You are on page 1of 503

−ν

(σ  ∂τ ∂τ yz ∂σ∇z ⋅ σρ+ ρb = 0
y
[ σ
y +σ
)]  ∂x ∂y ∂z



E z =

3
 y −ν

X l
−µ

2X
+ + + b =
(σ +


xz


y +σ
0 ε = 1 3 ,33
)]

X l 
z
 ∂x ∂y ∂z

2
3× −
[

2
]
z
σ 10 5 r r r

0
z =

l
t (nˆ ) =∇σ⋅⋅σn

3
x
σ )

l 2 3 
(σ + −µ
ˆ + ρb = 0
3

2
X

,3 323x×1 a − λ −r4ax 2 − λ − 4ax 2 z − ν σ


E
( =−
z

−µ

2X


2

nˆ )∂σ = 0∂τ 2 x1 a ∂τ γ
2

2,31
µ X

]
X 2

x σ 10 −5 (

z )= xz y ρ= 1 = x +σ
−µ l

t = σ n
ˆ
3

0 8× −
− 4ax 2 2ax1 − λx +
)]
xy x
+ + b =
l2

−2  − 4 ax 2 ax 1x τ
− λ 0 10 5

−µ
y =

2 3
∂ ∂ ∂

µ X 0
G

2X
2
, 3  x y z y = −
8⇒× (2 x1−a − λ )(2ax1 − λ )− 16
x
1
PROBLEMAS RESUELTOS DE
+σ x(22ax γa=−0 λ )(2ax − λ )2−,16
2 2
4,
)]  ∂τ xy ∂σ⇒ ∂1τ xyzy = ρ1 1 2 x1 a5−×xλ120a −5−=40ax 2 348 × −

X 2
2 2
10 5  ∂σ x ∂τ xy ∂τ

−µ l

3
y =
 0

ε −4,13 ( )  (4ax 2+) ⇒ +


y
+G2bτy = 0 − 2 =0 10 6 + +

0

l2


⇒ − λ 2
= 2
x =
2 x a ∂y (γ2 x∂1xaz − λ ) =xy(4=ax02 )x41 ax 2 2ax1 − λ
48 ×
[ ∂x ∂x ∂y ∂
1

0

=
E σx −10 −6
0 5 − dx1 t
(
2 x1 a − λ = ±4∂ax
 τ  λ
⇒ ∂τ 1yz + ∂σ
= 2x x
y =
a − 1 4 ax ∫
⇒ (2λx11 a==−2 xλd1)(
∫ a − 41ax−2λ )− 16 x 2 a = 0 2 2 
 ∂τ xy ∂σ y ∂τ


⇒ νσ 1 2
τ=ρxb±4ax t 2ax⇒

0
ε 2 +  ⇒2

ij
ε
y = 1
xz
λ = x a −z
G λ
+
+ = = 0 ⇒ X1 x1 ε   + +
[ − yλ + σ− 4ax2 ∂x x =l2n1


)] ∂y2 2 x∂1za 4ax 2 0 ⇒ (λ =−02λx )a2 + ( ) [ x1  ∂x ∂y ∂x
t 1 y z 2
σ 2 x a = 4 ax  
∫ 2 x a 4 ax  σ  = t
MECÁNICA DEL ∫MEDIO CONTINUO
dt E
1
z = = 0 x 21 1 2 2
⇒ y −ν−x(4ax 2 2ax1 −3λ,3
( E
2
dx2 Xx − ⇒  ∂τ ∂τ yz ∂σ
0ε λ 1 1=2 xν1 aσ− 4ax 2 x1 =
t
ln 1σ  33 × ε
z = 1
[ ∫ = d  + +
xz
x =+ σ t = 1
σ 

⇒ X 1 2 x a z ) =2ax1
(  t − λ ⇒)( ] − λ ) −
10 −52 2 t (nˆ ) = σ ⋅ nˆ X⇒
16 x a =
r
0
x22 x1 a − λ = ±4ax 2 ⇒ 
0 2t + 3
y
⇒ [
E σλlyn2−=x22 x1 a + 4axz2 = 3
y +σ
)] 

X ∂ 1xexp t ∂y ∂
E z −
νσ  1 (
2

−2 2xx1 =a −X λ 2− 4ax
dt  ( = ln
ν
(
x =

ε = 1 ,33
x (+2σx1 a − λ )2 = (,43ax 1182×)2 1 exp t X3
 X2 σ

2t=+ 13
3
⇒ ⇒

3× −
] ) ( )
2
=0
)] [ x + σ 2t +

z
10 5 r

1 
τ l n  x  − 4 ax 0 − 2 ax − λ σ ) 3 − t(

 ==ln−4
l 2 

l
(
5
G xy = y n
 E
( =
2
z −ν
,3 +⇒2 (⇒ λ−1∇=λ ⋅)(
2
2 x1 a − r4axr2
1
2

l 2 
z

−2 3
) ( )
0

2,5 ×⇒ X
2


3

+ 1ρ−b λ=)0− 16 x 22 a 2 = 0∇ ⋅ σ +γxρb



σ
X

10 2−5x21a − λ = ±24t4ax
r r
2 3 

8 3× 2−xln1−aλ = 2σ2xax
0

,31
3

1
X

l 2 3  = = 1
0 + x

8
+ σ
)] ×


1 a 4 ax
−µ

y
2X

x

G τxy = x1 0 6 23 2 ⇒ τ 10 −5
−µ

2X


 1 2
⇒ (2 x1 a − λ ) = (4ax 2x ) G xy = =−
X

2 y


0 dx
X

2 = X γ 4,3
l

2,5 ×

X l 
∫ 2
−µ

t
48 ×

2
 l2 3

= 1
µ X 0
2X

1 r r
∫ 1
−µ

0
= + x
2 3

3
λ 1 = 23x1 a −14ax 2
2 y
⋅ −
0+ ρb = 0
µ X 0
2X

τx
X
10 − 6

l 2 3 
dt τ ∇ σ 5

2
X1 x1 ⇒ ⇒  2 x a − λ = ±4ax ⇒
2

y =
ln x1 1  ∂σ x + ∂2τ xy +λ∂τ2 xz= 2+xρ1 ab+γ∂=4σax
−µ Eduardo
Gτ xy = x1
∂τ0xz dρx W. V. Chaves 2 x1 a − λ − 4a

2X
X 2


3

0 x2 ∂
µ X 2 µ X

0

3
0

2

  = t  x x 0x= 2+1

xy
+ + 1b = 0
0

l2

t

 ∂⇒ ∂y ∂z  y − 4ax 2 − 2λax1
l

dx2  X1 
∫ x ∂zX x =x dt ∫
0

∂x τ∂xy

t
−µ

2 x1 a

−µ

2 3


0


µ X 0

=
2X

x

dt = G ⇒
ij =

y =  x⇒ (

X 2 x2  ∂τ xy r ∂σ1 y X1∂eτxyzp t 1
− λ2 )(2a2
 ij =


l
1

+ ρby ∂=τ xy 0 n x−1 a4ax


0 + ∂σ y + ∂τ xyz2 +∂ρ  2
 00

r
X 2

0
3



ε

ε0 2t +13  x ∇ ⋅ σ + ρ+b = 0 + σbx =∂0τ xy 2∂τx1xza −λρ −=4tax


1

0

l

3
ε
2

= l n ε 
[   x =∂x1 ∂y ∂x z d 
l 2 3

yt+ + X
+ b1 =20x a −⇒
x3 = X x
([ ) ∫ ∂z4ax 2 ⇒x2(ax (2λx1λa=)−02 λ=
0

∂x z x2∂=
2

2

 2
l
2X

 ∂x ∂y

E σx −  X 2  = ln 2σ d∂ty ⇒1−
l


ij = 

−µ

x −


0

(
3

(( )
X

t +∂3τ x
X 2 

νσ ∂τE ln ∂σ3z + ρb  ∂=2τx0xz1 a − ∂λτ yz − 4∂ax


1
 xz +x − yzν− σ
X2
ε σ z2 2 ∂ρτ xy0 2t ∂+σ3y ∂⇒ ⇒ (2−x−λ1 aλ=)−±

y + σ εy =  1∂
+ += 0b z += 0 + ⇒τ(yz2 x+1 aln ρ−b⇒λx)( −λ4
µ

y = 1
2 3

[

X l 

 =2x01ax
µ X 0
2X

+ +
 0

)] [ ∂y y ∂+zε ⇒ 22 a1
(

z
x3 = X
ε

x 

)] [
σ r1 ∂rxx
0

σ ∂ ∂ =
3

σ, x σy∂τ−xy ∂τ∇xz ⋅ σxρz+=ρ=b = 0− 42 ax = 2X y 2ax 2 1 −z λ 3∂x  X  ln


y

z = ∂y ∂x
X 2

E y − ∂3E
2

ν(σ ⇒ (z2 x1 a − λ2)2⇒= 2(4xaxa 22−)t 2λ+


l
3

l 2 3

ε2 x=1 a 1− λ − 4ax 3 +3 ×∂y −(σ ∂zr nˆ + + = σ +


−µ

b
2X

ν 3 t
0


l2

2
 3 0
[
2
x =+0σ εz = 1∂x
( E
x ,3⇒3x 3(−2 νx a − λ3)(2ax − ∂λτ)− 16 x∂τ2 ayz2 = ∂0σ
1 1
=

−µ

] [
z
1 5 x +t ε ⋅ × 1σ − ρ
z )=
 0
0

1 
σ λ
]
− 4Eax 2 2ax1 − λ σ
( )
= σ n
ˆ 1 r xz
+ 2
+ z
+ b =
 σ∂)x⋅2nˆ ∂y ⇒∂z2 x1 a − λz = ±−4λ
0
2∂τσxyz − ∂σ y ∂τ yzyz =) = σ ⇒ (2 x a y− +λ )σ2 t= (4ax
1 0 5
[

( =
( )
z −ν
)]
−µ

− ⇒
2 3

− 4aλ
( E ax
0
2X

γ

σ +ρ Eb−y2=,y30−
= ε(12x x=1 a 1− λ )(2axx 1 +−σλ )γ−xy 16= x 221a 2,∂3=108 +×ν∂σx +
2 x1 a
0

2
z =
ij

xy ⇒ + 1 2

µ X

)] ε∂x∂=z σ x1 ∂τ xy 18 ×∂(τσxz − ρ ν
X 2

3

,33 λ = 2 x a − 4ax − 4ax


[


−µ l

)] [ x 10y−5 σ

τ  3 ×1 2 ax


σ)2 = (4ax )2 = −G τx + + 10x ++5 b = 0
0

l2

⇒G(2 xx1yaE=− 2λ

z
]
y 1 r 2 1
4,3 ∂yτ = 2 ∂τ yz ∂yσ ∂E=x −ρ4σ ∂y ⇒ 2∂xz1 a − σλz =)x±=4ax 2 ⇒ λ10=−52 x a t+(nˆ4)ax
2
= σ ⋅n


x5 − ν
(

γ ˆ
2 ⇒ (2 x1 a − λ )(2a

l 2  
,
0

×
(
10 σ−γ5y xy+ =λ1 =  8 ×−1 +,5− ×1 γ+−  ∂τ+ b,z3z∂4−σ=8ν×0 σ ∂τ
2
xy = ε1

4  2

2
−2

23 X  
xz z

0
1
y = 1
[

3
ij =

X ll 2  3 
,31
τ
)]
σ 2x1∂ax 04ax6∂2y 0 xy5 =∂z 1 xy

y 10 + ρ
0

G⇒ x2yEx1=a σ− λx1 = ±4ax 2 ε⇒ Gz τ= − 8 ⇒ (2 x1 a − λ )2 =


)] ×1 −5
1 yz
σ + by = 0
ε

 τ + + x 6

−µ


2
γ 0 y −dxν γ t x =λ12 =xy32,=x10a +x14ax 2 0

1(σ [ ∂ ∂ ∂

0
=−

[ ∫
xy = ε1 3 G x x y x y
=xxy+d= 1E σ 33 × 1dx1γ− = t r (nˆ ) y = 2,5 ×

l 2 3 
X
(∫ ∫

2
z 00 − µ 0l 2 2 X  
z = 1 4,3
τ ∫ ]
− µµ
σ −

2X
y =dtt ∂τ=xzσ ⋅ n 4


G xyE = σ 1 1 ε Gz )τ⇒ x t 0 1 ∂ τ ∂ σ
 µ X 2 l 2 3
ν 0 + b = 0 8 × 10 − 6
0
2

x
 5
x ˆ 1 ρ ⇒ 2 x1 a − λ = ±4
xy −= ln 2Xxσ
X −
x
0

x111 a21 − λ − λ− 4τax


( 0y = 1 =
µ X3

yz
⇒ +

0 z −x2 ν  + 5z
X

− µ2 XX 

[ +x a − 4 ax x  l 22 X3332
)] l

γ 0   G 2
= n 
dxσ
2 ,31 Xx2− γ = tσ y 0 
 ∂y−xλ = 0 ∂1=y 0  ∂=z
0 z

−µ

1 x 2
σ 1
µ

x

xy = 1 2
+
∫ )
σ E y − ν ×1d1x02xy−−524=ax
]
8 ⇒
= 1
0

4 ax 2 ax dx  t t


X l 

t

(σ⇒+ = t−1λ2τdt3,323ax3x11−×=−λXλX−1xeXx1p1 =t2r 2dt= ⇒ x1 = X ∇ ⋅ σ + ρbr = 0r


x z

X

G τxy =X 2 x2 =εz = y d1t = −


2
2


l
− µ l ij =  


X 2


0
3

X 2x x2(2σx1 aG )( =)(12ax10)1−−λ516
X

l
l 2 3 

[
3
X 2 

 x 1 ex p t
3

γ 2,5 4,⇒
] ⇒ (2 x a x−y2λax ) x16 2 a(n xˆ )2 a02 =⇒
µ X 0

ε 3  0
2X

⋅ 0ˆ
l2
0 2t +
x3 = X× 10 −5 E 3σz − 48 ×x1ln − x2 z)0 =21t−+ 23 0 ⇒1x2 2 ln x0t2 2 = σ n ln 1  =
3 1 −

−µ

2X

xy = 1

l  

−µ

(
X l 

( )∫
ε

(620⇒
rx1 a=−lnλ2), = (24ax 2 )dx 2 t  ε
2

G τxy = x1 γx3y = 1 ν σ 3⇒ 
r0=XX
( )
23  (2 x1 a −2 3tλ1+)8 = (4ax22 ) X  = ln  X 1  t
0  

0

x = 1
[
3


X



ij 2=
l 2 3 

∇ σ + ρ b =
0

+
( )∫


2

)]
−µ

× σ
2 3

0 dx σ =
( )
3 − 2 + x
µ X 0

=
0

x d
2X


1X 20 lxn−52λ31 = 2 x1 a − 4ax 2 3 − ln t

t
−µ


2X

τ X1 εE
[(

2
2= 1 e tx − 
0

t 1
x
X l 

x1=a2−−
=

p
⇒ ⇒ 0 λ21t⇒  ∂σ x ⇒∂τ xy εy ∂=τ xz1 x ρ= σν σ 
=+2 x1 a −⇒4ax 2  3

=0 = G 1
1

y
x =
2

2,5 ×  ⇒ 2⇒ λ=±
X 2

G τxy = X1 γx1 dt y ⇒ x41,a34− λ4=ax±x24ax


l

3 x

 l
[
X

x n  x+ = X +σ 2bEx = 0x −y +
l 2 3 

λ = +

2+ =
8 × −3 = X 3 2 λ22 x=1 a2 x1 4aax X 22 2   = +ε
(
 0

2
0

0 x2 xy = 01
l2

ij

(

−µ

2 2
ε
3

ln1 0x1−5 

24 ax
X

 Xt 2+1∂x ln x 2∂=ty +1 2 ∂εE ν


µ

10 6
)
2X
−µ

τ  
[ ( )[ z 2 y t−+
0

=
2

y = 31 ν1(σ

=


2
X

dx2 Gt xy = x∂1 σ  X∂1 τxy t ∂τ ⇒ 3


X 2

ε 3σ− ln

−µ l

γ 0 dxx + t  + xz + ρbx == 0  ∂τ xy ∂σE


3

x1 a − 4ax 2 zy = ∂x1τ −
∫ [ ( 3 σ + 

= d
∫ ρ
l2

+ σ ν+Eσ b y y⇒ =−0ν xσ


−µ

 1
 0

X 2 yx2=
1 t
3

x
ij

yz
µ X 0

ε
ε

+
2X

X1 eεxp t

x 1x
 (σ2
∂ x = ∂
∫ y ∂ z r r
∇ ⋅ σ + ρb = 0  ∂x y = ∂1y εEz ∂x=z z 1− ν
[ [ ( )]
0 τ2t + x = 1
x1 a + 4ax 2 X ⇒x  dt
[ y +σ
2
X 2

x3 = X G xy = 30 1 ∂τ1xy ln0∂σx2y  ⇒ x  σ γ σ σ


l

∂ τ
( l n  E x −
(  ∂τ E∂xyτ = y1−∂σE σ x + σ= 3x
X 2 − − 20 3  

= ln yz + ρb 1 = =0
0

( )]
2

εz xz= +1 yzγG+ τxν(zσ+ ρzb −


 0

 ν
+ +
) ( )[
x2
ε  νσ y ,

dx X  εt
µ Xµ l X 2  

σ
l

3
−µ

x = 1
∫ ∂2x t ∂y2  ∂x z 2t +X31y − y = ⇒ z =0
[
0

1
[
− µ 2lX32   

+
=

 l + =
 n 3 σ⇒ x1 r= Xr e tσz )]  ∂xγ σ∂y xy = ∂z1 2, σ
]
2

=
∫ 5 ×z ) =− x + σ
y y x
3 

dt
E σx −
0

∇ ⋅ σ y+−ρb = 01 xp
2X  

X 2 x2
X

( = Exy = z 1−
(

∂ τ E τ
 0

ν ∂ τ ∂ σ 1 −
ij

ν(xσ2 = X 2 γ ,33 ν
l 2 3 

⇒ + ρlnbz ε= 3 τx εσGxx =+xy1 = 2 0 5 2,3


ε σ  xz 0+ t + 3 + 2 yz
−µ

z xz 20= 1
y = 1
[ y + σ x3 = X∂
[( + = 3 1× γ G [)]

)]   σ 3 t + 1 γ τ10 x−y5 = 1t=(nˆ0) E =σσdy σ


,5
ˆ ×t 10 −5
x
∂y ∂z
]
r
2

2 y
 3x  X 2 E= lnσ 2t + ⋅
x y x
E σy − )
() ( )
n
1
X

z =
∫ (
l

z −ν 3 − = G y = xy1 = τx x =
1 −−ν
ν(σ
z
ln 3 γ −2,3
µ Xl 2 0 3

x x
3,33 2ε,G
0


2X

ε r= 1r γ σ = x1= 4d,3
tσ4yt8+×
µ X 22 X3

τ 5
xy =y ×=10 X−15 01
1
[ 3 × 1 −5
[ ∫ x
y
ρz b = 0 x +σ xy = 1 xτ + σ xy = 118 γG
  µ X


]
r
z )=
t = σ ⋅n  ∂σ
)]
∂ × d x
32

0 n ∂τ 1x0y −=5 21 0 E x2 σ
E σz −
( ˆ )
ˆ

G τxy = x ∂+σ x + ∂τ+xyy +=xz∂−τ+xzρb+xG ρ=bx20τx==y X


01

( ∫ = d
l

xy
y −ν
ij =  

3
0

−µl 2

x1 τ
(

νσ −2 = x
ε  0 0

ε t
l2

0 G d 1σ
γ ∂ 2 ∂ ∂ γ 4 0
∫ t + = x
2
,3 x
  5∂x× , y z ,
xy∂z=341 z3 = x1 1
X t
∫ [∫ d 1
x x
= 1 x +σ 18 × ∂y y
0 x2 = xdt0+ σ

1

2
xy = 1

t

)] 8
0  0

τ 10 −5  ∂τ 1
∂σ

0 5 ∂τ × 1 −6 =Xσ x
G τxy = xyx1∂τ+xy y ∂+σ y yz∂τ+yzρGb yτρx=y 00= γ X1 x1E 2z d−t2 ν d0⇒
ij =

∫( ∫
y =
∂G
x22t +t z
τ xy xy ∂=τ xz2, ρ −4

ε

ε
 0

, ε γ 0 + + + b =0 σ = l3nd

x = 1 ∂x d∂x1 ∂yt ∂ ∂x z ∂


5+× b x− = 0 348
[ [ 0 x 3 = X
0
1 + xy x2= 1 X 2 x2x + σ 
∂y ∂z 1 0 5 × 10 − 6 E σ σ x = 1 x y = 1  ∫ x
 ∂Xτ  x ∂=τ dt ∂σ ∫
y x
y
dx2 τ t )] 0 y2t + X
G τxy = x1 ∫
3
x −ν G τ
z

∂σ y 0∂τ yz dx ρ t ε ε 1
y =
E x −( (
σν σ x y = ∂ τ ⇒
0  x2  ∂τ+xz +0 +yz + ∂+σ z bl+nzρ=bx10γ=xy 0=X 2 1x2
1 xz 1 yz z ρ  G =xy = xdt =
∫ 23,5 ×X 3
τx ∂y
+ ∫
∂Xx1 z x1
+1 =b y = 0
dt ∫ E
y = 1
σ σ [ [ −
y +σ
y +
z σ)] )]∫
=

 ∂ x
dx2∂x t ∂y ∂ y ∂ z ∂z  X 1
 z  = t
 x3G= ⇒ τx 0 2 t + 3 10⇒ −5
ln
y = ⇒ ε x  E yν(−σ 3,3=33 = X 3y = x1 =x1

y z d
0 l n z 1=ε 1 ν (+σ X x32,3 ×33
t γ 0 X
dx1 expt t
[ ∫
0
∂τ ∂x   1  1
Nomenclature III

Problemas Resueltos de
´
Mecanica del Medio Continuo
EDUARDO WALTER VIEIRA CHAVES
IV MECÁNICA DEL MEDIO CONTINUO
Presentación

´
Presentacion

s
eratur

Co

lo
nv

Sue
ec a
ció ic
ul
Temp

n
-di á
dr
. de
fus
ió n Hi
Mec
Flu

as
Vi g
jo

ras
os

Estructu Placa
s
uid
Fl

s
li do

PCVI y tratamiento numérico


Mov. Sólido Rígido
Ecuaciones Constitutivas

Ecuaciones Fundamentales de MMC

Tensiones

Cinemática del continuo

Tensores
VI MECÁNICA DEL MEDIO CONTINUO

´ para el Alumno
Guia

1) NO SE MEMORIZA EJERCICIO.

2) Una vez que la teoría haya sido estudiada, intentar resolver los ejercicios sin mirar la
solución. Es importante que el alumno ante un nuevo problema desarrolle la habilidad de
dar la solución al problema con los conocimientos adquiridos.

3) Tener en cuenta que, en general, un ejercicio es un caso particular de la teoría. Es muy


importante saber reconocer cuando estamos ante una aproximación del caso general.

4) A veces, la solución de un ejercicio se puede obtener por varios caminos. Una vez
resuelto el ejercicio, intentar verificar si existe otra forma de resolverlo.

5) Cuidado, puede haber erratas, seáis críticos...


Contenido

Contenido

PRESENTACIÓN V
CONTENIDO VII
NOMENCLATURA IX
OPERADORES XIII
UNIDADES XIV

1 TENSORES1
1.1 EJERCICIOS RESUELTOS 1
1.1.1 Vectores, Notación Indicial 1
1.1.2 Operaciones con Tensores de Orden Superior 9
1.1.3 Transpuesta 14
1.1.4 Simetría y Antisimetría 14
1.1.5 Cofactor. Adjunta. Traza. Tensores Particulares. Determinante 18
1.1.6 Descomposición Aditiva de Tensores 24
1.1.7 Ley de Transformación. Invariantes. 25
1.1.8 Autovalores y Autovectores 31
1.1.9 Representación Espectral 38
1.1.10 Teorema de Cayley-Hamilton 42
1.1.11 Tensores Isótropos y Anisótropos 44
1.1.12 Descomposición Polar 44
1.1.13 Tensor Esférico y Desviador 45
1.1.14 Otros 46
1.1.15 Función de Tensores. Campo de Tensores. 47
1.1.16 Teoremas con Integrales 57
1.2 EJERCICIOS PROPUESTOS 59

2 CINEMÁTICA DEL CONTINUO 65


2.1 EJERCICIOS RESUELTOS 65
2.1.1 Descripción del Movimiento, Derivada Material, Velocidad, Aceleración 65
2.1.2 Tensores de Deformación Finita, Deformación Homogénea 89
2.1.3 Descomposición Polar del Gradiente de Deformación 121
2.1.4 Deformación Infinitesimal 142
2.2 EJERCICIOS PROPUESTOS 152

3 TENSIONES 157
3.1 EJERCICIOS RESUELTOS 157
3.1.1 Fuerza, Tensor de Tensiones, Vector Tensión 157
3.1.2 Ecuación de Equilibro, Tensiones y Direcciones Principales 162
3.1.3 Otras Medidas de Tensión 170
3.1.4 Máxima Tensión de Corte, Círculo de Mohr 171
3.1.5 Particularidades del Tensor de Tensiones 179
3.1.6 Estado Tensional en Dos Dimensiones 192
VIII PROBLEMAS RESUELTOS DE MECÁNICA DEL MEDIO CONTINUO

3.1.7 Tensiones En Coordenadas Cilíndricas y Esféricas 198


3.2 EJERCICIOS PROPUESTOS 202

4 LEYES FUNDAMENTALES DE LA MECÁNICA DEL MEDIO CONTINUO 207


4.1 EJERCICIOS RESUELTOS 207
4.2 EJERCICIOS PROPUESTOS 218

5 INTRODUCCIÓN A LAS ECUACIONES CONSTITUTIVAS 219


5.1 EJERCICIOS RESUELTOS 219
5.2 EJERCICIOS PROPUESTOS 224

7 ELASTICIDAD LINEAL 225


7.1 EJERCICIOS RESUELTOS 225
7.2 EJERCICIOS PROPUESTOS 280

11 FLUIDOS 283
11.1 Ejercicios Resueltos283
11.2 Ejercicios Propuestos 296
Abbreviations

Abreviaturas

PVCI Problema de Valor de Contorno Inicial


PVC Problema de Valor de Contorno
MEF Método de los Elementos Finitos
MEC Método de los Elementos de Contorno
MDF Método de las Diferencias Finitas

Latin

i.e. id est es decir


et al. et alii y otros
e.g. exempli gratia por ejemplo
etc. et cetera y así sucesivamente
Q.E.D. Quod Erat Demonstrandum lo que se quería demostrar
v., vs. versus versus
viz. vidilicet a saber

Alfabeto griego

(a) α Α - alfa (n) ν Ν - nu


(b) β Β - beta (o) ο Ο - ómicron
(c) χ Χ - ji (p) π Π - pi
(d) δ ∆ - delta (q) θ Θ - theta
(e) ε Ε - épsilon (r) ρ Ρ - ro (rho)
(f) φ Φ - fi (s) σ Σ - sigma
(g) γ Γ - gamma (t) τ Τ - tau
(h) η Η - eta (u) υ Υ - ypsilon
(i) ι Ι - iota (v) ϖ ς - sigma
(j) ϕ ϑ - fi (w) ω Ω - omega
(k) κ Κ - kappa (x) ξ Ξ - xi
(l) λ Λ - lambda (y) ψ Ψ - psi
(m) µ Μ - mu (z) ζ Ζ - dseta
Operadores

Operadores

• +•
〈•〉 = paréntesis de MacAuley
2
• norma Euclidiana de •
Tr (•) traza de (•)
(•) T transpuesta de (•)
(•) −1 inversa de (•)
(•) −T inversa de la transpuesta de (•)
(•) sym parte simétrica de (•)
(•) anti parte antisimétrica de (•)
(•) esf parte esférica de (•) o parte hidrostática
(•) dev parte desviadora de (•)
• módulo de •
[[•]] salto de •
⋅ producto escalar
det(•) ≡ • determinante de (•)
cof (•) Cofactor de • ;
Adj(•) adjunta de (•)
Tr (•) traza de (•)
: doble producto escalar
∇2 operador diferencial escalar (Laplaciano)
⊗ producto tensorial
∇ • ≡ grad(•) gradiente de •
∇ ⋅ • ≡ div (•) divergencia de •
∧ producto vectorial
I • , II • , III • Primer, segundo y tercer invariantes del tensor •
D•
≡ •& Derivada material de •
Dt
r Vector

•ˆ Vector unitario (versor)
1 Tensor identidad de segundo orden
I Tensor identidad de cuarto orden
I sym ≡ I Parte simétrica del tensor identidad de cuarto orden
Unidades

Unidades (SI)
longitud m - metro energía, trabajo, calor J = Nm - Joules
J
masa kg - kilogramo potencia ≡ W Vatio
s
tiempo s - segundo permeabilidad m2
temperatura K - Kelvin viscosidad dinámica Pa × s
m kg
velocidad flujo de masa
s m2s
m J
aceleración flujo de energía
s2 m2s
W
energía J = Nm - Joules conductividad térmica:
mK
1
fuerza N - Newton frecuencia ≡ Hz Hertz
s
N kg
presión, tensión Pa ≡ - Pascal densidad de masa
m2 m3
J
densidad de energía
m3

Prefijo Símbolo Potencia Prefijo Símbolo Potencia


10 10
pico p 10 −12 kilo k 10 3
nano η 10 −9 Mega M 10 6
micro µ 10 −6 Giga G 10 9
mili m 10 −3 Tera T 1012
centi c 10 −2
deci d 10
XII PROBLEMAS RESUELTOS DE MECÁNICA DEL MEDIO CONTINUO
Nomenclatura

´
Notacion

r r r r m
A( X , t ) ≡ a ( X , t ) Aceleración (configuración de referencia)
s2
A Matriz de transformación de base
r r m
a ( x, t ) Aceleración (configuración actual)
s2
B0 Medio continuo en la configuración de referencia - t = 0
B Medio continuo en la configuración actual - t
∂B Contorno de B
r r N
b( x , t ) Fuerzas másicas (por unidad de masa)
m3
b Tensor izquierdo de deformación de Cauchy-Green,
tensor de deformación de Finger
B Tensor de deformación de Piola
J
B Entropía creada interiormente
sK
Manantial de entropía local por unidad de masa y por J
b unidad de tiempo kg s K
Ce Tensor constitutivo elástico Pa
[C ] Matriz elástica (notación de Voigt) Pa
C in Tensor constitutivo inelástico Pa
c Tensor de deformación de Cauchy
Cv Calor específico a volumen constante
Cp Calor específico a presión constante
c Cohesión Pa
mol
cc Concentración
m3
C Tensor derecho de deformación de Cauchy-Green
m
DV Deformación volumétrica
m
D Tensor velocidad de deformación o tensor tasa de
deformación o tensor tasa de deformación Euleriana o
tensor estiramiento
r
dA Diferencial de área en la configuración de referencia m2
r
da Diferencial de área en la configuración actual m2
dV Diferencial de volumen m3
XIV PROBLEMAS RESUELTOS DE MECÁNICA DEL MEDIO CONTINUO

E Tensor material de deformación Green-Lagrange, tensor


m
de deformación de Green, tensor de deformación
Green-St. Venant m
Tensor de deformación finita Euleriana o tensor de m
e
deformación de Almansi m
E Módulo de elasticidad longitudinal o módulo de Young Pa
ê i Base Cartesiana en notación simbólica
ˆi , ˆj, kˆ Base Cartesiana
m
F Gradiente de deformación
m
G Módulo de elasticidad transversal Pa
H Tensor de deformación de Biot
J
H Entropía total
K
r kgm 2
HO Momento angular = Js
s
m3
J Determinante del Jacobiano
m3
r m
J ( X , t) Tensor gradiente material de los desplazamientos
m
r m
j ( x, t ) Tensor gradiente espacial de los desplazamientos
m
r mol
J Tensor de difusividad
m2s
W J
K Tensor de conductividad térmica =
mK smK
K Energía cinética J
r kg m
L Cantidad de movimiento lineal
s
m
l Tensor gradiente espacial de velocidad
sm
m Masa total kg
M Tensor de tensiones de Mandel Pa
Vector unitario normal a una superficie (configuración

actual)
Vector unitario normal a una superficie (configuración
N̂ de referencia)
r
NO Momento angular
r N
p Fuerza másicas por unidad de volumen
m3
P Primer tensor de tensiones de Piola-Kirchhoff, tensor
de tensiones nominales o tensor de tensiones Pa
Lagrangiano
p Presión media Pa
p Presión termodinámica Pa
NOTACIÓN XV

r r J
q( x , t ) Flujo de calor o vector del flujo no convectivo
m2s
Q Tensor ortogonal
Q Potencia calorífica J
r
r ( x, t ) Función escalar que describe en forma espacial el calor J
generado por las fuentes internas por unidad de masa kg s
R Tensor ortogonal de la descomposición polar
S Segundo tensor de tensiones de Piola-Kirchhoff Pa
r J
s Flujo de entropía
kg s m 2
T Tensor de tensiones de Biot Pa
r (nˆ ) r
t ( x , t , nˆ ) Vector tracción (configuración de referencia) Pa
r (Nˆ )
t0 Pseudo vector tensión (configuración de referencia) Pa
r
T ( x, t ) Temperatura K
t Tiempo s
t0 ≡ t = 0 Tiempo inicial s
J
U& Tasa de la energía interna =W
s
J
u Energía interna específica
kg
r r
u( x , t ) Vector desplazamiento (Euleriana) m
r r
u( X , t ) Vector desplazamiento (Lagrangiana) m
r
U( X , t Tensor derecho de estiramiento, o tensor de
estiramiento Lagrangiano, o tensor de estiramiento
material
r
V ( x, t ) Tensor izquierdo de estiramiento, o tensor de
estiramiento Euleriano, o tensor de estiramiento espacial
r r r r m
V ( X , t ) ≡ v ( X , t ) Velocidad (configuración de referencia)
s
r r m
v ( x, t ) Velocidad (configuración actual)
s
m rad
W Tensor spin o tensor velocidad de rotación =
ms s
J
w int Potencia tensorial =W
s
r
X Vector posición coordenada material m
r
x Vector posición coordenada espacial m
1
α Coeficiente de expansión térmica
K
δ ij Delta de Kronecker
ε1 , ε 2 , ε 3 Deformaciones principales
m
ε Alargamiento unitario
m
 ijk Símbolo de permutación, componentes del tensor Levi-
Civita
XVI PROBLEMAS RESUELTOS DE MECÁNICA DEL MEDIO CONTINUO

Deformación volumétrica (para pequeñas m


εV
deformaciones) m
m
ε Tensor de deformación infinitesimal
m
J
η Entropía específica
kg K
κ Módulo de deformación volumétrico Pa
m2
κ Difusividad térmica
s
m
λ Estiramiento
m
λ, µ Constante de Lamé Pa
ν Coeficiente de Poisson
kg
ρ Densidad de masa
m3
kg
ρS Densidad de masa de la solución
m3
kg
ρf Densidad de masa del fluido
m3
r kg
ρ 0 ( x, t ) Densidad de masa en la configuración de referencia
m3
r kg
ρ ( x, t ) Densidad de masa en la configuración actual
m3
Tensor de tensiones de Cauchy o tensor de tensiones
σ Pa
verdaderas
r
σN Componente normal del vector tracción Pa
r
σS Componente tangencial del vector tracción Pa
σm Tensión media Pa
σ1 , σ 2 , σ 3 Tensiones principales Pa
r
σ oct Tensión normal octaédrica Pa
r Tensión tangencial octaédrica o tensión de corte
τ oct Pa
octaédrica
τ max Tensión de corte máximo Pa
τ Tensor de tensiones de Kirchhoff Pa
φ Ángulo de fricción interno
J
ψ Energía libre de Helmholtz por unidad de masa
kg
Energía libre de Helmholtz por unidad de volumen J
Ψ (densidad de energía) m3
J
Ψ (ε ) = Ψ e Densidad de energía de deformación
m3
ψ Ángulo de dilatancia
Ω Tensor tasa del tensor de rotación material
r rad
ω Tensor de vorticidad
s
1 Tensores
La notación indicial fue introducida por ‘Einstein (1916, sec. 5),
who later jested to a friend, "I have made a great discovery in
mathematics; I have suppressed the summation sign every
time that the summation must be made over an index which
occurs twice..." (Kollros 1956; Pais 1982, p. 216). ‘
Ref. (Wolfram MathWorld (Einstein Summation))

1.1 Ejercicios Resueltos

1.1.1 Vectores, Notación Indicial

Ejemplo 1.1
r r
Probar que si a y b son vectores se cumple que:
(ar ∧ br )⋅ (ar ∧ br ) = (ar ⋅ ar )(br ⋅ br ) − (ar ⋅ br )
2

Solución:

(ar ∧ br )⋅ (ar ∧ br )
r r 2 r r
= a ∧ b = a b sin θ ( 2
)
r 2 r 2 r 2 r 2 r 2 r r 2
( ) r
2 2
= a b sin 2 θ = a b 1 − cos 2 θ = a b − a b cos 2 θ
r 2 r 2 r r
( 2
= a b − a b cos θ = a b − a ⋅ b
r 2 r 2 r r 2
) ( )
r r r r
( ) ( )
r r 2
= (a ⋅ a) b ⋅ b − a ⋅ b
r r r 2 r r r 2
donde hemos considerado que a ⋅ a = a y b ⋅ b = b .

Ejemplo 1.2
r r r r
Probar que: si c = a + b , el módulo de c puede ser expresado a través de la siguiente relación:
r r 2 r r r 2
c = a + 2 a b cos β + b
r r
donde β es el ángulo que forman los dos vectores a y b .
Solución:
2 PROBLEMAS RESUELTOS DE MECÁNICA DEL MEDIO CONTINUO

Partiendo de la definición del módulo de un vector se cumple que:


r r
a+b
2
(
r r
= a+b )⋅ (ar + br ) = ar ⋅ ar + ar ⋅ br + br ⋅ ar + br ⋅ br
r r r r r r 2 r r r r
Teniendo en cuenta que a ⋅ a = a , b ⋅ b = b y que a ⋅ b = b ⋅ a (conmutativo), concluimos
2

que:
r r 2 r r r r r r r r r
a+b = a⋅a + a⋅b + b ⋅ a + b ⋅b b
r r r r 2
+ 2a ⋅ b + b
2
= a β r
r 2 r r r 2 a
= a + 2 a b cos β + b

r r r 2 r r r 2
con lo cual demostramos que a+b = a + 2 a b cos β + b . Luego es de fácil
r r r 2 r r r 2
demostración que a − b = a − 2 a b cos β + b .
r r 2 r r r r 2
NOTA: Partiendo de la expresión a + b = a + 2a ⋅ b + b
2
podemos concluir que el
r r 2
valor a + b será máximo cuando β = 0º resultando que
r r 2 r r r r 2
+ 2a ⋅ b + b β = 0º
2
a+b = a
r
r 2 r r r 2
r b
= a +2a b + b a r r r r
a+b = a + b
r
( r
= a + b )
2

r r r r
Luego para cualquier otro valor de 0º < β ≤ 180 º el valor a + b será menor que a + b .
r r r r
luego, a + b ≤ a + b :

r r r r r r
b r r r c = a+b ≤ a + b
c = a+b
r
b
r
a

r r r r r r
De forma análoga se puede demostrar que a ≤ c + b y b ≤ a + c que es la conocida
desigualdad triangular, donde se cumple que:

a<c+b
c b<a+c
b
c<a+b

Universidad de Castilla- La Mancha Draft Por: Eduardo W. V. Chaves (2012)


Ciudad Real - España
1 TENSORES 3

Ejemplo 1.3
1
Verificar si para las siguientes transformaciones σ(ε) = Eε y ψ(ε) = Eε 2 son
2
transformaciones lineales.
Solución:
σ(ε 1 + ε 2 ) = E [ε1 + ε 2 ] = Eε 1 + Eε 2 = σ(ε1 ) + σ(ε 2 ) (transformación lineal)

σ( ε)
σ (ε 1 + ε 2 ) = σ (ε 1 ) + σ ( ε 2 )

σ (ε 2 )

σ (ε 1 )

ε1 ε2 ε1 + ε 2 ε

1
La transformación ψ(ε) = Eε 2 se demuestra fácilmente que no es una transformación lineal
2
ya que:
1
2
1
[
ψ(ε1 + ε 2 ) = E [ε1 + ε 2 ]2 = E ε12 + 2ε1ε 2 + ε 22
2
]
1 2 1 2 1
= Eε1 + Eε 2 + E 2ε 1ε 2
2 2 2
= ψ ( ε 1 ) + ψ ( ε 2 ) + Eε 1 ε 2 ≠ ψ ( ε 1 ) + ψ ( ε 2 )

ψ ( ε)

ψ (ε1 + ε 2 )

ψ (ε1 ) + ψ (ε 2 )
ψ (ε 2 )

ψ (ε1 )

ε1 ε2 ε1 + ε 2 ε

Universidad de Castilla- La Mancha Draft Por: Eduardo W. V. Chaves (2012)


Ciudad Real - España
4 PROBLEMAS RESUELTOS DE MECÁNICA DEL MEDIO CONTINUO

Ejemplo 1.4
Considérense los puntos A(1,3,1) , B (2,−1,1) , C (0,1,3) y D(1,2,4 ) .
Se pide:
→ →
1) Encontrar el área del paralelogramo definido por AB y AC ;
→ → →
2) Encontrar el volumen del paralelepípedo definido por: AB , AC y AD ;
→ →
3) Encontrar el vector proyección del vector AB sobre el vector BC .
Solución:
→ →
1) Primero se calculan los vectores AB y AC :
r → → →
( ) ( )
a = AB = OB − OA = 2ˆi − 1ˆj + 1kˆ − 1ˆi + 3ˆj + 1kˆ = 1ˆi − 4ˆj + 0kˆ

b = AC = OC − OA = (0ˆi + 1ˆj + 3kˆ ) − (1ˆi + 3ˆj + 1kˆ ) = −1ˆi − 2ˆj + 2kˆ


r → → →

Utilizando la definición del producto vectorial se obtiene el producto vectorial:


ˆi ˆj kˆ
r r
a∧b= 1 − 4 0 = ( −8)ˆi − 2ˆj + ( −6)kˆ
−1 − 2 2

El área del paralelogramo será igual al módulo del vector resultante del producto
vectorial:
r r
A = a ∧ b = (−8) 2 + (−2) 2 + ( −6) 2 = 104 (unidades cuadradas)


2) Calculando vector AD :
r → → →
( ) (
c = AD = OD − OA = 1ˆi + 2ˆj + 4kˆ − 1ˆi + 3ˆj + 1kˆ = 0ˆi − 1ˆj + 3kˆ )
Utilizando la definición:
r r r r r r
V (a, b, c ) = c ⋅ a ∧ b ( ) = (0ˆi − 1ˆj + 3kˆ )⋅ (− 8ˆi − 2ˆj − 6kˆ )
= 0 + 2 − 18 = 16 (unidades cúbicas)

3) A continuación calculamos el vector BC :
→ → →
( ) (
BC = OC − OB = 0ˆi + 1ˆj + 3kˆ − 2ˆi − 1ˆj + 1kˆ = −2ˆi + 2ˆj + 2kˆ )
→ →
Luego el vector proyección de AB sobre BC viene dado por:

proj

AB =

BC ⋅ AB


BC =
(− 2ˆi + 2ˆj + 2kˆ )⋅ (1ˆi − 4ˆj + 0kˆ ) (− 2ˆi + 2ˆj + 2kˆ )

BC →
BC
1
42 ⋅4
BC
3

(− 2ˆi + 2ˆj + 2kˆ )⋅ (− 2ˆi + 2ˆj + 2kˆ )
→ 2
BC

=
(− 2 − 8 + 0 ) (− 2ˆi + 2ˆj + 2kˆ )
(4 + 4 + 4 )

Universidad de Castilla- La Mancha Draft Por: Eduardo W. V. Chaves (2012)


Ciudad Real - España
1 TENSORES 5

→ 5ˆ 5ˆ 5 ˆ
proj →
AB = i − j− k
BC 3 3 3

Ejemplo 1.5
Reescribir en notación indicial las siguientes expresiones:
1) a1 x1 x 3 + a 2 x 2 x 3 + a 3 x 3 x 3
Solución: a i xi x 3 (i = 1,2,3)
2) x1 x1 + x2 x2
Solución: xi x i (i = 1,2)

 a11 x + a12 y + a13 z = b x



3) a 21 x + a 22 y + a 23 z = b y

 a 31 x + a 32 y + a 33 z = b z
Solución:
 a11 x1 + a12 x 2 + a13 x 3 = b1  a1 j x j = b1
 
índice índice

a 21 x1 + a 22 x 2 + a 23 x 3 = b2
mudo j
 → a 2 j x j = b2 libre
i → a ij x j = bi
a x + a x + a x = b 
 31 1 32 2 33 3 3  a 3 j x j = b3

Ejemplo 1.6
a) Demostrar que: δ 3 p v p = v3 ;
b) Demostrar que: δ 3i A ji = A j 3 ;
c) Obtener el resultado de δ ij  ijk ;
d) Obtener el resultado de δ i 2 δ j 3 Aij .
Solución:
Las componentes de la delta de Kronecker son:
δ 11 δ 12 δ 13  1 0 0
δ ij = δ 21 δ 22 δ 23  = 0 1 0 (1.1)
δ 31 δ 32 δ 33  0 0 1
a) La expresión ( δ 3 p v p ) no tiene índice libre, luego el resultado es un escalar:
δ 3 p v p = δ 31v1 + δ 32 v 2 + δ 33 v 3 = v3 (1.2)
b) La expresión δ 3i A ji tiene un índice libre ( j ), luego el resultado es un vector:
δ 3i A ji = δ 31 A j1 + δ 32 A j 2 + δ 33 A j 3 = A j 3 (1.3)
c) La expresión δ ij  ijk tiene un índice libre ( k ), luego el resultado es un vector:

Universidad de Castilla- La Mancha Draft Por: Eduardo W. V. Chaves (2012)


Ciudad Real - España
6 PROBLEMAS RESUELTOS DE MECÁNICA DEL MEDIO CONTINUO

δ ij  ijk = δ 1 j 1 jk + δ 2 j  2 jk + δ 3 j  3 jk
123 1424 3 123
δ 1111k + δ 21 21k + δ 31 31k
+ + + (1.4)
δ 12 12 k + δ 22  22 k + δ 32  32 k
+ + +
δ 13 13k + δ 23  23k + δ 33  33k

luego, δ ij  ijk = 0 k (vector nulo).


d)
δ i 2 δ j 3 Aij = A23 (1.5)

Ejemplo 1.7
Expandir la expresión: Aij x i x j (i, j = 1,2,3)

Solución: Los índices i, j son índices mudos (indican suma), no hay índice libre, y como
resultado tenemos un escalar. Expandimos primero el índice mudo i y a continuación el
índice j , resultando así:
Aij x i x j expandiend
   oi
→ A1 j x1 x j + A2 j x 2 x j + A3 j x 3 x j
1424 3 1 424 3 1 424 3
A11 x1 x1 A21 x 2 x1 A31 x 3 x1
+ + +
expandiendo j

A12 x1 x 2 A22 x 2 x 2 A32 x 3 x 2


+ + +
A13 x1 x 3 A23 x 2 x 3 A33 x 3 x 3
Reagrupando los términos anteriores obtenemos:
Aij x i x j = A11 x1 x1 + A12 x1 x 2 + A13 x1 x 3 + A21 x 2 x1 + A22 x 2 x 2 +
A23 x 2 x 3 + A31 x 3 x1 + A32 x 3 x 2 + A33 x 3 x 3

Ejemplo 1.8
Desarrollar las siguientes expresiones y obtener el valor numérico correspondiente:
1) δ ii δ jj
Solución: δ ii δ jj = (δ 11 + δ 22 + δ 33 )(δ 11 + δ 22 + δ 33 ) = 3 × 3 = 9
2) δ α1δ αγ δ γ1
Solución: δ α1δ αγ δ γ1 = δ γ1δ γ1 = δ 11 = 1
NOTA: Observar que es incorrecto hacer la siguiente operación
δ γ1δ γ1 ≠ δ γγ = 3 ≠ δ 11 = 1 , ya que lo que se reemplaza es el índice repetido ■

Universidad de Castilla- La Mancha Draft Por: Eduardo W. V. Chaves (2012)


Ciudad Real - España
1 TENSORES 7

Ejemplo 1.9
a) Probar que a)  ijk  pjk = 2δ ip ; b)  ijk  ijk = 6 c)  ijk a j a k = 0 i ; d) Obtener el valor numérico
de la siguiente expresión  ijk δ 2 j δ 3k δ 1i .
Solución:
a) Utilizando la expresión:
 ijk  pqk = δ ip δ jq − δ iq δ jp
y haciendo q = j , resulta:
 ijk  pjk = δ ip δ jj − δ ij δ jp
= δ ip 3 − δ ip = 2δ ip
b) Partiendo del resultado anterior, es trivial la siguiente comprobación:
 ijk  ijk = 2δ ii = 6
c) Observemos que  ijk = − ikj , es decir, es antisimétrico en jk y observemos que a j a k
resulta un tensor de segundo orden simétrico. Como sabemos el doble producto escalar de un
tensor simétrico y otro antisimétrico es cero luego:
r r
 ijk a j a k =  ijk (a ⊗ a ) jk = 0 i
r r
= (a ∧ a ) i = 0 i
d)  ijk δ 2 j δ 3k δ 1i = 123 = 1

Ejemplo 1.10
Obtener el valor de las siguientes expresiones:
a)  ijk δ i1δ j 2 δ 3k
b)  ijk  pqk = δ ip δ jq − δ iq δ jp para los siguientes casos
b.1) i = 1, j = q = 2, p = 3
b.2) i = q = 1, j = p = 2
c) ( ijk A jp c p A kq c q + δ i1 )( ist A sa c a A tb c b + δ i1 )
donde  ijk es el símbolo de permutación y δ ij es la Delta de Kronecker.
Solución: ijk = 1
a)  ijk δ i1δ j 2 δ 3k = 123 = 1 1
12 k  32 k = 121 321 + 122  322 + 123  323 =
b.1) ijk = −1
= 0 × (−1) + 0 × 0 + 0 × 0 = 0
3 2
12 k  21k = 121 211 + 122  212 + 123  213
b.2)
= 0 × 0 + 0 × 0 + 1 × (−1) = −1
c) Observemos que la operación A jp c p = b j resulta un vector y verificamos también que
r r
 ijk A jp c p A kq c q = [( A ⋅ c) ∧ ( A ⋅ c )]i = (b ∧ b) i = 0 i , con lo cual resulta que:
r r

Universidad de Castilla- La Mancha Draft Por: Eduardo W. V. Chaves (2012)


Ciudad Real - España
8 PROBLEMAS RESUELTOS DE MECÁNICA DEL MEDIO CONTINUO

( ijk A jp c p A kq c q + δ i1 )( ist A sa c a A tb c b + δ i1 ) = (0 i + δ i1 )(0 i + δ i1 ) = δ i1δ i1 = δ 11 = 1

Ejemplo 1.11
r
Escribir en notación indicial: a) el módulo del vector a ; b) cos θ , donde θ es el ángulo que
r r
forman los vectores a y b .
Solución:
r
r r r
= a ⋅ a = a i eˆ i ⋅ a j eˆ j = a i a j δ ij = a i a i = a j a j
2
a ⇒ a = ai ai
r
luego, también cumple que b = b i b i .
r r r r
Por definición a ⋅ b = a b cos θ , donde:
r
a ⋅ b = a i eˆ i ⋅ b j eˆ j = a i b j δ ij = a i b i = a j b j

Teniendo en cuenta que un índice no puede aparecer más que dos veces en un término de la
expresión, podemos expresar cos θ como:
r r
a⋅b a jb j
cos θ = r r =
a b ai ai b k b k

Ejemplo 1.12
(r r ) (r r )
Escribir la siguiente relación a ∧ b ⋅ c ∧ d sin emplear el producto vectorial.
(r r )
Solución: Observemos que el producto vectorial a ∧ b lo podemos expresar de la siguiente
(r r )
forma: a ∧ b = a j eˆ j ∧ b k eˆ k =  ijk a j b k eˆ i , cuyo resultado será un vector. De esta forma hemos
utilizado la definición del símbolo de permutación. Análogamente podemos expresar el
r r
( ) r r
( )
producto vectorial c ∧ d como c ∧ d =  nlm c l d m eˆ n , por lo tanto:
(ar ∧ b)⋅ (cr ∧ dr ) = 
r
ˆ ⋅ ( nlm c l d m eˆ n )
ijk a j b k e i )
=  ijk  nlm a j b k c l d m eˆ i ⋅ eˆ n
=  ijk  nlm a j b k c l d m δ in
=  ijk  ilm a j b k c l d m

Teniendo en cuenta que  ijk  ilm =  jki  lmi y aplicando la relación


 jki  lmi = δ jl δ km − δ jm δ kl =  jki  ilm , concluimos que:
 ijk  ilm a j b k c l d m = (δ jl δ km − δ jm δ kl ) a j b k c l d m = a l b m c l d m − a m b l c l d m

Puesto que el subíndice mudo indica el producto escalar: a l c l = (a ⋅ c ) y b m d m = b ⋅ d , luego:


r r
(r r )
(ar ∧ br )⋅ (cr ∧ dr ) = (ar ⋅ cr ) (br ⋅ dr ) − (ar ⋅ dr )(br ⋅ cr )
r r r r
Observemos que, cuando c = a y d = b obtenemos que:
(ar ∧ br )⋅ (ar ∧ br ) = (ar ⋅ ar ) (br ⋅ br ) − (ar ⋅ br )(br ⋅ ar ) = (ar ⋅ ar ) (br ⋅ br ) − (ar ⋅ br )
2

Que es la misma expresión obtenida en el Ejemplo 1.1.

Universidad de Castilla- La Mancha Draft Por: Eduardo W. V. Chaves (2012)


Ciudad Real - España
1 TENSORES 9

Ejemplo 1.13
Probar que
a)  ijk a i a j b k = 0
r r
b)  ijk (a k b 3δ i1δ j 2 + a j b 2 δ i1δ k 3 + a i b1δ j 2 δ k 3 ) = a ⋅ b
c) Aij A ji es un invariante
Solución:
a)  ijk a i a j b k =  ij1 a i a j b1 +  ij 2 a i a j b 2 +  ij 3 a i a j b 3 . Para el término  ij1 a i a j b1 tenemos que:
 ij1a i a j b1 = 1 j1 a1 a j b1 +  2 j1a 2 a j b1 +  3 j1 a 3 a j b1
= 111 a1 a1b1 +  211 a 2 a1b1 +  311 a 3 a1b1 +
+  121 a1a 2 b1 +  221 a 2 a 2 b1 +  321 a 3 a 2 b1 +
+  131 a1a 3b1 +  231 a 2 a 3 b1 +  331 a 3 a 3 b1
=  321 a 3 a 2 b1 +  231 a 2 a 3 b1 = −a 3 a 2 b1 + a 2 a 3b 1
=0
Análogamente para los términos  ij 2 a i a j b 2 =  ij 3 a i a j b 3 = 0 . Es interesante observar que
 ijk a i a j b k representa el determinante con dos filas iguales:
a1 a2 a3
 ijk a i a j b k = a1 a2 a3 = 0
b1 b 2 b3
b)
 ijk a k b 3 δ i1δ j 2 +  ijk a j b 2 δ i1δ k 3 +  ijk a i b1δ j 2 δ k 3 =
r r
12 k a k b 3 +  1 j 3 a j b 2 +  i 23 a i b1 = a 3b 3 + a 2 b 2 + a1b1 = a i b i = a ⋅ b

Ejemplo 1.14
(r r ) (r r ) r (r
Probar que: a ∧ b ∧ c ∧ d = c d ⋅ a ∧ b − d c ⋅ a ∧ b
r r
) r (r r r
)
Solución: Expresaremos en notación indicial el segundo miembro de la expresión:
[cr (dr ⋅ ar ∧ br ) − dr (cr ⋅ ar ∧ br )] p [ ( )] [ (
= c p d i  ijk a j b k − d p c i  ijk a j b k )]
⇒  ijk a j b k c p d i −  ijk a j b k c i d p
(
⇒  ijk a j b k c p d i − c i d p )
Si utilizamos la propiedad de la delta de Kronecker:
(
⇒  ijk a j b k δ pm c m d n δ ni − δ im c m d n δ np )
⇒ ( a b ) c
ijk j k mdn (δ pm δ ni − δ im δ np )
y si consideramos que δ pm δ ni − δ im δ np =  pil  mnl . Reemplazamos en la expresión anterior y
obtenemos:
( )
⇒  ijk a j b k c m d n  pil  mnl ( )
[(
⇒  pil  ijk a j b k ( mnl c m d n ) ) ]

Universidad de Castilla- La Mancha Draft Por: Eduardo W. V. Chaves (2012)


Ciudad Real - España
10 PROBLEMAS RESUELTOS DE MECÁNICA DEL MEDIO CONTINUO

(r r )
Dado que las componentes de a ∧ b son  ijk a j b k y las componentes de c ∧ d son (r r )
 mnl c m d n , obtenemos que:

[(r r ) (r r )]
 pil [( ijk a j b k ) ( mnl c m d n )] = a ∧ b ∧ c ∧ d p

Ejemplo 1.15
r r r
a) Si a , b , c son vectores linealmente independientes y que se cumple que:
r r r r
v = αa + βb + γ c componente
 s → v i = αa i + βb i + γ c i ≠ 0 i
Probar que los escalares α , β , γ son dados por:
 ijk v i b j c k  ijk a i v j c k  ijk a i b j v k
α= ; β= ; γ=
 pqr a p b q c r  pqr a p b q c r  pqr a p b q c r
b) Dados tres vectores linealmente independientes, demostrar que al intercambiar 2 filas ó 2
r r r
columnas el signo del determinante a ⋅ (b ∧ c ) cambia.
r r r
Solución: a) Haciendo el producto escalar del vector v por el vector ( b ∧ c ) obtenemos que:
r r r r r r r r r r r r
v ⋅ (b ∧ c ) = αa ⋅ (b ∧ c ) + β b ⋅ (b ∧ c ) + γ c ⋅ (b ∧ c )
14243 14243
=0 =0
Obtenemos entonces el valor de α como:
r r r
v ⋅ (b ∧ c )
α= r r r
a ⋅ (b ∧ c )
En componentes:
v1 v2 v3 v1 b1 c1
b1 b2 b3 v2 b2 c2
c1 c2 c3 v3 b3 c3  ijk v i b j c k
α= = =
a1 a2 a3 a1 b1 c1  pqr a p b q c r
b1 b2 b3 a2 b2 c2
c1 c2 c3 a3 b3 c3

Análogamente podemos obtener los parámetros β , γ , es decir, hacemos el producto escalar


r r r r r
del vector v por los vectores a ∧ c y a ∧ b , respectivamente, i.e.:
r r r r r r r r r r r r
v ⋅ (a ∧ c ) = α a ⋅ (a ∧ c ) + β b ⋅ ( a ∧ c ) + γ c ⋅ ( a ∧ c )
14243 14243
=0 =0
r r r r r r
v ⋅ (a ∧ c )  v a c
ijk i j k −  a v
jik j i k c a ⋅ (v ∧ c)
⇒β = r r r = = = r r r
b ⋅ (a ∧ c )  pqr b p a q c r −  qpr a q b p c r a ⋅ (b ∧ c )
r r r r r r r r r r r r
v ⋅ (a ∧ b ) = α a ⋅ ( a ∧ b ) + β b ⋅ ( a ∧ b ) + γ c ⋅ (a ∧ b )
14243 14243
=0 =0
r r r r r r
v ⋅ (a ∧ b)  ijk v i a j b k  jki a j b k v i a ⋅ (b ∧ v )
⇒γ= r r r = = = r r r
c ⋅ (a ∧ b)  pqr c p a q b r  qrp a q b r c p a ⋅ (b ∧ c )
r
NOTA 1: Podemos reestructurar las componentes del vector v de la siguiente forma:

Universidad de Castilla- La Mancha Draft Por: Eduardo W. V. Chaves (2012)


Ciudad Real - España
1 TENSORES 11

 v 1   a1 b1 c 1  α   a1 b1 c 1   z1 
     
v i = v 2  = a 2 b2 c 2  β  = a 2 b2 c 2  z 2  = B ij z j
 v  a b3 c 3  γ   a 3 b3 c 3  z 3 
 3  3
donde hemos denotado por z1 = α , z 2 = β , z 3 = γ . Teniendo en cuenta que:
v1 b1 c1 a1 v1 c1
v2 b2 c2 a2 v2 c2
 ijk v i b j c k v3 b3 c3 B (1)  ijk a i v j c k a3 v3 c3 B (2)
α =z1 = = = ; β =z 2 = = =
 pqr a p b q c r a1 b1 c1 B  pqr a p b q c r a1 b1 c1 B
a2 b2 c2 a2 b2 c2
a3 b3 c3 a3 b3 c3

a1 b1 v1
a2 b2 v2
 ijk a i b j v k a3 b 3 v3 B (3)
γ =z 3 = = =
 pqr a p b q c r a1 b 1 c1 B
a2 b2 c2
a3 b3 c3

donde B (i ) es el determinante de la matriz resultante al reemplazar la columna (i) de la


r
matriz B por las componentes del vector v . Con eso, podemos decir que:
B (i )
Dado v i = B ij z j ⇒ zi = Regla de Cramer
B

NOTA 2: Aunque hemos demostrado para una matriz 3 × 3 , este procedimiento es válido
para matrices de n-dimensiones y es conocido en la literatura como Regla de Cramer.
NOTA 3: La solución ( z i ) solo es posible si B ≠ 0 .

NOTA 4: Si v i = 0 i tenemos que B ij z j = 0 i y B (i ) = 0 i , con eso, según la regla de Cramer


tenemos que:
z i B = B (i ) = 0 i

Notar que, la solución non-trivial z i ≠ 0 i solo es posible si y solo si B = 0 , (ver Ejemplo


1.50).
r r r r r r
b) El determinante definido por a ⋅ (b ∧ c ) = [a, b, c ] en notación indicial queda  ijk a i b j c k ,
además sabiendo que se cumple que:
 ijk =  jki =  kij
 ijk = − ikj = − kji = − jik
i

k j

Universidad de Castilla- La Mancha Draft Por: Eduardo W. V. Chaves (2012)


Ciudad Real - España
12 PROBLEMAS RESUELTOS DE MECÁNICA DEL MEDIO CONTINUO

r r r r r r r r r
 ijk a i b j c k = [a, b, c ] = − ikj a i b j c k = −[a, c , b] =  jki a i b j c k = [b, c, a]
Luego
a1 a2 a3 a1 a2 a3
 ijk a i b j c k = b 1 b 2 b 3 = − c 1 c2 c 3 = −  ikj a i b j c k
c1 c2 c3 b1 b2 b3

b1 b2 b3
= c1 c2 c 3 =  jki a i b j c k
a1 a2 a3

Ejemplo 1.16
Probar las relaciones:
r r r r r r r r r r r r r r
( ) (
a ∧ (b ∧ c ) = (a ⋅ c ) b − a ⋅ b c = b ⊗ c − c ⊗ b ⋅ a
r r
)
r r r
a ∧ (b ∧ a) = [(a ⋅ a)1 − a ⊗ a]⋅ b
r r r

r r
(
Solución: Representando el producto vectorial b ∧ c i =  ijk b j c k , luego: )
[ar ∧ (br ∧ cr )]
r =  rsi a s ( ijk b j c k )
=  rsi  ijk a s b j c k =  rsi  jki a s b j c k
= (δ rj δ sk − δ rk δ sj ) a s b j c k
= δ rj δ sk a s b j c k − δ rk δ sj a s b j c k = asb r c s − asb s c r
= ak br c k − a jb j c r = (b r c s − b s c r )a s
r r
( )
r r
= b r (a ⋅ c ) − c r a ⋅ b [(
r r r r r
= b ⊗ c − c ⊗b ⋅a r ) ]
[r r r rr r
( )]
= b(a ⋅ c ) − c a ⋅ b r
Comprobando que:
r r r
( )
r r r r r r r r r r r
( ) (
a ∧ b ∧ c = (a ⋅ c ) b − a ⋅ b c = b ⊗ c − c ⊗ b ⋅ a )
r r
En el caso particular cuando a = c podemos decir que:
r r r
[ ( )]
a ∧ b ∧ a r = (a k a k )b r − (a j b j )a r = (a j a j )b p δ rp − (a j b p δ jp )a r
[ ] [
= (a j a j )δ rp − (a j δ jp )a r b p = (a j a j )δ rp − a p a r b p ]
{ r r r
= [(a ⋅ a)1 − a ⊗ a]⋅ b r
r r
}
Ejemplo 1.17
Demostrar la identidad de Jacobi:
r r r r
( )r r r r r r
a ∧ b ∧ c + b ∧ (c ∧ a) + c ∧ a ∧ b = 0 ( )
r
Solución: A través del ejercicio anterior demostramos que a ∧ b ∧ c = (a ⋅ c ) b − a ⋅ b c , luego, (r r ) r r r
(r r ) r
también es válido que:

Universidad de Castilla- La Mancha Draft Por: Eduardo W. V. Chaves (2012)


Ciudad Real - España
1 TENSORES 13

r r r
b ∧ (c ∧ a) = (br ⋅ rar )cr − (br ⋅ cr )arr
r r r
(
c ∧ a∧b = ) (cr ⋅ b)ar − (cr ⋅ ar )b
Luego, teniendo en cuenta que el producto escalar entre dos vectores es conmutativo, es decir,
r r r r
( ) ( )( ) ( )
r r r r r r r r
(a ⋅ c ) = (c ⋅ a) , a ⋅ b = b ⋅ a , b ⋅ c = c ⋅ b , concluimos que:
(a ⋅ c )b − (a ⋅ b )c
r r r r r r
r r r+ r r r r
(
r r r r
)
r r r r r
( ) ( ) ( )
a ∧ b ∧ c + b ∧ (c ∧ a) + c ∧ a ∧ b = b ⋅ a c − b ⋅ c a = 0
r r r+ r r r
c ⋅ b a − (c ⋅ a) b ( )
1.1.2 Operaciones con Tensores de Orden Superior

Ejemplo 1.18
¿ Cuál es el orden de los tensores representados por sus componentes: v i , Φ ijk , Fijj , ε ij , C ijkl ,
σ ij ? Determinar cuantas componentes independientes tiene el tensor C .

Solución: El orden del tensor viene dado por el número de subíndices libres, luego:
r r
Tensores de orden uno: v , F
Tensores de segundo orden: ε , σ
Tensor de tercer orden: Φ
Tensor de cuarto orden: C
El número de componentes de un tensor viene dado por el máximo valor del rango del
subíndice, 3 si ( i = 1,2,3 ), elevado al número de subíndices libres. Es decir, para el tensor de
cuarto orden, el número de índices libres es 4, luego:
3 4 = (i = 3) × ( j = 3) × (k = 3) × (l = 3) = 81
El tensor de cuarto orden C ijkl tiene 81 componentes independientes.

Ejemplo 1.19
(r r ) r (r r ) r (r r ) (r r ) (r r ) r
Demostrar que a) a ⊗ b ⋅ c = b ⋅ c a ; b) a ⊗ b ⋅ c ⊗ d = b ⋅ c a ⊗ d
r

Solución:
(ar ⊗ br )⋅ cr = (a eˆ
i i ⊗ b j eˆ j ) ⋅ c k eˆ k
= a i eˆ i b j c k δ jk
a)
= (b k c k )a i eˆ i
r r r r r r
= (b ⋅ c )a ≡ (b ⋅ c ) ⊗ a
r r r r
( )( )
b) La expresión a ⊗ b ⋅ c ⊗ d , que resulta un tensor de segundo orden, expresamos
directamente en notación indicial:

Universidad de Castilla- La Mancha Draft Por: Eduardo W. V. Chaves (2012)


Ciudad Real - España
14 PROBLEMAS RESUELTOS DE MECÁNICA DEL MEDIO CONTINUO

[(ar ⊗ br )⋅ (cr ⊗ dr )] ij ( )
= (a i b k ) c k d j = a i b k c k d j
= b k c k ai d j
r r r
= (b k c k )(a i d j ) = (b ⋅ c )(a ⊗ d) ij
123
escalar

Ejemplo 1.20
Desarrollar y simplificar lo posible la expresión A ij xi x j para los siguientes casos:
a) A ij = A ji
b) A ij = − A ji
Solución:
Expandiendo A ij xi x j obtenemos:

A ij xi x j = A 1 j x1 x j + A 2 j x 2 x j + A 3 j x3 x j =
= A 11 x1 x1 + A 21 x 2 x 1 + A 31 x3 x 1 +
(1.6)
A 12 x1 x 2 + A 22 x 2 x 2 + A 32 x 3 x 2 +
A 13 x1 x 3 + A 23 x 2 x 3 + A 33 x3 x 3

a) A ij = A ji (simetría)

A ij xi x j = A 11 x12 + 2 A 12 x1 x 2 +2 A 13 x1 x 3 +
(1.7)
A 22 x 22 + 2 A 23 x 2 x 3 + A 33 x32

b) A ij = − A ji (antisimetría)

A ij xi x j = 0 (1.8)
lo que era de esperar ya que:
r r r r
A ij xi x j = x ⋅ A ⋅ x = A : ( x ⊗ x ) (1.9)
r r
Si A antisimétrico y ( x ⊗ x ) resulta simétrico, el doble producto escalar de un tensor
simétrico y uno antisimétrico resulta ser siempre igual a cero.

Ejemplo 1.21
Si las componentes de los tensores de segundo orden ε y T son representadas
respectivamente por:
 5 2 4  3 1 2
ε ij =  − 1 2 1  ; Tij = 4 2 1  (1.10)
 4 3 6 1 3 8 

Obtener T : ε .
Solución:
T : ε = Tij ε ij (1.11)

Universidad de Castilla- La Mancha Draft Por: Eduardo W. V. Chaves (2012)


Ciudad Real - España
1 TENSORES 15

Tij ε ij = T j ε1 j + T2 j ε 2 j + T3 j ε 3 j
112 3 123 123
T11ε11 + T21ε 21 + T31ε 31
+ + + (1.12)
T12 ε12 + T22 ε 22 + T32 ε 32
+ + +
T13 ε13 + T23 ε 23 + T33 ε 33

luego,
Tij ε ij = 5 × 3 + 2 × 1 + 4 × 2 + (−1) × 4 + 2 × 2 + 1 × 1 + 4 × 1 + 3 × 3 + 6 × 8 = 87 (1.13)

Ejemplo 1.22
Dadas las componentes del tensor B en el sistema de coordenadas cartesianas:
3 2 4
B ij = 1 5 3 (1.14)
5 7 9

Obtener:
a) C ij = B ik B kj ; b) D ij = B ik B jk ; c) E ij = B ki B kj ; d) C ii , D ii , E ii
Solución:

3 2 4 3 2 4  31 44 54 
C = B ⋅B ⇒ C ij = B ik B kj = 1 5 3 1 5 3 =  23 48 46  (1.15)
5 7 9  5 7 9  67 108 122
T
3 2 4  3 2 4  29 25 65 
D = B ⋅ BT ⇒ D ij = B ik B jk    
= 1 5 3 1 5 3 =  25 35 67  (1.16)
5 7 9  5 7 9  65 67 155
T
3 2 4 3 2 4 35 46 60 
E = BT ⋅ B ⇒ E ij = B ki B kj = 1 5 3 1 5 3 = 46 78 86  (1.17)
5 7 9  5 7 9  60 86 106

Luego:
C ii = C 11 + C 22 + C 33 = 31 + 48 + 122 = 201
D ii = D11 + D 22 + D 33 = 29 + 35 + 155 = 219 (1.18)
E ii = E11 + E 22 + E 33 = 35 + 78 + 106 = 219

NOTA: Verificamos que se cumple que: Tr (B ⋅ B T ) = Tr (B T ⋅ B) = B : B

Ejemplo 1.23
Dadas las componentes cartesianas del tensor de segundo orden B :

Universidad de Castilla- La Mancha Draft Por: Eduardo W. V. Chaves (2012)


Ciudad Real - España
16 PROBLEMAS RESUELTOS DE MECÁNICA DEL MEDIO CONTINUO

1 0 2
B ij = 0 1 2
3 0 3

Obtener: a) B kk b) B ij B ij c) B jk B kj
Solución:
a) B kk = B 11 + B 22 + B 33 = 1 + 1 + 3 = 5
b) B ij B ij = B 1 j B 1 j + B 2 jB 2 j + B 3 jB 3 j
123 123 123
B 11B 11 + B 21B 21 + B 31B 31
+ + +
B 12B 12 + B 22B 22 + B 32B 32
+ + +
B 13B 13 + B 23B 23 + B 33B 33
Resultando:
B ij B ij = 1 × 1 + 0 × 0 + 2 × 2 + 0 × 0 + 1 × 1 + 2 × 2 + 3 × 3 + 0 × 0 + 3 × 3 = 28

c) B jk B kj = B 1k B k1 + B 2k B k 2 + B 3k B k 3
123 123 123
B 11B 11 + B 21B 12 + B 31B 13
+ + +
B 12B 21 + B 22B 22 + B 32B 23
+ + +
B 13B 31 + B 23B 32 + B 33B 33
B jk B kj = B 11B 11 + B 22B 22 + B 33B 33 + 2B 21B 12 + 2B 31B 13 + 2B 32B 23
= 1 × 1 + 1 × 1 + 3 × 3 + 2(0 × 0) + 2(3 × 2 ) + 2(0 × 2 ) = 23

Ejemplo 1.24
Obtener las componentes del tensor D resultante de la siguiente operación D = A : B , para
los siguientes casos:
 2 3 2  2 3 1
a) con Aij = 4 1 1  ; Bij = 1 2 1
1 1 5 1 2 5

 7 13 14 13 9 17 
b) con Aik Bkj = 11 18 11 ; Aik B jk = 15 9 13 
16 27 31 18 12 32

Solución:
a) A : B = 2 × 2 + 3 × 3 + 2 × 1 + 4 × 1 + 1 × 2 + 1 × 1 + 1 × 1 + 1 × 2 + 5 × 5 = 50
b) Teniendo en cuenta la expresión Tr ( A ⋅ B T ) = Tr ( AT ⋅ B) = A : B y que Aik B jk = A ⋅ B T ,
concluimos que A : B = Tr ( A ⋅ B T ) = 13 + 9 + 32 = 54 .

Universidad de Castilla- La Mancha Draft Por: Eduardo W. V. Chaves (2012)


Ciudad Real - España
1 TENSORES 17

Ejemplo 1.25
Considérese un tensor de segundo orden T = Tr ( E )1 + ( F : E ) E o en notación indicial
Tij = E kk δ ij + ( Fkp E kp ) E ij . Si las componentes de los tensores E y F vienen dadas por:

 2 1 4 4 3 1
E ij = 1 5 0 ; Fij =  2 0 3
2 0 1   2 0 0

a) Obtener las componentes del tensor T . b) ¿Son los tensores T y E coaxiales?


Demuéstralo.
Solución:
Obtenemos primero los siguientes escalares:
Tr ( E ) = 2 + 5 + 1 = 8
F : E = 2 × 4 + 1 × 3 + 4 × 1 + 1 × 2 + 5 × 0 + 0 × 3 + 2 × 2 + 0 × 0 + 1 × 0 = 21
Luego
1 0 0  2 1 4 50 21 84 
 
Tij = 80 1 0 + 211 5 0 =  21 113 0 
0 0 1  2 0 1   42 0 29

Dos tensores son coaxiales cuando presentan los mismos autovectores o cuando se cumple
que T ⋅ E = E ⋅ T :
50 21 84   2 1 4  289 155 284
Tik E kj =  21 113 0  1 5 0 = 155 586 84 
42 0 29  2 0 1  142 42 197 
2 1 4 50 21 84   289 155 284
E ik Tkj = 1 5 0   21 113 0  = 155 586 84 
2 0 1   42 0 29 142 42 197 
Con lo cual concluimos que son coaxiales.

Ejemplo 1.26
Obtener el resultado de las siguientes operaciones: I : I , I : I , I : I , I : I , I : I , I : I , I : I ,
I : I , I sym : I sym , I sym : I , I : I sym , donde

I = 1⊗1 = I ijkl eˆ i ⊗ eˆ j ⊗ eˆ k ⊗ eˆ l con I ijkl = δ ik δ jl (1.19)


I = 1⊗1 = I ijkl eˆ i ⊗ eˆ j ⊗ eˆ k ⊗ eˆ l con I ijkl = δ il δ jk (1.20)

I = 1 ⊗ 1 = I ijkl eˆ i ⊗ eˆ j ⊗ eˆ k ⊗ eˆ l con I ijkl = δ ij δ kl (1.21)

Solución:
(I : I ) ijkl = I ijpq I pqkl = δ ip δ jq δ pk δ ql = δ ik δ jl = I ijkl

( I : I ) ijkl = I ijpq I pqkl = δ iq δ jp δ pl δ qk = δ ik δ jl = I ijkl

Universidad de Castilla- La Mancha Draft Por: Eduardo W. V. Chaves (2012)


Ciudad Real - España
18 PROBLEMAS RESUELTOS DE MECÁNICA DEL MEDIO CONTINUO

( I : I ) ijkl = I ijpq I pqkl = δ ij δ pq δ pq δ kl = δ qq δ ij δ kl = 3I ijkl

( I : I ) ijkl = I ijpq I pqkl = δ iq δ jp δ pk δ ql = δ il δ jk = I ijkl

(I : I ) ijkl = I ijpq I pqkl = δ ip δ jq δ pl δ qk = δ il δ jk = I ijkl

(I : I ) ijkl = I ijpq I pqkl = δ ip δ jq δ pq δ kl = δ iq δ jq δ kl = δ ij δ kl = I ijkl

( I : I ) ijkl = I ijpq I pqkl = δ iq δ jp δ pq δ kl = δ iq δ jq δ kl = δ ij δ kl = I ijkl

Resumiendo lo anterior en notación tensorial:


I : I = (1⊗1) : (1⊗1) = 1⊗1 = I

I : I = (1⊗1) : (1⊗1) = 1⊗1 = I

I : I = (1 ⊗ 1) : (1 ⊗ 1) = 3(1 ⊗ 1) = 3I

I : I = (1⊗1) : (1⊗1) = 1⊗1 = I

I : I = (1⊗1) : (1⊗1) = 1⊗1 = I

I : I = (1⊗1) : (1 ⊗ 1) = 1 ⊗ 1 = I

I : I = (1⊗1) : (1 ⊗ 1) = 1 ⊗ 1 = I

Teniendo en cuenta la definición: I sym =


1
2
( 1
) ( )
I + I = 1⊗1 + 1⊗1 , concluimos que:
2

I sym : I sym =
1
4
( )(
1⊗1 + 1⊗1 : 1⊗1 + 1⊗1 )
1
[( ) (
= 1⊗1 : 1⊗1 + 1⊗1 : 1⊗1 + 1⊗1 : 1⊗1 + (1⊗1 : 1⊗1)
4
) ( ) ]
1
[
= 1⊗1 + 1⊗1 + 1⊗1 + 1⊗1
4
]
1
(
= 1⊗1 + 1⊗1
2
)
= I sym

I sym : (1 ⊗ 1) = I sym : I =
1
2
( 1
2
) 1
( ) ( )
I + I : I = I : I + I : I = I + I = I =1 ⊗1
2
(1 ⊗ 1) : I sym = I : I sym
1
2
( 1
2
) 1
( ) ( )
= I : I + I = I :I + I : I = I + I = I =1 ⊗1
2

1.1.3 Transpuesta

Ejemplo 1.27
Demostrar que la siguiente propiedad es válida:
( )
A : (B ⋅ C ) = B T ⋅ A : C = A ⋅ C T : B ( )
donde A , B , C son tensores de segundo cualesquiera.
Solución: Demostraremos esta identidad a través de sus componentes:

Universidad de Castilla- La Mancha Draft Por: Eduardo W. V. Chaves (2012)


Ciudad Real - España
1 TENSORES 19

(
A : (B ⋅ C ) = A ij eˆ i ⊗ e j : B lk eˆ l ⊗ e k ⋅ C pq eˆ p ⊗ eˆ q )
(
= A ij B lk C pq eˆ i ⊗ e j : δ kp eˆ l ⊗ eˆ q )
= A ij B lk C pq δ kp δ il δ jq = A ij B ik C kj

Observemos que cuando trabajamos en notación indicial el orden no importa, es decir:


A ij B ik C kj = B ik A ij C kj = A ij C kj B ik

Podemos ahora observar que la operación B ik A ij resultará un tensor de segundo orden cuyas
componentes son (B T ⋅ A ) kj luego, B ik A ij C kj = (B T ⋅ A ): C . Análogamente podemos decir que
( )
A ij C kj B ik = A ⋅ C T : B .

Ejemplo 1.28
r r
Demostrar que, si u , v son vectores y A un tensor de segundo orden, la siguiente relación es
válida:
r r r r
u⋅ AT ⋅ v = v ⋅ A ⋅u
Solución:
r r r r
u⋅ AT ⋅ v = v ⋅ A ⋅u
u i eˆ i ⋅ A jl eˆ l ⊗ eˆ j ⋅ v k eˆ k = v k eˆ k ⋅ A jl eˆ j ⊗ eˆ l ⋅ u i eˆ i
u i A jl δ il v k δ jk = v k δ kj A jl u i δ il
u l A jl v j = v j A jl u l
c.q.d.

1.1.4 Simetría y Antisimetría

Ejemplo 1.29
Si σ es un tensor de segundo orden simétrico y W es un tensor de segundo orden
antisimétrico. Demostrar que σ : W = 0 .
Solución:
σ :W = σ ij (eˆ i ⊗ eˆ j ) : Wlk (eˆ l ⊗ eˆ k )
= σ ij Wlk δ il δ jk
= σ ij Wij

Desarrollando
σ ij Wij = σ1 j W1 j + σ 2 j W2 j + σ 3 j W3 j
123 1424 3 1
424 3
σ11W11 σ21W21 σ31W31
+ + +
σ12 W12 σ 22 W22 σ32 W32
+ + +
σ13W13 σ23W23 σ33W33

Universidad de Castilla- La Mancha Draft Por: Eduardo W. V. Chaves (2012)


Ciudad Real - España
20 PROBLEMAS RESUELTOS DE MECÁNICA DEL MEDIO CONTINUO

Considerando la propiedad de un tensor simétrico σ12 = σ 21 , σ 31 = σ13 , σ 32 = σ 23 y


antisimétrico W11 = W22 = W33 = 0 , w 21 = − w 12 , W31 = − W13 , W32 = − W23 , resultando:
σ :W =0
c.q.d.

Ejemplo 1.30
Demostrar que:
r r r r
a) M ⋅ Q ⋅ M = M ⋅ Q sym ⋅ M ;
b) A : B = A sym : B sym + A anti : B anti ;
r
donde, M es un vector, y Q , A , y B son tensores de segundo orden arbitrarios.
c) Demostrar que si se cumple que  ijk T jk = 0 i , T es simétrico, es decir, Tij = T ji .
Solución:
a)
( )
r r r r
M ⋅ Q ⋅ M = M ⋅ Q sym + Q anti ⋅ M
r r r r
= M ⋅ Q sym ⋅ M + M ⋅ Q anti ⋅ M
r r r r
(
Ya que el producto: M ⋅ Q anti ⋅ M = Q anti : M ⊗ M = 0 , resulta que: )
r r r r
M ⋅ Q ⋅ M = M ⋅ Q sym ⋅ M c.q.d.
r r
NOTA: Podemos hacer la interpretación geométrica de M ⋅ Q anti ⋅ M = 0 . Notar que la
r r r r r r r r
operación algebraica Q anti ⋅ M = q (M) resulta un vector, luego M ⋅ Q anti ⋅ M = M ⋅ q (M) = 0 , que
r r r
implica que M y q (M) son vectores ortogonales. Con eso, concluimos que: la proyección de
r r r
un tensor antisimétrico de segundo grado según una dirección ( M ) resulta un vector ( q (M) )
r
que es ortogonal a M , ver figura abajo:

r
Q ⋅M r r r
q (M ) ⋅ M = 0

r r r r
M q (M) = Q anti ⋅ M r
M

b)
A :B = ( A sym + A anti ) : (B sym + B anti )
= A sym : B sym + 1
A sym
42
anti
43 + 1
:B A anti
42
sym
43 + A
:B anti
: B anti
=0 =0
= A sym : B sym + A anti : B anti
Luego como consecuencia tenemos que:
A : B sym = A sym : B sym ; A : B anti = A anti : B anti
c.q.d.

Universidad de Castilla- La Mancha Draft Por: Eduardo W. V. Chaves (2012)


Ciudad Real - España
1 TENSORES 21

c)
 ijk T jk =  ij1 T j1 +  ij 2 T j 2 +  ij 3 T j 3 = 0 i
=  i11 T11 +  i 21 T21 +  i 31 T31 +  i12 T12 +  i 22 T22 +  i 32 T32 +  i13 T13 +  i 23 T23 +  i 33 T33
=  i 21 T21 +  i 31 T31 +  i12 T12 +  i 32 T32 +  i13 T13 +  i 23 T23 = 0 i
Luego, las componentes del vector resultante quedan:
i =1 ⇒ 1 jk T jk = 132 T32 + 123 T23 = − T32 + T23 = 0 ⇒ T32 = T23
i=2 ⇒  2 jk T jk =  231 T31 +  213 T13 = T31 − T13 = 0 ⇒ T31 = T13
i=3 ⇒  3 jk T jk =  321 T21 +  312 T12 = − T21 + T12 = 0 ⇒ T21 = T12

con lo cual demostrando que si  ijk T jk = 0 i , T es simétrico, T = T T .

Ejemplo 1.31
Dado un tensor de segundo orden arbitrario A donde se conocen las componentes de su
parte simétrica en el sistema Cartesiano:
 4 2 0
A ijsym =  2 1 0
0 0 3

ˆ , donde las componentes del versor N̂ son Nˆ = [1 0 0] .


ˆ ⋅ A ⋅N
Obtener N i

Solución:
ˆ ⋅ A ⋅N
En el Ejemplo 1.30 se ha demostrado que N ˆ ⋅ A sym ⋅ N
ˆ =N ˆ con lo cual:

 4 2 0 1 
N ⋅ A ⋅ N = N ⋅ A ⋅ N = N i A ij N i = [1 0 0] 2 1 0 0 = 4
ˆ ˆ ˆ sym ˆ sym

0 0 3 0

Ejemplo 1.32
Si W es un tensor antisimétrico. a) Demostrar que W ⋅ W resulta un tensor de segundo
orden simétrico. b) Demostrar también que (W T ⋅ W ⋅ W) : 1 = 0
Solución:
a) Si demostramos que (W ⋅ W ) anti = 0 , demostramos que W ⋅ W resultar ser simétrico:

(W ⋅ W ) anti =
1
2
[ 1
] [
( W ⋅ W) − ( W ⋅ W) T = (W ⋅ W) − W T ⋅ W T
2
]
1
= [(W ⋅ W ) − W ⋅ W ]
2
=0
donde hemos aplicado la propiedad del tensor antisimétrico W = −W T .
Soluciones alternativas a) Teniendo en cuenta la definición de un tensor antisimétrico donde se
cumple que W = − W T :
W ⋅ W = −W T ⋅ W = W T ⋅ W T = (W ⋅ W ) T

Universidad de Castilla- La Mancha Draft Por: Eduardo W. V. Chaves (2012)


Ciudad Real - España
22 PROBLEMAS RESUELTOS DE MECÁNICA DEL MEDIO CONTINUO

También se puede comprobar a través de sus componentes:


 0 W12 W13   0 W12 W13 

( W ⋅ W ) ij =  − W12 0  
W23   − W12 0 W23 
 − W13 − W23 0   − W13 − W23 0 
 − W122 − W132 − W13 W23 W12 W23 
 2 2 
=  − W13 W23 − W12 − W23 − W12 W13 
 W12 W23 − W12 W13 2 
− W132 − W23
 

b) (W T ⋅ W ⋅ W ) : 1 = (W pi W pk Wkj )δ ij = W pi (W pk Wki ) = W : (W ⋅ W) = 0 , ya que el doble


producto escalar entre un tensor simétrico (W ⋅ W ) y uno antisimétrico ( W ) resulta cero.

Ejemplo 1.33
1
Sea B un tensor de segundo orden tal que B pq =  pqs a s con a i =  ijk B jk . Demostrar que B
2
es un tensor antisimétrico.
Solución:
1  1 1
B pq =  pqs a s =  pqs   sjk B jk  =  pqs  sjk B jk =  pqs  jks B jk
2  2 2
Recurriendo a la relación  pqs  jks = δ pj δ qk − δ pk δ qj
1 1
B pq =  pqs  jks B jk = (δ pj δ qk − δ pk δ qj )B jk
2 2
1
= (δ pj δ qk B jk − δ pk δ qj B jk )
2
1
= (B pq − B qp )
2
= B anti
pq

Solución Alternativa:
Teniendo en cuenta que B qp =  qps a s , y que por definición se cumple que  pqs = − qps ,
concluimos que:
B pq =  pqs a s = − qps a s = −B qp ∴ B = −B T (antisimétrico)

Ejemplo 1.34
Demostrar que la operación A anti ⋅ A sym + A sym ⋅ A anti resulta un tensor antisimétrico.
Solución:
Denominando por B = A anti ⋅ A sym + A sym ⋅ A anti , y teniendo en cuenta que se cumple que
A anti = −(A anti ) T , A sym = (A sym ) T , concluimos que:

Universidad de Castilla- La Mancha Draft Por: Eduardo W. V. Chaves (2012)


Ciudad Real - España
1 TENSORES 23

B = A anti ⋅ A sym + A sym ⋅ A anti


= A anti ⋅ A sym − A sym ⋅ ( A anti ) T
= A anti ⋅ A sym − ( A anti ⋅ A sym ) T
= 2( A anti ⋅ A sym ) anti

Ejemplo 1.35
r r r
¿La relación n ⋅ T = T ⋅ n es válida siempre? Siendo T un tensor de segundo orden y n un
vector. En el supuesto de que la relación no sea válida, ¿para qué caso particular lo sería?
Solución:
r r
n ⋅ T = n i eˆ i ⋅ Tkl (eˆ k ⊗ eˆ l ) T ⋅ n = Tlk (eˆ l ⊗ eˆ k ) ⋅ n i eˆ i
= n i Tkl δ ik eˆ l y = n i Tlk δ ki eˆ l
= n k Tkl eˆ l = n k Tlk eˆ l
Con lo que comprobamos que n k Tkl ≠ n k Tlk , luego:
r r
n⋅ T ≠ T ⋅n
r r
La relación n ⋅ T = T ⋅ n sólo será válida cuando el tensor T sea simétrico.

Ejemplo 1.36
r r r r
Obtener el vector axil w asociado al tensor antisimétrico ( x ⊗ a ) anti . Expresar w en función
r r
de x y a .
r
Solución: Sea z un vector arbitrario, se cumple que:
r r r r r
( x ⊗ a ) anti ⋅ z = w ∧ z
r r r
donde w es el vector axil asociado a ( x ⊗ a ) anti . Teniendo en cuenta que:
r r 1 r r
[ r r
]
1 r r r r
( x ⊗ a ) anti = ( x ⊗ a ) − ( x ⊗ a ) T = [ x ⊗ a − a ⊗ x ]
2 2
podemos aún decir que:
1 r r r r r r r
[x ⊗ a − a ⊗ x ] ⋅ z = w ∧ z ⇒ [xr ⊗ ar − ar ⊗ xr ] ⋅ zr = 2wr ∧ zr
2
r r r r r r r r r r r
Recordar que, dados tres vectores a , b , c se cumple que: a ∧ (b ∧ c ) = (b ⊗ c − c ⊗ b) ⋅ a ,
r r r r r r r r
ver Ejemplo 1.16. Luego, se cumple que [x ⊗ a − a ⊗ x ] ⋅ z = z ∧ ( x ∧ a ) . Retomando nuestra
expresión anterior:
[xr ⊗ ar − ar ⊗ xr ] ⋅ zr = zr ∧ ( xr ∧ ar ) = (ar ∧ xr ) ∧ zr = 2wr ∧ zr
con lo cual, concluimos que:
r 1 r r r r
w = (a ∧ x ) es el vector axil asociado al tensor ( x ⊗ a ) anti
2

Universidad de Castilla- La Mancha Draft Por: Eduardo W. V. Chaves (2012)


Ciudad Real - España
24 PROBLEMAS RESUELTOS DE MECÁNICA DEL MEDIO CONTINUO

Ejemplo 1.37
Consideremos dos tensores antisimétricos W (1) y W ( 2) y sus vectores axil representados,
r r
respectivamente, por w (1) y w ( 2) . Demostrar que:
r r r r
W (1) ⋅ W ( 2 ) = ( w ( 2 ) ⊗ w (1) ) − ( w (1) ⋅ w ( 2) )1
[ ] r r
Tr W (1) ⋅ W ( 2 ) = −2( w (1) ⋅ w ( 2 ) )
Solución:
Teniendo en cuenta las propiedades de tensor antisimétrico, podemos decir que:
r r r
W (1) ⋅ a = w (1) ∧ a
r r r
a ⋅ W (1) = −a ∧ w (1)
T
r r r
r r r y W ( 2) ⋅ a = w ( 2 ) ∧ a
− a ⋅ W (1) = −a ∧ w (1)
r r r
a ⋅ W (1) = a ∧ w (1)
A continuación hacemos el producto escalar (a ⋅ W (1) )⋅ (W ( 2) ⋅ a), obteniendo que:
r r

(ar ⋅ W )⋅ (W
(1) ( 2)
⋅ a) = (a ∧ w (1) ) ⋅ ( w ( 2) ∧ a)
r r r r r

Continuaremos el desarrollo en notación indicial:


(a i Wij(1) )(W (jk1) a k ) = ( ijk a j wk(1) )( ipq w (p2) a q )
a i (Wij(1) W (jk1) )a k = a j ( ijk  ipq wk(1) w (p2 ) )a q
[
= a j (δ jp δ kq − δ jq δ kp ) wk(1) w (p2 ) a q ]
=aj [δ jp δ kq wk(1) w (p2) − δ jq δ kp wk(1) w (p2) a q]
=aj [w (1) ( 2 )
q wj − δ jq wk(1) wk( 2 ) a q ]
En notación tensorial la expresión anterior queda:
r
[ ]
r r r
[r r r
a ⋅ W (1) ⋅ W ( 2 ) ⋅ a = a ⋅ ( w ( 2 ) ⊗ w (1) ) − ( w (1) ⋅ w ( 2 ) )1 ] ⋅ ar
r r r r
con lo cual demostramos que: W (1) ⋅ W ( 2) = ( w ( 2) ⊗ w (1) ) − ( w (1) ⋅ w ( 2) )1 .
b)
[
Tr W (1) ⋅ W ( 2) ] [ r r r r
= Tr ( w ( 2) ⊗ w (1) ) − ( w (1) ⋅ w ( 2 ) )1 ]
[ r r
] r
[ r
= Tr ( w ( 2) ⊗ w (1) ) − Tr ( w (1) ⋅ w ( 2 ) )1 ]
r r r r
= ( w ( 2 ) ⋅ w (1) ) − ( w (1) ⋅ w ( 2) ) 1 [3
Tr21]
=3
r r
= −2( w (1) ⋅ w ( 2) )
Solución alternativa
En esta solución alternativa vamos utilizar las componentes. Para ellos consideremos:
 0 W12(1) W13(1)   0 − w3(1) w2(1) 
 (1)   (1) 
Wij(1) = − W12(1) 0 W23  =  w3 0 − w1(1) 
− W (1) − W12(1) 0   − w2(1) w1(1) 0 
 12

Universidad de Castilla- La Mancha Draft Por: Eduardo W. V. Chaves (2012)


Ciudad Real - España
1 TENSORES 25

 0 W12( 2 ) W13( 2 )   0 − w3( 2 ) w2( 2 ) 


 (2)   (2) 
Wij( 2 ) = − W12( 2 ) 0 W23  =  w3 0 − w1( 2 ) 
− W ( 2 ) − W12( 2 ) 0  − w2( 2) w1( 2 ) 0 
 12

Con eso podemos obtener que:


 0 − w3(1) w2(1)   0 − w3( 2 ) w2( 2 ) 
[W (1)
⋅ W ( 2) ]ij = Wik(1) Wkj( 2 )

=  w3(1) 0

− w1(1)   w3( 2 ) 0

− w1( 2 ) 
− w (1) w1(1) 0  − w2( 2) w1( 2 ) 0 
 2
− w3(1) w3( 2 ) − w2(1) w2( 2 ) w2(1) w1( 2 ) w3(1) w1( 2 ) 
 
⇒ Wik(1) Wkj( 2 ) = w1(1) w2( 2) − w3( 2 ) w3(1) − w1(1) w1( 2 ) w2( 2) w3(1) 
 w3( 2 ) w1(1) w3( 2 ) w2(1) − w2(1) w2( 2) − w1( 2 ) w1(1) 

En el término (11) sumamos y restamos el término w1( 2) w1(1) , en el término (22) sumamos y
restamos el término w2( 2) w2(1) y en el término (33) sumamos y restamos el término w3( 2) w3(1) .
Con lo cual quedamos con:
 w1( 2) w1(1) w1( 2) w2(1) w1( 2 ) w3(1) 
 
Wik(1) Wkj( 2 ) =  w2( 2) w1(1) w2( 2) w2(1) w2( 2 ) w3(1)  +
 w ( 2) w (1) w3( 2) w2(1) w3( 2 ) w3(1) 
 3 1
 − w1(1) w1( 2 ) − w2(1) w2( 2) − w3(1) w3( 2) 0 0 
 
+ 0 − w1(1) w1( 2 ) − w2(1) w2( 2 ) − w3(1) w3( 2 ) 0 
 0 0 − w1(1) w1( 2 ) − w2(1) w2( 2 ) − w3(1) w3( 2 ) 

Que es igual a:
Wik(1) Wkj( 2 ) = wi( 2) w (j1) − ( w1(1) w1( 2) + w2(1) w2( 2) + w3(1) w3( 2) )δ ij = wi( 2) w (j1) − ( wk(1) wk( 2) )δ ij

Dejamos al lector comprobar la traza.


NOTA: La solución alternativa fue realizada solo como comprobación. El lector debe dar
preferencia a la solución tensorial o indicial, porque ni siempre la solución a través de
componentes es tan sencilla.

1.1.5 Cofactor. Adjunta. Traza. Tensores Particulares.


Determinante
Ejemplo 1.38
r r r r
Demostrar que Tr (a ⊗ b) = a ⋅ b .
Solución:
r r
[
Tr (a ⊗ b) = Tr (a i eˆ i ) ⊗ (b j eˆ j ) ]
[
= a i b j Tr eˆ i ⊗ eˆ j ]
= a i b j (eˆ i ⋅ eˆ j )
= a i b j δ ij
= aibi
r r
= a⋅b

Universidad de Castilla- La Mancha Draft Por: Eduardo W. V. Chaves (2012)


Ciudad Real - España
26 PROBLEMAS RESUELTOS DE MECÁNICA DEL MEDIO CONTINUO

Ejemplo 1.39
1
Dado que Tij = λE kk δ ij + 2µE ij , W = Tij E ij , y P = Tij Tij . Demostrar que:
2
λ
W = µE : E + [Tr( E )]2
2
P = 4µ 2 E : E + λ (3λ + 4µ)[Tr ( E )]
2

Solución 1: (Notación indicial)


1
2
1
2
( 1
2
) ( 1
) (
W = Tij E ij = λE kk δ ij + 2µE ij E ij = λE kk δ ij E ij + 2µE ij E ij = λE kk E ii + 2µE ij E ij
2
)
λ
Como E kk = E ii = Tr (E ) , y Eij Eij = E : E , concluimos que W = µE : E + [Tr( E )]2 .
2
( )(
P = Tij Tij = λE kk δ ij + 2µE ij λE qq δ ij + 2µE ij )
= λE kk δ ij λE qq δ ij + λE kk δ ij 2µE ij + 2µE ij λE qq δ ij + 2µE ij 2µE ij
= λ2 E kk δ ii E qq + 2µλE kk E ii + 2µλE ii E qq + 4µ 2 E ij E ij
= 3λ2 E kk E qq + 4µλE kk E ii + 4µ 2 E ij E ij
= λ (3λ + 4µ )E kk E qq + 4µ 2 E ij E ij

Con lo cual demostramos que P = 4µ 2 E : E + λ(3λ + 4µ)[Tr ( E )]2 .


Solución 2: (Notación tensorial)
En notación tensorial tenemos que:
1
T = λTr ( E )1 + 2µE , W = T : E , y P = T : T
2
Luego:
1 1 1
W = T : E = (λTr ( E )1 + 2µE ) : E = (λTr ( E )1 : E + 2µE : E )
2 2 2
1
= (λTr ( E ) Tr ( E ) + 2µE : E )
2
λ
= [Tr ( E )] + µE : E
2

2
P = T : T = (λTr ( E )1 + 2µE ) : (λTr ( E )1 + 2µE )
= [λTr ( E )] 1
2
{ : 1 + 2µλTr ( E ) 1
{ : E + 2µλTr ( E ) {
E : 1 + (2µ) 2 E : E
=3 = Tr ( E ) = Tr ( E )

= 3λ [Tr ( E )] + 4µλ[Tr ( E )] + 4µ E : E
2 2 2 2

= λ (3λ + 4µ )[Tr ( E )] + 4µ 2 E : E
2

Ejemplo 1.40
Sea un tensor de segundo orden σ ij que es una función del tensor ε ij , σ ij = σ ij (ε ij ) , y viene
dado por:
σ ij = λε kk δ ij + 2µε ij Tensorial
 → σ = λTr (ε )1 + 2µε

Universidad de Castilla- La Mancha Draft Por: Eduardo W. V. Chaves (2012)


Ciudad Real - España
1 TENSORES 27

donde λ , µ son constantes positivas. Partiendo de la expresión anterior, obtener la expresión


de ε ij en función de σ ij , es decir, ε ij = ε ij (σ ij ) . Expresar el resultado en notación indicial y
tensorial.
Solución:
Notación Indicial Notación Tensorial
σ ij = λε kk δ ij + 2µε ij σ = λTr (ε )1 + 2µε
⇒ 2µε ij = σ ij − λε kk δ ij ⇒ 2µε = σ − λTr (ε )1
1 λ 1 λ
⇒ ε ij = σ ij − ε kk δ ij ⇒ε= σ− Tr (ε )1
2µ 2µ 2µ 2µ

Tenemos que obtener la siguiente traza ε kk , para ello obtenemos la traza de σ ij :

Notación Indicial Notación Tensorial


σ ij = λε kk δ ij + 2µε ij
⇒ σ ii = λε kk δ ii + 2µε ii = λε kk 3 + 2µε kk σ : 1 = λTr (ε )1 : 1 + 2µε : 1
⇒ σ kk = (3λ + 2µ )ε kk Tr (σ ) = λTr (ε )3 + 2µTr (ε )
1 1
⇒ ε kk = σ kk ⇒ Tr (ε ) = Tr (σ )
(3λ + 2µ ) (3λ + 2µ )

Luego:
Notación Indicial Notación Tensorial
1 λ 1 λ
ε ij = σ ij − ε kk δ ij ε= σ− Tr (ε )1
2µ 2µ 2µ 2µ
1 λ 1 1 λ
= σ ij − σ kk δ ij ε= σ− Tr (σ )1
2µ 2µ (3λ + 2µ) 2µ 2µ(3λ + 2µ)

Ejemplo 1.41
Demostrar las siguientes identidades:
(T ) = (T )
m T T m
; ( )
Tr T T
m
( )
= Tr T m .
Solución:
(T )
m T
= (T ⋅ T L T ) = T T ⋅ T T L T T = T T
T
( ) m
c.q.d.
Para la segunda demostración utilizaremos la propiedad de la traza Tr (T T ) = Tr (T )

( )
Tr T T
m
= Tr T m ( ) T
( )
= Tr T m
c.q.d.

Ejemplo 1.42
Demostrar que: T : 1 = Tr (T ) .
Solución:

Universidad de Castilla- La Mancha Draft Por: Eduardo W. V. Chaves (2012)


Ciudad Real - España
28 PROBLEMAS RESUELTOS DE MECÁNICA DEL MEDIO CONTINUO

T : 1 = Tij eˆ i ⊗ eˆ j : δ kl eˆ k ⊗ eˆ l
= Tij δ kl δ ik δ jl
= Tij δ ij = Tii = T jj
= Tr ( T ) c.q.d.

Ejemplo 1.43
Probar que, si σ y D son tensores de segundo orden, la siguiente relación es válida:
σ ⋅ ⋅ D = Tr (σ ⋅ D )
Solución: Partimos de la siguiente definición:
σ ⋅ ⋅ D = σ ij D ji
= σ kj D jl δ ik δ il = σ kj D jl δ lk
= σ kj D jl δ lk
123
( σ⋅D )
kl
= (σ ⋅ D) kl δ lk = (σ ⋅ D) kk = (σ ⋅ D) ll
= Tr (σ ⋅ D) c.q.d.
Una segunda alternativa para la demostración sería:
σ ⋅ ⋅ D = σ ij D ji = σ ij D jk δ ik
= (σ ⋅ D ) : 1
= Tr (σ ⋅ D )
c.q.d.

Ejemplo 1.44
Demostrar que:
1
det (S ) ≡ S =  rjk  tpq S rt S jp S kq (1.22)
6
Solución:
det (S ) =  ijk S 1i S 2 j S 3k (1.23)
 pqr det(S ) =  ijk S pi S qj S rk (1.24)
 pqr  pqr det (S ) =  pqr  ijk S pi S qj S rk
1
424
3 (1.25)
6

1
det (S ) =  pqr  ijk S pi S qj S rk (1.26)
6

Ejemplo 1.45
Demostrar que: A  tpq =  rjk A rt A jp A kq
Solución: Sabemos que:
A =  rjk A r1 A j 2 A k 3
(1.27)
A  tpq =  rjk  tpq A r1 A j 2 A k 3

Universidad de Castilla- La Mancha Draft Por: Eduardo W. V. Chaves (2012)


Ciudad Real - España
1 TENSORES 29

La expresión  rjk  tpq podrá ser expresada en función de la delta de Kronecker como:

δ rt δ rp δ rq
 rjk  tpq = δ jt δ jp δ jq
(1.28)
δ kt δ kp δ kq
= δ rt δ jp δ kq + δ rp δ jq δ kt + δ rq δ jt δ kp − δ rq δ jp δ kt − δ jq δ kp δ rt − δ kq δ jt δ rp
Reemplazando en la expresión anterior (1.28) en la expresión (1.27), y utilizando la propiedad
del operador de sustitución obtenemos que:
A  tpq = A t1 A p 2 A q 3 + A p1 A q 2 A t 3 + A q1 A t 2 A p 3 − A q1 A p 2 A t 3 − A t1 A q 2 A p 3 − A p1 A t 2 A q 3
( ) ( ) (
= A t1  1 jk A pj A qk + A t 2  2 jk A pj A qk + A t 3  3 jk A pj A qk )
=  rjk A rt A jp A kq =  rjk A tr A pj A qk
c.q.d.

Ejemplo 1.46
1
Demostrar que: A =  rjk  tpq A rt A jp A kq
6
Solución:
Partiendo del problema anterior: A  tpq =  rjk A rt A jp A kq y multiplicando ambos lados por  tpq ,
resulta:
A  tpq  tpq =  rjk  tpq A rt A jp A kq (1.29)
Utilizando la propiedad  tpq  tpq = δ tt δ pp − δ tp δ tp = δ tt δ pp − δ tt = 6 . Luego, la relación (1.29)
resulta:
1
A =  rjk  tpq A rt A jp A kq
6 c.q.d.

Ejemplo 1.47
Demostrar que la siguiente propiedad es válida:
r r r r r r r r r r r r
(B ⋅ a) ⋅ (b ∧ c) − (B ⋅ b) ⋅ (a ∧ c) + (B ⋅ c ) ⋅ (a ∧ b) = Tr (B) a ⋅ (b ∧ c) [ ] (1.30)

Solución:
El lado derecho de la ecuación en notación indicial queda:
r r r
 ijk (B ⋅ a) i b j c k −  ijk (B ⋅ b) i a j c k +  ijk (B ⋅ c ) i a j b k =
⇒=  ijk [(B i1a1 + B i 2 a 2 + B i 3 a 3 )b j c k − (B i1b1 + B i 2 b 2 + B i 3b 3 )a j c k +
+ (B i1 c 1 + B i 2 c 2 + B i 3 c 3 )a j b k ]
⇒=  ijk [(B i1a1b j c k + B i 2 a 2 b j c k + B i 3 a 3b j c k ) − (B i1b1 a j c k + B i 2 b 2 a j c k + B i 3b 3 a j c k ) +
+ (B i1 c 1 a j b k + B i 2 c 2 a j b k + B i 3 c 3 a j b k )]
⇒=  ijk [B i1 (a1b j c k − b1a j c k + c 1a j b k ) + B i 2 (a 2 b j c k − b 2 a j c k + c 2 a j b k ) +
+ B i 3 (a 3b j c k − b 3 a j c k + c 3 a j b k )]

Universidad de Castilla- La Mancha Draft Por: Eduardo W. V. Chaves (2012)


Ciudad Real - España
30 PROBLEMAS RESUELTOS DE MECÁNICA DEL MEDIO CONTINUO

⇒= (1 jk B 11 +  2 jk B 21 +  3 jk B 31 )(a1b j c k − b1a j c k + c 1 a j b k ) +


+ (1 jk B 12 +  2 jk B 22 +  3 jk B 32 )(a 2 b j c k − b 2 a j c k + c 2 a j b k ) + (1.31)
+ (1 jk B 13 +  2 jk B 23 +  3 jk B 33 )(a 3b j c k − b 3 a j c k + c 3 a j b k )

Notar que:
a1 a 2 a3
1 jk (a1b j c k − b1a j c k + c 1a j b k ) = b1 b 2 b 3 =  ijk a i b j c k
c1 c2 c3

 2 jk (a1b j c k − b1a j c k + c 1a j b k ) =  3 jk (a1b j c k − b1a j c k + c 1a j b k ) = 0


Con lo cual la ecuación (1.31) queda:

[ r r r
B 11 ijk a i b j c k + B 22  ijk a i b j c k + B 33  ijk a i b j c k = (B 11 + B 22 + B 33 ) ijk a i b j c k = Tr (B) a ⋅ (b ∧ c ) ]
Notar que también se cumple que:
r r r r r r r r r r r r
(B T ⋅ a) ⋅ (b ∧ c ) − (B T ⋅ b) ⋅ (a ∧ c ) + (B T ⋅ c ) ⋅ (a ∧ b) = Tr (B) a ⋅ (b ∧ c ) [ ]
ya que Tr (B) = Tr (B T ) . También es válido que:
r r r r r r r r r r r r
(B ⋅ a) ⋅ (b ∧ c ) + a ⋅ ((B ⋅ b) ∧ c) + a ⋅ (b ∧ (B ⋅ c )) = Tr (B) a ⋅ (b ∧ c ) [ ] (1.32)
r r r r r r r r r r r r
⇒ [(B ⋅ a), b, c ] + [a, (B ⋅ b), c ] + [a, b, (B ⋅ c )] = I B [a, b, c ]

Ejemplo 1.48
Demostrar que la siguiente propiedad es válida:
r
[ r r
] r r r
[
( A ⋅ a) ⋅ ( A ⋅ b) ∧ ( A ⋅ c ) = det ( A ) a ⋅ (b ∧ c ) ] (1.33)
r r r
donde A es un tensor de segundo orden no-singular, y a , b y c son vectores linealmente
independientes.
Solución:
A tensor no-singular ⇒ det( A ) ≡ A ≠ 0
r r r r r r
a , b , c linealmente independientes ⇒ a ⋅ b ∧ c ≠ 0 . ( )
Escribimos el triple producto escalar en notación indicial, i.e. a ⋅ b ∧ c =  ijk a i b j c k , y
r
(r r )
multiplicamos por ambos lados de la igualdad por el determinante del tensor A , resultando:
r r r
( )
a ⋅ b ∧ c A =  ijk a i b j c k A

Fue demostrado en el Ejemplo 1.46 que se cumple que A  ijk =  pqr A pi A qj A rk , con lo cual:

Universidad de Castilla- La Mancha Draft Por: Eduardo W. V. Chaves (2012)


Ciudad Real - España
1 TENSORES 31

r r r
( )
a ⋅ b ∧ c A =  ijk a i b j c k A
=  pqr A pi A qj A rk a i b j c k
=  pqr ( A pi a i )( A qj b j )( A rk c k )
r
[ r
= ( A ⋅ a) ⋅ ( A ⋅ b) ∧ ( A ⋅ c )
r
]
Ejemplo 1.49
Demostrar que:

( r r
)
r r
det µ1 + αa ⊗ b = µ 3 + µ 2 α a ⋅ b (1.34)
r r
donde α y µ son escalares, a y b vectores y 1 es el tensor identidad de segundo orden.
Solución: Si denotamos por A ij µδ ij + αa i b j , el determinante de A viene dado por
A =  ijk A i1 A j 2 A k 3 , donde A i1 = µδ i1 + αa i b1 , A j 2 = µδ j 2 + αa j b 2 y A k 3 = µδ k 3 + αa k b 3 ,
luego podemos decir que:
( r r
) (
det µ1 + αa ⊗ b =  ijk (µδ i1 + αa i b1 ) µδ j 2 + αa j b 2 (µδ k 3 + αa k b 3 ) ) (1.35)
Desarrollando la expresión (1.35) obtenemos que:
( r r
) [
det µ1 + αa ⊗ b =  ijk µ 3 δ i1δ j 2 δ k 3 + µ 2 αa k b 3 δ i1δ j 2 + µ 2 αa j b 2 δ i1δ k 3 + µ 2 αa i b 1δ j 2 δ k 3 +
+ µα 2 a j b 2 a k b 3 δ i1 + µα 2 a i a k b 1b 3 δ j 2 + µα 2 a i a j b1b 2 δ k 3 + α 3 a i a j a k b1b 2 b 3 ]
Observemos que:
µ 3  ijk δ i1δ j 2 δ k 3 = µ 3 123 = µ 3
µ 2 α ( ijk a k b 3 δ i1δ j 2 +  ijk a j b 2 δ i1δ k 3 +  ijk a i b1δ j 2 δ k 3 ) =
r r
µ 2 α (12 k a k b 3 + 1 j 3 a j b 2 +  i 23 a i b1 ) = µ 2 α (a 3 b 3 + a 2 b 2 + a1b1 ) = µ 2 α (a k b k ) = µ 2 α (a ⋅ b)
 ijk a i a k b1b 3 δ j 2 =  i 2 k a i a k b1b 3 = a1a 3b1b 3 − a 3 a1b1b 3 = 0
 ijk a i a j b1b 2 δ k 3 =  ij 3 a i a j b1b 2 =  123 a1 a 2 b1b 2 −  213 a 2 a1b1b 2 = 0
 ijk a i a j a k b1b 2 b 3 = 0
Fijemos que no hacía falta expandir los términos  ijk a i a k b1b 3 δ j 2 ,  ijk a i a j b1b 2 δ k 3 ,
 ijk a i a j a k b1b 2 b 3 , para saber que son iguales a cero, ya que
r r
 ijk a i a k b1b 3 δ j 2 = (a ∧ a) j b1b 3 δ j 2 = 0 y análogamente para los otros términos. Con lo que
hemos demostrado que:
( r r r r
det µ1 + αa ⊗ b = µ 3 + µ 2 α a ⋅ b) c.q.d.
Para µ = 1 tenemos que:

( r r r r
det 1 + αa ⊗ b = 1 + α a ⋅ b )
Análogamente, se puede demostrar que:
r r
( )
det αa ⊗ b = α 3  ijk a i a j a k b1b 2 b 3 = 0

NOTA: Podemos extrapolar la expresión (1.34) de tal forma que:

Universidad de Castilla- La Mancha Draft Por: Eduardo W. V. Chaves (2012)


Ciudad Real - España
32 PROBLEMAS RESUELTOS DE MECÁNICA DEL MEDIO CONTINUO

( )
det µI sym + αA ⊗ B = µ 3 + µ 2 α A : B (1.36)

donde I sym es el tensor identidad de cuarto orden simétrico, A y B son tensores de segundo
orden. Notar que det ( I sym ) = (1) 3 + (1) 2 (0)(0 : 0 ) = 1 y det (1 ⊗ 1) = (0) 3 + (0) 2 (1)(1 : 1) = 0 .

Ejemplo 1.50
r r r r
Dado un tensor A , demostrar que existe un vector no nulo n ≠ 0 tal que A ⋅ n = 0 si y solo si
det ( A ) = 0 , Chadwick (1976).

Solución: Partimos del hecho que det ( A ) ≡ A = 0 y también escogemos una base arbitrario
r r r
{f , g, h} (linealmente independiente):

(
r r r
) r r
[ r
f ⋅ g ∧ h A = ( A ⋅ f ) ⋅ ( A ⋅ g) ∧ ( A ⋅ h) ] (ver Ejemplo 1.48)
Por el hecho que det ( A ) ≡ A = 0 , eso implica que:
r r
[
r
( A ⋅ f ) ⋅ ( A ⋅ g) ∧ ( A ⋅ h) = 0 ]
r r r
Con lo cual concluimos que los vectores ( A ⋅ f ) , ( A ⋅ g) , ( A ⋅ h) son linealmente
dependientes. Esto implica que existen escalares no nulos α ≠ 0 , β ≠ 0 , γ ≠ 0 tal que:
r r r r
α ( A ⋅ f ) + β ( A ⋅ g) + γ ( A ⋅ h) = 0
r r
(
⇒ A ⋅ αf + β g + γ h = 0
r r
)
r r
⇒ A ⋅n = 0
r r r r r
donde n = αf + βg + γh ≠ 0 .
r r r
Ahora escogemos dos vectores k , m que no son linealmente dependientes con n y
r r r r r r
reemplazamos esta base {k , m, n} en lugar de los vectores {a, b, c} :
r r r
( ) r
k ⋅ m ∧ h A = ( A ⋅ k ) ⋅ [( A ⋅ m) ∧ ( A ⋅ n)]
r r

r r r r r
( ) r r r
Considerando que A ⋅ n = 0 y que k ⋅ m ∧ h ≠ 0 , ya que la base {k , m, n} está constituida por
vectores linealmente independientes, obtenemos que:
(
r r r
k⋅ m∧h A =0
14243
) ⇒ A =0
≠0 c.q.d.

Universidad de Castilla- La Mancha Draft Por: Eduardo W. V. Chaves (2012)


Ciudad Real - España
1 TENSORES 33

Ejemplo 1.51
Sea un tensor de segundo orden arbitrario F . Demostrar que los tensores resultantes
C = F T ⋅ F y b = F ⋅ F T son tensores simétricos y semi-definidos positivos. Verificar también en que
condiciones C y b son tensores definidos positivos.
Solución: Para demostrar que los tensores son simétricos, tenemos que demostrar que C = C T
y b = bT :
C T = (F T ⋅ F )T = F T ⋅ (F T )T = F T ⋅ F = C
(simetría)
b T = (F ⋅ F T ) T = (F T )T ⋅ F T = F ⋅ F T = b

Con lo cual hemos demostrado que los tensores C = F T ⋅ F y b = F ⋅ F T son simétricos.


Para demostrar que los tensores C = F T ⋅ F y b = F ⋅ F T son semi-definidos positivos,
partimos de la definición de un tensor semi-definido positivo, es decir, un tensor A es semi-
r r r r
definido positivo si se cumple que x ⋅ A ⋅ x ≥ 0 , para todo x ≠ 0 . Luego:
r r r r r r r r
x ⋅ C ⋅ x = x ⋅ (F T ⋅ F ) ⋅ x x ⋅ b ⋅ x = x ⋅ (F ⋅ F T ) ⋅ x
r r r r
= (F ⋅ x ) ⋅ (F ⋅ x ) = (F T ⋅ x ) ⋅ (F T ⋅ x )
r 2 r 2
= F ⋅x ≥0 = FT ⋅x ≥0

En notación indicial:
x i C ij x j = x i ( Fki Fkj ) x j x i bij x j = x i ( Fik F jk ) x j
= ( Fki x i )( Fkj x j ) = ( Fik x i )( F jk x j )
2 2
= Fki x i ≥0 = Fik x i ≥0

Con lo cual demostramos que C = F T ⋅ F y b = F ⋅ F T son semi-definidos positivos.


r r r r r r r
Observemos que x ⋅ C ⋅ x = F ⋅ x sólo será igual a cero, con x ≠ 0 , si F ⋅ x = 0 , y por
2

r r r r
definición F ⋅ x = 0 con x ≠ 0 , si y solo si det ( F ) = 0 , ver Ejemplo 1.50. Luego, los tensores
C = F T ⋅ F y b = F ⋅ F T serán tensores definidos positivos si y solo si det ( F ) ≠ 0 .

Ejemplo 1.52
r r r r r r
Consideremos los siguientes vectores dX (1) , dX ( 2) , dX (3) , dx (1) , dx ( 2) , dx (3) , en el cual están
r r r r
relacionados entre si a través de las transformaciones dx (1) = F ⋅ dX (1) , dx ( 2) = F ⋅ dX ( 2) ,
r r
dx (3) = F ⋅ dX (3) , donde F es un tensor de segundo orden no-singular y ∃F −1 . a.1) Obtener
la relación entre los escalares dV y dV0 en función de F , sabiendo que
dV = dx (1) ⋅ (dx ( 2 ) ∧ dx (3) ) ≠ 0 , dV0 = dX (1) ⋅ (dX ( 2 ) ∧ dX (3) ) ≠ 0 . a.2) Obtener la relación entre
r r r r r r
r r r r r r r r
c = dX ( 2 ) ∧ dX (3) ≠ 0 y c * = dx ( 2 ) ∧ dx (3) ≠ 0 .
Solución
a.1) Teniendo en cuenta el enunciado se cumple que:
r r r r r
[ r
dV = dx (1) ⋅ (dx ( 2 ) ∧ dx (3) ) = ( F ⋅ dX (1) ) ⋅ ( F ⋅ dX ( 2 ) ) ∧ ( F ⋅ dX (3) ) ]
Recordar que en el Ejemplo 1.48 hemos demostrado que

Universidad de Castilla- La Mancha Draft Por: Eduardo W. V. Chaves (2012)


Ciudad Real - España
34 PROBLEMAS RESUELTOS DE MECÁNICA DEL MEDIO CONTINUO

(
r r r
) r r
[ r
a ⋅ b ∧ c A = ( A ⋅ a) ⋅ ( A ⋅ b) ∧ ( A ⋅ c ) ]
r r
( r
) r r
[r
⇒ dX (1) ⋅ dX ( 2 ) ∧ dX (3) F = ( F ⋅ dX (1) ) ⋅ ( F ⋅ dX ( 2 ) ) ∧ ( F ⋅ dX (3) ) ]
Con eso concluimos que:
r r r r r
[ r
dV = dx (1) ⋅ (dx ( 2 ) ∧ dx (3) ) = ( F ⋅ dX (1) ) ⋅ ( F ⋅ dX ( 2 ) ) ∧ ( F ⋅ dX (3) ) ]
r
[ r
= F dX (1) ⋅ (dX ( 2) ∧ dX (3) )
r
]
Luego:
dV = F dV0
a.2) Teniendo en cuenta la ecuación anterior, podemos obtener que:
dV = F dV0
r r r r r r
⇒ dx (1) ⋅ (dx ( 2 ) ∧ dx (3) ) = F dX (1) ⋅ dX ( 2 ) ∧ dX (3) [ ]
r r r r r r
⇒ dx (1) ⋅ (dx ( 2 ) ∧ dx (3) ) = F ( F −1 ⋅ dx (1) ) ⋅ dX ( 2) ∧ dX (3)[ ]
r r r r
(
r r
⇒ dx (1) ⋅ (dx ( 2 ) ∧ dx (3) ) = dx (1) ⋅ F F −T ⋅ dX ( 2 ) ∧ dX (3) [ ])
r r r r
⇒ (dx ( 2 ) ∧ dx (3) ) = F F −T ⋅ dX ( 2 ) ∧ dX (3)[ ]
r r
⇒ c * = F F −T ⋅ c
r r
NOTA: Notar que c * ≠ F ⋅ c . Podemos reescribir la ecuación anterior como
r r r
[ r
d x ( 2 ) ∧ d x ( 3 ) = F F −T ⋅ d X ( 2 ) ∧ d X ( 3 ) ]
r r r r
⇒ ( F ⋅ dX ( 2 ) ) ∧ ( F ⋅ dX (3) ) = F F −T ⋅ dX ( 2 ) ∧ dX (3) [ ]
El tensor F F −T es conocido como tensor cofactor de F , i.e. cof ( F ) = F F −T , con eso
definimos la inversa de un tensor como:
cof ( F ) = F F −T

[
⇒ F F −T ]
T
= [cof ( F )]
T

⇒ F F −1 = [cof ( F )]
T

1
⇒ F −1 = [cof (F )]T = 1 [adj(F )]
F F

F r r
r
dX (1) dx (1) = F ⋅ dX (1)

r r r
r c * = dx ( 2 ) ∧ dx ( 3 )
dX ( 3 ) r r
r r r c* ≠ F ⋅c r r
c = dX ( 2) ∧ dX (3) dx (3) = F ⋅ dX (3)
r r
c * = [cof ( F )] ⋅ c

dV = F dV0
r r r
dX ( 2 ) dx ( 2 ) = F ⋅ dX ( 2 )

r r r
dV0 = dX (1) ⋅ (dX ( 2 ) ∧ dX (3) ) ≠ 0 ⋅ (dx ( 2) ∧ dx (3) ) ≠ 0
r r r
−1 dV = dx (1)
F

Universidad de Castilla- La Mancha Draft Por: Eduardo W. V. Chaves (2012)


Ciudad Real - España
1 TENSORES 35

Ejemplo 1.53
Demostrar que si A y B son tensores ortogonales, el tensor resultante de la operación
C = A ⋅ B resulta ser otro tensor ortogonal.
Solución: Por definición, un tensor ( C ) es ortogonal cuando se cumple que C −1 = C T :
C −1 = ( A ⋅ B) −1 = B −1 ⋅ A −1 = B T ⋅ A T = ( A ⋅ B) T = C T c.q.d.

Ejemplo 1.54
Demostrar que adj( A ⋅ B) = adj(B) ⋅ adj( A ) y cof( A ⋅ B) = [cof( A )] ⋅ [cof(B)] .
Solución:
Partiendo de la propia definición de la inversa podemos decir que:

B −1 ⋅ A −1 =
[adj(B)] ⋅ [adj(A )]
B A
⇒ A B B −1 ⋅ A −1 = [adj(B)] ⋅ [adj( A )] = [cof(B)] ⋅ [cof( A)]T
T

⇒ A B (A ⋅ B ) = [adj(B)] ⋅ [adj( A )] =
−1
( [cof(A)]⋅ [cof(B)] ) T
(1.37)

⇒AB
[adj(A ⋅ B)] = [adj(B)] ⋅ [adj(A)] = ([cof(A)] ⋅ [cof(B)])T
A ⋅B
⇒ adj( A ⋅ B) = [adj(B)] ⋅ [adj( A )] = ([cof( A )] ⋅ [cof(B)])
T

donde hemos utilizado la propiedad que A ⋅ B = A B . Además teniendo en cuenta la


definición de adjunta y cofactor concluimos que:

adj( A ⋅ B) = ([cof( A ⋅ B)]) = ([cof( A )] ⋅ [cof(B)]) ⇒ [cof( A ⋅ B)] = [cof( A )] ⋅ [cof(B)]


T T
(1.38)

Ejemplo 1.55
Demostrar que:
r r r r
( A ⋅ a) ∧ ( A ⋅ b) = [cof( A )] ⋅ (a ∧ b) (1.39)

Solución:
Partiendo de la relación A  tpq =  rjk A rt A jp A kq (ver Ejemplo 1.45), y multiplicando ambos
lados por a t b p , resultando:
A  tpq a t b p =  rjk A rt A jp A kq a t b p =  rjk ( A rt a t )( A jp b p ) A kq

Multiplicamos ambos lados por A −qs1 obtenemos que:

A  tpq a t b p A −qs1 =  rjk ( A rt a t )( A jp b p ) A kq A −qs1 =  rjk ( A rt a t )( A jp b p )δ ks =  rjs ( A rt a t )( A jp b p )

−1
[cof ( A )] sq
Notar que A qs = , con lo cual la ecuación anterior queda:
A

Universidad de Castilla- La Mancha Draft Por: Eduardo W. V. Chaves (2012)


Ciudad Real - España
36 PROBLEMAS RESUELTOS DE MECÁNICA DEL MEDIO CONTINUO

−1
[cof ( A )] sq
A  tpq a t b p A qs = A  tpq a t b p = [cof ( A )] sq  tpq a t b p =  sjk ( A rt a t )( A jp b p )
A
r r r r
⇒ [cof( A )] ⋅ (a ∧ b) = ( A ⋅ a) ∧ ( A ⋅ b )

Ejemplo 1.56
Demostrar que:
r
[ r r
] r r
[ r r
] r r
[ ] r r r
a ⋅ ( A ⋅ b) ∧ ( A ⋅ c ) + ( A ⋅ a) ⋅ b ∧ ( A ⋅ c ) + ( A ⋅ a) ⋅ ( A ⋅ b) ∧ c = Tr ([cof ( A )]) a ⋅ (b ∧ c ) [ ]
(1.40)
Solución:
r r r r
En el ejemplo anterior hemos demostrado que [cof( A )] ⋅ (a ∧ b) = ( A ⋅ a) ∧ ( A ⋅ b) , luego se
cumplen que:
r r r r
[r r
a ⋅ [cof( A )] ⋅ (b ∧ c ) = a ⋅ ( A ⋅ b) ∧ ( A ⋅ c ) ]
r r r r
[
− b ⋅ [cof( A )] ⋅ (a ∧ c ) = −b ⋅ [( A ⋅ a) ∧ ( A ⋅ c ) ] = ( A ⋅ a) ⋅ b ∧ ( A ⋅ c )
r r r r r
]
r r r r
[r r
]r r
c ⋅ [cof( A )] ⋅ (a ∧ b) = c ⋅ ( A ⋅ a) ∧ ( A ⋅ b) = ( A ⋅ a) ⋅ ( A ⋅ b) ∧ c [
r
]
Sumando las relaciones anteriores, podemos obtener que:
r r r r r r r r r
a ⋅ [cof( A )] ⋅ (b ∧ c ) − b ⋅ [cof( A )] ⋅ (a ∧ c ) + c ⋅ [cof( A )] ⋅ (a ∧ b) =
r
[ r r
]r r
[r r
] r r
= a ⋅ ( A ⋅ b) ∧ ( A ⋅ c ) + ( A ⋅ a) ⋅ b ∧ ( A ⋅ c ) + ( A ⋅ a) ⋅ ( A ⋅ b) ∧ c [ ]
Según el Ejemplo 1.47 se cumple que:
([cof( A )] ⋅ ar )⋅ (b ∧ cr ) − ([cof( A )] ⋅ b)⋅ (ar ∧ cr ) + ([cof( A )] ⋅ cr )⋅ (ar ∧ b) = Tr ([cof( A )])[rcr ⋅ (ar ∧ b)]
T
r T
r T
r r
r r
= II A [c ⋅ (a ∧ b)]
donde II A = Tr [cof( A )] es el segundo invariante principal de A . Con lo cual demostramos
que:
r
[ r r
] r r
[ r r
] r r
[ r r r
]
a ⋅ ( A ⋅ b) ∧ ( A ⋅ c ) + ( A ⋅ a) ⋅ b ∧ ( A ⋅ c ) + ( A ⋅ a) ⋅ ( A ⋅ b) ∧ c = II A a ⋅ (b ∧ c ) [ ]
NOTA 1: Podemos resumir que:
r r r r
[ r r r r
] [ r
] [
r r r
( A ⋅ a) ⋅ (b ∧ c) + a ⋅ ( A ⋅ b) ∧ c ) + a ⋅ b ∧ ( A ⋅ c ) = I A a ⋅ (b ∧ c ) ] (ver Ejemplo 1.47) (1.41)

a ⋅ [( A ⋅ b) ∧ ( A ⋅ c)] + ( A ⋅ a) ⋅ [b ∧ ( A ⋅ c )] + ( A ⋅ a) ⋅ [( A ⋅ b) ∧ c ] = II [a ⋅ (b ∧ c )]
r r r r r r r r r r r r
A
(1.42)

( A ⋅ a) ⋅ [( A ⋅ b) ∧ ( A ⋅ c )] = III [a ⋅ (b ∧ c )] (ver Ejemplo 1.48)


r r r r r r
A
(1.43)

donde I A = Tr (A ) , II A = Tr ([cof( A )]) , III A = det (A ) .


r r r r r r
NOTA 2: Si consideramos tres vectores linealmente independientes [a ⋅ (b ∧ c )] ≡ [a, b, c ] ≠ 0 ,
y tres vectores tal que:

Universidad de Castilla- La Mancha Draft Por: Eduardo W. V. Chaves (2012)


Ciudad Real - España
1 TENSORES 37

r r r r
f = α 1a + α 2 b + α 3 c
 r r r r
g = β 1 a + β 2 b + β 3 c
r r r r
h = γ 1a + γ 2 b + γ 3 c
Según la regla de Cramer, (ver Ejemplo 1.15), se cumple que:
r r r r r r r r r r r r
f ⋅ (b ∧ c ) [ f , b, c ] [a, f , c ] [a, b, f ]
α1 = r r r ≡ r r r ; α2 = r r r ; α3 = r r r
a ⋅ (b ∧ c ) [a, b, c ] [a, b, c ] [a, b, c ]
r r r r r r r r r
[g, b, c ] [a, g, c ] [a, b, g]
β1 = r r r ; β2 = r r r ; β3 = r r r
[a, b, c ] [a, b, c ] [a, b, c ]
r r r r r r r r r
[h, b, c ] [a, h, c ] [a, b, h]
γ1 = r r r ; γ2 = r r r ; γ3 = r r r
[a, b, c ] [a, b, c ] [a, b, c ]
r r r r r r
Desarrollando el triple producto escalar [ f ⋅ (g ∧ h)] ≡ [f , g, h] , podemos obtener que:

r r r α 1 α2 α3 
β r r r
[ f ⋅ (g ∧ h)] =  1 β 2 β 3  [a, b, c ]
 γ 1 γ 2 γ 3 
r r r r r r r r r
 [ f , b, c ] [a, f , c ] [a, b, f ] 
1  r r r r r r r r r  r r r r r r
= r r r [g, b, c ] [a, g, c ] [a, b, g] [a, b, c ] = P [a, b, c ]
[a, b, c ]  r r r r r r r r r
[h, b, c ] [a, h, c ] [a, b, h]
 
r r r r r r
Para el caso cuando f = A ⋅ a , g = A ⋅ b , h = A ⋅ c , los invariantes principales de P vienen
dados por:
1
( r r r r r r r r
I P = Tr ( P ) = r r r [ A ⋅ a, b, c] + [a, A ⋅ b, c] + [a, b, A ⋅ c ] = I A
[a, b, c]
r
)
r r r r r r r r r r r r r r r r r r
1  [a , A ⋅ b, c ] [ a , b , A ⋅ b ] [ A ⋅ a, b , c ] [ a , b , A ⋅ a ] [ A ⋅ a , b, c ] [a, A ⋅ a, c] 
II P = r r r  r r r r r r + r rr r r r + r r r r r r
([a, b, c ]) 2  [a, A ⋅ c, c ] [a, b, A ⋅ c ] [ A ⋅ c, b, c] [a, b, A ⋅ c ] [ A ⋅ b, b, c ] [a, A ⋅ b, c ] 
= II A
III P = III A = det (A )

1.1.6 Descomposición Aditiva de Tensores

Ejemplo 1.57
Encontrar un tensor de cuarto orden P tal que se cumpla que:
P : A = A dev
Solución: Teniendo en cuenta la descomposición aditiva de un tensor en una parte esférica y
otra desviadora, podemos obtener que:
Tr ( A ) Tr ( A )
A = A esf + A dev = 1 + A dev ⇒ A dev = A − 1
3 3

Universidad de Castilla- La Mancha Draft Por: Eduardo W. V. Chaves (2012)


Ciudad Real - España
38 PROBLEMAS RESUELTOS DE MECÁNICA DEL MEDIO CONTINUO

Recurriendo a la definición de los tensores identidades de cuarto orden definidos por:


I = 1⊗1 = δ ik δ jl eˆ i ⊗ eˆ j ⊗ eˆ k ⊗ eˆ l = I ijkl eˆ i ⊗ eˆ j ⊗ eˆ k ⊗ eˆ l (1.44)
I = 1⊗1 = δ il δ jk eˆ i ⊗ eˆ j ⊗ eˆ k ⊗ eˆ l = I ijkl eˆ i ⊗ eˆ j ⊗ eˆ k ⊗ eˆ l (1.45)

I = 1 ⊗ 1 = δ ij δ kl eˆ i ⊗ eˆ j ⊗ eˆ k ⊗ eˆ l = I ijkl eˆ i ⊗ eˆ j ⊗ eˆ k ⊗ eˆ l (1.46)

donde se cumple que:


( )(
I : A = δ ik δ jl eˆ i ⊗ eˆ j ⊗ eˆ k ⊗ eˆ l : A pq eˆ p ⊗ eˆ q )
(
= δ ik δ jl A pq δ kp δ lq eˆ i ⊗ eˆ j )
(
= δ ik δ jl A kl eˆ i ⊗ eˆ j ) (1.47)
(
= A ij eˆ i ⊗ eˆ j )
=A
( )(
I : A = δ ij δ kl eˆ i ⊗ eˆ j ⊗ eˆ k ⊗ eˆ l : A pq eˆ p ⊗ eˆ q )
(
= δ ij δ kl A pq δ kp δ lq eˆ i ⊗ eˆ j )
(
= δ ij δ kl A kl eˆ i ⊗ eˆ j ) (1.48)
(
= A kk δ ij eˆ i ⊗ eˆ j )
= Tr ( A )1
Entonces, podemos decir que:
Tr ( A ) 1  1   1 
A dev = A − 1 = I : A − I : A =  I − I  : A =  I − 1 ⊗ 1 : A
3 3  3   3 
Con lo cual, concluimos que:
1
P = I − 1 ⊗1
3
El tensor P es conocido como tensor proyección de cuarto orden, Holzapfel (2000).

1.1.7 Ley de Transformación. Invariantes.


Ejemplo 1.58
Probar que I T = Tii es un invariante bajo un cambio de base.
Solución: Considerando la ley de transformación para un tensor de segundo orden, podemos
decir que: Tij′ = a ik a jl Tkl . Luego, Tii′ vendrá dado por:
Tii′ = a ik a il Tkl = δ kl Tkl = Tkk = I T
Lo que demuestra que I T es un invariante.

Ejemplo 1.59
Bajo la transformación de base eˆ ′i = a ij eˆ j y de las componentes del tensor de segundo orden
simétrico T en esta nueva base Tij′ = a ik a jl Tkl . Mostrar que I T , II T , III T son invariantes:

I T = Tr ( T ) = Tii ; II T =
1
2
{2
}
I T − Tr ( T 2 ) ; III T = det ( T )

Universidad de Castilla- La Mancha Draft Por: Eduardo W. V. Chaves (2012)


Ciudad Real - España
1 TENSORES 39

Solución:
=j
a) Tij′ = a ik a jl Tkl i→ Tii′ = a ik a il Tkl = δ kl Tkl = Tkk = Tll

b) Como ya hemos demostrado que I T es un invariante, para demostrar que II T es invariante


es suficiente demostrar que Tr ( T 2 ) = Tr ( T ⋅ T ) = Tij T ji es un invariante.
Tij′ T ′ji = (a ik a jl Tkl )(a jp a iq T pq ) = a ik a iq a jl a jp Tkl T pq = δ kq δ lp Tkl T pq = Tqp T pq = Tkl Tlk
123 123
=δ kq =δ lp

con lo cual, demostramos que Tr ( T 2 ) = Tr ( T ⋅ T ) = Tij T ji es un invariante.


c) det ( Tij′ ) = det(a ik a jl Tkl ) = det (a ik )det (a jl )det ( Tkl ) = det( Tkl )
1
424
31424
3
=1 =1

Acabamos de demostrar que Tkk = Tr (T ) , Tkl Tlk = Tr ( T ⋅ T ) , det (T ) son invariantes.

Ejemplo 1.60
Demostrar que las siguientes relaciones son invariantes:
C12 + C 22 + C 32 ; C13 + C 23 + C 33 ; C14 + C 24 + C 34
donde C1 , C 2 , C 3 son los autovalores del tensor de segundo orden C .
Solución:
Cualquier combinación de los invariantes principales será un invariante. Intentaremos expresar
las relaciones anteriores en función de los invariantes principales.
Consideremos la siguiente relación:
2
1444424444
(
I C2 = (C1 + C 2 + C 3 ) = C12 + C 22 + C 32 + 2 C1 C 2 + C1 C 3 + C 2 C 3
3
)
II C

⇒ C12 + C 22 + C 32 = I C2 − 2 II C

Comprobando que C12 + C 22 + C 32 es un invariante. Análogamente, podemos obtener que:


C13 + C 23 + C 33 = I C3 − 3 II C I C + 3 III C
C14 + C 24 + C 34 = I C4 − 4 II C I C2 + 4 III C I C + 2 II C2

Ejemplo 1.61
Obtener las componentes de la siguiente operación:
T′ = A ⋅ T ⋅ AT
donde Tij y a ij son las componentes de los tensores T y A , respectivamente.

Solución: La expresión T ′ = A ⋅ T ⋅ A T en notación simbólica queda:


′ (eˆ a ⊗ eˆ b ) = a rs (eˆ r ⊗ eˆ s ) ⋅ T pq (eˆ p ⊗ eˆ q ) ⋅ a kl (eˆ l ⊗ eˆ k )
Tab
= a rs T pq a kl δ sp δ ql (eˆ r ⊗ eˆ k )
= a rp T pq a kq (eˆ r ⊗ eˆ k )

Universidad de Castilla- La Mancha Draft Por: Eduardo W. V. Chaves (2012)


Ciudad Real - España
40 PROBLEMAS RESUELTOS DE MECÁNICA DEL MEDIO CONTINUO

Para obtener las componentes de T ′ es suficiente hacer el doble producto escalar por la base
(eˆ i ⊗ eˆ j ) , resultando:

′ (eˆ a ⊗ eˆ b ) : (eˆ i ⊗ eˆ j ) = a rp T pq a kq (eˆ r ⊗ eˆ k ) : (eˆ i ⊗ eˆ j )


Tab
′ δ ai δ bj = a rp T pq a kq δ ri δ kj
Tab
Tij′ = a ip T pq a jq

Observemos que esta operación viene representada en forma matricial como:


T ′ = A T AT
Si A es la matriz de transformación entre bases ortonormales se cumple que A −1 = A T
luego, se cumple que T = A T T ′ A , y la representación de las componentes se muestran en
la Figura abajo:

T ′ = A T AT x3′

T33


T23

T32 ′
T22
x3
T33 T13′


T31 T12′ x2′

T13 ′
T21
T23 T11′
T32
T31 T22

T12
T21 x2 x1′
T11

x1
T = AT T ′ A

Ejemplo 1.62
Consideremos que las componentes de un tensor de segundo orden T , en el sistema de
referencia (x1 , x 2 , x 3 ) , están representadas por:
 3 − 1 0
(T )ij = Tij = T =  − 1 3 0 
 0 0 1 

Sabiendo que la matriz de transformación de coordenadas del sistema (x1 , x 2 , x 3 ) al sistema


(x1′ , x 2′ , x 3′ ) viene dada por:

Universidad de Castilla- La Mancha Draft Por: Eduardo W. V. Chaves (2012)


Ciudad Real - España
1 TENSORES 41

 
 0 0 1
 2 2 
A= 0
 2 2 
 2 2 
− 0
 2 2 
Obtener las componentes del tensor Tij en el nuevo sistema de coordenadas (x1′ , x 2′ , x 3′ ) .

Solución: La ley de transformación para un tensor de segundo orden es:


Tij′ = aik a jl Tkl

Para que la operación anterior sea posible en forma matricial:


Tij′ = ai k Tk l ( al j ) T
Luego:
T ′ = A T AT

 0  2 2
0 1 0 − 
  2 2 
 2   3 − 1 0  
2 2 2 
T ′=  0  − 1 3 0  0
 2 2   2 2 
 2 2   0 0 1   
− 0 1 0 0 
 2 2   
Efectuando la operación de matrices obtenemos que:
1 0 0 
T ′ = 0 2 0 
0 0 4 

Ejemplo 1.63
Encontrar la matriz de transformación del sistema ( x, y , z ) al sistema x′′′, y ′′′, z′′′ , ver Figura 1.1:

z = z′
z ′′ = z ′′′

y ′′′
β
γ y ′ = y ′′
α y

x α x′

γ
x ′′′
x ′′
Figura 1.1: Rotación.

Universidad de Castilla- La Mancha Draft Por: Eduardo W. V. Chaves (2012)


Ciudad Real - España
42 PROBLEMAS RESUELTOS DE MECÁNICA DEL MEDIO CONTINUO

Solución: Recordar que se cumple que: si tenemos un espacio inicial y sucesivas


transformaciones hasta el espacio final, la ley de transformación del espacio inicial al espacio
final viene formada por el producto de las transformaciones en el sentido contrario. Es decir,
situamos en el espacio final y seguimos el sentido contrario de las flechas hasta el espacio
inicial, ver figura abajo.

r
a′
A r
a′′

B −1
r
a A −1
C −1 C
espacio
inicial
r espacio
CBA a′′′
final

A −1B −1 C −1 = (CBA ) −1 
si ortogonale s
 → A T B T C T = ( CBA ) T

Podemos observar que la obtención del sistema x′′′, y ′′′, z′′′ es una combinación de rotaciones
mostradas a continuación:

♦ Rotación según eje z

z = z′

del sistema x, y , z al x′, y ′, z′


y′
α  cos α sin α 0
α
y A = − sin α cos α 0
 0 0 1 
x x′
con 0 ≤ α ≤ 360 º

Universidad de Castilla- La Mancha Draft Por: Eduardo W. V. Chaves (2012)


Ciudad Real - España
1 TENSORES 43

♦ Rotación según eje y′


del sistema x′, y ′, z′ al x′′, y ′′, z′′
z = z′

z ′′ cos β 0 − sin β 
B =  0 1 0 
 sin β 0 cos β 

β y ′ = y ′′ con 0 ≤ β ≤ 180 º

α y z = z′ z ′′
x α x′

x′
β

x ′′
x ′′

♦ Rotación según eje z ′′


z = z′
z ′′ = z ′′′

y ′′′ del sistema x′′, y ′′, z′′ al x′′′, y ′′′, z′′′


β
γ y ′ = y ′′
α y  cos γ sin γ 0

x α x′ C = − sin γ cos γ 0 


 0 0 1 
γ
con 0 ≤ γ ≤ 360º
x ′′′
x ′′
La matriz de transformación del sistema ( x, y , z ) para el sistema x′′′, y ′′′, z′′′ será dada por:
D = CBA
Resultando:
 (cos α cos β cos γ − sin α sin γ ) (sin α cos β cos γ + cos α sin γ ) − sin β cos γ 
D = (− cos α cos β sin γ − sin α cos γ ) (− sin α cos β sin γ + cos α cos γ ) sin β sin γ 

 cos α sin β sin α sin β cos β 

Los ángulos α , β , γ son conocidos como los ángulos de Euler.

Ejemplo 1.64
Si a ij representan las componentes de la matriz de transformación de base demostrar que las
siguientes ecuaciones se cumplen:

Universidad de Castilla- La Mancha Draft Por: Eduardo W. V. Chaves (2012)


Ciudad Real - España
44 PROBLEMAS RESUELTOS DE MECÁNICA DEL MEDIO CONTINUO

a11
2 2
+ a12 + a132
=1 a11
2 2
+ a 21 2
+ a 31 =1
 2 2 2
 2 2 2
a 21 + a 22 + a 23 = 1 a12 + a 22 + a 32 = 1
 2 2 2  2 2 2
a 31 + a 32 + a 33 = 1 a13 + a 23 + a 33 = 1
 ó 
a11 a 21 + a12 a 22 + a13 a 23 = 0 a11 a12 + a 21 a 22 + a 31 a 32 = 0
 
a 21 a31 + a 22 a 32 + a 23 a 33 = 0 a12 a13 + a 22 a 23 + a 32 a 33 = 0
a a + a a + a a = 0 a a + a a + a a = 0
 11 31 12 32 13 33  11 13 21 23 31 33

Solución:
Partimos del principio que la matriz de transformación de base es una matriz ortogonal, i.e.
a ik a jk = a ki a kj = δ ij . Con lo cual:

(i = 1, j = 1) 2
a11 2
+ a12 2
+ a13 =1
 2 2 2
(i = 2, j = 2) a 21 + a 22 + a 23 =1
 2 2 2
(i = 3, j = 3) a31 + a 32 + a 33 =1
a ik a jk = a i1 a j1 + a i 2 a j 2 + a i 3 a j 3 = δ ij ⇒ 
(i = 1, j = 2) a11 a 21 + a12 a 22 + a13 a 23 = 0

(i = 2, j = 3) a 21 a 31 + a 22 a 32 + a 23 a 33 = 0
(i = 1, j = 3) a11 a 31 + a12 a 32 + a13 a 33 = 0

Solución Alternativa:
 a11 a12 a13   a11 a 21 a 31  1 0 0 
AA = 1 T
⇒ a a 22 a 23   a12 a 22 a 32  = 0 1 0 
 21
 a 31 a 32 a 33   a13 a 23 a 33  0 0 1 

Al efectuar la multiplicación de matrices obtenemos que:


 2
a11 2
+ a12 2
+ a13 a11 a 21 + a12 a 22 + a13 a 23 a11 a 31 + a12 a 32 + a13 a 33  1 0 0 
 
 a11 a 21 + a12 a 22 + a13 a 23
2
a 21 2
+ a 22 2
+ a 23 a 21 a 31 + a 22 a 32 + a 23 a 33  = 0 1 0 
a a + a a + a a a 21 a 31 + a 22 a 32 + a 23 a 33 2
a 31 + a 32
2
+ a 33
2  0 0 1 
 11 31 12 32 13 33   

Universidad de Castilla- La Mancha Draft Por: Eduardo W. V. Chaves (2012)


Ciudad Real - España
1 TENSORES 45

1.1.8 Autovalores y Autovectores

Ejemplo 1.65
Demostrar que si Q es un tensor de segundo orden ortogonal propio, y E es un tensor de
segundo orden, los autovalores de E no cambian con la transformación:
E* = Q ⋅ E ⋅ QT
Solución:
Los autovalores ( λ i ) del tensor E obtenemos a partir del determinante característico:

(
0 = det E * − λ1 ) (
0 = det E *ij − λδ ij )
= det (Q ⋅ E ⋅ Q − λ1 )
T
= det (Q ik E kp Q jp − λδ ij )
= det (Q ⋅ E ⋅ Q − Q ⋅ λ1 ⋅ Q )
T T
= det (Q − λQ ik Q jp δ kp )
ik E kp Q jp
= det [Q ⋅ (E − λ1 ) ⋅ Q ] T
[ (
= det Q ik E kp − λδ kp Q jp ) ]
= det
12Q
(3) det (E − λ1 ) det
1424
(Q3) T
(
= (Q ik )det E kp − λδ kp )det (Q ) jp
1 1
= det (E − λ1 ) (
= det E kp − λδ kp )
Con lo que comprobamos que E y E * tienen los mismos autovalores.

Ejemplo 1.66
Sea A un tensor de segundo orden y Q un tensor ortogonal. Si la ley de transformación
ortogonal aplicada a A viene dada por A * = Q ⋅ A ⋅ Q T , demostrar que A 2 = Q ⋅ A 2 ⋅ Q T .
*

Solución:
A 2 = A* ⋅ A* ( A 2 ) ij = ( A * ⋅ A * ) ij = A *ik A *kj
* *

= ( Q ⋅ A ⋅ Q T ) ⋅ (Q ⋅ A ⋅ Q T ) = (Q ip A pr Q kr )(Q ks A st Q jt )
= Q⋅A ⋅Q ⋅Q⋅A ⋅Q
T T
= Q ip A pr Q kr Q ks A st Q jt
123 123
=1
=δ rs
= Q ⋅ A ⋅ A ⋅ QT
= Q ip A pr δ rs A st Q jt = Q ip A ps A st Q jt
= Q ⋅ A 2 ⋅ QT
= Q ip ( A ⋅ A ) pt Q jt
= (Q ⋅ A 2 ⋅ Q T ) ij

Ejemplo 1.67
Dadas las componentes del tensor T :
5 3 3
Tij = 2 6 3
2 2 4

Se pide:
a) Obtener los invariantes principales de T ;

Universidad de Castilla- La Mancha Draft Por: Eduardo W. V. Chaves (2012)


Ciudad Real - España
46 PROBLEMAS RESUELTOS DE MECÁNICA DEL MEDIO CONTINUO

b) Obtener el polinomio característico asociado a T ;


c) Si λ 1 , λ 2 y λ 3 son los autovalores de T y λ 1 = 10 . Obtener λ 2 y λ 3 > 2 .
Solución:
a) Los invariantes principales de T son:
I T = Tr ( T ) = 5 + 6 + 4 = 15

6 3 5 3 5 3
II T = + + = 56
2 4 2 4 2 6
III T = det ( T ) = 60
b) El polinomio característico se obtiene al resolver el determinante:
5−λ 3 3
2 6−λ 3 =0 ⇒ λ3 − λ2 I T + λ II T − III T = 0 luego:
2 2 4−λ

λ3 − 15λ2 + 56λ − 60 = 0
c) En el espacio principal se cumple que:
λ 1 = 10 0 0 
Tij′ =  0 λ2 0 

 0 0 λ 3 > 2
donde los invariantes principales son
I T = Tr ( T ) = λ 1 + λ 2 + λ 3 = 15 ⇒ λ2 + λ3 = 5
III T = det ( T ) = λ 1λ 2 λ 3 = 60 ⇒ λ 2λ3 = 6
Combinando estas dos ecuaciones:
λ 2λ3 = 6  2
λ(31) = 3
 ⇒ (5 − λ 3 )λ 3 = 6 ⇒ λ 3 − 5λ 3 + 6 = 0 ⇒  ( 2 )
λ 2 + λ 3 = 5 λ 3 = 2

Descartamos la solución λ(32) = 2 por la imposición del problema. Luego, λ 3 = 3 . Resumiendo


así:
10 0 0  I T = 10 + 2 + 3 = 15

Tij′ =  0 2 0 donde se puede comprobar que:  II T = 2 × 3 + 10 × 3 + 10 × 2 = 56
 0 0 3  III = 10 × 2 × 3 = 60
 T

Ejemplo 1.68
Determinar los valores principales y las direcciones principales del tensor cartesiano de
segundo orden T , cuyas componentes se representan matricialmente por:
 3 − 1 0
(T )ij = Tij = T =  − 1 3 0 
 0 0 1 

Universidad de Castilla- La Mancha Draft Por: Eduardo W. V. Chaves (2012)


Ciudad Real - España
1 TENSORES 47

Solución: Buscamos soluciones no triviales para (Tij − λδ ij ) n j = 0 i , con la restricción de que


n j n j = 1 . Como ya hemos visto, la solución no trivial requiere la condición:

Tij − λδ ij = 0

Explícitamente, la expresión anterior queda:


T11 − λ T12 T13 3 − λ −1 0
T21 T22 − λ T23 = −1 3 − λ 0 =0
T31 T32 T33 − λ 0 0 1− λ

Desarrollando el determinante anterior obtenemos la ecuación cúbica:


[
(1 − λ ) (3 − λ ) 2 − 1 = 0 ]
3 2
λ − 7 λ + 14λ − 8 = 0
Podríamos haber obtenido directamente la ecuación característica anterior a través de los
invariantes:
I T = Tr ( Tij ) = Tii = T11 + T22 + T33 = 7

T T23 T11 T13 T11 T12


II T =
1
2
( )
Tii T jj − Tij Tij = 22
T32 T33
+
T31 T33
+
T21 T22
= 14

III T = Tij =  ijk Ti1 T j 2 Tk 3 = 8

utilizando la ecuación característica será:


λ3 − λ2 I T + λ II T − III T = 0 → λ3 − 7λ2 + 14λ − 8 = 0
Resolviendo la ecuación cúbica podemos obtener las tres raíces reales, puesto que la matriz T
es simétrica:
λ 1 = 1; λ 2 = 2; λ3 = 4
Podemos además comprobar si los invariantes están bien calculados utilizando la expresión de
los invariantes en función de los autovalores:
I T = λ1 + λ 2 + λ 3 = 1 + 2 + 4 = 7 ✓
II T = λ 1 λ 2 + λ 2 λ 3 + λ 3 λ 1 = 1 × 2 + 2 × 4 + 4 × 1 = 14 ✓
III T = λ 1 λ 2 λ 3 = 8 ✓
Con lo que podemos comprobar que los invariantes son los mismos que los obtenidos
anteriormente.
Cálculo de las direcciones principales:
Para obtener las direcciones principales, utilizamos la definición de autovalor-autovector,
donde cada autovalor λ i está asociado a un autovector nˆ (i ) .
ŒPara λ 1 = 1
3 − λ 1 −1 0   n 1  3 − 1 − 1 0   n1   0 
 −1 3 − λ1 0  n 2  =  − 1 3 − 1 0  n 2  = 0 
   

 0 0 1 − λ 1  n 3   0 0 1 − 1 n 3  0 

resultando el siguiente sistema de ecuaciones:

Universidad de Castilla- La Mancha Draft Por: Eduardo W. V. Chaves (2012)


Ciudad Real - España
48 PROBLEMAS RESUELTOS DE MECÁNICA DEL MEDIO CONTINUO

2n1 − n 2 = 0 
  ⇒ n1 = n 2 = 0
 − n1 + 2n 2 = 0
0n = 0
 3
n i n i = n12 + n 22 + n 32 = 1

Luego, podemos obtener que: λ1 = 1 ⇒ nˆ (1) = [ 0 0 ±1] .


NOTA: Esta solución podría haberse determinado previamente por la situación particular que
presentan las componentes del tensor. Al ser los términos T13 = T23 = T31 = T32 = 0 , T33 = 1 ya
es un valor principal, como consecuencia esta dirección ya es una dirección principal. ■
Para λ 2 = 2
3 − λ 2 −1 0   n 1  3 − 2 − 1 0   n1   0 
 −1 3 − λ2 0  n  =  − 1 3 − 2 0  n 2  = 0 
  2  
 0 0 1 − λ 2  n 3   0 0 1 − 2  n 3  0 

n1 − n 2 = 0 ⇒ n1 = n 2

 − n1 + n 2 = 0
− n = 0
 3
Podemos observar que las dos primeras ecuaciones son linealmente dependientes.
Necesitamos entonces de una ecuación adicional:
1
n i n i = n12 + n 22 + n 32 = 1 ⇒ 2n12 = 1 ⇒ n1 = ±
2
Luego:
 1 1 
λ2 = 2 ⇒ nˆ (2) =  ± ± 0
 2 2 
Para λ 3 = 4
3 − λ 3 −1 0   n 1  3 − 4 − 1 0   n1   0 
 −1 3 − λ3    
0  n 2  =  − 1 3 − 4 0  n 2  = 0 

 0 0 1 − λ 3  n 3   0 0 1 − 4  n 3  0 

 − n1 − n 2 = 0 
  ⇒ n 2 = −n 2
 − n1 − n 2 = 0 
− 3n = 0
 3

1
n i n i = n12 + n 22 + n 32 = 1 ⇒ 2n12 = 1 ⇒ n1 = ±
2
Resultando:
 1 1 
λ3 = 4 ⇒ nˆ (3) =  ± m 0
 2 2 
Podemos entonces resumir que las direcciones principales correspondientes a sus valores
principales son:

Universidad de Castilla- La Mancha Draft Por: Eduardo W. V. Chaves (2012)


Ciudad Real - España
1 TENSORES 49

λ1 ⇒ nˆ (1) = [ 0 0 ±1]

 1 1 
λ2 ⇒ nˆ (2) = ± ± 0
 2 2 
 1 1 
λ3 ⇒ nˆ (3) = ± m 0
 2 2 

Ejemplo 1.69
Dado un tensor ortogonal propio Q , a) demostrar que Q tiene un autovalor real e igual a 1 .
b) Demostrar también que Q puede ser representado en función de un ángulo θ tal que:
Q = pˆ ⊗ pˆ + cos θ(qˆ ⊗ qˆ + rˆ ⊗ rˆ ) − sin θ(qˆ ⊗ rˆ − rˆ ⊗ qˆ )

donde p̂ , q̂ , r̂ , son versores que constituyen una base ortonormal, siendo p̂ la dirección
correspondiente al autovalor λ = 1 , es decir, p̂ es autovector de Q . c) Obtener los invariantes
r
principales de Q en función del ángulo θ . d) Dado el vector posición x , determinar el nuevo
r
vector formado por la transformación ortogonal Q ⋅ x en el espacio p̂ , q̂ .
Solución:
a) Teniendo en cuenta la definición de tensor ortogonal, podemos decir que:
QT ⋅ Q = 1
⇒ QT ⋅ Q − QT = 1 − QT
⇒ Q T ⋅ (Q − 1) = −(Q T − 1)
⇒ Q T ⋅ (Q − 1) = −(Q − 1) T
A continuación obtenemos el determinante de los dos tensores anteriores:
[ ] [
det Q T ⋅ (Q − 1) = det − (Q − 1) T ] [
= ( −1) 3 det (Q − 1) T ]
[ ] [ ]
⇒ det Q T det[(Q − 1)] = −det (Q − 1) T = −det[(Q − 1)]
1424 3
= detQ =1

⇒ det[(Q − 1)] = −det[(Q − 1)]

donde hemos utilizado las siguientes propiedades del determinante: det[αA ] = α 3 det[A ] ,
[ ]
det A T = det[A ] , det[A ⋅ B ] = det[A ]det[B ] . El único escalar que cumple la expresión
anterior es el cero, luego:
det[(Q − 1)] = 0
Teniendo en cuenta la definición de autovalor, det[(Q − λ1)] = 0 , concluimos que cuando
λ = 1 cumple det[(Q − 1)] = 0 , luego λ = 1 es autovalor de Q . Además, existe una dirección
(autovector) que cumple que Q ⋅ eˆ 1* = λeˆ 1* = eˆ 1* .
b) Vamos considerar que pˆ ≡ eˆ 1* , qˆ ≡ eˆ *2 , rˆ ≡ eˆ *3 constituye una base ortonormal.

Universidad de Castilla- La Mancha Draft Por: Eduardo W. V. Chaves (2012)


Ciudad Real - España
50 PROBLEMAS RESUELTOS DE MECÁNICA DEL MEDIO CONTINUO

ê 3
eˆ 1* ≡ pˆ
qˆ ≡ eˆ *2
Q ⋅ eˆ 1* = eˆ 1*

ê 2

ê1 rˆ ≡ eˆ *3

La representación simbólica del tensor en la base ê1* , ê *2 , ê *3 , viene dada por:

Q = Q *ij eˆ *i ⊗ eˆ *j
* ˆ*
= Q11 e1 ⊗ eˆ 1* + Q12
* ˆ*
e1 ⊗ eˆ *2 + Q13
* ˆ*
e1 ⊗ eˆ *3 +
(1.49)
+ Q *21eˆ *2 ⊗ eˆ 1* + Q *22 eˆ *2 ⊗ eˆ *2 + Q *23 eˆ *2 ⊗ eˆ *3 +
+ Q *31eˆ *3 ⊗ eˆ 1* + Q *32 eˆ *3 ⊗ eˆ *2 + Q *33 eˆ *3 ⊗ eˆ *3

Teniendo en cuenta que ê1* es autovector de Q asociado al autovalor λ = 1 , se cumple que


Q ⋅ eˆ 1* = λeˆ 1* = eˆ 1* , además haciendo la proyección de Q , dado por (1.49), según dirección ê1* ,
obtenemos que:
Q ⋅ eˆ 1* = eˆ 1*
Q ⋅ eˆ 1* = [ Q11
* ˆ*
e1 ⊗ eˆ 1* + Q12
* ˆ*
e1 ⊗ eˆ *2 + Q13
* ˆ*
e1 ⊗ eˆ *3 +
+ Q *21eˆ *2 ⊗ eˆ 1* + Q *22 eˆ *2 ⊗ eˆ *2 + Q *23 eˆ *2 ⊗ eˆ *3 +
+ Q *31eˆ *3 ⊗ eˆ 1* + Q *32 eˆ *3 ⊗ eˆ *2 + Q *33 eˆ *3 ⊗ eˆ *3 ] ⋅ eˆ 1*
* ˆ*
= Q11 e1 + Q *21eˆ *2 + Q *31eˆ *3
*
Con lo cual concluimos que Q11 = 1 , Q *21 = 0 , Q *31 = 0 .
Recordar que dos tensores coaxiales tienen las mismas direcciones principales. Un tensor y su
inversa siempre serán tensores coaxiales, luego si Q −1 = Q T , eso implica que Q T y Q son
coaxiales, y ê1* también será dirección principal de Q T , luego se cumple que:
Q T ⋅eˆ 1* = eˆ 1*
Q T ⋅eˆ 1* = [ Q11
* ˆ*
e1 ⊗ eˆ 1* + Q *21eˆ 1* ⊗ eˆ *2 + Q *31eˆ 1* ⊗ eˆ *3 +
* ˆ*
+ Q12 e 2 ⊗ eˆ 1* + Q *22 eˆ *2 ⊗ eˆ *2 + Q *32 eˆ *2 ⊗ eˆ *3 +
* ˆ*
+ Q13 e 3 ⊗ eˆ 1* + Q *23 eˆ *3 ⊗ eˆ *2 + Q *33 eˆ *3 ⊗ eˆ *3 ] ⋅ eˆ 1*
* ˆ* * ˆ* * ˆ*
= Q11 e1 + Q12 e 2 + Q13 e3
* * *
Con lo cual concluimos que Q11 = 1 , Q12 = 0 , Q13 = 0 . Luego, la expresión (1.49) queda:

Q = eˆ 1* ⊗ eˆ 1* + Q *22 eˆ *2 ⊗ eˆ *2 + Q *23 eˆ *2 ⊗ eˆ *3 + Q *32 eˆ *3 ⊗ eˆ *2 + Q *33 eˆ *3 ⊗ eˆ *3 (1.50)

Universidad de Castilla- La Mancha Draft Por: Eduardo W. V. Chaves (2012)


Ciudad Real - España
1 TENSORES 51

En forma de matriz, las componentes de Q en la base eˆ *i vienen dadas por:

x 2*
Q *22
1 0 0 
Q ij = 0 Q *22
*
Q *23 
*
0 Q *32 Q *33  Q11 =1
Q *32
Q *23 x1*

Q *33

x3*
Recurrimos una vez más a la condición de ortogonalidad Q T ⋅ Q = Q ⋅ Q T = 1 , o en función de
las componentes en el espacio eˆ *i :
1 0 0  1 0 0  1 0 0
Q *ki Q *kj = δ ij ⇒ 0 Q * Q *32  0 Q *22 Q *23  = 0 1 0
 22
0 Q *23 Q *33  0 Q *32 Q *33  0 0 1 
(1.51)
1 0 0  1 0 0
⇒ 0 [(Q )
* 2
22 + (Q *32 ) 2 ] [Q Q *
22 23
*
+ Q *32 Q *33 ]
 = 0 1 0 
  
0 [Q Q
*
22 23
*
+ Q *32 Q *33 ] [(Q ) * 2
33 + (Q *23 ) 2 ] 0 0 1

El determinante de un tensor ortogonal propio es det (Q) = +1 :


1 0 0 
0 Q * Q *23  = 1 ⇒ Q *22 Q *33 − Q *23 Q *32 = 1
 22 (1.52)
0 Q *32 Q *33 

Teniendo en cuenta (1.51) y (1.52) tenemos el siguiente conjunto de ecuaciones:


(Q *22 ) 2 + (Q *32 ) 2 = 1 cos 2 θ + sin 2 θ = 1
 * * * * 
Q 22 Q 23 + Q 32 Q 33 = 0 cos θ(− sin θ) + sin θ cos θ = 0
 * 2 * 2  2 2
(Q 33 ) + (Q 23 ) = 1 cos θ + sin θ = 1
 * * * * cos θ cos θ − (− sin θ)(sin θ) = 1
Q 22 Q 33 − Q 23 Q 32 = 1 
Con lo cual hemos demostrado la existencia de un ángulo θ que cumpla con las condiciones
anteriores.
1 0 0  1 0 0 
Q *ij = 0 Q *22 * 
Q 32  = 0 cos θ − sin θ

(1.53)
0 Q *23 Q *33  0 sin θ cos θ 
Retomando la expresión (1.50), y teniendo en cuenta (1.53), concluimos que:
Q = eˆ 1* ⊗ eˆ 1* + (cos θ) eˆ *2 ⊗ eˆ *2 + (− sin θ)eˆ *2 ⊗ eˆ *3 + (sin θ)eˆ *3 ⊗ eˆ *2 + (cos θ) eˆ *3 ⊗ eˆ *3
[ ] [
= eˆ 1* ⊗ eˆ 1* + cos θ eˆ *2 ⊗ eˆ *2 + eˆ *3 ⊗ eˆ *3 − sin θ eˆ *2 ⊗ eˆ *3 − eˆ *3 ⊗ eˆ *2 ]

Universidad de Castilla- La Mancha Draft Por: Eduardo W. V. Chaves (2012)


Ciudad Real - España
52 PROBLEMAS RESUELTOS DE MECÁNICA DEL MEDIO CONTINUO

Considerando que pˆ ≡ eˆ 1* , qˆ ≡ eˆ *2 , rˆ ≡ eˆ *3 , demostramos que:


Q = pˆ ⊗ pˆ + cos θ(qˆ ⊗ qˆ + rˆ ⊗ rˆ ) − sin θ(qˆ ⊗ rˆ − rˆ ⊗ qˆ )
Es interesante verificar que la descomposición aditiva de Q en una parte simétrica y otra
antisimétrica, en el espacio eˆ *i , resulta:
1 0 0  0 0 0 
Q *ijsym
= 0 cos θ 0  ; Q *ijanti
= 0 0 − sin θ
0 0 cos θ 0 sin θ 0 
1444 4244443 1444 44244444 3
[pˆ ⊗pˆ +cos θ(qˆ ⊗qˆ +rˆ ⊗rˆ )]ij [−sin θ(qˆ ⊗rˆ −rˆ ⊗qˆ ) ]ij
anti
Observemos que el formato de Q *ij tiene el mismo formato que presenta un tensor
antisimétrico ( W ) en el espacio definido por su vector axil:
0 0 0 
Wij* = 0 0 − ω 

0 ω 0 

donde ω es el módulo del vector axil.


c) Teniendo en cuenta (1.53), queda de fácil demostración que I Q = II Q = 1 + 2 cos θ , III Q = 1 .
r
d) Representamos el vector posición x a través de sus componentes y la base p̂ , q̂ , r̂ :
r
x = ppˆ + qqˆ + rrˆ .
r r r
Luego, se cumple que: x ⋅ pˆ = ( ppˆ + qqˆ + rrˆ ) ⋅ pˆ = p ; x ⋅ qˆ = q ; x ⋅ rˆ = r
Luego, ver Figura 1.2, se cumple que:
r
~ r
[
x = Q ⋅ x = pˆ ⊗ pˆ + cos θ(qˆ ⊗ qˆ + rˆ ⊗ rˆ ) − sin θ(qˆ ⊗ rˆ − rˆ ⊗ qˆ ) ]⋅ [ppˆ + qqˆ + rrˆ ]
= ppˆ + (q cos θ − r sin θ)qˆ + (r cos θ + q sin θ)rˆ

eˆ 1* ≡ pˆ r
x
qˆ ≡ eˆ *2
O r r
~
x =Q⋅ x

rˆ ≡ eˆ *3
Figura 1.2

Universidad de Castilla- La Mancha Draft Por: Eduardo W. V. Chaves (2012)


Ciudad Real - España
1 TENSORES 53

Ejemplo 1.70
r r r r
Considérense las transformaciones tensoriales p ′ = U ⋅ p y p ′′ = R ⋅ p ′ , donde R es un tensor
ortogonal de segundo orden y U es un tensor de segundo orden con U ⋅ U −1 = 1 , i.e. ∃ U −1 .
r r
Obtener las leyes de transformación entre p y p ′′ .
Solución:
El problema planteado se puede esquematizar a través de la siguiente figura:

r R
U p′

r
r p ′′
p
?

Teniendo en cuenta que R −1 = R T (tensor ortogonal), es decir, existe la inversa de R y


r r
considerando p ′′ = R ⋅ p ′ obtenemos que:
r r
p ′′ = R ⋅ p ′
r r
⇒ R −1 ⋅ p ′′ = R −1 ⋅ R ⋅ p ′
r r r
⇒ R −1 ⋅ p ′′ = 1 ⋅ p ′ = p ′
r r r r
Reemplazando p ′ = R −1 ⋅ p ′′ en p ′ = U ⋅ p , obtenemos que:
r r r r
p′ = U ⋅ p p′ = U ⋅ p
r r r r
⇒ R −1 ⋅ p ′′ = U ⋅ p ⇒ R −1 ⋅ p ′′ = U ⋅ p
r r r r
⇒ R ⋅ R −1 ⋅ p ′′ = R ⋅ U ⋅ p ⇒ U −1 ⋅ R −1 ⋅ p ′′ = U −1 ⋅ U ⋅ p (1.54)
r r r r r
⇒ 1 ⋅ p ′′ = R ⋅ U ⋅ p ⇒ (R ⋅ U) −1 ⋅ p ′′ = 1 ⋅ p = p
r r r r
⇒ p ′′ = (R ⋅ U) ⋅ p ⇒ p = (R ⋅ U) −1 ⋅ p ′′

o en Notación indicial:
p ′i = U ij p j p ′i = U ij p j
⇒ R ij−1p ′′j = U ij p j ⇒ R ij−1p ′′j = U ij p j
⇒ R ki R ij−1p ′′j = R ki U ij p j ⇒ U ki−1R ij−1p ′′j = U ki−1U ij p j (1.55)
−1
⇒ δ kj p ′′j = R ki U ij p j ⇒ (R ki U ij ) p ′′j = δ kj p j = p k
⇒ p ′k′ = (R ki U ij )p j ⇒ p k = (R ki U ij ) −1 p ′′j

Universidad de Castilla- La Mancha Draft Por: Eduardo W. V. Chaves (2012)


Ciudad Real - España
54 PROBLEMAS RESUELTOS DE MECÁNICA DEL MEDIO CONTINUO

r
U p′ R

U −1
R −1 = R T
r r
p (R ⋅ U) p ′′

(R ⋅ U) −1 = U −1 ⋅ R T

1.1.9 Representación Espectral

Ejemplo 1.71
Sea w un tensor antisimétrico de segundo orden y V un tensor de segundo orden definido
positivo cuya representación espectral viene dado por:
3
V= ∑λa =1
a nˆ ( a ) ⊗ nˆ ( a )

Demostrar que el tensor antisimétrico w puede ser representado por:


3
w = ∑ w ab nˆ (a ) ⊗ nˆ (b)
a ,b =1
a ≠b

Demostrar también que se cumple la relación:


3
w ⋅ V − V ⋅ w = ∑ w ab (λ b − λ a ) nˆ ( a ) ⊗ nˆ (b)
a ,b =1
a ≠b

Solución:
Es cierto que
 3
 3
w ⋅ 1 = w ⋅  ∑ nˆ ( a ) ⊗ nˆ ( a )  = ∑ w ⋅ nˆ ( a) ⊗ nˆ (a )
 a =1  a =1

∑( )
3
r
= w ∧ nˆ ( a ) ⊗ nˆ ( a )
a =1

∑ w (nˆ )
3
= b
(b )
∧ nˆ ( a ) ⊗ nˆ ( a )
a ,b =1
r r
donde hemos aplicado la propiedad de un tensor antisimétrico w ⋅ nˆ = w ∧ nˆ , donde w es el
vector axil asociado al tensor w . Expandiendo la expresión anterior obtenemos que

Universidad de Castilla- La Mancha Draft Por: Eduardo W. V. Chaves (2012)


Ciudad Real - España
1 TENSORES 55

w = wb (nˆ (b) ∧ nˆ (1) ) ⊗ nˆ (1) + wb (nˆ (b) ∧ nˆ ( 2) ) ⊗ nˆ ( 2) + wb (nˆ (b) ∧ nˆ (3) ) ⊗ nˆ (3) =
( ) ( )
= w1 nˆ (1) ∧ nˆ (1) ⊗ nˆ (1) + w2 nˆ ( 2) ∧ nˆ (1) ⊗ nˆ (1) + w3 nˆ (3) ∧ nˆ (1) ⊗ nˆ (1) + ( )
+ w (nˆ
1
(1)
) ( )
∧ nˆ ( 2 ) ⊗ nˆ ( 2 ) + w2 nˆ ( 2 ) ∧ nˆ ( 2) ⊗ nˆ ( 2) + w3 nˆ (3) ∧ nˆ ( 2) ⊗ nˆ ( 2 ) + ( )
+ w (nˆ
1
(1)
∧ nˆ ( 3)
) ⊗ nˆ ( 3)
+ w2 (nˆ ( 2)
∧ nˆ ( 3)
) ⊗ nˆ ( 3)
+ w3 (nˆ ( 3)
∧ nˆ ( 3)
) ⊗ nˆ ( 3)

Simplificando la expresión anterior resulta que:


w = −w2 (nˆ (3) ) ⊗ nˆ (1) + w3 (nˆ ( 2) ) ⊗ nˆ (1) +
( )
+ w1 nˆ (3) ⊗ nˆ ( 2 ) − w3 nˆ (1) ⊗ nˆ ( 2 ) + ( )
− w1 (nˆ ) ⊗ nˆ
( 2) ( 3)
+ w2 (nˆ ) ⊗ nˆ
(1) ( 3)

Además teniendo en cuanta que w1 = −w 23 = w 32 , w2 = w13 = −w 31 , w3 = −w12 = w 21 , w


aún puede ser expresado por:
w = w 31 nˆ (3) ⊗ nˆ (1) + w 21 nˆ ( 2) ⊗ nˆ (1) +
+ w 32 nˆ (3) ⊗ nˆ ( 2 ) + w12 nˆ (1) ⊗ nˆ ( 2 ) +
+ w 23 nˆ ( 2 ) ⊗ nˆ (3) + w13 nˆ (1) ⊗ nˆ (3)
el cual es exactamente igual a
3
w = ∑ w ab nˆ (a ) ⊗ nˆ (b)
a ,b =1
a ≠b

Los términos w⋅V y V ⋅ w pueden ser obtenidos como sigue a continuación:


 3 
   3 
 a ,b =1

w ⋅ V =  w ab n ⊗ n  ⋅  λ b nˆ (b) ⊗ nˆ (b) 
ˆ (a) ˆ (b )

  b =1 

 a≠b 
3 3
= ∑ λ b w ab nˆ ( a ) ⊗ nˆ (b ) ⋅ nˆ (b ) ⊗ nˆ (b ) =
a ,b =1
∑λ w
a ,b =1
b ab nˆ ( a ) ⊗ nˆ (b )
a ≠b a≠b

y
 3 
 3 (a ) 
 
 a =1
ˆ ∑
V ⋅ w =  λ a n ⊗ n  ⋅ 
(a) ˆ
  a ,b =1
w ab n ⊗ n 
ˆ (a) ˆ (b )



 a≠b 
3
= ∑λ w
a ,b =1
a ab nˆ ( a ) ⊗ nˆ (b )
a ≠b

Luego,
 3   3 
   
 a ,b =1
ˆ (a) ˆ ∑
w ⋅ V − V ⋅ w =  λ b w ab n ⊗ n  −  λ a w ab n ⊗ n 
(b )

  a ,b =1
ˆ (a) ˆ (b )



 a ≠b   a≠b 
3
= ∑w
a ,b =1
ab (λ b − λ a ) nˆ ( a ) ⊗ nˆ (b )
a ≠b

Análogamente, es posible demostrar que:

Universidad de Castilla- La Mancha Draft Por: Eduardo W. V. Chaves (2012)


Ciudad Real - España
56 PROBLEMAS RESUELTOS DE MECÁNICA DEL MEDIO CONTINUO

3
w ⋅ V 2 − V 2 ⋅ w = ∑ w ab (λ2b − λ2a ) nˆ ( a ) ⊗ nˆ (b)
a ,b =1
a ≠b

Ejemplo 1.72
Dado un tensor definido positivo C , cuyas componentes cartesianas de este tensor vienen
dadas por:
2 0 1 
C ij = 0 4 0
1 0 2

Obtener los siguientes tensores: a) C 2 ; b) U = C . c) Comprobar si los tensores C y U son


coaxiales.
Solución:
Observemos que los tensores C 2 y U = C son tensores coaxiales con el tensor C .
Haciendo la representación espectral del tensor C :
3
C= ∑ γ Nˆ
a =1
a
(a) ˆ (a )
⊗N

ˆ ( a ) son los autovectores del tensor C . Luego,


donde γ a son los autovalores del tensor C , y N
se cumple que:
3 3
C2 = ∑
a =1
ˆ (a) ⊗ N
γ a2 N ˆ (a) ; U= C = ∑
a =1
ˆ (a) ⊗ N
γaN ˆ (a )

Cálculo de los autovalores y autovectores del tensor C .


Podemos verificar que según la estructura de las componentes del tensor C ya conocemos un
auto valor γ 2 = 4 que está asociado a la dirección Nˆ i( 2) = [0 ± 1 0] . Para calcular los restantes
autovalores obtenemos el determinante característico siguiente:
2−γ 1 γ 1 = 2 − 1 = 1
=0 ⇒ ( 2 − γ ) 2 = 12 ⇒ ( 2 − γ ) = ±1 ⇒ 
1 2−γ γ 3 = 2 + 1 = 3
Asociado al autovalor γ 1 = 1 tenemos el siguiente autovector:

2 − γ 1 1  Nˆ 1(1)  0 1 1 Nˆ 1(1)  0


 1  = ⇒ 1 1  ˆ (1)  = 0 ⇒ Nˆ 1(1) = −Nˆ 3(1)
 2 − γ 1  Nˆ 3(1)  0   N 3   

con la restricción Nˆ i(1) Nˆ i(1) = 1 , resultando que:


Nˆ 1(1) Nˆ 1(1) + Nˆ (21) Nˆ (21) + Nˆ 3(1)Nˆ (31) = 1
⇒ Nˆ (1) Nˆ (1) + Nˆ (1)Nˆ (1) = 1
1 1 1 1
1
⇒ Nˆ 1(1) = ±
2
1
⇒ Nˆ 3(1) = −Nˆ 1(1) = m
2

Universidad de Castilla- La Mancha Draft Por: Eduardo W. V. Chaves (2012)


Ciudad Real - España
1 TENSORES 57

Asociado al autovalor γ 3 = 3 tenemos el siguiente autovector:

2 − γ 3 1  Nˆ 1(3)  0 − 1 1  N ˆ (3)  0


 1  = ⇒  1 − 1 ˆ (3)  = 0
 1
⇒ Nˆ 1(3) = Nˆ 3(3)
 2 − γ 3  Nˆ (33)  0  N
  3   

con la restricción Nˆ i(3) Nˆ i(3) = 1 , resultando que:

Nˆ 1(3)Nˆ 1(3) + Nˆ (23) Nˆ (23) + Nˆ 3(3)Nˆ (33) = 1


1
⇒ Nˆ 1(3)Nˆ 1(3) + Nˆ 1(3) Nˆ 1(3) = 1 ⇒ Nˆ 1(3) = ±
2
1
⇒ Nˆ 3(3) = Nˆ 1(3) = ±
2
Resumiendo tenemos que:

ˆ (1) =  ± 1 1   1 1 
γ1 = 1 ⇒ N i  0 m  0 −
 2 2   
  2 2
γ2 = 4 ⇒ ˆ ( 2 ) = [0
N ± 1 0] Matriz
 de transformación
 → A= 0 1 0 
i 
1   1 1 
γ3 = 3 ⇒ ˆ ( 3) =  ±
N
1
0 ± 0
i    2 2 
 2 2   

Luego se cumple que:


C′ = A C AT ⇒ C = AT C′ A
En el espacio principal tenemos que:
 1 0 0
 2 

C ij = 0 16 0
1 0 0  0 0 9

C ij′ = 0 4 0 ⇒ 
 ± 1 0 0  1 0 0
0 0 3  
U′ = C ′ = 0 ± 4 0  = 0 2 0 
 ij ij

  0 0 ± 3  0 0 3 
 
Observemos que el tensor C es un tensor definido positivo, luego sus autovalores son
positivos. En espacio original tenemos las siguientes componentes:
T
 1 1   1 1 
 0 −  1 0 0  2 0 − 
 2 2 2  5 0 4 
2
C ij =  0 1 0  0 16 0  0 1 0  = 0 16 0
1 1   
 0 0 0 9  1 0
1  
4 0 5
 2 2   2 2 
 
Observemos que este resultado podría haber sido obtenido fácilmente a través de la operación
C 2 = C ⋅ C , o en componentes:
 2 0 1   2 0 1   5 0 4
C ij2 = C ik C kj = 0 4 0 0 4 0 = 0 16 0
1 0 2 1 0 2  4 0 5

Análogamente:

Universidad de Castilla- La Mancha Draft Por: Eduardo W. V. Chaves (2012)


Ciudad Real - España
58 PROBLEMAS RESUELTOS DE MECÁNICA DEL MEDIO CONTINUO

T
 1 1   1 1   3 +1 3 − 1
 0 −  1 0 0  0 −   0 
 2 2 2 2  2 2 
U ij =  0 1 0  0 2 0  0 1 0 = 0 2 0 
1 1    1 1   3 −1
 0 0 0 3   0 3 + 1
0

 2 2  
 2 2   2 2 

c) Los tensores C y U son coaxiales ya que hemos obtenido los autovalores de U en el


espacio principal de C . También podemos comprobar que son tensores coaxiales porque se
cumple que C ⋅ U = U ⋅ C , en componentes.
 3 +1 3 − 1
2 0 1  0  3,098 0 2,098
   2 2  
C ik U kj = 0 4 0  0 2 0 = 0 8 0 
1 0 2  3 − 1 3 + 1  2,098 0 3,098
 2 0  
 2 

 3 +1 3 − 1
 0  2 0 1  3,098 0 2,098
 2 2 
U ik C kj =  0 2 0  0 4 0 =  0 8 0 
 3 −1 0 3 + 1  1 0 2 2,098 0 3,098 
   
 2 2 

Ejemplo 1.73
Sea C un tensor de segundo orden simétrico y R un tensor ortogonal propio. Las
componentes de estos tensores en el sistema Cartesiano vienen dadas por:
 
 0 0 1
2 0 1   
2 2
C ij = 0 4 0 ; R ij =  0
 2 2 
1 0 2  
2 2
− 0
 2 2 
a) Obtener los siguientes tensores: a.1) C 8 ; a2) U = C .
b) Obtener también los invariantes principales de C .
c) Teniendo en cuenta que los tensores b y C están relacionados entre si a través de la
siguiente transformación ortogonal C = R T ⋅ b ⋅ R , obtener el tercer invariante principal de b .
Solución:
 3281 0 3280

a) Análogo al Ejemplo 1.72. Respuesta: C =  0 655368
0 
3280 0 3281
b)
I C = Tr (C ij ) = C ii = C11 + C 22 + C 33 = 8

II C =
1
2
( )
C ii C jj − C ij C ij =
4 0 2 1 2 0
+ +
0 2 1 2 0 4
= 19 ; III C = C =  ijk C i1C j 2 C k 3 = 12

Universidad de Castilla- La Mancha Draft Por: Eduardo W. V. Chaves (2012)


Ciudad Real - España
1 TENSORES 59

c) Teniendo en cuenta las propiedades de determinante, el tercer variante principal de b puede


ser expresado por:
C ≡ det (C ) = det (R T ⋅ b ⋅ R ) = det (R T )det (b)det (R ) = det (b) = III b = 12
1424 3 123
= +1 = +1

Ejemplo 1.74
Sea un tensor de segundo orden simétrico S con det (S ) ≠ 0 . Considerando que S tiene dos
autovalores iguales, i.e. S 2 = S 3 y S1 ≠ S 2 , demostrar que S puede ser representado por:
S = S 1nˆ (1) ⊗ nˆ (1) + S 2 (1 − nˆ (1) ⊗ nˆ (1) )

donde n̂ (1) es el autovector de S asociado al autovalor S 1 , 1 es el tensor identidad de


segundo orden.
Solución:
Partimos de la representación espectral de S :
3
S= ∑ S nˆ
a =1
a
(a)
⊗ nˆ ( a )
(1.56)
= S1nˆ (1) ⊗ nˆ (1) + S 2 nˆ ( 2 ) ⊗ nˆ ( 2 ) + S 3 nˆ (3) ⊗ nˆ (3)
= S1nˆ (1) ⊗ nˆ (1) + S 2 (nˆ ( 2 ) ⊗ nˆ ( 2 ) + nˆ (3) ⊗ nˆ (3) )

Recordar que 1 es un tensor esférico, con lo cual cualquier dirección es una dirección
principal. Partiendo de este principio adoptamos el espacio principal de S para hacer la
representación espectral de 1 :
3
1= ∑ nˆ
a =1
(a)
⊗ nˆ ( a ) = nˆ (1) ⊗ nˆ (1) + nˆ ( 2 ) ⊗ nˆ ( 2 ) + nˆ (3) ⊗ nˆ (3)
(1.57)
⇒ nˆ ( 2)
⊗ nˆ ( 2)
+ nˆ ( 3)
⊗ nˆ ( 3)
= 1 − nˆ (1)
⊗ nˆ (1)

Reemplazando lo anterior en (1.56), obtenemos que:


S = S 1nˆ (1) ⊗ nˆ (1) + S 2 (nˆ ( 2) ⊗ nˆ ( 2) + nˆ (3) ⊗ nˆ (3) )
= S 1nˆ (1) ⊗ nˆ (1) + S 2 (1 − nˆ (1) ⊗ nˆ (1) )

1.1.10 Teorema de Cayley-Hamilton

Ejemplo 1.75
Dado un tensor de segundo orden T , demostrar el teorema de Cayley-Hamilton.
Solución:
Partimos de la ecuación característica del tensor: λ3 − λ2 I T + λ II T − III T = 0 , cuya ecuación se
cumple para los autovalores de T , λ 1 , λ 2 , λ 3 , luego:
λ31 − λ21 I T + λ 1 II T − III T = 0
λ32 − λ22 I T + λ 2 II T − III T = 0
λ33 − λ23 I T + λ 3 II T − III T = 0

Universidad de Castilla- La Mancha Draft Por: Eduardo W. V. Chaves (2012)


Ciudad Real - España
60 PROBLEMAS RESUELTOS DE MECÁNICA DEL MEDIO CONTINUO

Reestructurando las ecuaciones anteriores en forma matricial, obtenemos que:


λ31 0 0  λ21 0 0 λ 1 0 0 1 0 0  0 0 0 
     
0 λ32 0−0 λ22 0 I T +  0 λ2 0  II T − 0 1 0 III T = 0 0 0
 
0 0 λ33   0 0 λ23  0 0 λ 3  0 0 1  0 0 0
 
Notar que en el espacio principal se cumple que:
λ 1 0 0
 
( T ) ij = Tij =  0 λ2 0
0 0 λ 3 

λ 1 0 0  λ 1 0 0  λ21 0 0
    
( T 2 ) ij = Tik Tkj =  0 λ2 0  0 λ2 0=0 λ22 0
0 0 λ 3   0 0 λ 3   0 0 λ23 

λ 1 0 0  λ 1 0 0  λ 1 0 0  λ31 0 0
     
( T 3 ) ij = Tik Tkp T pj =  0 λ2 0  0 λ2 0  0 λ2 0 =0 λ32 0
0 0 λ 3   0 0 λ 3   0 0 λ 3   0 0 λ33 

Con lo cual podemos concluir que: T 3 − T 2 I T + T II T − III T 1 = 0 .

Ejemplo 1.76
Partiendo del teorema de Cayley-Hamilton obtener la inversa de un tensor T en función de
potencia de tensores.
Solución:
El teorema de Cayley-Hamilton afirma que:
T 3 − T 2 I T + T II T − III T 1 = 0

Haciendo el producto escalar de la expresión anterior por el tensor T −1 obtenemos que:


T 3 ⋅ T −1 − T 2 ⋅ T −1 I T + T ⋅ T −1 II T − III T 1 ⋅ T −1 = 0 ⋅ T −1
T 2 − TI T + 1 II T − III T T −1 = 0

⇒ T −1 =
1
III T
(
T 2 − TI T + 1 II T )

Ejemplo 1.77
Dado el tensor T representado por sus componentes en el sistema cartesiano:
5 0 0 
T = 0 2 0
0 0 1 

Comprobar el teorema de Cayley-Hamilton.


Solución:
El teorema de Cayley-Hamilton también se aplica para las componentes del tensor:

Universidad de Castilla- La Mancha Draft Por: Eduardo W. V. Chaves (2012)


Ciudad Real - España
1 TENSORES 61

T 3 − T 2 I T + T II T − III T 1 = 0
donde:
I T = 5 + 2 + 1 = 8 ; II T = 10 + 2 + 5 = 17 ; III T = 10
luego:
5 3 0 0  125 0 0  5 2 0 0  25 0 0 
   
T 3
=0 23 0  =  0 8 0  ; T 2
=0 22 0  =  0 4 0 
0 0 1   0 0 1  0 0 1   0 0 1 
 
Aplicando el teorema de Cayley-Hamilton verificamos que:
125 0 0  25 0 0 5 0 0  1 0 0  0 0 0 
 0 8 0 − 8  0 4 0 + 17 0 2 0  − 10 0 1 0 = 0 0 0 
       
 0 0 1 
1  0 0 1 
 0 0 1 
 0 0 1 
 0 0 0 
444444444442444444444443

0 0 0  0 0 0 
0 0 0  = 0 0 0 
 
0 0 0 0 0 0 
c.q.d.

Ejemplo 1.78
Dada una matriz P representada por sus componentes por Pij (i, j = 1,2,3,4) . a) Obtener la
inversa, b) los invariantes, y c) la ecuación característica. Aplicar los apartados para la matriz
P:
1 2 3 1 1 0 0 0
2 2 1 2  0 1 0 0 
P= . Considerar que 1=
4 1 5 3 0 0 1 0
   
3 1 2 4 0 0 0 1
Solución:
Aplicando el Teorema de Cayley-Hamilton se cumple que:
P 4 + P 3 I1 + P 2 I 2 + P I 3 + I 41 = 0
(
⇒ P P 3 + P 2 I1 + P I 2 + 1 I 3 + I 41 = 0 )
⇒ P (P (P 2
) )
+ P I1 + 1 I 2 + 1 I 3 + I 41 = 0
   
  
4 4 2 443
( 
)
⇒ P  P  P P + 1 I 1 + 1 I 2  + 1 I 3  + I 41 = 0
1
   
  C1  
 

4 4 2
(
44 3

)
⇒ P  P C1 + 1 I 2 + 1 I 3  + I 4 1 = 0
1
 
 C2 
(
⇒ P C2 + 1 I 3 + I 4 1 = 0 )
⇒ C3 + I 4 1 = 0
donde hemos denotado:

Universidad de Castilla- La Mancha Draft Por: Eduardo W. V. Chaves (2012)


Ciudad Real - España
62 PROBLEMAS RESUELTOS DE MECÁNICA DEL MEDIO CONTINUO

C0 = P
(
C1 = P C0 + 1 I 1 )
C2 = P (C 1 +1 I )
2

C3 = P (C 2 +1 I )
3

Podemos sacar la traza de C3 + I 41 = 0 , resultando que:


Tr (C3 + I 4 1) = Tr (0 )
⇒ Tr (C3 ) + Tr ( I 4 1) = Tr (C3 ) + I 4 Tr (1) = Tr (C3 ) + 4 I 4 = 0
− Tr (C3 )
⇒ I4 =
4
Análogamente podemos definir que:
− Tr (C2 ) − Tr (C1 ) − Tr (C0 )
I3 = ; I2 = ; I1 =
3 2 1
Con eso obtenemos que:
− Tr (C0 )
I1 = = −(1 + 2 + 5 + 4) = −12
1
con eso podemos definir la matriz C1 = P C0 + 1 I 1 : ( )
1 2 3 1   1 2 3 1 1 0 0 0   8 − 14 − 14 6 
2   
2 1 2   2 
2 1 2 0 
1 0 0    − 8 − 13 5 − 7 
C1 =  − 12  =
4 1 5 3   4 1 5 3 0 0 1 0   − 13 6 − 16 − 3 
       
3 1 2 4   3 1 2 4 0 0 0 1    − 11 2 4 − 21

− Tr (C1 ) − (8 − 13 − 16 − 21) − (−42)


I2 = = = = 21
2 2 2
A su vez obtenemos C2 = P C1 + 1 I 2 ( )
1 2 3 1   8 − 14 − 14 6  1 0 0 0   − 37 22 15 − 17 
2    0   
2 1 2   − 8 − 13 5 −7 1 0 0   7 − 2 − 5 − 5 
C2 =  + 21 =
4 1 5 3   − 13 6 − 16 − 3  0 0 1 0    10 − 12 − 14 2 
       
3 1 2 4   − 11 2 4 − 21 0 0 0 1   9 − 14 − 11 5 

− Tr (C2 ) − ( −37 − 2 − 14 + 5) − ( −48)


I3 = = = = 16
3 3 3
A su vez obtenemos C3 = P C2 + 1 I 3 ( )
1 2 3 1    − 37 22 15 − 17  1 0 0 0   32 0 0 0 
2    0 
2 1 2   7 −2 −5 −5 1 0 0    0 32 0 0 
C3 =  + 16  =
4 1 5 3   10 − 12 − 14 2  0 0 1 0    0 0 32 0 
       
3 1 2 4   9 − 14 − 11 5  0 0 0 1    0 0 0 32 

− Tr (C3 ) − 4(32)
I4 = = = −32 = det (P )
4 4
Luego, la ecuación característica queda:

Universidad de Castilla- La Mancha Draft Por: Eduardo W. V. Chaves (2012)


Ciudad Real - España
1 TENSORES 63

P 4 + P 3 I1 + P 2 I 2 + P I 3 + I 41 = 0 ⇒ P 4 − 12P 3 + 21P 2 + 16P − 321 = 0


Los coeficientes de la ecuación característica podrían haber sido obtenidos al resolver:
det ( P − λ1) ≡ P − λ1 = 0

c) La inversa puede ser obtenida partiendo de una de las relaciones obtenida anteriormente:
P (C2 + 1 I 3 ) + I 4 1 = 0
( )
⇒ P −1 P C2 + 1 I 3 + I 4 P −11 = 0 ⇒ ( )
⇒ C2 + 1 I 3 + I 4 P −1 = 0

⇒ P −1 = −
1
I4
(
C2 + 1 I 3 ) ⇔ P −1 =
1
det (P )
adj[P ] ∴ (
adj[P ] = − C2 + 1 I 3 )
Luego:
  − 37 22 15 − 17  1 0 0 0  − 21 22 15 − 17 
  0   
1  7 −2 −5 −5  1 0 0  1  7 14 − 5 − 5 
P −1 =−  + 16 =
(−32)   10 − 12 − 14 2  0 0 1 0  32  10 − 12 2 2 

 9     
 − 14 − 11 5  0 0 0 1   9 − 14 − 11 21 

NOTA 1: Este procedimiento que acabamos de realizar, en la literatura, se conoce como


Método de Faddeev-Leverrier.

Universidad de Castilla- La Mancha Draft Por: Eduardo W. V. Chaves (2012)


Ciudad Real - España
64 PROBLEMAS RESUELTOS DE MECÁNICA DEL MEDIO CONTINUO

NOTA 2: También podemos obtener los coeficientes del polinomio característico a través del
siguiente procedimiento. Considerando P 4 − P 3 I 1 + P 2 I 2 − P I 3 + I 4 1 = 0
El último coeficiente, en el caso I 4 = det (P ) = −32 .
El coeficiente I 3 se obtiene por la suma de los determinantes de las matrices resultantes al
eliminar 1 fila y 1 columna asociados a la diagonal principal, i.e.:
1 2 3 1  1 2 3 1  1 2 3 1  1 2 3 1
2 2 1 2 2 2 1 2 2 2 1 2 2 2 1 2
I3 =  + + +
4 1 5 3  4 1 5 3  4 1 5 3  4 1 5 3
       
3 1 2 4  3 1 2 4  3 1 2 4  3 1 2 4
2 1 2 1 3 1  1 2 1  1 2 3
= 1 5 3 +  4 5 3 +  2 2 2 +  2 2 1 = −16
1 2 4  3 2 4  3 1 4  4 2 5

El coeficiente I 2 se obtiene por la suma de los determinantes de las matrices resultantes al


eliminar 2 filas y 2 columnas asociados a la diagonal principal, i.e.:
1 2 3 1 1 2 3 1 1 2 3 1
2 2 1 2 2 2 1 2 2 2 1 2
I2 =  +  +  +
4 1 5 3 4 1 5 3 4 1 5 3
     
3 1 2 4 3 1 2 4 3 1 2 4
1 2 3 1  1 2 3 1
2 2 1 2 2 2 1 2
+  + +
4 1 5 3  4 1 5 3
   
3 1 2 4  3 1 2 4
1 2 3 1
2 2 1 2
+ 
4 1 5 3
 
3 1 2 4
5 3  2 2 2 1 1 1  1 3 1 2
=  +   +   +   +  +   = 21
 2 4 1 4 1 5 3 4  4 5  2 2
El coeficiente I 1 se obtiene por la suma de los determinantes de las matrices resultantes al
eliminar 3 filas y 3 columnas asociados a la diagonal principal, i.e.:
1 2 3 1  1 2 3 1  1 2 3 1  1 2 3 1
2 2 1 2  2 2 1 2 2 2 1 2 2 2 1 2
I1 =  + + +
4 1 5 3  4 1 5 3  4 1 5 3  4 1 5 3
       
3 1 2 4  3 1 2 4  3 1 2 4  3 1 2 4
= [4] + [1] + [2] + 5 = 12 = Tr (P )

Universidad de Castilla- La Mancha Draft Por: Eduardo W. V. Chaves (2012)


Ciudad Real - España
1 TENSORES 65

Ejemplo 1.79
Demostrar que si A es un tensor de segundo orden se cumple que:

a) II A =
1
2
{
[Tr( A )]2 − Tr( A 2 ) }
b) det ( A ) =
1
6
{
[Tr (A )]3 + 2 Tr( A 3 ) − 3Tr(A ) Tr(A 2 ) }
Solución:
a) Fue demostrado en el Ejemplo 1.76 que III A A −1 = (A 2 − AI A + 1 II A ) , luego, aplicando el
doble producto escalar con el tensor identidad, obtenemos que:
( )
III A A −1 : 1 = A 2 − AI A + 1 II A : 1 = A 2 : 1 − A : 1 I A + 1 : 1 II A
−1
III A Tr ( A ) = Tr ( A ) − Tr ( A ) I A + Tr (1) II A = Tr ( A 2 ) − ( I A ) 2 + 3 II A
2

Teniendo en cuenta la inversa de un tensor A −1 =


[cof ( A )]T , podemos decir que:
III A


III A Tr ( A −1 ) = Tr ( III A A −1 ) = Tr  III A
[cof ( A )]T 
( )
 = Tr [cof ( A )]T = Tr ([cof ( A ) ]) = II A
 III A 
 
Con lo cual, podemos decir que:
III A Tr ( A −1 ) = II A = Tr ( A 2 ) − ( I A ) 2 + 3 II A
⇒ II A − 3 II A = Tr ( A 2 ) − ( I A ) 2

⇒ II A =
1
2
{
( I A ) 2 − Tr ( A 2 ) }
b) Partiendo del teorema de Cayley-Hamilton, que afirma que todo tensor cumple su propia
ecuación característica:
A 3 − A 2 I A + AII A − III A 1 = 0 (1.58)

donde I A = [Tr (A )] , II A =
1
2
{ }
[Tr( A )]2 − Tr( A 2 ) , III A = det (A ) son los invariantes principales

del tensor A . Haciendo el doble producto escalar con el tensor identidad de segundo orden
( 1 ) en la expresión (1.58) obtenemos que:
A 3 : 1 − A 2 : 1 I A + A : 1II A − III A 1 : 1 = 0 : 1
Tr ( A 3 ) − Tr ( A 2 ) I A + Tr ( A ) II A − III A [Tr(1)] = [Tr(0)]
Tr ( A 3 ) − Tr ( A 2 ) Tr ( A ) + Tr ( A )
1
2
{
[Tr(A )]2 − Tr(A 2 ) − III A 3 = 0}
1 1
Tr ( A 3 ) − Tr ( A 2 ) Tr ( A ) + [Tr ( A )] − Tr ( A ) Tr ( A 2 ) − III A 3 = 0
3

2 2
1
2
{ }
2 Tr ( A 3 ) − 3 Tr ( A 2 ) Tr ( A ) + [Tr ( A )] − III A 3 = 0
3

Con lo cual obtenemos que:

III A = det ( A ) =
1
6
{
[Tr( A )]3 + 2 Tr( A 3 ) − 3Tr (A 2 ) Tr( A ) }
o en notación indicial:

Universidad de Castilla- La Mancha Draft Por: Eduardo W. V. Chaves (2012)


Ciudad Real - España
66 PROBLEMAS RESUELTOS DE MECÁNICA DEL MEDIO CONTINUO

III A = det ( A ) =
1
6
{
A ii A jj A kk + 2A ij A jk A ki − 3A ij A ji A kk }
NOTA: Es interesante destacar que los invariantes principales del tensor A vienen formados
por la combinación de los tres invariantes fundamentales del tensor ( Tr (A ) , Tr ( A 2 ) y
Tr ( A 3 ) ), es decir:

I A = Tr ( A )

II A =
1
2
2
{
I A − Tr ( A 2 ) }
1
{
III A = [Tr ( A )] + 2 Tr ( A 3 ) − 3 Tr ( A 2 ) Tr ( A )
6
3
}

Ejemplo 1.80

Demostrar que II T = III T Tr ( T −1 ) , donde II T =


1
2
{ }
[Tr( T )]2 − Tr(T 2 ) es el segundo invariante
principal de T , III T es el tercer invariante principal, es decir, el determinante de T .

Solución: Fue demostrado en el Ejemplo 1.76 que T −1 =


1
(
III T
)
T 2 − TI T + 1 II T , luego,

aplicando el doble producto escalar con el tensor identidad, obtenemos que:

T −1 : 1 =
1
III T
(
T 2 − TI T + 1 II T : 1 =
1
)
III T
(
T 2 : 1 − T : 1 I T + 1 : 1 II T )
Tr ( T −1 ) =
1
III T
(
Tr ( T 2 ) − Tr ( T ) I T + Tr (1) II T )
⇒ III T Tr ( T −1 ) = Tr ( T 2 ) − I T2 + 3 II T
14243
= −2 II T
−1
⇒ III T Tr ( T ) = II T

Ejemplo 1.81
Demostrar que:
r r
r r 1 β (c ⊗ b )
(α1 + β c ⊗ b) −1 = 1 − r r (1.59)
α α (α + βc ⋅ b)
r r
donde c , b son vectores, 1 es el tensor identidad de segundo orden, y α y β escalares.
Solución:
r r
Haciendo que T = (α1 + β c ⊗ b) , y teniendo en cuenta la expresión de la inversa obtenida en
el Ejemplo 1.76:

T −1 =
1
III T
(
T 2 − TI T + 1 II T ) (1.60)

A continuación obtenemos T 2 :

Universidad de Castilla- La Mancha Draft Por: Eduardo W. V. Chaves (2012)


Ciudad Real - España
1 TENSORES 67

r r r r
T 2 = T ⋅ T = (α1 + β c ⊗ b) ⋅ (α1 + β c ⊗ b)
r r r r r r r r
= α 2 1 ⋅ 1 + αβ1 ⋅ (c ⊗ b) + αβ (c ⊗ b) ⋅ 1 + β 2 (c ⊗ b) ⋅ (c ⊗ b)
r r r r r r r r
donde se cumple que (c ⊗ b) ⋅ (c ⊗ b) = (c ⋅ b)(c ⊗ b) , ver Ejemplo 1.19. Luego, la expresión
anterior puede ser rescrita como:
r r r r r r
T 2 = α 2 1 + 2αβ (c ⊗ b) + β 2 (c ⋅ b)(c ⊗ b)
y la traza viene dada por:

[ r r r r r r
Tr ( T 2 ) = Tr α 2 1 + 2αβ (c ⊗ b) + β 2 (c ⋅ b)(c ⊗ b) ]
r r r r r r
= α 2 Tr (1) + 2αβTr (c ⊗ b) + β 2 (c ⋅ b) Tr (c ⊗ b)
r r r r r r
= 3α 2 + 2αβ (c ⋅ b) + β 2 (c ⋅ b)(c ⋅ b)
r r r r
= 3α 2 + 2αβ (c ⋅ b) + β 2 (c ⋅ b) 2
A continuación calculamos los invariantes principales de T
r r r r r r
I T = Tr (α1 + β c ⊗ b) = αTr (1) + βTr (c ⊗ b) = 3α + β (c ⋅ b)

[ ]
r r 2 r r r r
( I T ) 2 = 3α + β (c ⋅ b) = 9α 2 + 6β (c ⋅ b) + β 2 (c ⋅ b) 2

II T =
1
2
{ 2
} { 1
2
r r r r
[ r r r r
I T − Tr ( T 2 ) = 9α 2 + 6β (c ⋅ b) + β 2 (c ⋅ b) 2 − 3α 2 + 2αβ (c ⋅ b) + β 2 (c ⋅ b) 2 ]}
r r
= 3α + 2αβ (c ⋅ b)
2

r r r r
III T = det (α1 + β c ⊗ b) = α 3 + α 2 β c ⋅ b (ver Ejemplo 1.49)
Luego, la expresión (1.60) queda:
III T T −1 = T 2 − I T T + II T 1
r r r r r r
= α 2 1 + 2αβ (c ⊗ b) + β 2 (c ⋅ b)(c ⊗ b)
[ r r
] r r
[ r r
− 3α + β (c ⋅ b) (α1 + β c ⊗ b) + 3α 2 + 2αβ (c ⋅ b) 1 ]
r r r r r r r r r r
= α 2 1 + 2αβ (c ⊗ b) + β 2 (c ⋅ b)(c ⊗ b) − 3α 2 1 − 3αβ (c ⊗ b) − αβ (c ⋅ b)1
r r r r r r (1.61)
− β 2 (c ⋅ b)(c ⊗ b) + 3α 2 1 + 2αβ (c ⋅ b)1
r r r r
= 1α 2 + αβ (c ⋅ b)1 − αβ (c ⊗ b)
r r r r
= (α 2 + αβ c ⋅ b)1 − αβ (c ⊗ b)
1 r r r r
= (α 3 + α 2 β c ⋅ b)1 − αβ (c ⊗ b) = [adj( T ) ] = [cof ( T ) ]
T

α
r r r r
Teniendo en cuenta que T = (α1 + β c ⊗ b) , III T = α 3 + α 2 β c ⋅ b , la expresión anterior
queda:
1  III T r r 
T −1 =  1 − αβ (c ⊗ b) 
III T  α 
r r r r (1.62)
1 III T αβ (c ⊗ b) 1 αβ (c ⊗ b)
= 1− = 1− r r
α III T III T α (α 3 + α 2 β c ⋅ b )

o aun:

Universidad de Castilla- La Mancha Draft Por: Eduardo W. V. Chaves (2012)


Ciudad Real - España
68 PROBLEMAS RESUELTOS DE MECÁNICA DEL MEDIO CONTINUO

r r 1 β r r
(α1 + β c ⊗ b) −1 = 1 − r r (c ⊗ b) Notación Tensorial (1.63)
α α (α + β c ⋅ b)
1 β
(α δ ij + β c i b j ) −1 = δ ij − (c b ) Notación Indicial (1.64)
α α (α + β c k b k ) i j

[α[1] + β [{c}{b} ] ] T −1
=
1
α
[ 1] −
β
α (α + β {c} {b})
T
{c}{b}T [ ] Notación matricial (1.65)

NOTA 1: La expresión anterior también es válida para matrices de n-dimensiones.


Para el caso particular cuando α = 1 , β = 1 , obtenemos que:
r r
r r −1 (c ⊗ b)
(1 + c ⊗ b) = 1 − r r (1.66)
1+ c ⋅b

NOTA 2: La expresión (1.63) puede ser reescrita como:


r r 1 β r r
T −1 = (α1 + β c ⊗ b) −1 = 1 − r r (c ⊗ b)
α α (α + β c ⋅ b )
= r
1 r r
[r r
r (α 2 + αβ c ⋅ b)1 − βα (c ⊗ b) =
(α + α β c ⋅ b )
3 2
1
det ( T )
[adj( T )] ]
r r r r r r
con lo cual concluimos que adj(α1 + β c ⊗ b) = (α 2 + αβ c ⋅ b)1 − βα (c ⊗ b) .
NOTA 3: Podemos extender la ecuación (1.63) de tal forma que:
1 β
(α I sym + β A ⊗ B ) −1 = I sym − ( A ⊗ B)
α α (α + β A : B )

donde ahora tenemos que I sym es el tensor identidad de cuarto orden, A y B son tensores de
segundo orden, y α y β son escalares. Con eso es de fácil demostración que ( I sym ) −1 = I sym .

Ejemplo 1.82
r r 1 β r r
Teniendo en cuenta que (α1 + β c ⊗ b) −1 = 1− r r (c ⊗ b) , demostrar que:
α α (α + β c ⋅ b )
r r 1
(αA + β a ⊗ b) −1 = A −1 −
α
r
β
r
α (α + β b ⋅ A −1 ⋅ a)
r
[ r
( A −1 ⋅ a) ⊗ (b ⋅ A −1 ) ] (1.67)

r r
donde a , b son vectores, A es un tensor de segundo orden, con det ( A ) ≠ 0 ( ∃A −1 ), y α , β
son escalares.
Solución:
r r
Observemos que el término (αA + β a ⊗ b) , puede ser reescrito como:
r r r r
(αA + β a ⊗ b) = A ⋅ (α1 + β A −1 ⋅ a ⊗ b)

Utilizando la propiedad de la inversa tal que ( A ⋅ B ) −1 = B −1 ⋅ A −1 , podemos decir que:


r r
[ r r −1 r r
]
(αA + β a ⊗ b) −1 = A ⋅ (α1 + β A −1 ⋅ a ⊗ b) = (α1 + β A −1 ⋅ a ⊗ b) −1 ⋅ A −1

Universidad de Castilla- La Mancha Draft Por: Eduardo W. V. Chaves (2012)


Ciudad Real - España
1 TENSORES 69

r
Observemos que el resultado de la operación A −1 ⋅ a resulta un vector y lo denotamos por el
r r
vector c = A −1 ⋅ a , con lo cual podemos reescribir la expresión anterior como:
r r r r
(αA + β a ⊗ b) −1 = (α1 + β A −1 ⋅ a ⊗ b) −1 ⋅ A −1
r r
= (α1 + β c ⊗ b) −1 ⋅ A −1
1 β r r 
r r (c ⊗ b)  ⋅ A
−1
= 1−
 α α (α + β c ⋅ b ) 
1 β r r
= 1 ⋅ A −1 − r r (c ⊗ b) ⋅ A
−1
α α (α + β c ⋅ b )
1 β r r
= A −1 − r r c ⊗b⋅A
−1
α α (α + β c ⋅ b )
β r
1 −1 r
= A −1 − r r ( A ⋅ a) ⊗ (b ⋅ A )
−1
α α (α + β c ⋅ b )
1 β r r
= A −1 − r r ( A −1 ⋅ a) ⊗ (b ⋅ A −1 )
α α (α + β b ⋅ A −1 ⋅ a)
La expresión anterior en notación indicial queda
1 β
(αA ij + β a i b j ) −1 = A ij−1 − ( A ik−1 a k )(b s A −sj1 )
α α (α + β b p A −pq1 a q )
r r r r
Cuidado con la operación ( A −1 ⋅ a) ⋅ b ≠ 1
A −1 ⋅ (a ⋅ b) , esta última no tiene consistencia, ya
4243
Expresión errónea
r r
que no podemos tener un producto escalar (contracción) con un escalar (a ⋅ b) . En notación
r r r
indicial se puede comprobar c ⋅ b = c i b i = ( A −1 ⋅ a) i b i = A ik−1 a k b i , luego, las expresiones
r r
posibles son ( A −1 ⋅ a) ⋅ b = b i A ik−1a k = a k A ik−1b i = A ik−1b i a k = A ik−1 a k b i = A ik−1a k b i .
1424 r3 1 424 3 1 424r 3 1424
r r 3 1424 3
r r r r r r
b⋅A −1⋅a a⋅A −T ⋅b A −1:(b⊗a) A −T :( a⊗b ) A −1:( a⊗b )T

Para el caso particular cuando α = 1 , β = 1 , recaemos en la fórmula de Sherman-Morrison:


r r
r r −1 ( A −1 ⋅ a) ⊗ (b ⋅ A −1 ) Fórmula de Sherman-Morrison
−1
( A + a ⊗ b) = A − r r (1.68)
1 + b ⋅ A −1 ⋅ a (notación tensorial)

En notación matricial la ecuación anterior queda:

{ }{
 [ A] −1 {a} {b}T [ A] −1 } 
T
Fórmula de Sherman-Morrison
[[ A] + [{a}{b} ] ]
T −1
= [ A] −1 −

1 + {b}T [ A] −1 {a} (notación matricial)
(1.69)

r r r r
NOTA 1: Observar que si (αA + β a ⊗ b) = A ⋅ (α1 + β A −1 ⋅ a ⊗ b) , el determinante viene
definido por:
r r
[ r r
]
r r
det (αA + β a ⊗ b) = det A ⋅ (α1 + β A −1 ⋅ a ⊗ b) = det [A ]det (α1 + β A −1 ⋅ a ⊗ b)
r
[ ]
r
= det [A ](α 3 + α 2 β b ⋅ A −1 ⋅ a)
con lo cual, la expresión (1.67) puede aun ser reescrita como:

Universidad de Castilla- La Mancha Draft Por: Eduardo W. V. Chaves (2012)


Ciudad Real - España
70 PROBLEMAS RESUELTOS DE MECÁNICA DEL MEDIO CONTINUO

r r
(α A + β a ⊗ b) −1 =
1
{ r r r r
A (α 2 + αβ b ⋅ A −1 ⋅ a) A −1 − A αβ ( A −1 ⋅ a) ⊗ (b ⋅ A −1 )
γ
[ ]}
r r r r
con γ = det (αA + β a ⊗ b) = A (α 3 + α 2 β b ⋅ A −1 ⋅ a) . (1.70)
con eso concluimos que:
r r
{ r r r r
adj(αA + β a ⊗ b) = A (α 2 + αβ b ⋅ A −1 ⋅ a) A −1 − A αβ ( A −1 ⋅ a) ⊗ (b ⋅ A −1 ) [ ]}
NOTA 2: Podemos extrapolar la expresión (1.67) de tal forma que:

(α D + β A ⊗ B ) −1 =
1
α
D −1 −
β
α (α + β B : D −1 : A )
[(D −1
: A ) ⊗ (B : D −1 ) ] (1.71)

donde ahora tenemos que D es un tensor de cuarto orden, A y B son tensores de segundo
orden, y α , β son escalares. Aun podemos decir que:

(α D + β A ⊗ B ) −1 =
1
γ
{D (α 2
+ αβ B : D −1 : A )D −1 − D αβ (D −1 : A ) ⊗ (B : D −1 )[ ]}
con γ = det (αD + β A ⊗ B ) = D (α 3 + α 2 β B : D −1 : A ) . (1.72)
donde podemos concluir que:
det (αD + β A ⊗ B ) = det (D )(α 3 + α 2 β B : D −1 : A ) (1.73)
{
adj(αD + β A ⊗ B ) = D (α 2 + αβ B : D −1 : A )D −1 − αβ D (D −1 : A ) ⊗ (B : D −1 ) [ ]} (1.74)

Ejemplo 1.83
r r r r
a) Dado un tensor de segundo orden C = (α1 + β a ⊗ b + γ c ⊗ d) , demostrar que:
r r r r r r r r
α1 + β a ⊗ b + γ c ⊗ d = α 3 + α 2 γ (c ⋅ d) + α 2 β (a ⋅ b) + αβγ (a ⋅ b)(c ⋅ d) − (a ⋅ d)(b ⋅ c ) [r r r r r r r r
]
(1.75)
r r r r r r r r
donde α1 + β a ⊗ b + γ c ⊗ d = det (α1 + β a ⊗ b + γ c ⊗ d) representa el determinante del
r r r r
tensor C . b) Para el caso particular cuando α = 1 , d = a , c = b , demostrar que:
r r r r r r r r 2
det (1 + β a ⊗ b + γ b ⊗ a) = 1 + (β + γ )(a ⋅ b) − βγ a ∧ b (1.76)

Solución:
r r r r
Definimos un tensor auxiliar D = α1 + β a ⊗ b y a su vez tenemos que C = (D + γ c ⊗ d) .
r r r r
Según el Ejemplo 1.82, ecuación (1.70), se cumple que det (D + γ c ⊗ d) = D (1 + γ d ⋅ D −1 ⋅ c ) ,
donde:
r r r r
det (D) ≡ D = det (α1 + β a ⊗ b ) = α 3 + α 2 β (a ⋅ b) y
r r 1 β r r
(D ) −1 = (α1 + β a ⊗ b) −1 = 1 − r r (a ⊗ b)
α α (α + β a ⋅ b )

Universidad de Castilla- La Mancha Draft Por: Eduardo W. V. Chaves (2012)


Ciudad Real - España
1 TENSORES 71

Con eso, podemos decir que:


r r r r
det (D + γ c ⊗ d) = D (1 + γ d ⋅ D −1 ⋅ c )

[ r r 
] r 1
= α 3 + α 2 β (a ⋅ b ) (1 + γ d ⋅  1 −
 α
β
r
r r  r
r (a ⊗ b)  ⋅ c 
α (α + β a ⋅ b )  
  

[ r r 
] 1 r r
= α 3 + α 2 β (a ⋅ b ) 1 + γ  d ⋅ 1 ⋅ c −
 α
β
r
r r r r 
r d ⋅ (a ⊗ b) ⋅ c  
α (α + β a ⋅ b ) 
 

[ r r 
] 1 r r
= α 3 + α 2 β (a ⋅ b ) 1 + γ  d ⋅ c −
 α
β
r
r r r r 
r (d ⋅ a) ⊗ (b ⋅ c )  
α (α + β a ⋅ b ) 
 

[ r r 
]
γ r r
= α 3 + α 2 β (a ⋅ b ) 1 + (c ⋅ d) −
αβγ
r
r r r r 
r (a ⋅ d)(b ⋅ c ) 
α 2 (α + β a ⋅ b )
 α 
r r r r r r r r r r r r
Notar que (d ⋅ a) ⊗ (b ⋅ c ) = (a ⋅ d) ⊗ (b ⋅ c ) = (a ⋅ d)(b ⋅ c ) .
123 123
escalar escalar

r r
[ r r 
] γ r r 
[ r r 
det (D + γ c ⊗ d) = α 3 + α 2 β (a ⋅ b) 1 + (c ⋅ d)  − α 3 + α 2 β (a ⋅ b)  ]
αβγ
r
r r r r 
r (a ⋅ d)(b ⋅ c ) 
 α   α (α + β a ⋅ b)
2


[ r r 
 α
] γ r r 

r r r r
= α 3 + α 2 β (a ⋅ b) 1 + (c ⋅ d)  − αβγ (a ⋅ d)(b ⋅ c )

3
[ 2 r r 2 r r r r r r
] r r r r
= α + α γ (c ⋅ d) + α β (a ⋅ b) + αβγ (a ⋅ b)(c ⋅ d) − αβγ (a ⋅ d)(b ⋅ c )
Luego:
r r r r r r r r r r r r
[ r r r r
det (α1 + β a ⊗ b + γ c ⊗ d) = α 3 + α 2 γ (c ⋅ d) + α 2 β (a ⋅ b ) + αβγ (a ⋅ b )( c ⋅ d) − (a ⋅ d)(b ⋅ c ) ]
Con eso demostramos la ecuación (1.75).
r r r r
Para el caso particular cuando d = a , c = b , tenemos que:
r r r r r r r r r r r r
[ r r r r
det (α1 + β a ⊗ b + γ c ⊗ d) = α 3 + α 2 γ (c ⋅ d) + α 2 β (a ⋅ b) − αβγ (a ⋅ d)(b ⋅ c ) − (a ⋅ b)(c ⋅ d) ]
r r r r r r r r r r r r
[ r r r r
det (α1 + β a ⊗ b + γ b ⊗ a) = α 3 + α 2 γ (b ⋅ a) + α 2 β (a ⋅ b) − αβγ (a ⋅ a)(b ⋅ b) − (a ⋅ b)(b ⋅ a) ]
r r
[
r r r r r r r r
= α 3 + α 2 (β + γ )(a ⋅ b) − αβγ (a ⋅ a)(b ⋅ b) − (a ⋅ b)(a ⋅ b) ]
r r 2 r 2 r 2 r r 2
En el Ejemplo 1.1 hemos demostrado que a ∧ b = a b − a ⋅ b . Con eso ( )
demostramos que:
r r r r r r r r 2
det (α1 + β a ⊗ b + γ b ⊗ a) = α 3 + α 2 (β + γ )(a ⋅ b) − αβγ a ∧ b

Para el caso particular cuando α = 1 , obtenemos que:


r r r r r r r r 2
det (1 + β a ⊗ b + γ b ⊗ a) = 1 + (β + γ )(a ⋅ b) − βγ a ∧ b

Ejemplo 1.84
r r r r
a) Obtener la inversa del tensor de segundo orden C = (α1 + β a ⊗ b + γ c ⊗ d) .

Universidad de Castilla- La Mancha Draft Por: Eduardo W. V. Chaves (2012)


Ciudad Real - España
72 PROBLEMAS RESUELTOS DE MECÁNICA DEL MEDIO CONTINUO

r r r r
p ⊗ p (B ⋅ q) ⊗ (B ⋅ q)
b.1) Dado el tensor D = B + r r − r r donde B = B T y ∃B −1 , demostrar que
p⋅q q ⋅ B ⋅q
se cumple que:
r r r r
( p ⋅ q + p ⋅ B −1 ⋅ p ) r r
D −1 = B −1 + r r 2
( p ⋅ q) ( p ⋅ q)
[
[q ⊗ q ] − r 2 r qr ⊗ ( B −1 ⋅ pr ) ]sym
(1.77)

b.2) Si B es un tensor definido positivo, determinar las condiciones para que D sea un tensor
no-singular.
Solución:
r r r r
a) Denotando por A = (α1 + β a ⊗ b) quedamos con C = ( A + γ c ⊗ d) , y teniendo en cuenta
que
r r 1
(αA + β a ⊗ b) −1 = A −1 −
α
r
β
r
α (α + β b ⋅ A −1 ⋅ a)
[ r r
( A −1 ⋅ a) ⊗ (b ⋅ A −1 ) ] (1.78)

Ver Ejemplo 1.82, ecuación (1.67). Luego


r r
( A + γ c ⊗ d) −1 = A −1 − r
γ
r
(1 + γ d ⋅ A −1 ⋅ c )
[ r r
( A −1 ⋅ c ) ⊗ (d ⋅ A −1 ) ] (1.79)

En el Ejemplo 1.81 hemos demostrado que:


r r
r r 1 β (c ⊗ b )
(α1 + β c ⊗ b) −1 = 1 − r r (1.80)
α α (α + βc ⋅ b)

Con eso concluimos que:


r r
r r 1 β (a ⊗ b)
A −1 = (α1 + β a ⊗ b) −1 = 1 − r r
α α (α + β a ⋅ b )
Además tenemos que
r r r r r r
r 1 β ( a ⊗ b )  r 1 r β ( a ⊗ b ) r 1 r β (b ⋅ c) r
A ⋅c =  1 −
−1
r r  ⋅ c = 1 ⋅ c − r r ⋅c = c − r r a
α α (α + β a ⋅ b )  α α (α + β a ⋅ b ) α α (α + β a ⋅ b )

r r r r r r
r r 1 β ( a ⊗ b )  1 r r β ( a ⊗ b ) 1 r β ( d ⋅ a) r
d⋅A = d⋅ 1 −
−1
r r  = d ⋅1 − d ⋅ r r = d− r r b
α α (α + βa ⋅ b)  α α (α + β a ⋅ b ) α α (α + β a ⋅ b )

Con eso concluimos que
r r r r r r
[
r r r r
] [ r r r r
(α1 + β a ⊗ b + γ c ⊗ d) −1 = θ (1) 1 + θ ( 2 ) (a ⊗ b) + θ ( 3) θ (1) c + θ ( 2 ) (b ⋅ c )a ⊗ θ (1) d + θ ( 2 ) (a ⋅ d)b ]
(1.81)
donde
1
θ (1) =
α
−β
θ ( 2) = r r
α (α + β a ⋅ b )

Universidad de Castilla- La Mancha Draft Por: Eduardo W. V. Chaves (2012)


Ciudad Real - España
1 TENSORES 73

−γ
θ ( 3) = r r
(1 + γ d ⋅ A −1 ⋅ c )
r r 1 r r β r r r r
d ⋅ A −1 ⋅ c = (d ⋅ c ) − r r (d ⋅ a )(b ⋅ c )
α α (α + β a ⋅ b )
b.1) Podemos reescribir el tensor D como:
r r r r r r r r
p⊗ p (B ⋅ q) ⊗ (B ⋅ q) p⊗ p (B ⋅ q) ⊗ (B ⋅ q)
D = B ⋅1 + 1 ⋅ r r − 1 ⋅ r r = B ⋅ 1 + (B ⋅ B ) ⋅ r r − (B ⋅ B ) ⋅
−1 −1
r r
p⋅q q ⋅ B ⋅q p⋅q q ⋅B⋅q
r r r r r r r r
 p⊗ p −1 ( B ⋅ q ) ⊗ ( B ⋅ q )   ( B −1 ⋅ p) ⊗ p ( B −1 ⋅ B ⋅ q ) ⊗ ( B ⋅ q ) 
= B ⋅ 1 + B ⋅ r r − B ⋅
−1
r r  = B ⋅ 1 + r r − r r 
 p⋅q q ⋅B ⋅q   p⋅q q ⋅B⋅q 
r r r r
 ( B −1 ⋅ p ) ⊗ p q ⊗ ( B ⋅ q ) 
= B ⋅ 1 + r r − r r 
 p⋅q q ⋅B⋅q 
Denotando por
r r r r r r r r 1 −1
a = ( B −1 ⋅ p ) ; b=p ; c =q ; d = (B ⋅ q) ; β= r r ; γ= r r
p⋅q q ⋅B⋅q

[ r r r r
D = B ⋅ 1 + β a ⊗ b + γc ⊗ d ] = B ⋅C ⇒ D −1 = ( B ⋅ C ) −1 = C −1 ⋅ B −1

[ r r r r
]
donde C = 1 + βa ⊗ b + γc ⊗ d . La inversa del tensor C puede ser obtenido a través del
apartada (a) con α = 1 . Además tenemos que:
θ (1) = 1 ,
−β −β −1 1 −1
θ ( 2) = r r = r r = r r = r r r −1 r
α (α + βa ⋅ b) (1 + βa ⋅ b) p ⋅ q (1 + r r ( B −1 ⋅ p ) ⋅ p ) ( p ⋅ q + p ⋅ B ⋅ p )
1 r r
p⋅q
r r 1 r r β r r r r
d ⋅ A −1 ⋅ c = (d ⋅ c ) − r r ( a ⋅ d )(b ⋅ c )
α α (α + β a ⋅ b )
r r −1 r r r r
= (( B ⋅ q ) ⋅ q ) + r r r −1 r
(( B −1 ⋅ p) ⋅ ( B ⋅ q ) )( p ⋅ q )
( p ⋅ q + p ⋅ B ⋅ p)
r r r r
r r − ( p ⋅ ( B −T ⋅ B ) ⋅ q ) ( p ⋅ q )
= q ⋅B ⋅q + r r r r
( p ⋅ q + p ⋅ B −1 ⋅ p)
−γ 1 1
θ ( 3) = r
−1 r
= r r r r r r
(1 + γ d ⋅ A ⋅ c ) q ⋅ B ⋅ q  −1 r r − ( p ⋅ ( B −T ⋅ B ) ⋅ q ) ( p ⋅ q ) 
1 + r
 r q ⋅ B ⋅ q + r r r r 

 q ⋅B⋅q ( p ⋅ q + p ⋅ B −1 ⋅ p ) 
r r r −1 r
( p ⋅ q + p ⋅ B ⋅ p)
= r r r r
( p ⋅ ( B −T ⋅ B ) ⋅ q ) ( p ⋅ q )
r r r r
−1 ( p ⋅ q + p ⋅ B −1 ⋅ p ) −1
θ ( 2 ) θ ( 3) = r r r −1 r r r r r = r r r r
( p ⋅ q + p ⋅ B ⋅ p) ( p ⋅ ( B ⋅ B ) ⋅ q ) ( p ⋅ q ) ( p ⋅ ( B ⋅ B ) ⋅ q ) ( p ⋅ q )
−T −T

r r −1 −1 r r
θ ( 2 ) θ ( 3) (a ⋅ d ) = r r r r (( B ⋅ p) ⋅ ( B ⋅ q ))
( p ⋅ (B −T
⋅ B) ⋅ q) ( p ⋅ q )
−1 r r −1
r r r ( p ⋅ (B ⋅ B) ⋅ q ) = r r
−T
= r
( p ⋅ ( B −T ⋅ B) ⋅ q) ( p ⋅ q ) ( p ⋅ q)

Universidad de Castilla- La Mancha Draft Por: Eduardo W. V. Chaves (2012)


Ciudad Real - España
74 PROBLEMAS RESUELTOS DE MECÁNICA DEL MEDIO CONTINUO

r r −1 r r −1
θ ( 2 ) θ ( 3) (b ⋅ c ) = r r r r ( p ⋅ q) = r r
( p ⋅ (B −T
⋅ B) ⋅ q ) ( p ⋅ q ) ( p ⋅ (B ⋅ B) ⋅ q )
−T

r r r r −1 −1 r r r r
θ ( 2 ) θ ( 3) (b ⋅ c )(a ⋅ d ) = r r r r (( B ⋅ p) ⋅ ( B ⋅ q ))( p ⋅ q ) = −1
( p ⋅ (B −T
⋅ B) ⋅ q ) ( p ⋅ q )
La expresión (1.81) queda:
r r r
[ r r r r
] [
r r r
C −1 = 1 + θ ( 2 ) (a ⊗ b ) + θ ( 3) c + θ ( 2 ) (b ⋅ c )a ⊗ d + θ ( 2 ) (a ⋅ d )b ]
r r
[ ] r r
[ ] r r r r
[
r r r r
C −1 = 1 + θ ( 2 ) a ⊗ b + θ ( 3) c ⊗ d + θ ( 3) θ ( 2 ) (a ⋅ d ) c ⊗ b + θ ( 3) θ ( 2 ) (b ⋅ c ) a ⊗ d +] [ ]
r r r r r r
+ θ ( 3) θ (22 ) (b ⋅ c )(a ⋅ d ) a ⊗ b [ ]
{ r r r r r r
}[r r
] [
r r r r
]
C −1 = 1 + θ ( 2 ) + θ ( 3) θ (22 ) (b ⋅ c )(a ⋅ d ) a ⊗ b + θ ( 3) c ⊗ d + θ ( 3) θ ( 2 ) (a ⋅ d ) c ⊗ b + [ ]
r r r r
+ θ ( 3) θ ( 2 ) (b ⋅ c ) a ⊗ d [ ]
notar que: {θ ( 2 ) + θ ( 3) θ (22 ) (b ⋅ c )(a ⋅ d )}= θ ( 2 ) {1 + θ ( 3) θ ( 2 ) (b ⋅ c )(a ⋅ d )}= θ ( 2 ) {1 − 1} = 0
r r r r r r r r

r r
[ ]
r r r r
[ r r r r
]
C −1 = 1 + θ ( 3) c ⊗ d + θ ( 3) θ ( 2 ) (a ⋅ d ) c ⊗ b + θ ( 3) θ ( 2 ) (b ⋅ c ) a ⊗ d [ ]
r r
[ ]r r r r
[ r r r r
]
C −1 = 1 + θ ( 3) c ⊗ d + θ ( 3) θ ( 2 ) (a ⋅ d ) c ⊗ b + θ ( 3) θ ( 2 ) (b ⋅ c ) a ⊗ d [ ]
r r r r
( p ⋅ q + p ⋅ B −1 ⋅ p) r r −1 r r
=1 + r r r r [q ⊗ ( B ⋅ q ) ] + r r [q ⊗ p ] + L
( p ⋅ (B ⋅ B) ⋅ q ) ( p ⋅ q )
−T
( p ⋅ q)

L+ r
−1 −1 r
r ( B ⋅ p) ⊗ ( B ⋅ q )
( p ⋅ (B ⋅ B) ⋅ q )
−T
r
[ ]
Con eso, podemos obtener que:
D −1 = C −1 ⋅ B −1
r r r r
 ( p ⋅ q + p ⋅ B −1 ⋅ p ) r r −1 r r
= 1 + r r r r [q ⊗ ( B ⋅ q )] + r r [q ⊗ p ] + L
 ( p ⋅ (B ⋅ B) ⋅ q ) ( p ⋅ q )
−T
( p ⋅ q)

L+ r
−1
( p ⋅ (B ⋅ B) ⋅ q )
−T
−1 r r  −1
r ( B ⋅ p) ⊗ ( B ⋅ q )  ⋅ B [ ]

r r r r
( p ⋅ q + p ⋅ B ⋅ p)
−1
r r −1 r r
= B −1 + r r r r [q ⊗ ( B ⋅ q ) ] ⋅ B + r r [q ⊗ p ] ⋅ B + L
−1 −1

( p ⋅ (B ⋅ B) ⋅ q ) ( p ⋅ q )
−T
( p ⋅ q)

L+ r
−1 −1 r r
[
r ( B ⋅ p) ⊗ ( B ⋅ q ) ⋅ B
( p ⋅ (B ⋅ B) ⋅ q )
−T
−1
]
notar que:
{[qr ⊗ ( B ⋅ qr )]⋅ B } = [qr ⊗ ( B ⋅ qr )] B = [q ( B ⋅ qr ) ]B
−1
ij ik
−1
kj i k
−1
kj [ ]
= q i ( B kp q p ) B kj−1 = q i ( B kp B kj−1 q p )
= [q ⊗ ( B ⋅ B ) ⋅ q ) ]
r r −T
ij

[( B ⋅ pr ) ⊗ ( B ⋅ qr )]⋅ B = ( B ⋅ pr ) ⊗ ( B ⋅ B ) ⋅ qr
−1 −1 −1 −T

Si ahora consideramos que el tensor B es simétrico, i.e. B = B T , quedamos con:

Universidad de Castilla- La Mancha Draft Por: Eduardo W. V. Chaves (2012)


Ciudad Real - España
1 TENSORES 75

r r r r
( p ⋅ q + p ⋅ B −1 ⋅ p ) r r
D −1 = B −1 + r r r r
( p ⋅ q) ( p ⋅ q)
[q ⊗ q ] + r− 1r
( p ⋅ q)
[qr ⊗ ( pr ⋅ B )] + ( pr−⋅1qr ) [( B
−1 −1
⋅ p) ⊗ q ]
r r

{[qr ⊗ ( pr ⋅ B )] + [( B }
r r r −1 r
( p ⋅ q + p ⋅ B ⋅ p) r r
[q ⊗ q ] + r− 1r ⋅ p) ⊗ q ]
r r
= B −1 + r r r r
−1 −1
( p ⋅ q) ( p ⋅ q) ( p ⋅ q)
r r r −1 r
( p ⋅ q + p ⋅ B ⋅ p) r r
= B −1 + r r 2
( p ⋅ q)
[q ⊗ q ] − r 2 r
( p ⋅ q)
[qr ⊗ ( B −1
⋅ p)]
r sym

r r r
Notar que, debido la simetría de B se cumple que p ⋅ B −1 = B −1 ⋅ p = t y B −T ⋅ B = 1 .
b.2) Un tensor será no-singular si det (D ) ≠ 0 . Utilizando la expresión obtenida anteriormente:

[ r r r r
D = B ⋅ 1 + βa ⊗ b + γ c ⊗ d ]
⇒ ( [ r r r r
]) r r r r
det (D) = det B ⋅ 1 + βa ⊗ b + γc ⊗ d = det ( B )det 1 + βa ⊗ b + γc ⊗ d [ ]
Como B es un tensor definido positivo esto implica que det ( B ) > 0 . Luego, la condición para
que D sea no-singular es que det 1 + βa ⊗ b + γc ⊗ d [ r r r r
] ≠ 0 . Utilizando el determinante
obtenido en el Ejemplo 1.83:
r r r r r r r r r r
[ r r r r
det (α1 + β a ⊗ b + γ c ⊗ d ) = α 3 + α 2 γ (c ⋅ d ) − αβγ (a ⋅ d )( b ⋅ c ) − (a ⋅ b )( c ⋅ d ) ]
r r r r r r r r r r r r r r r r
donde α = 1 , a ⋅ b = ( B −1 ⋅ p ) ⋅ p = p ⋅ B −1 ⋅ p , a ⋅ d = ( B −1 ⋅ p ) ⋅ ( B ⋅ q ) = p ⋅ q , b ⋅ c = p ⋅ q
r r r r r r r r −1 r r
c ⋅ d = q ⋅ ( B ⋅ q ) = q ⋅ B ⋅ q , γ (c ⋅ d ) = r r q ⋅ B ⋅ q = −1 ,
q ⋅B ⋅q

[r r r r r r r r
]
βγ (a ⋅ d )( b ⋅ c ) − (a ⋅ b )( c ⋅ d ) = r r r
1 −1
[
r r r r r −1 r r r
r ( p ⋅ q )( p ⋅ q ) − ( p ⋅ B ⋅ p )( q ⋅ B ⋅ q )
p⋅q q ⋅B ⋅q
]
Con eso podemos decir que:

[ r r r r
det 1 + βa ⊗ b + γc ⊗ d = r r r ] 1
( p ⋅ q )(q ⋅ B ⋅ q )
r r r r
[ r −1 r r r
r ( p ⋅ q )( p ⋅ q ) − ( p ⋅ B ⋅ p )(q ⋅ B ⋅ q ) ≠ 0 ]
r r r r r r r r
Luego, las condiciones son: p ≠ 0 , q ≠ 0 , ( p ⋅ q ) ≠ 0 , i.e. p y q no pueden ser vectores
ortogonales. Otra condición que hay que cumplir es:
r r r r r r r r
( p ⋅ q )( p ⋅ q ) − ( p ⋅ B −1 ⋅ p )(q ⋅ B ⋅ q ) ≠ 0
142 4 43 4 144424443
>0 >0
r r
Notar que por el hecho que B sea definido positivo, el término (q ⋅ B ⋅ q ) > 0 siempre será
r r r r
positivo cualquier que sea q ≠ 0 . Lo mismo ocurre con ( p ⋅ B −1 ⋅ p ) > 0 , ya que si un tensor es
definido positivo su inversa también lo es. Observar también que para que D sea definido
r r r r r r r r
positivo ( det (D ) > 0 ) hay que cumplir ( p ⋅ q ) 2 > ( p ⋅ B −1 ⋅ p )(q ⋅ B ⋅ q ) y ( p ⋅ q ) > 0 . Estas dos
r r r r r r
condiciones pueden ser reemplazadas por ( p ⋅ q ) > ( p ⋅ B −1 ⋅ p )(q ⋅ B ⋅ q ) .

Ejemplo 1.85
Dado un tensor de segundo orden A = A (τ) y un escalar τ , demostrar que se cumple que:

dA  dA
= A Tr ⋅ A −1  (1.82)
dτ  dτ 

Universidad de Castilla- La Mancha Draft Por: Eduardo W. V. Chaves (2012)


Ciudad Real - España
76 PROBLEMAS RESUELTOS DE MECÁNICA DEL MEDIO CONTINUO

Solución:
En el Ejemplo 1.79 y Ejemplo 1.76, hemos demostrado, respectivamente, que se cumplen:

III A = det ( A ) = A =
1
6
{
[Tr( A )]3 + 2 Tr(A 3 ) − 3Tr( A 2 ) Tr(A ) } (1.83)

III A A −1 = A 2 − AI A + II A 1 (1.84)

donde I A = Tr (A ) , II A =
1
2
{
[Tr ( A )]2 − Tr ( A 2 ) . }
Notar también que las siguientes derivadas se cumplen:
d [I A ] d [Tr ( A )] d [A kk ] d [A ik δ ik ] d [A ik ] dA  dA 
= = = = δ ik = : 1 = Tr  
dτ dτ dτ dτ dτ dτ  dτ 
[
d Tr ( A 2 ) ]
 d (A 2 )  
 = Tr  2A ⋅
dA   dA 

= Tr 
d τ dτ  = 2 Tr  A ⋅ dτ 
     

[
d Tr ( A 3 ) ]
= 3Tr  A 2 ⋅
dA 
dτ  dτ 
Tomando la derivada de (1.83) con respecto a τ obtenemos que:
d ( III A ) 1 d

=
6 dτ
{
[Tr( A )]3 + 2 Tr(A 3 ) − 3Tr( A 2 ) Tr(A ) }
1
= 3[Tr ( A )]
2 d [Tr ( A )]
+2
d Tr ( A 3 )
−3
[
d Tr ( A 2 )] [ ]
Tr ( A ) − 3 Tr ( A 2 )
d [Tr ( A )]

6 dt dτ dτ dτ 
1  dA   2 dA   dA   dA  
3[Tr ( A )] Tr   + 6 Tr  A ⋅ − 6 Tr  A ⋅
2
=   Tr ( A ) − 3Tr ( A 2 ) Tr   
6  dτ   dτ   dτ   dτ  

= Tr  A 2 ⋅

dA 

dτ 
 dA 
− Tr  A ⋅


dτ 
1
2
2
{  dA 
Tr ( A ) + [Tr ( A )] − Tr ( A 2 ) Tr  
 dτ 
}
o aun
d ( III A )  dA   dA   dA 
= Tr  A 2 ⋅  − Tr  A ⋅ dτ  I A + II A Tr  dτ  (1.85)
dτ  d τ     
dA
Haciendo el producto escalar de la ecuación (1.84) con , obtenemos que:

III A A −1 ⋅
dA

(
= A 2 − AI A + II A 1 ⋅
dA

= A2 ⋅)dA

−A⋅
dA

I A + II A
dA

y sacando la traza con obtenemos que:
 dA   2 dA dA dA 
Tr  A −1 ⋅  III A = Tr  A ⋅ −A⋅ I A + II A 
 dτ   dτ dτ dτ 
(1.86)
 dA   dA   dA 
= Tr  A 2 ⋅  − Tr  A ⋅  + Tr   II A
 dτ   dτ   dτ 
Si comparamos las ecuaciones (1.85) y (1.86) concluimos que:
 dA   dA
⋅ A −1 
d ( III A )
= III A Tr  A −1 ⋅  = III A Tr 
dτ  d τ   d τ 

Universidad de Castilla- La Mancha Draft Por: Eduardo W. V. Chaves (2012)


Ciudad Real - España
1 TENSORES 77

1.1.11 Tensores Isótropos y Anisótropos

Ejemplo 1.86
Sea el tensor de cuarto orden C , cuyas componentes vienen dados por:
C ijkl = λδ ij δ kl + µδ ik δ jl + γδ il δ jk (1.87)
a) ¿Qué tipo de simetría presenta el tensor C ? b) Que condiciones hay que cumplir para que
C sea un tensor simétrico?
Solución:
El tensor presenta simetría mayor si se cumple que C ijkl = C klij . Teniendo en cuenta (1.87),
concluimos que:
C klij = λδ kl δ ij + µδ ki δ lj + γδ kj δ li = C ijkl

Verificamos ahora si el tensor presenta simetría menor, por ejemplo C ijkl = C ijlk
C ijlk = λδ ij δ lk + µδ il δ jk + γδ ik δ jl ≠ C ijkl

Se puede comprobar este hecho fácilmente por adoptar i = 2 , j = 1 , k = 1 , l = 2 , con eso:


C ijkl = C 2112 = λδ 21δ 12 + µδ 21δ 12 + γδ 22 δ 11 = γ

C ijlk = C 2121 = λδ 21δ 21 + µδ 22 δ 11 + γδ 21δ 12 = µ

Luego, el tensor C solo será simétrico (simetría menor y mayor) si µ = γ , resultando:


C ijkl = λδ ij δ kl + µ(δ ik δ jl + δ il δ jk )

Verifiquemos que δ ij δ kl presenta simetría mayor y menor, mientras que los tensores δ ik δ jl ,
δ il δ jk no son simétricos. Fijemos también que (δ ik δ jl + δ il δ jk ) = 2I ijkl
sym
.

Ejemplo 1.87
Sea el tensor de cuarto orden C , cuyas componentes vienen dadas por:
C ijkl = λδ ij δ kl + µ(δ ik δ jl + δ il δ jk ) (1.88)
donde λ y µ son constantes. Demostrar que C es un tensor isótropo.
Solución: Un tensor de cuarto orden será isótropo si se cumple que C ′ijkl = C ijkl , donde C′ijkl
son las componentes del tensor debido a transformación de coordenadas. Teniendo en cuenta
que la ley de transformación de las componentes de un tensor de cuarto orden viene dada por:
C ′ijkl = a im a jn a kp a lq C mnpq (1.89)

donde aij es la matriz de transformación de base, luego:

Universidad de Castilla- La Mancha Draft Por: Eduardo W. V. Chaves (2012)


Ciudad Real - España
78 PROBLEMAS RESUELTOS DE MECÁNICA DEL MEDIO CONTINUO

[
C ′ijkl = a im a jn a kp a lq λδ mn δ pq + µ (δ mp δ nq + δ mq δ np ) ]
= λa im a jn a kp a lq δ mn δ pq + µ(a im a jn a kp a lq δ mp δ nq + a im a jn a kp a lq δ mq δ np ) (1.90)
= λa in a jn a kq a lq + µ(a ip a jq a kp a lq + a iq a jp a kp a lq )

Observemos que a ik a jk = δ ij , o en notación matricial AA T = 1 , ya que la matriz de


transformación es una matriz ortogonal, por lo que tenemos:
C ′ijkl = λδ ij δ kl + µ(δ ik δ jl + δ il δ jk ) = C ijkl (1.91)

luego C es isótropo.

Ejemplo 1.88
Sea C un tensor de cuarto orden simétrico e isótropo representado por:
(
C ijkl = λδ ij δ kl + µ δ ik δ jl + δ il δ jk ) (notación indicial)
C = λ1 ⊗ 1 + 2µI (notación tensorial)
donde λ , µ son escalares constantes, 1 es el tensor identidad de segundo orden, I es la
parte simétrica del tensor identidad de cuarto orden, es decir, I ≡ I sym .
Se pide:
a) Dado un tensor de segundo orden simétrico ε , obtener σ que viene dado por la
siguiente operación σ = C : ε . Expresar el resultado en notación tensorial e indicial.
b) Demostrar que σ y ε presentan los mismos autovectores (mismas direcciones
principales).
c) Si γ σ son los autovalores (valores principales) del tensor σ , obtener también los
autovalores del tensor ε .

Solución: a)
Notación tensorial: Notación indicial
σ ij = C ijkl ε kl
σ = C :ε [ ( )]
= λδ ij δ kl + µ δ ik δ jl + δ il δ jk ε kl
= (λ1 ⊗ 1 + 2µI ) : ε (
= λδ ij δ kl ε kl + µ δ ik δ jl ε kl + δ il δ jk ε kl )
= λ1 ⊗ 1
{ : ε + 2µI{
Tr (ε )

ε sym
(
= λδ ij ε kk + µ ε ij + ε ji )
= λTr (ε )1 + 2µε ( )
= λδ ij ε kk + 2µ ε ijsym
= λδ ij ε kk + 2µε ij

donde hemos considerado la simetría del tensor ε = ε T .


b) y c) Partiendo de la definición de autovalor y autovector del tensor σ :
σ ⋅ nˆ = γ σ nˆ
Reemplazando el valor de σ obtenido anteriormente podemos decir que:

Universidad de Castilla- La Mancha Draft Por: Eduardo W. V. Chaves (2012)


Ciudad Real - España
1 TENSORES 79

(λTr(ε)1 + 2µε ) ⋅ nˆ = γ σ nˆ
⇒ λTr (ε )1 ⋅ nˆ + 2µε ⋅ nˆ = γ σ nˆ
⇒ λTr (ε )nˆ + 2µε ⋅ nˆ = γ σ nˆ
⇒ 2µε ⋅ nˆ = γ σ nˆ − λTr (ε )nˆ = (γ σ − λTr (ε ) )nˆ
 γ − λTr (ε ) 
⇒ ε ⋅ nˆ =  σ nˆ
 2µ 
⇒ ε ⋅ nˆ = γ ε nˆ
Con lo cual concluimos que σ y ε presentan los mismos autovectores (son coaxiales). Y
además los autovalores de ε se pueden obtener como:
γ σ − λTr (ε )
γε =

Si denominamos que γ ε(1) = ε1 , γ ε( 2) = ε 2 , γ (ε3) = ε 3 y γ σ(1) = σ1 , γ σ( 2) = σ 2 , γ (σ3) = σ 3 . La forma


explícita de la relación anterior viene dada por:
ε 1 0 0 σ1 0 0  1 0 0
0 ε2 0=  1 
0 σ2 0  − λTr (ε ) 0 1 0
 2µ   2µ  
 0 0 ε 3   0 0 σ 3  0 0 1

σ1 0 0  1 0 0 ε 1 0 0
donde también se cumple que:  0 σ2 0  = λTr (ε ) 0 1 0 + 2µ  0
    ε2 0 
 0 0 σ 3  0 0 1  0 0 ε 3 

Ejemplo 1.89
a) Obtener la inversa del tensor de cuarto orden C = 2µI + λ1 ⊗ 1 donde I ≡ I sym es el tensor
identidad simétrico de cuarto orden, 1 es el tensor identidad de segundo orden, y µ > 0 , λ
son escalares.

b) Obtener el determinante de C . Además si consideramos que λ = ,
(1 + ν)(1 − 2ν)
E
µ= , que valores deben asumir E y ν para que el tensor sea definido positivo.
2(1 + ν)
c) Obtener la expresión inversa de σ = C : ε en función de µ > 0 , λ , donde σ y ε son
tensores simétricos de segundo orden.
Solución:
a) Recurrimos a la ecuación obtenida en (1.71):

(α D + β A ⊗ B ) −1 =
1
α
D −1 −
β
α (α + β B : D −1 : A )
[(D −1
: A ) ⊗ (B : D −1 ) ]
Haciendo que D = I , A = B = 1 , α = 2µ , β = λ , obtenemos que:

C −1 = ( 2µI + λ1 ⊗ 1) −1 =
1 −1

I −
λ
−1
2 µ ( 2 µ + λ 1 : I : 1)
[
(I −1 : 1) ⊗ (1 : I −1 ) ]

Universidad de Castilla- La Mancha Draft Por: Eduardo W. V. Chaves (2012)


Ciudad Real - España
80 PROBLEMAS RESUELTOS DE MECÁNICA DEL MEDIO CONTINUO

Recordar que se cumple que I −1 = I , (I −1 : 1) = I : 1 = 1 . A continuación obtenemos el valor


del escalar 1 : I −1 : 1 = 1 : I : 1 = 1 : 1 = Tr (1) = 3 . También lo expresamos en notación indicial:
1
1 : I −1 : 1 = 1 : I : 1 = δ ij I ijkl
sym
δ kl = δ ij (δ ik δ jl + δ il δ jk )δ kl
2
1
= (δ ij δ ik δ jl δ kl + δ ij δ il δ jk δ kl )
2
1
= (δ jj + δ jj ) = 3
2
Resultando que:
1 λ
C −1 = ( 2µI + λ1 ⊗ 1) −1 = I− (1 ⊗ 1)
2µ 2µ ( 2µ + 3λ )

Verifiquemos si se cumple que C : C −1 = I sym ≡ I :


 1 λ 
C : C −1 = ( 2µI + λ1 ⊗ 1) :  I− (1 ⊗ 1) 
 2µ 2µ ( 2µ + 3λ ) 
2µ 2µ λ λ λ2
C : C −1 = ( I:I − I : (1 ⊗ 1) + (1 ⊗ 1) : I − (1 ⊗ 1) : (1 ⊗ 1)
2µ 2µ ( 2µ + 3λ ) 2µ 2µ ( 2µ + 3λ )
Según el Ejemplo 1.26 se cumplen que I : I = I , I : (1 ⊗ 1) = (1 ⊗ 1) : I = 1 ⊗ 1 , y
(1 ⊗ 1) : (1 ⊗ 1) = 3(1 ⊗ 1) . Con eso obtenemos que:

 − 2µ λ λ 3λ2 
C : C −1 = I +  + − (1 ⊗ 1) = I

2 µ ( 2 µ + 3λ
1444444424444444) 2 µ 2 µ ( 2 µ + 3λ )
3
=0

b) Podemos utilizar directamente la expresión (1.36) del Ejemplo 1.50


( )
det αI sym + βA ⊗ B = α 3 + α 2 β A : B

Haciendo α = 2µ , β = λ , A = B = 1 concluimos que:


det (2µI + λ1 ⊗ 1) = (2µ) 3 + (2µ) 2 λ 1 : 1 = ( 2µ) 3 + (2µ) 2 λ 3 = (2µ) 2 (2µ + 3λ)

Para que C sea positivo definido, los autovalores tienen que ser positivos, i.e.:
E
µ>0⇒µ= >0
2(1 + ν)
E Eν E
2µ + 3λ > 0 ⇒ 2 +3 = >0
2(1 + ν ) (1 + ν )(1 − 2ν ) (1 − 2ν )
Denotando por y1 = (1 + ν) ≠ 0 , y 2 = (1 − 2ν ) ≠ 0 , podemos concluir que:

Universidad de Castilla- La Mancha Draft Por: Eduardo W. V. Chaves (2012)


Ciudad Real - España
1 TENSORES 81

 E > 0

E E  y1 > 0
µ= = >0 ⇒ 
2(1 + ν ) 2 y1  E < 0
 y < 0
 1
 E > 0

E E  y 2 > 0
2µ + 3λ = = >0 ⇒ 
(1 − 2ν ) y2  E < 0
 y < 0
 2
Las condiciones anteriores tienen que cumplir simultáneamente. Luego, a través de la gráfica
abajo, podemos concluir que E > 0 y − 1 < ν < 0,5 .

y (ν)
y2 = (1 − 2ν ) ≠ 0

y1 = (1 + ν) ≠ 0

zona no factible
zona no factible

1
ν ≠ −1

( y 2 > 0 ⇒ E > 0) E >0 ( y 1 > 0 ⇒ E > 0)

ν
( y 1 < 0 ⇒ E < 0) ( y 2 < 0 ⇒ E < 0)
ν ≠ 0,5

c)
σ = C:ε ⇒ C −1 : σ = C −1 : C : ε ⇒ C −1 : σ = I sym : ε = ε sym = ε
⇒ ε = C −1 : σ
 1 λ  1 λ
⇒ε= I− 1 ⊗ 1 : σ = I:σ − 1⊗1:σ
 2µ 2µ ( 2µ + 3λ )  2µ 2µ ( 2µ + 3λ )
1 λ
⇒ε= σ− Tr (σ )1
2µ 2µ ( 2µ + 3λ )
Ver Figura 1.3. Es interesante comparar esta última expresión con el Ejemplo 1.40.

Universidad de Castilla- La Mancha Draft Por: Eduardo W. V. Chaves (2012)


Ciudad Real - España
82 PROBLEMAS RESUELTOS DE MECÁNICA DEL MEDIO CONTINUO

x3 C x3
ε 33 σ 33
σ = C:ε

ε13 σ13
ε 23 ε 32 σ 23 σ 32
ε 31 ε 22 σ 31 σ 22

ε12 σ12
ε 21 x2 σ 21 x2
ε11 σ11

x1 ε = C −1 : σ
x1

C −1

Figura 1.3

Ejemplo 1.90
ˆ ) un tensor de segundo orden, denominado de tensor acústico elástico, y definido
Sea Q e (N
como:
Q e (N ˆ ⋅ Ce ⋅N
ˆ) =N ˆ

donde C e es un tensor de cuarto orden simétrico e isótropo dado por C e = λ(1 ⊗ 1) + 2µI ,
cuyas componentes son: C ijkle
= λδ ij δ kl + µ(δ ik δ jl + δ il δ jk ) . Obtener las componentes del
tensor acústico en función de las constantes λ , µ .
Solución:
Utilizando notación simbólica obtenemos que:
Q e (N ˆ ⋅ Ce ⋅N
ˆ) =N ˆ = Nˆ eˆ
i i ( )⋅ (C e ˆ
pqrs e p ⊗ eˆ q ⊗ eˆ r ⊗ eˆ s )⋅ (Nˆ eˆ )
j j

(
= Nˆ i C epqrs Nˆ j δ ip δ sj eˆ q ⊗ eˆ r = N)
ˆ C e Nˆ eˆ ⊗ eˆ
p pqrs s q r ( )
ˆ ) son:
Luego, las componentes de Q e (N
Q e qr = Nˆ p C epqrs Nˆ s
[
= Nˆ p λδ pq δ rs + µ(δ pr δ qs + δ ps δ qr ) N
ˆ
s ]
= λδ pq δ rs Nˆ p Nˆ s + µ (Nˆ p δ pr δ qs Nˆ s + Nˆ p δ ps δ qr Nˆ s )
= λNˆ q N
ˆ + µ(Nˆ Nˆ + Nˆ δ Nˆ )
r r q s qr s

Ya que N̂ es un versor se cumple que N s N s = 1 , resultando que:

Q e qr = µδ qr + (λ + µ )Nˆ q Nˆ r ; ˆ ) = µ1 + (λ + µ )N
Q e (N ˆ ⊗N
ˆ

Universidad de Castilla- La Mancha Draft Por: Eduardo W. V. Chaves (2012)


Ciudad Real - España
1 TENSORES 83

Ejemplo 1.91
Sea Q un tensor de segundo orden simétrico y dado por:
ˆ ) = µ1 + (λ + µ )N
Q (N ˆ ⊗N
ˆ

donde λ , µ son constante y N̂ es un versor.


ˆ ) y determinar las restricciones de λ e µ para que exista la
a) Obtener los autovalores de Q (N
ˆ ) , i.e. ∃ Q −1 .
inversa de Q (N
Eν E
b) Teniendo en cuenta que λ = , µ= , determinar los valores posibles de
(1 + ν)(1 − 2ν) 2(1 + ν)
ˆ ) sea un tensor definido positivo.
( E , ν ) para que Q (N
ˆ).
c) Obtener la inversa de Q (N
Solución:
r r
a) Fue demostrado en el Ejemplo 1.49 que, dados dos vectores a y b se cumple que:
( r r
) r r
det β1 + αa ⊗ b = β 3 + β 2 αa ⋅ b
Los autovalores se obtienen al resolver el determinante característico det (Q − γ1) = 0 , donde
γ i son los autovalores de Q . Luego:

(
det µ1 + (λ + µ)N ˆ ⊗N ˆ − γ1 = 0 )
(
det (µ − γ )1 + (λ + µ)Nˆ ⊗Nˆ =0 )
Haciendo β = (µ − γ ) y α = (λ + µ) concluimos que:

(
det (µ − γ )1 + (λ + µ )N ˆ ⊗Nˆ =0 )
ˆ ⋅N
(µ − γ ) 3 + (µ − γ ) 2 (λ + µ)N ˆ =0
{
=1

(µ − γ ) [(µ − γ ) + (λ + µ)] = 0
2

(µ − γ ) 2 [(λ + 2µ) − γ ] = 0
La ecuación característica anterior, ecuación cúbica en γ , tiene las siguientes soluciones:
 γ 1 = µ
(µ − γ ) = 0 ⇒ 
2
(µ − γ ) [(λ + 2µ) − γ ] = 0
2 solución
 
→  γ 2 = µ
[(λ + 2µ) − γ ] = 0 ⇒ γ = (λ + 2µ)
 3

En el espacio principal de Q , las componentes de Q vienen dadas por:


µ 0 0 

Qij′ =  0 µ 0 
 0 0 (λ + 2µ )

Para que haya la inversa de Q , el determinante de Q tiene que ser distinto de cero:
µ ≠ 0
Q = µ 2 × (λ + 2µ) ≠ 0 ⇒ 
λ + 2µ ≠ 0 ⇒ λ ≠ −2µ

Universidad de Castilla- La Mancha Draft Por: Eduardo W. V. Chaves (2012)


Ciudad Real - España
84 PROBLEMAS RESUELTOS DE MECÁNICA DEL MEDIO CONTINUO

b) Un tensor será definido positivo si sus autovalores son mayores que cero, luego:
 E
µ = 2(1 + ν) > 0


λ + 2µ = Eν E E (1 − ν)
+2 = >0
 (1 + ν)(1 − 2ν) 2(1 + ν) (−2ν 2 − ν + 1)

(1 + ν) ≠ 0 ⇒ ν ≠ −1

Verifiquemos que  2 ν ≠ −1
(−2ν − ν + 1) ≠ 0 ⇒ ν ≠ 0,5
 

Denotando por y1 = (1 + ν) ≠ 0 , y 2 = (1 − ν) ≠ 0 , y 3 = ( −2ν 2 − ν + 1) ≠ 0 , podemos reescribir


las condiciones como:
  E > 0
 
 E  y1 > 0
µ = 2 y > 0 ⇒  E < 0
 1 
  y < 0
 1

   y 2 , y3 > 0
 E > 0 ⇒ 
λ + 2µ = Ey 2 > 0 ⇒   y 2 , y3 < 0
 y3   y 2 > 0, y 3 < 0
  E <0⇒
   y 2 < 0, y 3 > 0
Resumiendo:
E > 0 ⇒ ν ⊂ ]− 1 ; 0,5[ ∪ ] 1 ; ∞ [

E < 0 ⇒ ν ⊂ ]− ∞ ; − 1[

y 2 = (1 − ν) ≠ 0 y (ν)

y1 = (1 + ν) ≠ 0
zona no factible

1 ν =1
ν = −1

E<0 E >0 E >0


ν

ν = 0,5

y 3 = ( −2ν 2 − ν + 1) ≠ 0

Universidad de Castilla- La Mancha Draft Por: Eduardo W. V. Chaves (2012)


Ciudad Real - España
1 TENSORES 85

ˆ ) en el espacio principal de Q (N
c) Las componentes de la inversa de Q (N ˆ ) viene dadas por:

1 
 0 0 
µ 0 0  µ 
 1 1
Qij′ =  0 µ 0  inversa
→ Qij′ −1 =0 0  ∴ Q −1 =
 µ  µ ( λ + 2µ )
2
 0 0 (λ + 2µ)  
0 1 
0
 (λ + 2µ) 

ˆ ) −1 son Q ′ −1 = Q ′ −1 = 1 1
Los valores principales de Q (N 1 2 , Q3′ −1 = . Recordar que un
µ ( λ + 2µ )
tensor y su inversa comparten el mismo espacio principal, i.e. son tensores coaxiales. Además
ˆ ) −1 como:
podemos hacer la representación espectral de Q (N
3
Q −1 = ∑Q
a =1
−1 ˆ ( a )
a N
ˆ (a)
⊗N

= Q1−1Nˆ (1) ⊗ N
ˆ (1) + Q −1N ˆ (2) ⊗ N ˆ ( 2 ) + Q −1Nˆ ( 3) ⊗ N ˆ ( 3)
2 3
ˆ (1) ⊗ N
= Q −1 (N ˆ (1) + N
ˆ ( 2) ⊗ N ˆ ( 2 ) ) + Q −1Nˆ ( 3) ⊗ N ˆ ( 3)
1 3
ˆ ( 3) ⊗ N
= Q1−1 (1 − N ˆ (3) ) + Q −1N
ˆ (3) ⊗ N
ˆ ( 3)
3
ˆ ⊗N
= Q −1 (1 − N ˆ ) + Q −1N ˆ ⊗N ˆ
1 3

ˆ ( 3) = N
donde hemos considerado que N ˆ . Es interesante ver el Ejemplo 1.74. Luego:

ˆ ⊗N
Q −1 = Q1−1 (1 − N ˆ ) + Q −1N
ˆ ⊗N
ˆ
3

1 ˆ ⊗N
ˆ)+ 1 ˆ ⊗N
ˆ
= (1 − N N
µ ( λ + 2µ )
1 1 ˆ ˆ+ 1 ˆ ⊗N
ˆ
= 1− N⊗N N
µ µ ( λ + 2µ )
1 1 1 ˆ ˆ
= 1 −  −  N ⊗ N
µ  µ ( λ + 2 µ ) 
1  λ+µ  ˆ ˆ
= 1 −   N ⊗ N
µ  µ ( λ + 2 µ ) 

Notar que (
Q −1 = N ˆ −1 ≠ N
ˆ ⋅ Ce ⋅N )
ˆ ⋅ C e −1 ⋅ N
ˆ, donde Ce
−1
=
1

I−
λ
2µ( 2µ + 3λ )
(1 ⊗ 1) .
−1
ˆ ⋅ Ce ⋅N
Calculamos Qinv = N ˆ:

Universidad de Castilla- La Mancha Draft Por: Eduardo W. V. Chaves (2012)


Ciudad Real - España
86 PROBLEMAS RESUELTOS DE MECÁNICA DEL MEDIO CONTINUO

ˆ ⋅ C −1 ⋅ N
Qinv = N ˆ
 1 1 λ 
⇒ (Qinv ) jk = Nˆ i C ijkl
−1 ˆ
N l = Nˆ i  (δ ik δ jl + δ il δ jk ) − δ ij δ kl Nˆ l
 2µ 2 2µ( 2µ + 3λ ) 
1 1 ˆ λ
⇒ (Qinv ) jk = (N i δ ik δ jl Nˆ l + Nˆ i δ il δ jk Nˆ l ) − ˆ δ δ Nˆ
N i ij kl l
2µ 2 2µ(2µ + 3λ )
1 ˆ ˆ λ
⇒ (Qinv ) jk = (N k N j + Nˆ l Nˆ l δ jk ) − Nˆ j Nˆ k
4µ 2µ(2µ + 3λ )
1  1 λ ˆ ˆ
⇒ (Qinv ) jk = δ jk +  − N j N k
4µ  4µ 2µ(2µ + 3λ ) 
1  2µ + λ ˆ ˆ
⇒ (Qinv ) jk = δ jk +  N j N k
4µ  4µ(2µ + 3λ ) 
Luego:
1  2µ + λ ˆ ˆ
Qinv = 1 +   N ⊗ N
4µ  4µ(2µ + 3λ ) 
Notar que µ ≠ 0 e (2µ + 3λ ) ≠ 0 y que además son las mismas condiciones para que ∃C −1
(ver Ejemplo 1.89).

Universidad de Castilla- La Mancha Draft Por: Eduardo W. V. Chaves (2012)


Ciudad Real - España
1 TENSORES 87

1.1.12 Descomposición Polar

Ejemplo 1.92
Considérese un tensor F que tiene inversa ( det ( F ) ≠ 0 ), y que puede ser descompuesto
como:
F = Q ⋅U = V ⋅Q
ˆ ( a ) , y V tiene los autovalores µ
Si U tiene los autovalores λ a asociados a los autovectores N a
ˆ (a )
asociados a los autovectores n , probar que:
µa = λa
ˆ ( a ) y nˆ ( a ) .
Obtener también la relación que hay entre los autovectores N
Solución:
Partiendo de la definición de autovalor, autovector del tensor U :
U⋅N
ˆ (a) = λ N
a
ˆ (a) (aquí el índice no indica suma)
Por la definición de F podemos obtener las siguientes relaciones:
QT ⋅ F = QT ⋅ Q ⋅ U = QT ⋅ V ⋅ Q
⇒ QT ⋅ F = U = QT ⋅ V ⋅ Q
Reemplazando en la definición de autovalor, autovector:
U⋅N ˆ (a ) = λ N
a
ˆ (a)

QT ⋅ V ⋅ Q ⋅ N
ˆ (a ) = λ N
a
ˆ (a)

Q ⋅ QT ⋅ V ⋅ Q ⋅ N
ˆ (a ) = λ Q ⋅ N
a
ˆ (a)
123
1

Resultando:
V ⋅ Q ⋅N
ˆ (a) = λ Q ⋅ N
a
ˆ (a)

V ⋅ nˆ ( a ) = λ a nˆ ( a )

donde hemos considerado que nˆ ( a ) = Q ⋅ N


ˆ ( a ) . Además comparando las dos definiciones de
autovalor y autovector de los tensores U y V podemos comprobar que tienen los mismos
autovalores y distintos autovectores y que están relacionados por la transformación ortogonal
nˆ ( a ) = Q ⋅ N
ˆ (a) .

1.1.13 Tensor Esférico y Desviador

Ejemplo 1.93
Considérese un tensor de segundo orden simétrico σ y su parte desviadora s ≡ σ dev .
∂s
a) Obtener el resultado de la operación s : .
∂σ

Universidad de Castilla- La Mancha Draft Por: Eduardo W. V. Chaves (2012)


Ciudad Real - España
88 PROBLEMAS RESUELTOS DE MECÁNICA DEL MEDIO CONTINUO

b) Demostrar también que los tensores σ y σ dev son tensores coaxiales.


Solución:
a) Teniendo la definición de un tensor desviador σ = σ esf + σ dev = σ esf + s . Obtenemos que:

s=σ− 1 . Luego:
3
 I 
∂ σ − σ 1 
∂s
= 
3  = ∂[σ ] − 1 ∂[I σ ] 1
∂σ ∂σ ∂σ 3 ∂σ
En notación indicial:
∂s ij ∂σ ij 1 ∂ [I σ ] 1
= − δ ij = δ ik δ jl − δ kl δ ij
∂σ kl ∂σ kl 3 ∂σ kl 3
Con lo cual
∂s ij  1  1 1
s ij = s ij  δ ik δ jl − δ kl δ ij  = s ij δ ik δ jl − s ij δ kl δ ij = s kl − δ kl s ii = s kl
∂σ kl  3  3 3 {
=0

∂s
s: =s
∂σ
b) Para demostrar que dos tensores son coaxiales, hay que cumplir que:
σ dev ⋅ σ = σ ⋅ σ dev

= σ ⋅ (σ − σ esf ) = σ ⋅ σ − σ ⋅ σ esf = σ ⋅ σ − σ ⋅ 1
3
Iσ I
= σ ⋅σ − σ ⋅ 1 = σ ⋅σ − σ 1⋅σ
3 3
 I 
=  σ − σ 1  ⋅ σ = σ dev ⋅ σ
 3 

Con lo cual demostramos que los tensores σ y σ dev son coaxiales, es decir, tienen las mismas
direcciones principales.

1.1.14 Otros

Ejemplo 1.94
1 1
Considere J = [det (b )] 2 = ( III b ) 2 , donde b es un tensor de segundo orden simétrico, b = b T .
Obtener la derivada de J y de Ln(J ) con respecto a b .
Solución:

Universidad de Castilla- La Mancha Draft Por: Eduardo W. V. Chaves (2012)


Ciudad Real - España
1 TENSORES 89

 1

∂ ( III b ) 2 
∂J
⇒ =  
∂b ∂b
1 1 ∂ III 1 1
= ( III b )− 2 b
= ( III b )− 2 III b b −T
2 ∂b 2
1 1 1
= ( III b ) 2 b −1 = J b −1
2 2
  1 

∂ Ln III b 2  
∂[Ln(J )]   1 ∂ III b 1 −1
⇒ =  = = b
∂b ∂b 2 III b ∂b 2

1.1.15 Notación de Voigt

Ejemplo 1.95
a) Escribir la relación σ = C : ε en notación de Voigt, donde C = λ1 ⊗ 1 + 2µI es un tensor
de cuarto orden, y los tensores simétricos σ y ε están estructurados según notación de Voigt
como:
 σ11   ε11 
σ  ε 
 22   22 
σ  ε 
{σ} =  33  ; {ε} =  33 
 σ12   2ε 12 
σ 23  2ε 23 
   
 σ13   2ε 13 
b) Escribir la ecuación ε = C −1 : σ en notación de Voigt, donde el tensor C −1 (ver Ejemplo
1.89) viene dado por:
1 λ
C −1 = I− 1 ⊗1
2µ 2µ ( 2µ + 3λ )
Solución:
Podemos escribir la relación σ = (λ1 ⊗ 1 + 2µI ) : ε en notación indicial:
 1
( )

[ ( )]
σ ij = λδ ij δ kl + 2µ δ ik δ jl + δ il δ jk  ε kl = λδ ij δ kl + µ δ ik δ jl + δ il δ jk ε kl
2
 
El tensor identidad de segundo orden en notación de Voigt queda:
1
1
1 0 0   
1
δ ij = 0 1 0 Voigt
 →{δ} =  
0 0 1 0 
0 
 
0

Universidad de Castilla- La Mancha Draft Por: Eduardo W. V. Chaves (2012)


Ciudad Real - España
90 PROBLEMAS RESUELTOS DE MECÁNICA DEL MEDIO CONTINUO

Luego el término (1 ⊗ 1)ij = δ ij δ kl en notación de Voigt queda:

1 1 1 1 0 0 0
1 1 1 1 0 0 0
  
I ijkl [] 1  1
→ I =  [1 1 1 0 0 0] = 
= δ ij δ kl Voigt

1 1 0 0 0
 = {δ}{δ}
T

0  0 0 0 0 0 0
0  0 0 0 0 0 0
   
0 0 0 0 0 0 0

El tensor identidad de cuarto orden, I ijkl =


1
2
( )
δ ik δ jl + δ il δ jk , en notación de Voigt:

 I1111 I1122 I1133 I1112 I1123 I1113  1 0 0 0 0 0


I  1 0 0 0 0 
 2211 I 2222 I 2233 I 2212 I 2223 I 2213  0
I 3311 I 3322 I 3333 I 3312 I 3323 I 3313  0 0 1 0 0 0
I ijkl →[I ] = 
Voigt = 
 I1211 I1222 I1233 I1212 I1223 I1213  0 0 0 12 0 0 
I 2311 I 2322 I 2333 I 2312 I 2323 I 2313  0 0 0 0 12 0 
   
 I1311 I1322 I1333 I1312 I1323 I1313  0 0 0 0 0 12 

Con eso podemos decir que C = λ1 ⊗ 1 + 2µI y en notación de Voigt queda:


1 1 1 0 0 0 1 0 0 0 0 0  λ + 2µ λ λ 0 0 0
1  0  
 1 1 0 0 0  1 0 0 0 0
 λ λ + 2µ λ 0 0 0 
1 1 1 0 0 0 0 0 1 0 0 0  λ λ λ + 2µ 0 0 0 
[C ] = λ   + 2µ  = 
0 0 0 0 0 0 0
1
0 0 2 0 0  0 0 0 µ 0 0
0 0 0 0 0 0 0 0 0 0 12 0   0 0 0 0 µ 0
     
0 0 0 0 0 0 0
1
0 0 0 0 2   0 0 0 0 0 µ 

Luego:
 σ11  λ + 2µ λ λ 0 0 0   ε 11 
σ   λ λ + 2µ λ 0 0 0   ε 22 
 22  
 σ 33   λ λ λ + 2µ 0 0 0   ε 33 
σ = (λ1 ⊗ 1 + 2µI ) : ε Voigt
 →  =  
 σ12   0 0 0 µ 0 0   2ε12 
σ 23   0 0 0 0 µ 0   2ε 23 
    
 σ13   0 0 0 0 0 µ   2ε13 
144444444444244444444444 3

{σ } = [C ]{ε }
b)
ε = C −1 : σ
 1 λ  1 λ
⇒ε= I− 1 ⊗ 1 : σ = I:σ − 1 ⊗1:σ
 2µ 2µ ( 2µ + 3λ )  2µ 2µ ( 2µ + 3λ )
1 λ
⇒ε= σ− Tr (σ )1
2µ 2µ ( 2µ + 3λ )
1 λ
⇒ ε ij = σ ij − σ kk δ ij
2µ 2µ ( 2µ + 3λ )

Universidad de Castilla- La Mancha Draft Por: Eduardo W. V. Chaves (2012)


Ciudad Real - España
1 TENSORES 91

Notar que:
1 λ  µ+λ  λ
ε 11 = σ11 − (σ 11 + σ 22 + σ 33 )δ 11 =  σ 11 − (σ 22 + σ 33 )
2µ 2µ ( 2µ + 3λ )  µ ( 2µ + 3λ )  2µ ( 2µ + 3λ )
1 λ  µ+λ  λ
ε 22 = σ 22 − (σ 11 + σ 22 + σ 33 )δ 22 =  σ 22 − (σ 11 + σ 33 )
2µ 2µ ( 2µ + 3λ )  µ ( 2µ + 3λ )  2µ ( 2µ + 3λ )
1 λ  µ+λ  λ
ε 33 = σ 33 − (σ 11 + σ 22 + σ 33 )δ 33 =  σ 33 − (σ11 + σ 22 )
2µ 2µ ( 2µ + 3λ )  µ ( 2µ + 3λ )  2µ ( 2µ + 3λ )
1 λ 1 1
ε 12 = σ 12 − (σ kk )δ{12 = σ 12 ⇒ 2ε 12 = σ 12
2µ 2µ ( 2µ + 3λ ) =0
2µ µ
1
2ε 23 = σ 23
µ
1
2ε 13 = σ
µ 13
Reestructurando las ecuaciones anteriores en notación de Voigt, obtenemos:
 µ+λ λ λ 
 µ ( 2µ + 3λ ) − − 0 0 0
2µ ( 2µ + 3λ ) 2µ ( 2µ + 3λ )
 
 λ µ+λ λ
 11   −
ε − 0 0 0  σ 11 

   2µ ( 2µ + 3λ ) µ ( 2µ + 3λ ) 2µ ( 2µ + 3λ )
 σ 22 
 ε 22   λ λ µ+λ  
ε 33   − 2µ ( 2µ + 3λ ) − 0 0 0 
2µ ( 2µ + 3λ ) µ ( 2µ + 3λ )  σ 33 
 = 1  σ 
 2ε 12   0 0 0 0 0   12 
 2ε 23   µ  σ 23 
  1  
 2ε 13   0 0 0 0 0  σ 13 
 µ
 
 1
0 0 0 0 0
 µ 
{ε } = [C ] −1 {σ }

Ejemplo 1.96
r
Considérese T ( x , t ) un tensor simétrico de segundo orden, el cual es función de la posición
r
( x ) y del tiempo (t ) . Considérese también que las componentes del tensor según la dirección
x3 son iguales a cero, i.e. T13 = T23 = T33 = 0 .
r r
NOTA: Definimos T ( x , t ) como un campo tensorial, i.e. el valor de T ( x , t ) depende de la
posición y del tiempo. Si el tensor es independiente de una dirección para todo el dominio
r r
( x ) , e.g. si T ( x , t ) es independiente de la dirección x3 , ver Figura 1.4, el problema puede ser
considerado como bidimensional (estado plano) simplificando bastante las ecuaciones. ■

Universidad de Castilla- La Mancha Draft Por: Eduardo W. V. Chaves (2012)


Ciudad Real - España
92 PROBLEMAS RESUELTOS DE MECÁNICA DEL MEDIO CONTINUO

2D
x2 T T12 
Tij =  11
 T12 T22 
x2
T22
T12 T22

T12 T12
T11 T11
T11
x1
T12
x1
T22
x3

Figura 1.4: Problema bidimensional (2D).

Obtener las componentes T11′ , T22


′ , T12′ , tras un cambio de base en el plano x1 − x 2 tal como
se indica en la figura abajo

 a11 a12 0  cos θ sin θ 0


x2
a ij =  a 21 a 22 0 = − sin θ cos θ 0
x 2′ x1′  0 0 1  0 0 1

θ
x1

Obtener también el valor de θ correspondiente a las dirección principales de T .


OBS.: Utilizar notación de Voigt, y expresar los resultados en función de 2θ .
Solución:

Podemos utilizar directamente la ley de transformación en la notación de Voigt


{T ′} = [M] {T } , donde
 T11′   T11 
T′  T 
 22   22 
T′  T 
{T ′} =  33′  ; {T } =  33 
 T12   T12 
 T23
′   T23 
   
 T13′   T13 

Universidad de Castilla- La Mancha Draft Por: Eduardo W. V. Chaves (2012)


Ciudad Real - España
1 TENSORES 93

 a11 2 a12 2 a13 2 2a11 a12 2a12 a13 2a11 a13 


 2 
 a 21 a 22 2 a 23 2 2a 21 a 22 2a 22 a 23 2a 21 a 23 
 2
a 32 2 a 33 2 
[M] =  a 31 2a 31 a 32 2a 32 a 33 2a 31 a 33

 a 21 a11 a 22 a12 a13 a 23 (a11 a 22 + a12 a 21 ) (a13 a 22 + a12 a 23 ) (a13 a 21 + a11 a 23 )
a a a 32 a 22 a 33 a 23 (a 31 a 22 + a 32 a 21 ) (a 33 a 22 + a 32 a 23 ) (a 33 a 21 + a 31 a 23 )
 31 21
 a 31 a11 a 32 a12 a 33 a13 (a 31 a12 + a 32 a11 ) (a 33 a12 + a 32 a13 ) (a 33 a11 + a 31 a13 )
En este caso particular la matriz de transformación [M] tras eliminar filas y columnas
asociadas con la dirección x3 queda:

 T11′   a11   T11 


2 2
a12 2a11 a12
T′  =  a 2 2  
 22   21 a 22 2a 21 a 22   T22 
 T12′   a 21 a11 a 22 a12 a11 a 22 + a12 a 21   T12 
 
La matriz de transformación ( a ij ) en el plano viene dada en función de un único parámetro,
θ:
 cos θ sin θ 0
a ij = − sin θ cos θ 0
 0 0 1
Resultando así que:
 T11′   cos θ sin 2 θ 2 cos θ sin θ   T11 
2

T′  =  
 22   sin θ
2
cos θ
2
− 2 sin θ cos θ   T22 
 T12′   − sin θ cos θ cos θ sin θ cos 2 θ − sin 2 θ   T12 
 
Tomando partido de las siguientes relaciones trigonométricas, 2 cos θ sin θ = sin 2θ ,
1 − cos 2θ 1 + cos 2θ
cos 2 θ − sin 2 θ = cos 2θ , sin 2 θ = , cos 2 θ = , obtenemos que:
2 2
 1 + cos 2θ   1 − cos 2θ  
    sin 2θ 
 T11′   2   2    T11 
T′  =   1 − cos 2θ   1 + cos 2θ   
 22      − sin 2θ   T22 
2   2 
 T12′     T12 
  − sin 2θ   sin 2θ 
   cos 2θ 
  2   2  
Explícitamente las componentes vienen dadas por:
  1 + cos 2θ   1 − cos 2θ 
 T11′ =   T11 +   T22 + T12 sin 2θ
  2   2 
  1 − cos 2θ   1 + cos 2θ 
′ =
 T22  T11 −   T22 − T12 sin 2θ
  2   2 
  sin 2θ   sin 2θ 
 T12′ =  −  T11 +   T22 + T12 cos 2θ
  2   2 
Reestructurando la expresión anterior aún podemos decir que:

Universidad de Castilla- La Mancha Draft Por: Eduardo W. V. Chaves (2012)


Ciudad Real - España
94 PROBLEMAS RESUELTOS DE MECÁNICA DEL MEDIO CONTINUO

  T11 + T22   T11 − T22 


 T11′ =  +  cos 2θ + T12 sin 2θ
  2   2 
  T + T22   T11 − T22 
′ =  11
 T22 −  cos 2θ − T12 sin 2θ
  2   2 
  T − T22 
 T12′ = − 11  sin 2θ + T12 cos 2θ
  2 

Recordemos que las direcciones principales se caracterizan por la ausencia de las componentes
tangenciales, es decir, Tij = 0 para i ≠ j . Si queremos encontrar las direcciones principales en
el caso plano hacemos que T12′ = 0 , obteniendo así:
 T − T22   T11 − T22 
T12′ = − 11  sin 2θ + T12 cos 2θ = 0 ⇒   sin 2θ = T12 cos 2θ
 2   2 

sin 2θ 2 T12 2 T12 1  2 T12 


⇒ = ⇒ tg(2θ ) = ⇒ θ = arctg 
cos 2θ T11 − T22 T11 − T22 2  T11 − T22 

Ejemplo 1.97
Obtener los valores principales y direcciones principales de la parte simétrica del tensor T ,
cuyas componentes en el sistema cartesiano vienen dadas por:
5 1 
Tij =   (i, j = 1,2)
3 4
Solución:
La parte simétrica del tensor viene dada por:
5 2 
Tijsym =
1
( )
Tij + T ji =  
2  2 4
Valores principales:
5−λ 2
=0 ⇒ λ2 − 9λ + 16 = 0
2 4−λ
La solución de la ecuación cuadrática viene dada por:
− 9 ± (−9) 2 − 4 × (1) × (16) λ1 ≡ T1 = 6,5615
λ (1, 2 ) = ⇒
2 ×1 λ 2 ≡ T2 = 2,4385

Universidad de Castilla- La Mancha Draft Por: Eduardo W. V. Chaves (2012)


Ciudad Real - España
1 TENSORES 95

Podemos dibujar el círculo de Mohr del tensor T sym :

TSsym

(T11sym , T12sym )


TII = 2,4385 TI = 6,5615 T Nsym

En el caso plano, la dirección principal se puede obtener directamente a través de la relación:


2 T12sym 2×2
tan(2θ) = = =4 ⇒ θ = 37,982º
T11sym − sym
T22 5−4

1.1.16 Campo de Tensores.

Ejemplo 1.98
Encuentre el gradiente de la función f ( x1 , x2 ) = sin( x1 ) + exp x1x2 en el punto (0,1).
Solución:
Por definición el gradiente de una función escalar viene definido de la forma:
∂f ˆ ∂f ˆ
∇ xr f = e1 + e2
∂x1 ∂x 2
donde:
∂f ∂f
= cos( x1 ) + x 2 exp x1x2 ; = x1 exp x1x2
∂x1 ∂x 2

[ ] [
∇ xr f ( x1 , x 2 ) = cos( x1 ) + x 2 exp x1 x2 eˆ 1 + x1 exp x1x2 eˆ 2 ]
∇ xr f (0,1) = [2] eˆ 1 + [0] eˆ 2 = 2eˆ 1

Ejemplo 1.99
r
Supongamos que v y ϕ son respectivamente vector y escalar, y dos veces diferenciables
continuamente. Usando notación indicial, demostrar que:
r r
a) ∇ xr ⋅ (∇ xr ∧ v ) = 0
b) ∇ xr ⋅ (∇ xr ϕ ) = ∇ xr 2ϕ

Universidad de Castilla- La Mancha Draft Por: Eduardo W. V. Chaves (2012)


Ciudad Real - España
96 PROBLEMAS RESUELTOS DE MECÁNICA DEL MEDIO CONTINUO

c) ∇ xr (φµ) = µ∇ xr φ + φ∇ xr µ
r r r
d) ∇ xr ⋅ (φv ) = ∇ xr φ ⋅ v + φ∇ xr ⋅ v
e) ∇ xr ⋅ ( A ⋅ B) = ∇ xr A : B + A ⋅ (∇ xr ⋅ B)
Solución:
a) Considerando que
r r
∇ xr ∧ v =  ijk v k , j ê i (1.92)
∂ (•)
∇ xr ⋅ (•) = ⋅ ê l (1.93)
∂xl

luego
r r ∂ ∂
∇ xr ⋅ (∇ xr ∧ v ) = ( ijk v k , j eˆ i ) ⋅ eˆ l = ( ijk v k , j δ il )
∂xl ∂x l
∂ (1.94)
= ( ljk v k , j )
∂xl
=  ljk v k , jl

Observemos que  ljk es un tensor antisimétrico en lj y vk , jl es simétrico con lj , luego:

 ljk v k , jl = 0 (1.95)

b)
∂ ∂
∇ xr ⋅ (∇ xr ϕ ) = (ϕ ,i eˆ i ) ⋅ eˆ j = (ϕ ,i δ ij )
∂x j ∂x j
∂ϕ , j
= = ϕ , jj (1.96)
∂x j

∂  ∂ϕ  ∂ 2ϕ
=  = 2
= ∇ xr ϕ
∂x j  ∂x j  ∂x 2
  j

c)
[∇ xr (φµ)]i = (φµ) ,i = φ ,i µ + φµ ,i
(1.97)
= µ[∇ xr φ ]i + φ[∇ xr µ]i
r
d) El resultado de la operación ∇ xr ⋅ (φv ) resulta un escalar, luego:
r
∇ xr ⋅ (φv ) = (φvi ) ,i = φ ,i v i + φv i , i
r r
= ∇ xr φ ⋅ v + φ∇ xr ⋅ v
e) Considerando que ( A ⋅ B) ij = A ik B kj , [∇ xr ⋅ ( A ⋅ B)]i = ( A ⋅ B) ij , j = ( A ik B kj ) , j , luego
( A ik B kj ) , j = A ik , j B kj + A ik B kj , j = [∇ xr A : B ]i + [A ⋅ (∇ xr ⋅ B)]i

Universidad de Castilla- La Mancha Draft Por: Eduardo W. V. Chaves (2012)


Ciudad Real - España
1 TENSORES 97

Ejemplo 1.100
Probar la identidad:
r r r r
∇ xr ⋅ (a + b) = ∇ xr ⋅ a + ∇ xr ⋅ b
Solución:
r r ∂
Considerando que a = a j eˆ j y b = b k eˆ k y ∇ xr = ê i podemos expresar el primer
∂x i
miembro de la identidad como:
∂ (a j eˆ j + b k eˆ k ) ∂a j ∂b k ∂a ∂b r r
⋅ eˆ i = eˆ j ⋅ eˆ i + eˆ k ⋅ eˆ i = i + i = ∇ xr ⋅ a + ∇ xr ⋅ b
∂x i ∂x i ∂x i ∂x i ∂x i
c.q.d.

Ejemplo 1.101
r r
Obtener las componentes de (∇ xr a) ⋅ b .
Solución:
r r ∂
Considerando: a = a j eˆ j ; b = b k eˆ k y ∇ xr = ê i ( i = 1,2,3 ) podemos decir que:
∂x i

r r  ∂ (a j eˆ j )   ∂a j  ∂a j ∂a j
(∇ xr a) ⋅ b =  ⊗ eˆ i  ⋅ (b k eˆ k ) =  eˆ j ⊗ eˆ i  ⋅ (b k eˆ k ) = b k δ ik eˆ j = b k eˆ j
 ∂x i  ∂ x ∂ x ∂ x
   i  i k

Expandiendo el índice mudo k :


∂a j ∂a j ∂a j ∂a j
bk = b1 + b2 + b3
∂x k ∂x1 ∂x 2 ∂x 3
luego:
∂a1 ∂a ∂a
j =1 ⇒ b1 + b2 1 + b3 1
∂x1 ∂x 2 ∂x 3
∂a 2 ∂a ∂a
j = 2 ⇒ b1 + b2 2 + b3 2
∂x1 ∂x 2 ∂x 3
∂a 3 ∂a ∂a
j = 3 ⇒ b1 + b 2 3 + b3 3
∂x1 ∂x 2 ∂x 3

Ejemplo 1.102
Probar que la siguiente relación es válida:
r
q 1 r 1 r
∇ xr ⋅   = ∇ xr ⋅ q − 2 q ⋅ ∇ xr T
T  T T
r r r
donde q( x , t ) es un campo vectorial arbitrario y T ( x , t ) un campo escalar.
Solución:

Universidad de Castilla- La Mancha Draft Por: Eduardo W. V. Chaves (2012)


Ciudad Real - España
98 PROBLEMAS RESUELTOS DE MECÁNICA DEL MEDIO CONTINUO

r
 q  ∂  qi   qi 
∇ xr ⋅   =  = 
 T  ∂x i  T   T  ,i
1 1
= q i ,i − 2 q i T,i
T T
1 r r 1 r
= ∇ x ⋅ q − 2 q ⋅ ∇ xr T
T T

Ejemplo 1.103
Demostrar que:
r r r r r r
a) rot (λa) = ∇ xr ∧ (λa) = λ(∇ xr ∧ a) + (∇ xr λ ∧ a) (1.98)
r r r r r r r r r r r
b) ∇ xr ∧ (a ∧ b) = (∇ xr ⋅ b)a − (∇ xr ⋅ a)b + (∇ xr a) ⋅ b − (∇ xr b) ⋅ a (1.99)
r r r r r
c) ∇ xr ∧ (∇ xr ∧ a) = ∇ xr (∇ xr ⋅ a) − ∇ xr a
2
(1.100)

d) ∇ xr ⋅ (ψ∇ xr φ ) = ψ∇ xr φ + (∇ xr ψ ) ⋅ (∇ xr φ)
2
(1.101)

Solución:
r r
a) El resultado de la operación ∇ xr ∧ (λa) será un vector, cuyas componentes vienen dadas
por:
[∇r r
x
r
∧ (λa) i ] =  ijk (λa k ) , j
=  ijk (λ , j a k + λa k , j )
=  ijk λa k , j  ijk λ , j a k (1.102)
r r
= λ (∇ x ∧ a) i  ijk (∇ x λ ) j a k
r r
r r r
= λ (∇ xr ∧ a) i  (∇ xr λ ∧ a) i
r r r r r r
con lo que comprobamos la identidad: rot (λa) = ∇ xr ∧ (λa) = λ(∇ xr ∧ a) + (∇ xr λ ∧ a) .
r r r r
Las componentes del producto vectorial (a ∧ b) vienen dadas por (a ∧ b) k =  kij a i b j . Luego:

[∇r r
x
r r
]
∧ (a ∧ b) l =  lpk ( kij a i b j ) , p
(1.103)
=  kij  lpk (a i , p b j + a i b j , p )

b) Considerando que  kij =  ijk , el resultado de  ijk  lpk = δ il δ jp − δ ip δ jl y reemplazando en la


expresión anterior obtenemos que:

[∇r r
x
r r
]
∧ (a ∧ b) l =  kij  lpk (a i , p b j + a i b j , p )
= (δ il δ jp − δ ip δ jl )(a i , p b j + a i b j , p )
(1.104)
= δ il δ jp a i , p b j − δ ip δ jl a i , p b j + δ il δ jp a i b j , p − δ ip δ jl a i b j , p
= al , p b p − a p, p b l + al b p, p − a p b l , p

Podemos observar que [(∇ x a)


r
r
⋅ b]l
r
= al , p b p , [(∇ r
x ⋅ a)b]l
r r
= a p, p b l , [(∇ r
x ⋅ b)a]l
r r
= al b p, p ,

[ r r
]
(∇ xr b) ⋅ a l = a p b l , p .

Universidad de Castilla- La Mancha Draft Por: Eduardo W. V. Chaves (2012)


Ciudad Real - España
1 TENSORES 99

r r r r
c) Las componentes del producto vectorial (∇ xr ∧ a) vienen dadas por (∇ xr ∧ a) i =  ijk a k , j .
123
ci
Luego:
[∇r r
x
r r
]
∧ (∇ xr ∧ a) q =  qli c i ,l
=  qli ( ijk a k , j ) ,l (1.105)
=  qli  ijk a k , jl

Considerando que  qli  ijk =  qli  jki = δ qj δ lk − δ qk δ lj , la expresión anterior queda:

[∇r r
x
r r
]
∧ (∇ xr ∧ a) q =  qli  ijk a k , jl
= (δ qj δ lk − δ qk δ lj )a k , jl
(1.106)
= δ qj δ lk a k , jl − δ qk δ lj a k , jl
= a k ,kq − a q ,ll

Podemos observar que [∇ xr (∇ xr ⋅ a)]q = a k , kq y ∇ xr 2 a q = a q ,ll .


r
[ r
]
d)
∇ xr ⋅ (φ∇ xr ψ ) = (φψ ,i ) ,i
= φψ ,ii + φ ,i ψ ,i (1.107)
= φ∇ xr ψ + (∇ xr φ ) ⋅ (∇ xr ψ )
2

donde φ y ψ son funciones escalares.


Otra identidad interesante que origina de la anterior es:
∇ xr ⋅ (φ∇ xr ψ ) = φ∇ xr ψ + (∇ xr φ ) ⋅ (∇ xr ψ )
2

(1.108)
∇ xr ⋅ (ψ∇ xr φ ) = ψ∇ xr φ + (∇ xr ψ ) ⋅ (∇ xr φ)
2

Restando las dos identidades anteriores obtenemos que:


∇ xr ⋅ (φ∇ xr ψ ) − ∇ xr ⋅ (ψ∇ xr φ) = φ∇ xr ψ − ψ∇ xr φ
2 2

(1.109)
⇒ ∇ xr ⋅ (φ∇ xr ψ − ψ∇ xr φ ) = φ∇ xr ψ − ψ∇ xr φ
2 2

Ejemplo 1.104
a) Probar que ∇ xr ⋅ (∇ xr ∧ v ) = 0 y que ∇ xr ∧ (∇ xr φ ) = 0 , donde φ es un campo escalar, y v es
r r r r r

un campo vectorial;
[r r r
] r
[
b) Demostrar que ∇ xr ∧ (∇ xr ∧ v ) ∧ v = (∇ xr ⋅ v )(∇ xr ∧ v ) + ∇ xr (∇ xr ∧ v ) ⋅ v − (∇ xr v ) ⋅ (∇ xr ∧ v ) ;
r r r r r
] r r r r

r r r r r r r r
c) Teniendo en cuenta que ω = ∇ xr ∧ v , demostrar que ∇ xr ∧ (∇ xr 2 v ) = ∇ xr 2 (∇ xr ∧ v ) = ∇ xr 2 ω .
Solución:
r r
Considerando: ∇ xr ∧ v =  ijk v k , j ê i

Universidad de Castilla- La Mancha Draft Por: Eduardo W. V. Chaves (2012)


Ciudad Real - España
100 PROBLEMAS RESUELTOS DE MECÁNICA DEL MEDIO CONTINUO

⋅ (∇ xr )
r ∂
∇ xr
r
∧v =
∂x l
(
 ijk v k , j eˆ i )⋅ eˆ l

∂ ∂
=  ijk
∂x l
( )
v k , j δ il =  ijk
∂x i
( )
vk , j

=  ijk v k , ji
r
La segunda derivada de v es simétrica en ij , i.e. v k , ji = v k ,ij , mientras que  ijk es antisimétrico
en ij , i.e.,  ijk = − jik , luego:
 ijk v k , ji =  ij1v1, ji +  ij 2 v 2, ji +  ij 3 v3, ji = 0
Observar que  ij1v1, ji es el doble producto escalar de un tensor simétrico con un antisimétrico,
cuyo resultado es cero.
Análogamente demostramos que:
r r
∇ xr ∧ (∇ xr φ ) =  ijk φ , kj eˆ i = 0 i eˆ i = 0

NOTA: El rotacional del gradiente de un escalar resulta ser igual al vector nulo, y la
divergencia del rotacional de un vector resulta ser igual a cero.
r r r
b) Denominamos por ω = ∇ xr ∧ v , con eso, quedamos con
r r
[(r r
∇ xr ∧ ∇ xr ∧ v ∧ v ) ]
r r r
= ∇ xr ∧ (ω ∧ v )
Recurrimos a la identidad (1.99), luego, se cumple que:
r r r r r r r r r r r
∇ xr ∧ (ω ∧ v ) = (∇ xr ⋅ v ) ω − (∇ xr ⋅ ω)v + (∇ xr ω) ⋅ v − (∇ xr v ) ⋅ ω
r r r
Fijemos que el término ∇ xr ⋅ ω = ∇ xr ⋅ (∇ xr ∧ v ) = 0 , que fue demostrado en el apartado a).
Luego, concluimos que:
r r r r r r r r r
∇ xr ∧ (ω ∧ v ) = (∇ xr ⋅ v )ω + (∇ xr ω) ⋅ v − (∇ xr v ) ⋅ ω
r r r r r r
[ r r
]r
= (∇ xr ⋅ v )(∇ xr ∧ v ) + ∇ xr (∇ xr ∧ v ) ⋅ v − (∇ xr v ) ⋅ (∇ xr ∧ v )
c) Recurriendo a la identidad (1.100) podemos decir que:
2r r r r r
∇ xr v = ∇ xr (∇ xr ⋅ v ) − ∇ xr ∧ (∇ xr ∧ v )
r r r
= ∇ xr (∇ xr ⋅ v ) − ∇ xr ∧ ω
Aplicando el rotacional a la expresión anterior obtenemos que:
r 2r
r r r r r
∇ xr ∧ (∇ xr v ) = ∇ xr ∧ [∇ xr (∇ xr ⋅ v )] − ∇ xr ∧ (∇ xr ∧ ω)
14442r 444 3
=0
donde hemos tenido en cuenta que el rotacional del gradiente de un escalar resulta el vector
r r r
nulo. Recurrimos una vez más la identidad (1.100) para expresar el término ∇ xr ∧ (∇ xr ∧ ω) ,
resultando:
r 2r
r r r r 2 r
∇ xr ∧ (∇ xr v ) = −∇ xr ∧ (∇ xr ∧ ω) = −∇ xr (∇ xr ⋅ ω) + ∇ xr ω
r r
[ 2 r
= −∇ xr ∇ xr ⋅ (∇ xr ∧ v ) + ∇ xr ω
1442443
]
=0
2
r r
= ∇ xr (∇ xr ∧ v )

Universidad de Castilla- La Mancha Draft Por: Eduardo W. V. Chaves (2012)


Ciudad Real - España
1 TENSORES 101

Ejemplo 1.105
Demostrar que:
r r r r r r r r
∇ xr ⋅ (a ∧ b) = (∇ xr ∧ a) ⋅ b + a ⋅ (∇ xr ∧ b)
a) r r r r (1.110)
= rot (a) ⋅ b + a ⋅ rot (b)
Solución:
r r
La operación ∇ xr ⋅ (a ∧ b) resulta un escalar:
r r
∇ xr ⋅ (a ∧ b) = ( ijk a j b k ) ,i
=  ijk a j ,i b k +  ijk b k ,i a j
12r3 12r3
( ∇ ∧ a) k (∇ ∧b ) j
r r r r r r
= (∇ xr ∧ a) ⋅ b + a ⋅ (∇ xr ∧ b)

Ejemplo 1.106
r r r r r
Sea v un campo vectorial que es función de x , i.e. v = v ( x ) , donde sus componentes vienen
dadas por:
v1 = x1 − 5 x 2 + 2 x3

v 2 = 5 x1 + x 2 − 3 x3
v = −2 x + 3 x + x
 3 1 2 3
r r
a) Obtener el gradiente de v ; b) Obtener ∇ xr v : 1 ; c) Hacer la descomposición aditiva del
r
tensor ∇ xr v a través de su parte simétrica y otra antisimétrica.; d) Obtener el vector axil
r
asociado al tensor antisimétrico (∇ xr v ) anti .
Solución: a)
 ∂v1 ∂v1 ∂v1 
 
r  ∂x1 ∂x 2 ∂x3   1 − 5 2 
r ∂v ∂v  ∂v ∂v 2 ∂v 2  
1 − 3
r
∇ xr v = r componente
 s →(∇ xr v ) ij = i =  2 = 5
∂x ∂x j ∂x ∂x 2 ∂x3  
 1   1 
 ∂v3 ∂v 3 ∂v3   − 2 3
 ∂x1 ∂x 2 ∂x3 
r r
b) ∇ xr v : 1 = Tr (∇ xr v ) = 1 + 1 + 1 = 3 . Observar que la traza del gradiente es igual a la divergencia,
r ∂v1 ∂v 2 ∂v3
i.e.: ∇ xr ⋅ v = vi ,i = v1,1 + v 2, 2 + v 3,3 = + + = 3.
∂x1 ∂x 2 ∂x3
r r r
c) ∇ xr v = (∇ xr v ) sym + (∇ xr v ) anti =
1
[ r r 1
] [
r r
(∇ xr v ) + (∇ xr v ) T + (∇ xr v ) − (∇ xr v ) T
2 44424443 1 2 44424443
]
1 r r
=(∇ xr v ) sym =(∇ xr v ) anti
r r
Luego, las componentes de (∇ xr v ) sym y (∇ xr v ) anti vienen dadas respectivamente por:
1 0 0  0 −5 2 
1  ∂vi ∂v j    1  ∂v i ∂v j  
0 − 3
r sym r anti
(∇ xr v ) ij =  +  = 0 1 0 ; (∇ xr v ) ij =  − = 5
2  ∂x j ∂x i   2  ∂x j ∂xi  
0 0 1   − 2 3 0 

Universidad de Castilla- La Mancha Draft Por: Eduardo W. V. Chaves (2012)


Ciudad Real - España
102 PROBLEMAS RESUELTOS DE MECÁNICA DEL MEDIO CONTINUO

d) Recordar que
 1  ∂v1 ∂v 2  1  ∂v1 ∂v3  
 0  −   − 
 2  ∂x 2 ∂x1  2  ∂x 3 ∂x1  
 1  ∂v 1  ∂v 2 ∂v 3  
[ r
(W ) ij ≡ (∇ xr v ) anti ]
ij ≡ vianti
,j
∂v 
=   2 − 1  0  − 
2  ∂x3 ∂x 2  
 2  ∂x1 ∂x 2 
 1  ∂v ∂v  1  ∂v3 ∂v 2   (1.111)
  3 − 1   −  0 
 2  ∂x1 ∂x 3  2  ∂x 2 ∂x 3  
 0 W12 W13   0 − w3 w2 
W12 W13   0
=  W21 0 W23  =  − W12 00 − w1  W23  =  w3
 W31 W32 0   − W13 − W23
w1 0  0   − w2
r
donde w1 , w2 , w3 son las componentes del vector axil w correspondiente al tensor
r
antisimétrico W ≡ (∇ xr v ) anti , luego para el problema propuesto:
 0 w2   0 − 5 2 
− w3  w1 = 3
 w 
 30 − w1  =  5 0 − 3 ⇒  w2 = 2
− w2
w1 0   − 2 3 0  w = 5
 3
r
El vector axil en la base Cartesiana queda: w = 3eˆ 1 + 2eˆ 2 + 5eˆ 3 .
1 r r
Solución Alternativa d) Recurriendo al Ejemplo 1.36 donde hemos demostrado que (a ∧ x )
2
r r
es el vector axil asociado al tensor antisimétrico ( x ⊗ a ) anti . Luego, el vector axil asociado al
r
tensor antisimétrico (∇ xr v ) anti = (v ) ⊗ (∇ xr ) [r r
] anti
es el vector w =
r
2
(
1 rr r
∇x ∧v . )
eˆ 1 eˆ 2 eˆ 3
r 1 ∂ ∂ ∂ 1  ∂v ∂v   ∂v ∂v   ∂v ∂v  
w= =  3 − 2 eˆ 1 −  3 − 1 eˆ 2 +  2 − 1 eˆ 3 
2 ∂x1 ∂x 2 ∂x3 2  ∂x 2 ∂x3   ∂x1 ∂x 3   ∂x1 ∂x 2  
v1 v2 v3

=
1
2
[
(3 − (−3) )eˆ 1 − ((−2) − (2) )eˆ 2 + (5 − (−5))eˆ 3 = 3eˆ 1 + 2eˆ 2 + 5eˆ 3 ]

Ejemplo 1.107
r r
Sea un tensor de segundo orden definido por l = ∇ xr v . Teniendo en cuenta que D = (∇ xr v ) sym
r
y W = (∇ xr v ) anti demostrar que
r r
W ⋅ D + D ⋅ W = 2(D ⋅ W ) anti = (∇ xr v ⋅ ∇ xr v ) anti = ( l ⋅ l ) anti
Solución:
En el Ejemplo 1.34 hemos demostrado que, dado un tensor de segundo orden arbitrario l
se cumple que
l
anti
⋅l sym
+l sym
⋅l anti
= 2( l anti
⋅l sym anti
)

Universidad de Castilla- La Mancha Draft Por: Eduardo W. V. Chaves (2012)


Ciudad Real - España
1 TENSORES 103

Luego, se cumple que W ⋅ D + D ⋅ W = 2(D ⋅ W ) anti . Teniendo en cuenta la definición de


simetría y antisimetría, D =
1
2
[l +l
T
] , W = 12 [l − l ] , podemos concluir que:
T

W ⋅ D + D ⋅ W = 2(D ⋅ W ) anti
2
[
= (l + l T ) ⋅ (l − l T )
4
anti
]
1
[
= l ⋅l + l ⋅l T − l T ⋅l − l T ⋅l T
2
anti
]
1
= 1
2 4 4 4 [
l ⋅l − l ⋅l
T
2
T
4 44
anti
3
1
2
]
+ l ⋅l − l T ⋅l [ ]
T anti

=0
1
[
= l ⋅ l − (l ⋅ l ) T
2
anti
]
1
[
= 2( l ⋅ l ) anti
2
]
anti

r r
= ( l ⋅ l ) anti = (∇ xr v ⋅ ∇ xr v ) anti

OBS.: Fijemos que el tensor resultante l ⋅l T −l T


⋅ l es simétrico, ya que:
(l ⋅ l T
−l T
⋅ l )T = l ⋅l T
−l T
⋅l .

Ejemplo 1.108
r
Considérese un campo vectorial representado por su campo vector unitario bˆ ( x ) , ver Figura
r r
1.5. Obtener un tensor proyección de segundo orden P tal que se cumpla que p = P ⋅ u ,
r r r
donde u es un vector arbitrario y p es ortogonal al campo definido por el versor bˆ ( x ) .

r
bˆ ( x )

Figura 1.5: Campo vectorial

Universidad de Castilla- La Mancha Draft Por: Eduardo W. V. Chaves (2012)


Ciudad Real - España
104 PROBLEMAS RESUELTOS DE MECÁNICA DEL MEDIO CONTINUO

Solución:
El problema planteado se puede apreciar en la figura abajo:

r r r
p = P ⋅u u
r
a

r
bˆ ( x )

r r r r
Luego, a través de suma de vectores se cumple que: u = a + p . Además el vector a puede ser
r r r r
obtenido a través de la proyección de u según la dirección b̂ : a = a bˆ = (u ⋅ bˆ ) bˆ . Con eso
podemos decir que:
r r r
p=u−a p i = ui − ai
r r r r
= u − (u ⋅ bˆ ) bˆ = u − (u ⋅ bˆ ) ⊗ bˆ = u i − (u k bˆ k )bˆ i
r r
= 1 ⋅ u − (bˆ ⊗ bˆ ) ⋅ u = u δ − u bˆ bˆ
k ik k k i

[ ]
= 1 − (bˆ ⊗ bˆ ) ⋅ u
r
r
= (δ ik − bˆ k bˆ i )u k
= P ⋅u = Pik u k
Con lo cual concluimos que el tensor proyección de segundo orden viene dado por:
P = 1 − bˆ ⊗ bˆ
Este mismo resultado podría haber sido obtenido a través del producto vectorial. Dibujando
el problema planteado en otra perspectiva para mejor visualización, tenemos que:
r
u ∧ bˆ
r
bˆ ( x )
r
u

r
bˆ ∧ (u ∧ bˆ )
r r
Teniendo en cuenta que a ∧ (b ∧ a) = [(a ⋅ a)1 − a ⊗ a]⋅ b , ver Ejemplo 1.16, podemos decir
r r r r r r

r
[ ] r
[ r r
que: bˆ ∧ (u ∧ bˆ ) = (bˆ ⋅ bˆ )1 − bˆ ⊗ bˆ ⋅ u = 1 − bˆ ⊗ bˆ ⋅ u = p ]
Ejemplo 1.109
r r
Dado un campo vectorial v ( x ) , demostrar que se cumple la siguiente relación:
r r 1 r r r
∇ xr v ⋅ v = ∇ xr (v 2 ) − v ∧ (∇ xr ∧ v )
2
r
donde v es el módulo de v .
Solución:

Universidad de Castilla- La Mancha Draft Por: Eduardo W. V. Chaves (2012)


Ciudad Real - España
1 TENSORES 105

Fijemos que
1 r 2
2
[ ] 1
2
r r 1
2
1
2
r r
∇ x (v ) i = [∇ xr (v ⋅ v )]i = (v k v k ) ,i = (v k ,i v k + v k v k ,i ) = v k v k ,i = (v ⋅ ∇ xr v ) i .
r r
En un punto del campo vectorial v vamos considerar un plano normal a v y recordar que la
r
proyección de un tensor de segundo orden según una dirección ( v ) resulta un vector el cual
r
no necesariamente tiene la misma dirección que ( v ), con eso vamos representar los siguientes
r r r r
vectores ∇ xr v ⋅ v y v ⋅ ∇ xr v :

r r
∇ xr v ⋅ v

r r r
r c⊥v
(∇ xr ∧ v ) r
r r
r c ⊥ (∇ xr ∧ v )
v r r
v ⋅ ∇ xr v

r r r r
c = v ∧ (∇ xr ∧ v )

Verifiquemos que a través de suma de vectores se cumple que:


r r r r r
∇ xr v ⋅ v + c = v ⋅ ∇ xr v
r r r r r
⇒ c = v ⋅ ∇ xr v − ∇ xr v ⋅ v
r r r r r
⇒ c = v ⋅ ∇ xr v − v ⋅ ∇ xr v T
r r r r r r
⇒ c = v ⋅ (∇ xr v − ∇ xr v T ) = v ⋅ 2(∇ xr v ) anti
r r
Si consideramos que w es el vector axil asociado al tensor antisimétrico (∇ xr v ) anti se cumple
r r r r r r r r
que: (∇ xr v ) anti ⋅ v = w ∧ v ⇒ v ⋅ (∇ xr v ) anti = v ∧ w . Además se cumple también que
r r r r
rot (v ) ≡ ∇ xr ∧ v = 2 w . Luego,
r r r r r r r r
c = v ⋅ 2(∇ xr v ) anti = v ∧ 2w = v ∧ (∇ xr ∧ v ) (1.112)
con eso concluimos que:
r r r r r
∇ xr v ⋅ v + c = v ⋅ ∇ xr v
r r r r r
⇒ ∇ xr v ⋅ v = v ⋅ ∇ xr v − c
r r 1 r r r
⇒ ∇ xr v ⋅ v = ∇ xr (v 2 ) − v ∧ (∇ xr ∧ v )
2
r r r
Es interesante observar que cuando (∇ xr v ) resulta ser un tensor simétrico, (∇ xr v ) = (∇ xr v ) sym ,
r r r r r r r r r r
se cumple que (∇ xr v ) anti = 0 , c = 0 , (∇ xr ∧ v ) = 0 , ∇ xr v ⋅ v = v ⋅ ∇ xr v y tiene la misma
r
dirección que v .
r r r r r r r
En el caso que se cumpla (∇ xr v ) = (∇ xr v ) anti tenemos que: c = v ⋅ 2(∇ xr v ) anti = 2v ⋅ (∇ xr v ) , ver
r r r r r
expresión (1.112). Con eso se cumple también que v ⋅ ∇ xr v = −∇ xr v ⋅ v , y además, v es
r r
perpendicular al vector (∇ xr ∧ v ) , ver figura abajo:

Universidad de Castilla- La Mancha Draft Por: Eduardo W. V. Chaves (2012)


Ciudad Real - España
106 PROBLEMAS RESUELTOS DE MECÁNICA DEL MEDIO CONTINUO

r r
(∇ xr v ) = (∇ xr v ) anti r r
∇ xr v ⋅ v r r
c⊥v
r r r
r c ⊥ (∇ xr ∧ v )
v

r r
(∇ xr ∧ v )

r r
v ⋅ ∇ xr v

r r r r r r
c = v ∧ (∇ xr ∧ v ) = 2v ⋅ (∇ xr v )

Solución Alternativa:
r r r r r r r r r
∇ xr v ⋅ v = (∇ xr v sym + ∇ xr v anti ) ⋅ v = ∇ xr v sym ⋅ v + ∇ xr v anti ⋅ v
r r r r r r r r
= ∇ xr v sym ⋅ v + ∇ xr v anti ⋅ v + (∇ xr v anti ⋅ v − ∇ xr v anti ⋅ v )
r r r r r r
= (∇ xr v sym ⋅ v − ∇ xr v anti ⋅ v ) + 2∇ xr v anti ⋅ v
1
[
r r r r
] r
= (∇ xr v + ∇ xr v T ) − (∇ xr v − ∇ xr v T ) ⋅ v + 2∇ xr v anti ⋅ v
2
r r

1 r r r r
= (2∇ xr v T ) ⋅ v + 2∇ xr v anti ⋅ v
2
r r r r
= v ⋅ ∇ xr v + 2∇ xr v anti ⋅ v
1 r r r
= ∇ xr (v 2 ) − v ∧ (∇ xr ∧ v )
2
r r r r r r r r r
Recordar que (∇ xr v anti ) T = −∇ xr v anti , luego 2∇ xr v anti ⋅ v = −v ⋅ 2∇ xr v anti = −v ∧ (∇ xr ∧ v )

Ejemplo 1.110
r r
Considérese un campo vectorial estacionario u( x ) . Obtener las componentes del diferencial
r r r
total du . Considerando que u( x ) representa el campo de desplazamientos y es independiente
de la componente x3 , hacer la representación gráfica del campo de desplazamiento en un
elemento diferencial de área dx1 dx 2 .

Universidad de Castilla- La Mancha Draft Por: Eduardo W. V. Chaves (2012)


Ciudad Real - España
1 TENSORES 107

Solución: Según la definición de diferencial total y de gradiente se cumple que:

r r r r r r
u( x ) dx u( x + dx )

r r r r r r
r du ≡ u( x + dx ) − u( x )
x2 x r r r r r
x + dx du = ∇ xr u ⋅ dx

x1
x3

Luego, las componentes vienen dadas por:

 ∂u1 ∂u1 ∂u1   ∂u ∂u ∂u


 du1 = 1 dx1 + 1 dx 2 + 1 dx3
  ∂x1 ∂x 2 ∂x3
 du1   ∂x1 ∂x 2 ∂x3   dx  
 ⇒ du = ∂u 2 dx + ∂u 2 dx + ∂u 2 dx
1
∂u i du  =  ∂u 2 ∂u 2 ∂u 2  
du i = dx j ⇒  2   ∂x dx   2 ∂x
∂x3  
2 1 2 3
∂x j ∂x 2 ∂x 2 ∂x3
 du 3   1   dx3   1

 ∂u 3 ∂u 3 ∂u 3   ∂u ∂u ∂u
 ∂x1 ∂x 2 ∂x3  du 3 = 3 dx1 + 3 dx 2 + 3 dx3
 ∂x1 ∂x 2 ∂x3
con
du1 = u1 ( x1 + dx1 , x 2 + dx 2 , x3 + dx3 ) − u1 ( x1 , x 2 , x3 )

du 2 = u 2 ( x1 + dx1 , x 2 + dx 2 , x3 + dx3 ) − u 2 ( x1 , x 2 , x3 )
du = u ( x + dx , x + dx , x + dx ) − u ( x , x , x )
 3 3 1 1 2 2 3 3 3 1 2 3

Para el caso plano, es decir, cuando el campo es independiente de x3 , el campo de


desplazamientos en el elemento diferencial de área viene definido por:
 ∂u1 ∂u1
du1 = u1 ( x1 + dx1 , x 2 + dx 2 ) − u1 ( x1 , x 2 ) = ∂x dx1 + ∂x dx 2
 1 2

du = u ( x + dx , x + dx ) − u ( x , x ) = ∂u 2 dx + ∂u 2 dx
 2 2 1 1 2 2 2 1 2
∂x1
1
∂x 2
2

o aún:
 ∂u1 ∂u1
u1 ( x1 + dx1 , x 2 + dx 2 ) = u1 ( x1 , x 2 ) + ∂x dx1 + ∂x dx 2
 1 2

u ( x + dx , x + dx ) = u ( x , x ) + ∂u 2 dx + ∂u 2 dx
 2 1 1 2 2 2 1 2
∂x1
1
∂x 2
2

Observemos que la expresión anterior es equivalente a la expansión en serie de Taylor


teniendo en cuenta solo hasta términos lineales. La representación del campo de
desplazamiento en el elemento diferencial de área se muestra en la Figura 1.6.

Universidad de Castilla- La Mancha Draft Por: Eduardo W. V. Chaves (2012)


Ciudad Real - España
108 PROBLEMAS RESUELTOS DE MECÁNICA DEL MEDIO CONTINUO

∂u 2 ∂u 2 ∂u
u2 + dx 2 u2 + dx1 + 2 dx 2
∂x 2 ∂x1 ∂x 2

( x1 , x 2 + dx 2 ) ( x1 + dx1 , x 2 + dx 2 )

∂u1 ∂u1 ∂u
u1 + dx 2 u1 + dx1 + 1 dx 2
∂x 2 ∂x1 ∂x 2
r
du
dx 2

∂u1
u1 + dx1
(u1 ) ∂x1

( x1 , x 2 ) ( x1 + dx1 , x 2 )
x2
(u 2 ) ∂u 2
u2 + dx1
∂x1
dx1
x1

144444444444444444424444444444444444443

=
644444444444444444474444444444444444448

x 2 ,u 2
∂u1
dx2
∂x2
∂u 2
u2 + dx2
∂x2 B′

B B B′

dx 2 dx 2
+
O′ A′ A′
∂u 2
u2 A
dx1
O A ∂x1
dx1 O′
dx1
u1

∂u1
u1 + dx1
∂x1
x1 ,u1

Figura 1.6

Universidad de Castilla- La Mancha Draft Por: Eduardo W. V. Chaves (2012)


Ciudad Real - España
1 TENSORES 109

Ejemplo 1.111
r
Dado un campo tensorial de segundo orden, T ( x ) . Demostrar que si no hay una fuente de
r r r
T ( x ) se cumple que la divergencia de T ( x ) es igual a cero, i.e. ∇ xr ⋅ T = 0 . Para la
demostración considerar el campo tensorial en un elemento diferencial de volumen
dV = dx1 dx 2 dx 3 en el sistema Cartesiano.
Solución:
r
Primero vamos establecer el campo T ( x ) en el diferencial de volumen. Para ello, partimos de
r
la definición del diferencial de T ( x ) que viene definido a través del gradiente como:
r r r
dT ≡ T ( x + dx ) − T ( x )  r r r r r r r r
r  ⇒ T ( x + dx ) − T ( x ) = ∇ xr T ⋅ dx ⇒ T ( x + dx ) = T ( x ) + ∇ xr T ⋅ dx
dT = ∇ xr T ⋅ dx 
En componente la expresión anterior queda:
r r r
Tij ( x + dx ) = Tij ( x ) + Tij ,k dx k
r
= Tij ( x ) + Tij ,1 dx1 + Tij , 2 dx 2 + Tij ,3 dx3
r ∂Tij ∂Tij ∂Tij
= Tij ( x ) + dx1 + dx 2 + dx3
∂x1 ∂x 2 ∂x 3
r r
La representación de las componentes del campo Tij ( x + dx ) se pueden apreciar en la Figura
r ∂Ti1
1.7. Observar que en la cara normal a x1 + dx1 actúan las componentes Ti1 ( x ) + dx1 , ya
∂x1
que según nuestra convención el primer índice indica la dirección hacía donde apunta y el
segundo índice indica el plano normal.
r r
Una vez establecido el campo de Tij ( x + dx ) en el elemento diferencial de volumen,
r r
aplicamos el balance total de las componentes del campo Tij ( x + dx ) según las direcciones x1 ,
x 2 , x3 .
r r
Balance total de Tij ( x + dx ) en dV según dirección x1 es igual a cero (no hay fuente):

 ∂T   ∂T   ∂T 
 T11 + 11 dx1  dx 2 dx3 +  T13 + 13 dx 3 dx1 dx 2 +  T12 + 12 dx 2 dx1 dx3 − T11 dx 2 dx3
 ∂x1   ∂x3   ∂x 2 
− T13 dx1 dx 2 − T12 dx1 dx3 = 0
Simplificando la expresión anterior obtenemos que:
∂T11 ∂T ∂T
dx1 dx 2 dx3 + 13 dx3 dx1 dx 2 + 12 dx 2 dx1 dx3 = 0
∂x1 ∂x3 ∂x 2
∂T11 ∂T12 ∂T13
⇒ + + =0
∂x1 ∂x 2 ∂x3
Análogamente según las direcciones x 2 y x3 vamos obtener, respectivamente, que:
∂T21 ∂T22 ∂T23 ∂T31 ∂T32 ∂T33
+ + =0 y + + =0
∂x1 ∂x 2 ∂x3 ∂x1 ∂x 2 ∂x3

Universidad de Castilla- La Mancha Draft Por: Eduardo W. V. Chaves (2012)


Ciudad Real - España
110 PROBLEMAS RESUELTOS DE MECÁNICA DEL MEDIO CONTINUO

x3 Cara oculta
T11
∂T33 T21
T33 + dx3
Cara oculta ∂x3
∂T23
T23 + dx3
∂x3 T31
∂T13
T13 + dx3
∂x3 ∂T32 dx 3
T32 + dx2
∂x2
T12
T22 ∂T31
T31 + dx1 ∂T22
T22 + dx2
∂x1 ∂T12 ∂x2 x2
T12 + dx2
∂x2
T32
∂T21
T21 + dx1 dx1
∂T11 ∂x1
T11 + dx1
∂x1

T13
x1 T23
Cara oculta

T33

dx 2

Figura 1.7: Componentes del campo tensorial en un elemento diferencial de volumen.

Luego, tenemos el siguiente conjunto de ecuaciones que hay que cumplir simultáneamente:
 ∂T11 ∂T12 ∂T13
 + + =0
 ∂x1 ∂x 2 ∂x3
 T11,1 + T12, 2 + T13,3 = 0  T1 j , j = 0
 ∂T21 ∂T22 ∂T23  
 + + =0 ⇒  T21,1 + T22, 2 + T23,3 = 0 ⇒  T2 j , j = 0 ⇒ Tij , j = 0 i
 ∂x1 ∂x 2 ∂x 3  
 ∂T31 ∂T32 ∂T33  T31,1 + T32, 2 + T33,3 = 0  T3 j , j = 0
 + + =0
 ∂x1 ∂x 2 ∂x3
Demostrando así que en la ausencia de fuente la divergencia es igual a cero:
r
Tij , j = 0 i ⇔ (∇ xr ⋅ T ) i = 0 i tensorial
  → ∇ xr ⋅ T = 0

NOTA: Si tenemos un campo tensorial, el orden de la fuente (o sumidero) es de un orden


menor que el campo tensorial, e.g. si el campo tensorial es un vector, la fuente de este campo
tensorial será un escalar.
NOTA: Si la divergencia de un campo tensorial es positiva indica que tenemos una fuente del
campo tensorial, caso contrario, si la divergencia es negativa tenemos un sumidero.

Universidad de Castilla- La Mancha Draft Por: Eduardo W. V. Chaves (2012)


Ciudad Real - España
1 TENSORES 111

Ejemplo 1.112
Demostrar que:
[(∇ xr T ) ⋅ ur ] ⋅ ar = [∇ xr ( T ⋅ ar )] ⋅ ur (1.113)
r r r r
donde T = T (x ) es un campo tensorial de segundo orden, u = u( x ) es un campo vectorial, y
r r
a es un vector arbitrario (independiente de ( x ) ).
Solución:
r r
Observar que la operación [(∇ xr T ) ⋅ u] ⋅ a resulta un vector, que en notación indicial queda:
{[(∇ xr T ) ⋅ ur ] ⋅ ar}i = [(∇ xr T ) ⋅ ur ]ik (ar ) k [ ] [ ]
= (∇ xr T ) ikp u p a k = Tik , p u p a k = Tik , p u p a k (1.114)
Observar también que:
r r r
( T ⋅ a ) i = Tik a k  → [∇ xr ( T ⋅ a )]ij = ( T ⋅ a ) i , j = ( Tik a k ), j
gradiente

r
⇒ [∇ xr ( T ⋅ a )]ij = ( Tik a k ), j = Tik , j a k + Tik a k , j = Tik , j a k
{
=0
r r
donde hemos considerado que a es independiente de (x ) . Con eso, podemos concluir que:
{ }
{[∇ xr ( T ⋅ ar )] ⋅ ur}i = [∇ xr ( T ⋅ ar )]ij u j = Tik , j a k u j = Tik , p u p a k (1.115)
Si comparamos (1.114) con (1.115) demostramos (1.113).

Ejemplo 1.113
r r
Demostrar que si el módulo de un vector, ω = ω(t ) , es constante en el tiempo eso implica que
r
r dω
ω es ortogonal a para todo tiempo t .
dt
Solución:
r r r
Partimos de la definición del módulo de un vector: ω = ω ⋅ ω , con lo cual:
2

r
d ω( ) = d (ωr ⋅ ωr ) = d (ωr ) ⋅ ωr + ωr ⋅ d (ωr ) = 2ωr ⋅ d (ωr ) = 0
2


r dω
ω⊥
r

dt dt dt dt dt dt
NOTA: Un caso particular de este ejemplo es el movimiento circular donde:

r
v r
x = constante r
r  r dx
r r dx ⇒ x ⊥
x v=  dt
dt 

Universidad de Castilla- La Mancha Draft Por: Eduardo W. V. Chaves (2012)


Ciudad Real - España
112 PROBLEMAS RESUELTOS DE MECÁNICA DEL MEDIO CONTINUO

1.1.17 Teoremas con Integrales

Ejemplo 1.114
r
Comprobar el Teorema de la divergencia (Teorema de Gauss) para el campo vectorial F
cuyas componentes Cartesianas viene dadas por Fi = xi + ( x32 − x 3 )δ i 3 . Considerar la frontera
definida por el cilindro x12 + x 22 ≤ 1 , 0 ≤ x3 ≤ 1 .
Solución:
El Teorema de la divergencia afirma que:
r r
∫ ∇ xr ⋅ F dV = F ⋅ nˆ dS

V S

donde n̂ es la normal a la superficie y apunta hacia fuera.


S (2)
n̂ ( 2 )
x12 + x 22 ≤ 1 x3

r r
r =1 r S (1)

h =1 r n̂ (1)
x

x2

x1 n̂ (3) S ( 3)

Figura 1.8.
r
Cálculo de ∫ ∇ xr ⋅ F dV :
V
r
[
∇ xr ⋅ F = Fi ,i = xi + ( x 32 − x3 )δ i 3 ],i = xi ,i + ( x32 − x3 ) ,i δ i 3 = δ ii + ( x32 − x3 ) ,3
= 3 + (2 x3 − 1) = 2 x3 + 2
Luego:
x3 =1
r
∫ ∇ xr ⋅ F dV = (2 x3 + 2) dV =
∫ ∫A x ∫=(02 x 3 ∫
+ 2)dx3 dA = 3 dA = 3(πr 2 ) = 3π
V V 3 A

donde A viene definido por el círculo x12 + x 22 ≤ 1 .


r
Cálculo de ∫ F ⋅ nˆ dS
S

Universidad de Castilla- La Mancha Draft Por: Eduardo W. V. Chaves (2012)


Ciudad Real - España
1 TENSORES 113

Separamos la frontera en tres superficies: S (1) , S ( 2) , S (3) , ver Figura 1.8. Luego,
r r r r
∫ F ⋅ nˆ dS = ∫ F ⋅ nˆ ∫ F ⋅ nˆ ∫ F ⋅ nˆ
(1)
dS (1) + ( 2)
dS ( 2 ) + ( 3)
dS (3)
S S ( 1) S ( 2) S ( 3)
r
Las componentes explícitas
son: F1 = x1 + ( x32 − x3 )δ 13 = x1 ,
de F2 = x 2 ,
F
r
F3 = x3 + ( x3 − x 3 )δ 33 = x3 . La representación de F en la base Cartesiana viene dada por:
2 2
r
F = x1eˆ 1 + x 2 eˆ 2 + x32 eˆ 3 . Las normales correspondientes a cada superficie vienen definidas a
continuación:
r 1
nˆ (1) // r ⇒ nˆ (1) = ( x1 eˆ 1 + x 2 eˆ 2 ) ; nˆ ( 2 ) = eˆ 3 ; nˆ (3) = −eˆ 3
x12 + x 22

En la superficie S (1) se cumple que:


r 1
∫ F ⋅ nˆ (1) dS (1) = ∫ ( x eˆ 1 1 + x 2 eˆ 2 + x 32 eˆ 3 ) ⋅ ( x1 eˆ 1 + x 2 eˆ 2 )dS (1)
S ( 1) S ( 1) x12 + x 22
x12 + x 22
∫ dS (1) = ∫ 1dS
(1)
= = 2πrh = 2π
S ( 1) x12 + x 22 S (1 )

donde hemos considerado el área del cilindro ( 2πrh = 2π ).


En la superficie S ( 2) se cumple que x3 = 1 :
r
∫ F ⋅ nˆ ∫ ( x eˆ + x 2 eˆ 2 + 1eˆ 3 ) ⋅ (eˆ 3 )dS ( 2 ) = ∫ 1dS
( 2)
dS ( 2 ) = 1 1
(2)
= πr 2 = π
S (2) S (2) S (2)

donde hemos considerado el área del círculo ( πr 2 = π ).


En la superficie S (3) se cumple que x3 = 0 :
r
∫ F ⋅ nˆ ∫ ( x eˆ + x 2 eˆ 2 + 0eˆ 3 ) ⋅ (−eˆ 3 )dS (3) = ∫ 0dS
( 3)
dS (3) = 1 1
( 3)
=0
(3) (3) (3)
S S S
r r r r
Con lo cual: ∫ F ⋅ nˆ dS = ∫ F ⋅ nˆ (1) dS (1) + ∫ F ⋅ nˆ ( 2 ) dS ( 2 ) + ∫ F ⋅ nˆ
( 3)
dS (3) = 3π
S S (1 ) S (2) S ( 3)
r r
Luego, comprobando así el Teorema de la divergencia: ∫ ∇ xr ⋅ F dV = ∫ F ⋅ nˆ dS = 3π .
V S

Ejemplo 1.115
Sea un dominio de área Ω delimitado por el contorno Γ como muestra figura abajo:



x2

x1

Universidad de Castilla- La Mancha Draft Por: Eduardo W. V. Chaves (2012)


Ciudad Real - España
114 PROBLEMAS RESUELTOS DE MECÁNICA DEL MEDIO CONTINUO

Considérese también que m es un campo tensorial de segundo orden y ω un campo escalar.


Demostrar que se cumple la siguiente relación:

∫ [m : ∇

x (∇ x ω )
r r ]dΩ = ∫ [(∇ xr ω ) ⋅ m] ⋅ nˆ dΓ − ∫ [(∇ xr ⋅ m)∇ xr ∇ xr ω ]dΩ
Γ Ω

Solución: Se puede aplicar directamente la definición de integración por partes para la


demostración. Pero partiremos de la definición del teorema de la divergencia. Luego dado un
r
tensor v se cumple que:
r r
∫Ω ∇ r
x ⋅v dΩ = ∫Γ v ⋅ nˆ dΓ → Ω∫ v
indicial
j, j ∫
dΩ = v j nˆ j dΓ
Γ
r r
Pero si consideramos que el tensor v es el resultante de la operación v = ∇ xr ω ⋅ m y lo
equivalente en notación indicial v j = ω , i m ij y reemplazándolo en la expresión anterior
obtenemos que:

∫Ω v j, j ∫
dΩ = v j nˆ j dΓ
Γ
⇒ ∫Ω [ω, i m ij ],j
Γ

dV = ω , i m ij nˆ j dΓ

⇒ ∫ [ω,

ij ] ∫
m ij + ω , i m ij , j dΩ = ω , i m ij nˆ j dΓ
Γ

⇒ ∫Ω [ω, ij ] ∫
m ij dΩ = ω , i m ij nˆ j dΓ −
Γ
∫Ω [ω, i ]
m ij , j dΩ

Lo equivalente en notación tensorial:

∫ [m : ∇

x (∇ x ω )
r r ]dΩ = ∫ [(∇ xr ω ) ⋅ m] ⋅ nˆ dΓ − ∫ [∇ xr ω ⋅ (∇ xr ⋅ m)]dΩ
Γ Ω c.q.d.
NOTA: Si consideramos ahora un dominio de volumen V delimitado por una superficie S
r
con normal n̂ y sea N un vector y T un escalar también se cumple que:

∫ N T,
V
i ij ∫
dV = N i T , i nˆ j dS − N i , j T , i dV
S V

r r r
⇒ N ⋅ ∇ xr (∇ xr T )dV = (∇ xr T ⋅ N ) ⊗ n̂dS − ∇ xr T ⋅ ∇ xr NdV
∫ ∫ ∫
V S V

donde hemos aplicado directamente la definición de integración por partes.

Ejemplo 1.116
r r r
Si un campo vectorial se define como: b = ∇ xr ∧ v , probar que:

∫ λb nˆ
S
i i ∫
d S = λ, i  ijk v k , j dV = λ, i b i dV
V

V
r r
donde λ es una función únicamente de x , i.e., λ = λ( x ) .
r r r
Solución1: Si b = ∇ xr ∧ v , luego b i =  ijk v k , j . Reemplazando en la integral de superficie anterior
resulta:

Universidad de Castilla- La Mancha Draft Por: Eduardo W. V. Chaves (2012)


Ciudad Real - España
1 TENSORES 115

∫ λb nˆ
S
i i ∫
dS = λ ijk v k , j nˆ i dS
S

Aplicando el teorema de la divergencia de Gauss, resulta:

∫ λb nˆ
S
i i ∫
dS = λ ijk v k , j nˆ i dS
S

= ( ijk λv k , j ), i dV
V


= ( ijk λ, i v k , j +  ijk λv k , ji ) dV
V

∫ 424 3 1424 3 ∫
= (λ, i  ijk v k , j + λ  ijk v k , ji ) dV = λ, i b i dV
1
V bi 0 V c.q.d.
Solución 2:

∫ λb nˆ
S
i i ∫
dS = (λb i ), i dV = (λ, i b i + λb i , i ) dV
V

V

como b i =  ijk v k , j ⇒ b i ,i =  ijk v k , ji =  ijk v k ,ij = 0

∫ λb nˆ
S
i i ∫
dS = λ, i b i dV = λ, i  ijk v k , j dV
V

V

Ejemplo 1.117
Sea un dominio de volumen V delimitado por la superficie S . a) Demostrar que:
r r
∫ ( x ⊗ nˆ + nˆ ⊗ x) dS = 2V 1
S

donde n̂ es el versor normal exterior a la superficie S . b) Demostrar también que:


r r
∫ (∇ r
x ⋅ σ ) ⊗ x dV = ∫ (σ ⋅ nˆ ) ⊗ x ∫
dS − σ dV
V S V

∫σ
V
ik , k ∫
x j dV = σ ik nˆ k x j dS − σ ij dV
S V

y
r r
∫ x ⊗ (∇ r
x ⋅ σ ) dV = ∫ x ⊗ (σ ⋅ nˆ ) ∫
dS − σ T dV
V S V

∫x σ
V
i jk , k ∫
dV = xi σ jk nˆ k dS − σ ji dV
S

V

donde σ es un tensor de segundo orden arbitrario.

Universidad de Castilla- La Mancha Draft Por: Eduardo W. V. Chaves (2012)


Ciudad Real - España
116 PROBLEMAS RESUELTOS DE MECÁNICA DEL MEDIO CONTINUO

S r
dS = n̂dS
x2 n̂
V dS
B

r
x

x1
x3

Solución:
a) Teniendo en cuenta solo el primer término del integrando, podemos decir que:
r r r
∫ ( x ⊗ nˆ ) dS = ∫ ( x ⊗ 1 ⋅ nˆ ) dS = ∫ ( x ⊗ 1) ⋅ nˆ dS
S S S

Aplicando el teorema de la divergencia obtenemos que:


r r r
∫ ( x ⊗ nˆ ) dS = ∫ ( x ⊗ 1) ⋅ nˆ dS = ∫ ∇
S S V
r
x ⋅ ( x ⊗ 1) dV

Seguiremos el desarrollo en notación indicial:

∫ x nˆ
S
i j ∫
dS = x i δ jk nˆ k dS = (δ jk x i ) ,k dV
S V


= (δ jk ,k xi + δ jk x i ,k ) dV
V

Teniendo en cuenta que δ jk ,k = 0 j , xi ,k = δ ik , concluimos que:

∫ x n̂
S
i j ∫
dS = δ ji dV = δ ji dV = δ jiV
V

V

r
∫ ( x ⊗ nˆ ) dS = V 1
T
= V1
S

r
Análogamente, concluimos que ∫ (nˆ ⊗ x ) dS = V 1 . Con lo cual es cierto que:
S

r r
∫ ( x ⊗ nˆ + nˆ ⊗ x) dS = 2V 1
S

b) Verifiquemos que se cumple que

Universidad de Castilla- La Mancha Draft Por: Eduardo W. V. Chaves (2012)


Ciudad Real - España
1 TENSORES 117

( x j σ ik ) ,k = x j ,k σ ik + x j σ ik ,k
{
= δ jk

⇒ x j σ ik ,k = ( x j σ ik ) ,k − σ ij
r r
⇒ (∇ xr ⋅ σ ) ⊗ x = ∇ xr ⋅ (σ ⊗ x ) − σ
Con eso podemos decir que:

∫ (∇ r
x
r r
⋅ σ ) ⊗ x dV = ∫ ∇ xr ⋅ (σ ⊗ x ) dV − ∫ σ dV ∫x σ
V
j ik , k ∫
V

dV = ( x j σ ik ) ,k dV − σ ij dV
V
V V V

∫ (∇ r
x
r r
⋅ σ ) ⊗ x dV = ∫ (σ ⊗ x ) ⋅ nˆ dS − ∫ σ dV ∫x σ
V
j ik , k ∫
S

dV = x j σ ik nˆ k dS − σ ij dV
V
V S V

r
= (σ ⋅ nˆ ) ⊗ x dS − σ dV
∫ ∫ ∫
S

= (σ ik nˆ k ) x j dS − σ ij dV
V
S V

donde hemos aplicado el teorema de la divergencia a la primera integral del lado derecho de la
igualdad.
Teniendo en cuenta que
[(∇ xr ⋅ σ ) ⊗ xr ]T = [∇ xr ⋅ (σ ⊗ xr ) − σ ]T
r r T
⇒ x ⊗ (∇ xr ⋅ σ ) = [∇ xr ⋅ (σ ⊗ x )] − σ T
En indicial
xi σ jk ,k = ( xi σ jk ) ,k − σ ji

Con eso podemos decir que:

r r
∫ x ⊗ (∇ r
x ⋅ σ ) dV = ∫ [∇ xr ⋅ (σ ⊗ x )]T ∫
dV − σ T dV ∫x σ i jk , k ∫ ∫
dV = ( xi σ jk ) , k dV − σ ji dV
V V V V V V

r r
∫ x ⊗ (∇ r
x ⋅ σ ) dV = ∫ ( x ⊗ σ ) ⋅ nˆ dS − ∫ σ T dV ∫x σ i jk , k ∫ ∫
dV = ( xi σ jk )nˆ k dS − σ ji dV
V S V V S V

r
= x ⊗ (σ ⋅ nˆ ) dS − σ T dV
∫ ∫ ∫ ∫
= xi (σ jk nˆ k ) dS − σ ji dV
S V S V

Ejemplo 1.118
Sea un escalar φ que viene dado por:
GM
φ=− r
a
r r r
donde G y M son escalares y constantes, y a es el módulo del vector a ≠ 0 . a) Obtener el
r r
gradiente de φ . b) Obtener el gradiente de φ para el caso particular cuando a = x y dibujar el
campo ∇ xr φ en el espacio Cartesiano.
Solución:

Universidad de Castilla- La Mancha Draft Por: Eduardo W. V. Chaves (2012)


Ciudad Real - España
118 PROBLEMAS RESUELTOS DE MECÁNICA DEL MEDIO CONTINUO

 − GM   
(∇ xr φ ),i ≡  ∂φr  ≡ φ ,i =  r
 a
 = −GM  − 1  ( ar ) ,i
  ar 2 
(1.116)
 ∂x  ,i   ,i  
Fijemos que:
r  r r 1 1 r r
−1
r r 1 r r
−1
( a ) ,i =  ( a ⋅ a ) 2  = (a ⋅ a ) 2 ( a ⋅ a ) ,i = ( a ⋅ a ) 2 ( a k a k ) ,i
  ,i 2 2
−1 −1
1 r r 2 r r 1
= (a ⋅ a ) ( a k ,i a k + a k a k ,i ) = (a ⋅ a ) 2 ( a k ,i a k ) = r ( a k , i a k )
2 a
o en notación tensorial:
r 1 r r
∇ xr ( a ) = r (a ⋅ ∇ xr a ) (1.117)
a

Luego, la expresión (1.116) queda:


 −1  r
(∇ xr φ ),i ≡  ∂φr  ≡ φ ,i = −GM  r 2  ( a ) ,i =
 a 
 ∂x  ,i  
 1  1 GM
= GM  r 2  r (a k ,i a k ) = r 3 (a k ,i a k ) (1.118)
 a  a a
 
GM r r
= r 3 (a ⋅ ∇ xr a ) i
a
r
r a
Además, teniendo en cuenta que el versor según la dirección de a viene dado por a = r , aún
ˆ
a
podemos decir que:

(∇ xr φ ),i = GM r r
r GM r
r
r 3 (a ⋅ ∇ x a ) i = r 2 (aˆ ⋅ ∇ x a ) i (1.119)
a a
r r
b) Para el caso particular cuando a = x tenemos que:
r 1 1 1 r
( x ) ,i = r ( x k ,i x k ) = r (δ ki x k ) = r ( xi ) donde r = x = x12 + x 22 + x32
x x x
o en notación tensorial:
r 1 r r 1 r 1 r
∇ xr ( x ) = r ( x ⋅ ∇ xr x ) = r ( x ⋅ 1) = r ( x ) = xˆ
x x x

Con lo cual
 − GM    r
(∇ xr φ )i ≡  ∂φr ≡ φ ,i =  r
 x
 = −GM  − 1
  xr 2
 ( x ) = GM ( xr )
 ,i r3 i (1.120)
 ∂x  i   ,i   x

o en notación tensorial:
 − GM  GM r GM
∇ xr φ = ∇ xr  r = x = r 2 xˆ (1.121)
 x  xr 3 x
 

Universidad de Castilla- La Mancha Draft Por: Eduardo W. V. Chaves (2012)


Ciudad Real - España
1 TENSORES 119

Observar que el campo vectorial ∇ xr φ es radial, i.e. es normal a las superficies de las esferas
r r 2
definidas por x y disminuye con x = r 2 . La ecuación (1.121) también puede ser reescrita
como:
r  − GM  GM ˆ ∂  − GM  ˆ ∂φ(r ) ˆ
∇φ = −b = ∇  = 2 r =  r = r = φ ′(r )rˆ (1.122)
 r  r ∂r  r  ∂r

x3

Esferas

x̂ ∇ xr φ
xˆ = 1

r
x

r x2
b
∇ xr φ
∇φ

x1
∇ xr φ

Figura 1.9
GM
NOTA: Este ejemplo φ = − r representa el potencial gravitacional que tiene la siguiente
x
r m3
propiedad b = −∇ xr φ , ver Figura 1.9, donde G = 6,67384 × 10 −11 es la constante
kg s 2
gravitacional, M es la masa total del planeta. Verificamos las unidades:
  m 3 kg kg m m N m J
[φ] = − GM
r = = 2 = = ( Unidad de energía por unidad de masa )
 x  kg s 2 m s kg kg kg (energía específica)

[br ]= [− ∇ φ] =  ∂∂φxr  = mJkg = mN kgm = skgmkg = sm (Unidad(unidad


r
x 2 2
de fuerza por unidad de masa)
de aceleración)

r r r r
Es interesante comprobar también que ∇ xr ∧ b = ∇ xr ∧ [− ∇ xr φ ] = 0 , ver Ejemplo 1.104.
r
Podemos obtener b en la superficie de la Tierra a través de

Universidad de Castilla- La Mancha Draft Por: Eduardo W. V. Chaves (2012)


Ciudad Real - España
120 PROBLEMAS RESUELTOS DE MECÁNICA DEL MEDIO CONTINUO

r GM
b = −∇ xr φ = − r 2 xˆ
x

donde la masa total de la Tierra es M ≈ 5,98 × 10 24 kg y el radio aproximado R ≈ 6,37 × 10 6 m ,


resultando
r GM GM
b = − r 2 xˆ = − 2 xˆ ≈ −9,82 xˆ
x R
r m
su módulo denotamos por g = b ≈ 9,82 .
s2
r
Adoptando por x ′ el sistema que tiene su origen en el centro del cuerpo de masa M , e
r r
invocando la ley de Newton ( F = ma ), podemos obtener la fuerza que está sometido un
r
cuerpo de masa ( m ) que se encuentra bajo la influencia del campo gravitacional b = −∇ xr φ :
r r r GMm
F = ma = mb = − r 2 xˆ ′ (1.123)
x′
Podemos expresar la relación anterior en un sistema genérico tal y como se indica abajo:

x 2′

M x1′
r
r x′ r
x3′ F (Mm ) F (mM )
m

r
x (m )
x2 r r r
x ( M ) + x ′ = x ( m)
r r r
⇒ x ′ = x (m) − x (M )

x1
x3

r
Luego, para el sistema x la fuerza viene dada por:
r r r
GMm ( x ( m) − x ( M ) ) Ley de gravitación “universal” de
F ( mM ) = − r r (1.124)
r ( m) r ( M ) 2
x ( m) − x ( M ) Newton
x −x
r
donde utilizamos la nomenclatura F (mM ) para indicar que es la fuerza en m debido a la
influencia de M . Observar también que en M tenemos la misma fuerza en módulo y
r
dirección, pero de sentido contrario F (Mm ) .

Universidad de Castilla- La Mancha Draft Por: Eduardo W. V. Chaves (2012)


Ciudad Real - España
1 TENSORES 121

Ejemplo 1.119
1 r
Considerando que φ = donde r = x = x12 + x 22 + x32 , se pide:
r
a) Demostrar que:

[ r r
] ∂ 2φ ∂ 2φ ∂ 2φ
∇ xr ⋅ ∇ xr φ ( x − 0) ≡ ∇ 2 φ ≡ 2 + 2 + 2 = 0 Ecuación de Laplace
∂x1 ∂x 2 ∂x3
(1.125)

r r
[ ]
para r ≠ 0 . Utilizamos la nomenclatura ∇ xr φ ( x − 0) para indicar que el origen no está
incluido.
b) Dada una superficie cerrada S que contiene el origen, demostrar que:

∫ (∇ φ )⋅ nˆ dS = −4π
S
r
x (1.126)

donde n̂ es el versor normal a la superficie.


Solución:
Fue obtenido en el Ejemplo 1.118 que
 − GM  GM r GM
∇ xr φ = ∇ xr  r = x = r 2 xˆ (1.127)
 x  xr 3 x
 
Haciendo que GM = −1 obtenemos que:
 1  −1 r −1
∇ xr φ = ∇ xr  r  = r 3 x = r 2 xˆ (1.128)
 x  x x
 
o en notación indicial:
 −1 r −1
(∇ xr φ ) i =  r 3 x  = r 3 xi (1.129)
 x  x
 i
Calculando la divergencia de la relación anterior quedamos con:
−x   1 
⋅ (∇ xr φ ) = φ ,ii
x i ,i
∇ xr =  r 3i  = − r 3 − xi  r 3 
 x  x  x 
  ,i   ,i
(1.130)
x i ,i  −3 r 
= − r 3 − x i  r 4 ( x ) ,i 
x  x 
 
r 1 r
En el Ejemplo 1.118 hemos demostrado que ∇ xr ( x ) = r ( x ) y además teniendo en cuenta
x
que xi ,i = δ ii = 3 , podemos decir que:

Universidad de Castilla- La Mancha Draft Por: Eduardo W. V. Chaves (2012)


Ciudad Real - España
122 PROBLEMAS RESUELTOS DE MECÁNICA DEL MEDIO CONTINUO

 −3 r 
∇ xr ⋅ (∇ xr φ ) = − 3
r3 − x i
 r 4 ( x ) ,i 
x  x 

3  −3 x 
= − r 3 − xi  r 4 ri 
x  x x 
  (1.131)
3 3 x x
= − r 3 + ri 5i
x x
r 2
3 3x
=− r 3 + r 5 =0
x x

c) Adoptamos una esfera arbitraria de radio r cuya área de la superficie es 4πr 2 . Luego:
 
∫ (∇ φ )⋅ nˆ dS = ∫  r
−1  ⋅ nˆ dS = − 1 xˆ ⋅ nˆ dS = − 1
r
x
S x
2
ˆ
x
 r 2
x S
∫ r 2
x
∫ dS
S  S (1.132)
−1 −1
= 2 × ( Área) = 2 × (4πr 2 ) = −4π
r r
Observar que xˆ ⋅ nˆ = 1 ya que para la esfera se cumple que xˆ // nˆ .
Es interesante verificar que a través del teorema de la divergencia hay que cumplir que:

∫ ∇ ⋅ [∇ φ ]φdV = ∫ (∇ φ )⋅ nˆ dS
V
r
x
r
x
S
r
x ∫φ
V
,ii dV ∫
= φ ,i ni dS
S
(1.133)

[ r r
]
Hemos demostrado anteriormente que ∇ xr ⋅ ∇ xr φ ( x − 0) = 0 , pero eso solo es válido para
r r
todo x ≠ 0 (no está incluido el origen). Es decir, teniendo en cuenta el resultado (1.132), y
r r
para que (1.133) tenga consistencia, en x = 0 tenemos una fuente (manantial o sumidero) e
igual a ( − 4π ). Con eso es muy intuitivo concluir que cualquier superficie cerrada que no
∫(
contenga el origen se cumple que ∇ xr φ ⋅ nˆ dS = 0 .
S
)

Ejemplo 1.120
a) Demostrar que:

∫ (∇φ ) ⋅ nˆ dS = 4πGM (r )
S
(1.134)

 − GM 
donde φ =   es el potencial gravitacional, y M (r ) es la masa total contenida en la
 r 
esfera de radio r , donde la superficie de contorno de la esfera denotamos por S .
b) Considerando un planeta que tiene forma de esfera de radio r = a , obtener la masa total del
planeta en función de la densidad de masa, donde la densidad de masa es función del radio, i.e.
ρ = ρ (r ) .
c) Obtener el potencial gravitacional para r < a y r ≥ a . En este apartado considerar la
densidad de masa constante en el planeta ρ = ρ 0
Solución:
a) En el Ejemplo 1.119 hemos demostrado que:

Universidad de Castilla- La Mancha Draft Por: Eduardo W. V. Chaves (2012)


Ciudad Real - España
1 TENSORES 123

∫ (∇ φ )⋅ nˆ dS = ∫ ∇ r  ⋅ nˆ dS = −4π


  1 
(1.135)
S S

Multiplicamos ambos lados de la igualdad por GM (r ) obtenemos que:


  1 
− GM (r ) ∇   ⋅ nˆ dS = 4πGM (r )
∫ r
S   

  − GM (r )  

⇒ ∇    ⋅ nˆ dS = 4πGM (r ) (1.136)
S   r 
⇒ ∫ [∇φ]⋅ nˆ dS = 4πGM (r )
S

b)

Planeta esférico

r=a

ρ (r )
r

La masa total viene definida por:


M = ρ (r )dV
V
(1.137)

Notar que V = 43 πr 3 ⇒ dV = 43 π3r 2 dr = 4πr 2 dr . Con lo cual:


r =a


M = ρ (r )dV = ∫ ρ(r )4πr
2
dr (1.138)
V r =0

c) Recordar que en el Ejemplo 1.118 (ver ecuación (1.122)) hemos obtenido que
r  − GM  GM ˆ ∂  − GM  ˆ ∂φ(r ) ˆ
∇φ = −b = ∇  = 2 r =  r = r = φ ′(r )rˆ (1.139)
 r  r ∂r  r  ∂r
Utilizando la ecuación (1.136) podemos decir que:

Universidad de Castilla- La Mancha Draft Por: Eduardo W. V. Chaves (2012)


Ciudad Real - España
124 PROBLEMAS RESUELTOS DE MECÁNICA DEL MEDIO CONTINUO

∫ [∇φ] ⋅ nˆ dS = 4πGM (r )
S
r
⇒ − b ⋅ nˆ dS = φ ′(r ) rˆ ⋅ nˆ dS = φ ′(r ) dS = φ ′(r )(4πr 2 ) = 4πGM (r )
∫ ∫ ∫
123
S S =1 S (1.140)
⇒ φ ′(r )r = GM (r )
2

GM (r )
⇒ φ ′(r ) =
r2
donde M (r ) = Vρ 0 = 43 πr 3 ρ 0 . Luego:
GM (r ) 4Gπρ 0
φ ′(r ) = = r
r2 3
dφ(r ) 4Gπρ 0
⇒ = r (1.141)
dr 3
4Gπρ 0
⇒ dφ ( r ) = rdr
3
Integrando la ecuación anterior obtenemos que:
4Gπρ 0
∫ dφ = ∫ 3
rdr =

4Gπρ 0 r 2
⇒ φ(r ) = +C (1.142)
3 2
2Gπρ 0 2
⇒ φ (1) (r ) = r +C
3
donde hemos denotado que φ (r ) = φ (1) (r ) para r < a . Para valores de r ≥ a el potencial
gravitacional viene dado por
− GM − 4Gπa 3 ρ 0
φ= = = φ (2) ; r≥a (1.143)
r 3r
donde M es la masa total del planeta cuyo valor es M = Vρ 0 = 43 πa 3 ρ 0 . Notar que el potencial
φ tiene que ser continuo en r = a , (ver Parker (2003)), con lo cual:
φ (1) (r = a ) = φ ( 2) (r = a )
2Gπρ 0 2 − 4Gπa 3 ρ 0
⇒ a +C =
3 3a (1.144)
− 2Gπa ρ 0 − 2Gπa 3 ρ 0 4 3 − 2GM 3 − 3MG
3
⇒C = = = =
a a 34 a 4 2a
Con lo cual la ecuación (1.142) queda
2Gπρ 0 2 2Gπρ 0 2 3MG MG 2 3MG MG  r 2 3
φ (1) (r ) = r +C = r − = 3r − = 2  2 −  (1.145)
3 3 2a 2a 2a 2a  2a 2
resumimos:
 MG  r 2 3
φ(r ) =  −  para r<a
2 
 2a  2a 2
2
 (1.146)
 MG
φ(r ) = r
para r≥a

Universidad de Castilla- La Mancha Draft Por: Eduardo W. V. Chaves (2012)


Ciudad Real - España
1 TENSORES 125

φ (r )

superficie del planeta

− MG
a
punto de inflexión

− 3MG
2a

Figura 1.10: Potencial gravitacional vs. radio.

Figura 1.11: Potencial gravitacional (Ref. Wikipedia: “Gravitational potential”).

Ejemplo 1.121
a) Demostrar que la órbita de un planeta se realiza en un plano. b) Demostrar las Leyes de
Kepler, (Johannes Kepler (1571-1630):
b.1) Primera Ley de Kepler (1609): La órbita de cada planeta es una elipse, teniendo el Sol en unos
de los focos de la elipse;
b.2) Segunda Ley de Kepler (1609): El vector posición Sol=>Planeta en movimiento describe una
área a una tasa constante;
b.3) Tercera Ley de Kepler (1618): Si T (periodo orbital) representa el tiempo necesario para que
un planeta realice una vez su órbita elíptica, cuyo eje mayor es 2a , se cumple que T 2 = κa 3 ,
donde κ es una constante.

Universidad de Castilla- La Mancha Draft Por: Eduardo W. V. Chaves (2012)


Ciudad Real - España
126 PROBLEMAS RESUELTOS DE MECÁNICA DEL MEDIO CONTINUO

Recordatorio: Expresiones relacionados con la elipse

x2

r
x
b
θ f2

f1 x1
b

a a

r p
Ecuación de la elipse: x = r =
1 + e cos θ

a 2 − b2 p2
Excentricidad: e = ; 0 < e < 1 , donde se cumple que a 2 = .
a2 (1 − e 2 ) 2
Área de la elipse: A = πab
Solución:

M - masa del Sol


m - masa del planeta
r
x
xˆ = r
x
x2 r r r
r dx r d 2 x dv
v= , a= 2 =
x3 r r r dt dt dt
r c = x∧v
dx r r
=v a // xˆ
dt

r r r
x F // a
θ t=0
Sol r
h x1

Figura 1.12: Órbita del planeta.

a) Para demostrar que la órbita se realiza en un plano tenemos que demostrar que la normal
r r
formada por el plano x̂ y v no cambia con el tiempo, es decir, si el vector c no varía con el
tiempo, i.e. es una constante.
En el Ejemplo 1.118 en la ecuación (1.123) hemos demostrado que:

Universidad de Castilla- La Mancha Draft Por: Eduardo W. V. Chaves (2012)


Ciudad Real - España
1 TENSORES 127

r r r GMm r GM
F = ma = mb = − r 2 xˆ ; a = − r 2 xˆ (1.147)
x x
r r r
Obtenemos la tasa del vector c = x ∧ v :
r
dc d r r d r r r d r r r r r r
= ( x ∧ v ) = ( x ) ∧ v + x ∧ (v ) = v12
∧3v + 1∧3
x2 a =0
dt dt dt dt r
=0
r
=0
r r r
Con lo cual hemos demostrado que el vector c = x ∧ v no varía con el tiempo, implicando
que la órbita se realiza en un plano.
b.1) Primera Ley de Kepler
Ya que la órbita del planeta se realiza en un plano, adoptamos como dicho plano el x1 − x 2 ,
r
luego el vector c tiene misma dirección que x3 , ver Figura 1.12.
r
Expresamos c en función de x̂ .
r r
r dx d r d( x ) r dxˆ
v= = ( x x) =
ˆ xˆ + x
dt dt dt dt
y
r
r r r r  d( x ) r dxˆ 
c = x ∧ v = ( x xˆ ) ∧  xˆ + x =
 dt dt 
 
r
r d( x ) r 2 dxˆ
= x 1 ∧3xˆ + x xˆ ∧
xˆ 2
dt r
=0
dt
r 2 dxˆ
= x xˆ ∧
dt
r GM r r
Teniendo en cuenta que a = − r 2
xˆ , calculamos el vector a ∧ c que tiene misma dirección
x
r r r r
que v , i.e. (a ∧ c ) // v :

r r  GM   r 2 dxˆ   dxˆ   dxˆ dxˆ 


a ∧ c =  − r 2 xˆ  ∧  x xˆ ∧  = −GM xˆ ∧  xˆ ∧  = −GM ( xˆ ⋅ ) xˆ − ( xˆ ⋅ xˆ ) 
 x   dt   dt   dt dt 
 
dxˆ
= GM
dt
r r r
( )
r r r r r r
donde hemos utilizando la propiedad a ∧ b ∧ c = (a ⋅ c )b − (a ⋅ b)c , ver Ejemplo 1.16. Notar
dxˆ dxˆ
también que se cumple que xˆ ⋅ = 0 ya que xˆ ⊥ siempre se cumple, y xˆ ⋅ xˆ = xˆ = 1 .
2

dt dt
Teniendo en cuenta que GM es una constante, también se cumple que:
r r dxˆ d (GM xˆ )
a ∧ c = GM =
dt dt
r
Ya que el vector c no varía con el tiempo, también se cumple que:
r r r
r r dv r d ( v ∧ c )
a∧c = ∧c =
dt dt
Luego:

Universidad de Castilla- La Mancha Draft Por: Eduardo W. V. Chaves (2012)


Ciudad Real - España
128 PROBLEMAS RESUELTOS DE MECÁNICA DEL MEDIO CONTINUO

r r
d (v ∧ c ) d (GM xˆ )
=
dt dt
Integrando en el tiempo la ecuación anterior obtenemos que:
r r r
v ∧ c = GM xˆ + h
r r
donde h es el vector constante de integración y no depende del tiempo. Notar que h se
r r
encuentra en el plano x1 − x 2 , ya que (v ∧ c ) y x̂ también se encuentran en el plano x1 − x 2 ,
ver Figura 1.12. Calculamos entonces:
r r
xˆ ⋅ h = xˆ h cos θ = h cos θ
r
donde hemos denotado por h = h . Con lo cual:
r r r r r r r r r
= c ⋅ c = ( x ∧ v ) ⋅ c = (v ∧ c ) ⋅ x
2
c2 = c
( r
) r r r r r r
= GM xˆ + h ⋅ ( x xˆ ) = x GM xˆ ⋅ xˆ + x h ⋅ xˆ = x GM + x h cos θ
r
= x (GM + h cos θ)
= r (GM + h cos θ)
r
donde hemos considerado que r = x . Luego, obtenemos la siguiente ecuación de la elipse:
c2
c2 GM p
⇒r= = =
(GM + h cos θ) (GM + h cos θ ) 1 + e cos θ
GM
donde hemos considerado que:
c2 h
p= y e= (1.148)
GM GM
b.2) Segunda Ley de Kepler

r 1 r r 1 r r
A = x ∧ ∆S ∆
S →0
→ dA = x ∧ ds
2 2
x2
∆S

A
r
x

x1

r
La tasa de dA queda:

Universidad de Castilla- La Mancha Draft Por: Eduardo W. V. Chaves (2012)


Ciudad Real - España
1 TENSORES 129

r r r r r
D (dA) 1 D( x ∧ ds ) 1 D( x ) r 1 r D ( ds )
= = ∧ ds + x ∧
Dt 2 Dt 2 Dt 2 Dt
r
1 D( x ) r 1 r r
= ∧ ds + x∧v
2 1Dt
4243 2
r
=0

1r
= c (constante)
2
y su módulo:
r
D (dA) D(dA) 1 r 1
= = c = c
Dt Dt 2 2

NOTA: Como consecuencia de la segunda ley tenemos que si las áreas de dos sectores son
iguales el tiempo necesario para recoger sus trayectos serán iguales. Luego, según la Figura
1.13 si las áreas de los sectores OCD y EFO son iguales los tiempos recogido de C → D y
E → F también serán iguales. Como consecuencia, el planeta cuando esté más cerca del Sol
tendrá mayor velocidad que cuando esté más alejado.

sector EFO sector OCD


E
D
O
A A

C
F

Figura 1.13: Órbita del planeta.

b.3) Tercera Ley de Kepler


Si T es el tiempo total para una órbita completa (periodo orbital), obtenemos que:
T T
D (dA) 1 1
A=
0
∫ Dt
dt =
0
2 ∫
c dt = cT
2

1
Teniendo en cuenta el área de la elipse: A = πab , concluimos que cT = πab , con lo cual:
2
2πab 4π 2 a 2 b 2
T= ⇒ T2 = (1.149)
c c2
A través de las relaciones de la elipse se cumple que:
a 2 − b2
e= ⇒ b2 = a 2 − a 2e2 ⇒ b 2 = a 2 (1 − e 2 )
a2

Universidad de Castilla- La Mancha Draft Por: Eduardo W. V. Chaves (2012)


Ciudad Real - España
130 PROBLEMAS RESUELTOS DE MECÁNICA DEL MEDIO CONTINUO

p2 p
y teniendo en cuenta que a 2 = 2 2
⇒a= 2
⇒ (1 − e 2 )a = p en la relación anterior,
(1 − e ) (1 − e )
podemos decir que:
b2
b 2 = a 2 (1 − e 2 ) ⇒ b 2 = ap ⇒ p=
a
Con lo cual la ecuación (1.149) puede ser reescrita como:
4π 2 a 2 b 2 4π 2 a 2 ab 2 4π 2 a 3 p 4π 2 3
T2 = = = = a = κ a3 (1.150)
c2 c 2a c2 GM
p 1
donde hemos tenido en cuenta que 2
= , ver ecuación (1.148).
c GM

Universidad de Castilla- La Mancha Draft Por: Eduardo W. V. Chaves (2012)


Ciudad Real - España
1 TENSORES 131

1.2 Ejercicios Propuestos

Problema 1.1
Obtener el resultado de las siguientes expresiones, si dichas expresiones SON CORRECTAS:
• δ ii δ kl
• δ ij δ ij
• δ ij δ ik δ ij

Problema 1.2
Demostrar que:
a) δ ii = 3 ;
b) δ ij δ ij = 3 ;
c)  ijk  jki = 6 ;
d)  ijk A j Ak = 0 ;
e) δ ij δ jk = δ ik ;
f) δ ij  ijk = 0 .

Problema 1.3
Utilizando las propiedades del operador de permutación (  ijk ) y de la delta de Kronecker ( δ ij ),
obtener el resultado de las siguientes expresiones:
a)  3 jk a j a k
b)  ijk δ kj
c)  ijk a 2Tkj
d) 1 jk δ 3 j v k

Problema 1.4
Utilizando notación indicial probar que:
r r r r r r r r r r r r
(s ∧ t ) ⋅ (u ∧ v ) = (s ⋅ u)(t ⋅ v ) − (s ⋅ v )(t ⋅ u) (1.151)

Universidad de Castilla- La Mancha Draft Por: Eduardo W. V. Chaves (2012)


Ciudad Real - España
132 PROBLEMAS RESUELTOS DE MECÁNICA DEL MEDIO CONTINUO

Problema 1.5
Dadas las componentes del tensor A :
1 1 0
A ij = 1 2 2 (1.152)
0 2 3

Obtener los siguientes valores:


a) A ii ;
b) A ij A ij ;
c) A ij A jk para i = 1 , k = 1 y para i = 1 , k = 2 .

Problema 1.6
Dadas las componentes del tensor R :
 12 9 4
 25 −
25 5
3 4 
R ij =  − 0 (1.153)
5 5 
 16 12 3
 25 25 5 

¿ El tensor R es un tensor ortogonal propio? Demostrar.

Problema 1.7
Encontrar la matriz de rotación A que describe primero una rotación sobre el eje x1 de 90º ,
y a continuación efectúa una rotación de 45º sobre el eje rotado x3′ .

Problema 1.8
1 1 ˆ
Dadas dos bases cartesianas (eˆ i ) y (eˆ *i ) , donde eˆ 1* = (2eˆ 1 + 2eˆ 2 + eˆ 3 ) y eˆ *2 = (e1 − eˆ 2 ) .
3 2
Se pide:
a) Expresar ê*3 en función de ê i ;
b) Expresar ê i en función de eˆ *i ;
r r
c) Si v = 6eˆ 1 − 6eˆ 2 + 12eˆ 3 encontrar v i* .

Problema 1.9
La siguiente tabla muestra los ángulos entre los ejes originales xi y los ejes transformados xi* :

Universidad de Castilla- La Mancha Draft Por: Eduardo W. V. Chaves (2012)


Ciudad Real - España
1 TENSORES 133

x1 x2 x3

x1* 135º 60º 120 º

x 2* 90º 45º 45º

x3* 45º 60º 120 º

a) Encontrar la matriz de transformación A ;


b) Si B es un tensor de segundo orden dado por sus componentes en la base xi :

 3 − 4 2
B ij =  − 4 0 1  (1.154)
 2 1 3

Encontrar las componentes de B *ij con respecto a los ejes rotados xi* .

Problema 1.10
Utilizando notación indicial probar que:
r
a) ∇ ∧ ∇φ = 0 ;
r
b) ∇ ⋅ ∇ ∧ v = 0 .

Problema 1.11
Si φ ( x1 , x 2 , x3 ) = a ij x i x j , con aij constante. Demostrar que:

φ, i = (a ij + a ji ) x j (1.155)
φ, ij = aij + a ji (1.156)

Problema 1.12
Demostrar que:
∇ 2 (φψ ) = φ∇ 2 ψ + 2(∇φ) ⋅ (∇ψ ) + ψ∇ 2 φ (1.157)

Problema 1.13

Probar que si σ y D son tensores de segundo orden la siguiente relación es válida:


σ : D = Tr (σ ⋅ D T ) (1.158)

Problema 1.14

Universidad de Castilla- La Mancha Draft Por: Eduardo W. V. Chaves (2012)


Ciudad Real - España
134 PROBLEMAS RESUELTOS DE MECÁNICA DEL MEDIO CONTINUO

Si T y D son tensores de segundo orden ¿La relación


T :D = D : T (1.159)
es válida siempre?
En caso positivo, ¿Cuándo la relación anterior no es válida?
En caso negativo, ¿Cuándo la relación anterior es válida?

Problema 1.15
Si T , D y S son tensores de segundo orden y n un vector, ¿se puede afirmar que las
relaciones
1) T : D = D : T
2) T ⋅ n = n ⋅ T
3) Tr ( T ⋅ D ⋅ S) = Tij D jk S kl
son siempre válidas?. Razona la respuesta.

Problema 1.16
a) Demostrar que
r r
( r r
det 1 + a ⊗ b = 1 + a ⋅ b )
r r
donde 1 es el tensor identidad de segundo orden, y a , b son vectores.
b) Verificar si se cumple la relación (demostrar):

[(ar ⊗ br )⋅ (cr ⊗ dr ): 1] = (br ⋅ cr )(ar ⋅ dr )


Problema 1.17
Demostrar que tras una transformación ortogonal aplicadas a vectores, estos preservan su
módulo y preserva los ángulos entre vectores.

Problema 1.18
Considérense los siguientes tensores:
1
K ijkl = δ ij δ kl ; J ijkl = I ijkl − K ijkl
3
donde δ ij son las componentes del tensor identidad de segundo orden 1 , y

I ijkl =
1
2
( )
δ ik δ jl + δ il δ jk es la parte simétrica del tensor identidad de cuarto orden.

Demuestre que el resultado del doble producto escalar de cualquier tensor de segundo orden
simétrico A con:
a) el tensor J resulta ser la parte desviadora de A ;
b) el tensor K resulta ser la parte esférica de A .

Universidad de Castilla- La Mancha Draft Por: Eduardo W. V. Chaves (2012)


Ciudad Real - España
1 TENSORES 135

Problema 1.19
Demostrar que tras una transformación ortogonal aplicadas a vectores, estos preservan sus
módulos y preservan los ángulos entre vectores

Problema 1.20
Obtener las componentes de T ′ , cuyo tensor se obtiene por la siguiente operación:
T ′ = B ⋅ T ⋅ BT
Siendo Tij y B ij las componentes de los tensores T y B , respectivamente.
NOTA: Utilizar notación simbólica y notación indicial.

Problema 1.21
Demostrar que
(
r r
) ( r r
det Q + a ⊗ b = det (Q) 1 + Q −1 ⋅ a ⋅ b )
r r
donde Q es un tensor de segundo orden, y a , b son vectores.

Problema 1.22
Dado un tensor de segundo orden T demostrar que las siguientes expresiones son invariantes:
ƒ Tr (T )
ƒ Tr ( T ⋅ T )
ƒ det (T )

Problema 1.23
Dadas las componentes de un tensor de segundo orden:
1 1 0
A ij = 1 1 0
0 0 2

y sus valores principales:


 2 0 0
A ′ij = 0 2 0
0 0 0

Obtener los invariantes del tensor desviador del tensor A .

Problema 1.24
Cuantas componentes independientes tiene el tensor cuarto orden A si:
a) A es no simétrico;

Universidad de Castilla- La Mancha Draft Por: Eduardo W. V. Chaves (2012)


Ciudad Real - España
136 PROBLEMAS RESUELTOS DE MECÁNICA DEL MEDIO CONTINUO

b) A presenta sólo simetría menor;


c) A presenta simetría mayor y menor.

Problema 1.25
Considérese un tensor de cuarto orden C , teniendo su representación tensorial e indicial
como se sigue:
C = λ1 ⊗ 1 + 2µI
(
C ijkl = λδ ij δ kl + µ δ ik δ jl + δ il δ jk )
y su inversa viene dada por:
−λ 1
C −1 = 1 ⊗1 + I
2µ(3λ + 2µ ) 2µ
−λ
−1
C ijkl =
2µ(3λ + 2µ)
δ ij δ kl +
1

(
δ ik δ jl + δ il δ jk )
Demostrar que C : C −1 = I sym ≡ I , donde I es el tensor identidad de cuarto orden simétrico.

Problema 1.26
Considérese un tensor de cuarto orden B , cuyas componentes vienen dadas por la siguiente
expresión:
B ijkl = λδ ij δ kl + µδ ik δ jl + βδ il δ jk

donde λ , µ y β son constantes. Demostrar que el tensor B es isótropo.

Problema 1.27
El campo del tensor de tensiones de Cauchy de un medio continuo viene representado por:
 3 x1 5 x 22 0 
r  2 
σ ij ( x ) = 5 x 2 0 2 x3 
 0 2 x3 0 

Obtener las fuerzas másicas (por unidad de volumen) para que el medio continuo esté en
equilibrio.
10 x 2 + 3
Respuesta: ρb i = −  2 
 0 

Problema 1.28

Universidad de Castilla- La Mancha Draft Por: Eduardo W. V. Chaves (2012)


Ciudad Real - España
2 Cinemática del Continuo
2.1 Ejercicios Resueltos

2.1.1 Descripción del Movimiento, Derivada Material,


Velocidad, Aceleración

Ejemplo 2.1
Un continuo viene definido por un cuadrado de lado b , y está sometido a movimiento de
sólido rígido definido por una rotación antihoraria por un ángulo de 30º . Encontrar las
ecuaciones de movimiento. Obtener también la nueva posición de la partícula D .
r r
Nota: Considerar los sistemas x y X superpuestos.

X 2 , x2
x2′
C′

D′ C x1′
D
30º
B′
b
30º B
A = A′ b X 1 , x1

r r r r
Solución: Aplicamos las ecuaciones de movimiento de un sólido rígido x = c + Q ⋅ X = Q ⋅ X ,
r r
con c = 0 . Las componentes de Q son las mismas que las componentes de la matriz de
r r
transformación del sistema x ′ para el sistema x , i.e.:
cos θ − sin θ 0
Q ij =  sin θ cos θ 0
 0 0 1
Luego, las partículas vienen gobernadas por las ecuaciones de movimiento:
138 PROBLEMAS RESUELTOS DE MECÁNICA DEL MEDIO CONTINUO

 x1  cos 30º − sin 30º 0  X 1 


    
 x 2  =  sin 30º cos 30º 0  X 2 
x   0 0 1  X 3 
 3 
La partícula que inicialmente estaba en D ( X 1 = 0 , X 2 = b , X 3 = 0 ) mueve para la siguiente
posición:
 x1D  cos 30º − sin 30º 0 0 − b sin 30º 
 D     
 x 2  =  sin 30º cos 30º 0 b  =  b cos 30º 
x D   0 0 1 0  0 
 3   

Ejemplo 2.2
Un movimiento del medio continuo en la descripción material viene dado por:
 x1 = exp t X 1 − exp −t X 2
 t −t
 x 2 = exp X 1 + exp X 2 (2.1)
x = X
 3 3

para t > 0 . Encontrar las componentes de la velocidad y aceleración en coordenadas


espaciales y materiales.
Solución:
Velocidad:

r r V1 = exp t X 1 + exp − t X 2


r r Dx ( X , t ) componentes 
V ( X , t) =    →V2 = exp t X 1 − exp −t X 2 (2.2)
Dt V = 0
 3

Aceleración:
 A1 = exp t X 1 − exp −t X 2
 t −t
 A2 = exp X 1 + exp X 2 (2.3)
A = 0
 3

Para encontrar las componentes de la velocidad y la aceleración reemplazamos las


ecuaciones del movimiento:
Velocidad (descripción espacial)
v1 = x2

v2 = x1 (2.4)
v = 0
 3
Aceleración (descripción espacial)
a1 = x1 = v 2

a 2 = x 2 = v1 (2.5)
a = 0
 3

Universidad de Castilla- La Mancha Draft Por: Eduardo W. V. Chaves (2012)


Ciudad Real - España
2 CINEMÁTICA DEL CONTINUO 139

Ejemplo 2.3
El campo de velocidad de un fluido viene dado por:
r
v = x1eˆ 1 + x2 eˆ 2 + x3 eˆ 3 (2.6)
y el campo de temperatura es:
r
T ( x , t ) = 3 x 2 + x3 t (2.7)
Encontrar la tasa de cambio en el tiempo de la temperatura.
Solución:
r
La tasa de cambio de una propiedad viene dada por la derivada material T ( x , t ) :
r r
DT ∂T ( x , t ) ∂T ( x , t ) ∂T  ∂T ∂T ∂T 
= + vj = +  v1 + v2 + v3  (2.8)
Dt ∂t ∂x j ∂t  ∂x1 ∂x 2 ∂x3 
DT
= x3 + (0 × x1 + 3 × x2 + tx3 )
Dt (2.9)
= x3 + (3x2 + tx3 )

Ejemplo 2.4
Dado el movimiento:
xi = X i + 0,2tX 2 δ 1i (2.10)
y el campo de temperatura (estacionario):
r
T ( x ) = 2 x1 + x 22 (2.11)
a) Encontrar el campo de temperatura en la descripción material;
b) Encontrar la tasa de cambio de la temperatura para una partícula que en la configuración
de referencia ocupaba la posición (0,1,0) .
Solución:
Según las ecuaciones del movimiento tenemos que:
x1 = X 1 + 0,2tX 2 δ 11 = X 1 + 0,2tX 2
x 2 = X 2 + 0,2tX 2 δ 12 = X 2
x3 = X 3 + 0,2tX 2 δ 13 = X 3
Luego:
r r r
[ r 2
]
T ( x ( X , t )) = 2 x1 ( X , t ) + x 2 ( X , t )
= 2( X 1 + 0,2tX 2 ) + ( X 2 )
2

r
= 2 X 1 + ( X 2 + 0,4t )X 2 = T ( X , t )
b) La derivada material de la temperatura viene dada por:
r
DT ( X , t ) & r
≡ T ( X , t ) = 0,4 X 2
Dt

Universidad Castilla- La Mancha Draft Por: Eduardo W. V. Chaves (2012)


Ciudad Real - España
140 PROBLEMAS RESUELTOS DE MECÁNICA DEL MEDIO CONTINUO

Para la partícula ( X 1 = 0; X 2 = 1; X 3 = 0) tenemos que:


T& (( X 1 = 0; X 2 = 1; X 3 = 0), t ) = 0,4 X 2 = 0,4

Ejemplo 2.5
r r
Determinar el campo de velocidad V ( X , t ) en la descripción material y el campo de
r r
aceleración A( X , t ) de una partícula en el tiempo t en función de la tasa de cambio de los
r r
desplazamientos U ( X , t )
Solución:
r r D r r r&
V ( X , t) = U ( X , t) = U (2.12)
Dt
r r D r r r&
A( X , t ) = V ( X , t) = V =
Dt
(2.13)
D2 r r &r&
= 2 U ( X , t) = U
Dt
r& &r&
A =V = U (2.14)

Ejemplo 2.6
Considérense las siguientes ecuaciones del movimiento en la descripción Lagrangiana:
r
 x1 ( X , t ) = X 2 t 2 + X 1  x1  1 t
2
0  X 1 
 r Forma Matricial     
 x 2 ( X , t ) = X 3 t + X 2     →  x 2  = 0 1 t  X 2  (2.15)
 r  x  0 0
 x 3 ( X , t ) = X 3  3  1   X 3 

¿Es este un movimiento posible? Si así es, encontrar los campos de desplazamiento,
velocidad y aceleración en la descripción Lagrangiana y Euleriana. Considérese un partícula
P , que en el tiempo t = 0 ocupaba la posición X 1 = 2, X 2 = 1, X 3 = 3 , encontrar la
velocidad de P en los tiempos t = 1s y t = 2 s .
Solución:
El movimiento es posible si J ≠ 0 . Verificamos que el movimiento es posible:
∂x1 ∂x1 ∂x1
∂X 1 ∂X 2 ∂X 3 1 t 2 0
∂xi ∂x 2 ∂x 2 ∂x 2
J= = =0 1 t =1≠ 0
∂X j ∂X 1 ∂X 2 ∂X 3
∂x3 ∂x3 ∂x3 0 0 1
∂X 1 ∂X 2 ∂X 3
r r r
El campo vectorial de desplazamiento viene dado por la definición u = x − X . Utilizando
las ecuaciones del movimiento (2.15) obtenemos que:

Universidad de Castilla- La Mancha Draft Por: Eduardo W. V. Chaves (2012)


Ciudad Real - España
2 CINEMÁTICA DEL CONTINUO 141

u1

(Xrr , t ) = x ( Xrr, t ) − X
1 1 = X 2t 2
u 2 (Xr , t ) = x ( Xr , t ) − X
2 2 = X 3t (2.16)

u 3 (X , t ) = x ( X , t ) − X
3 3 =0

que son las componentes del desplazamiento en la descripción Lagrangiana. La velocidad y


la aceleración vienen dadas por:

V1 =
r
( )
du 1 X , t
=
d
(
X 2t 2 = 2 X 2t )  dV1
dt dt  A1 = dt = 2 X 2
 

V 2 =
r
( )
du 2 X , t d
= ( X 3t ) = X 3 ;

 A2 =
dV 2
=0 (2.17)
 dt dt  dt
( )
r
 du 3 X , t d  dV 3
V3 = = (X 2t ) = 0  A3 = dt = 0
 dt dt 

La forma inversa de (2.83) nos proporcionan las ecuaciones del movimiento en la


descripción Euleriana:
r
 X 1  1 − t 2 t 3   x1   X 1 ( x , t ) = x1 − t 2 x 2 + t 3 x 3
      r
 X 2  = 0 1 − t   x 2  ⇒  X 2 ( x , t ) = x 2 − tx 3 (2.18)
 X  0 r
 3  0 1   x 3   X 3 ( x , t ) = x 3

Luego, los campos de desplazamiento, velocidad y aceleración en la descripción Euleriana


se obtienen al reemplazar las ecuaciones (2.18) en las expresiones (2.16) y (2.17), i.e.:

(Xr ( xr , t ), t ) = X ( xr , t )t = ( x − tx )t = u ( xr , t )
r
u1 2
2
2 3
2
1

u 2 (Xr ( xr , t ), t ) = X ( xr , t )t = x t = u ( xr , t )
3 3 2 (2.19)

u 3 (X ( xr , t ), t ) = u ( xr , t ) = 0
3

(Xr ( xr , t ), t ) = 2 X ( xr , t )t = 2( x − tx )t = v ( xr , t )
r
V1 2 2 3 1

V 2 (Xr ( xr , t ), t ) = X ( xr , t ) = x = v ( xr , t )
3 3 2 (2.20)

V3 (X ( xr , t ), t ) = v ( xr , t ) = 0
3

(Xr ( xr , t ), t ) = 2 X ( xr , t ) = 2( x
r r
 A1 2 2 − tx 3 ) = a1 ( x , t )

 A2 (Xr ( xr , t ), t ) = a ( xr , t ) = 0
2 (2.21)

 A3 (X ( xr , t ), t ) = a ( xr , t ) = 0
3

Teniendo en cuenta la descripción Lagrangiana de la velocidad dada por (2.17), la velocidad


de la partícula P ( X 1 = 2, X 2 = 1, X 3 = 3 ) en el tiempo t = 1s viene dada por:
r r r
V1 ( X , t ) = 2 X 2 t = 2 m / s ; V 2 ( X , t ) = X 3 = 3m / s ; V3 ( X , t ) = 0
Observemos que en el tiempo t = 1s la partícula P ocupa una nueva posición definida por:
x1 = X 2 t 2 + X 1 = 3 ; x 2 = X 3t + X 2 = 4 ; x3 = X 3 = 3
Luego, la velocidad de la partícula P también puede ser obtenida por (2.20), i.e.:

Universidad Castilla- La Mancha Draft Por: Eduardo W. V. Chaves (2012)


Ciudad Real - España
142 PROBLEMAS RESUELTOS DE MECÁNICA DEL MEDIO CONTINUO

r
v1 ( x , t ) = 2( x 2 − tx 3 )t = 2( 4 − 1 × 3) × 1 = 2 m / s
 r
v 2 ( x , t ) = x 3 = 3m / s
v ( xr , t ) = 0
 3
Observemos que, la velocidad de la partícula es la misma sea utilizando la descripción
Lagrangiana o la Euleriana, ya que la velocidad es una propiedad intrínseca de la partícula.
La velocidad de la partícula P en el tiempo t = 2 s viene dada por:
V1 (Xrr , t ) = 2 X t = 2 × 2 × 1 = 4m / s
2

V 2 (Xr , t ) = X = 3m / s
3

V3 (X , t ) = 0
En el tiempo t = 2 s la nueva posición de la partícula P queda definida por:
r
 x1 ( X , t ) = X 2 t 2 + X 1 = 6
 r
 x2 ( X , t ) = X 3t + X 2 = 7
 r
 x 3 ( X , t ) = X 3 = 3
Como podemos verificar en la figura abajo, la descripción Lagrangiana del movimiento
r r
x ( X , t ) describe la trayectoria de la partícula P .

Trayectoria de la partícula P
r
viP ( x , t = 1s) = [2;3;0]
r
Vi P ( X P , t = 1s) = [2;3;0]
t = 1s
r Partícula P
t0 Vi ( X P , t = 2s ) = [4;3;0]
P

X iP = [2;1;3]
P
xiP = [3;4;3]

Partícula P

t = 2s
xiP = [6;7;3]

r
viP ( x , t = 2s ) = [4;3;0]

NOTA: Notar que la velocidad espacial no se puede obtener a través de


r r
DX ( x , t ) r r r
= 0 ≠ v ( x , t ) . Podemos verificar este hecho con el ejemplo propuesto:
Dt
r r r r r
DX i ( x , t ) ∂X i ( x , t )  ∂X i ( x , t ) r ∂X i ( x , t ) r ∂X i ( x , t ) r 
= + v1 ( x , t ) + v 2 ( x, t ) + v3 ( x , t )
Dt ∂t  ∂x1 ∂x 2 ∂x 3 
Luego:

Universidad de Castilla- La Mancha Draft Por: Eduardo W. V. Chaves (2012)


Ciudad Real - España
2 CINEMÁTICA DEL CONTINUO 143

r r r r r
DX 1 ( x , t ) ∂X 1 ( x , t )  ∂X 1 ( x , t ) r ∂X 1 ( x , t ) r ∂X 1 ( x , t ) r 
= + v1 ( x , t ) + v 2 ( x, t ) + v3 ( x , t )
Dt ∂t  ∂x1 ∂x 2 ∂x 3 
( ) [
= − 2tx 2 + 3t 2 x3 + 1 × 2( x 2 − tx 3 )t − t 2 × x3 + t 3 × 0 = 0 ]
r r r r r
DX 2 ( x , t ) ∂X 2 ( x, t )  ∂X 2 ( x , t ) r ∂X 2 ( x , t ) r ∂X 2 ( x, t ) r 
= + v1 ( x , t ) + v 2 ( x, t ) + v 3 ( x, t )
Dt ∂t  ∂x1 ∂x 2 ∂x3 
= (− x3 ) + [0 × 2( x 2 − tx 3 )t + 1 × x3 − t × 0] = 0
r r r r r
DX 3 ( x , t ) ∂X 3 ( x , t )  ∂X 3 ( x, t ) r ∂X 3 ( x , t ) r ∂X 3 ( x , t ) r 
= + v1 ( x , t ) + v 2 ( x, t ) + v3 ( x, t )
Dt ∂t  ∂x1 ∂x 2 ∂x3 
= (0 ) + [0 × 2( x 2 − tx 3 )t + 0 × x3 + 1 × 0] = 0
r r r
Además, teniendo en cuenta que u = x − X , se cumple que:
r r r r
r r
v ( X , t) =
Dx ( X , t ) D r r
Dt
=
Dt
( r r
u( X , t ) − X ( x , t ) = )Du( X , t ) r& r
Dt
≡ u( X , t )

Además, también se cumple que:


r r r r r r
r r r& r Du( x , t ) ∂u( x , t ) ∂u( x , t ) r r
v ( x , t ) = u( x , t ) ≡ = + r ⋅ v ( x, t )
Dt ∂t ∂x

Ejemplo 2.7
El campo de velocidad de un medio continuo, expresado en forma Euleriana es el
siguiente:
x1 2 x2 3 x3
v1 = ; v2 = ; v3 = (2.22)
1+ t 1+ t 1+ t
Se pide:
r
a) Determinar la relación entre las coordenadas espaciales y materiales xi = xi ( X , t ) ;
b) Obtener las componentes de la aceleración cuando se utiliza la descripción espacial
del movimiento.
c) Obtener las componentes de la aceleración cuando se utiliza la descripción
Lagrangiana del movimiento.
Solución:
dxi
a) Considerando que vi =
dt
dx1 x dx dt
v1 = = 1 ⇒ 1 = (2.23)
dt 1 + t x1 1 + t
1 1
∫x 1
dx1 = ∫ 1 + t dt ⇒ Lnx 1 = Ln(1 + t ) + Ln(C1 ) ⇒
(2.24)
⇒ x1 = C1 (1 + t )

La condición inicial t = 0 ⇒ x1 = X 1 implica que C1 = X 1

x1 = X 1 (1 + t ) (2.25)

Universidad Castilla- La Mancha Draft Por: Eduardo W. V. Chaves (2012)


Ciudad Real - España
144 PROBLEMAS RESUELTOS DE MECÁNICA DEL MEDIO CONTINUO

dx 2 2 x 2 dx 2dt
v2 = = ⇒ 2 = (2.26)
dt 1 + t x2 1 + t
1 2
∫x 2
dx 2 = ∫ 1 + t dt ⇒ Lnx 2 = 2Ln(1 + t ) + LnC 2 ⇒
(2.27)
2
⇒ x2 = C 2 (1 + t )

para t = 0 ⇒ x 2 = X 2 ⇒ C 2 = X 2

x2 = X 2 (1 + t ) 2 (2.28)

dx3 3 x3 dx 3dt
v3 = = ⇒ 3 = (2.29)
dt 1 + t x3 1 + t
1 3
∫x 3
dx3 = ∫ 1 + t dt ⇒ Lnx 3 = 3Ln(1 + t ) + LnC 3 ⇒
(2.30)
3
⇒ x3 = C3 (1 + t )

y para t = 0 ⇒ x3 = X 3 ⇒ C 3 = X 3

x3 = X 3 (1 + t ) 3 (2.31)
Las ecuaciones del movimiento:
 x1 = X 1 (1 + t )
 2
 x2 = X 2 (1 + t ) (2.32)
 3
 x3 = X 3 (1 + t )
r r
b) Conocido v ( x , t ) en la descripción espacial (Euleriana), podemos aplicar la
derivada material:
r r
r r ∂v ( x , t ) r r r r
a ( x, t ) = + ∇v ( x , t ) ⋅ v ( x , t ) (2.33)
∂t
∂vi
ai = + (v i , k )v k
∂t
(2.34)
∂v
a i = i + (vi ,1 v1 + vi , 2 v 2 + vi ,3 v3 )
∂t
luego,
x1  x 1 
a1 = − + 1 + 0 + 0 = 0
1 + t 1 + t
2
(1 + t ) 
2x2  2x 2  2 x2
a2 = − + 0 + 2 + 0 = (2.35)
(1 + t ) 2
 1+ t 1+ t  (1 + t )
2

3 x3  3x 3  6 x3
a3 = − + 0 + 0 + 3 =
(1 + t ) 
2
1 + t 1 + t  (1 + t ) 2

c) La velocidad en la descripción Lagrangiana viene dada por:

Universidad de Castilla- La Mancha Draft Por: Eduardo W. V. Chaves (2012)


Ciudad Real - España
2 CINEMÁTICA DEL CONTINUO 145

V1 = X 1

V2 = 2 X 2 (1 + t ) (2.36)
 2
V3 = 3 X 3 (1 + t )
luego,
dV1
a1 = =0
dt
dV
a2 = 2 = 2 X 2 (2.37)
dt
dV
a3 = 3 = 6 X 3 (1 + t )
dt

Ejemplo 2.8
Respecto a un conjunto de ejes materiales X i y espaciales xi superpuestos, el campo de
desplazamientos de un cuerpo continuo viene dado por:
 x1 = X 1

 x2 = X 2 + AX 3 (2.38)
 x = X + AX
 3 3 2

en las que A es constante. Hallar las componentes del vector desplazamiento en las formas
material y espacial.
Solución:
Vector desplazamiento:
r r r
u= x−X (2.39)
u1 = x1 − X 1 = 0

u 2 = x 2 − X 2 = X 2 + AX 3 − X 2 = AX 3 (2.40)
u = x − X = X + AX − X = AX
 3 3 3 3 2 3 2

Las ecuaciones del movimiento inverso son obtenidas a continuación:


 x1  1 0 0   X1 
 x  = 0 1 A  X 2 
 2  (2.41)
 x3  0 A 1   X 3 

1 0 0
det 0 1 A = 1 − A 2 (2.42)
0 A 1 

la inversa:
1 − A 2 0 0 
1  
 0 1 − A (2.43)
1 − A2 
 0 − A 1 

luego,

Universidad Castilla- La Mancha Draft Por: Eduardo W. V. Chaves (2012)


Ciudad Real - España
146 PROBLEMAS RESUELTOS DE MECÁNICA DEL MEDIO CONTINUO

 X1  1 − A 2 0 0   x1 
X  = 1  
 2  1 − A2  0 1 − A  x2  (2.44)
 X 3   0 − A 1   x3 


 X 1 = x1

 1
X 2 = ( x 2 − Ax3 ) (2.45)
 1 − A2
 1
X 3 = ( x3 − Ax 2 )
 1 − A2
Componentes del vector desplazamientos en coordenadas espaciales:

u1 = x1 − X 1 = 0

 1 A( x3 − Ax 2 )
u 2 = x 2 − X 2 = x 2 − 2
( x 2 − Ax3 ) = (2.46)
 1− A 1 − A2
 1 A( x 2 − Ax3 )
u1 = x3 − X 3 = x3 − 2
( x3 − Ax 2 ) =
 1− A 1 − A2

Ejemplo 2.9
Considérese las ecuaciones del movimiento:
 x1 = X 1

 x2 = X 2 + X 3t (2.47)
x = X + X t
 3 3 3

Determinar las velocidades de las partículas que pasan por el punto (0,1,2) en los tiempos
t1 = 0 s y t 2 = 1 s
Solución:
El campo de velocidad viene dado por:
r r r r
Dx ( X , t )
V ( X ,t) = (2.48)
Dt
en componentes:
V1 = 0

V2 = X 3 (2.49)
V = X
 3 3

r r
Para t = 0 s tenemos que x = X , luego, ( X 1 = 0, X 2 = 1, X 3 = 2)

V1 = 0

V2 = 2 (unidades de velocidad) (2.50)
V = 2
 3

Universidad de Castilla- La Mancha Draft Por: Eduardo W. V. Chaves (2012)


Ciudad Real - España
2 CINEMÁTICA DEL CONTINUO 147

Para t = 1 s , la partícula que está pasando por ( x1 = 0, x 2 = 1, x3 = 2) en la configuración de


referencia ocupaba la posición:
x1 = 0 = X 1 

x 2 = 1 = X 2 + X 3  ⇒ ( X 1 = 0; X 2 = 0; X 3 = 1) (2.51)
x3 = 2 = X 3 + X 3 

luego,
V1 = 0

V2 = 1 (unidades de velocidad) (2.52)
V = 1
 3

Ejemplo 2.10
Dado un sistema de referencia ê i , el movimiento de una partícula del medio continuo está
definido por las siguientes ecuaciones:
 ct   ct 
x1 = X 1 sin  2  + X 2 cos
2   2

2 
 X1 + X 2   X1 + X 2 
 ct   ct  (2.53)
x 2 = − X 1 cos 2  + X 2 sin 
2   2 2


 X1 + X 2   X1 + X 2 
x3 = X 3

donde c es una constante.


Determinar las componentes de la velocidad en coordenadas materiales y espaciales.
Solución:
Las componentes de la velocidad en la descripción material (Lagrangiana) son:
r c   ct   ct 
V1 ( X , t ) =  X cos  − X 2 sin  
2 2
X 1 + X 2 
1  2 2  X2 +X2 
 X1 + X 2   1 2  
r c   ct   ct  (2.54)
V2 ( X , t ) =  X sin   + X 2 cos 
2 2
X 1 + X 2 
1  2 2  X2 + X2 
 X1 + X 2   1 2 
r
V3 ( X , t ) = 0

Teniendo en consideración (2.53), podemos notar que se cumple la siguiente relación:


x12 + x22 = X 12 + X 22 (2.55)
Luego, las componentes de la velocidad en la descripción espacial (Euleriana) son:
r c x2
v1 ( x , t ) = −
x12 + x 22
r c x1 (2.56)
v 2 ( x, t ) =
x12 + x 22
r
v3 ( x , t ) = 0

Las ecuaciones inversas del movimiento son:

Universidad Castilla- La Mancha Draft Por: Eduardo W. V. Chaves (2012)


Ciudad Real - España
148 PROBLEMAS RESUELTOS DE MECÁNICA DEL MEDIO CONTINUO

  ct   ct  
 sin  2  − cos
2 

 x 2 + x 2  0
  x1 + x 2   1 2  
 X1     x1 
 X  = cos c t  sin  c t  0  x 
 2    x2 + x2  (2.57)
 x2 + x2   
2
 X 3    1 2   1 2  x 
0 0 1  3 
 
 
 

Ejemplo 2.11
El campo de velocidad tiene las siguientes componentes:
v1 = x1

 x2
v2 = (2.58)
 2t + 3
v3 = 0

en la descripción Euleriana. Encontrar las ecuaciones paramétricas de la trayectoria de la


partícula que en la configuración de referencia estaba en ( X 1 , X 2 , X 3 ).
Solución:
Para encontrar la trayectoria debemos resolver el sistema:
 dx1
 dt = x1

 dx 2 x
 = 2 (2.59)
 dt 2t + 3
 dx3
 =0
 dt
con las condiciones iniciales
 x1 (t = 0) = X 1

 x2 (t = 0) = X 2 (2.60)
 x (t = 0) = X
 3 3

x1 t
dx1  x 

X1
x1 0 ∫
= dt ⇒ Ln 1  = t
 X1 
⇒ x1 = X 1 exp t

x2
(2.61)
( ) ( )
t
dx 2 dt  x  2

X2
x2
=∫0
2t + 3
⇒ Ln 2
 X2
 = Ln 2t + 3 − Ln 3

⇒ x2 = X 2
3
t +1

x3 = X 3

Luego, las ecuaciones del movimiento vienen dadas por:


2
x1 = X 1exp t ; x2 = X 2 t + 1 ; x3 = X 3 (2.62)
3

Universidad de Castilla- La Mancha Draft Por: Eduardo W. V. Chaves (2012)


Ciudad Real - España
2 CINEMÁTICA DEL CONTINUO 149

Ejemplo 2.12
Considérense las siguientes ecuaciones del movimiento:
 x1 = X 1

 x2 = 2 t X 3 + X 2 (2.63)
x = X
 3 3

r
y una cantidad física representada por el campo escalar q ( x , t ) en la descripción Euleriana:
r
q ( x , t ) = 2 x1 + x 2 − x3 + 1 (2.64)
Se pide:
a) Obtener la descripción Lagrangiana de esta cantidad física;
b) Obtener la velocidad en las descripciones Lagrangiana y Euleriana;
c) Obtener la tasa de cambio de la cantidad física en cuestión.
d) Obtener la tasa de cambio local de q en el punto espacial (1,3,2) .
Solución:
r r r r
a) La descripción Lagrangiana es inmediato, q ( x , t ) = q ( x ( X , t ), t ) = Q( X , t ) , es decir,
r
reemplazamos las ecuaciones del movimiento (2.63) en la expresión de la variable q ( x , t )
dada por (2.64):
r
Q( X , t ) = 2 X 1 + X 2 + ( 2t − 1) X 3 + 1 (2.65)
b) La velocidad
r r r r
Dx ( X , t )
V ( X , t) = (2.66)
Dt
Descripción Lagrangiana
V1 = 0

V2 = 2 X 3 (2.67)
V = 0
 3
Las ecuaciones del movimiento inversa:
 x1 = X 1  X 1 = x1
 
 x2 = 2 t X 3 + X 2 ⇒  X 2 = x2 − 2 t x3
x = X X = x
 3 3  3 3

Luego, la descripción Euleriana de la velocidad viene dada por:


v1 = 0

v 2 = 2 x3 (2.68)
v = 0
 3
c) La tasa de cambio de la variable viene dada por la derivada material
D r
Q& = Q( X , t ) = 2 X 3 (2.69)
Dt

Universidad Castilla- La Mancha Draft Por: Eduardo W. V. Chaves (2012)


Ciudad Real - España
150 PROBLEMAS RESUELTOS DE MECÁNICA DEL MEDIO CONTINUO

o
r
∂q( x , t ) r
q& = + ∇ xr q ⋅ v
1
42∂t4 3 (2.70)
= 0 ( estacionario )

q& = 0 + q, i v i
 ∂q ∂q ∂q 
=0+ v1 + v2 + v3  = [(2)(0) + (1)(2 x 3 ) + ( −1)(0)] (2.71)
 ∂x1 ∂x 2 ∂x 3 
= 2 x3

Podríamos haber obtenido este resultado partiendo de que Q& = 2X 3 y reemplazando


X 3 = x3 , obteniendo:
r r r
q& ( x , t ) = Q& ( X ( x , t ), t )
r (2.72)
q& ( x , t ) = 2 x3
r
d) Observemos que el campo de la cantidad física en cuestión es estacionario, i.e. q = q ( x ) ,
r
∂q ( x )
luego la tasa local = 0 para cualquier punto espacial.
∂t

Ejemplo 2.13
Dado el campo de desplazamientos (descripción Lagrangiana):
u1 = ktX 2 ; u2 = 0 ; u3 = 0

y el campo de la temperatura (descripción Euleriana):


r
T ( x , t ) = ( x1 + x 2 ) t

a) Encontrar la tasa de cambio de la temperatura para una partícula que en el tiempo t = 1s


está pasando por el punto (1,1,1) .
Solución:
r r
r ∂T ∂T ∂x r dT ( X , t )
&
Podemos aplicar las dos definiciones: T ( x , t ) = + r⋅ &
ó T ( X , t) =
∂t ∂x ∂t ∂t
A través de la relación u i = xi − X i podemos obtener las ecuaciones del movimiento:
u1 = x1 − X 1 ⇒ x1 = X 1 + ktX 2
u 2 = x2 − X 2 ⇒ x2 = X 2
u 3 = x3 − X 3 ⇒ x3 = X 3
El campo de temperatura en la descripción material queda:
r r r
T ( x ( X , t ), t ) = ( x1 + x 2 ) t = (( X 1 + ktX 2 ) + ( X 2 ) ) t = X 1t + kX 2 t 2 + X 2 t = T ( X , t )
Luego, la derivada material viene dada por:
r
T& ( X , t ) = X 1 + 2kX 2 t + X 2
Si queremos encontrar la tasa de la temperatura para una partícula que está pasando por el
punto x1 = 1, x 2 = 1, x3 = 1 en t = 1s , tenemos dos posibilidades. 1) encontrar la posición de
la partícula en la configuración de referencia y reemplazar en la ecuación anterior. 2)

Universidad de Castilla- La Mancha Draft Por: Eduardo W. V. Chaves (2012)


Ciudad Real - España
2 CINEMÁTICA DEL CONTINUO 151

obtener la expresión de la tasa de la temperatura en la descripción espacial, para esto


r r
necesitamos de las ecuaciones de movimiento X ( x , t ) :
 x1 = X 1 + ktX 2  X 1 = x1 − ktx 2
 
 x2 = X 2 ⇒  X 2 = x2
x = X X = x
 3 3  3 3
r r r
T& ( X ( x , t ), t ) = X 1 + 2kX 2 t + X 2 = ( x1 − ktx 2 ) + 2kt ( x 2 ) + ( x 2 ) = T& ( x , t )
r
Simplificando tenemos que T& ( x , t ) = x1 + ktx 2 + x 2 . Luego:
T& ( x1 = 1, x 2 = 1, x3 = 1, t = 1) = (1 − k ) + 2k + 1 = k + 2
Solución Alternativa:
r
r ∂T ∂T ∂x ∂T ∂T ∂x i
T& ( x , t ) = + r⋅ = + (escalar )
∂t ∂x ∂t ∂t ∂x i ∂t
 ∂T ∂x1 ∂T ∂x 2 ∂T ∂x3 
= ( x1 + x 2 ) +  + + 
 ∂x1 ∂t ∂x 2 ∂t ∂x3 ∂t 
= ( x1 + x 2 ) + (tkX 2 + t (0) + (0)(0) )
= x1 + x 2 + tkX 2
Observemos que x 2 = X 2 , luego:
r
T& ( x , t ) = x1 + x 2 + tkx 2

Ejemplo 2.14
Considérese las siguientes ecuaciones del movimiento:

 x1 = X 1

 t
 x2 = X 2 + X 3 (2.73)
 2
 t
 x3 = X 3 + 2 X 2

Se pide:
a) ¿Para que valores de t > 0 (tiempo) este movimiento es posible y que tenga sentido
físico?
b) Determinar las componentes de la velocidad en la descripción Lagrangiana y
Euleriana;
c) Obtener la ecuación de la trayectoria.
Solución:
a) Obteniendo el determinante del Jacobiano:
1 0 0
∂xi t2
J=F = = 0 1 2t = 1 − (2.74)
∂X j 4
0 2t 1

con lo que el movimiento es posible para t < 2 s , ya que:

Universidad Castilla- La Mancha Draft Por: Eduardo W. V. Chaves (2012)


Ciudad Real - España
152 PROBLEMAS RESUELTOS DE MECÁNICA DEL MEDIO CONTINUO

t2
J =1− >0⇒t <2 s (2.75)
4
b) Velocidad en la descripción Lagrangiana:

V = 0
 1
 D t  X3
V2 =  X2 + X3  = (2.76)
 Dt  2  2
 D t  X2
V3 =  X3 + X2  =
 Dt  2  2

La inversa del tensor F viene dada por:


T
 1 t
0 t 0 1 
 t
2
− 2
t 
 2 1 0 1 0 2 

1 1 1 0 0 1 0 1 0
F −1 = adj( F ) = [cof (F )]T ⇒ Fij−1 = − t − t

F F J 2 1 0 1 0 2 
 0 0 1 0 1 0  (2.77)
 t
− 
 1 2
0 2t 0 1 
J 0 0 
1 
⇒ Fij−1 = 0 1 − 2t 
J
0 − 2t 1 

r r
Las ecuaciones inversas del movimiento X = F −1 ⋅ x :
 x1  1 0 0   X 1   X1  J 0 0   x1 
 x  = 0 1 t   X  ⇒ X  = 1 0 1

− 2t   x 2 
 2  2  2   2 J  (2.78)
 x3  0 2 1   X 3 
 t  X 3  0
 − 2t 1   x3 

Reemplazando X i en la expresión de la velocidad, obtenemos la velocidad en la


descripción espacial:
t t
x2x3 − x2 −
x3
2 2 x − tx 2 = 2 x 2 − tx 3
v1 = 0 ; v 2 = 2
= 3 22 ; v3 = (2.79)
t 4−t t2 4 − t2
2− 2−
2 2
c) La trayectoria se obtiene eliminando t de las ecuaciones del movimiento (2.73):
 x1 = X 1

 X2 X2 (2.80)
( x3 − X 3 ) X 3 = ( x 2 − X 2 ) X 2 ⇒ x3 =
X3
x2 − 2 + X 3
X3

Ejemplo 2.15
r
La velocidad en un punto x de un fluido estacionario viene dada por:

Universidad de Castilla- La Mancha Draft Por: Eduardo W. V. Chaves (2012)


Ciudad Real - España
2 CINEMÁTICA DEL CONTINUO 153

r b 2 ( x12 − x 22 ) b 2 x1 x 2
v =U ˆ 1 + 2U
e eˆ 2 + Veˆ 3 (2.81)
( x12 + x 22 ) 2 ( x12 + x 22 ) 2

donde U y V son constantes.


r r
Mostrar que ∇ xr ⋅ v = 0 y encontrar la aceleración de la partícula en x .
Solución:
r ∂v ∂v ∂v
∇ xr ⋅ v = vi ,i = 1 + 2 + 3
∂x1 ∂x 2 ∂x3
x1 ( x12 − 3 x 22 ) x1 ( x12 − 3 x 22 )
= −2Ub 2 + 2Ub 2 =0
( x12 + x 22 ) 3 ( x12 + x 22 ) 3
La aceleración:
r
r ∂v r r
a= + ∇ xr v ⋅ v
∂t
r r
= ∇ xr v ⋅ v
Las componentes del gradiente espacial de la velocidad vienen dadas por:
 x1 (3 x 22 − x12 ) − x 2 (3 x12 − x 22 ) 0
r 2Ub  2

(∇ xr v ) ij = 2 2 3 
− x 2 (3 x12 − x 22 ) − x1 (3 x 22 − x12 ) 0
( x1 + x 2 ) 
 0 0 0
r r
Las componentes de la aceleración vienen dadas por a i = (∇ xr v ) ij (v ) j :

 − 2 x1U 2 b 4 
 2 2 3 
 ( x1 + x 2 ) 
 
a i =  − 2 x 2U 2 b 4 
 2 2 3 
 ( x1 + x 2 ) 
 
 0 

Ejemplo 2.16
Para el siguiente campo de velocidades:

v1 = 0 ; v2 = 0 ; v3 = f ( x1 , x 2 ) x3

Se pide:
a) Hallar las trayectorias de las partículas;
b) Determinar la densidad de masa ( ρ ), sabiendo que en t = 0 , ρ = f ( x1 , x 2 ) .
Solución:
dx1
= v1 = 0 ⇒ x1 (t ) = C1 para t = 0 ⇒ x1 = X 1 ⇒ x1 (t = 0) = C1 = X 1 ;
dt
dx 2
= v 2 = 0 ⇒ x 2 (t ) = C 2 para t = 0 ⇒ x 2 = X 2 ⇒ x 2 (t = 0) = C 2 = X 2
dt

Universidad Castilla- La Mancha Draft Por: Eduardo W. V. Chaves (2012)


Ciudad Real - España
154 PROBLEMAS RESUELTOS DE MECÁNICA DEL MEDIO CONTINUO

dx3 dx3
dt
= v3 = f ( x1 , x 2 ) x3 = f (C1 , C 2 ) x3 ⇒
x3
= ∫ ∫ f (C , C 1 2 ) dt ⇒ Ln( x 3 ) = f (C1 , C 2 )t + k

haciendo k = Ln(C 3 ) obtenemos que:


Ln( x3 ) − Ln(C 3 ) = f ( X 1 , X 2 )t
x  x
⇒ ln 3  = f ( X 1 , X 2 )t ⇒ 3 = exp f ( X 1 , X 2 )t ⇒ x3 = C 3 exp f ( X 1 , X 2 )t
 C3  C3
para t = 0 ⇒ x3 = X 3 ⇒ x3 (t = 0) = C 3 = X 3
Resumiendo:
x1 = X 1
x2 = X 2 (2.82)
f ( X 1 , X 2 )t
x3 = X 3 exp

Densidad de masa:
ρ0 ∂xi
ρ= con Fij =
F ∂X j

1 0 0
Fij = 0 1 0 = exp f ( X 1 , X 2 )t
? ? exp f ( X 1 , X 2 )t
Los valores ( ? ) no son necesarios para obtener el determinante. Luego:
ρ0 f (X1, X 2 )
ρ= =
F exp f ( X 1 , X 2 )t
Observemos que según el enunciado, t = 0 , ρ = f ( x1 , x 2 ) , y según expresiones (2.82)
concluimos que ρ 0 = f ( X 1 , X 2 ) .

Ejemplo 2.17
D
Calcular la derivada material para la propiedad φ cuando esta esté descrita en las
Dt
siguientes coordenadas:
r
ƒ Materiales: φ ( X , t ) = X 1t 2 ;
r x1t 2
ƒ Espaciales: φ ( x , t ) = .
(1 + t )
Solución:
r
a) Derivada material de φ ( X , t ) = X 1t 2 :
D r r
φ( X , t ) ≡ φ& ( X , t ) = 2 X 1t
Dt
r x1t 2
b) Derivada material de φ ( x , t ) = :
(1 + t )

Universidad de Castilla- La Mancha Draft Por: Eduardo W. V. Chaves (2012)


Ciudad Real - España
2 CINEMÁTICA DEL CONTINUO 155

r r r
D r ∂φ( x , t ) r ∂φ( x , t ) ∂φ( x , t )
φ( x , t ) = + ∇ xφ ⋅ v =
r + vi
Dt ∂t ∂t ∂xi
r r r r
∂φ( x , t )  ∂φ( x , t ) ∂φ( x , t ) ∂φ( x , t ) 
= + v1 + v2 + v3 
∂t  ∂x1 ∂x 2 ∂x3 
r
∂  x t 2   ∂φ( x , t ) 
=  1  +  v1 + 0 + 0
∂t  (1 + t )   ∂x1 
Necesitamos conocer la velocidad v1 . Partimos del principio de que una propiedad es algo
intrínsico a la partícula luego:
r
φ ( X , t ) = X 1t 2
r r r x t2 x1
⇒ φ( X ( x, t ), t ) = φ ( x , t ) = 1 ⇒ X1 =
(1 + t ) (1 + t )
La velocidad queda:

( )
r D r x
v( X , t) = X 1t 2 = 2 X 1t eˆ 1 ⇒ v ( x , t ) = 2 1 t eˆ 1
Dt (1 + t )
Retomando a la derivada material tenemos que:
r
D r ∂  x t 2   ∂φ( x , t ) 
φ( x , t ) =  1  +  v1 
Dt ∂t  (1 + t )   ∂x1 
 2 x1t x t2   t2 
=  − 1 2 +
 X1
 (1 + t ) (1 + t )   (1 + t ) 
 2 x1t x t2   t2 x 
=  − 1 2 +
 2 1 t
 (1 + t ) (1 + t )   (1 + t ) (1 + t ) 
2 x1t
=
(1 + t )
También podríamos haber obtenido este mismo resultado partiendo que se cumple que
D r r
φ( X , t ) ≡ φ& ( X , t ) = 2 X 1t y además conociendo las ecuaciones del movimiento
Dt
x1
X1 = , obtenemos que:
(1 + t )
D r r
φ( X , t ) ≡ φ& ( X , t ) = 2 X 1t
Dt
D r r r r r x
⇒ φ( X ( x , t ), t ) ≡ φ& ( X ( x , t ), t ) ≡ φ& ( x , t ) = 2 1 t
Dt (1 + t )

Ejemplo 2.18
Sean las siguientes ecuaciones del movimiento en la descripción Lagrangiana:

 x1 = X 1t 2 + 2 X 2 t + X 1  x1  t + 1 2t
2
0  X 1 
 2
forma
Matricial     
 x 2 = 2 X 1 t + X 2 t + X 2   →  x 2  =  2t 2 t +1 0  X 2  (2.83)
x = 1 X t + X x   0 0 1
t + 1  X 3 
 3 2 3 3  3  2

Universidad Castilla- La Mancha Draft Por: Eduardo W. V. Chaves (2012)


Ciudad Real - España
156 PROBLEMAS RESUELTOS DE MECÁNICA DEL MEDIO CONTINUO

Encontrar las componentes del vector desplazamiento en la descripción Lagrangiana y


Euleriana.
Solución:
r r r
El vector desplazamiento viene dado por u = x − X . Reemplazando las ecuaciones del
movimiento (2.83) obtenemos:
u1 = x1 − X 1 = X 1t 2 + 2 X 2 t
 2
u 2 = x 2 − X 2 = 2 X 1t + X 2 t

u 3 = x 3 − X 3 = 2 X 3 t
1

que son las componentes del vector desplazamiento en la descripción Lagrangiana


(material).
Para obtener la descripción Euleriana debemos obtener las ecuaciones inversas de
(2.83), resultando:
 2tx 2 − x1 (1 + t )
  X1 = 3
 − (1 + t ) x   3t − 1 − t − t 2
 1
X 2t 0
  1  2
  1
2 x1t 2 − x 2 (1 + t 2 )
X 2  = 3 2 
2t − (1 + t 2 ) 0  x2  ⇒  X 2 =
 X  3t − 1 − t − t  3t 3 − 1 − t − t 2   x 3   3t 3 − 1 − t − t 2
 3  0 0   2 x3

1
2
(t + 2)  X 3 =
 (t + 2)
r r r
Podemos utilizar, una vez más, la definición u = x − X , pero ahora reemplazamos las
coordenadas materiales, obteniendo así las componentes del vector desplazamiento en
coordenadas espaciales (descripción Euleriana):
 2tx 2 − x1 (1 + t )
u1 = x1 − X 1 = x1 − 3
 3t − 1 − t − t 2
 2 x1t 2 − x 2 (1 + t 2 )
u 2 = x 2 − X 2 = x 2 −
 3t 3 − 1 − t − t 2
 2 x3
u 3 = x 3 − X 3 = x 3 −
 (t + 2)

Universidad de Castilla- La Mancha Draft Por: Eduardo W. V. Chaves (2012)


Ciudad Real - España
2 CINEMÁTICA DEL CONTINUO 157

Ejemplo 2.19
Las siguientes ecuaciones X 2 , x2
describen el movimiento de las Configuración de referencia
partículas de un cuerpo (medio
continuo): 1 B
C
1
 x1 = X 1 + 0,2 X 2 t
 E
 x2 = X 2 1
x = X
 3 3 A
O
X 1 , x1
D
G

X 3 , x3
Figura 2.1: Configuración de referencia t = 0 .
En t = 0 , este cuerpo tiene forma de cubo de lado unitario con un vértice en el origen,
punto O, como se indica en la Figura 2.1. Determinar la configuración del cuerpo en el
instante t = 2 s .
Solución:
Para obtener la configuración actual del cuerpo para el instante t = 2 s analizaremos
independientemente el movimiento de las partículas. La partícula que ocupa posición O
(origen) en t = 0 tiene coordenadas materiales:
X1 = 0

X 2 = 0
X = 0
 3
Sustituyendo en la expresión del movimiento:
 x1 = 0

x i ( X 1 = 0, X 2 = 0, X 3 = 0, t ) ⇒  x 2 = 0
x = 0
 3
Concluyendo que la partícula del origen no cambia de posición durante el movimiento.
Las partículas que ocupan la línea OA en la configuración inicial tienen como coordenadas
de referencia: ( X 1 , X 2 = 0, X 3 = 0) . En coordenadas espaciales:
 x1 = X 1 + 0,2 X 2 t = X 1

x2 = X 2 = 0
x = X = 0
 3 3

Es decir, todas las partículas que están en la línea OA no se mueven durante el


movimiento. Análogamente, podemos verificar que la recta ( X 1 , X 2 = 0, X 3 = 1) en la
configuración de referencia ( X 1 , X 2 = 0, X 3 = 1) no se mueve:
 x1 = X 1 + 0,2 × 0 × 2 = X 1

x2 = X 2 = 0
x = X = 0
 3 3

Universidad Castilla- La Mancha Draft Por: Eduardo W. V. Chaves (2012)


Ciudad Real - España
158 PROBLEMAS RESUELTOS DE MECÁNICA DEL MEDIO CONTINUO

Las partículas que están en la línea CB ( X 1 , X 2 = 1, X 3 = 0) en el tiempo t = 2 s ocuparán


las posiciones:
 x1 = X 1 + 0,2 × 1 × 2 = X 1 + 0,4

x2 = X 2 = 1
x = X = 0
 3 3

Luego, todas las partículas que están en la línea CB se desplazarán 0,4 según la dirección
x1 .

Las partículas que están en la línea OC en t = 0 , tras el movimiento ocuparán las


posiciones:
 x1 = X 1 + 0,2 X 2 t = 0 + 0,2 × 2 × X 2 = 0,4 X 2

x2 = X 2
x = X = 0
 3 3

Siguiendo el mismo procedimiento para las partículas restantes, se obtiene la configuración


final ( t = 2 s ) del cuerpo representada por la Figura 2.2.

x2

0,4 0,4

C C’ 1 B B’ Configuración del cuerpo


en t = 2 s
E E’ 1
A=A’
O
1 x1
D
G=G’

x3

Figura 2.2: Configuración actual t .

Ejemplo 2.20
Considérense las ecuaciones del movimiento dadas por:
 x1 = X 1 + t 2 X 2  x1   1 t2 0  X 1 
 2
Forma
Matricial     
 x 2 = t X 1 + X 2   →  x 2  = t 2 1 0  X 2 
x = X x   0 0 1   X 3 
 3 3  3 

Se pide:
1) Determinar la trayectoria de una partícula Q que originalmente, en t 0 , estaba en
X i = (1,2,1) ;
2) Considerando la configuración actual t = 0,5s . Determinar las componentes de la
velocidad y de la aceleración de una partícula P que originalmente estaba en
X i = (16 ; −4 ;1) ;
15 15

Universidad de Castilla- La Mancha Draft Por: Eduardo W. V. Chaves (2012)


Ciudad Real - España
2 CINEMÁTICA DEL CONTINUO 159

3) Obtener las ecuaciones del movimiento en la descripción Euleriana;


4) Obtener las componentes de la velocidad y la aceleración de una partícula que en el
tiempo ( t = 0,5s ) pasa por el punto x i = (1,0,1) .
NOTA: Considerar el sistema internacional de unidades.
Solución:
1)Utilizando las ecuaciones del movimiento y reemplazando las coordenadas materiales del
punto X i = (1,2,1) , obtenemos:
 x1 = 1 + 2t 2
 2
x2 = 2 + t
x = 1
 3

Las ecuaciones anteriores representan la trayectoria de la partícula. Para obtener la ecuación


de la partícula, eliminamos el tiempo de las ecuaciones de movimiento, obteniendo así:
 x1 − 2 x 2 = −3

 x3 = 1
Lo cual indica que la partícula se mueve en línea recta según la ecuación ( x1 − 2 x 2 = −3) en
el plano x 3 = 1 , ver figura siguiente.

X 3 , x3 trayectoria de la partícula

( x1 − 2 x 2 = −3)
x3 = 1

X 2 , x2

X 1 , x1

2) Las componentes de la velocidad y de la aceleración de la partícula P vienen dadas


respectivamente por:
r V1 = 2tX 2
r r Dx componente s 
V ( X , t) =    → V 2 = 2tX 1
Dt V = 0
 3

r  A1 = 2 X 2
r r Dv componente s 
A( X , t ) =    →  A2 = 2 X 1
Dt A = 0
 3
Luego, para la partícula situada originalmente en X i = (16 ; −4 ;1) , en t = 0,5s tiene:
15 15

Universidad Castilla- La Mancha Draft Por: Eduardo W. V. Chaves (2012)


Ciudad Real - España
160 PROBLEMAS RESUELTOS DE MECÁNICA DEL MEDIO CONTINUO

V1 = 2 × 0,5 × 15
−4
= 15
−4
m/s  A1 = 2 × 15
−4
= 15
−8
m / s2
 
V 2 = 2 × 0,5 × (15 ) = 15 m / s  A2 = 2 × (15 ) = 15 m / s
16 16 16 32 2
y
V = 0 A = 0
 3  3

3) Invirtiendo las ecuaciones del movimiento resulta:


 x1 − t 2 x 2
 1 X =
 x1 = X 1 + t 2 X 2 ⇒ X 1 = x1 − t 2 X 2  1− t4
 2 2
 x − t 2 x1
 x2 = t X 1 + X 2 ⇒ X 2 = x2 − t X 1 ⇒ X 2 = 2
x = X ⇒ X = x  1− t4
 3 3 3 3
 X 3 = x3


4) Podemos obtener la velocidad y la aceleración de la partícula que en el tiempo ( t = 0,5s )
pasa por el punto xi = (1,0,1) simplemente obteniendo la expresión de la velocidad y
aceleración en coordenadas espaciales:
Velocidad:
 x 2 − t 2 x1  −4
 1 v = 2t
 1− t4 v1 = 15 m / s
 1
V = 2tX 

2
sustituyen do
 x − t 2 x2 = 0,5 s  16
V 2 = 2 X 1t X → v 2 = 2t 1 4
t → v 2 = m / s
V = 0 1 , X 2
 1− t x (1, 01)
 15
 3 v 3 = 0 v 3 = 0
 
 

Aceleración:
 x 2 − t 2 x1  8
 1 a = 2 2

 1− t4 a1 = − 15 m / s
 1
A = 2 X 

2
sustituyen do
 x − t 2 x2 =0,5 s  32
 A2 = 2 X 1 X → a 2 = 2 1 4
t → a 2 = m / s2
A = 0 1 , X 2
 1− t x (1, 01)
 15
 3 a 3 = 0 a 3 = 0
 
 

Podemos obtener la posición inicial de esta partícula utilizando las ecuaciones inversas del
movimiento obtenidas en el apartado 3, x i (1,0,1) , resultando:
 x1 − t 2 x 2 1 − (0,5 2 )(0) 16
 1X = = =
 1− t4 1 − (0,5) 4 15
 x 2 − t 2 x1 0 − (0,5 2 )(1)
 4
X 2 = 4
= 4
=−
 1− t 1 − (0,5) 15
X = x = 1
 3 3



Podemos verificar que es la misma partícula P referida en el apartado 2. Es lógico que


hayamos encontrado las mismas velocidades y aceleraciones utilizando las coordenadas
materiales o espaciales, ya que la velocidad y aceleración son propiedades intrínsecas de una
partícula.

Universidad de Castilla- La Mancha Draft Por: Eduardo W. V. Chaves (2012)


Ciudad Real - España
2 CINEMÁTICA DEL CONTINUO 161

Ejemplo 2.21
La aceleración de una partícula en un medio continuo está descrita por:
r r
r r D v ∂v r r
a ( x , t) = = + ∇ xr v ⋅ v
Dt ∂t
Demostrar que la aceleración también se puede escribir como:
r r r
Dv ∂v  v2  r r r ∂v  v2  r r
= + ∇ xr   − v ∧ (∇ x ∧ v ) =

r + ∇ r
x
 − v ∧ rot v

Dt ∂t  2  ∂t  2 
Solución:
Para demostrar la relación anterior es suficiente demostrar por identificación de términos:
r r  v2  r r r
∇ xr v ⋅ v = ∇ xr   − v ∧ (∇ xr ∧ v )

 2 
En notación simbólica:
 v2  r r r 1 ∂  ∂ ˆ
∇ xr   − v ∧ (∇ xr ∧ v ) = eˆ i
 ∂
( )
v j v j  − (v i eˆ i ) ∧

e r ∧ (v s eˆ s )
 2  2  x i  x r

Utilizado la definición del operador de permutación (Capítulo 1) podemos expresar el


producto vectorial como:
 v2  r r 1 ∂  ∂v
∇ xr 
r
 − v ∧ (∇ xr ∧ v ) = eˆ i
 2  ∂x i
( )
v j v j  − (v i eˆ i ) ∧  rst s eˆ t
∂x r
 2  
1 ∂v j  ∂v s
= eˆ i 2v j  −  rst  itk v i eˆ k
2 ∂x i  ∂x r

En el capítulo 1 del libro se demostró que  rst  itk =  rst  kit = δ rk δ si − δ ri δ sk , luego:
 v2  r r r ∂v j ∂v
∇ xr   − v ∧ (∇ xr ∧ v ) = v j
 eˆ i − (δ rk δ si − δ ri δ sk )v i s eˆ k
 2  ∂x i ∂x r
∂v j  ∂v s ∂v 
=vj eˆ i −  δ rk δ si v i − δ ri δ sk v i s eˆ k
∂x i  ∂x r ∂x r 
∂v j  ∂v ∂v 
=vj eˆ i −  v s s − v i k eˆ k
∂x i  ∂x k ∂x i 

 v2  r r r ∂v j ∂v ∂v
∇ xr   − v ∧ (∇ xr ∧ v ) = v j
 eˆ i − v s s eˆ k + v i k eˆ k
 2  ∂x i ∂x k ∂x i
∂v j ∂v ∂v
= δ sj v s eˆ i − v s s δ ik eˆ i + v i k eˆ k
∂x i ∂x k ∂x i
∂v s ∂v s ∂v k
= vs eˆ i − v s eˆ i + v i eˆ k
∂x i ∂x i ∂x i
∂v eˆ
= k k vi
∂x i
r r
= ∇ xr v ⋅ v
NOTA: Es interesante que el lector compare éste desarrollo con el Ejemplo 1.109
(capítulo 1).

Universidad Castilla- La Mancha Draft Por: Eduardo W. V. Chaves (2012)


Ciudad Real - España
162 PROBLEMAS RESUELTOS DE MECÁNICA DEL MEDIO CONTINUO

Ejemplo 2.22
r r r
Consideremos las ecuaciones de movimiento x ( X , t ) y el campo de temperatura T ( x , t )
dado por:
 x1 = X 1 (1 + t )
 r
 x 2 = X 2 (1 + t ) ; T ( x ) = x12 + x 22
x = X
 3 3

Encontrar la tasa de cambio de la temperatura de la partícula P en el tiempo t = 1s , dicha


partícula P estaba en el punto ( X 1 = 3, X 2 = 1, X 3 = 0) en el tiempo t = 0 .
Solución 1:
En esta primera solución obtenemos la derivada material de la temperatura Lagrangiana,
r
para ello tenemos que obtener la temperatura en la descripción Lagrangiana T ( X , t )
(Temperatura Lagrangiana):
r
T ( x ) = x12 + x 22

sustituyen do
las ecuaciones del movimiento

r
T ( X , t ) = X 12 (1 + t ) 2 + X 22 (1 + t ) 2
La derivada material de la temperatura Lagrangiana viene dada por:
r
r DT dT ( X , t)
T& ( X , t ) ≡ = = 2 X 12 (1 + t ) + 2 X 22 (1 + t )
Dt dt
Sustituyendo t = 1s , ( X 1 = 3, X 2 = 1, X 3 = 0) , en la ecuación anterior obtenemos que:
r
⇒ T& ( X , t ) = 2 X 12 (1 + t ) + 2 X 22 (1 + t ) = 2(3) 2 (1 + 1) + 2(1) 2 (1 + 1) = 40
Solución 2:
En esta solución alternativa, utilizamos directamente la definición de derivada material de
r r
& r DT ∂T ( x ) ∂T ( x ) r
una propiedad Euleriana, i.e. T ( x , t ) = = + vk ( x, t ) .
Dt ∂t ∂x k
De las ecuaciones del movimiento obtenemos:
r
 x1 = X 1 (1 + t ) v1 ( X , t ) = X 1
 velocidad
 r
 x 2 = X 2 (1 + t )  → v 2 ( X , t ) = X 2
x = X  r
 3 3 v 3 ( X , t ) = 0
Las ecuaciones del movimiento en la descripción Euleriana vienen dadas por:
 x1
 X 1 = (1 + t )
 x1 = X 1 (1 + t ) 
 movimiento inverso  x2
 x2 = X 2 (1 + t )     →  X 2 =
x = X  (1 + t )
 3 3  X 3 = x3


Luego, es posible obtener la velocidad Euleriana como:

Universidad de Castilla- La Mancha Draft Por: Eduardo W. V. Chaves (2012)


Ciudad Real - España
2 CINEMÁTICA DEL CONTINUO 163

 r r r x1 r
V1 ( X ( x , t ), t ) = X 1 ( x , t ) = (1 + t ) = v1 ( x , t )

 r r r x2 r
V 2 = ( X ( x , t ), t ) = X 2 ( x , t ) = = v 2 ( x, t )
 (1 + t )
r
V3 = v 3 ( x , t ) = 0


r
Con eso, la derivada material de la temperatura Euleriana, T ( x , t ) , viene dada por:
r r
DT ( x , t ) & r ∂T ( x )  ∂T ∂T ∂T 
⇒ ≡ T ( x, t ) = + v1 + v2 + v3 
Dt 12 ∂t3  ∂x1 ∂x 2 ∂x 3 
= 0 (Campo estacionar io)

r x x r 2x 2 2x 2 2
⇒ T& ( x , t ) = 2 x1 1 + 2 x 2 2 + 0 ⇒ T& ( x , t ) = 1 + 2 = ( x12 + x 22 )
1+ t 1+ t 1+ t 1+ t 1+ t
La posición de la partícula P en el tiempo t = 1s viene dada por:
 x1 = X 1 (1 + t ) = 3(1 + 1) = 6

 x 2 = X 2 (1 + t ) = 1(1 + 1) = 2
x = X = 0
 3 3

Luego, sustituyendo las coordenadas espaciales en la expresión de la derivada material de la


temperatura obtenemos que:
r 2 2
T& ( x , t ) = T& ( x1 = 6, x 2 = 2, x 3 = 0, t = 1) = ( x12 + x 22 ) = (6 2 + 2 2 ) = 40
1+ t 1+1
r
Alternativamente, la expresión T& ( x , t ) puede también ser obtenido como sigue:
r
T& ( X , t ) = 2 X 12 (1 + t ) + 2 X 22 (1 + t )
2 2
r r r 2 r 2  x   x 
T& ( X ( x , t ), t ) = 2[ X 1 ( x , t )] (1 + t ) + 2[X 2 ( x , t ) ] (1 + t ) = 2  1  (1 + t ) + 2  2  (1 + t )
 (1 + t )   (1 + t ) 
2 r
= ( x12 + x 22 ) = T& ( x , t )
(1 + t )

Ejemplo 2.23
Sea el movimiento:
 x i = X i (1 + t )

t > 0
Determinar el campo de velocidad en la descripción espacial.
Solución:
La velocidad será obtenida a través de la derivada temporal de las ecuaciones del
movimiento:
d
Vi = x& i = [X i (1 + t )] = X i (2.84)
dt
Para hallar la velocidad en la descripción espacial tenemos que obtener la inversa de las
ecuaciones del movimiento y reemplazar en la ecuación (2.84):

Universidad Castilla- La Mancha Draft Por: Eduardo W. V. Chaves (2012)


Ciudad Real - España
164 PROBLEMAS RESUELTOS DE MECÁNICA DEL MEDIO CONTINUO

 xi
 x i = X i (1 + t ) ⇒ X i = (1 + t )

v = X ( xr , t ) = x i
 i i
1+ t

Ejemplo 2.24
r
Considérese el campo de temperatura T ( x ) en la descripción espacial y las ecuaciones del
movimiento siguientes:
T = 2( x12 + x 22 )

 xi = X i (1 + t ) i ∈ {1,2}

Encuentre en t = 1s la tasa de cambio de temperatura de una partícula que ocupaba la


posición (1,1) en la configuración de referencia.
NOTA: Podemos observar que el campo de temperatura es un campo estacionario, es
r
decir T = T ( x ) . ■
Solución 1:
En esta primera solución obtendremos la ecuación de la temperatura en la descripción
material:
r
 T ( x ) = 2( x12 + x 22 )
 ↓
reemplazan do las ecuaciones
 del movimiento

 r
 [
T ( X , t ) = 2 X 2 (1 + t ) 2 + X 2 (1 + t ) 2
1 2 ]
La derivada material viene dada por:
r
r
⇒ T& ( X , t ) =
DT
Dt
=
dT (
dt
X , t)
[
= 2 2 X 12 (1 + t ) + 2 X 22 (1 + t ) ]
Reemplazando t = 1s y las coordenadas materiales ( X 1 = 1; X 2 = 1) obtenemos:
⇒ T& ( X 1 = 1; X 2 = 1; t = 1) = 16
Solución 2:
En esta segunda solución usaremos directamente la definición de derivada material para
propiedades descritas en coordenadas espaciales:
r
T ( x ) = 2( x12 + x12 ) x i = (1 + t ) X i
; i ∈{1,2}
r r
& r DT ∂T ( x ) ∂T ( x ) ∂x k
⇒ T ( x, t ) = = + i ∈ {1,2}
Dt ∂t ∂x k ∂t
r
r ∂T ( x )
dado que T ( x ) no es función del tiempo =0:
∂t
r ∂T ∂x1 ∂T ∂x 2
⇒ T& ( x , t ) = 0 + +
∂x1 {
∂t ∂x 2 {
∂t
V1 = X 1 V2 = X 2

Universidad de Castilla- La Mancha Draft Por: Eduardo W. V. Chaves (2012)


Ciudad Real - España
2 CINEMÁTICA DEL CONTINUO 165

r x x
⇒ T& ( x , t ) = 0 + 4 x1 1 + 4 x 2 2
1+ t 1+ t
2 2
r 4x 4x
⇒ T& ( x , t ) = 1 + 2
1+ t 1+ t
La partícula que en la configuración de referencia ocupaba la posición (1,1) , en el tiempo
t = 1s estará en la posición x i = (1 + t ) X i = 2 X i , es decir, ( x1 = 2; x 2 = 2 ):

4( 2) 2 4( 2) 2
T& ( x1 = 2; x 2 = 2; t = 1) = + = 16
1+1 1+1

Ejemplo 2.25
Dadas las siguientes ecuaciones del movimiento:
 x1 = X 1 exp t + X 3 (exp t − 1)
 t −t
 x 2 = X 2 + X 3 (exp − exp )
x = X
 3 3

Se pide:
Determinar las componentes de la velocidad y de la aceleración en coordenadas materiales
y espaciales.
Solución:
Primero obtenemos la inversa de la ecuación del movimiento:
 x1 = X 1 exp t + X 3 (exp t − 1)  x1 − X 1 exp t = x 3 (exp t − 1)
 −t

t
 x 2 = X 2 + X 3 (exp − exp ) →  x 2 − X 2 = x 3 (exp t − exp −t )

x = X ⇒ X = x x = X ⇒ X = x
 3 3 3 3
 3 3 3 3

resultando:
 X 1 = x1 exp − t − exp − t (exp t − 1)
 2t −t
 X 2 = x 2 − x 3 (exp − 1)exp (2.85)
X = x
 3 3

o
 x1  exp  X 1   X 1  exp − exp −t (exp t − 1)   x1 
t −t
0 (exp t − 1) 0
          
x2  =  0 1 (exp t − exp −t )   X 2  inversa
 →  X 2  =  0 1 − (exp 2 t − 1)exp −t   x 2 
x   0 0 1    X   0 x 
 3   X 3   3  0 1  3 

a) La velocidad en la descripción material viene dada a través de sus componentes:


V1 = X 1 exp t + X 3 exp t
D r 
V i= → V 2 = X 3 exp t + X 3 exp − t = X 3 (exp t + exp − t )
x j ( X , t)  (2.86)
Dt V = 0
 3

b) La aceleración en la descripción material viene dada por:

Universidad Castilla- La Mancha Draft Por: Eduardo W. V. Chaves (2012)


Ciudad Real - España
166 PROBLEMAS RESUELTOS DE MECÁNICA DEL MEDIO CONTINUO

r  A1 = X 1exp t + X 3 exp t
r DV i ( X , t ) 
Ai ( X , t ) = →  A2 = X 3 (exp t − exp − t )
 (2.87)
Dt A = 0
 3

Para obtener la velocidad y la aceleración en la descripción espacial es suficiente sustituir en


las ecuaciones (2.86) y (2.87) los valores de X 1 , X 2 , X 3 , dados por la ecuación (2.85),
resultando:
v1 = x1 + x 3 a1 = x1 + x 3
 −t  −t
v 2 = x 3 (exp + exp )
t
a 2 = x 3 (exp − exp )
t
;
v = 0 a = 0
 3  3
Velocidad en la Aceleració n en la
descripció n espacia l descripció n espacial

Ejemplo 2.26
El movimiento de un medio continuo viene definido por las siguientes ecuaciones:
 x1 = 12 ( X 1 + X 2 )exp t + 12 ( X 1 − X 2 )exp − t
 t −t
 x 2 = 2 ( X 1 + X 2 )exp − 2 ( X 1 − X 2 )exp
1 1

x = X
 3 3

0 ≤ t ≤ constante
Expresar las componentes de la velocidad en la descripción material y espacial.
Solución:
Las componentes de la velocidad utilizando la descripción material son:
r
 Dx1 ( X , t ) 1 1
V1 = = ( X 1 + X 2 )exp t − ( X 1 − X 2 )exp −t
 Dt 2 2
r
 Dx 2 ( X , t ) 1 1
V 2 = = ( X 1 + X 2 )exp t + ( X 1 − X 2 )exp −t (2.88)
 Dt 2 2
V3 = 0



Para expresar las componentes de la velocidad en la descripción espacial tenemos que


obtener la inversa de las ecuaciones de movimiento, x i = x i ( X 1 , X 2 , X 3 ) resultando:
 (exp t + exp − t ) (exp t − exp − t ) 
 0
 x1   2 2  X 1 
   (exp
t
− exp − t ) (exp t + exp − t )  
x2  =  0  X 2 
2 2 
x    
 3 0 0 1  X 3 
 
 
 X1   (exp 2t + 1)exp − t − (exp 2t − 1)exp − t 0   x1 
  1   
→  X 2  =  − (exp 2t − 1)exp − t
inversa (exp 2 t + 1)exp −t 0  x2 
X  2  0 0 2   x 3 
 3 
Para obtener la velocidad en la descripción espacial es suficiente reemplazar las ecuaciones
anteriores en las expresiones de la velocidad (2.88), resultando:

Universidad de Castilla- La Mancha Draft Por: Eduardo W. V. Chaves (2012)


Ciudad Real - España
2 CINEMÁTICA DEL CONTINUO 167

v1 = x 2

v 2 = x1
v = 0
 3
Ejemplo 2.27
Dado el movimiento:
x i = ( X 1 + ktX 2 )δ i1 + X 2 δ i 2 + X 3 δ i 3 i ∈ {1,2,3}
y la temperatura
T = x1 + x 2
Encontrar la tasa de cambio de T para la partícula que en la configuración actual está
situada en el punto (1,1,1) .
Solución:
Explícitamente las ecuaciones de movimiento son:
 x1 = X 1 + ktX 2

x2 = X 2
x = X
 3 3

Reemplazando x i en la expresión de la temperatura, se obtiene la temperatura en la


configuración material:
r r
T ( x ) = x1 + x 2 ⇒ T ( X , t ) = X 1 + ktX 2 + X 2
La derivada material de la temperatura viene dada por:
r DT D ( X 1 + ktX 2 + X 2 )
T& ( X , t ) = = = kX 2 = k x 2 ( → T& = k
1,1,1)
Dt Dt
Solución alternativa:
La derivada material para una propiedad expresada en la descripción espacial viene dada
por:
DT ∂T ∂T ∂x k
T& ( x1 , x 2 , x 3 , t ) = = +
Dt ∂t ∂x k ∂t
Considerando T = x1 + x 2 , obtenemos:
∂T  ∂T ∂x1 ∂T ∂x 2 ∂T ∂x 3 
T& ( x1 , x 2 , x 3 , t ) = + + + 
∂t  ∂x1 ∂t
{ ∂x 2 {
∂t ∂x 3 {
∂t 
=0
{ =0 =0
=0

⇒ T& ( x1 , x 2 , x 3 , t ) = kX 2
Hallando la inversa de las ecuaciones del movimiento:
 x1 = X 1 + ktX 2  X 1 = x1 − ktx 2
 inversa 
x2 = X 2  →  X 2 = x 2
x = X X = x
 3 3  3 3

⇒ T& ( x1 , x 2 , x 3 , t ) = kX 2 = kx 2
Para la partícula que en la configuración actual pasa por el punto (1,1,1) :

Universidad Castilla- La Mancha Draft Por: Eduardo W. V. Chaves (2012)


Ciudad Real - España
168 PROBLEMAS RESUELTOS DE MECÁNICA DEL MEDIO CONTINUO

T& ( x1 = 1, x 2 = 1, x 3 = 1, t ) = k

Ejemplo 2.28
Dado un campo de velocidad estacionario. Se pregunta: ¿Las velocidades de las partículas
son constantes? Justificar la respuesta. En caso negativo, en situación se cumple.
Solución:
r
Un campo φ ( x , t ) se dice que es estacionario si la tasa local no varía con el tiempo:
r
∂φ ( x , t ) r
=0 ⇒ φ = φ( x ) Campo estacionario (2.89)
∂t
Un campo de velocidad estacionario (movimiento estacionario) se indica en la Figura 2.3.
Luego, como se verifica en la Figura 2.3, la representación del campo para los tiempos t1 y
t 2 no cambia. Pero, eso no implica que las velocidades de las partículas no estén
r
cambiando a lo largo del tiempo. Fijemos nuestra atención en un punto espacial fijo x * . En
r r
el tiempo t1 la partícula Q está pasando por x * y tiene velocidad v * , consideremos
también una partícula P que está pasando por otro punto y que tiene velocidad tal que
r r r
v P (t1 ) ≠ v * . Para un tiempo t 2 la partícula P está pasando ahora por el punto x * y si el
r r
campo es estacionario, la velocidad de la partícula P en x * tiene que ser v * , i.e.
r r
v P (t 2 ) = v * . Esto se puede verificar fácilmente a través de la derivada material de la
velocidad (que está asociada siempre con la misma partícula):
r r r r
Dv ( x , t ) r r ∂v ( x , t ) r r r r r r r r
≡ a ( x, t ) = + ∇ xr v ⋅ v ( x , t ) = ∇ xr v ⋅ v ( x ) = a ( x )
Dt 142∂t4 3 (2.90)
r
= 0 (Estaciona rio)

Para que la aceleración de la partícula sea igual a cero, además de campo de velocidad
r
estacionario, el campo de velocidad también tiene que ser homogéneo, i.e. ∇ xr v = 0 .
Verifiquemos también que, aunque la velocidad espacial sea independiente del tiempo la
material no necesariamente lo será, ya que:
r r r r r r r
v ( x ) = v ( x ( X , t )) = v ( X , t ) (2.91)

Universidad de Castilla- La Mancha Draft Por: Eduardo W. V. Chaves (2012)


Ciudad Real - España
2 CINEMÁTICA DEL CONTINUO 169

t1 r r
v ( x)

r r r r
v ( x * , t1 ) = v * = v Q

Partícula - P Partícula - Q
r r
v P ≠ v*
r
x*

t2 r r
v ( x)

r r r r
v ( x * , t2 ) = v * = v P

Partícula - P

r
x*

Figura 2.3: Campo estacionario.

2.1.2 Tensores de Deformación Finita, Deformación


Homogénea

Ejemplo 2.29
Una barra (considerada como un sólido de una dimensión) sufre un estiramiento uniforme
de todos sus puntos dado por:
λ = exp at (2.92)
donde a = ctte .
Se pide:
r r r
a) Obtener las ecuaciones del movimiento x = x ( X , t ) ;
b) Obtener las componentes del tensor velocidad de deformación D .

Universidad Castilla- La Mancha Draft Por: Eduardo W. V. Chaves (2012)


Ciudad Real - España
170 PROBLEMAS RESUELTOS DE MECÁNICA DEL MEDIO CONTINUO

x1

λ = exp at

Figura 2.4.

Solución:
Haciendo el planteamiento en 1D :
ds dx
λ= = = exp at ⇒ dx = exp at dX (2.93)
dS dX

∫ dx = ∫ exp
at
dX (2.94)

x1 = exp at X 1 + C (2.95)
para t = 0 ⇒ x = X , luego
x = exp 0 X 1 + C ⇒ X = X + C ⇒ C = 0 (2.96)
Obtenemos así las ecuaciones del movimiento:
 x1 = exp at X 1

 x2 = X 2 (2.97)
x = X
 3 3

El campo de velocidad:
 dx1 at
v1 = dt = a X 1 exp = a x1

v 2 = 0 (2.98)
v = 0
 3

Tensor velocidad de deformación:
 a 0 0
1  ∂v ∂v j 
D ij =  i +  ⇒ D ij =  0 0 0 (2.99)
2  ∂x j ∂xi 
  0 0 0

Ejemplo 2.30
Considérese la ley del movimiento dada por las siguientes expresiones:
 x1 = X 1 + 2 X 3

 x2 = X 2 − 2 X 3
x = X − 2 X + 2 X
 3 3 1 2

Universidad de Castilla- La Mancha Draft Por: Eduardo W. V. Chaves (2012)


Ciudad Real - España
2 CINEMÁTICA DEL CONTINUO 171

Determinar las componentes del tensor de deformación de Green-Lagrange ( E ).


Solución 1:
Campo de desplazamiento
u1 = x1 − X 1 = 2 X 3

u 2 = x 2 − X 2 = −2 X 3
u = x − X = −2 X + 2 X
 3 3 3 1 2

Partiendo de las componentes del tensor de deformación de Green-Lagrange en función


del gradiente material de los desplazamientos:
1  ∂u i ∂u j ∂u k ∂u k 

E ij = + +
2  ∂X j ∂X i ∂X i ∂X j 

1  ∂u i ∂u j  1  ∂u k ∂u k
+ 


= +
2  ∂X j ∂X i  2  ∂X i ∂X j
 


sym
 ∂u  1  ∂u k ∂u k 
= i  + 
 ∂X j  2  ∂X i ∂X j 
  
donde el gradiente material de los desplazamientos viene dados por:
 ∂u1 ∂u1 ∂u1 
 
 ∂X 1 ∂X 2 ∂X 3   0 0 2 
∂u i  ∂u 2 ∂u 2 ∂u 2  
= =  0 0 − 2
∂X j  ∂X 1 ∂X 2 ∂X 3 
   
 ∂u 3 ∂u 3 ∂u 3  − 2 2 0 
 ∂X 1 ∂X 2 ∂X 3 
Verifiquemos que el gradiente material de los desplazamientos es un tensor antisimétrico.
Es decir, la parte simétrica es el tensor nulo. Sólo queda el término:
  0 0 2  T  0 0 2    2 − 2 0
1  ∂u ∂u k  1  
E ij =  k  =  0 0 − 2   0 0 − 2  =  − 2 2 0 
2  ∂X i ∂X j  2      
   − 2 2 0  − 2 2 0    0 0 4
 
Solución 2:
Podemos aplicar directamente la definición:

E ij =
1
2
( 1
) (
C ij − δ ij = Fki Fkj − δ ij
2
)
donde:
 ∂x1 ∂x1 ∂x1 
 
 ∂X 1 ∂X 2 ∂X 3   1 0 2 
∂xi  ∂x 2 ∂x 2 ∂x 2  
Fij = = = 0 1 − 2
∂X j  ∂X 1 ∂X 2 ∂X 3  
   
 ∂x3 ∂x3 ∂x3  − 2 2 1 
 ∂X 1 ∂X 2 ∂X 3 
Luego:

Universidad Castilla- La Mancha Draft Por: Eduardo W. V. Chaves (2012)


Ciudad Real - España
172 PROBLEMAS RESUELTOS DE MECÁNICA DEL MEDIO CONTINUO

  1 0 2  T  1 0 2  1 0 0   2 − 2 0
1 
E ij =   0 1 − 2  0 1 − 2 − 0 1 0  = − 2 2 0
2 
  − 2 2 1  − 2 2 1  0 0 1   0 0 4
 

Ejemplo 2.31
Consideremos una transformación homogénea definida por las siguientes ecuaciones:
 x1 = X 1 + 2 X 2 + X 3

 x2 = 2 X 2 (2.100)
x = X + 2 X
 3 1 3

Demostrar que para una transformación homogénea, vectores paralelos en la configuración


de referencia, siguen paralelos tras la deformación.
Para la demostración considere dos partículas A y B cuyos vectores posición en la
configuración de referencia son:
r
X A = eˆ 1 + eˆ 2
r (2.101)
X B = 2eˆ 1 + 2eˆ 2 + eˆ 3

Solución:
El vector que une las dos partículas en la configuración de referencia viene dado por:
r r r
V = B − A = eˆ 1 + eˆ 2 + eˆ 3 (2.102)
El gradiente de deformación:
1 2 1 
∂xi 
Fij = = 0 2 0 (2.103)
∂X j 
1 0 2

Podemos obtener los vectores posición de las partículas en la configuración actual:


r r
dx = F ⋅ dX
⇒ Transformación homogénea (2.104)
r r
x=F⋅X

luego,
1 2 1  1 3 1 2 1   2 7 
xiA = 0 2 0 1 = 2 ; xiB = 0 2 0  2 =  4 (2.105)
1 0 2 0 1  1 0 2 1   4

y el vector que une estos dos puntos es:


r r r
v = x B − x A = 4eˆ 1 + 2eˆ 2 + 3eˆ 3 (2.106)
r
luego cualquier vector paralelo a V , por ejemplo 2eˆ 1 + 2eˆ 2 + 2eˆ 3 , después de la
r
transformación: 8eˆ 1 + 4eˆ 2 + 6eˆ 3 , es paralelo a v .

Universidad de Castilla- La Mancha Draft Por: Eduardo W. V. Chaves (2012)


Ciudad Real - España
2 CINEMÁTICA DEL CONTINUO 173

Ejemplo 2.32
Considere una base ortonormal cartesiana ê i y considere una deformación de corte puro
representada por la deformación homogénea:
r r
x = X + k t X 2ê1 (2.107)
o explícitamente:
 x1 = X 1 + k t X 2

 x2 = X 2 (2.108)
x = X
 3 3

Obtener la forma geométrica en la configuración actual de la Figura 2.5 representada por


un rectángulo en la configuración de referencia.
X2

B C

O A X1
Figura 2.5

Solución:
El gradiente de deformación:
1 k t 0
∂xi 
Fij = = 0 1 0 (2.109)
∂X j 
0 0 1
r r r r r
Verificamos que se trata de un caso de deformación homogénea, x = F ⋅ X + c con c = 0 .
Determinante del Jacobiano:
J = F =1 (2.110)
Verificamos que para este caso no hay dilatancia.
Para la línea BC , que tiene como coordenadas en la configuración de referencia ( X 1 , X 2 ,0)
tenemos que:
 x1 = X 1 + k t X 2

 x2 = X 2 (2.111)
x = 0
 3
Para la línea OA , de coordenadas ( X 1 ,0,0) :
 x1 = X 1

 x2 = 0 (2.112)
x = 0
 3

Universidad Castilla- La Mancha Draft Por: Eduardo W. V. Chaves (2012)


Ciudad Real - España
174 PROBLEMAS RESUELTOS DE MECÁNICA DEL MEDIO CONTINUO

luego no se mueve. La deformación final se puede apreciar en la Figura 2.6.

x2

B B′ C C′

O A x1
Figura 2.6

Ejemplo 2.33
Sean las ecuaciones del movimiento:
2 2
x1 = X 1 + X2 ; x2 = X1 + X 2 ; x3 = X 3 (2.113)
2 2
Se pide:
a) Probar que esta deformación es un ejemplo de transformación homogénea;
r
b) Determinar las componentes del campo de desplazamientos u en coordenadas
materiales y espaciales;
c) Determinar en la configuración actual la figura geométrica formada por las
partículas que en la configuración de referencia formaban un círculo:
X 12 + X 22 = 2 X3 = 0
d) Obtener las componentes del tensor derecho de deformación de Cauchy-Green
( C ) y del tensor de deformación de Green-Lagrange ( E ).
e) Obtener los valores principales de los tensores C y E .
Solución:
a) Una transformación homogénea si es el del tipo:
xi = Fij X j (2.114)
donde
 2 
 1 0
 2 
∂xi  2 
Fij = = 1 0 (2.115)
∂X j  2
 0 0 1
 
 

Universidad de Castilla- La Mancha Draft Por: Eduardo W. V. Chaves (2012)


Ciudad Real - España
2 CINEMÁTICA DEL CONTINUO 175

 2 
 1 0
 x1   2   X1 
x  =  2 
 2  1 0  X 2  (2.116)
2
 x3   0 0 1  X 3 
 
 

Comprobando que es una transformación homogénea. Su forma inversa se obtiene las


ecuaciones del movimiento en coordenadas espaciales:

 X1   2 − 2 0  x1   X 1 = 2 x1 − 2 x 2
 X  = − 2  
 2  2 0  x 2  ⇒  X 2 = − 2 x1 + 2 x 2 (2.117)
 X 3   0 0 1  x3  X = x
   3 3

b) El campo de desplazamientos viene dado por:


 2 2
u1 = x1 − X 1 = X 1 + X 2 − X1 = X2
 2 2
r r r  2 2
u= x − X componente
  s → u 2 = x 2 − X 2 = X1 + X 2 − X 2 = X1 (2.118)
 2 2
u 3 = x 3 − X 3 = 0



En coordenadas espaciales queda:

(
u1 = x1 − X 1 = x1 − 2 x1 − 2 x 2 = − x1 + 2 x 2 )
(
u 2 = x 2 − X 2 = x 2 − − 2 x1 + 2 x 2 = 2 x1 − x 2 ) (2.119)
u 3 = x 3 − X 3 = x3 − x3 = 0

c) Dada la ecuación de las partículas que en la configuración de referencia formaba un


círculo:
X 12 + X 22 = 2 ; X3 = 0 (2.120)
En la configuración actual queda:

(2 x
1 − 2 x2 ) + (−
2
2 x1 + 2 x 2 ) 2
=2 (2.121)

Desarrollando obtenemos que:


3x12 + 3 x 22 − 4 2 x1 x 2 = 1 (ecuación de una elipse) (2.122)
Ver Figura 2.7. Es interesante que el lector verifique las posiciones de las partículas
P ( X 1 = 0; X 2 = 2 ; X 3 = 0) y Q( X 1 = 2 ; X 2 = 2 ; X 3 = 0) en la configuración deformada.

Universidad Castilla- La Mancha Draft Por: Eduardo W. V. Chaves (2012)


Ciudad Real - España
176 PROBLEMAS RESUELTOS DE MECÁNICA DEL MEDIO CONTINUO

1,5
Conf. de Referencia
Conf. Deformada
1

0,5

0
x2

-2 -1 0 1 2

-0,5

-1

-1,5

-2
x1

Figura 2.7: Curva material

d) El tensor derecho de deformación de Cauchy-Green y el tensor de deformación de


Green-Lagrange vienen dados, respectivamente, por:
1
C = FT ⋅F ; E= (C − 1) (2.123)
2
Luego las componentes de C son:
 2  2 
 1 0  1 0  3
2

0
2 2 
    2

 2  2  3
C ij =  1 0  1 0 =  2 0 (2.124)
2 2  2 
 0 0 1  0 0 1  0 0 1
    
    

Obteniendo los autovalores de (2.124). Ya conocemos un autovalor C 3 = 1 . Para obtener


los otros autovalores es suficiente resolver:

3  3+ 2 2
−C 2 C1 = ≈ 2,91421
2 2 1  2
=0 ⇒ C − 3C + = 0 ⇒  (2.125)
3 4 3−2 2
2 −E 
2 C 2 = 2
≈ 0,08579

Resumiendo:
3 3
C1 = + 2 ; C 2 = − 2 ; C3 = 1 (2.126)
2 2

Universidad de Castilla- La Mancha Draft Por: Eduardo W. V. Chaves (2012)


Ciudad Real - España
2 CINEMÁTICA DEL CONTINUO 177

Las componentes del tensor E son:


 3  
 2 0 
 2 1 0 0    1 2 2 0
1   1 
1
(
E ij = C ij − δ ij ) = 
2 
2
3   
0  − 0 1 0   =  2 2 1 0 (2.127)
2 2  4
 0 0 1 0 0 1  0 0 0
   

 
Los valores principales de E son obtenidos por:

1 2  1+ 2 2
−E E1 = ≈ 0,95711
4 2 2 E 7  4
=0 ⇒ E − − =0⇒ (2.128)
2 1 2 16  1− 2 2
−E
2 4 E 2 = 4
≈ −0,45711

Luego, los valores principales de E son:


1+ 2 2 1− 2 2
E1 = ; E2 = ; E3 = 0 (2.129)
4 4
Solución Alternativa:
Recordar que los tensores C y E son coaxiales, es decir, podemos trabajar en el espacio
principal para obtener los autovalores de E :
 3 + 2 2   1 + 2 2 
 0 0   0 0
 2  1 0 0   4 
1  3−2 2   1− 2 2
E ij′ =
1
2
(
C ij′ − δ ij ) =  0
2 2

0 − 0 1 0  =  0
4

0 (2.130)
 
 0 0 1 0 0 1   0 0 0
  
    
  

Ejemplo 2.34
Consideremos las siguientes ecuaciones del movimiento:
11
x1 = X 1 + X1 + X 2
X2 ; ; x2 =
x3 = X 3 (2.131)
22
r
a) Obtener el campo de desplazamiento ( u ) en las descripciones Lagrangiana y Euleriana;
b) Determinar la curva material en la configuración actual de una circunferencia material
definido en la configuración de referencia como:
X 12 + X 22 = 2 X3 = 0
c) Obtener las componentes del tensor derecho de deformación de Cauchy-Green y el
tensor de deformación de Green-Lagrange;
d) Obtener los estiramientos principales.
Solución:
El gradiente de deformación viene dado por:

Universidad Castilla- La Mancha Draft Por: Eduardo W. V. Chaves (2012)


Ciudad Real - España
178 PROBLEMAS RESUELTOS DE MECÁNICA DEL MEDIO CONTINUO

 2 1 0
∂xi 1
Fij = = 1 2 0 ; J = F = 0.75
∂X j 2 
0 0 2

y combinando con las ecuaciones del movimiento dada en (2.131), obtenemos que:
 x1  2 1 0  X 1 
 x  = 1 1 2 0   X  ; x i = Fij X j
 2 2   2 
 x3  0 0 2  X 3 

Con eso, podemos verificar que el ejemplo propuesto es un caso de deformación


r r
homogénea con c = 0 . La forma inversa de la ecuación anterior es:
 4 2
 X 1 = 3 x1 − 3 x 2
 X1   4 − 2 0  x1  
 X  = 1  − 2 4 0  x   2 4
 2 3  2  ⇒  X 2 = − x1 + x 2 (2.132)
 3 3
 X 3   0 0 3  x3 
 X 3 = x3


r r r
El campo de desplazamiento viene definido por u = x − X , con lo cual las componentes
del desplazamiento Lagrangiano resulta ser:
 r 1 1
u1 ( X , t ) = x1 − X 1 = X 1 + 2 X 2 − X 1 = 2 X 2
 r
 1 1
u i = xi − X i ⇒ u 2 ( X , t ) = x 2 − X 2 = X 1 + X 2 − X 2 = X 1 (2.133)
 2 2
r
u 3 ( X , t ) = x3 − X 3 = 0


Las componentes del desplazamiento Euleriana pueden ser obtenidas por reemplazar las
ecuaciones del movimiento Euleriano (2.132) en (2.133), luego:
 r r 1 r 1 2 4  r
 u1 ( X ( x, t ), t ) = X 2 ( x , t ) =  − x1 + x 2  = u1 ( x , t )
 2 2 3 3 
 r r 1 r 1 2 4  r
u 2 ( X ( x , t ), t ) = X 1 ( x , t ) =  − x1 + x 2  = u 2 ( x , t ) (2.134)
 2 2 3 3 
r r
u ( X ( x , t ), t ) = u ( xr , t ) = 0
 3 3


Las partículas que pertenecen a la circunferencia X 12 + X 22 = 2 en la configuración de


referencia, formarán una nueva curva en la configuración actual y que viene definida por:
2 2
4 2   2 4 
X 12 + X 22 = 2 ⇒  x1 − x 2  +  − x1 + x 2  = 2 ⇒ 20 x12 − 32 x1 x 2 + 20 x 22 = 18
 3 3   3 3 
el cual es la ecuación de una elipse (Figura 2.8 muestra la curva material en las diferentes
configuraciones).
Las componentes de C y E pueden ser obtenidas a través de las definiciones C = F T ⋅ F
1
y E= (C − 1) :
2

Universidad de Castilla- La Mancha Draft Por: Eduardo W. V. Chaves (2012)


Ciudad Real - España
2 CINEMÁTICA DEL CONTINUO 179

 2 1 0  2 1 0 1.25 1 0
1
C ij = Fki Fkj ⇒ C ij = 1 2 0  1 2 0 =  1 1.25 0
4
0 0 2 0 0 2  0 0 1

 1.25 1 0 1 0 0  0.125 0.5 0


1  
1
(
E ij = C ij − δ ij
2
) ⇒ E ij =   1 1.25 0 − 0 1 0  =  0.5 0.125 0
2
  0 0 1  0 0 1    0 0 0

En el espacio principal de C sus componentes vienen dadas por:


λ21 0 0 λ 1 0 0
   
C ij′ =  0 λ22 0 ⇒ C ij′ =  0 λ2 0
0 0 λ23  0 0 λ 3 
 
donde λ i son los estiramientos principales. A continuación, calculamos los autovalores de
C:
1.25 − C 1 C1 = 2.25
= 0 ⇒ C 2 − 2.5C + 0.5625 = 0 ⇒ 
1 1.25 − C C 2 = 0.25
λ21 0 0  2.25 0 0 λ 1 0 0  1.5 0 0
    
C ij′ =  0 λ22 0= 0 0.25 0 ⇒ 0 λ2 0  =  0 0.5 0
0 0 λ23   0 0 1 0 0 λ 3   0 0 1 
 

2.0
curva material
1.5
Reference Conf.
Current Conf.
1.0

0.5

0.0
x2

-2 -1 0 1 2
-0.5

-1.0

-1.5

-2.0
x1

Figura 2.8: Curva material.

Universidad Castilla- La Mancha Draft Por: Eduardo W. V. Chaves (2012)


Ciudad Real - España
180 PROBLEMAS RESUELTOS DE MECÁNICA DEL MEDIO CONTINUO

Ejemplo 2.35
Probar que

⋅ [(detF ) F −T ] = 0
r
∇ Xr (2.135)
r r r
Datos: Relación de Nanson da = J F −T ⋅ dA , o da = da nˆ = J F −T ⋅ N
ˆ dA .

Solución:
Considerando la relación de Nanson en notación indicial da nˆ i = J Fki−1Nˆ k dA , donde
J = detF . Podemos integrar en toda la superficie:
−1 ˆ
∫ nˆ da = ∫ J F
S
i
S0
ki N k dA (2.136)

Fijemos que dado una función escalar f , se cumple que:


∂f
∫ nˆ
S
i f da = ∫
V
f ,i dV = ∫
V
∂xi
dV

Haciendo f = 1 , obtenemos que:

∫ nˆ
S
i da = 0 i

Retomando la ecuación (2.136), y aplicando el teorema de la divergencia de Gauss para la


segunda integral, obtenemos que:

∫ (J F ), ( )

∫ nˆ da = 0 = ∫ J F
−1 ˆ

−1
i i ki N k dA = ki k dV0 = J Fki−1 dV0 = 0 i
S S0 V0 V0
∂X k
(2.137)
⋅ [(detF ) F −T ] dV0 = 0
r

V0
∇ Xr

Luego si es válido para todo el volumen, tiene que ser válido localmente:

⋅ [(detF ) F −T ] = 0
r
∇ Xr (2.138)

Ejemplo 2.36

[
Demostrar que a) E& = F −T ⋅ ∇ Xr u& ( X , t)
r r
]sym
[
y b) D = ∇ xr u& ( x , t) , donde E es el tensor de
r r
]sym

deformación de Green-Lagrange y D es el tensor velocidad de deformación.


Solución:
a)

E& ≡
D
Dt
E=
D 1 T

Dt  2
( 1
) 1
(
F ⋅ F − 1  = F& T ⋅ F + F T ⋅ F& = ( F T ⋅ F& ) T + ( F T ⋅ F& ) = F T ⋅ F& ) [ ] [ ]sym

 2 2
Notar que:

F&ij =
D  ∂xi ( X , t ) 
Dt  ∂X j  ∂X j
=
∂ Dxi ( X , t )
Dt
=
∂ &
∂X j
[ ∂u& ( X , t )
u i ( X , t )] = i
∂X j
r r
= ∇ Xr u& ( X , t ) ij ( )

Universidad de Castilla- La Mancha Draft Por: Eduardo W. V. Chaves (2012)


Ciudad Real - España
2 CINEMÁTICA DEL CONTINUO 181

con lo cual, demostramos que:

E& ≡
D
Dt
[
E = F T ⋅ F& ] sym
[ r r sym
]
= F T ⋅ ∇ Xr u& ( X , t )

b)

D=l
1
2
= T
[ 1
2
] [
r r T r r
l + ( l ) = ∇ xr v + (∇ xr v ) = (∇ xr v ( x , t ))
sym sym
] r r sym
= ∇ xr u& ( x , t ) [ ]
r r r r
donde hemos considerado que v ( x , t ) = u& ( x , t ) .

Ejemplo 2.37
Considérese el siguiente campo de velocidad:
v1 = −5 x 2 + 2 x3

v 2 = 5 x1 − 3 x3
v = −2 x + 3 x
 3 1 2

Demostrar que dicho movimiento corresponde a un movimiento de sólido rígido.


Solución:
En primero vamos obtener el gradiente espacial de la velocidad (l ), cuyas componentes
vienen dadas por:
 ∂v1 ∂v1 ∂v1 
 
r  ∂x1 ∂x 2 ∂x3   0 − 5 2 
∂vi ( x, t )  ∂v 2 ∂v 2 ∂v 2  
l ij = = = 5 0 − 3 (2.139)
∂x j ∂x ∂x 2 ∂x3  
 1   0 
 ∂v3 ∂v3 ∂v3   − 2 3
 ∂x1 ∂x 2 ∂x3 

Recordar que ( l ) podemos descomponer en una parte simétrica ( D ) y otra antisimétrica


( W ). También podemos verificar que l = D + W = W . Ya que D = 0 , el movimiento es de
sólido rígido.

Ejemplo 2.38
r
Dada una función escalar en la configuración de referencia φ ( X , t ) . Obtener la relación
r r
entre el gradiente material de φ ( X , t ) , es decir, ∇ Xr φ( X , t ) , y el gradiente espacial de
r r
φ ( x , t ) , es decir, ∇ xr φ( x , t ) .
Solución:
Recordemos que una variable que está en la configuración de referencia la podemos
expresar en la configuración actual a través de las ecuaciones del movimiento, es decir,
r r r r
φ ( X , t ) = φ( X ( x , t ), t ) = φ( x , t ) . Luego, partiendo de la definición del gradiente de un
escalar podemos obtener que:
r r r r r
r ∂φ( X , t ) ∂φ ( X ( x , t ), t ) ∂x ∂φ ( x , t ) r
∇ Xr φ( X , t ) = r = r ⋅ r= r ⋅ F = ∇ xr φ ( x , t ) ⋅ F
∂X ∂x ∂X ∂x
Y la forma inversa:

Universidad Castilla- La Mancha Draft Por: Eduardo W. V. Chaves (2012)


Ciudad Real - España
182 PROBLEMAS RESUELTOS DE MECÁNICA DEL MEDIO CONTINUO

r r r r r
r ∂φ ( x , t ) ∂φ ( x ( X , t ), t ) ∂X ∂φ ( X , t) r
∇ xr φ( x , t ) = r = r ⋅ r= r ⋅ F −1 = ∇ Xr φ( X , t ) ⋅ F −1
∂x ∂X ∂x ∂X

Ejemplo 2.39
El campo de desplazamientos de un cuerpo viene descrito por las siguientes ecuaciones:
u1 = 3 X 12 + X 2
 2
u 2 = 2 X 2 + X 3
 2
u 3 = 4 X 3 + X 1
r
Determinar el vector dx (configuración actual) cuyo vector en la configuración de
r
referencia estaba representado por dX y pasaba por el punto P(1,1,1) .
X 3 , x3
 dX 1 
dX k =  dX 2 
Q  dX 3 
r
dX
P
X 2 , x2

X 1 , x1

Solución:
r
Para determinar el vector dx necesitamos obtener el gradiente de deformación F . Las
componentes del gradiente de deformación material pueden obtenerse utilizando
directamente la ecuación:
∂u i
Fij = δ ij +
∂X j

1 + 6 X 1 1 0 

Fij =  0 1 + 4X 2 1 

 1 0 1 + 8 X 3 

Las componentes del gradiente de deformación en el punto P(1,1,1) son:


7 1 0 
Fij = 0 5 1 
P
1 0 9 
r
Una vez obtenido el gradiente de deformación F , las componentes del vector dx vienen
dadas por:
dx i = Fij dX j

 dx1  7 1 0   dX 1   7 dX 1 + dX 2 
 dx  = 0 5 1   dX  = 5dX + dX 
 2   2   2 3
 dx 3  1 0 9   dX 3   dX 1 + 9 dX 3 

Universidad de Castilla- La Mancha Draft Por: Eduardo W. V. Chaves (2012)


Ciudad Real - España
2 CINEMÁTICA DEL CONTINUO 183

Ejemplo 2.40
Dadas las componentes del campo de desplazamientos siguientes:
u1 = 2 X 12 + X 1 X 2
 2
u 2 = X 2 para X 1 ≥ 0; X 2 ≥ 0
u = 0
 3
Se pide:
a) Encontrar el vector en la configuración de referencia cuyo vector en la configuración

actual es dx i = (OP ′) i = (1,0,0) ;
b) Encontrar el estiramiento de un elemento de línea que en la configuración actual es el

vector dx i = (OP ′) i = (1,0,0) y que pasa por el punto P(1,0,0) .
Solución:
a) Dadas las componentes del desplazamiento podemos obtener las componentes del
movimiento según la ecuación:
u i = xi − X i

 x1 = u1 + X 1  x1 = X 1 + 2 X 12 + X 1 X 2
 
x2 = u 2 + X 2 sustituyen
 do
→  x 2 = X 2 + X 22
valores de u1 ,u 2 ,u 3
x = u + X x = X
 3 3 3  3 3

Podemos verificar que no se trata de una deformación homogénea, ya que una recta en la
configuración de referencia no sigue siendo una recta en la configuración deformada.
Como ejemplo consideremos que unas partículas que ocupan una recta en la configuración
de referencia, tras la deformada estas partícula ya no formarán una recta en la configuración
actual, ver Figura 2.9.

2,5

2
P

1,5
x2

P
1

Q Conf. Actual
Q
0,5 Conf. Referencia

0
0 0,5 1 1,5 2 2,5 3 3,5 4 4,5
x1

Figura 2.9: Deformación de la recta (1,1).

Universidad Castilla- La Mancha Draft Por: Eduardo W. V. Chaves (2012)


Ciudad Real - España
184 PROBLEMAS RESUELTOS DE MECÁNICA DEL MEDIO CONTINUO

Reemplazando el punto P ( x1 = 1, x 2 = 0, x 3 = 0) en las ecuaciones del movimiento anterior,


resulta:
1 = X 1 + 2 X 12 + X 1 X 2
 2
0 = X 2 + X 2
0 = X
 3

Podemos decir de inmediato que X 3 = 0 y X 22 = − X 2 , debido a la restricción del problema


X 2 ≥ 0 , luego la única solución posible es X 2 = 0 . Sustituyendo los valores de X 3 = 0 y
X 2 = 0 en la primera ecuación resulta:

resolviend o
 X 1 = −1
1 = X 1 + 2 X 12   → 
 X 1 = 12

Debido a la restricción X 1 ≥ 0 , la única solución posible es X 1 = 12 . Así:


( X 1 = 12 ; X 2 = 0; X 3 = 0)
Es decir, la partícula que en la configuración actual ocupa ( x1 = 1, x 2 = 0, x 3 = 0) , en la
configuración de referencia ocupaba el lugar ( X 1 = 12 ; X 2 = 0; X 3 = 0;) .
r
Calculemos el incremento en esta dirección dX , según la ecuación dx j = F jk dX k y su
forma inversa:
 dX 1   dx1 
dX  = F −1  dx 
 2 jk  2
 dX 3   dx 3 

Para lo cual debemos calcular antes las componentes del gradiente de deformación material
F jk . Podemos hacerlo directamente partiendo de la definición:

 ∂x1 ∂x1 ∂x1 


 
 ∂X 1 ∂X 2 ∂X 3  (1 + 4 X + X )
1 2 X1 0
 ∂x ∂x 2 ∂x 2  
F jk = 2 = 0 1 + 2X 2 0 
∂X ∂X 2 ∂X 3  
 1   0 0 1 
 ∂x 3 ∂x 3 ∂x 3 
 ∂X 1 ∂X 2 ∂X 3 

Para el punto P ( X 1 = 12 ; X 2 = 0; X 3 = 0;) obtenemos las componentes del gradiente de


deformación:
3 12 0
 
F jk = 0 1 0 
P
0 0 1 
 
Calculamos la inversa ( F jk ) −1 ≡ F jk−1 . Por definición:
T
 1 0 0 1 − 0,5 0
1 1  1
F jk−1 = adj( F jk ) =  − 0,5 3 0  = 0 3 0
F jk 3 3
 0 0 3 0 0 3

Luego:

Universidad de Castilla- La Mancha Draft Por: Eduardo W. V. Chaves (2012)


Ciudad Real - España
2 CINEMÁTICA DEL CONTINUO 185

 dX 1   dx1  1 − 0,5 0 1   13 
 dX  = F −1  dx  = 1 0  
 2 jk  2  3 0 0 = 0
3
 dX 3   dx 3  0 0 3 0 0

b) El estiramiento viene dado por:


r
dx 12 + 0 + 0
λ= r = =3
dX (13 )2 + 0 + 0

0,1
0,08
0,06 Conf. Actual
x2

0,04 Conf. Referencia


0,02
0
0 0,5 1 1,5 2 2,5 3 3,5
x1

Ejemplo 2.41
DF
Utilizar la definición
D
Dt
[det (F )] = ij cof Fij
Dt
( ) para obtener la expresión
D
[det (F )] = Jv i ,i .
Dt
∂x i
Solución: Considerando que Fij = , luego:
∂X j

 
D
[det (F )] = D  ∂xi cof Fij ( )
Dt Dt  ∂X j 
D  ∂x i 
= 
∂X j  Dt 
cof Fij ( )
=
D
∂X j
( )
(vi )cof Fij
r
o aún considerando que v i ( x ( X , t ), t ) , podemos decir que:
D
[det (F )] = ∂vi ∂x k cof Fij ( )
Dt ∂x k ∂X j

Y considerando la definición del cofactor: [cof (Fij )]T = (Fij )−1 det (Fij ) , obtenemos que:

Universidad Castilla- La Mancha Draft Por: Eduardo W. V. Chaves (2012)


Ciudad Real - España
186 PROBLEMAS RESUELTOS DE MECÁNICA DEL MEDIO CONTINUO

D
[det (F )] = ∂v i ∂x k Fij ( ) −T
( )
det Fij
Dt ∂x k ∂X j
∂v i
=
∂x k
Fkj F ji( )−1
( )
det Fij

∂v i ∂v
=
∂x k
( )
δ ki det Fij = i det Fij
∂x i
( )
= Jv i ,i

Ejemplo 2.42
r
Dado el diferencial dx , hallar su derivada material.
Solución:
D r D r
dx = ( F ⋅ dX )
Dt Dt
D r D r
= ( F ) ⋅ dX + F ⋅ ( dX )
Dt Dt
1
424 3 r
0
r
= l ⋅1⋅ d3
F2 X
r
dx
r
= l ⋅ dx
r r
= ∇ xr v ⋅ dx
Las componentes vienen dadas por:
 D r ∂v
 dx  = v i , k dx k = i dx k
 Dt  i ∂x k

Ejemplo 2.43
Considerando las ecuaciones del movimiento:
 x1 = X 1 + 4 X 1 X 2
 2
x2 = X 2 + X 2
 2
 x3 = X 3 + X 3
Encontrar el tensor de deformación de Green-Lagrange E .
Solución:
El tensor de deformación de Green-Lagrange viene dado por:
1
E= ( F T ⋅ F − 1)
2
(2.140)
1
E ij = ( Fki Fkj − δ ij )
2

Considerando las ecuaciones del movimiento podemos obtener las componentes del
gradiente de deformación material F :

Universidad de Castilla- La Mancha Draft Por: Eduardo W. V. Chaves (2012)


Ciudad Real - España
2 CINEMÁTICA DEL CONTINUO 187

 ∂x1 ∂x1 ∂x1 


 
 ∂X 1 ∂X 2 ∂X 3  (1 + 4 X )
2 4X1 0 
∂x  ∂x ∂x 2 ∂x 2  
Fkj = k =  2 = 0 1 + 2X 2 0 
∂X j ∂X ∂X 2 ∂X 3  
 1   0 0 1 + 2 X 3 
 ∂x 3 ∂x 3 ∂x 3  
 ∂X 1 ∂X 2 ∂X 3 

(1 + 4 X 2 ) 0 0  (1 + 4 X 2 ) 4X1 0 


Fki Fkj 
=  4X1 1 + 2X 2 0    0 1+ 2X 2 0 
 0 0 1 + 2 X 3   0 0 1 + 2 X 3 
 (1 + 4 X 2 ) 2
(1 + 4 X 2 ) 4 X 1 0 
 2 2 
= (1 + 4 X 2 ) 4 X 1 ( 4 X 1 ) + (1 + 2 X 2 ) 0 
 0 0 2
(1 + 2 X 3 ) 

Reemplazando la relación anterior en la ecuación (2.140) obtenemos que las componentes
del tensor de deformación de Green-Lagrange vienen dadas por:
(1 + 4 X 2 ) 2 − 1 (1 + 4 X 2 ) 4 X 1 0 
1 2 2 
E ij =  (1 + 4 X 2 ) 4 X 1 ( 4 X 1 ) + (1 + 2 X 2 ) − 1 0 
2 2 
 0 0 (1 + 2 X 3 ) − 1

Ejemplo 2.44
Obtener los invariantes principales de E en función de los invariantes principales de C y
b.
Solución:
Los invariantes principales de E son:

I E = Tr ( E ) ; II E =
2
[
1 2
I E − Tr ( E 2 ) ] ; III E = det ( E )

1
Considerando que E = (C − 1) , resulta que:
2
Primer Invariante:
1  1 1 1
I E = Tr ( E ) =  (C − 1)  = Tr (C − 1) = [Tr (C ) − Tr (1) ] = (I C − 3)
2  2 2 2
Segundo Invariante:

II E =
1 2
2
[
I E − Tr ( E 2 ) ]
donde
2
1  1
(
I E2 =  (I C − 3) = I C2 − 6 I C + 9 )
2  4

Universidad Castilla- La Mancha Draft Por: Eduardo W. V. Chaves (2012)


Ciudad Real - España
188 PROBLEMAS RESUELTOS DE MECÁNICA DEL MEDIO CONTINUO

[ ]
2
1  1 1
Tr ( E 2 ) = Tr  (C − 1)  = Tr (C − 1) 2 = Tr C 2 − 2C + 1 ( )
2  4 4
1
[ ( )
= Tr C 2 − 2 Tr (C ) + Tr (1 )
4
]
1
[ ( )
= Tr C 2 − 2 I C + 3
4
]
Para obtener Tr (C 2 ) , adoptaremos el espacio de las direcciones principales donde se
cumple que:
C12 0 0 

C ⋅C = C 2 =  0 C 22

( )
0  ⇒ Tr C 2 = C12 + C 22 + C 32
 0 0 C 32 

Pero considerando la siguiente relación:
I C2 = (C1 + C 2 + C 3 ) = C12 + C 22 + C 32 + 2 C1 C 2 + C1 C 3 + C 2 C 3
2
1444424444 3
( )
II C

⇒ C12 + C 22 + C 32 = I C2 − 2 II C
Luego:

Tr ( E 2 ) =
1 2
4
(
I C − 2 II C − 2 I C + 3 )
Con lo cual concluimos que el segundo invariante viene dado por:

II E =
1 1 2

2 4
( 1
) 
I C − 6 I C + 9 − I C2 − 2 II C − 2 I C + 3 
4
( )

1
= (− 2 I C + II C + 3)
4
Tercer Invariante:
3
1  1
III E = det ( E ) = det  (C − 1)  =   det [(C − 1)]
2  2
Trabajando en las direcciones principales tenemos que:
C1 − 1 0 0
det (C − 1) = 0 C2 − 1 0 = (C1 − 1)(C 2 − 1)(C 3 − 1)
0 0 C3 − 1
= C1 C 2 C 3 − C 1 C 2 − C1 C 3 − C 2 C 3 + C 1 + C 2 + C 3 − 1
= III C − II C + I C − 1
luego:
1
III E = ( III C − II C + I C − 1)
8
Resumiendo:

Universidad de Castilla- La Mancha Draft Por: Eduardo W. V. Chaves (2012)


Ciudad Real - España
2 CINEMÁTICA DEL CONTINUO 189

1
IE = (I C − 3 ) I C = 2I E + 3
2
1
II E = (− 2 I C + II C + 3) INVERSA
 → II C = 4 II E + 4 I E + 3
4
1
III E = ( III C − II C + I C − 1) III C = 8 III E + 4 II E + 2 I E + 1
8

Ejemplo 2.45
Sea Ψ = Ψ (I C , II C , III C ) una función de valor-escalar, donde I C , II C , III C son los
invariantes principales del tensor derecho de deformación de Cauchy-Green C . Obtener la
derivada de Ψ con respecto a C y con respecto a b . Comprobar que la siguiente igualdad
es válida F ⋅ Ψ ,C ⋅ F T = Ψ ,b ⋅ b .
Solución:
Utilizando la regla de la cadena podemos obtener que:
∂Ψ (I C , II C , III C ) ∂Ψ ∂I C ∂Ψ ∂ II C ∂Ψ ∂ III C
Ψ ,C = = + + (2.141)
∂C ∂I C ∂C ∂ II C ∂C ∂ III C ∂C

Considerando las derivadas parciales de los invariantes vistas en el capítulo 1, podemos


decir que:
∂I C ∂ II C ∂ III C
=1 , = IC 1 − C T = IC 1 − C , = III C C −T = III C C −1 , luego:
∂C ∂C ∂C

∂Ψ ∂Ψ
Ψ ,C = 1+ (I C 1 − C ) + ∂Ψ III C C −1
∂I C ∂ II C ∂ III C
(2.142)
 ∂Ψ ∂Ψ  ∂Ψ ∂Ψ
Ψ ,C =  + I C 1 − C+ III C C −1

 C I ∂ II C  ∂ II C ∂ III C

También es válido que:

 ∂Ψ ∂Ψ  ∂Ψ ∂Ψ
Ψ ,b =  + I b 1 − b+ III b b −1 (2.143)
 ∂I b ∂ II b  ∂ II b ∂ III b

Haciendo una contracción por la izquierda con F y por la derecha por F T en la relación
(2.142) obtenemos que:
 ∂Ψ ∂Ψ  ∂Ψ ∂Ψ
F ⋅ Ψ ,C ⋅ F T =  + I C  F ⋅ 1 ⋅ F T − F ⋅C ⋅ F T + III C F ⋅ C −1 ⋅ F T
 ∂I C ∂ II C  ∂ II C ∂ III C
(2.144)
Y considerando las siguientes relaciones:
⇒ F ⋅1 ⋅ F T = F ⋅ F T = b
C = F T ⋅ F ⇒ F ⋅C ⋅ F T = F ⋅ F T ⋅ F ⋅ F T = b ⋅ b = b2
Y considerando la relación C −1 = F −1 ⋅ b −1 ⋅ F concluimos que:
C −1 = F −1 ⋅ b −1 ⋅ F ⇒ F ⋅ C −1 ⋅ F T = F ⋅ F −1 ⋅ b −1 ⋅ F ⋅ F T = b −1 ⋅ b

Universidad Castilla- La Mancha Draft Por: Eduardo W. V. Chaves (2012)


Ciudad Real - España
190 PROBLEMAS RESUELTOS DE MECÁNICA DEL MEDIO CONTINUO

Luego la expresión (2.144) puede ser rescrita como:


 ∂Ψ ∂Ψ  ∂Ψ 2 ∂Ψ
F ⋅ Ψ ,C ⋅ F T =  + I C b − b + III C b −1 ⋅ b

 CI ∂ II C  ∂ II C ∂ III C

  ∂Ψ ∂Ψ  ∂Ψ ∂Ψ 
F ⋅ Ψ ,C ⋅ F T =  + I C 1 − b+ III C b −1  ⋅ b
 ∂I C ∂ II C  ∂ II C ∂ III C 
También es válido que:
 ∂Ψ ∂Ψ  ∂Ψ ∂Ψ 
F ⋅ Ψ ,C ⋅ F T =  + I b 1 − B+ III b b −1  ⋅ b
 ∂I b ∂ II b  ∂ II b ∂ III b 

F ⋅ Ψ ,C ⋅ F T = Ψ ,b ⋅ b c.q.d.

Verificando la expresión (2.143) podemos concluir que la relación Ψ ,b ⋅ b = b ⋅ Ψ ,b es válida,


indicando que los tensores Ψ ,b y b son coaxiales.

Ejemplo 2.46
Demostrar que el tensor de deformación de Green-Lagrange ( E ) y el tensor derecho de
deformación de Cauchy-Green ( C ) son tensores coaxiales.
Solución:
Dos tensores son coaxiales cuando presentan las mismas direcciones principales. También
se puede demostrar que son coaxiales cuando se cumpla la relación:
C ⋅ E = E ⋅C
Partiendo de la definición C = 1 + 2 E concluimos que:
C ⋅ E = (1 + 2 E ) ⋅ E = 1 ⋅ E + 2 E ⋅ E = E ⋅ 1 + 2 E ⋅ E = E ⋅ (1 + 2 E ) = E ⋅ C
Con lo cual se demuestra que los tensores E y C son tensores coaxiales.

Ejemplo 2.47
r r
Obtener la relación E& = F T ⋅ D ⋅ F , partiendo de la definición (ds ) 2 − (dS ) 2 = dX ⋅ 2 E ⋅ dX .
Obtener también la relación entre
D
Dt
[
(ds ) 2 y D . ]
Solución:
r r
Tomando la derivada material de la relación (ds ) 2 − (dS ) 2 = dX ⋅ 2 E ⋅ dX :
D
Dt
[
(ds ) 2 − (dS ) 2 ] =
D
Dt
[ ]
(ds ) 2 =
D r
Dt
[ r
dX ⋅ 2 E ⋅ dX ]
D r r r& r r r r r&
= [dx ⋅ dx ] = 2d{ X ⋅ E ⋅ dX + 2dX ⋅ E& ⋅ dX + 2dX ⋅ E ⋅ d{X
Dt =0 =0
r D r r r
= 2 dx ⋅ [dx ] = 2dX ⋅ E& ⋅ dX
Dt
D r
El término [dx ] , puede expresarse de la siguiente forma:
Dt

Universidad de Castilla- La Mancha Draft Por: Eduardo W. V. Chaves (2012)


Ciudad Real - España
2 CINEMÁTICA DEL CONTINUO 191

D D  ∂x k 
 [dx k ] = dX i 
D r
 Dt [dx ] =
D
Dt r
[
F ⋅ dX
r
]  Dt

Dt  ∂X i 
  D  ∂x k  D  ∂x k 
 = F& ⋅ dX Indicial
 →  =  dX i =  dX i
 = l ⋅ F ⋅ dX
r  Dt  ∂X i  DX i  ∂t 
  ∂v k
  = dX i
 ∂X i

Con lo cual podemos decir que:


r r r D r
2dX ⋅ E& ⋅ dX = 2dx ⋅ [dx ]
r Dt r
= 2dx ⋅ l ⋅ F ⋅ dX
r r
= 2 F ⋅ dX ⋅ l ⋅ F ⋅ dX
r r
= 2dX ⋅ F T ⋅ l ⋅ F ⋅ dX

Podemos descomponer de forma aditiva el tensor gradiente espacial de velocidad ( l ) en


una parte simétrica ( D ) y otra antisimétrica ( W ):
r r r r
2dX ⋅ E& ⋅ dX
= 2 dX ⋅ F T ⋅ l ⋅ F ⋅ dX
r r
= 2dX ⋅ F T ⋅ (D + W ) ⋅ F ⋅ dX
r r r r
= 2 dX ⋅ F T ⋅ D ⋅ F ⋅ dX + 2 dX ⋅ F T ⋅ W ⋅ F ⋅ dX
r r
= 2 dX ⋅ F T ⋅ D ⋅ F ⋅ dX
r r r r r r
Observemos que dX ⋅ F T ⋅ W ⋅ F ⋅ dX = dx ⋅ W ⋅ dx = W : (dx ⊗ dx ) = 0 , ya que el tensor W
r r
es antisimétrico y (dx ⊗ dx ) un tensor simétrico. Con lo que concluimos que:
E& = F T ⋅ D ⋅ F

Con lo cual la relación entre


D
Dt
[ ]
(ds ) 2 y D queda:

D
Dt
[ ] r r r r
(ds ) 2 = 2dX ⋅ F T ⋅ D ⋅ F ⋅ dX = 2dx ⋅ D ⋅ dx

Ejemplo 2.48
Obtener la tasa del determinante del Jacobiano ( J& ) en función de la tasa del tensor de
deformación de Green-Lagrange ( E& ) y también en función de la tasa del tensor derecho
de deformación de Cauchy-Green ( C& ).
Solución:
Considerando que J& = J Tr (D ) , donde D es el tensor tasa de deformación, y está
relacionado con E& por D = F −T ⋅ E& ⋅ F −1 , luego:
( ) (
J& = J Tr (D) = J Tr F −T ⋅ E& ⋅ F −1 = J F −T ⋅ E& ⋅ F −1 : 1 )
En notación indicial queda:
J& = J Fki−1 E& kp F pj−1δ ij = J Fki−1 F pi−1 E& kp
= J F −1 ⋅ F −T : E& = J C −1 : E&
J
= C −1 : C&
2

Universidad Castilla- La Mancha Draft Por: Eduardo W. V. Chaves (2012)


Ciudad Real - España
192 PROBLEMAS RESUELTOS DE MECÁNICA DEL MEDIO CONTINUO

Aún podemos expresar J& en función de F& , para ello consideremos la siguiente relación
1
( )
E& kp = F&sk Fsp + Fsk F&sp . Luego J& aún puede se expresado por:
2

(1
J& = J Fki−1 F pi−1 E& kp = J Fki−1 F pi−1 F&sk Fsp + Fsk F&sp
2
)
(
J −1 −1 &
= Fki F pi Fsk Fsp + Fki−1 F pi−1 Fsk F&sp
2
)
J
( ) J
(
= δ si Fki−1 F&sk + δ si F pi−1 F&sp = Fks−1 F&sk + F ps−1 F&sp
2 2
)
−1 &
= JFts Fst
= JF −T : F&
Resumiendo, podemos expresar la tasa del determinante del Jacobiano como:
J −1 &
J& = J Tr (D ) = J C −1 : E& = C :C = JF −T : F&
2
J
= J Tr (C −1 ⋅ E& ) = Tr (C −1 ⋅ C& ) = J Tr ( F& ⋅ F −1 )
2

Ejemplo 2.49
Las componentes del campo de desplazamiento de un medio continuo son:
u1 = 0,1 X 22

u 2 = 0
u = 0
 3
Se pide:
a) ¿Es una deformación posible en un cuerpo continuamente deformable? Justifique su
respuesta;
b) Determinar el tensor derecho de deformación de Cauchy-Green;
r
c) Encontrar los vectores deformados relativos a los vectores materiales b = 0,01eˆ 1 y
r
c = 0,015 eˆ 2 , los cuales pasaban por el punto P (1,1,0) en la configuración de referencia;
r r
d) Determinar los estiramientos de estos vectores, b y c , en el punto P (1,1,0) ;
e) Determinar el cambio sufrido por el ángulo comprendido entre los dos vectores.
Solución:
a) Para que un movimiento sea posible, el determinante del Jacobiano tiene que ser
positivo. El gradiente de deformación material viene dado por:
1 0 0  0 0,2 X 2 0 1 0,2 X 2 0
∂u i 
Fij = δ ij + = 0 1 0  + 0 0 0 = 0 1 0
∂X j 
0 0 1  0 0 0 0 0 1 

Calculando el determinante: Fij = J = 1 > 0 . Luego es un movimiento posible.

b) El tensor derecho de deformación de Cauchy-Green viene definido por C = F T ⋅ F ,


luego las componentes viene dadas por:

Universidad de Castilla- La Mancha Draft Por: Eduardo W. V. Chaves (2012)


Ciudad Real - España
2 CINEMÁTICA DEL CONTINUO 193

 1 0 0 1 0,2 X 2 0  1 0,2 X 2 0
C ij = 0,2 X 2 1 0 0 1 0 = 0,2 X 2 0,2 X 2 + 1 0
2 2

 0 0 1 0 0 1  0 0 1


r
c) El vector b = 0,01eˆ 1 en el punto P(1,1,0) se deforma según al criterio:
r r
b′ = F P
⋅b
 b1′  1 0,2 × 1 0 0,01 0,01
b ′  = 0 1 0  0  =  0 
 2 
b ′3  0 0 1   0   0 
r
y el vector c = 0,015 eˆ 2 en la configuración actual queda:
 c 1′  1 0,2 × 1 0   0  0,003
c ′  = 0 1 0  0,015  = 0,015 
 2 
 c ′3  0 0 1   0   0 

d) Para obtener el estiramiento, utilizamos directamente la ecuación:


r
b′ 0,01 2
λ br = r = =1
b 0,01
r
El estiramiento del vector c viene dado por:
r
c′ 0,003 2 + 0,015 2
λ cr = r = = 1,0198 ≈ 1,02
c 0,015

Solución Alternativa: Teniendo en cuenta que λ Mˆ = Mˆ ⋅ C ⋅ Mˆ y evaluando C en el punto


P obtenemos que:

 1 0,2 X 2 0  1 0,2 0
C ij ( X 1 = 1, X 2 = 1, X 3 = 0) = 0,2 X 2 0,22
+ 1 0
X 22 = 0,2 1,04 0
 0 0 1  0 0 1
P

Luego aplicando λ bˆ = bˆ ⋅ C ⋅ bˆ y λ cˆ = cˆ ⋅ C ⋅ cˆ , podemos obtener que:


 1 0, 2 0   1 
= [1 0 0] 0,2 1,04 0  0  = 1

2
λ bˆ ⇒ λ bˆ = 1
 0 0 1  0 

 1 0, 2 0   0 
= [0 1 0] 0,2 1,04 0  1  = 1,04

2
λ cˆ ⇒ λ cˆ = 1,0198
 0 0 1  0 
r r
e) En la configuración actual el ángulo que forman los vectores, b ′ y c ′ , puede obtenerse
según la relación:
r r
b′ ⋅ c ′
cos θ = r r
b′ c ′

Universidad Castilla- La Mancha Draft Por: Eduardo W. V. Chaves (2012)


Ciudad Real - España
194 PROBLEMAS RESUELTOS DE MECÁNICA DEL MEDIO CONTINUO

(0,01eˆ 1 + 0eˆ 2 + 0eˆ 3 ) ⋅ (0,003eˆ 1 + 0,015 eˆ 2 + 0eˆ 3 ) 0,00003


cos θ = = = 0,196116135
0,01 2 0,003 2 + 0,015 2 0,01 0,000234

θ = arccos(0,196116135) ≈ 78,69º
Estos dos vectores, en la configuración de referencia, formaban un ángulo de 90 º , luego el
cambio de ángulo será:
∆θ = 90 º −78,69 º = 11,3º
Solución Alternativa: Dadas dos direcciones en la configuración de referencia representadas
por sus versores M̂ y N̂ , el ángulo formado por estos versores en la configuración actual
(tras la deformada) viene dado por:
Mˆ ⋅ C ⋅ Nˆ Mˆ ⋅ C ⋅ Nˆ
cos θ = =
Mˆ ⋅ C ⋅ Mˆ Nˆ ⋅ C ⋅ Nˆ λ Mˆ λ Nˆ

Haciendo Mˆ = b̂ , Nˆ = ĉ se cumple que:


 1 0, 2 0   0 
b ⋅ C ⋅ cˆ = [1 0 0] 0,2 1,04 0 1  = 0,2
ˆ 
 0 0 1  0 

Luego,
bˆ ⋅ C ⋅ cˆ bˆ ⋅ C ⋅ cˆ 0,2
cos θ = = = = 0,196116135
bˆ ⋅ C ⋅ bˆ cˆ ⋅ C ⋅ cˆ λ bˆ λ cˆ 1 1,04

Ejemplo 2.50
Obtener una expresión de la densidad de masa en función del tercer invariante del tensor
de deformación de Green ρ 0 = ρ 0 ( III C ) .
Solución:
Partiendo de la ecuación:
r r r
ρ 0 ( X ) = ρ ( x, t ) J ( x, t )
y considerando que el tercer invariante viene dado por III C = det (C ) = J 2 , obtenemos que:
J = III C , luego:

ρ0 = ρ III C (2.145)

Ejemplo 2.51
En un cierto instante, el campo de desplazamientos de un medio continuo es:
u1 = (a1 − 1) X 1 ; u 2 = (a 2 − 1) X 2 + a1αX 1 ; u 3 = (a 3 − 1) X 3
donde α es una constante. Determinar a1 , a 2 y a 3 sabiendo que el sólido es
incompresible, que un segmento paralelo al eje X 3 no se alarga y que el área de un
elemento situado en el plano X 1 − X 3 no se ha modificado.
Solución:

Universidad de Castilla- La Mancha Draft Por: Eduardo W. V. Chaves (2012)


Ciudad Real - España
2 CINEMÁTICA DEL CONTINUO 195

r r r
Partiendo de la definición del campo de desplazamientos ( u = x − X ):
u1 = x1 − X 1 = (a1 − 1) X 1 ⇒ x1 = a1 X 1
u 2 = x 2 − X 2 = (a 2 − 1) X 2 + a1αX 1 ⇒ x 2 = a 2 X 2 + a1αX 1
u 3 = x 3 − X 3 = (a 3 − 1) X 3 ⇒ x3 = a3 X 3
Luego las ecuaciones del movimiento son:
 x1 = a1 X 1  x1   a1 0 0  X 1 
     
 x 2 = a 2 X 2 + a1αX 1 ⇒  x 2  = a1α a 2 0   X 2  (deformación homogénea)
x = a X x   0 0 a 3   X 3 
 3 3 3  3 
Pudiendo sacar la información del determinante F = a1 a 2 a 3 > 0 .
Con la condición de incompresibilidad dV = F dV0 ⇒ F ≡ J = 1 , con lo cual obtenemos la
siguiente relación:
a1 a 2 a 3 = 1

Que un segmento paralelo al eje X 3 ( M ˆ = [0 0 1] ) no se alarga conlleva a que el


i
estiramiento según esta dirección es unitario λ Mˆ = 1 :

ˆ ⋅ E ⋅M
λ Mˆ = 1 + 2M ˆ = 1 + 2E = 1 ⇒ E 33 = 0
33

Las componentes del tensor de deformación de Green-Lagrange ( E =


2
(
1 T
F ⋅ F − 1 ): )
 a1 a1α 0   a1 0 0  1 0 0 
1    
E ij =  0 a 2 0   a1α a 2 0  − 0 1 0 
2
 0 0 a 3   0 0 a 3  0 0 1  
 a12 + a12 α 2 − 1 a1 a 2 α 0 
1 
=  a1 a 2 α 2
a2 − 1 0 
2
 0 0 a 32 − 1

Luego:
E 33 = a 32 − 1 = 0 ⇒ a 3 = ±1
Área en el plano X 1 − X 3 no se ha modificado
 x1   a1 0 0  X 1 
    
 x 2  =  a1α a 2 0   X 2 
x   0 0 a 3   X 3 
 3 

con Nˆ i(1) = [1 0 0] y Nˆ i(3) = [0 0 1] obtenemos que:


 a1 0 0  1  a1   a1 0 0  0  0 
        
n i(1) = a1α a 2 0  0 = a1α  ; n i(3) 
= a1α a 2 0  0 =  0 
 0 0 a 3  0  0   0 0 a 3  1 a 3 
Luego el área en la configuración actual:

Universidad Castilla- La Mancha Draft Por: Eduardo W. V. Chaves (2012)


Ciudad Real - España
196 PROBLEMAS RESUELTOS DE MECÁNICA DEL MEDIO CONTINUO

eˆ 1 eˆ 2 eˆ 3
r (1) r (3)
n ∧ n = a1 a1α 0 = a1αeˆ 1 − a1 a 3 eˆ 2 + 0eˆ 3
0 0 a3
r r
y su módulo no se modifica N (1) ∧ N (3) = n (1) ∧ n (3) = 1 :
r r
n (1) ∧ n (3) = 1 = (a1α ) 2 + (−a1 a 3 ) 2 ⇒ a12 a 32 α 2 + a12 a 32 = 1

Hemos obtenido anteriormente que a 32 = 1 , con lo cual obtenemos que:


1 1
a12 a 32 α 2 + a12 a 32 = 1 ⇒ a12 α 2 + a12 = 1 ⇒ a12 = ⇒ a1 = ±
(1 + α )
2
(1 + α 2 )
Con lo cual concluimos que:
1
a1 = ; a 2 = (1 + α 2 ) ; a3 = 1
(1 + α )2

Ejemplo 2.52
El sólido de la Figura 2.10 sufre una deformación uniforme (homogénea).
Se pide:
a) Obtener la expresión general de la descripción material del campo de
r r
desplazamientos U ( X , t ) en función del tensor gradiente material de los
desplazamientos J .
b) Obtener dicha expresión sabiendo que, además, se cumple las siguientes
condiciones de contorno:
r r
u 2 ( X , t ) = u3 ( X , t ) = 0 ∀X 1 , X 2 , X 3
u1 ( X 1 = 0, X 2 , X 3 , t ) = 0
u1 ( X 1 = L, X 2 , X 3 , t ) = δ
c) Justificar los valores posibles (positivos y negativos) que puede tomar δ .
d) Calcular los tensores material y espacial de deformación y el de deformación
infinitesimales;

x3

L δ

x1

x2
Figura 2.10:

Universidad de Castilla- La Mancha Draft Por: Eduardo W. V. Chaves (2012)


Ciudad Real - España
2 CINEMÁTICA DEL CONTINUO 197

Solución:
r
Una deformación uniforme viene caracterizada por F ( X , t ) = F (t ) . Además sabemos que:
r r
F ( X , t ) = 1 + J ( X , t ) Deformació
  n → F (t ) = 1 + J (t )
uniforme

donde J es el gradiente material de los desplazamientos y para una deformación uniforme


no es dependiente de la posición, con lo cual podemos decir que:
r r
∂u( X , t ) r r r r r r r
J (t ) = r ⇒ J (t ) ⋅ dX = du( X , t ) ⇒
∫ ∫ u( X , t ) = J (t ) ⋅ X + c (t )
∂X
r
donde c (t ) es una constante de integración. Luego:
r r r r
u( X , t ) = J (t ) ⋅ X + c (t )
En componentes:
u1   J 11 X 1 + J 12 X 2 + J 13 X 3   c1 
     
u 2  = J 21 X 1 + J 22 X 2 + J 23 X 3  + c 2 
u   J X + J X + J X  c 
 3   31 1 32 2 33 3   3
b) De las condiciones del apartado b) podemos decir que:
r r
condición 1) u 2 ( X , t ) = u 3 ( X , t ) = 0 ∀X 1 , X 2 , X 3 :

 u1   J 11 X 1 + J 12 X 2 + J 13 X 3   c1 
      J 21 = 0; J 22 = 0; J 23 = 0, c 2 = 0
u 2 = 0 = J 21 X 1 + J 22 X 2 + J 23 X 3  + c 2  ⇒ 
u = 0   J X + J X + J X  c  J 31 = 0; J 32 = 0; J 33 = 0, c3 = 0
 3   31 1 32 2 33 3   3
condición 2) u1 ( X 1 = 0, X 2 , X 3 , t ) = 0 :
u1 = 0 J 11 X 1 + J 12 X 2 + J 13 X 3  c1 
     
 u2  =  0  +  0  ⇒ {J 12 = 0; J 13 = 0, c1 = 0
 u   0  0
 3     
condición 3) u1 ( X 1 = L, X 2 , X 3 , t ) = δ
 u1  J 11 L  0
       δ
 u 2  =  0  + 0 ⇒ J 11 =
u = δ   0  0  L
 3     
Con lo cual podemos decir que las componentes del gradiente material de los
desplazamientos son:
δ 
L 0 0
J ij =  0 0 0
 
0 0 0
 
Y además el campo de desplazamientos:
δ 
 X1
r r r r r L
s → u i ( X , t ) =  0 
u( X , t ) = J (t ) ⋅ X + c (t ) componente

 
 0 
 

Universidad Castilla- La Mancha Draft Por: Eduardo W. V. Chaves (2012)


Ciudad Real - España
198 PROBLEMAS RESUELTOS DE MECÁNICA DEL MEDIO CONTINUO

c) Para que el movimiento sea posible y tenga significado físico hay que cumplir que
F >0:

 δ 
1 + L 0 0
δ
s → Fij =  0
F (t ) = 1 + J (t ) componente
 1 0 ⇒ F = 1 + > 0 ⇒ δ > − L
  L
 0 0 1
 
d)
Tensor material de deformación (Tensor de deformación de Green-Lagrange):
δ 1 δ2 
 + 2
0 0
L 2 L
E=
1 T
2
(
F ⋅F −1 ) componente
 s → E ij =  0

0 0
 0 0 0
 
 
Tensor espacial de deformación (Tensor de deformación de Almansi):
δ 1 δ2 
 + 2  
1 0 0
e=
1
2
(
1− F ⋅FT ) componente
 s → eij =
L 2 L  
2

0 0 0
 δ
1 +  0 0 0
 L
Tensor de deformación infinitesimal:
δ 
L 0 0
ε ij =  0 0 0
 
0 0 0
 

Ejemplo 2.53
Sobre el tetraedro de la Figura 2.11 se produce una deformación uniforme ( F = ctte ) con
las siguientes consecuencias:
1. Los puntos O , A y B no se mueven;
2. El volumen del sólido pasa a ser " p" veces el volumen inicial;
p
3. La longitud del segmento AC pasa a ser veces la inicial;
2
4. El ángulo AOC pasa a ser de 45º .
Se pide:
a) Justificar por qué no puede utilizar la teoría de deformación infinitesimal;
b) Obtener el tensor gradiente de deformación, los posibles valores de " p" y el campo
de desplazamiento en su forma material y espacial;
c) Dibujar el sólido deformado

Universidad de Castilla- La Mancha Draft Por: Eduardo W. V. Chaves (2012)


Ciudad Real - España
2 CINEMÁTICA DEL CONTINUO 199

x3

O a
B x2
a

x1
Figura 2.11.
Solución:
a) el ángulo AOC pasa de 90º a 45º por lo que, evidentemente, no se trata de una
pequeña deformación, ya que en el caso de pequeñas deformaciones ∆φ << 1 , y en este
π
problema tenemos que ∆φ << ≈ 0,7854
4
b) Estamos en un caso de deformación homogénea. Luego, las ecuaciones del movimiento
viene dadas por:
r r r
x = F (t ) ⋅ X + c (t )

 x1   F11 F12 F13   X 1   c1 


      
 x 2  =  F21 F22 F23   X 2  + c 2 
x  F F32 F33   X 3  c3 
 3   31
El punto O( X 1 = 0, X 2 = 0, X 3 = 0) no se mueve:
0  F11 F12 F13  0  c1   c1  0
          
0 =  F21 F22 F23  0 + c 2  ⇒ c 2  = 0
0  F F32 F33  0 c3  c  0
   31  3  
El punto A( X 1 = a, X 2 = 0, X 3 = 0) no se mueve:
a   F11 F12 F13  a  a   aF11   F11 = 1
         
0  =  F21 F22 F23  0  ⇒ 0  = aF21  ⇒  F21 = 0
0   F F32 F33  0  0  aF  F = 0
   31    31   31
El punto B ( X 1 = 0, X 2 = a, X 3 = 0) no se mueve:
0  1 F12 F13  0  0   aF12   F12 = 0
         
a  = 0 F22 F23  a  ⇒ a  = aF22  ⇒  F22 = 1
 0  0 F F33  0  0  aF  F = 0
   32    32   32
Agrupando las informaciones anteriores:

Universidad Castilla- La Mancha Draft Por: Eduardo W. V. Chaves (2012)


Ciudad Real - España
200 PROBLEMAS RESUELTOS DE MECÁNICA DEL MEDIO CONTINUO

1 0 F13 
Fij = 0 1 F23  ; F = F33
0 0 F33 

El volumen del sólido pasa a ser " p" veces el volumen inicial. La relación del diferencial de volumen
en la configuración de referencia y actual viene dada por:
dV = F dV0 ⇒ ∫ dV = ∫ F dV 0 ⇒ V final = F Vinicial = F33Vinicial

donde hemos tenido en cuenta que la deformación es homogénea. Con lo cual, concluimos
que F33 = p
p
(La longitud del segmento AC pasa a ser veces la inicial). Como estamos con deformación
2
homogénea, una recta en la configuración de referencia seguirá siendo una recta en la
configuración deformada.
El punto C ( X 1 = 0, X 2 = 0, X 3 = a ) se desplaza como:
 x1C  1 0 F13  0   x1C   aF13 
 C     C  
 x 2  = 0 1 F23  0  ⇒  x 2  = aF23 
 x C  0 0 p  a   x C   ap 
 3     3  

La longitud del segmento AC en la configuración de referencia es L AC = a 2 . El vector


que une los puntos A′ ≡ A( x1 = a, x 2 = 0, x3 = 0) , C ′( x1 = aF13 , x 2 = aF23 , x3 = ap) en la
configuración deformada viene dado por:
AC = (aF13 − a )eˆ 1 + (aF23 )eˆ 2 + (ap)eˆ 3
Su módulo:
AC = l AC = (a ( F13 − 1)) 2 + (aF23 ) 2 + (ap) 2 = a ( F13 − 1) 2 + ( F23 ) 2 + ( p) 2

p
Utilizando la información proporcionada por el problema: l AC = L AC . Con lo cual:
2
p
l AC = L AC
2
p
a ( F13 − 1) 2 + ( F23 ) 2 + ( p ) 2 = a 2
2
( F13 − 1) 2 + ( F23 ) 2 + ( p ) 2 = p

Resultando:
( F13 − 1) 2 + ( F23 ) 2 + p 2 = p 2
⇒ ( F13 − 1) 2 + ( F23 ) 2 = 0
 F13 = 1
⇒
 F23 = 0
Resultando:

Universidad de Castilla- La Mancha Draft Por: Eduardo W. V. Chaves (2012)


Ciudad Real - España
2 CINEMÁTICA DEL CONTINUO 201

1 0 1
Fij = 0 1 0 
0 0 p 
El ángulo AOC pasa a ser de 45º .
dx1(1)  1 0 1  1 1
 (1)      
dX i(1) = [1 0 0] ⇒ dxi(1) = Fij dX (j1) ⇒ dx 2  = 0 1 0  0 = 0
dx (1)  0 0 p  0 0
 3      

dx1( 2)  1 0 1  0  1 
 ( 2)      
dX i( 2 ) = [0 0 1] ⇒ dxi( 2 ) = Fij dX (j 2 ) ⇒ dx 2  = 0 1 0  0 =  0 
dx ( 2)  0 0 p  1   p 
 3  
r r
dx (1) ⋅ dx ( 2 ) 2
cos( AOC ′) = cos(45º ) = r (1) r ( 2 ) =
dx dx 2

r r r r
donde dx (1) = 1 , dx ( 2) = 1 + p 2 , dx (1) ⋅ dx ( 2) = 1 . Luego:

1 2
= ⇒ p = ±1
1+ p2 2

Como el determinante del Jacobiano tiene que ser mayor que cero F = p > 0 , eso implica
que p = 1 :
1 0 1
Fij = 0 1 0
0 0 1

Las ecuaciones del movimiento quedan:


 x1  1 0 1  X 1   X 1 + X 3 
      
 x 2  = 0 1 0  X 2  =  X 2 
 x  0 0 1   X   X 
 3   3   3 
El campo de desplazamientos material queda:
u1   X 1 + X 3   X 1   X 3 
r r r r        
u( X , t ) = x − X ⇒ u 2  =  X 2  −  X 2  =  0 
u   X  X   0 
 3  3   3  
El campo de desplazamientos espacial:
u1   x3 
   
u 2  =  0 
u   0 
 3  
c)

Universidad Castilla- La Mancha Draft Por: Eduardo W. V. Chaves (2012)


Ciudad Real - España
202 PROBLEMAS RESUELTOS DE MECÁNICA DEL MEDIO CONTINUO

x3

C
 x1C   aF13  a 
 C    
 x 2  = aF23  = 0  C′
 x C   ap  a 
 3    

a a
O
B = B′ x2
a

A = A′

x1

Ejemplo 2.54
Considérense las siguientes ecuaciones del movimiento:
x1 = X 1
x 2 = X 2 − αX 3
x 3 = X 3 + αX 2
Se pide:
a) El gradiente de deformación, el tensor derecho de deformación de Cauchy-Green, el
tensor izquierdo de deformación de Cauchy-Green, el tensor de deformación de Green-
Lagrange y el tensor de deformación de Almansi. Verificar si se trata de un caso de
deformación homogénea.
b) El tensor derecho de estiramiento, el tensor de rotación de la descomposición polar y la
base principal del tensor izquierdo de deformación de Cauchy-Green de la descomposición
polar.
c) La longitud final de un elemento de longitud inicial 2 que se encuentra en la dirección
X 3 , y la distorsión angular de un ángulo que inicialmente es de 30º y está en el plano
X1 − X 2 .
d) Obtener el tensor de deformación considerando el caso de pequeñas deformaciones.

Solución:
a) Gradiente de deformación ( F )
 ∂x1 ∂x1 ∂x1 
 
 ∂X 1 ∂X 2 ∂X 3  1 0 0 
∂xi  ∂x 2 ∂x 2 ∂x 2  
Fij = = = 0 1 − α 
∂X j  ∂X 1 ∂X 2 ∂X 3  
   
 ∂x3 ∂x3 ∂x3  0 α 1 
 ∂X 1 ∂X 2 ∂X 3 

Universidad de Castilla- La Mancha Draft Por: Eduardo W. V. Chaves (2012)


Ciudad Real - España
2 CINEMÁTICA DEL CONTINUO 203

r r
En general tenemos que dx = F ⋅ dX , y si estamos en un caso de deformación homogénea
r r r
(caso particular del movimiento) se cumple que x = F ⋅ X + c , hecho que se puede
r r
comprobar a través de las ecuaciones del movimiento en forma matricial donde c = 0 :
 x1  1 0 0   X1 
 x  = 0 1 − α   X 
 2    2
 x3  0 α 1   X 3 

Tensor derecho de deformación de Cauchy-Green ( C = F T ⋅ F ):


1 0 0  1 0 0  1 0 0 
    
C ij = Fki Fkj = 0 1 α  0 1 − α  = 0 1 + α 2
0 
0 − α 1  0 α 1  0 0 1 + α 2 

Tensor izquierdo de deformación de Cauchy-Green ( b = F ⋅ F T )


1 0 0  1 0 0  1 0 0 
    
bij = Fik F jk = 0 1 − α  0 1 α  = 0 1 + α 2
0 
0 α 1  0 − α 1  0 0 1 + α 2 

1
El tensor de deformación de Green-Lagrange ( E = (C − 1) ) y el tensor de deformación
2
1
de Almansi ( e = (1 − b −1 ) ) vienen definidos a través de sus componentes por:
2
 1 0 0   1 0 0  0 0 0 
1 1     1 
E ij = (C ij − δ ij ) =  0 1 + α 2
0  −  0 1 0   = 0 α 2 0 
2 2  2
 0
 0 1 + α 2   0 0 1   0 0 α 2 

    
  0

1 0 0 1 0 0  0 0 
  
1 1    1  1
 = 0 α 2
−1
eij = (δ ij − bij ) =  0 1 0  − 0 0 0 
2 2   1+ α2  2  1 + α 2 

 0 0 1   1   α 2 
 0 0 
2  0 0 
  1 + α   1+ α2 
Se puede verificar el resultado a través de la relación E = F T ⋅ e ⋅ F :
 
 0  1 0
1 0 0  0 0
 0  0 0 0
1 α 2
1
E ij = 0 1 α  0 0   
0 1 − α  = 0 α 2 0 
2  1+ α2  2
0 − α 1  
α 2  0 α 1  0 0 α 2 
0 0 
 1+ α2 
b) Según el formato de las componentes cartesianas de C , podemos verificar que ele
espacio original ya es el espacio principal de C , es decir, las direcciones principales son
Nˆ i(1) = [1 0 0] , Nˆ i(1) = [0 1 0] , Nˆ i(1) = [0 0 1] . Por definición, el tensor derecho de
estiramiento viene dado por U = C , y sus componentes:

Universidad Castilla- La Mancha Draft Por: Eduardo W. V. Chaves (2012)


Ciudad Real - España
204 PROBLEMAS RESUELTOS DE MECÁNICA DEL MEDIO CONTINUO

 
 
 1 0 0  1 0 0 
   1 
U ij =  0 1+ α 2
→ U ij−1
0  inversa = 0 0 
0 0 1+ α2   1+ α2 
   1 
0 0
2 
 1+ α 

A través de la descomposición polar por la derecha ( F = R ⋅ U ⇒ R = F ⋅ U −1 ), luego:


 
 
1 0 0  1 0 0   1+ α2 0 0 
 1  1  
R ij = Fik U −kj1 = 0 1 − α  0 0 =  0 1 − α
0 α 1   1+ α2  1+ α2  0 α 1 
 1   
0 0
2 
 1+ α 
Observemos que según el formato de las componentes cartesianas de b , tenemos como
direcciones principales [1 0 0] , [0 1 0] , [0 0 1] , pero esta no es la base principal del
tensor b de la descomposición polar. Como hay dos autovalores iguales correspondientes a
las direcciones [0 1 0] , [0 0 1] , cualquier dirección en el plano x 2 − x 3 será una
dirección principal.

X 2 , x2

Cualquier dirección en el plano


x 2 − x 3 es una dirección
principal de b n̂ (1) -dirección principal única,
asociada al autovalor b1 = 1 .

X 1 , x1

X 3 , x3

Figura 2.12: Espacio principal de b .


Recordar que la descomposición polar es única, es decir, sólo habrá una única base
principal de b para la descomposición polar. A través de la relación nˆ ( a ) = R ⋅ Nˆ ( a )
podemos encontrar la base de b de la descomposición polar:
 1+ α2 0 0  0  0
nˆ (i 2 ) (
= R ⋅N
ˆ (2) )
i =
1 
 0 1

− α  1 =
1 1
 
1+ α2  0 α 1  0 1+ α2 α 
 
 1 + α2 0 0  0   0 
nˆ i(3) (
= R ⋅N
ˆ ( 3)
)
i =
1 
 0 1

− α  0 =
1 − α 
 
1+ α2  0 α 1  1  1+ α2  1 
 

Universidad de Castilla- La Mancha Draft Por: Eduardo W. V. Chaves (2012)


Ciudad Real - España
2 CINEMÁTICA DEL CONTINUO 205

3
Verifiquemos además que se cumple la relación R = ∑ nˆ ( a ) ⊗ N
ˆ (a ) :
a =1

R ij = nˆ i(1)Nˆ (j1) + nˆ i( 2) Nˆ (j2 ) + nˆ (i 3) Nˆ (j3)


1  0  0 
1 1
= 0[1 0 0] +  1 [0
  1 0] +  − α [0 0 1]
 
0 1+ α2
α  1+ α2  1 
1 0 0 0 0 0 0 0 0 
  1 0 1 1
= 0 0 0 + 0 + 0 0 − α 
2  
0 0 0 1 + α 0 α 0 1+ α2 0 0 1 

 1+ α2 0 0 
1  
=  0 1 − α
1 + α2  0 α 1 
 
r
dx ds
c) A través de la relación de estiramiento según una dirección M̂ , λ Mˆ = r = , y
dX dS
teniendo en cuenta que el estiramiento no depende de la integral de línea (deformación
homogénea), se cumple que:

∫ ∫ ∫
L final = ds = λ Mˆ dS = λ Mˆ dS = λ Mˆ Linicial

El estiramiento según la dirección X 3 viene definido por:

λ X = C 33 = 1 + 2 E 33 = 1 + α 2
3

Luego:
2


L final = λ Mˆ dX 2 = 1 + α 2 ( Linicial ) = 2 1 + α 2
0

Como se trata de un caso de deformación homogénea, una recta en la configuración de


referencia sigue sendo una recta en la configuración actual, ver Figura 2.13.

X 3 , x3

 x1A  1 0 0   X 1A 
A′ A  A    A
 x 2  = 0 1 − α   X 2 
 x3A  0 α 1   X 3A 
Linicial = 2     
L final
1 0 0  0  0 
= 0 1 − α  0 =  − 2α 
− 2α O X 2 , x2 0 α 1   2  2 

X 1 , x1
Figura 2.13.

Universidad Castilla- La Mancha Draft Por: Eduardo W. V. Chaves (2012)


Ciudad Real - España
206 PROBLEMAS RESUELTOS DE MECÁNICA DEL MEDIO CONTINUO

Según la Figura 2.13, podemos comprobar que:


2
Linicial = 2 2 + (−2α ) 2 = 4(1 + α 2 )
⇒ Linicial = 2 1 + α 2
Para obtener el ángulo en la configuración actual que forman dos versores, podemos
utilizar la expresión:
cos Θ + 2 Mˆ ⋅ E ⋅ Nˆ
cos θ = (2.146)
λ Mˆ λ Nˆ

donde Θ es el ángulo que forman los versores M̂ y N̂ en la configuración de referencia, y


θ es el ángulo que estos versores forman en la configuración actual.
Teniendo en cuenta que el tensor de deformación de Green-Lagrange es independiente de
r
X , adoptaremos dos versores en el plano X 1 − X 2 que forman un ángulo de Θ = 30º :
Nˆ i = [1 0 0] , Mˆ i = [cos 30º sin 30º 0] . Con estos datos tenemos que:

0 0 0  cos 30º 
1
ˆ ⋅ E ⋅ Nˆ = [1 0 0]0 α 2
M  0   sin 30º  = 0
2
0 0 α 2   0 
Los estiramientos:
1 0 0  1 
λ2Mˆ ˆ 
= M ⋅ C ⋅ M = [1 0 0] 0 1 + α
ˆ 2
0  0 = 1 ⇒ λ Mˆ = 1
0 0 1 + α 2  0

1 0 0  cos 30º 
λ2Nˆ ˆ ˆ 
= N ⋅ C ⋅ N = [cos 30º sin 30º 0] 0 1 + α 2
0   sin 30º 
0 0 1 + α 2   0 
= cos 2 30º + (1 + α 2 ) sin 2 30º
= 1 + α 2 sin 2 30º

Luego, λ Nˆ = 1 + α 2 sin 2 30º . Resultando que:

cos Θ + 2 Mˆ ⋅ E ⋅ Nˆ
cos θ =
λ Mˆ λ Nˆ
cos 30º
=
1 + α 2 sin 2 30º
Como se trata de un caso de deformación homogénea, podemos adoptar dos rectas en la
configuración de referencia y obtener estas dos rectas en la configuración actual y obtener
el ángulo que forman. Por ejemplo, adoptando las rectas OB = [cos 30º 0 0] , y
OC = [cos 30º sin 30º 0] . Según las ecuaciones del movimiento, el punto O no se mueve.
A continuación obtenemos las nuevas posiciones de los puntos B y C , ver Figura 2.14:

Universidad de Castilla- La Mancha Draft Por: Eduardo W. V. Chaves (2012)


Ciudad Real - España
2 CINEMÁTICA DEL CONTINUO 207

 x1B  1 0 0   X 1B  1 0 0  cos 30º  cos 30º 


 B    B     
 x 2  = 0 1 − α   X 2  = 0 1 − α   0  =  0 
 x B  0 α 1   X B  0 α 1   0   0 
 3   3      

 x1C  1 0 0   X 1C  1 0 0  cos 30º   cos 30º 


 C    C     
 x 2  = 0 1 − α   X 2  = 0 1 − α   sin 30º  =  sin 30º 
 x C  0 α 1   X C  0 α 1   0  α sin 30º 
 3   3      

X 3 , x3

A′ α sin 30º

C′

θ O sin 30º
X 2 , x2
cos 30º 30º
B = B′
C

X 1 , x1

Figura 2.14.

A continuación obtenemos el ángulo que forman los nuevos vectores O ′B ′ y O ′C ′ :


O ′B ′ ⋅ O ′C ′ = O ′B ′ O ′C ′ cos θ

cos 2 30º = cos 2 30º cos 2 30º + sin 2 30º +α 2 sin 2 30º cos θ
cos 30º
⇒ cos θ =
1 + α 2 sin 2 30º
d)
0 0 0 
ε ij = 0 0 0
0 0 0

Ejemplo 2.55
Un movimiento de cuerpo rígido está caracterizado por presentar la siguiente ecuación de
movimiento:
r r r
x = c(t ) + Q(t ) ⋅ X (2.147)

Universidad Castilla- La Mancha Draft Por: Eduardo W. V. Chaves (2012)


Ciudad Real - España
208 PROBLEMAS RESUELTOS DE MECÁNICA DEL MEDIO CONTINUO

r
Encontrar la velocidad y aceleración en función de ω que es el vector asociado con el
& ⋅ QT .
tensor antisimétrico Ω = Q
Solución:
r D r r& r& & r
v= x = x = c + Q⋅ X
Dt
& ⋅ QT ⇒ Q
Considerando que Ω = Q & = Ω ⋅ Q , la relación anterior puede aún ser escrita
como:
r r r
v = c& + Ω ⋅ Q ⋅ X
r r r r
v = c& + Ω ⋅ ( x − c )
Utilizando la propiedad que para un tensor antisimétrico Ω el producto escalar de éste con
r r r r r
un vector a podrá ser representado por Ω ⋅ a = ω ∧ a , donde ω es el vector asociado al
tensor antisimétrico Ω . Luego:
r r r r
v = c& + Ω ⋅ ( x − c )
r r r r (2.148)
= c& + ω ∧ ( x − c )

La aceleración vendrá dada por:


r r r r && r
a = v& = &x& = &c& + Q ⋅X
Considerando que Q & ⋅Q +Ω ⋅Q
&& = Ω & la expresión anterior podrá ser representada por:
r r r
& ⋅Q +Ω⋅Q
a = &c& + (Ω & )⋅ X
r & r r
= &c& + Ω ⋅ Q ⋅ X + Ω ⋅ Q& ⋅ X
r & r r
= &c& + Ω ⋅ Q ⋅ X + Ω ⋅Ω ⋅ Q ⋅ X
r & r r r r
= &c& + Ω ⋅ ( x − c) + Ω ⋅ Ω ⋅ ( x − c)
r r r
Una vez más utilizando la propiedad Ω ⋅ a = ω ∧ a , podemos decir que:

∧ ( x − c) + ω ∧ [ω ∧ ( x − c)]
r r r& r r r r r r
a = &c& + ω (2.149)
r r r r r
Para un movimiento de sólido rígido con c = 0 , la velocidad viene dada por v = ω ∧ x , y el
tensor tasa de deformación D viene dado por:
1  ∂vi ∂v j  1  ∂ ( ipq ω p x q ) ∂ ( jpq ω p x q )  1 
= 
∂x ∂x 
 =   ipq ω p q +  jpq ω p q 
D ij = + +
2  ∂x j ∂xi  2
  ∂x j ∂xi  2
  ∂x j ∂xi 

=
1
2
(
 ipq ω p δ qj ) (
1
2
) 1
( )
+  jpq ω p δ qi =  ipj ω p +  jpi ω p =  ipj ω p −  ipj ω p = 0 ij
2
Una vez más hemos demostrado que D = 0 para un movimiento de sólido rígido.

Ejemplo 2.56
r
a) Un sólido gira con una velocidad angular constante ω = ω 3 ê 3 . Se pide:
a.1) Obtener las componentes de la velocidad en la descripción espacial y material;
a.2) Obtener la aceleración en la descripción espacial (Euleriana);

Universidad de Castilla- La Mancha Draft Por: Eduardo W. V. Chaves (2012)


Ciudad Real - España
2 CINEMÁTICA DEL CONTINUO 209

a.3) Si ω 3 = 3rad / s encontrar la posición, velocidad y aceleración en el tiempo t = 2,5s de


una partícula que en la configuración de referencia ocupaba la posición (1,1,0) .
b) Teniendo en cuenta el resultado del Ejemplo 1.118 donde hemos obtenido el vector
r GM r
fuerzas másicas b = − r xˆ donde g = b es la aceleración de la gravedad debido al
x
campo gravitacional. Si consideramos la Tierra como una esfera que gira alrededor de su eje
r
con velocidad angular ω = ω 3 ê 3 , obtener la aceleración de la gravedad ( g φ ) a nivel del mar
en función de la latitud φ .
Solución:

X 3 , x3
r r
r r = r eˆ r = reˆ r
ω = ω 3 ê 3
ω3

r ê3
r
êθ
r ê r
x

X 2 , x2

X 1 , x1

Figura 2.15.
r r r r
a.1) A través del ejercicio anterior podemos decir que v ( x , t ) = ω ∧ x , o en notación
indicial:
vi =  ijk ω j x k =  i1k ω
{1 x k +  i 2 k ω
{2 x k +  i 3k ω 3 x k =  i 3k ω 3 x k
=0 =0

=  i 31ω3 x1 +  i 32 ω 3 x 2 +  i 33 ω3 x3 =  i 31ω 3 x1 +  i 32 ω 3 x 2
{
=0

Luego:
v1 = 132 ω3 x 2 = −ω3 x 2

v 2 =  231ω3 x1 = ω3 x1 (2.150)
v = 0
 3
r
Solamente por verificación, recordar que el vector vorticidad ( ω ) es igual al rotacional del
r r r r
campo de velocidad, ∇ xr ∧ v = rot v = ω y es igual a 2 veces el vector velocidad angular
r r r
(vector axil) ω = 2ω = 2 w :

Universidad Castilla- La Mancha Draft Por: Eduardo W. V. Chaves (2012)


Ciudad Real - España
210 PROBLEMAS RESUELTOS DE MECÁNICA DEL MEDIO CONTINUO

eˆ 1 eˆ 2 eˆ 3
r r ∂ ∂ ∂  ∂v ∂v   ∂v ∂v   ∂v ∂v 
∇ xr ∧ v = =  3 − 2 eˆ 1 −  3 − 1 eˆ 2 +  2 − 1 eˆ 3
∂x1 ∂x 2 ∂x3  ∂x 2 ∂x3   ∂x1 ∂x3   ∂x1 ∂x 2 
v1 v2 v3
r
= (ω 3 − (−ω 3 ) )eˆ 3 = 2ω 3 eˆ 3 = ω
r 11 r r 1 r r 1
La velocidad angular viene definida por ω = ∇ xr ∧ v = rot v = ω = ( 2ω 3 eˆ 3 ) = ω 3 eˆ 3 .
22 2 2
r r r r r
Observar que el campo v ( x , t ) es estacionario, es decir, v = v ( x ) .
Para un movimiento de sólido rígido las ecuaciones del movimiento vienen gobernadas
por:
r r
x = Q(t ) ⋅ X
donde las componentes del tensor ortogonal viene dada por las componentes de la matriz
r r
de transformación del sistema x ′ al sistema x , luego:
 x1  cos θ(t ) − sin θ(t ) 0  X 1  cos θ(t ) X 1 − sin θ(t ) X 2 
      
 x 2  =  sin θ(t ) cos θ(t ) 0  X 2  = sin θ(t ) X 1 + cos θ(t ) X 2 
x   0 0 1  X 3   X3 
 3  
dθ(t )
Teniendo en cuenta que ω = y al integrar obtenemos que:
dt

∫ dθ(t ) = ∫ ωdt ⇒ θ(t ) = ωt

Pudiendo así reescribir las ecuaciones del movimiento como:


 x1  cos θ(t ) − sin θ(t ) 0  X 1   X 1 cos(ωt ) − X 2 sin(ωt ) 
      
 x 2  =  sin θ(t ) cos θ(t ) 0  X 2  =  X 1 sin(ωt ) + X 2 cos(ωt ) (2.151)
x   0 0 1  X 3   X3 
 3  
Para obtener la expresión de la velocidad en la descripción material (Lagrangiana),
reemplazamos las ecuaciones del movimiento (2.151) en las expresiones (2.150):
r
v1 ( X , t ) = −ω 3 ( X 1 sin(ωt ) + X 2 cos(ωt ))
 r
v 2 ( X , t ) = ω 3 ( X 1 cos(ωt ) − X 2 sin(ωt )) (2.152)
 r
v3 ( X , t ) = 0
a.2) La aceleración Euleriana obtenemos a través de la definición de derivada material de la
r r
velocidad v ( x , t ) :
r r r r r r
r r ∂v ( x , t ) ∂v ( x , t ) ∂x ( X , t ) r r r
a ( x, t ) = + r ⋅ = ∇ xr v ⋅ v ( x , t )
142∂t4 3 ∂x ∂t
r
0

donde las componentes del gradiente espacial de la velocidad vienen dadas por:

Universidad de Castilla- La Mancha Draft Por: Eduardo W. V. Chaves (2012)


Ciudad Real - España
2 CINEMÁTICA DEL CONTINUO 211

 ∂v1 ∂v1 ∂v1 


 
r r  ∂x1 ∂x 2 ∂x 3   0 − ω3 0
 ∂v ( x , t )   ∂v ∂v 2 ∂v 2  
0 (antisimétrico)
r
 r  = (∇ xr v )ij = 2 = ω3 0
 ∂x  ij ∂x ∂x 2 ∂x 3  
 1   0

 v3 ∂v 3 ∂v 3   0 0
 ∂x1 ∂x 2 ∂x 3 
Comprobando que realmente se trata de un movimiento de sólido rígido. Luego, las
componentes de la aceleración Euleriana vienen dadas por:
 0 − ω3 0 − ω3 x2   − ω32 x1 
   
ai ( x , t ) = [∇ x v ⋅ v ( x , t ) ]i = ω3 0  ω3 x1  = − ω32 x2 
r r r r
r 0
 0 0 0  0   0 
r r
Podemos expresar la aceleración a ( x , t ) = −ω 32 x1 eˆ 1 − ω 32 x 2 eˆ 2 en el sistema cilíndrico, ver
Figura 2.15. Recordar del capítulo 1 de los apuntes que se cumplen que:
x1 = r cos θ , x1 = r cos θ , eˆ 1 = eˆ r cos θ − eˆ θ sin θ , eˆ 2 = eˆ r sin θ + eˆ θ cos θ . Luego, la
aceleración en el sistema cilíndrico queda:
r
a = − ω 32 x1eˆ 1 − ω 32 x 2 eˆ 2
= − ω 32 ( r cos θ)(eˆ r cos θ − eˆ θ sin θ) − ω 32 ( r sin θ)(eˆ r sin θ + eˆ θ cos θ)
= − ω 32 r (cos 2 θ + sin 2 θ)eˆ r
= − ω 32 reˆ r
r v
= − ω 32 r = a ctpe
v
Esta última conocida como la aceleración centrípeta ( actpe ).
a.3) La partícula que en la configuración de referencia ocupaba la posición (1,1,0) describe
una trayectoria circular de radio r = 2 en el plano x1 − x 2 , ver Figura 2.16.

Partícula P en t = 2,5s X 2 , x2
r r
v ( X , t = 0)
Partícula P
r r
v P ( x , t = 2,5) 1
r
r X
x

1 X 1 , x1

Trayectoria de la partícula P

Figura 2.16.
r r
En la configuración de referencia ( t = 0 ) se cumple que X = x . Para la partícula P
tenemos que:

Universidad Castilla- La Mancha Draft Por: Eduardo W. V. Chaves (2012)


Ciudad Real - España
212 PROBLEMAS RESUELTOS DE MECÁNICA DEL MEDIO CONTINUO

r
v1P ( x , t = 0) = −ω3 x 2 = −ω 3 X 2 = −(3)(1) = −3
 P r
v 2 ( x , t = 0) = ω 3 x1 = ω 3 X 1 = (3)(1) = 3
 P
v3 = 0

 − ω 32 X 1  − 9 
r    
a iP ( x , t = 0) = − ω 32 X 2  = − 9 
 0  0 
   
Para el tiempo t = 2,5s la posición, velocidad, y aceleración de la partícula P vienen dadas
por:
 x1P   X 1 cos(ωt ) − X 2 sin(ωt )  cos(3 × 2,5) − sin(3 × 2,5)  − 0,59136
 P      
 x 2  =  X 1 sin(ωt ) + X 2 cos(ωt ) = sin(3 × 2,5) + cos(3 × 2,5) =  1,28464 
x P   X3   0   0 
 3      
r
v1P ( x , t = 2,5) = −ω3 x 2 = −(3)(−0,59136) = −3,85391
 P r
v 2 ( x , t = 2,5) = ω 3 x1 = (3)(1,28464) = −1,77409
 P
v3 = 0

 − ω 32 x1   5,322 
r    
a iP ( x , t = 2,5) = − ω 32 x 2  = − 11,562 
 0   0 
   
b) Para una partícula situada en la superficie de la Tierra, debido a la rotación se sentirá
como si estuviera siendo proyecta hacia fuera según dirección de r , ver figura abajo. Hay
que tener en cuenta que la fuerza real es la Centrípeta debido a la aceleración centrípeta.
Por conveniencia, adoptamos una fuerza ficticia, fuerza centrífuga, que sería la aparente
causa de esta proyección hacia afuera. Asociada a esta fuerza tenemos la aceleración
v v
centrífuga ( a ctfu ) que es igual pero de sentido contrario a la aceleración centrípeta ( a ctpe ).

x3 , z
x3 , z
ω3 ω3

r v
x3 r a ctfu
r α
R b
φ
φ
x2 , y

x1 , x

Universidad de Castilla- La Mancha Draft Por: Eduardo W. V. Chaves (2012)


Ciudad Real - España
2 CINEMÁTICA DEL CONTINUO 213

La aceleración de la gravedad para una latitud definida por φ viene dada por:
v r
g φ = a ctfu + b

Recordar que dados dos vectores se cumple que


v r v 2 v r r 2
a ctfu +b = a ctfu + 2 a ctfu b cos α + b , ver Ejemplo 1.02. Para este caso en
r v v r
particular tenemos que, b = g , a ctpe = a ctfu = − ω32 r = ω32 r . Verificar también que
r = R cos φ y que cos α = cos( π − φ ) = − cos φ . Con eso, obtenemos que:

v r v 2 v r r 2
g φ = a ctfu + b = a ctpe − 2 a ctpe b cos φ + b

= (ω 32 r ) 2 − 2(ω 32 r ) g cos φ + g 2

= (ω 32 R cos φ ) 2 − 2(ω 32 R cos φ ) g cos φ + g 2


Resultando
g φ = g 2 − 2 gω 32 R cos 2 φ + ω 34 R 2 cos 2 φ

Observar que en los polos ( φ = 90 º ) se cumple que g φPol = g y en la línea del ecuador se
cumple que g φEcu = g 2 − 2 gω 32 R + ω 34 R 2 = ( g − ω 32 R ) 2 = g − ω 32 R .

Ejemplo 2.57
Considérese una barra sometida a sucesivos desplazamientos como se indica la Figura abajo

B0 B B

L0 L0
L(1) L( 2 ) L(f1) L ≡ L( 2 )

∆L(1)
∆L
∆L( 2)

Demostrar que la deformación Ingenieril (deformación de Cauchy) no es aditiva para


incremento sucesivos de deformación, es decir, ε (1) + ε ( 2) ≠ ε .
Solución:
La deformación de Cauchy fue definida como:
∆L L − L 0
εC = = = λ −1
L0 L0

Luego, la deformación total sufrida por el cuerpo, es decir, de la configuración B0 hasta la


configuración B es

Universidad Castilla- La Mancha Draft Por: Eduardo W. V. Chaves (2012)


Ciudad Real - España
214 PROBLEMAS RESUELTOS DE MECÁNICA DEL MEDIO CONTINUO

L( 2 ) − L0 L( 2 )
εC = = −1
L0 L0

En la configuración B la deformación Ingenieril queda:


L(1) − L0 L(1)
ε C(1) = = −1
L0 L0

En la configuración B teniendo en cuenta solo el incremento de desplazamiento u ( 2) ,


tenemos que:
L( 2 ) − L(1) L( 2 )
ε C( 2 ) = = (1) − 1
L(1) L
Luego
 L(1) L( 2 )   L( 2) 
ε C(1) + ε C( 2 ) =  − 1 + (1) − 1 ≠  − 1 = ε C
 L0 L   L0 
Un requerimiento esencial de toda deformación es que pueda caracterizar los
desplazamientos reales, en el caso la longitud final:
L0 L0
 L(1)  
∫0
ε C(1) dx
0

= 
L0
− 1dx = L(1) − L0 = ∆L(1) 
  (1) ( 2)
L1 L1  ⇒ ∆L + ∆L = ∆L
L (2)
 

0 0

ε C( 2 ) dx =  (1) − 1 dx = L( 2 ) − L(1) = ∆L( 2 ) 
L  
L0 L
0
 L 

0
ε C dx = ∫
0 0
L
− 1 dx = L − L0 = ∆L

Deformación de Green-Lagrange
Observemos que la deformación de Green-Lagrange en la configuración B viene dado
por:

εG =
L2 − L20
= (
1 2
λ −1 )
2 L20 2
Podríamos haber obtenido esta misma expresión utilizando la relación
E=E (1)
+F (1)T
⋅ E ⋅ F , donde para el caso uniaxial tenemos que
( 2) (1)
E → εG , E (1)
→ ε G(1) ,
L(1)
E ( 2 ) → ε G( 2) , F (1) → λ(1) = . Luego:
L0

⋅ E ( 2) ⋅ F (1)
T
E = E (1) + F (1)
ε G = ε (G1) + λ(1) ε G( 2 ) λ(1)

1  L(1)   L(1)  1  L( 2 )   L(1)


2 2
  
=   − 1 +     − 1  
2  L0    L0  2  L(1)    L0 
       
 L( 2) 2 − L2 
=
 0 2
 L − L0
=
(2
)
2 L20 2 L20

Universidad de Castilla- La Mancha Draft Por: Eduardo W. V. Chaves (2012)


Ciudad Real - España
2 CINEMÁTICA DEL CONTINUO 215

2.1.3 Descomposición Polar del Gradiente de Deformación

Ejemplo 2.58
Considérense las componentes cartesianas del gradiente de deformación dadas por:
 5 3 3
Fij =  2 6 3
 2 2 4
Obtener los tensores derecho ( U ) e izquierdo ( V ) de estiramiento y el tensor de rotación
( R ) de la descomposición polar.
Solución:
Antes de obtener los tensores U , V , R , vamos analizar el tensor F .
Para que tenga sentido físico el determinante de F tiene que ser mayor que cero,
det ( F ) = 60 > 0 . Los autovalores y autovectores F se pueden obtener como:
F11′ = 10 asociado al autovector mˆ i(1) = [0,6396021491; 0,6396021491; 0,4264014327]
F22′ = 3 asociado al autovector mˆ i( 2 ) = [− 0,5570860145; 0,7427813527; − 0,3713906764]
F33′ = 2 asociado al autovector mˆ i(3) = [− 0,4082482905; − 0,4082482905; 0,8164965809]
Se puede verificar fácilmente que la base constituida por estos autovectores no forma una
ortonormal, es decir, mˆ i(1) mˆ i( 2) ≠ 0 , mˆ i(1) mˆ i(3) ≠ 0 , mˆ i( 2) mˆ i(3) ≠ 0 . Verificamos también que si
B es la matriz que contiene los autovectores de F :
 mˆ i(1)   0,6396021491; 0,6396021491; 0,4264014327 
 ( 2)  
B = mˆ i  = − 0,5570860145; 0,7427813527; − 0,3713906764
 mˆ (3)  − 0,4082482905; − 0,4082482905; 0,8164965809 
 i   
podemos hallar que det (B ) = 0,905 ≠ 1 , y que B −1 ≠ B T . Pero se cumple que:
10 0 0 5 2 2 5 2 2 10 0 0
−1  
B  0 3 0 B = 3 6 2 = FijT y  
B 3 6 2B =  0 3 0
−1

 0 0 2 3 3 4 3 3 4  0 0 2


Tensor derecho de deformación de Cauchy-Green, C = F T ⋅ F (tensor definido positivo):
33 31 29
C ij = Fki Fkj =  31 49 35
29 35 34 
Los autovalores y autovectores del tensor C son:
′ = 9,274739
C11  → ˆ (1) = [0,6861511933; − 0,7023576528 0,1894472683]
autovector
N i

′ = 3,770098
C 22 autovector
 → N i( 2 ) = [0,5105143234; 0,2793856273; − 0,8132215099]
ˆ

′ = 102,955163
C 33 autovector
 → Nˆ (3) = [− 0,518239; − 0,65470405; − 0,550264423]
i

Dichos autovectores constituye una base ortonormal luego, se cumple que AC−1 = ACT , y
det (AC ) = −1 , donde:
Nˆ i(1)  0,6861511933 − 0,7023576528 0,1894472683 
 
AC = Nˆ i( 2)  = 0,5105143234 0,2793856273 − 0,8132215099
Nˆ (3)   − 0,518239 − 0,65470405 − 0,550264423 
 i  
cumpliendo que:

Universidad Castilla- La Mancha Draft Por: Eduardo W. V. Chaves (2012)


Ciudad Real - España
216 PROBLEMAS RESUELTOS DE MECÁNICA DEL MEDIO CONTINUO

C11′ 0 0  33 31 29 33 31 29 C11′ 0 0 



T
A  0 ′
C 22      T 
0  AC =  31 49 35 = C ij ; AC  31 49 35 AC =  0 C 22 ′ 0 
C
 0 ′ 
0 C 33 29 35 34  29 35 34   0 ′ 
0 C 33
En el espacio principal de C obtenemos las componentes del tensor derecho de
estiramiento U como:
λ 1 0 0   C11′ 0 0  3,0454455 0 0 
     
′ ′
U = U ij =  0 λ 2 0  =  0 ′
C 22 0 = 0 1,9416741 0 
 0 0 λ 3   0 0 ′  
C 33 0 0 10,1466824
 
y su inversa:
1   1 
0  0 0 0 
 λ1   3,0454455 
1 1
U ′ −1 = U′ij−1 0 =
= 0 0 0 
λ2    1,9416741 
   1 
1 
0
0  0 0 
λ 3  
 10,1466824 
Pudiendo así obtener las componentes del tensor U en el espacio original a través de la ley
de transformación:
 4,66496626 2,25196988 2,48328843
A U ′AC = 2,25196988 6,00314487 2,80907159 = U ij
T
C
 2,48328843 2,80907159 4,46569091
y
 0,31528844 − 0,05134777 − 0,14302659
A U ′ AC =  2,25196988 0,24442627 − 0,12519889 = U ij−1
T
C
−1

− 0,14302659 − 0,12519889 0,38221833 


Luego, el tensor de rotación de la descomposición polar viene dado por la expresión
R = F ⋅ U −1 , que resulta en un tensor ortogonal propio det (R ) = 1 .
 0,9933191 0,10094326 0,05592536
R ij = =  − 0,10658955 0,98826538 0,10940847 
 Fik U kj−1
 − 0,04422505 − 0,11463858 0,9924224 
Tensor izquierdo de deformación de Cauchy-Green, b = F ⋅ F T (tensor definido positivo):
 43 37 28
bij = Fik F jk = 37 49 28
28 28 24
Los autovalores y autovectores del tensor b son:
′ = 9,274739
b11 nˆ (i1) = [0,6212637156 − 0,7465251613 0,238183919]
autovector
 →
′ = 3,770098
b22 autovector
 → nˆ i( 2 ) = [0,4898263742 0,1327190337 − 0,8616587383]
′ = 102,955163
b33 autovector
 → nˆ (i 3) = [− 0,611638389 − 0,6519860747 − 0,448121233]
Observemos que son los mismos autovalores del tensor C , pero con distintos
autovectores. Los autovectores del tensor b también constituye una base ortonormal
luego, se cumple que Ab−1 = AbT , y det (Ab ) = −1 , donde:
 nˆ (i1)   0,6212637156 − 0,7465251613 0,238183919 
 ˆ ( 2)  
Ab = n i  =  0,4898263742 0,1327190337 − 0,8616587383
nˆ (3)   − 0,611638389 − 0,6519860747 − 0,448121233 
 i   

Universidad de Castilla- La Mancha Draft Por: Eduardo W. V. Chaves (2012)


Ciudad Real - España
2 CINEMÁTICA DEL CONTINUO 217

cumpliendo que:

b11 0  43 37 28
0  43 37 28 ′
b11 0 0

A 0T  

b22    T 
0  Ab = 37 49 28 = bij ; Ab 37 49 28 Ab =  0 b22 ′ 0 
b
 0 0 b33′   28 28 24  28 28 24  0 0 b33 ′ 
Ya que los tensores C y b tienen los mismos autovalores se cumple que U′ij = Vij′ , es decir,
que tienen las mismas componentes en sus respectivos espacios principales. Y como
consecuencia U′ij−1 = Vij′ −1 . Luego se cumple que:
Pudiendo así obtener las componentes del tensor U en el espacio original a través de la ley
de transformación:
 5,3720129 2,76007379 2,41222612
A V ′Ab = A U ′Ab = 2,76007379 6,04463857 2,20098553 = Vij
T
b
T
b
 2,41222612 2,20098553 3,6519622 
y
 0,28717424 − 0,07950684 − 0,14176921
A V ′ Ab = A U ′ Ab = − 0,07950684 0,23396031 − 0,08848799 = Vij−1
T
b
−1 T
b
−1

 − 0,14176921 − 0,08848799 0,42079849 


El tensor de rotación de la descomposición polar ya obtenido anteriormente tiene ser el
mismo si utilizamos la expresión R = V −1 ⋅ F .
También podríamos haber obtenido los tensores U , V , R a través de su representación
espectral. Es decir, si conocemos los estiramientos principales λ i y los autovectores de C ,
ˆ (i ) , y los autovectores de b , nˆ (i ) , es de fácil demostración que:
N
3
U= ∑λ
a =1
a
ˆ (a ) ⊗ N
N ˆ (a) = λ N
1
ˆ (1) ⊗ N
ˆ (1) + λ N
2
ˆ (2) ⊗ N
ˆ (2) + λ N
3
ˆ ( 3) ⊗ N
ˆ ( 3)

3
V= ∑λ
a =1
a nˆ ( a ) ⊗ nˆ ( a ) = λ 1 nˆ (1) ⊗ nˆ (1) + λ 2 nˆ ( 2 ) ⊗ nˆ ( 2 ) + λ 3 nˆ (3) ⊗ nˆ (3)
3
R= ∑ nˆ
a =1
(a) ˆ ( a ) = nˆ (1) ⊗ N
⊗N ˆ (1) + nˆ ( 2) ⊗ N
ˆ ( 2 ) + nˆ (3) ⊗ N
ˆ ( 3)

3
F= ∑λ
a =1
a
ˆ ( a ) = λ nˆ (1) ⊗ N
nˆ ( a ) ⊗ N 1
ˆ (1) + λ nˆ ( 2 ) ⊗ N
2
ˆ ( 2 ) + λ nˆ (3) ⊗ N
3
ˆ ( 3)

3 3
= ∑
a =1
λa R ⋅N
ˆ (a) ⊗ N
ˆ (a) =
∑λ
a =1
a nˆ ( a ) ⊗ nˆ ( a ) ⋅ R

 3
ˆ ( a )  =  λ nˆ ( a ) ⊗ nˆ ( a )  ⋅ R
3


= R ⋅  λ a N
 a =1
ˆ (a ) ⊗ N
 
  a =1
a ∑ 

= R ⋅U = V ⋅R
Como podemos verificar la representación espectral de los tensores R y F no viene
presentada en el sentido estricto de la representación espectral, es decir, autovalor y
autovector del tensor.

Ejemplo 2.59
El tensor gradiente de deformación en un punto del cuerpo viene dado a través de la
siguiente combinación lineal de diádicas:
F = 0,2eˆ 1 ⊗ eˆ 1 − 0,1eˆ 1 ⊗ eˆ 2 + 0,3eˆ 2 ⊗ eˆ 1 + 0,4eˆ 2 ⊗ eˆ 2 + 0,1eˆ 3 ⊗ eˆ 3

Universidad Castilla- La Mancha Draft Por: Eduardo W. V. Chaves (2012)


Ciudad Real - España
218 PROBLEMAS RESUELTOS DE MECÁNICA DEL MEDIO CONTINUO

donde eˆ i (i = 1,2,3) representa la base cartesiana. Se pide:


a) Determinar los tensores de deformación b y C ;
b) Determinar los autovalores y autovectores de b y C ;
c) Escribir F en su representación espectral en función de los autovalores de C ( C a )
3
y verificar si se cumple que F = ∑ λ a nˆ ( a ) ⊗ N
ˆ ( a ) , siendo λ los estiramientos
a
a =1

principales, n̂ los autovectores de b , y N̂ los autovectores de C ;


d) Obtener la representación espectral y las componentes: del tensor de rotación ( R )
de la descomposición polar; y de los tensores de estiramientos U y V ;
Solución
Las componentes del gradiente de deformación en forma de matriz vienen dadas por:
F = Fij eˆ i ⊗ eˆ j = 0,2eˆ 1 ⊗ eˆ 1 − 0,1eˆ 1 ⊗ eˆ 2 + 0,3eˆ 2 ⊗ eˆ 1 + 0,4eˆ 2 ⊗ eˆ 2 + 0,1eˆ 3 ⊗ eˆ 3

0,2 − 0,1 0 
Fij =  0,3 0,4 0 
 0 0 0,1

a) Las componentes del tensor izquierdo de deformación de Cauchy-Green, b = F ⋅ F T ,


vienen dadas por:
T
0,2 − 0,1 0  0,2 − 0,1 0  0,05 0,02 0 
bij = Fik F jk =  0,3 0,4 0   0,3 0,4 0  = 0,02 0,25 0 
    (2.153)
 0 0 0,1  0 0 0,1  0 0 0,01

Las componentes del tensor derecho de deformación de Cauchy-Green, C = F T ⋅ F ,


vienen dadas por:
T
0,2 − 0,1 0 
0,2 − 0,1 0  0,13 0,1 0 
C ij = Fki Fkj =  0,3 0,4 0 
 0,3 0,4 0  =  0,1 0,17
   0  (2.154)
 0 0 0,1
 0 0 0,1  0 0 0,01
b) Determinar los autovalores y autovectores de b y C ;
C ⋅N
ˆ =C N
(a)
ˆ (a) ⇒ C − C1 = 0

donde el índice (a ) no indica suma. Observemos que ya conocemos un autovalor de C ,


C (3) = 0,01 , ver componentes de C (2.154). Luego, el determinante característico queda:

0,13 − C 0,1
=0
0,1 0,17 − C
⇒ (0,13 − C )(0,17 − C ) − 0,01 = 0
La solución de la ecuación cuadrática es:
C (1) = 0,25198 ; C ( 2) = 0,04802

Luego:

Universidad de Castilla- La Mancha Draft Por: Eduardo W. V. Chaves (2012)


Ciudad Real - España
2 CINEMÁTICA DEL CONTINUO 219

0,633399 − 0,77334
Cc (1) = 0,25198 ⇒ N i =  0,77334 
ˆ (1)
; C ( 2 ) = 0,04802 ⇒ N i =  0,63399 
ˆ (2)

 0   0 
0 
C (3) = 0,01 ⇒ Nˆ (i 3) = 0
1

Análogamente para obtener los autovalores y autovectores del tensor b :


b ⋅ nˆ = b( a ) nˆ ( a )

donde el índice (a ) no indica suma.. Luego


0,098538  − 0,995133
b(1) = 0,25198 ⇒ =  0,995133
nˆ i(1) ; b( 2 ) = 0,04802 ⇒ nˆ i( 2 ) =  0,098538 
 0   0 
0 
b(3) = 0,01 ⇒ nˆ i(3) = 0
1

Observemos que era de esperar que los tensores C y b presentan los mismos autovalores,
pero distintos autovectores.
0,252 0 0  0,252 0 0 

C ij′ =  0 0,048 0  ; 
bij′ =  0 0,048 0 
 0 0 0,01  0 0 0,01
Además la representación espectral de los tensores C y b viene dadas respectivamente
por:
3 3
C= ∑
a =1
ˆ (a ) ⊗ N
λ2a N ˆ (a) ; b= ∑ λ nˆ
a =1
2
a
(a)
⊗ nˆ ( a )

donde λ a son los estiramientos principales. Considerando que λ2a = C a son los autovalores
de C y de b , los estiramientos principales son:
λ (1) = 0,25198 ≈ 0,501976 ; λ ( 2 ) = 0,04802 ≈ 0,219134 ; λ (3) = 0,01 = 0,1
3
c) Para verificar si se cumple F = ∑ λ a nˆ ( a ) ⊗ N
ˆ ( a ) . Calculemos las componentes de
a =1

 3
ˆ ( a )  , con los resultados obtenidos anteriormente. Resultando:

 λ a nˆ ( a ) ⊗ N

 a =1

 ij

Universidad Castilla- La Mancha Draft Por: Eduardo W. V. Chaves (2012)


Ciudad Real - España
220 PROBLEMAS RESUELTOS DE MECÁNICA DEL MEDIO CONTINUO

 3 ˆ ( a )  = λ nˆ (1) ⊗ N

 λ a nˆ ( a ) ⊗ N

 a =1

 ij
1 i
ˆ (1) + λ nˆ ( 2 ) ⊗ N
j 2 i
ˆ ( 2 ) + λ nˆ (3) ⊗ Nˆ (3)
j 3 i j

 0,76958 − 0,6309 0 0,06247 0,0762 0


= 0,50197 − 0,0762 0,06247 0 + 0,219134  0,6309 0,76958 0 +
 
 0 0 0  0 0 0
0 0 0
+ 0,10 0 0
0 0 1
0,2 − 0,1 0 
=  0,3 0,4 0 
 0 0 0,1
Luego, resulta ser cierto.
d)

3
0,832 − 0,554 0
R= ∑ nˆ (a) ˆ (a )
⊗N componentes (R )ij = 0,554 0,832 0
a =1
 0 0 1
Que puede ser verificado con:
 0,76958 − 0,6309 0 0,06247 0,0762 0 0 0 0  0,832 − 0,5547 0
R ij =  − 0,0762 0,06247 0 +  0,6309 0,76958 0 + 0 0 0 = 0,5547 0,832 0
 0 0 0  0 0 0 0 0 1  0 0 1

3
0,333 0,139 0 
U= ∑ ˆ (a) ⊗ N
λ aN ˆ (a) componentes (U)ij ≈ 0,139 0,388 0 
a =1
 0 0 0,1

3
0,222 0,028 0 
V= ∑ λ nˆ a
(a)
⊗ nˆ (a)
componentes (V )ij ≈ 0,028 0,5 0 
a =1
 0 0 0,1

Ejemplo 2.60
Para un movimiento dado (deformación de corte):
 x1 = X 1 + kX 2 k − constante

 x2 = X 2
x = X
 3 3

Encontrar los tensores F (Gradiente de deformación), C (Tensor derecho de deformación


de Cauchy-Green), b (Tensor izquierdo de deformación de Cauchy-Green), E (Tensor de
deformación de Green-Lagrange), U (Tensor derecho de estiramiento), V (Tensor
izquierdo de estiramiento) y R (Tensor de rotación de la descomposición polar).
Solución:

Universidad de Castilla- La Mancha Draft Por: Eduardo W. V. Chaves (2012)


Ciudad Real - España
2 CINEMÁTICA DEL CONTINUO 221

Tensor gradiente de deformación:


 ∂x1 ∂x1 ∂x1 
 
 ∂X 1 ∂X 2 ∂X 3  1 k 0
∂xi  ∂x 2 ∂x 2 ∂x 2  
Fij = = = 0 1 0
∂X j  ∂X 1 ∂X 2 ∂X 3  
   
 ∂x3 ∂x3 ∂x3  0 0 1 
 ∂X 1 ∂X 2 ∂X 3 

Tensor derecho de deformación de Cauchy-Green ( C = F T ⋅ F ), cuyas componentes son:


 1 0 0 1 k 0   1 k 0
    
C ij = Fki Fkj = k 1 0 0 1 0 = k 1 + k 2 0
 0 0 1 0 0 1  0 0 1

Tensor izquierdo de deformación de Cauchy-Green ( b = F ⋅ F T ), cuyas componentes son:


1 k 0  1 0 0 1 + k 0
2
k
 
bij = Fik F jk = 0 1 0  k 1 0 =  k 1 0
0 0 1  0 0 1  0 0 1

1
Tensor material de deformación de Green-Lagrange, E = (C − 1) , cuyas componentes
2
son:
 1 k 0 1 0 0  0 k 0
1      1
E ij =   k 1 + k 2 0  − 0 1 0   =  k k 2 0
2 2
  0 0 1 0 0 1   0 0 0

Verifiquemos que sólo hay deformación en el plano x1 − x 2 .


Teniendo en cuenta la descomposición polar F = R ⋅ U = V ⋅ R , y que:
C = (V ⋅ R)T ⋅ (V ⋅ R)
= RT ⋅ VT ⋅ V ⋅R
= RT ⋅ V ⋅ V ⋅R
= RT ⋅ V 2 ⋅R
= RT ⋅ b ⋅R
Por simplicidad vamos trabajar sólo en el plano x1 − x 2 , con lo cual representaremos las
componentes del tensor de rotación como:
cos θ − sin θ c − s 
R ij =  =  (i, j = 1,2)
 sin θ cos θ   s c 
donde, se cumple que cos 2 θ + sin 2 θ = c 2 + s 2 = 1 . La relación C = R T ⋅ b ⋅ R queda:
1 k   c s  1 + k 2 k  c − s 
k 1 + k 2  =  − s c    
    k 1 s c 
(c 2 + c 2 k 2 + 2 sck + s 2 ) (− sck 2 − s 2 k + c 2 k ) 
= 2 2 2 
 (− sck − s k + c k ) (c 2 + s 2 k 2 − 2 sck + s 2 )

Universidad Castilla- La Mancha Draft Por: Eduardo W. V. Chaves (2012)


Ciudad Real - España
222 PROBLEMAS RESUELTOS DE MECÁNICA DEL MEDIO CONTINUO

−k
De la relación (c 2 + c 2 k 2 + 2 sck + s 2 ) = 1 ⇒ (c 2 k 2 + 2 sck + 1) = 1 obtenemos que s =
c.
2
−k
Ahora partiendo de la relación (− sck 2 − s 2 k + c 2 k ) = k y considerando que s = c
2
obtenemos que:
−k
1 2 2 −k
c= = ; s= =
k2 k2 + 4 k2 k2 + 4
+1 +1
4 4
Luego:
 2 k 
 2 0
2
 k +4 k +4 
−k 2
R ij =  0
 2 
 k +4 k2 + 4 
 0 0 1
 

De la descomposición polar F = R ⋅ U = V ⋅ R , podemos obtener que U = R T ⋅ F , y que


V = F ⋅ R T , cuyas componentes son:

 2 −k   2 k 
0  2 0
 2 2
 1 k 0  k + 4 k +4
2
 k +4 k +4 
k 2  k 2+ k2 
U ij = R ki Fkj =  0 0 1 0 =  0
2
 k +4 k2 + 4 2
 0 0 1  k + 4 k2 + 4 
 0 0 1   0 0 1
   
 

  2+k 
2
 2 −k k
 2 0  0
2 2 2
1 k 0  k + 4 k +4   k +4 k +4 
k 2  k 2 
Vij = Fik R jk = 0 1 0  0 =  0
2
0 0 1  k + 4 k2 + 4 2
  k +4 k2 + 4 
 0 0 1  0 0 1
   

Ejemplo 2.61
Un paralelepípedo deformable de dimensiones 2 × 2 × 1 se encuentra en su configuración
de referencia en la posición que indica la Figura 2.17. Este cuerpo se somete a una
deformación:
r r
x ( X , t ) = −exp X 2t eˆ 1 + tX 12 eˆ 2 + X 3 eˆ 3 (2.155)
siendo ( X 1 , X 2 , X 3 ) las coordenadas materiales y t el tiempo. Para este cuerpo se pide:
r
a) Obtener las componentes del gradiente de deformación F , en todo punto X e
instante t .

Universidad de Castilla- La Mancha Draft Por: Eduardo W. V. Chaves (2012)


Ciudad Real - España
2 CINEMÁTICA DEL CONTINUO 223

b) Lo mismo para el tensor derecho de deformación de Cauchy-Green C . ¿Cuáles


son los estiramientos principales?
c) Obtener también las componentes correspondientes al tensor derecho de
estiramiento U y al tensor de rotación R . Comprobar que este último es un tensor
ortogonal propio.
d) ¿Cuál es el volumen del paralelepípedo deformado en el instante t = 1s ?
X2

X3 X1
1 2
Figura 2.17.
Solución:
a) Según (2.155), las componentes del vector posición son x1 = −exp X 2t , x 2 = tX 12 ,
x3 = X 3 , luego las componentes del gradiente de deformación F vienen dadas por:

 0 − t exp X 2t 0
∂xi  
Fij = = 2tX 1 0 0
∂X j 
 0 0 1

b) El tensor derecho de deformación de Cauchy-Green C viene definido por C = F T ⋅ F ,


con componentes C ij = Fki Fkj :

 0 2tX 1 0  0 − t exp X 2t 0  4t 2 X 12 0 0
    
C ij = − t exp X 2t 0 0  2tX 1 0 0 =  0 2
t exp 2 X 2t
0
 0 0 1  0 0 1  0 0 1

Observemos que este espacio es el espacio de las direcciones principales de C . Si λ i son


los estiramientos principales se cumple la siguiente relación:
3 3
C = U2 = ∑
a =1
ˆ (a ) ⊗ N
λ2a N ˆ (a) ⇒ U= ∑λ
a =1
a
ˆ (a) ⊗ N
N ˆ (a)

Como estamos en el espacio principales, podemos obtener los estiramientos principales:


λ 1 = + 4t 2 X 12 ; λ 2 = + t 2 exp 2 X 2t ; λ3 = + 1
3
Ya que el tensor U = ∑ λ a N
ˆ (a ) ⊗ N
ˆ ( a ) es por definición un tensor definido positivo,
a =1
implicando que sus valores principales son positivos, luego:
λ 1 = 2tX 1 ; λ 2 = t exp X 2t ; λ3 = 1
c)

Universidad Castilla- La Mancha Draft Por: Eduardo W. V. Chaves (2012)


Ciudad Real - España
224 PROBLEMAS RESUELTOS DE MECÁNICA DEL MEDIO CONTINUO

 1 
 2tX 0 0
 2tX 1 0 0  1 
1
U ij =  0 t exp X 2t
0 ⇒ U ij−1 
= 0 0
 t exp X 2t 
 0 1  0 1
0
0
 
 

Según la descomposición polar F = R ⋅ U ⇒ R = F ⋅ U −1 , con eso podemos obtener las


componentes del tensor ortogonal propio R :
 1 
 0 0
 0 − t exp X 2t
0  2tX 1  0 − 1 0
  1
R ij =  2tX 1 0 0  0 0 = 1 0 0
 t exp X 2t 
 0 1  0 0 1
1 
 0
0 0
 
 

Verificamos que debe cumplir la ortogonalidad R ⋅ R −1 = R ⋅ R T = 1 :


0 − 1 0  0 1 0 1 0 0
R ik R jk = 1 0 0  − 1 0 0 = 0 1 0
0 0 1  0 0 1 0 0 1

y propio ya que det (R ) = 1 .


d) Para calcular el volumen final utilizaremos la relación dV = JdV0 , donde J = F es el
determinante del Jacobiano y viene dado por:
0 − t exp X 2t 0
J = 2tX 1 0 0 = 2t 2 X 1exp X 2t
0 0 1

Para el tiempo t = 1s tenemos que J = 2 X 1exp X 2 . Luego el volumen en el tiempo t = 1s


vendrá dado por:

∫ dV = ∫ JdV V0
0

2 2 1

∫ ∫ ∫ (2 X exp )dX
X2
= 1 3 dX 2 dX 1
X 1 = 0 X 2 = 0 X 3 =0

= 4(exp − 1) ≈ 25,556
2

Obs.: No se puede utilizar la expresión V = JV0 porque no se trata de un caso de


deformación homogénea.

Ejemplo 2.62
Un cuerpo continuo experimenta la deformación:
x1 = X 1 ; x 2 = X 2 + kX 3 ; x3 = X 3 + kX 2
donde k es una constante.

Universidad de Castilla- La Mancha Draft Por: Eduardo W. V. Chaves (2012)


Ciudad Real - España
2 CINEMÁTICA DEL CONTINUO 225

a) Determinar el gradiente de deformación ( F ); el tensor derecho de deformación de


Cauchy-Green ( C ); el tensor de deformación de Green-Lagrange ( E ).
b) Calcular el campo de desplazamiento, la longitud al cuadrado (dx) 2 de los lados
OA y OB , y de la diagonal OC , después de la deformación en el pequeño
rectángulo indicado en la figura abajo.

X3

dX 2
B C

dX 3

O A X2
X1

c) Considérese ahora un cuadrado como la figura abajo

x3 C′

A′
A C

θ 23 B′

O B
x2

c.1) Determinar los estiramientos según las direcciones OC y BA ; c.2) el ángulo θ 23 en la


configuración actual en función de k .
c.3) Aplicar la descomposición polar del tensor F , es decir, determinar U y R
Solución:
a) C = F T ⋅ F . Las componentes del gradiente de deformación son:
1 0 0 
∂xi 
Fij = = 0 1 k 
∂X j
0 k 1 

1 0 0  1 0 0  1 0 0 
    
C ij = Fki Fkj = 0 1 k  0 1 k  = 0 1 + k 2
2k 
0 k 1  0 k 1  0 2k 1 + k 2 

1
Tensor material de deformación de Green-Lagrange, E = (C − 1) , cuyas componentes
2
son :

Universidad Castilla- La Mancha Draft Por: Eduardo W. V. Chaves (2012)


Ciudad Real - España
226 PROBLEMAS RESUELTOS DE MECÁNICA DEL MEDIO CONTINUO

 1 0 0  1 0 0  0 0 0
1      1
E ij =  0 1 + k 2
2 k  − 0 1 0   = 0 k 2
2k 
2 2
 0 2k 1 + k 2  0 0 1  0 2k k 2 
r r r
b.1) Campo de desplazamientos, u = x − X , cuyas componentes son:
u1 = x1 − X 1 = 0 ; u 2 = x 2 − X 2 = kX 3 ; u 3 = x 3 − X 3 = kX 2
r
b.2) Cálculo de (dx ) 2 = dx 2
(dxr )2 = dxr ⋅ dxr
r r
= F ⋅ dX ⋅ F ⋅ dX
r r
= dX ⋅ F T ⋅ F ⋅ dX
r r
= dX ⋅ C ⋅ dX
Explícitamente:
1 0 0   dX 1 
(dx ) 2
= [dX 1 dX 2 
dX 3 ] 0 1 + k 2
2k   dX 2 
0 2k 1 + k 2   dX 3 
= (dX 1 ) 2 + (dX 2 ) 2 (1 + k 2 ) + (dX 3 ) 2 (1 + k 2 ) + 4k (dX 2 )(dX 3 )

Luego, para la diagonal OC tenemos que [0 dX 2 dX 3 ] , resultando que:


(dx )2 = (dX 2 ) 2 (1 + k 2 ) + (dX 3 ) 2 (1 + k 2 ) + 4k (dX 2 )(dX 3 )
Para el lado OA tenemos que [0 dX 2 0] , resultando:
(dx )2 = (dX 2 ) 2 (1 + k 2 )
Para el lado OB tenemos que [0 0 dX 3 ] , resultando que:
(dx )2 = (dX 3 ) 2 (1 + k 2 )
c) El estiramiento según una dirección N̂ (configuración de referencia) viene dado por el la
expresión (λ Nˆ )2 = N
ˆ ⋅ C ⋅N
ˆ.

 1 1 
c.1) Estiramiento según dirección OC : Nˆ i = 0  , con lo cual:
 2 2

 
 0 
1 0 0  
(λ ) 2 
= 0
1 1 
 0 1+ k
2
2k  
1 
= (1 + k ) 2
2  2
OC
 2
0 1 + k 2  
2k 1 
 
 2 

 1 −1
Estiramiento según dirección BA : Nˆ i = 0  , con lo cual:
 2 2

Universidad de Castilla- La Mancha Draft Por: Eduardo W. V. Chaves (2012)


Ciudad Real - España
2 CINEMÁTICA DEL CONTINUO 227

 
 0 
1 0 0  
 −1 
(λ )
BA
2
= 0
1
 0 1 + k
2
2k  

1 

= (1 − k ) 2
 2 2
0 2k
2
1 + k 2   − 1 
 
 2 
c.2) Variación del ángulo. Podemos utilizar directamente la expresión:
ˆ ⋅C ⋅N
M ˆ ˆ ⋅ C ⋅N
M ˆ
cos θ = =
ˆ ⋅ C ⋅M
M ˆ N ˆ ⋅ C ⋅N
ˆ λ Mˆ λ Nˆ

ˆ = [0 0 1] , y según dirección OA es
donde el versor según dirección OB es M i

Nˆ i = [0 1 0] . Con eso obtenemos que:

1 0 0  0 
(λ )
OB
2 
= [0 0 1] 0 1 + k 2
2k  0 = 1 + k 2
0 2k 1 + k 2  1

1 0 0  0 
(λ )
OA
2 
= [0 1 0] 0 1 + k 2
2k  1 = 1 + k 2
0 2k 1 + k 2  0

1 0 0  0
ˆ ˆ 
Mi C ij N j = [0 0 1] 0 1 + k 2
2k  1 = 2k
0 2k 1 + k 2  0
Resultando que:
ˆ ⋅ C ⋅N
M ˆ 2k
cos θ 23 = =
λ Mˆ λ Nˆ 1+ k2

c.3) Descomposición Polar F = R ⋅ U = V ⋅ R , donde:


3 3
C = U2 = ∑
a =1
ˆ (a ) ⊗ N
λ aN ˆ (a ) ⇒ U= C = ∑
a =1
ˆ (a) ⊗ N
λaN ˆ (a)

Cálculo de los valores principales de C . Verifiquemos que por el formato de las


componentes del tensor C , sólo hay deformación según el plano x 2 − x 3 . Además ya
conocemos un autovalor λ1 = 1 asociado a la dirección Ni(1) = [1 0 0] . Simplificando así el
determinando característico como:
(1 + k 2 ) − λ 2k
=0
2k (1 + k 2 ) − λ
( ) (
⇒ λ 2 − 2 1 + k 2 λ + 1 − 2k 2 + k 4 = 0 )
⇒ λ2 − 2(1 + k )λ + (1 − k )
2 2 2
=0

Las raíces son: λ 2 = 1 + k 2 + 2k = (1 + k ) 2 ; λ 3 = 1 + k 2 − 2k = (1 − k ) 2


Luego, en el espacio principal de C , tenemos que:

Universidad Castilla- La Mancha Draft Por: Eduardo W. V. Chaves (2012)


Ciudad Real - España
228 PROBLEMAS RESUELTOS DE MECÁNICA DEL MEDIO CONTINUO

1 0 0 

C ij′ = 0 (1 + k ) 2 0 

0 0 (1 − k ) 2 

 1 1   −1 1 
Las direcciones principales son λ 2 ⇒ Ni( 2) = 0 ( 3)
 , λ 3 ⇒ N i = 0 .
 2 2  2 2
Luego, la matriz de transformación entre el espacio original y el espacio principal queda:
 
 
1 0 0 
1 1 
a ij = A = 0
 2 2
 −1 1 
0 
 2 2 
Es decir, se debe cumplir que:
C′ = A C AT
T
   
   
1 0 0  1 0 0  1 0 0  1 0 0 
0 (1 + k ) 2 1 1  1 1 
 0  = 0  0 1+ k2 2k  0
 2 2   2 2
0 (1 − k ) 2    1 + k 2  
0 −1 1  0 2k −1 1 
0  0 
 2 2   2 2 
Luego, en el espacio principal de C , tenemos que:
1 0 0  + 1 0 0 
   
C ij′ = 0 (1 + k ) 2 0  ⇒ U ij =  0 + (1 + k ) 2 0 
0 0 (1 − k ) 2   0 0 2 
+ (1 − k ) 

1 0 0 

⇒ U ij = 0 (1 + k ) 0 
0 0 (1 − k )
La inversa en el espacio principal:
 
 
1 0 0 
1
U′ij−1 = 0 0 
 (1 + k ) 
 1 
0 0 
 (1 − k ) 

Las componentes del tensor U en el espacio original vienen dadas por:

Universidad de Castilla- La Mancha Draft Por: Eduardo W. V. Chaves (2012)


Ciudad Real - España
2 CINEMÁTICA DEL CONTINUO 229

U ′ −1 = A T U −1 A
T
      
      
1 0 0  1 0 0  1 0 0  1 0 0 
1 1  1 1 1   1 −k 
U ij = 0 0 0  0 = 0
 2 2  (1 + k )  2 2   (1 − k 2 ) (1 − k 2 ) 

 −1 1   1  −1 1   −k 1 
0  0 0  0  0
 2 2   (1 − k )   2 2   (1 − k 2 ) (1 − k 2 ) 

De la descomposición polar obtenemos que F = R ⋅ U ⇒ R = F ⋅ U −1


 
 
1 0 0  1 0 0  1 0 0 
 1 −k  
R ij = 0 1 k  0 = 0 1 0
(1 − k 2 ) (1 − k 2 )  
0 k 1     
0 −k 1  0 0 1 
 (1 − k 2 ) (1 − k 2 ) 

Ejemplo 2.63
Dada la siguiente ley de movimiento:
 x1 = λ 1 X 1

 x 2 = −λ 3 X 3
x = λ X
 3 2 2

Si pide:
a) Encontrar el volumen deformado para un cubo unitario;
b) Encontrar el área deformada de un cuadrado unitario en el plano X 1 − X 2 , y dibujar el
área deformada;
c) Aplicar la Descomposición Polar y obtener los tensores U , V y R
Solución:
a)
 x1  λ 1 0 0  X 1  λ 1 0 0 
    
 x2  =  0 0 − λ 3   X 2  ⇒ Fij =  0 0 − λ 3  (deformación Homogénea)
x   0 λ2 0   X 3   0 λ2 0 
 3 
El determinante de F viene dado por F ≡ J = λ 1λ 2 λ 3 , y el volumen deformado:

dV = F dV0 integrando
 → V final = F Vinicial = λ 1λ 2 λ 3

b) Aplicando la relación de Nanson y teniendo en cuenta que estamos en el caso particular


de deformación homogénea:
r r r r
da = JF −T ⋅ dA integrando
 → a final = JF −T ⋅ Ainicial

donde

Universidad Castilla- La Mancha Draft Por: Eduardo W. V. Chaves (2012)


Ciudad Real - España
230 PROBLEMAS RESUELTOS DE MECÁNICA DEL MEDIO CONTINUO

1 
 0 0 
eˆ 1 eˆ 2 eˆ 3 λ 2 λ 3 0 0   λ1 
r 1  0 1 
Ainicial = 1 0 0 = eˆ 3 ; Fij−1 = 0 λ 1 λ 3  =  0 0
λ 1λ 2 λ 3   λ2 
0 1 0  0 − λ 1λ 2 0   
0 −1
0 
 λ3 
Con lo cual el vector área deformada queda:
1 
 0 0 
 a1   λ1  0  0 
   − 1    
a 2  = λ 1 λ 2 λ 3  0 0 0 = − λ 1 λ 2 
a  λ 3   
 3   1  0 
0 1
0 
 λ2 
Su módulo queda:
r
a final = (−λ 1 λ 2 ) 2 = λ 1 λ 2

X 3 , x3
B ′(0,0, λ 2 )
r
a final = λ 1 λ 2

C ′(λ1 ,0, λ 2 )

O (0,0,0) B (0,1,0) X 2 , x2

r
Ainicial = 1
C (1,1,0)
A(1,0,0)
A′(λ 1 ,0,0)

X 1 , x1

donde los puntos A(1,0,0) , B(0,1,0) y C (1,1,0) se desplazan según la ley del movimiento:
 x1A  λ 1 0 0  1 λ 1   x1B  λ 1 0 0  0  0 
 A       B     
 x2  =  0 0 − λ 3  0 =  0  ;  x2  =  0 0 − λ 3  1 =  0 
x A   0 λ2 0  0  0  x B   0 λ2 0  0 λ 2 
 3   3 
 x1C  λ 1 0 0  1  λ 1 
 C     
 x2  =  0 0 − λ 3  1 =  0 
xC   0 λ2 0  0 λ 2 
 3 

Universidad de Castilla- La Mancha Draft Por: Eduardo W. V. Chaves (2012)


Ciudad Real - España
2 CINEMÁTICA DEL CONTINUO 231

c) Según la definición de la descomposición polar F = R ⋅U = V ⋅R donde


U= C = F T
⋅F y V = b = F ⋅F T

λ 1 0 0  λ 1 0 0  λ21 0 0 λ 1 0 0
 
C ij =  0 0 λ 2   0  2
0 − λ3  =  0 λ 2 0  ⇒ U ij =  0 λ2 0 
 0 − λ3 0   0 λ2 0   0 0 λ23   0 0 λ 3 

λ 1 0 0  λ 1 0 0  λ21 0 0 λ 1 0 0
 
bij =  0 0 − λ 3   0 0  2
λ2  =  0 λ3 0  ⇒ Vij =  0 λ3 0 
 0 λ2 0   0 − λ 3 0   0 0 λ22   0 0 λ 2 

Verifiquemos que el espacio original coincide con el espacio principal de C . Verifiquemos


también que C y b tienen los mismos autovalores pero direcciones principales distintas.
Para obtener el tensor de rotación de la descomposición polar R = F ⋅ U −1 = V −1 ⋅ F , con lo
cual:
1 
 0 0
λ 1 0 0   λ1  1 0 0 
1
R ij =  0 0 − λ 3   0 0  = 0 0 − 1
 λ2 
 0 λ2 0     
0 1  0 1 0 
0
 λ 3 
1 
 0 0 
 λ1  λ 1 0 0  1 0 0 
1
R ij =  0 0   0 0 − λ 3  = 0 0 − 1
 λ3 
  λ2 0  0 1 0 
0 1 0
0
 λ 2 

Ejemplo 2.64
Determinar para la deformación homogénea:
 x1 = 3 X 1

 x2 = 2 X 2

 x3 = 3 X 3 − X 2
el elipsoide de deformación material que resulta de la deformación de una esfera material
X 12 + X 22 + X 32 = 1 (ver Figura 2.18). Probar que este elipsoide en el espacio principal del
tensor izquierdo de estiramiento V tiene la forma:
x1′ 2 x 2′ 2 x3′ 2
+ + =1
λ21 λ22 λ23
donde λ 1 , λ 2 , λ 3 son los estiramientos principales.

Universidad Castilla- La Mancha Draft Por: Eduardo W. V. Chaves (2012)


Ciudad Real - España
232 PROBLEMAS RESUELTOS DE MECÁNICA DEL MEDIO CONTINUO

X 2 , x2
X 3 , x3

Superficie material
(Siempre constituida por las
mismas partículas)

X 1 , x1
Figura 2.18: Esfera material.

Solución:
La ley del movimiento y de su inversa vienen dadas por:
 3 
 0 0 
 x1   3 0 0  X 1   X1   3   x1 
       1  
 x2  =  0 2 → X 2  = 
0   X 2  inversa 0 0 x2 
x   0 −1 3   X 3   X   2  
 3    3 3 3   x3 
 0 
 6 3 
La ly del movimiento en la descripción espacial viene dada por:
 3
X1 = x1
 3
 x2
X 2 =
 2
 3 3
X 3 = x2 + x3
 6 3
Reemplazando en la ecuación de la esfera:
X 12 + X 22 + X 32 = 1
2 2 2
 3   x2   3 3 
 x  +  + x2 + x3 = 1
 3 1  2   
   6 3 
Tras la simplificación de la expresión anterior obtenemos que:
x12 + x 22 + x32 + x 2 x3 = 3
Que es la ecuación de un elipsoide. Tenemos ahora que representar la ecuación de este
elipsoide en el espacio principal del tensor derecho de estiramiento V . Recordemos que el
tensor V y el tensor b son coaxiales (tienen las mismas direcciones principales), y además
se cumple que:
V = b = F ⋅FT

Universidad de Castilla- La Mancha Draft Por: Eduardo W. V. Chaves (2012)


Ciudad Real - España
2 CINEMÁTICA DEL CONTINUO 233

A continuación obtenemos las componentes del tensor b , y sus autovalores y autovectores.


T
 3 0 0  3 0 0 3 0 0 
   
bij =  0 2 0  0 2 0  = 0 5 − 3 
0 −1  
3 0 −1 3 0 − 3 3 
  
Verificamos que ya conocemos un autovalor y autovector b1 = 3 , nˆ (i1) = [1 0 0] . Luego,
las otras direcciones principales estarán el plano x 2 − x 3 . Obteniendo los demás
autovalores y autovectores
 2 − 2
 → nˆ (i 2 ) = 0
b2 = 6 autovector 
 2 2 

 − 2 − 2
 → nˆ (i 3) = 0
b3 = 2 autovector 
 2 2 

Resultando así que:


 
1 0 0 
3 0 0  − 2 2 
bij′ = 0 6 0 Matriz
 de   → a ij = 0
Trasnforamción
 2 2 
0 0 2  2 2
0 
 2 2 
λ 1 = 3 0 0
 
Vij′ =  0 λ2 = 6 0 
 0 0 λ3 = 2
 
Luego, aplicando la ley de transformación del sistema x1 , x 2 , x3 al sistema x1′ , x 2′ , x3′ ,
obtenemos que:

 
T 
  x1 = x1′
 x1  1 0 0   x1′  
   − 2 2     − 2 2
 x2  = 0  x 2′  ⇒ x2 = x 2′ + x3′
x   2 2   x′   2 2
 3  2 2  3 
0  2 2
 2 2   x3 = x 2′ + x3′
 2 2
Con lo cual, la ecuación del elipsoide en el espacio principal de V viene representada por:
x12 + x 22 + x32 + x 2 x3 = 3
2 2
 2 2   2 2   2 2  2 2 
(x1′ ) 2
+ −
 2
x 2′ + x3′ + 
  2
x 2′ + x3′ +  −
  2
x 2′ + x3′
 2
x 2′ + x 3′ = 3

 2   2   2  2 
Simplificando la expresión anterior obtenemos que:
x1′ 2 x 2′ 2 x3′ 2 x′ 2 x′2 x′ 2 x′2 x′2 x′2
+ + = 1 2 + 2 2 + 3 2 = 12 + 22 + 32 = 1
3 6 2 ( 3) ( 6) ( 2) λ1 λ2 λ3

Universidad Castilla- La Mancha Draft Por: Eduardo W. V. Chaves (2012)


Ciudad Real - España
234 PROBLEMAS RESUELTOS DE MECÁNICA DEL MEDIO CONTINUO

X 3 , x3
X 2 , x2
x3′

λ3 = 2

λ2 = 6
λ1 = 3 x 2′

x1′

X 1 , x1

Figura 2.19: Elipsoide material (configuración deformada).

x2

x1
x3
R V

x 2′
X2 x2

F x1 x1′
X1 x3
X3

x3′

Figura 2.20: Descomposición polar por la izquierda.

Universidad de Castilla- La Mancha Draft Por: Eduardo W. V. Chaves (2012)


Ciudad Real - España
2 CINEMÁTICA DEL CONTINUO 235

2.1.4 Deformación Infinitesimal

Ejemplo 2.65
Dadas las ecuaciones del movimiento
 x1 = X 1 + 4 X 1 X 2 t
 2
 x2 = X 2 + X 2 t (2.156)
 2
 x3 = X 3 + X 3 t
Se pide:
a) Encontrar el campo de velocidad;
b) Encontrar el campo de deformación infinitesimal;
c) Para el tiempo t = 1 s , obtener el tensor de deformación infinitesimal.
Solución:
a) Velocidad:

r V1 = 4 X 1 X 2
r r dx 
V ( X , t) = ⇒ V 2 = X 22 (2.157)
dt  2
V3 = X 3
b) Aceleración:
r  A1 = 0
r r dV 
A( X , t ) = ⇒  A2 = 0 (2.158)
dt A = 0
 3
c) Campo de desplazamientos:
u1 = x1 − X 1 = X 1 + 4 X 1 X 2 − X 1 = 4 X 1 X 2
 2 2
u 2 = x2 − X 2 = X 2 + X 2 − X 2 = X 2 (2.159)
 2 2
u3 = x3 − X 3 = X 3 + X 3 − X 3 = X 3
Luego, las componentes del tensor de deformación infinitesimal vienen dadas por:

1  ∂u i ∂u j 

ε ij = + (2.160)
2  ∂x j ∂xi 

 ∂u1 ∂u1 ∂u1 
 
 ∂X 1 ∂X 2 ∂X 3  4 X
2 4X1 0 
∂u i  ∂u 2 ∂u 2 ∂u 2  
= = 0 2X 2 0  (2.161)
∂x j  ∂X 1 ∂X 2 ∂X 3  
   2 X 3 
 ∂u 3 ∂u 3 ∂u 3   0 0
 ∂X 1 ∂X 2 ∂X 3 

luego:

Universidad Castilla- La Mancha Draft Por: Eduardo W. V. Chaves (2012)


Ciudad Real - España
236 PROBLEMAS RESUELTOS DE MECÁNICA DEL MEDIO CONTINUO

4 X 2 2 X1 0 
ε ij =  2 X 1 2X 2 0  (2.162)
 0 0 2 X 3 

Ejemplo 2.66
Consideren el siguiente tensor de deformación infinitesimal:
 
0 0 0 
 2 
X X X
ε ij = 0 µ 2 2 3 − µ 23  (2.163)
 l l 
 X2 X2X3 
0 − µ 3 −µ 
 l2 l 2 
y el tensor de rotación infinitesimal:
 
0 0 0 
 
ωij = 0

0
µ
( X 22 − X 32 

) (2.164)
2l 2
 µ 2 2
0 − 2 X 2 − X 3 ( ) 0


 2l 
Hallar las componentes del campo de desplazamientos.
Solución:
El gradiente de los desplazamientos viene relacionado con el tensor de deformación
infinitesimal y el tensor spin como:
u i , j = ε ij + ω ij
(2.165)
ε ij =
1
2
(
ui, j + ui, j ) ; ω ij =
1
2
(
ui, j − ui, j )
luego:
0 0 0 
µ
ui, j = 2 0 2X 2 X 3 X 22 2
− 3X 3  (2.166)
2l
0 − (X 22 + X 32 ) − 2 X 2 X 3 

∂u1
=0
→ u1 = 0 (2.167)
∂x1
∂u 2 µ
= 2 (2 X 2 X 3 )
∂x 2 2l
(2.168)

⇒ ∂u 2 =
µ
∫ 2l (2 X
2 2X3 )∂x 2 ⇒ u 2 = µ
2l 2
[X 22 X 3 + C1 ( X 3 ) ]

Universidad de Castilla- La Mancha Draft Por: Eduardo W. V. Chaves (2012)


Ciudad Real - España
2 CINEMÁTICA DEL CONTINUO 237

∂u 3 µ
= − 2 (2 X 2 X 3 )
∂x3 2l
(2.169)

⇒ ∂u 3 = − ∫ 2l 2
µ
(2 X 2 X 3 )∂x3 ⇒ u 3 = − µ
2l 2
[
X 32 X 2 + C2 ( X 2 ) ]
Para determinar la constante C1 ( X 3 ) del resultado (2.168) derivamos con respecto a X 3 :

∂u 2 µ
= 2
∂X 3 2l
 2 ∂C1 ( X 3 ) 
X 2 +
∂X
µ 2 2
 = 2 X 2 − 3X 3 ⇒
∂X
[
∂C1 ( X 3 )
= −3 X 32 ]
 3  2 l 3 (2.170)
⇒ C1 ( X 3 ) = − X 33

Análogamente hacemos para determinar la constante C2 ( X 2 ) :

∂u 3
∂X 2
µ
=− 2
2l
 2 ∂C 2 ( X 2 ) 
X 3 +
∂X
µ 2 2
 = − 2 X2 + X3 ⇒
2 ∂ X
[
∂C 2 ( X 2 )
= X 22 ]
 2  l 2
(2.171)
X3
⇒ C2 ( X 2 ) = 2
3
Luego, el campo de desplazamientos viene dado por:

u1 = 0 ; u 2 =
µ
2l 2
[X 2
2 X3 − X 33 ] ; u3 = −
µ 

2l 2 
X 2
3 X 2 +
X 23 
3 
 (2.172)

Ejemplo 2.67
Demostrar que, para el caso de pequeñas deformaciones, la tasa del tensor de deformación
infinitesimal ( ε& ) es igual al tensor tasa de deformación ( D ).
Solución:
Consideremos la relación entre la tasa del tensor material de deformación de Green-
Lagrange ( E& ) y el tensor tasa de deformación ( D ):
E& = F T ⋅ D ⋅ F (2.173)
Para el caso de pequeñas deformaciones se cumple que F ≈ 1 , y además se cumple
también que E& ≈ e& ≈ ε& luego:
E& = ε& = D (2.174)

Ejemplo 2.68
Consideremos un cuerpo material bajo el régimen de pequeñas deformaciones, el cual está
sometido al siguiente campo de desplazamientos:
u1 = (−2 x1 + 7 x 2 ) × 10 −3 ; u 2 = (−10 x 2 − x1 ) × 10 −3 ; u 3 = x3 × 10 −3
a) Encontrar el tensor de deformaciones infinitesimales, y el tensor spin infinitesimal;
b) Encontrar los invariantes principales del tensor de deformación infinitesimal, y las
deformaciones principales;
c) Dibujar el círculo de Mohr en deformaciones, y obtener la deformación tangencial
máxima;

Universidad Castilla- La Mancha Draft Por: Eduardo W. V. Chaves (2012)


Ciudad Real - España
238 PROBLEMAS RESUELTOS DE MECÁNICA DEL MEDIO CONTINUO

d) Encontrar la deformación volumétrica lineal y el tensor de deformación infinitesimal


desviador.
Solución
a) El gradiente de los desplazamientos:
 ∂u1 ∂u1 ∂u 1 
 
 ∂x1 ∂x 2 ∂x 3  − 2 7 0
∂u i  ∂u 2 ∂u 2 ∂u 2  
(∇ u)ij = = =  − 1 − 10 0  × 10 −3
∂x j  ∂x1 ∂x 2 ∂x 3 
   1 
 ∂u 3 ∂u 3 ∂u 3   0 0
 ∂x1 ∂x 2 ∂x 3 
Tensor spin infinitesimal:
 0 4 0
1  ∂u ∂u j  
ωij = ∇ ( anti
u ij ) =  i −
2  ∂x j ∂x i
 = − 4 0 0 × 10 −3
  
  0 0 0
 
Tensor de deformación infinitesimal:
− 2 3 0
1  ∂u ∂u j  
ε ij = ∇ ( sym
)
u ij =  i +
2  ∂x j ∂xi
 = 3 − 10 0 × 10 −3
  
  0 0 1 

b) Las deformaciones principales (autovalores) se obtienen al resolver el determinante
característico:
− 2 − ε 3 0 
 3 − 10 − ε 0  × 10 −3 = 0

 0 0 1 − ε 

donde ε = ε × 10 −3 . Al desarrollar el determinante anterior obtenemos la ecuación


característica ε 3 − I ε ε 2 + II ε ε − III ε = 0 , donde los invariantes principales de ε vienen
definidos por I ε = Tr (ε ) , II ε =
1
2
{ }
[Tr(ε)]2 − Tr (ε 2 ) , III ε = det(ε) , ver capítulo 1. Luego,
para el problema propuesto los invariantes son:
I ε = Tr (ε ) = (−2 − 10 + 1) × 10 −3 = −11 × 10 −3
−2 3 0 −2 3 0 −2 3 0
 
1
{ }
II ε = [Tr (ε )] − Tr (ε ) =  3 − 10 0 + 3 − 10 0 + 3 − 10 0  × 10 −6 = −1 × 10 −6
2
2 2

 0 0 1 0 0 1 0 0 1 

III ε = det (ε ) = 11 × 10 −9
Resultando en la siguiente ecuación característica:
ε 3 − I ε ε 2 + II ε ε − III ε = 0
⇒ ε 3 + 11 × 10 −3 ε 2 + ε × 11 × 10 −6 − 11 × 10 −9 = 0
Al resolver la ecuación anterior obtenemos los autovalores de ε (deformaciones
principales). Pero, si nos fijamos en el formato de las componentes de ε verificamos que
ε 33 = 1 × 10 −3 ya es una deformación principal y que está asociada a la dirección

Universidad de Castilla- La Mancha Draft Por: Eduardo W. V. Chaves (2012)


Ciudad Real - España
2 CINEMÁTICA DEL CONTINUO 239

nˆ i = [0 0 ± 1] . Luego, para obtener los demás autovalores es suficiente resolver el


siguiente sistema:
− 2 − ε 3  −3
 3  × 10 = 0 ⇒ ε 2 + 12 × 10 −3 ε + 11 × 10 −6 = 0
 − 10 − ε 

2 −3 −6
ε 1 = −1,0 × 10 −3
ε + 12 × 10 ε + 11 × 10 =0 ⇒ 
ε 2 = −11,0 × 10 −3

c) Para dibujar el círculo de Mohr en deformaciones, ver Apéndice A, tenemos que


reestructurar las deformaciones principales tal que ε I ≥ ε II ≥ ε III , i.e.:
ε I = 1,0 × 10 −3 ; ε II = −1,0 × 10 −3 ; ε III = −11,0 × 10 −3
La deformación tangencial máxima viene dada por:
1 ε − ε III
γ max = ε S max = I = 6 × 10 −3
2 2
El círculo de Mohr en deformaciones se puede apreciar en la figura abajo.

ε S (×10 −3 )

ε S max = 12 γ max = 6

ε II = −1

ε III = −11
εI = 1 ε N (×10 −3 )

d) La deformación volumétrica lineal - εV :


ε V = I ε = Tr (ε ) = −12 × 10 −3
Haciendo la descomposición aditiva de ε en una parte esférica y otra desviadora,
ε = ε esf + ε dev , donde la parte esférica viene dada por:
− 4 0 0 
Tr (ε )
ε ijesf = 
δ ij =  0 − 4 0  × 10 −3
3
 0 0 − 4
Y, la parte desviadora por:
 − 2 3 0  − 4 0 0  2 3 0
 
ε ijdev = ε ij − ε ijesf =   3 − 10 0 −  0 − 4 0   × 10 = 3 − 6 0 × 10 −3
   −3

 0 0 0  0 0 − 4  0 0 4




Universidad Castilla- La Mancha Draft Por: Eduardo W. V. Chaves (2012)


Ciudad Real - España
240 PROBLEMAS RESUELTOS DE MECÁNICA DEL MEDIO CONTINUO

Ejemplo 2.69
Dado el movimiento
 x1 = X 1

( − 2t
 x 2 = X 2 + X 1 exp − 1 ) (2.175)

( − 3t
 x3 = X 3 + X 1 exp − 1 )
Encontrar el tensor tasa de deformación ( D ) y compararlo con la tasa del tensor
infinitesimal de deformación ( ε& ).
Solución:
Por definición el tensor tasa de deformación ( D ) es la parte simétrica del tensor gradiente
espacial de la velocidad:
1
D= (l + l T ) r r
2 y ε ( x , t ) = ∇ sym u (2.176)
r
l = ∇ xv

El tensor infinitesimal de deformación por definición es igual la parte simétrica del


gradiente de los desplazamientos:
r Dε
ε = (∇u) sym ⇒ ε& ≡ (2.177)
Dt
r r r
El campo de desplazamientos viene dado por u = x − X . Considerando las ecuaciones del
movimiento dadas, las componentes del campo de desplazamiento quedan:
u1 = x1 − X 1 = X 1 − X 1 = 0

(
− 2t
)
u 2 = x 2 − X 2 = X 2 + X 1 exp − 1 − X 2 = X 1 exp − 1 (
− 2t
)

(
−3t
)
u 3 = x 3 − X 3 = X 3 + X 1 exp − 1 − X 3 = X 1 exp − 1(
− 3t
)
r
 r Du 
El campo de velocidades viene definido por  v =  . Luego, las componentes del campo
 Dt 
de velocidades, en coordenadas materiales, son:
V1 = 0
 − 2t
V2 = X 1 (−2exp ) (2.178)
 − 3t
V3 = X 1 ( −3exp )
Teniendo en cuenta las ecuaciones inversas del movimiento:
 x1 = X 1  X 1 = x1
 − 2t  − 2t
 x 2 = X 2 + X 1 (exp − 1) ⇒  X 2 = x 2 − x1 (exp − 1) (2.179)
 − 3t  − 3t
 x3 = X 3 + X 1 (exp − 1)  X 3 = x3 − x1 (exp − 1)
podemos obtener el campo de velocidades en coordenadas espaciales:
v1 = 0
 − 2t
v 2 = −2 x1 exp (2.180)
 −3t
v3 = −3x1exp

Universidad de Castilla- La Mancha Draft Por: Eduardo W. V. Chaves (2012)


Ciudad Real - España
2 CINEMÁTICA DEL CONTINUO 241

Las componentes del tensor gradiente espacial de la velocidad ( l ) vienen dadas por:

 0 0 0
∂vi 
0 0
r
( l ) ij = (∇ x v ) ij = = − 2exp − 2t (2.181)
∂x j 
 − 3exp −3t 0 0

 0 0 0  0 0 0
T

1 1    

(D) ij = ( l ij + l ji ) = − 2exp − 2t
0 0 +  − 2exp − 2t 0 0 
2 2  
  − 3exp −3t 0 0  − 3exp −3t 0 0 
 
 3  (2.182)
 0 − exp − 2t − exp −3t 
2
 
=  − exp − 2t 0 0 
− 3 exp −3t 0 0 
 2 

También obtenemos el tensor spin W = l anti

 3 
 0 exp − 2t exp −3t 
2
Wij =
1
2
(
l ij − l ji ) 
=  − exp − 2t 0 0

 (2.183)
− 3 exp −3t 0 0 
 2 

Tensor de deformación Infinitesimal (ε )


Conocido el campo de desplazamiento:
u1 = 0
 − 2t
u 2 = x1 (exp − 1) (2.184)
 − 3t
u 3 = x1 (exp − 1)
Las componentes del gradiente de desplazamientos vienen dadas por:
 0 0 0
r ∂u i 
(∇u)ij = =  exp − 1 0 0
− 2t
( ) (2.185)
∂x j
 exp −3t − 1 0 0 ( )
r
Podemos decomponer (∇u) en una parte simétrica y una antisimétrica:

(∇u)ij = (∇ symu)ij + (∇ anti u)ij


r r r
(2.186)
= (ε )ij + (ω )ij

La parte simétrica:

Universidad Castilla- La Mancha Draft Por: Eduardo W. V. Chaves (2012)


Ciudad Real - España
242 PROBLEMAS RESUELTOS DE MECÁNICA DEL MEDIO CONTINUO

 0 0 0  0 0 0
T

1   
( r
∇ sym u ij ) =
2 
( − 2t
)  (

exp − 1 0 0 +  exp − 1 0 − 2t
) 0 
( ) (
  exp −3t − 1 0 0  exp −3t − 1 0

) 0


 (2.187)
 0 exp − 2t − 1 exp −3t − 1
1 
= exp − 2t − 1 0 0  = ε ij
2 − 3t 
 exp − 1 0 0 
También proporcionamos el tensor spin infinitesimal:

 0 − (exp −2t − 1) − (exp −3t − 1)0


1 
(ω )ij = (exp − 2t − 1) 0 0  (2.188)
2 − 3t 
(exp − 1) 0 0 

Luego, la tasa de ε :
  0 exp −2t − 1 exp −3t − 1 

(ε& )ij =
D
(ε )ij = D  1 exp − 2t − 1 0 0


Dt Dt  2   
 exp − 1 − 3t
0 0
  
 3  (2.189)
 0 − exp − 2t − exp −3t 
2
 
=  − exp − 2t 0 0 
− 3 exp −3t 0 0 
 2 

Con lo que concluimos que:


D = ε& (2.190)

Ejemplo 2.70
En un punto de un sólido el gradiente de los desplazamientos viene representado por sus
componentes como:
4 − 1 − 4
r
(∇u) ij = 1 − 4 2  × 10 −3 (2.191)
4 0 6 

Determinar:
a) Las componentes del tensor infinitesimal de deformación y rotación;
b) Las componentes de la parte esférica y desviadora del tensor infinitesimal de
deformación;
c) Los invariantes principales de ε : I ε , II ε , III ε ;
d) Los autovalores y autovectores del tensor de deformación.
Solución:
a) El tensor infinitesimal de deformación ( ε ) viene dado por la parte simétrica del
gradiente de los desplazamientos:

Universidad de Castilla- La Mancha Draft Por: Eduardo W. V. Chaves (2012)


Ciudad Real - España
2 CINEMÁTICA DEL CONTINUO 243

r 1
2
r
[ r
ε = ∇ sym u = (∇u) + (∇u) T ] (2.192)

Luego:
 4 − 1 − 4  4 1 4  8 0 0   4 0 0
1   1

ε ij =  1 − 4 2  +  − 1 − 4 0  × 10 = 0 − 8 2  × 10 = 0 − 4 1 × 10 −3
  −3  −3
2 2
 4 0 6   − 4 2 6  0 2 12 0 1 6
r
El tensor spin infinitesimal ω = ∇ anti u
  4 − 1 − 4  4 1 4   0 − 2 − 8  0 − 1 − 4
1      1
−3
ωij =  1 − 4 2  −  − 1 − 4 0  × 10 =  2 0 2  × 10 = 1 0
 −3
1  × 10 −3
2 2
  4 0 6   − 4 2 6  8 − 2 0   4 − 1 0 

b) Descomponiendo de forma aditiva el tensor en una parte esférica y una parte


desviadora:
ε = ε esf + ε dev (2.193)
donde la parte esférica viene dada por:
2 0 0
Tr (ε ) 6 × 10 −3
ε esf
= 1= 1 = 2 × 10 −31 ⇒ ε ijesf = 0 2 0 × 10 −3 (2.194)
3 3
0 0 2

La parte desviadora viene dada por:


 4 0 0 2 0 0  2 0 0
ε ijdev   −3  
= 0 − 4 1 × 10 − 0 2 0 × 10 = 0 − 6 1  × 10 −3
−3
(2.195)
0 1 6 0 0 2 0 1 4

c) Los invariantes principales del tensor ε son:


I ε = Tr (ε ) = 6 × 10 −3
−4 1 4 0 4 0 
II ε =  + +  × 10 − 6 = −17 × 10 −6
 (2.196)
 1 6 0 6 0 − 4 
III ε = (4 × (−4) × 6 − 4 ) × 10 −9 = −100 × 10 −9

d) Teniendo en cuenta las componentes del tensor de deformación:


 4 0 0
ε ij = 0 − 4 1  × 10 −3 (2.197)
0 1 6

Ya verificamos que ε1 = 4 × 10 −3 es un autovalor y la dirección [± 1,0,0] es el autovector


asociado a ε1 . Para encontrar los demás autovalores hay que obtener la solución del
determinante característico:
−4−λ 1
=0 (2.198)
1 6−λ

Universidad Castilla- La Mancha Draft Por: Eduardo W. V. Chaves (2012)


Ciudad Real - España
244 PROBLEMAS RESUELTOS DE MECÁNICA DEL MEDIO CONTINUO

(−4 − λ )(6 − λ ) − 1 = 0
λ2 − 2λ − 25 = 0
2
− b ± b 2 − 4ac 2 ± (−2) − 4 × 1 × (−25) 2 ± 4 + 4 × 25
λ= = = = 1 ± 26
2a 2 ×1 2 (2.199)

λ 1 = 6,0990
⇒
λ 2 = −4,099
Luego:

ε1 = 4 × 10 −3 ; ε 2 = 6,0990 × 10 −3 ; ε 3 = −4,099 × 10 −3 (2.200)

Reestructurando las deformaciones:

ε I = 6,0990 × 10 −3 ; ε II = 4 × 10 −3 ; ε III = −4,099 × 10 −3 (2.201)

Ejemplo 2.71
Encontrar el tensor de deformación infinitesimal y el tensor de rotación infinitesimal para
el siguiente campo de desplazamiento:
 x12 
 
u i =  x1 x 2 
 0 
 
Solución:
Tensor de deformación Infinitesimal
En el régimen de pequeñas deformaciones, el tensor de deformación viene dado por:
1  ∂u i ∂u j 

E ijL ≈ eijL ≈ ε ij = +
2  ∂x j ∂x i 

Tenemos que hallar el gradiente del desplazamiento:
 ∂u 1 ∂u1 ∂u1 
 
 ∂x1 ∂x 2 ∂x 3   2 x
1 0 0
∂u j  ∂u 2 ∂u 2 ∂u 2  
= = x2 x1 0
∂x k  ∂x1 ∂x 2 ∂x 3  
   0 0 0
 ∂u 3 ∂u 3 ∂u 3 
 ∂x1 ∂x 2 ∂x 3 

Con eso podemos obtener:


 x2
1  ∂u ∂u j   2 x 0 0  2 x1 x2 0   2 x1 0 
E ijL ≈ eijL ≈ ε ij =  i +  1  1  2

2  ∂x j ∂x i  =   x2 x1 0 +  0 x1 0  =  x 2 
 2  x1 0 
  0 0 0  0 0 0   2 
 0 0 0 
Tensor de rotación infinitesimal:

Universidad de Castilla- La Mancha Draft Por: Eduardo W. V. Chaves (2012)


Ciudad Real - España
2 CINEMÁTICA DEL CONTINUO 245

1  ∂u i ∂u j 

ω ij = −
2  ∂x j ∂x i 

 − x2 
0 0
  2 x1 0 0   2 x1 x2 0   2

1   x
=   x2 x1 0  −  0 x1 0   =  2 0 0
2 2 
  0 0 0   0 0 0    0 0 0
 
 

Ejemplo 2.72
En la Figura 2.21 se muestra la transformación que experimenta el cuadrado ABCD de
lado unitario.

X 2 , x2
x2′
D
C
D′
1
45º
B C′
A = A′ X 1 , x1
1
B′
x1′
Figura 2.21: Cuerpo sometido a una rotación.
Se pide:
a) Plantear las ecuaciones del movimiento;
b) ¿Es válida la teoría de pequeñas deformaciones? Probar y Justificar;
c) ¿Es válida la teoría de deformación finita (grandes deformaciones)? Probar.

Solución:
La ley de transformación entre los sistemas x ⇒ x ′ viene dada por:
 2 2 
 − 0
 x1′   cos θ sin θ 0  x1   x1′   2 2   x1 
      θ= −45º    2 2  
 x 2′  =  − sin θ cos θ 0  x 2   → x 2′  =  0  x 2 
x′   0 2 2
 3  0 1   x3   x′  
 3 0 0
 
1  x 3 
 
 

Si consideramos los sistemas materiales y espaciales superpuestos, las ecuaciones de


movimiento quedan definidas por la inversa de la expresión anterior, es decir, x ′ ⇒ x :

Universidad Castilla- La Mancha Draft Por: Eduardo W. V. Chaves (2012)


Ciudad Real - España
246 PROBLEMAS RESUELTOS DE MECÁNICA DEL MEDIO CONTINUO

 2 2 
 0
 x1   2 2  X 1 
   2 2  
 x 2  = − 0  X 2 
x   2 2  
 3 0 0 1  X 3 
 
 

 2 2
 x1 = X1 + X2
 2 2

 2 2
 x 2 = − 2 X 1 + 2 X 2

Por ejemplo, el punto C en la configuración de referencia tiene coordenadas materiales


2 2 2 2
X 1C = 1 , X 2C = 1 . Tras el movimiento: x1C = (1) + (1) = 2 , x 2C = − (1) + (1) = 0
2 2 2 2
Campo de desplazamientos:
 2 2  2  2
u1 = x1 − X 1 = X1 − X 2 − X 1 = X 1  − 1 − X2
 2 2  2  2

 2 2 2  2 
u 2 = x 2 − X 2 = X1 + X2 − X2 = X 1 + X 2  − 1
 2 2 2  2 
Gradiente material de los desplazamientos:
 ∂u1 ∂u1 ∂u1   2 2 
   −1 − 0
 ∂X 1 ∂X 2 ∂X 3   2 2 
∂u i  ∂u 2 ∂u 2 ∂u 2   2 2 
= = − 1 0
∂X j  ∂X 1 ∂X 2 ∂X 3   2 2
   
 ∂u 3 ∂u 3 ∂u 3   
 ∂X 1 ∂X 2 ∂X 3   0 0 0

El tensor de deformación infinitesimal viene definido como ε = ∇ symu =


r 1
2
[ r r
]
(∇u) + (∇u)T ,
con o obtenemos:
 2 
 −1 0 0
 2 
 2 
ε ij =  0 − 1 0 ≠ 0 ij
2
 0 0 0
 
 

Como para un movimiento de sólido rígido el tensor de deformación tiene que ser cero, es
decir, ε = 0 (tensor de deformación infinitesimal), E = 0 (tensor de deformación de
Green-Lagrange), e = 0 (tensor de deformación de Almansi). Calculando las componentes
del tensor de deformación de Green-Lagrange:
0 0 0
1  ∂u ∂u j ∂u k ∂u k  
 = 0 0 0
E ij =  i + +
2  ∂X j ∂X i ∂X i ∂X j   
 0 0 0
 

Universidad de Castilla- La Mancha Draft Por: Eduardo W. V. Chaves (2012)


Ciudad Real - España
2 CINEMÁTICA DEL CONTINUO 247

Ejemplo 2.73
Un rectángulo de base y altura b se gira en sentido antihorario 30º . Tras el giro el
rectángulo sufre una deformación de tal forma que la base mantiene su longitud inicial y la
altura se dobla. Calcular el gradiente de deformación, el tensor derecho de deformación de
Cauchy-Green, y el tensor de deformación de Green-Lagrange.
Solución:
X 2 , x2
x2′ C′

D′

D C x1′
2b 30º
B′
30º B
A = A′ b X 1 , x1

Figura 2.22: Cuerpo sometido rotación/deformación.

Fijaros que podemos hacer la descomposición del movimiento por: primero una
deformación y a continuación una rotación:
El movimiento de deformación
X 2 , x2 viene gobernado por el tensor
derecho de estiramiento de la
descomposición polar:
D′′ C ′′
1 0 0
U ij = 0 2 0
0 0 1
2b D C
donde hemos aplicado la definición
del estiramiento. Fijemos que son
los propios estiramientos
principales. A continuación
B B′
aplicamos una rotación, donde las
b X 1 , x1 componentes del tensor R son las
A = A′ mismas que la matriz de
r
transformación del sistema x ′ al
r
sistema x :

Universidad Castilla- La Mancha Draft Por: Eduardo W. V. Chaves (2012)


Ciudad Real - España
248 PROBLEMAS RESUELTOS DE MECÁNICA DEL MEDIO CONTINUO

cos θ − sin θ 0
R ij =  sin θ cos θ 0
 0 0 1 

Luego, aplicando la descomposición polar por la derecha F = R ⋅ U :


cos θ − sin θ 0 1 0 0 cos θ − 2 sin θ 0
Fij = R ik U kj =  sin θ cos θ 0 0 2 0 =  sin θ 2 cos θ 0
 0 0 1 0 0 1  0 0 1
Para el problema propuesto, tenemos que:
cos 30º − 2 sin 30º 0
Fij =  sin 30º 2 cos 30º 0
 0 0 1
r r r
Como se trata de un caso de deformación homogénea se cumple x = F ⋅ X + c , en este
r r
caso con c = 0 . Por ejemplo, para una partícula que en la configuración de referencia
ocupaba el punto D , en la configuración actual estará según:
 x1D  cos 30º − 2 sin 30º 0  X 1D  cos 30º − 2 sin 30º 0 0 − 2b sin 30º 
 D   D      
 x 2  =  sin 30º 2 cos 30º 0  X 2  =  sin 30º 2 cos 30º 0 b  =  2b cos 30º 
x D   0 0 1  X 3D   0 0 1 0  0 
 3   
hecho que se puede comprobar fácilmente a través de la Figura 2.22.
A través de la definición del tensor derecho de deformación de Cauchy-Green, C = F T ⋅ F ,
podemos obtener las componentes cartesianas:
 cos θ sin θ 0 cos θ − sin θ 0 1 0 0
C ij = Fki Fkj =  − sin θ cos θ 0  sin θ cos θ 0 = 0 4 0
 0 0 1   0 0 1 0 0 1

1
El tensor de deformación de Green-Lagrange, E = (C + 1) , y sus componentes
2
cartesianas quedan:
 1 0 0   1 0 0    0 0 0 
1      
E ij =  0 4 0  −  0 1 0   =  0 1,5 0 
2 
 0 0 1   0 0 1    0 0 0 
 
Observemos que el espacio original coincide con el espacio principal de deformación.
También podíamos haber obtenido las componentes de C y E a través de sus
3 3
1
representaciones espectrales: C = ∑ λ2a N
ˆ (a) ⊗ N
ˆ (a) , E =
∑ (λ2a − 1)Nˆ (a ) ⊗ Nˆ ( a) , donde
a =1 a =1 2
λ a son los estiramientos principales.

Universidad de Castilla- La Mancha Draft Por: Eduardo W. V. Chaves (2012)


Ciudad Real - España
2 CINEMÁTICA DEL CONTINUO 249

2.2 Ejercicios Propuestos

Problema 2.1
Considere el movimiento del continuo dado por las siguientes ecuaciones:

 x1 = X 1 (1 + αt 3 )

 x2 = X 2 (2.202)
x = X
 3 3

donde α es una constante. Determinar los campos de desplazamiento, velocidad y


aceleración en las descripciones material y espacial.

Problema 2.2
Un movimiento del medio continuo viene definido por las siguientes componentes de la
velocidad:
3 x1 x2 5 x32
v1 = ; v2 = ; v3 = (2.203)
1+ t 1+ t 1+ t
r r
Asumiendo que la configuración de referencia (t = 0) se cumple que x = X .
Se pide:
a) Obtener la trayectoria de la partícula;
b) Expresar las componentes de la velocidad en coordenadas materiales;
c) Obtener las componentes de la aceleración en coordenadas espaciales y materiales.

Problema 2.3
Un cuerpo se halla bajo deformación homogénea.
 3
 x1 = 2 X 1 + 4 2 X 2

 3 1
 x2 = − X 1 + X 2 + 2 X3 (2.204)
 4 4
 3 1
 x3 = X 1 − 4 X 2 + 4 2 X 3

Encontrar:
a) La dirección del elemento de línea en la configuración deformada cuyo elemento de
línea en la configuración de referencia estaba según la dirección (1,1,1) ;
b) El estiramiento del elemento de línea.

Universidad Castilla- La Mancha Draft Por: Eduardo W. V. Chaves (2012)


Ciudad Real - España
250 PROBLEMAS RESUELTOS DE MECÁNICA DEL MEDIO CONTINUO

Problema 2.4
Dados los siguientes campos de desplazamientos en el plano x − y :
r 1 1
a) u = γ x 2 eˆ 1 + γ x1 eˆ 2
2 2
r 1 1
b) u = − γ x 2 eˆ 2 + γ x1 eˆ 2
2 2
r
c) u = γ x1 eˆ 2
Para cada campo de desplazamiento con γ << 1 , dibujar los desplazamientos que sufren las
partículas que originalmente estaban en un cuadrado determinado por x1 = 0 , x1 = 1 ,
x2 = 0 y x2 = 1 . Ver Figura 2.23

x2

1 t=0

O 1 x1

Figura 2.23: Dominio en la configuración de referencia

Problema 2.5
Para un campo de desplazamientos dado en coordenadas cilíndricas por:
r
u = a r eˆ r + b r z eˆ θ + c sin θ eˆ z (2.205)
donde a , b y c son constantes. Determinar las componentes del tensor de deformación
infinitesimal en coordenadas cilíndricas.

Problema 2.6
r r r
Si el campo de aceleración a es la derivada material v& del campo de la velocidad v ,
encontrar la aceleración en el punto (1,1,0) en el tiempo t = 0 si la velocidad en la
descripción Euleriana viene dada por:
r
[ ]
v = C ( x13 + x1 x22 )eˆ 1 − ( x12 x2 + x23 )eˆ 2 exp −at (2.206)

donde C y a son constantes.

Problema 2.7
El campo de velocidad de un fluido viene dado por:
r
v = x1eˆ 1 + (2 x1 − x 2 )eˆ 2 + ( x1 − x3 )eˆ 3 (2.207)

Universidad de Castilla- La Mancha Draft Por: Eduardo W. V. Chaves (2012)


Ciudad Real - España
2 CINEMÁTICA DEL CONTINUO 251

y la distribución de temperatura es:


T = 3 x 2 + x3 t (2.208)
Encontrar la tasa de cambio en el tiempo de la temperatura.

Problema 2.8
Un movimiento de un fluido viene representado por las siguientes ecuaciones:
 x1 = X 1 + X 2 t + X 3t 2
 2
 x2 = X 2 + X 3t + X 1t (2.209)
 2
 x3 = X 3 + X 1t + X 2 t
Encontrar la velocidad y aceleración para los siguientes casos:
a) Para una partícula que estaba en el punto (1,1,1) en la configuración de referencia;
b) Para una partícula que para el tiempo t se encuentra en (1,1,1) .
Explicar por qué este movimiento pierde sentido físico cuando t → 1 .

Problema 2.9
Encontrar las componentes de los tensores F , C , B , F −1 , C −1 y B −1 para la siguiente
deformación:
 x1 = a1 ( X 1 + αX 2 )

 x2 = a2 X 2 (2.210)
x = a X
 3 3 3

donde a1 , a 2 , a3 y α son constantes.

Problema 2.10
Para la siguiente deformación
u1 = AX 1 + BX 1 ( X 12 + X 22 ) −1
 2 2 −1
u 2 = AX 2 + BX 2 ( X 1 + X 2 ) (2.211)
u = CX
 3 3

donde A , B y C son constantes.


Encontrar las componentes de los tensores F , E y Ω .

Problema 2.11
Dadas las ecuaciones del movimiento:
 x1 = X 1 + 4 X 1 X 2 t
 2
 x2 = X 2 + X 2 t
 2
 x3 = X 3 + X 3 t

Universidad Castilla- La Mancha Draft Por: Eduardo W. V. Chaves (2012)


Ciudad Real - España
252 PROBLEMAS RESUELTOS DE MECÁNICA DEL MEDIO CONTINUO

a) Hallar el campo de velocidad;


b) Hallar el campo de aceleración;
c) Para el instante de tiempo t = 1s , obtener el tensor de deformación infinitesimal.

Problema 2.12
Considérese un cuerpo que sufre un movimiento de cuerpo rígido, tal y como se muestra
en la siguiente figura:

x2 , X 2 x2

x1′
x 2′

X 1 , x1 x1

Figura 2.24: Cuerpo sometido a una rotación.

Se pide:
a) Plantear las ecuaciones del movimiento
b) Obtener el campo de desplazamientos
c) Obtener el tensor de deformación infinitesimal
d) Obtener el tensor de deformación de Green-Lagrange
e) ¿Qué podemos decir con respecto a los tensores de deformación para un movimiento de
cuerpo rígido?
f) ¿Qué aproximación tenemos que adoptar para que se cumpla la teoría de pequeñas
deformaciones para el movimiento dado?

Problema 2.13
¿Qué se entiende por deformación homogénea?

Problema 2.14
Definir: Estado de Tensión Plana y Estado de Deformación Plana. Para cada estado, ¿qué
simplificaciones son consideradas? Dar ejemplos prácticos en los que se pueda aplicar uno
y otro estado.

Problema 2.15
Plantear la Descomposición Polar del Gradiente de Deformación.

Universidad de Castilla- La Mancha Draft Por: Eduardo W. V. Chaves (2012)


Ciudad Real - España
2 CINEMÁTICA DEL CONTINUO 253

Problema 2.16
Justificar si son ciertas o falsas las siguientes afirmaciones:
a) Si el campo de velocidades es estacionario, entonces el campo de aceleración
también lo es.
b) Si el campo de velocidades es uniforme, entonces el campo de aceleraciones es
siempre nulo;
c) Si el campo de velocidades es estacionario y el medio es incompresible, el campo
de aceleraciones es siempre nulo.

Problema 2.17
Partiendo de las ecuaciones del movimiento de cuerpo rígido, obtener los siguientes
tensores:
F - Gradiente de deformación
C - Tensor derecho de deformación de Cauchy-Green
b - Tensor izquierdo de deformación de Cauchy-Green
E - Tensor de deformación de Green-Lagrange
e - Tensor de deformación de Almansi.

Problema 2.18
Obtener la tasa del determinante del Jacobiano
a) ( J& = f ( E& , C ) ) en función de la tasa del tensor de deformación de Green-Lagrange
( E& ) y del tensor derecho de deformación de Cauchy-Green;
b) ( J& = f (C& , C ) ) en función de la tasa del tensor derecho de deformación de Cauchy-
Green ( C& ),y del tensor derecho de deformación de Cauchy-Green.
Datos
D r
F ≡ J& = F ∇ ⋅v
Dt r r 1
= J ∇ ⋅v E ( X , t ) = ( F T ⋅ F − 1)
r
= J Tr (∇v )
2 E& = F T ⋅ D ⋅ F
1
= (C − 1)
= J Tr ( l ) 2
= J Tr (D)

Problema 2.19
Dado el campo de velocidad vi = 2 x 2 δ1i , obtener el tensor tasa de deformación y el tensor
spin.
Respuesta:
0 1 0  0 1 0
D ij = 1 0 0 ; Wij =  − 1 0 0
0 0 0  0 0 0

Universidad Castilla- La Mancha Draft Por: Eduardo W. V. Chaves (2012)


Ciudad Real - España
254 PROBLEMAS RESUELTOS DE MECÁNICA DEL MEDIO CONTINUO

Universidad de Castilla- La Mancha Draft Por: Eduardo W. V. Chaves (2012)


Ciudad Real - España
3 Tensiones
3.1 Ejercicios Resueltos

3.1.1 Fuerza, Tensor de Tensiones, Vector Tensión

Ejemplo 3.1
Ignorando la curvatura de la superficie de la tierra, el campo gravitacional puede suponerse
de la forma como se muestra en la Figura 3.1, donde g es la aceleración de la gravedad.
Obtener la fuerza resultante que actúa en el cuerpo B .

g
B

x3
x2

x1
Figura 3.1: Campo gravitacional.

Solución:
Todos los cuerpos situados en este campo se encontrarán sometidos a la fuerza:
 0 
b i ( x , t ) =  0 
r

− g 

La fuerza másica que actúa sobre el cuerpo es:


256 PROBLEMAS RESUELTOS DE MECÁNICA DEL MEDIO CONTINUO

 0 
 
r 0
Fi = ∫ ρb i ( x , t ) dV =  
 
V

 V

 − ρ g dV 

 kg   m  kg m
Podemos verificar la unidad de F : [F] = ∫  3  2 
dV = 2 = N ( Newton )
V  m  s  s

Ejemplo 3.2
Las componentes del tensor de tensiones en el punto P son:

 8 −4 1 
σ ij =  − 4 3 0,5 Pa
 1 0,5 2  x3

C (0,0,5)
a) Calcular el vector tensión en el punto P
según la dirección del plano ABC ,
como se indica en la Figura 3.2.
r B (0,2,0)
b) Obtener el vector tensión normal ( σ N ) O
r x2
y el vector tensión tangencial ( σ S ),
ver Apéndice A en Chaves(2007, 3ª edición). A(3,0,0)
x1

Figura 3.2: Plano ABC .


Solución:
En primer lugar, deberemos obtener la dirección normal a este plano, para ello escogemos
dos vectores pertenecientes al plano y hacemos el producto vectorial entre ellos:
→ → → → → →
BA = OA− OB = 3eˆ 1 − 2eˆ 2 + 0eˆ 3 ; BC = OC − OB = 0eˆ 1 − 2eˆ 2 + 5eˆ 3
El vector normal al plano ABC viene dado a través del producto vectorial de los vectores
definidos anteriormente:
eˆ 1 eˆ 2 eˆ 3
r → →
n = BC ∧ BA = 0 − 2 5 = 10eˆ 1 + 15eˆ 2 + 6eˆ 3
3 −2 0
r
r n 10 15 6
El versor asociado a n será: nˆ = r = eˆ 1 + eˆ 2 + eˆ 3
n 19 19 19
ˆ
Utilizando la ecuación t i(n) = σ ij nˆ j , podemos obtener las componentes del vector tensión
de la forma:

Universidad de Castilla- La Mancha Draft Por: Eduardo W. V. Chaves (2012)


Ciudad Real - España
3 TENSIONES 257

 t1   8 − 4 1  10   t1   26 
 t  = 1  − 4 3 0,5 15  Pa 1 
⇒  t 2  =  8  Pa
 
 2  19    19
 t 3   1 0,5 2   6   t 3   29,5
r ˆ
b) El vector tensión t (n) asociado a la dirección n̂ puede descomponerse en una componente
r r
normal σ N y en otra tangencial σ S tal como se indica en la Figura 3.3. La suma vectorial
de estos vectores resulta:
r ˆ r r r ˆ
t (n ) = σ N + σ S ó t (n) = σ N nˆ + σ S sˆ
r r
donde σ N y σ S son los módulos de σ N y de σ S , respectivamente.
Como visto en el Apéndice A, σ N puede ser obtenido a través de las siguientes relaciones
r ˆ
σ N = t (n) ⋅ nˆ = (σ ⋅ nˆ ) ⋅ nˆ = nˆ ⋅ σ ⋅ nˆ = σ : (nˆ ⊗ nˆ )
ˆ
= t i(n) nˆ i = (σ ij nˆ j )nˆ i = nˆ i σ ij nˆ j = σ ij (nˆ i nˆ j )

Luego:
10 
1
σN = t i nˆ i ⇒ σN = 2 [26 8 29,5] 15 ≈ 1,54 Pa
19
 6 
r ˆ
t (n)
x3
r
σN
r
σS
ŝ n̂
P
ê 3

ê 2 x2
ê1

x1

Figura 3.3: Componentes normal y tangencial del vector tensión.

La componente tangencial viene dada por:


r ˆ
σ S = t (n) ⋅ sˆ = (σ ⋅ nˆ ) ⋅ sˆ = sˆ ⋅ σ ⋅ nˆ = σ : (nˆ ⊗ nˆ )
ˆ
= t i(n) sˆ i = (σ ij nˆ j )sˆ i = sˆ i σ ij nˆ j = σ ij (sˆ i nˆ j )
La componente tangencial también puede ser obtenida a través del teorema de Pitágoras:
r ˆ 2 ˆ ˆ
t (n) = σ 2N + σ 2S ⇒ σ 2S = t (i n) t (i n) − σ 2N

donde

Universidad de Castilla- La Mancha Draft Por: Eduardo W. V. Chaves (2012)


Ciudad Real - España
258 PROBLEMAS RESUELTOS DE MECÁNICA DEL MEDIO CONTINUO

 26 
1
= 2 [26 8 29,5]  8  ≈ 4,46
ˆ ˆ
t i(n) t i(n)
19
29,5

Resultando que:
ˆ ˆ
σ S = t i(n) t i(n) − σ 2N = 4,46 − 2,3716 ≈ 2,0884 Pa

Ejemplo 3.3
El estado tensional en un punto del continuo viene representado a través de las
componentes del tensor de tensiones de Cauchy como:
2 1 0
σ ij = 1 2 0  Pa
0 0 2 

a) Obtener las componentes de σ en un nuevo sistema x1′ , x 2′ , x3′ , donde la matriz de


transformación viene dada por a ij , ver Figura 3.4.
b) Obtener los invariantes principales de σ ;
c) Obtener los autovalores y autovectores de σ . Verificar también si los autovectores
forman una matriz de transformación de base entre el sistema original y el principal;
d) Obtener la representación gráfica del tensor de tensiones de Cauchy, i.e. el círculo de
Mohr en tensiones (ver Apéndice A);
e) Obtener la parte esférica ( σ sph ) y la parte desviadora ( σ dev ) del tensor σ . También,
obtener los invariantes principales de σ dev y los autovalores de σ dev ;
f) Obtener la tensión normal octaédrica ( σ oct
N ) y la tensión tangencial octaédrico (o también conocida
oct
como tensión de corte octaédrica) ( σ S ).

x3
x 2′
3 0 − 4
1
γ1 a ij = A = 0 5 0 
x3′ 5
4 0 3 
x1′
ê′2 donde
ê 3
ê′3
ê′1 β1 a11 = cos α 1
ê 2 a12 = cos β 1
ê1 x2
a13 = cos γ 1
α1
M

x1

Figura 3.4: Matriz de transformación

Solución:

Universidad de Castilla- La Mancha Draft Por: Eduardo W. V. Chaves (2012)


Ciudad Real - España
3 TENSIONES 259

a) Como hemos visto en el capítulo 1 (Chaves (2007)), la ley de transformación de las


componentes del tensor de segundo orden viene dada por:
σ′ij = aik a jl σ kl Forma
 Matricial
→ σ′ = A σ A T
Luego:
T

 σ 11 σ12′ σ13′   3 0 − 4  1 1 0   3 0 − 4   2 0,6 0 
1 
σ ′
 12 σ ′22 σ 23  = 2 0 5 0   2 2 0  0 5 0  = 0,6 2 0,8
′      
5
σ13
′ σ ′23 σ ′33   4 0 3  0 0 2  4 0 3   0 0,8 2 

donde estas componentes se pueden apreciar en la Figura 3.5.


b) Los invariantes principales del tensor de tensiones de Cauchy stress tensor son obtenido
a través de las expresiones:
I σ = Tr (σ ) = σ ii = σ11 + σ 22 + σ 33

II σ =
1
2
[ 1
] (
( Trσ ) 2 − Tr (σ 2 ) = σ ii σ jj − σ ij σ ij
2
)
2 2
= σ11σ 22 + σ11σ 33 + σ 33 σ 22 − σ12 − σ13 − σ 223

III σ = det (σ ) =  ijk σ i1σ j 2 σ k 3 =


1
6
(
σ ii σ jj σ kk − 3σ ii σ jk σ jk + 2σ ij σ jk σ ki )
= σ11σ 22 σ 33 + 2σ12 σ 23 σ13 − σ11σ 223 − σ 22 σ132
− σ 33 σ122

Reemplazando los valores del problema propuesto obtenemos que:


2 0 2 0 2 1
Iσ = 6 ; II σ = + + = 11 ; III σ = 6
0 2 0 2 1 2

c) Los valores principales (autovalores) ( σ i ) y direcciones principales ( nˆ ( i ) ) son obtenidos a


través del siguiente sistema de ecuaciones:
2 − σ 1 0   n1   0 
 1 2−σ 0  n 2  = 0

 0 0 2 − σ n 3  0

Para obtener las soluciones no triviales de nˆ ( i ) , tenemos que resolver el siguiente


determinante característico:
2−σ 1 0
σ ij − σδ ij = 1 2−σ 0 =0
0 0 2−σ
Pero, si nos fijamos en las componentes del tensor de tensiones de Cauchy, verificamos
que ya conocemos un autovalor y autovector ya que las componentes tangenciales según
dirección x3 son iguales a cero, luego:
σ1 = 2 Dirección
  principal
 → n1(1) = n (21) = 0 , n3(1) = ±1
Para obtener los autovalores restantes, es suficiente resolver el determinante:
2−σ 1 σ 2 = 1
= (2 − σ ) − 1 = 0
2
⇒ 
1 2−σ σ 3 = 3
Expresando así las componentes del tensor de tensiones de Cauchy en el espacio principal:

Universidad de Castilla- La Mancha Draft Por: Eduardo W. V. Chaves (2012)


Ciudad Real - España
260 PROBLEMAS RESUELTOS DE MECÁNICA DEL MEDIO CONTINUO

2 0 0
σ ′ij′ = 0 1 0  Pa
0 0 3

Dirección principal asociada al autovalor σ 2 = 1 :

2 − 1 1 0  n1( 2 )  0 
 ( 2 )    n1 + n 2 = 0
( 2) ( 2)
 1 2 −1 
0  n 2  = 0  ⇒  ( 2 ) ⇒ n1( 2 ) = −n (22 )
 n + n 2 = 0
( 2)
 0 0 2 − 1 n 3( 2 )  0   1
2 2
Con n 3( 2 ) = 0 y utilizando la restricción n1( 2 ) + n (22 ) = 1 obtenemos que:
1  1 −1 
n1( 2 ) = −n (22 ) = , luego nˆ i( 2 ) =  0
2  2 2 
Dirección principal asociada al autovalor σ 2 = 3 :

2 − 3 1 0  n1( 3)  0
 ( 3)    − n1 + n 2 = 0
( 3) ( 3)
 1 2−3 
0  n 2  = 0 ⇒  ( 3) ⇒ n1( 3) = n (23)
 n − n 2 = 0
( 3)
 0 0 2 − 3 n 3( 3)  0  1
2 2
Con n 3(3) = 0 y utilizando la restricción n1( 3) + n (23) = 1 obtenemos que:
1  1 1 
n1( 3) = n (23) = luego nˆ i(3) =  0
2  2 2 
Como hemos visto en el capítulo 1, los autovectores del tensor constituye una matriz de
transformación, B , del espacio original al espacio principal, i.e. σ ′′ = B σ B T . Luego, hay
que cumplir que:
T
 0 0 1  0 0 1
   
σ 1 = 2 0 0    2 1 0  
 0 1 1 1 1
 σ2 = 1 0  =  0  1 2 0   0
 2 2   2 2 
 0 0 σ 3 = 3   0 0 2   1 
1 −1 −1
 0  0
 2 2   2 2 

Universidad de Castilla- La Mancha Draft Por: Eduardo W. V. Chaves (2012)


Ciudad Real - España
3 TENSIONES 261

x3
x′2
x3′
x2
P
x1 x1′

σ′ = A σ A T
x3

σ 33
x3′
σ′33
σ 23 σ′23 σ′23
σ 13 σ 23 σ′22 x′2
σ 13 ′
σ13
σ 12 σ 22 ′
σ12
x2 ′
σ13
σ 12

σ12
σ 11

σ 11

x1
x1′

σ = A T σ′ A

Figura 3.5: Ley de transformación de base.

d) La representación gráfica de un tensor de segundo orden simétrico, i.e. el círculo de


Mohr, puede ser obtenido tal y como se describe en el Apéndice A. Para ello, debemos
reestructurar los autovalores de tal forma que σ I > σ II > σ III , resultando:
σI = 3 ; σ II = 2 ; σ III = 1
Las tres circunferencias son definidas por:
1 1
Círculo 1 ⇒ ; (centro)C1 = (σ II + σ III ) = 1,5 ; (radio) R1 = (σ II − σ III ) = 0,5
2 2
1 1
Círculo 2 ⇒ ; (centro)C 2 = (σ I + σ III ) = 2,0 ; (radio) R2 = (σ I − σ III ) = 1,0
2 2
1 1
Círculo 3 ⇒ ; (centro)C 3 = (σ I + σ II ) = 2,5 ; (radio) R3 = (σ I − σ II ) = 0,5
2 2

Entonces, el círculo de Mohr en tensiones viene representado en la Figura 3.6.

Universidad de Castilla- La Mancha Draft Por: Eduardo W. V. Chaves (2012)


Ciudad Real - España
262 PROBLEMAS RESUELTOS DE MECÁNICA DEL MEDIO CONTINUO

σS

σ S max = 1

R2

R1 R3 σN
C3
σ III = 1 C1 σ II = 2 σ I = 3 = σ N max

Figura 3.6: Círculo de Mohr en tensiones.

e) Como definido en al capítulo 1, un tensor de segundo orden puede ser descompuesto de


forma aditiva en una parte esférica y otra desviadora:
Notación Tensorial Notación Indicial

σ = σ esf + σ dev 1
σ ij = σ ijesf + σ ijdev = σ kk δ ij + σ ijdev (3.1)
3
= σ m 1 + σ dev = σ m δ ij + σ ij dev

La representación esquemática de estas componentes se puede apreciar en la Figura 3.7. El


valor de σ m viene dado por:
σ11 + σ 22 + σ 33 σ1 + σ 2 + σ 3 1 1 I 6
σm = = = σ kk = Tr (σ ) = σ = = 2
3 3 3 3 3 3
Luego, la parte esférica queda definida por:
 2 0 0
σ ijesf = σ m δ ij = 2δ ij = 0 2 0
0 0 2
Y, la parte desviadora por:
 σ11 σ12 σ13  σ m 0 0 
σ ijdev = σ12 σ 22 σ 23  −  0 σ m 0 
σ13 σ 23 σ 33   0 0 σ m 
 13 (2σ11 − σ 22 − σ 33 ) σ12 σ13 
 
= σ12 1
3
(2σ 22 − σ11 − σ 33 ) σ 23 
 σ13 σ 23 1
(2σ 33 − σ11 − σ 22 )
 3

Luego,
2 − 2 1 0  0 1 0 
σ ijdev 
= 1 2−2 0  = 1 0 0
 0 0 2 − 2 0 0 0

Universidad de Castilla- La Mancha Draft Por: Eduardo W. V. Chaves (2012)


Ciudad Real - España
3 TENSIONES 263

Los tensores σ y σ dev son coaxiales (ver capítulo 1 del libro de texto), i.e., presentan las
mismas direcciones principales. Luego, podemos obtener los autovalores de σ dev
fácilmente si operamos en el espacio principal de σ :
σ1 0 0  σ m 0 0  0 0 0
σ′ijdev =  0 σ2 0 − 0
  σm 0  = 0 − 1 0
 0 0 σ 3   0 0 σ m  0 0 1

Los invariantes de σ dev vienen dados por:


I σ dev = Tr (σ dev ) = 0 ; II σ dev = −1 ; III σ dev = 0

Tradicionalmente, los invariantes del tensor de tensiones desviador viene denotados por:
J1 = I σ dev = 0

J 2 = − II
σ dev
1 2
3
= (
I σ − 3 II σ )
J 3 = III
σ dev
=
1
27
(
2 I σ3 − 9 I σ II σ + 27 III σ )

x3
σ 33

σ 23
σ 13 σ 23
σ 13
σ 12 σ 22
σ 12
σ 11 x2

x
14414444442444444443
x3 x3
σm dev
σ 33

σ 23
σ 13 σ 23
σ 13
σm
+ σ 12
σ 12
σ dev
22

σm x2 dev
σ 11 x2

x1 x1

σ esf σ dev

Figura 3.7: Parte esférica y desviadora de σ .

Universidad de Castilla- La Mancha Draft Por: Eduardo W. V. Chaves (2012)


Ciudad Real - España
264 PROBLEMAS RESUELTOS DE MECÁNICA DEL MEDIO CONTINUO

f) Las tensiones normal y tangencial octaédricas vienen dadas por:

σ oct
N =
1
(σ1 + σ 2 + σ 3 ) = 1 σ ii = I σ = σ m
3 3 3

σ oct ≡ τ oct =
1
2 I σ2 − 6 II σ =
2
J2 =
(σ ) + (σ ) + (σ )
dev 2
1
dev 2
2
dev 2
3
S
3 3 3

Reemplazando los valores del problema propuesto obtenemos que:


2 2
σ oct
N = σm = 6 ; τ oct = J2 =
3 3

Ejemplo 3.4
Las componentes del tensor de tensiones en un punto P son:
1 2 3
σ ij =  2 4 6 MPa (3.2)
 3 6 1

Encontrar:
r
a) El vector tracción t en P para un plano normal al eje x1 ;
r
b) El vector tracción t en P para un plano cuyo vector normal es (1,−1,2) ;
r
c) El vector tracción t en P para un plano paralelo al plano 2 x1 − 2 x 2 − x3 = 0 ;
d) Las tensiones principales en P ;
e) Las direcciones principales de σ en P .

Solución:
a) El vector normal al plano es (1,0,0) . Luego el vector tracción viene dado por:

1 2 3 1 1 
=  2 4 6 0 =  2
ˆ
t i(n) (3.3)
 3 6 1 0  3

b) El vector unitario (versor) asociado a la dirección (1,−1,2) es:

1
1 − 1
nˆ i = (3.4)
6 
 2 

luego,
1 2 3  1  5
ˆ 1     1  
t i(n) = 2 4 6  − 1 = 10 (3.5)
6 6  
3 6 1  2   − 1

Universidad de Castilla- La Mancha Draft Por: Eduardo W. V. Chaves (2012)


Ciudad Real - España
3 TENSIONES 265

c)
 2  1 2 3  2  −5
1 1 1
     
nˆ i =  − 2 ⇒ t i =  2 4 6  − 2 =  − 10
ˆ)
(n
(3.6)
3 3 3
 − 1   3 6 1  − 1   − 7 

d) Resolviendo el determinante característico


1− σ 2 3
2 4−σ 6 =0 (3.7)
3 6 1− σ

obtenemos que:
σ1 = 10 ; σ 2 = 0 ; σ 3 = −4 (3.8)
e) Las tensiones principales correspondientes son:
Para σ1 = 10

− 9n1 + 2n 2 + 3n 3 = 0 3 
 (1)  
 2n1 − 6n 2 + 6n 3 = 0 ⇒ n i = 6 (3.9)
 3n + 6n − 9n = 0 5
 1 2 3

Análogamente:
 − 2 1
n (i 2 ) =  1  ; n i =  2 
  ( 3)
(3.10)
 0  − 3

Normalización de las direcciones principales:


 3  − 2 1
n i(1) 1   n i( 2 ) 1   n (i 3) 1  
nˆ (i1) = r = 6 ; nˆ i( 2 ) = r = 1 ; nˆ i(3) = r = 2
n (1) 70   n ( 2) 5  n ( 3) 14  
5  0   − 3

Ejemplo 3.5
r r r
Probar que σ S = t (n) ⋅ (1 − nˆ ⊗ nˆ ) , donde t (n) es el vector tracción asociado al plano cuya
ˆ ˆ

r
normal es n̂ y σ S es la tensión tangencial asociada a este plano.

Solución 1:
r r ˆ r ˆ
[
σ S = t (n) − t (n ) ⋅ nˆ nˆ ]
r r ˆ r ˆ
σ S = t (n) − t (n) ⋅ nˆ ⊗ nˆ
r r ˆ
σ S = t (n) ⋅ (1 − nˆ ⊗ nˆ )

Solución 2:
Podemos resolver el problema anterior utilizando sólo las componentes de la ecuación
r ˆ
σ S = t (n) − [σ : (nˆ ⊗ nˆ )]nˆ :
r

Universidad de Castilla- La Mancha Draft Por: Eduardo W. V. Chaves (2012)


Ciudad Real - España
266 PROBLEMAS RESUELTOS DE MECÁNICA DEL MEDIO CONTINUO

σSi
ˆ
[ ]
= t i(n) − (nˆ k nˆ l σ kl ) nˆ i
ˆ ˆ
= t i(n) − nˆ i nˆ k t (kn)
ˆ
= t (kn) δ ik − nˆ i nˆ k t (kn)
= t (kn) (δ ik − nˆ i nˆ k )
ˆ

o en forma compacta:
r r ˆ
σ S = t (n) ⋅ (1 − nˆ ⊗ nˆ )

Ejemplo 3.6
El estado de tensión en un punto P del medio continuo se da esquemáticamente por:

x3

1
4
1
4 σ 22

1 1
x2

x1
Se pide:
Determinar el valor de la componente σ 22 del tensor de tensiones para que exista al menos
un plano que pase por P que esté libre de tensiones;
Determinar la dirección de dicho plano.
Solución:
r r
Buscamos un plano cuya dirección es n̂ tal que t (nˆ ) = 0 . Podemos relacionar el tensor de
tensiones con el vector tensión según expresión:
r ˆ
t (n) = σ ⋅ nˆ
luego:
 t1(nˆ )   0 1 4  n1  0 
 (nˆ )    
 t 2  =  1 σ 22 1  n 2  = 0 
 t (nˆ )   4 1 0  n3  0 
 3  
Resultando en el siguiente sistema de ecuaciones:
 1
n2 + 4n3 = 0 ⇒ n3 = − 4 n 2

n1 + σ 22n 2 + n3 = 0
 1
4n1 + n2 = 0 ⇒ n1 = − n 2
 4
Combinando las ecuaciones anteriores obtenemos que:

Universidad de Castilla- La Mancha Draft Por: Eduardo W. V. Chaves (2012)


Ciudad Real - España
3 TENSIONES 267

1 1
n1 + σ 22n 2 + n3 = 0 ⇒ − n2 + σ 22n 2 − n2 = 0
4 4
 1 1
 − + σ 22 − n 2 = 0
 4 4
r r
Luego, para n ≠ 0 , tenemos que:  − + σ 22 −  = 0 ⇒ σ 22 = .
1 1 1
 4 4 2
Para determinar la dirección del plano partimos de la restricción: n i n i = 1 , luego:
nini = 1 ∴ n12 + n 22 + n32 = 1
2 2
 1   1 
⇒  − n 2  + n 22 +  − n 2  = 1
 4   4 
2 2 2
⇒ n2 = ; n1 = n3 = −
3 6
r r
Obteniendo así la dirección de la normal al plano, cuando se cumple t (nˆ ) = 0 :
 − 1
2 4
nˆ i =
6  
 − 1

3.1.2 Ecuación de Equilibro, Tensiones y Direcciones


Principales

Ejemplo 3.7
El campo de tensión de un medio continuo viene representado por:
 1 0 2 x2 
σ ij =  0 1 4 x1  unidades de tensión (3.11)
 2 x2 4 x1 1 

donde xi son las coordenadas cartesianas.


Se pide:
a) Despreciando las fuerzas másicas, ¿está el cuerpo en equilibrio?
b) Determinar el vector tensión que actúa en un punto ( x1 = 1, x2 = 2, x3 = 3) según el
plano x1 + x 2 + x3 = 6 ;
c) Determinar la proyección del vector tensión según la dirección normal y tangencial
al plano x1 + x 2 + x3 = 6 ;
Solución:
Ecuación de equilibrio:
r r
∇ xr ⋅ σ + ρ
{=0
b
r (3.12)
=0

Universidad de Castilla- La Mancha Draft Por: Eduardo W. V. Chaves (2012)


Ciudad Real - España
268 PROBLEMAS RESUELTOS DE MECÁNICA DEL MEDIO CONTINUO

σ ij , j = 0 i (3.13)
expandiendo,
σ i1,1 + σ i 2, 2 + σ i 3,3 = 0 i (3.14)
σ11,1 + σ12, 2 + σ13,3 = 0

σ 21,1 + σ 22, 2 + σ 23,3 = 0 (3.15)

σ 31,1 + σ 32, 2 + σ 33,3 = 0
ya que:
∂σ11 ∂σ12 ∂σ13
=0 =0 =0
∂x1 ∂x2 ∂x3
∂σ 21 ∂σ 22 ∂σ 23
=0 =0 =0 (3.16)
∂x 2 ∂x2 ∂x3
∂σ 31 ∂σ 32 ∂σ 33
=0 =0 =0
∂x3 ∂x2 ∂x3

b) El versor normal al plano x1 + x 2 + x3 = 6 es:

1
1 1
nˆ i = (3.17)
3
1
r ˆ
El vector tensión t (n) :
r ˆ
t (n) = σ ⋅ nˆ (3.18)
1 0 4 
σ ij ( x1 = 1, x 2 = 2, x3 = 3) = 0 1 4 (3.19)
 4 4 1 

luego,
1 0 4  1 5
ˆ   1  1  
t i(n) = 0 1 4 1 = 5 (3.20)
3 3 
 4 4 1  1 9

c) Componente normal
1
r (nˆ ) 1
σN = t ⋅n = ˆ [5 5 9] 1 = (5 + 5 + 9) = 19
1  1
(3.21)
3 3 3 3
1

Componente tangencial
r ˆ r ˆ
σ 2S = −σ 2N + t (n) ⋅ t (n) (3.22)
5
r (nˆ ) r (nˆ ) 1
t ⋅t = [5 5 9] 5 1 = 131 (3.23)
3 3 3
9

Universidad de Castilla- La Mancha Draft Por: Eduardo W. V. Chaves (2012)


Ciudad Real - España
3 TENSIONES 269

luego
2
 19  131 32
σ S2 = −  + = (3.24)
 3 3 9

Ejemplo 3.8
Dado un cuerpo en equilibrio estático, donde el campo del tensor de tensiones de Cauchy
viene representado a través de las siguientes componentes cartesianas:
σ11 = 6 x13 + x 22 ; σ12 = x 32
σ 22 = 12 x13 + 60 ; σ 23 = x 2
σ 33 = 18 x 23 + 6 x33 ; σ 31 = x12
Determinar el vector de fuerzas másicas (por unidad de volumen) en el punto
( x1 = 2; x 2 = 4; x3 = 2 ).
Solución:
Ecuación de equilibrio:
r r
∇ xr ⋅ σ + ρb = 0 (3.25)
 ∂σ11 ∂σ12 ∂σ13 ∂σ ∂σ ∂σ
 + + + ρb1 = 0 ⇒ ρb1 = − 11 − 12 − 13
 ∂x1 ∂x 2 ∂x3 ∂x1 ∂x 2 ∂x3
 ∂σ 21 ∂σ 22 ∂σ 23 ∂σ 21 ∂σ 22 ∂σ 23
 + + + ρb 2 = 0 ⇒ ρb 2 = − − − (3.26)
 ∂x1 ∂x 2 ∂x3 ∂x1 ∂x 2 ∂x3
∂σ
 31 ∂σ ∂σ ∂σ ∂σ ∂σ 33
 ∂x + ∂x + ∂x + ρb 3 = 0 ⇒ ρb 3 = − ∂x − ∂x − ∂x
32 33 31 32

 1 2 3 1 2 3

ρb1 = −18 x12 − 0 − 0  − 18 x12 


  
ρb 2 = −0 − 0 − 0 ⇒ ρb i =  0  (3.27)
ρb = −2 x − 1 − 18 x 2  − 2 x − 1 − 18 x 2 
 3 1 2  1 2

Para el punto x1 = 2; x 2 = 4; x3 = 2 obtenemos que:

 − 72
ρb i =  0  (Fuerza por unidad de volumen) (3.28)
− 77

Ejemplo 3.9
El campo del tensor de tensiones de Cauchy viene representado por sus componentes
como:
 x12 x 2 (a 2 − x 22 ) x1 0 
 1 3 
σ ij = k (a 2 − x 22 ) x1 ( x 2 − 3a 2 x 2 ) 0  (3.29)
 3 
 0 0 2ax32 

donde k y a son constantes.

Universidad de Castilla- La Mancha Draft Por: Eduardo W. V. Chaves (2012)


Ciudad Real - España
270 PROBLEMAS RESUELTOS DE MECÁNICA DEL MEDIO CONTINUO

r
Encontrar el campo de fuerzas másicas b (por unidad de masa) necesario para que el
campo de tensión esté en equilibrio.
Solución:
 ∂σ11 ∂σ12 ∂σ13
 + + + ρb1 = 0 ⇒ ρb1 = −2 x1 x 2 k + 2 x1 x 2 k = 0
 ∂x1 ∂x 2 ∂x 3
 ∂σ 21 ∂σ 22 ∂σ 23 k
 + + + ρb 2 = 0 ⇒ ρb 2 = − k (a 2 − x 22 ) − (3 x 22 − 3a 2 ) = 0 (3.30)
 ∂x1 ∂x 2 ∂x 3 3
 ∂σ 31 ∂σ 32 ∂σ 33
 ∂x + ∂x + ∂x + ρb 3 = 0 ⇒ ρb 3 = −4kax3
 1 2 3

Luego:
0
4kax3  
bi = 0 (Fuerza por unidad de masa) (3.31)
ρ  
 − 1

Ejemplo 3.10
r
Suponga que las fuerzas másicas son b = − gê 3 , donde g es una constante. Considere el
siguiente tensor de tensiones:
 x2 − x3 0 
σ ij = α  − x3 0 − x 2  (3.32)
 0 − x2 p 

Encontrar p tal que cumpla con las ecuaciones de equilibrio. Considerar α una constante
y el campo homogéneo de densidad de masa, es decir, no depende del vector posición.
Solución:
Ecuación de equilibrio:
r r
∇ xr ⋅ σ + ρb = 0 (3.33)
 ∂σ11 ∂σ12 ∂σ13 
 + + + ρb 1 = 0
∂x ∂x ∂x 0 + 0 + 0 + ρb = 0 ⇒ b = 0
 1 2 3
 1 1
 ∂σ 21 ∂σ 22 ∂σ 23
 + + + ρb 2 = 0 ⇒ 0 + 0 + 0 + ρb 2 = 0 ⇒ b 2 = 0 (3.34)
 ∂x1 ∂x 2 ∂x 3  ∂σ
 ∂σ 31 ∂σ 32 ∂σ 33 0 − α + 33 + ρb 3 = 0
 ∂x + ∂x + ∂x + ρb 3 = 0  ∂x3
 1 2 3

∂σ 33 ∂(αp ) ∂p ∂p ρg
= =α = α − ρb 3 ⇒ =1 +
∂x3 ∂x3 ∂x 3 ∂x3 α
(3.35)
 ρg 
⇒ dp = 1 +  dx
 α 3
 ρg   ρg 

p = 1 +
 α
∂x 3 ⇒ p = 1 +

x
α 3

Universidad de Castilla- La Mancha Draft Por: Eduardo W. V. Chaves (2012)


Ciudad Real - España
3 TENSIONES 271

Verificación:
 ρg 
− α + α 1 +  − ρg = −α + α + ρg − ρg = 0 (3.36)
 α

Ejemplo 3.11
Muestre que para el siguiente campo de tensión:
σ11 = x 22 + ν ( x12 − x 22 ) ; σ12 = −2νx1 x 2 ; σ 23 = σ13 = 0
σ 22 = x1 + ν ( x 2 − x1 ) ; σ 33 = ν ( x1 + x 2 )
2 2 2 2 2

Satisface las ecuaciones de equilibrio con fuerzas másicas iguales a cero.


Solución:
Ecuaciones de equilibrio:
σ ij , j + ρb i = 0 i
{
=0i

σ ij , j = 0 i (i , j = 1,2,3)
σ i1,1 + σ i 2, 2 + σ i 3, 3 = 0 i

 ∂σ11 ∂σ12 ∂σ13


 + + =0
 ∂ x ∂ x ∂x
i = 1 σ 11,1 + σ 12 , 2 + σ 13,3 = 0 1 2 3
  ∂σ ∂σ 22 ∂σ 23
i = 2 σ 21,1 + σ 22 , 2 + σ 23, 3 = 0 ⇒  21 + + =0
i = 3 σ 31,1 + σ 32 , 2 + σ 33,3 = 0  ∂x1 ∂x 2 ∂x 3
  ∂σ 31 ∂σ 32 ∂σ 33
 + + =0
 ∂x1 ∂x 2 ∂x 3

Las ecuaciones de equilibrio quedan:


σ 11,1 + σ 12 , 2 + σ 31,3 = 2 x1ν − 2νx1 = 0

⇒ σ 12,1 + σ 22 , 2 + σ 23,3 = −2 x 2 ν + 2νx 2 = 0

σ 13,1 + σ 23, 2 + σ 33,3 = 0
Con lo cual se comprueba que el cuerpo está en equilibrio.

Ejemplo 3.12
Considérese el siguiente campo de tensiones:
 x1 + x 2 σ12 0
r 
σ ij ( x ) =  σ12 x1 − 2 x 2 0 
 0 0 x 2 

Considerando el medio en equilibrio, encontrar σ12 , sabiendo que es función de x1 , x 2 , i.e.


σ12 ( x1 , x 2 ) . Se sabe también que el medio está libre de fuerzas másicas y que el vector
r ˆ
tensión en el plano x1 = 1 viene dado por: t (n) = (1 + x 2 )eˆ 1 + (5 − x 2 )eˆ 2 .

Universidad de Castilla- La Mancha Draft Por: Eduardo W. V. Chaves (2012)


Ciudad Real - España
272 PROBLEMAS RESUELTOS DE MECÁNICA DEL MEDIO CONTINUO

Solución:
Como el cuerpo está en equilibrio debe satisfacer las ecuaciones de equilibrio:
σ ij , j + ρb i = 0 i
{
=0i

σ ij , j = 0 i (i , j = 1,2,3)
σ i1,1 + σ i 2, 2 + σ i 3, 3 = 0 i

Resultando:
 ∂σ11 ∂σ 12 ∂σ13 ∂σ
 + + = 1 + 12 + 0 = 0
 ∂x1 ∂x 2 ∂x 3 ∂x 2
 ∂σ ∂σ 22 ∂σ 23 ∂σ 12
⇒  21 + + = −2+0=0
 ∂x1 ∂x 2 ∂x 3 ∂x1
 ∂σ 31 ∂σ 32 ∂σ 33
 + + =0+0+0=0
 ∂x1 ∂x 2 ∂x 3
r ˆ
Como dato del problema tenemos que cuando x1 = 1 , t (n) = (1 + x 2 )eˆ 1 + (5 − x 2 )eˆ 2 , luego:
1 + x 2 σ12 0

σ ij ( x1 = 1, x 2 ) =  σ12 1 − 2 x 2 0 
 0 0 x 2 

1 + x 2 σ12 0  1 1 + x 2 
t (nˆ ) ˆ 
= σ ij ( x1 = 1, x 2 )n j =  σ12 1 − 2 x 2 0  0 = 5 − x 2  (3.37)
 0 0 x 2  0  0 
ˆ
t (n) = σ ij ( x1 = 1, x 2 ) nˆ j
 1 + x2 σ12 ( x1 = 1, x 2 ) 0  1  1 + x2  1 + x 2 

⇒ σ12 ( x1 = 1, x 2 ) 1 − 2 x2 0  0 = σ12 ( x1 = 1, x 2 ) = 5 − x 2 
   
 0 0 x 2  0  0   0 

A través de las ecuaciones de equilibrio:


∂σ12
∂x1
=2 ⇒ ∫ ∂σ 12 ∫
= 2∂x1 ⇒ σ12 ( x1 , x 2 ) = 2 x1 + C ( x 2 )

A través de la condición de contorno dada por (3.37) podemos obtener la constante de


integración:
σ12 ( x1 = 1, x 2 ) = 5 − x 2 = 2 + C ( x 2 ) ⇒ C ( x2 ) = 3 − x2
Luego:
σ12 ( x1 , x 2 ) = 2 x1 − x 2 + 3

Ejemplo 3.13
Obtener las ecuaciones de equilibrio (en notación ingenieril), partiendo de un elemento
r
diferencial ( dx ), donde la variación de las tensiones de punto a punto (campo de tensiones)
es la que se muestra en la Figura 3.8.

Universidad de Castilla- La Mancha Draft Por: Eduardo W. V. Chaves (2012)


Ciudad Real - España
3 TENSIONES 273

z Cara oculta
σx
τ xy
∂σ z
σz + dz
Cara oculta ∂z
∂σ yz
σ yz + dz τ xz
∂z
∂τ xz
τ xz + dz
∂z ∂τ yz dz
τ yz + dy
τ xy bz ∂y
σy by ∂σ y
∂τ xz σy + dy
τ xz + dx ∂y
∂x bx ∂τ xy
τ xy + dy y
∂y
τ yz ∂τ xy
τ xy + dx dx
∂σ ∂x
σ x + x dx
∂x

τ xz

x τ yz

Cara oculta
σz

dy

Figura 3.8: Tensiones en un elemento diferencial.


Solución:
Para obtener las ecuaciones de equilibro partiremos de que la suma de las fuerzas que
actúan en el diferencial sea cero. Haciendo el equilibrio de fuerzas según dirección x :

∑F x =0

 ∂σ x   ∂τ xy 
ρb x dxdydz +  σ x + dx  dydz − σ x dydz +  τ xy + dy dxdz
 ∂x   ∂y 
 ∂τ 
− τ xy dxdz +  τ xz + xz dz  dxdy − τ xz dxdy = 0
 ∂z 
Simplificando la ecuación anterior resulta:
∂σ x ∂τ xy ∂τ
ρb x dxdydz + dxdydz + dxdydz + xz dxdydz = 0
∂x ∂y ∂z

∂σ x ∂τ xy ∂τ xz
ρb x + + + =0
∂x ∂y ∂z

Universidad de Castilla- La Mancha Draft Por: Eduardo W. V. Chaves (2012)


Ciudad Real - España
274 PROBLEMAS RESUELTOS DE MECÁNICA DEL MEDIO CONTINUO

Resultante de fuerzas según dirección y : ∑ Fy = 0


 ∂σ y   ∂τ yz 
ρb y dxdydz +  σ y + dy  dxdz − σ y dxdz +  τ yz + dz  dxdy
 ∂ y   ∂ z 
 ∂τ xy 
− τ yz dxdy +  τ xy + dx  dydz − τ xy dydz = 0
 ∂x 
Simplificando la ecuación anterior resulta:
∂τ xy ∂σ y ∂τ yz
ρb y + + + =0
∂x ∂y ∂x z

Resultante de fuerzas según dirección z : ∑ Fz = 0


 ∂σ z   ∂τ 
ρb z dxdydz +  σ z + dz  dxdy − σ z dxdy +  τ xz + xz dx  dzdy
 ∂z   ∂x 
 ∂τ yz 
− τ xz dzdy +  τ yz + dy  dxdz − τ yz dxdz = 0
 ∂y 
Simplificando la ecuación anterior resulta:
∂τ xz ∂τ yz ∂σ z
ρb z + + + =0
∂x ∂y ∂z
Luego, las ecuaciones de equilibrio son:
 ∂σ x ∂τ xy ∂τ xz
 + + + ρb x = 0
 ∂x ∂y ∂z
 ∂τ xy ∂σ y ∂τ yz
 + + + ρb y = 0
 ∂x ∂y ∂x z
 ∂τ ∂τ ∂σ z
 xz + yz + + ρb z = 0
 ∂x ∂y ∂z

Ejemplo 3.14
Dado un medio continuo donde se conoce el estado tensional en un punto y que viene
representado a través de las componentes del tensor de tensiones de Cauchy:
1 1 0 
σ ij = 1 1 0  Pa
0 0 2 

Se pide:
a) Encontrar las tensiones principales y las direcciones donde se producen.
Solución:
Para obtener las tensiones principales λ i = σ i y direcciones principales nˆ ( i ) debemos
resolver el siguiente sistema de ecuaciones:

Universidad de Castilla- La Mancha Draft Por: Eduardo W. V. Chaves (2012)


Ciudad Real - España
3 TENSIONES 275

1 − λ 1 0   n1   0 
 1 1− λ 0  n 2  = 0 
 (3.38)
 0 0 2 − λ  n 3  0 

La obtención de soluciones no triviales de nˆ ( i ) , es equivalente a resolver:

σ ij − λδ ij = 0

Pero si nos fijamos en el formado de la matriz que contiene las componentes del tensor de
tensiones, podemos notar que ya tenemos una solución principal, ya que las componentes
tangenciales en la dirección 3 son cero, luego:
dirección
λ 1 = 2  → n1(1) = n (21) = 0 , n3(1) = ±1
Pero obtener las otras dos direcciones es suficiente con resolver:
1− λ 1
= −λ (2 − λ ) = 0
1 1− λ
Podemos fácilmente verificar que las raíces de la ecuación anterior son:
λ2 = 2 , λ3 = 0
Expresamos las componentes del tensor de tensiones en este nuevo sistema como:
2 0 0
σ ′ij = 0 2 0  Pa
0 0 0 

b) Direcciones principales
b.1) Para obtener la dirección principal asociada a la solución λ 2 = 2 , sustituimos esta
solución en la ecuación (3.38):
1 − 2 1 0  n1( 2 )  0 
 1  
 1− 2 0  n (22 )  = 0 
 0 0 2 − 2  n (32 )  0 
− n1( 2 ) + n (22 ) = 0
⇒  ( 2)
n1 − n (22 ) = 0
2 2
Resolviendo el sistema obtenemos n 3( 2) = 0 , n1( 2) = n (22) y utilizando n1( 2 ) + n (22 ) = 1 resulta:
1  1 1 
n1( 2 ) = n (22 ) = . nˆ ( 2 ) =  0
2  2 2 
b.2) Para la solución λ 3 = 0 , obtenemos que:

1 − 0 1 0  n1( 3)  0  n1( 3) + n (23) = 0


 1  n ( 3)  = 0  
 1 − 0 0  2    ⇒ n1( 3) + n (23) = 0
 0 0 2 − 0  n 3( 3)  0   (3)
2n 3 = 0
2 2
Resolviendo el sistema obtenemos n (33) = 0 , n1(3) = −n (23) y utilizando n1( 3) + n (23) = 1 ,
1 1  1 1 
resulta: n1(3) = , n (23) = − . nˆ (3) =  − 0 .
2 2  2 2 

Universidad de Castilla- La Mancha Draft Por: Eduardo W. V. Chaves (2012)


Ciudad Real - España
276 PROBLEMAS RESUELTOS DE MECÁNICA DEL MEDIO CONTINUO

Como hemos visto, los autovectores constituyen una matriz de transformación, A , entre
los dos sistemas, es decir, σ ′ = A σ A T , Así:
T
 0 0 1  0 0 1
   
σ 1 = 2 0 0    1 1 0   
 0 1 1 1 1
 σ2 = 2 0  =  0 1 1 0   0
 2 2   2 2 
 0 0 σ 3 = 0   1 1  0 0 2  1 1 
 − 0  − 0
 2 2   2 2 

Ejemplo 3.15
Una presa prismática está sometida a una presión ejercida por el agua. La presa tiene
espesor b y altura h , ver Figura 3.9. Obtener las restricciones de las componentes
cartesianas del tensor de tensiones de Cauchy en las caras BC , OB y AC .
x2
ρ a - densidad de masa del agua
g - aceleración de la gravedad

B C

ρa
ρ a g (h − x 2 )
h

O b A x1

Figura 3.9.

Solución:
La cara BC tiene como normal nˆ i( BC ) = [0 1 0] . Teniendo en cuenta que en esta cara no
hay vector tracción, concluimos que:
 σ11 σ12 σ13  0  σ12  0
t i( BC ) = 0 i = σ ij nˆ j ⇒ σ σ 22 σ 23  1 = σ 22  = 0
 21
σ 31 σ 32 σ 33  0 σ 32  0

Lo que es lo mismo que σ i 2 = 0 y debido a la simetría σ 2i = 0 .


La cara OB tiene como normal nˆ i( BC ) = [− 1 0 0] . Teniendo en cuenta que en esta cara el
vector tracción tiene como componentes t i(OB ) = [ρ a g (h − x 2 ) 0 0] , concluimos que:
ρ a g ( h − x 2 )   σ11 σ12 σ13  − 1  − σ11  ρ a g (h − x 2 )
t i(OB ) =  0  = σ nˆ
 ij j ⇒ σ
 21 σ 22 σ 23   0  = − σ 21  =  0 

 0 
 σ 31 σ 32 σ 33   0   − σ 31   0 

Lo que es lo mismo que σ i1 = ρ a g (h − x 2 )δ i1 .

Universidad de Castilla- La Mancha Draft Por: Eduardo W. V. Chaves (2012)


Ciudad Real - España
3 TENSIONES 277

La cara AC tiene como normal nˆ i( BC ) = [1 0 0] . Teniendo en cuenta que en esta cara no


hay vector tracción, concluimos que:
 σ11 σ12 σ13  1  σ11  0
t i( AC ) = 0 i = σ ij nˆ j ⇒ σ σ 22 σ 23  0 = σ 21  = 0
 21
σ 31 σ 32 σ 33  0 σ 31  0

Lo que es lo mismo que σ i1 = 0 y debido a la simetría σ1i = 0 .

3.1.3 Otras Medidas de Tensión

Ejemplo 3.16
Demostrar que se cumplen las siguientes relaciones:
P = J σ dev ⋅ F −T + Jσ m F −T ; S = JF −1 ⋅ σ dev ⋅ F −T + Jσ m C −1
donde P y S son el primer y segundo tensor de tensiones de Piola-Kirchhoff,
respectivamente, C es el tensor derecho de deformación de Cauchy-Green, F es el
gradiente de deformación, J es el determinante del Jacobiano, y σ m es la tensión media
del tensor de tensiones de Cauchy. Demostrar también que se cumplen las siguientes
relaciones:
P : F = S : C = 3Jσ m
Solución:
Teniendo en cuenta que P = J σ ⋅ F −T , y la descomposición de σ como σ = σ esf + σ dev ,
podemos obtener que:
P = J (σ dev + σ m 1) ⋅ F −T
= J σ dev ⋅ F −T + Jσ m 1 ⋅ F −T
= J σ dev ⋅ F −T + Jσ m F −T
Consideremos ahora la definición del segundo tensor de tensiones de Piola-Kirchhoff
S = JF −1 ⋅ σ ⋅ F −T , y teniendo en cuenta la descomposición aditiva de σ como
σ = σ esf + σ dev , obtenemos:
S = J F −1 ⋅ σ ⋅ F − T
= JF −1 ⋅ (σ dev + σ m 1) ⋅ F −T
= JF −1 ⋅ σ dev ⋅ F −T + JF −1 ⋅ σ m 1 ⋅ F −T
= JF −1 ⋅ σ dev ⋅ F −T + Jσ m C −1
Aplicando en doble producto escalar entre los tensores S y C , obtenemos que:
S : C = JF −1 ⋅ σ dev ⋅ F −T : C + Jσ m C −1 : C

donde el término JF −1 ⋅ σ dev ⋅ F −T : C queda:

Universidad de Castilla- La Mancha Draft Por: Eduardo W. V. Chaves (2012)


Ciudad Real - España
278 PROBLEMAS RESUELTOS DE MECÁNICA DEL MEDIO CONTINUO

JF −1 ⋅ σ dev ⋅ F −T : C = ( JF −1 ⋅ σ dev ⋅ F −T ) : {
C
F T ⋅F

( JF −1 ⋅ σ dev ⋅ F −T ) ij ( F T ⋅ F ) ij = J Fip−1σ dev −1


pk F jk Fqi Fqj

= J δ qp δ qk σ dev
pk

pk δ pk
= J σ dev
dev
=J σ
1
424 :1
3 =0
Tr (σ dev ) =0

Luego:
S : C = Jσ m C −1 : C = Jσ m Tr (C −1 ⋅ C ) = Jσ m Tr (1) = 3 Jσ m
Ahora, haciendo el doble producto escalar entre P y F , obtenemos que:
P : F = J σ dev ⋅ F −T : F + Jσ m F −T : F

Analizando el término J σ dev ⋅ F −T : F , concluimos que:


J σ dev ⋅ F −T : F = ( J σ dev ⋅ F −T ) ij ( F ) ij

= J σ ikdev F jk−1 Fij = Jσ ikdev δ ik


dev
=J σ
1
424 :1 = 0
3
Tr (σ dev )=0

Luego,
P : F = Jσ m F −T : F = Jσ m Tr ( F −T ⋅ F T ) = Jσ m Tr (1) = 3 Jσ m

3.1.4 Máxima Tensión de Corte, Círculo de Mohr

Ejemplo 3.17
¿Cuál es la tensión de corte máxima cuyo estado tensional en un punto es el siguiente?
x2

30 MPa

20 MPa
x1

x3
Figura 3.10.

Universidad de Castilla- La Mancha Draft Por: Eduardo W. V. Chaves (2012)


Ciudad Real - España
3 TENSIONES 279

Solución:
τ
Como los ejes xi son ejes
principales, dibujamos el círculo τ max (MPa)
de Mohr con las tensiones
principales σ I = 30 MPa ,
σ II = 20MPa y σ III = 0 .
σ N (MPa)
20 30

Figura 3.11.

30 − 0
τ max = = 15 MPa (3.39)
2

Ejemplo 3.18
Dado el estado tensional en un punto representado por el elemento infinitesimal mostrado
en la Figura 3.12. Se pide:
x2
a) Dibujar el círculo de Mohr;
b) Obtener la tensión normal máxima,
20 MPa
e indicar el plano en la que se produce;
c) Obtener la tensión tangencial máxima.
5 MPa
x1

10 MPa

x3
Figura 3.12:

Solución:
σS ≡ τ
τmax = 15
σ N max = 10MPa
10 − (−20)
σ S max = = 15MPa
2

σ N (MPa)
− 20 5 10

Universidad de Castilla- La Mancha Draft Por: Eduardo W. V. Chaves (2012)


Ciudad Real - España
280 PROBLEMAS RESUELTOS DE MECÁNICA DEL MEDIO CONTINUO

Ejemplo 3.19
Determinar para que valores de σ * son posibles 6
los siguientes estados tensionales en planos que
pasen por P :

σ*
Caso a) σ N = 4 ; τ=2 P

Caso b) σ N = 4 ; τ =1 2
Caso c) σ N = 7 ; τ=0

Figura 3.13.
Solución
Para que los pares de valores (σ N ; τ) sean factibles, tienen que pertenecer a la zona en gris
en el círculo de Mohr, ver Figura 3.14, o pertenecer a las circunferencias.

σ3 σ2 σ1 σN

Figura 3.14: Círculo de Mohr.

Universidad de Castilla- La Mancha Draft Por: Eduardo W. V. Chaves (2012)


Ciudad Real - España
3 TENSIONES 281

Caso a)

2 Caso b)

1 Caso c)

2 6 7 σN

Figura 3.15: Círculo de Mohr.

Caso a) :En este caso el par (σ N = 4; τ = 2) pertenece al círculo formado por la tensiones
principales 2 y 6 , luego σ * puede ser cualquiera, ver Figura 3.16.

Caso a)

2
1

σ −∞
* 2 σ* 6 σ∞
*
σN

Figura 3.16: Círculo de Mohr.

Caso b) En este caso podemos decir que la solución es:

σ *( 2) ≤ σ * ≤ σ *(1) (3.40)

donde σ *( 2) , σ *(1) están señalados en la Figura 3.17.

Universidad de Castilla- La Mancha Draft Por: Eduardo W. V. Chaves (2012)


Ciudad Real - España
282 PROBLEMAS RESUELTOS DE MECÁNICA DEL MEDIO CONTINUO

τ τ Casos límites

2 2
( 4,1) ( 4,1)
1 1

2 σ* ( x ) 6 σN 2 σ * ( 2) σ * (1) 6 σN

Figura 3.17: Círculo de Mohr.

Partiendo de la ecuación de la circunferencia:


( x − xC ) 2 + ( y − y C ) 2 = R 2 (3.41)

(σ *(1) + 2) (σ *(1) − 2)
Para el caso σ *(1) , tenemos: x = 4; x C = ; y = 1; y C = 0; R =
2 2
Reemplazando estos valores en la ecuación de la circunferencia, resulta:
( x − xC ) 2 + ( y − y C ) 2 = R 2
2 2
 ( σ * + 2)   ( σ * − 2) 
 4 − (1)  + (1 − 0)2 =  (1)  (3.42)
 2   2 
   
⇒ σ *(1) = 4,5

(6 + σ *( 2 ) ) (6 − σ *( 2 ) )
Para el caso σ *( 2 ) , tenemos: x = 4; xC = ; y = 1; y C = 0; R =
2 2
reemplazando estos valores en la ecuación de la circunferencia, resulta:
( x − xC ) 2 + ( y − y C ) 2 = R 2
2 2
 (6 + σ *( 2) )   (6 − σ *( 2 ) ) 
4 −  + (1 − 0)2 =   (3.43)
 2   2 
   
⇒ σ *( 2 ) = 3,5

luego:

3,5 ≤ σ * ≤ 4,5 (3.44)


Caso c) En este caso la única solución posible es que σ N sea una tensión principal, luego

σ* = 7 (3.45)

Universidad de Castilla- La Mancha Draft Por: Eduardo W. V. Chaves (2012)


Ciudad Real - España
3 TENSIONES 283

2 6 σ* = 7 σN

Figura 3.18: Círculo de Mohr.

Ejemplo 3.20
Obtener la máxima tensión normal y tangencial (de corte) y dibujar el círculo de Mohr
correspondiente para los siguientes estados tensionales:
a)
 τ τ 0
σ ij =  τ τ 0 (3.46)
0 0 0

b)
− 2τ 0 0 
σ ij =  0 τ 0  (3.47)
 0 0 − τ

Solución:
a) Valores principales. Si verificamos el formato de las componentes del tensor de
tensiones de Cauchy, ya podemos decir que un valor principal es λ (3) = 0 . Luego, es
suficiente obtener solo los dos otros autovalores:
τ τ τ−λ τ
τ τ → τ = (τ − λ) 2 − τ 2 = 0 ⇒ τ − λ = τ ⇒ λ = 0 (3.48)
  τ−λ

 λ (1) = 0
(τ − λ ) 2 − τ 2 = 0 ⇒ τ 2 − 2λτ + λ2 − τ 2 = 0 ⇒ λ (−2τ + λ ) = 0 ⇒  (3.49)
λ ( 2) = 2τ

Universidad de Castilla- La Mancha Draft Por: Eduardo W. V. Chaves (2012)


Ciudad Real - España
284 PROBLEMAS RESUELTOS DE MECÁNICA DEL MEDIO CONTINUO

τ max = τ

2τ σN

Figura 3.19:

σ N max = 2τ
(3.50)
τ max = τ

b)

− 2τ −τ τ σN

Figura 3.20.

σ N max = τ
τ − (−2τ) 3 (3.51)
τ max = = τ
2 2

Universidad de Castilla- La Mancha Draft Por: Eduardo W. V. Chaves (2012)


Ciudad Real - España
3 TENSIONES 285

Ejemplo 3.21
Hacer la representación del círculo de Mohr para los siguientes casos:
1) Caso unidimensional, estado de carga de tracción
2) Caso unidimensional, estado de carga de compresión
3) Caso bidimensional, estado de carga de tracción
4) Caso triaxial
5) estado de corte puro
Solución:
1) Caso unidimensional, estado de carga de tracción
τ
σx

σ I 0 0 σI σN
σx 0 0 0

 0 0 0

2) Compresión uniaxial
τ
σx

0 0 0
0 − σ 0
 II σ II σN
σx 0 0 0

3) Caso biaxial
σ II
τ

σI

σ II σI σN

σ I 0 0
0 σ II 0

 0 0 0

Universidad de Castilla- La Mancha Draft Por: Eduardo W. V. Chaves (2012)


Ciudad Real - España
286 PROBLEMAS RESUELTOS DE MECÁNICA DEL MEDIO CONTINUO

4) Caso triaxial

σ III τ

σ I 0 0 
σII 0 σ II 0 

 0 0 σ III  σ III σII σI
σN
σI

5) Corte puro
σS ≡ τ

σ 0 0 
 0 − σ 0 σ
 
 0 0 0

−σ σ σN

−σ

3.1.5 Particularidades del Tensor de Tensiones

Ejemplo 3.22
Las componentes del tensor de tensiones de Cauchy en un punto P vienen dadas por:
5 6 7 
σ ij = 6 8 9 GPa (3.52)
7 9 2

Obtener:
a) La tensión media;
b) La parte volumétrica y desviadora del tensor σ .
Solución:
σ kk 5 + 8 + 2
σm = = =5 (3.53)
3 3

Universidad de Castilla- La Mancha Draft Por: Eduardo W. V. Chaves (2012)


Ciudad Real - España
3 TENSIONES 287

σ m 0 0  5 0 0 
σ ijesf =  0 σm 0  = 0 5 0
 0 0 σ m  0 0 5
σ ij = σ ijesf + σ ijdev ⇒ σ ijdev = σ ij − σ ijesf (3.54)
0 6 7 
σ ijdev = 6 3 9 
7 9 − 3

Ejemplo 3.23
Considere las componentes del tensor de tensiones:
5 3 2
σ ij =  3 1 0  unidades de (3.55)
 2 0 3 tensión

dadas en el sistema constituido por la base (eˆ 1 , eˆ 2 , eˆ 3 ) .


Dada la ley de transformación de base entre los sistemas x y x' por:

x'1 x' 2 x '3

3 4
x1 0
5 5

x2 0 1 0

4 3
x3 − 0
5 5

donde el sistema x' está constituido por la base (eˆ '1 , eˆ ' 2 , eˆ ' 3 ) .
Se pide:
r ˆ
a) Obtener el vector tensión t ( e'2 ) según el plano cuya normal es eˆ ' 2 , expresado según el
sistema cartesiano (eˆ '1 , eˆ ' 2 , eˆ ' 3 ) con el siguiente formato:
r ˆ
t (e'2 ) = ( )eˆ 1′ + ( )eˆ ′2 + ( )eˆ ′3 (3.56)
b) Obtener la parte esférica y desviadora del tensor de tensiones.

Solución:
a) Como definimos, la primera fila de la matriz de transformación está formada por los
cosenos directores del eje x'1 con x1 , x2 y x3 , luego:

3 0 − 4
1
A = 0 5 0  (3.57)
5
4 0 3 

y la ley de transformación para las componentes de tensor de segundo orden:

Universidad de Castilla- La Mancha Draft Por: Eduardo W. V. Chaves (2012)


Ciudad Real - España
288 PROBLEMAS RESUELTOS DE MECÁNICA DEL MEDIO CONTINUO

σ' = A σ A T (3.58)
luego:
 53 0 − 45  5 3 2  53 0 4
5

9 9 2 
    1
σ′ij = 0 1 0   3 1 0  0 1 0  = 9 5 12 (3.59)
5
 4 0 3   2 0 3 − 4 0 35  2 12 31
5 5   5
9
ˆ 1 
t i( e' 2 ) = 5
5
12 (3.60)
r ˆ 9  12 
t (e'2 ) =  eˆ 1′ + (1)eˆ ′2 +  eˆ ′3
5 5

ya que:

′   t 1 ( e'1 ) 
ˆ ( eˆ ' 2 ) ( eˆ '3 )
 σ11′ σ12′ σ13 t1 t1
σ ′  ( eˆ '3 ) 
σ ′23  = t 2 1
( eˆ ' ) ( eˆ ' 2 )
 21 σ ′22 t2 t2  (3.61)
σ ′33   t 3 1
( eˆ ' ) ( eˆ ' ) ( eˆ '3 ) 
σ ′31 σ ′32 t3 2 t3
 
b)
σ ij = σ ijesf + σ ijdev
Iσ (3.62)
= δ ij + σ ijdev
3
Iσ = 5 +1+ 3 = 9 (3.63)
 3 0 0
σ esf
ij = 0 3 0 (3.64)
0 0 3

σ ijdev = σ ij − σ ijesf
5 − 3 3 2  2 3 2
 (3.65)
=  3 1− 3 0  = 3 − 2 0
 2 0 3 − 3 2 0 0

Ejemplo 3.24
El estado de tensión en un medio continuo (cuerpo) está dado por el tensor de tensiones
de Cauchy:
 0 Cx 3 0 
σ ij = Cx 3 0 − Cx1 
 0 − Cx1 0 
donde C es una constante. Considérese que el cuerpo esté libre de fuerzas másicas. Se
pide:
a) Probar si el cuerpo está en equilibrio;

Universidad de Castilla- La Mancha Draft Por: Eduardo W. V. Chaves (2012)


Ciudad Real - España
3 TENSIONES 289

b) Calcular el vector tensión en el punto P (4,−4,7) según un plano cuya normal viene dada
2 2 1
por nˆ = eˆ 1 + eˆ 2 − eˆ 3 .
3 3 3
c) Representar los círculos de Mohr del estado de tensión del punto P .
Solución:
a) Para que el medio continuo esté en equilibrio hay que cumplir las ecuaciones de
equilibrio:
r r
∇ ⋅ σ + ρb = 0 ; σ ij,j + ρ b i = 0 i (3.66)
 0 Cx 3 0 
σ ij = Cx 3 0 − Cx1  (3.67)
 0 − Cx1 0 

Para el problema propuesto ρb i = 0 i , luego:

i = 1 ⇒ 0 + 0 + 0 = 0
∂σ ij ∂σ i1 ∂σ i 2 ∂σ i 3 
σ ij,j = = + + ⇒ i = 2 ⇒ 0 + 0 + 0 = 0 (3.68)
∂x j ∂x1 ∂x 2 ∂x3 i = 3 ⇒ 0 + 0 + 0 = 0

σ ij,j = 0 i luego el cuerpo está en equilibrio.

b) El vector tensión viene dado por:


r ˆ
t (n) = σ ⋅ nˆ ; t i = σ ij nˆ j
(nˆ )
(3.69)
 0 Cx3 0  0 7C 0 
σ ij ( x1 = 4; x 2 = −4; x3 = 7) = Cx3 0 − Cx1  = 7C 0 − 4C  (3.70)
 0 − Cx1 0   0 − 4C 0 

2
1
nˆ j =  2  (3.71)
3
− 1

Resultando que:
0 7C 0  2  14C 
r (nˆ ) 1  1
t i = σ ij n j = 7C
ˆ 0 
− 4C   2  =  18C  (3.72)
3 3
 0 − 4C 0   − 1 − 8C 

c)
0 7 0 
σ ij = C 7 0 − 4

(3.73)
0 − 4 0 

Los autovalores (tensiones principales) vienen dados por:

Universidad de Castilla- La Mancha Draft Por: Eduardo W. V. Chaves (2012)


Ciudad Real - España
290 PROBLEMAS RESUELTOS DE MECÁNICA DEL MEDIO CONTINUO

σ 
C 0 0
0 7 0  σ 0 0  0 7 0   
σ
C 7 0 − 4 −  0 σ 0  = 0
 ⇒ C 7 0 − 4 − C  0 0 =0 (3.74)
 C 
0 − 4 0   0 0 σ  0 − 4 0   σ
 0 0
C 

Considerando que σ = C σ , obtenemos:


− σ 7 0  − σ 7 0 
C  7 − σ − 4  = C 3
  7 − σ − 4 = 0
  (3.75)
 0 − 4 − σ   0 − 4 − σ 

− σ 3 + 16 σ + 49 σ = 0 ⇒ − σ 2 + 65 = 0 ⇒ σ = ± 65 (3.76)

Con eso, obtenemos que σ = ±C 65 .


Resultando así un estado de corte puro, el círculo de Mohr viene representado por:

σS ≡ τ

C 65

σ III = −C 65 σ I = C 65 σN

Ejemplo 3.25
El estado tensional en un punto del cuerpo viene dado por las componentes del tensor de
tensiones de Cauchy representado en el sistema cartesiano como:

x3

r ˆ
t ( e 3 ) = 8eˆ 1

r ˆ
t ( e 2 ) = 6eˆ 1
ê 3
ê1
ê 2 x2

r ˆ
t ( e1 ) = 6eˆ 2 + 8eˆ 3
x1

Figura 3.21:

Universidad de Castilla- La Mancha Draft Por: Eduardo W. V. Chaves (2012)


Ciudad Real - España
3 TENSIONES 291

Se pide:
a) La tensión desviadora;
b) Determinar las tensiones principales ( σ I , σ II , σ III ) y las direcciones principales;
c) Dibujar el círculo de Mohr;
d) Obtener la máxima tensión de corte;
e) Encontrar el vector tensión en un plano que pasa por el punto dado cuya normal a este
6 ˆ
plano es nˆ = 0,75eˆ 1 + 0,25eˆ 2 − e3 ;
4
f) Obtener también la tensión normal y tangencial en este plano.
Solución: Según la Figura 3.21 podemos obtener las componentes del tensor de tensiones de
Cauchy como:
0 6 8 
σ ij = 6 0 0
8 0 0

a)
σ ij = σ ijesf + σ ijdev


La parte esférica σ ijesf = δ ij = 0 ij ya que I σ = 0 . Luego, la parte desviadora viene dada por:
3
0 6 8 
σ ijdev = σ ijesf − σ ij = 6 0 0
8 0 0

b) Los autovalores pueden ser determinados por el determinante característico:


−λ 6 8
6 −λ 0 =0 ⇒ − λ3 + 100λ = 0 ⇒ ( )
λ − λ2 + 100 = 0
8 0 −λ

Las soluciones son λ 1 = 0 , λ 2 = 10 , λ 3 = −10 , que son las tensiones principales. Las direcciones
principales quedan:
 → nˆ (i1) = [0 − 0,8 0,6]
σ1 = 0 autovalor
 → nˆ i( 2 ) = [− 0,707 0,424 0,566]
σ 2 = −10 autovalor
 → nˆ i(3) = [0,707 0,424 0,566]
σ 3 = 10 autovalor
σ I = 10 , σ II = 0 , σ III = −10

Universidad de Castilla- La Mancha Draft Por: Eduardo W. V. Chaves (2012)


Ciudad Real - España
292 PROBLEMAS RESUELTOS DE MECÁNICA DEL MEDIO CONTINUO

c) El círculo de Mohr viene dibujado en la figura abajo:


σS ≡ τ

τ max = 10

σ III = −10 σ II = 0 σ I = 10 σN

d) En el círculo de Mohr se puede obtener directamente la tensión de corte máxima:


τ max = 10
ˆ
e) Teniendo en cuenta que t i (n) = σ ij nˆ j , podemos obtener las componentes del vector tensión
6 ˆ
en el plano de normal nˆ = 0,75eˆ 1 + 0,25eˆ 2 − e3 :
4
 
 t 1 (nˆ )  0 6 8  0,75   − 3,39898
 (nˆ )      
t 2  = 6 0 0  0,25  ≈  4,5 
 t (n)  8 0 0 
ˆ
 3    − 6   6 
 4 

f)
r ˆ
t (n)

r
σN
r
σS
ŝ n̂
P

r r r
El módulo de σ N se puede obtener a través de la proyección σ N = t (n) ⋅ nˆ = t i (n) nˆ i ,
ˆ ˆ

luego:
 
 0,75 
r  
= t i nˆ i ≈ [− 3,39898 4,5 6]  0,25  ≈ −5,09847
(nˆ )
σN
 6
− 4 
r
El vector σ N viene dado por:
r r
σ N = σ N nˆ = −3,82385eˆ 1 − 1,27462eˆ 2 + 3,12216eˆ 3

Universidad de Castilla- La Mancha Draft Por: Eduardo W. V. Chaves (2012)


Ciudad Real - España
3 TENSIONES 293

r ˆ r r
Además como se cumple que t (n) = σ N + σ S , podemos obtener el vector tangencial a este
plano como:
r r ˆ r
σ S = t (n) − σ N
≈ (− 3,39898 + 3,82385)eˆ 1 + (4,5 + 1,27462 )eˆ 2 + (6 − 3,12216 )eˆ 3
≈ (0,42487 )eˆ 1 + (5,77462 )eˆ 2 + (2,87784 )eˆ 3
y su módulo:
r
σS ≈ (0,42487 )2 + (5,77462)2 + (2,87784)2
≈ 41,808713 = 6,465966
r r ˆ r ˆ r
= t (n) ⋅ t (n) − σ N
2 2
OBS.: También podríamos haber utilizado la expresión σ S para
r
obtener el módulo de σ S .

Ejemplo 3.26
El campo del tensor de tensiones de Cauchy de un medio continuo viene representado por:
 3 x1 5 x 22 0 
r  
σ ij ( x ) = σ 21 3x 2 2 x3 
σ σ 32 0 
 31
a) Obtener las fuerzas másicas (por unidad de volumen) para que el medio continuo esté en
equilibrio.
b) Para un punto particular ( x1 = 1, x 2 = 1, x3 = 0 ). Se pide:
b.1) Dibujar el Círculo de Mohr en tensiones. Obtener la tensión tangencial y normal
máximas.
 1 1 1 
b.2) Obtener el vector tensión en el plano definido por la normal ni =  
 3 3 3
b.2.1) Obtener la componente normal y tangencial en este plano.
Solución:
a) Debido a la simetría del tensor de tensiones de Cauchy tenemos que:
 3 x1 5 x 22 0 
r  2 
σ ij ( x ) = 5 x 2 3x 2 2 x3 
 0 2 x3 0 

σ + σ12, 2 + σ13,3 = −ρb1 3 + 10 x 2 + 0 = −ρb1
r r componentes  11,1 
∇ xr ⋅ σ + ρb = 0   →σ 21,1 + σ 22, 2 + σ 23,3 = −ρb1 ⇒ 0 + 3 + 2 = −ρb 2
 0 + 0 + 0 = −ρb
σ 31,1 + σ 32, 2 + σ 33,3 = −ρb1  3

con lo cual obtenemos que:

Universidad de Castilla- La Mancha Draft Por: Eduardo W. V. Chaves (2012)


Ciudad Real - España
294 PROBLEMAS RESUELTOS DE MECÁNICA DEL MEDIO CONTINUO

− 10 x 2 − 3
ρb i =  − 5  (Fuerza por unidad de volumen) (3.77)
 0 

b) Para el punto en particular ( x1 = 1, x 2 = 1, x3 = 0 ) tenemos que:


3 5 0
σ ij = 5 3 0
0 0 0

donde podemos verificar que σ 3 = 0 es un valor principal. Para obtener los otros
autovectores es suficiente con resolver:
3−σ 5 σ1 = 8
=0 ⇒ (3 − σ) = (5) 2 ⇒ 3 − σ = ±5 ⇒ 
5 3−σ σ 2 = −2
Reestructurando los autovalores:
σ I = 8 , σ II = 0 , σ III = −2
b.1) El círculo de Mohr viene dibujado en la figura abajo:

σS ≡ τ
τ max = 5

σ III = −2 σ II = 0 σI = 8 σN

En el círculo de Mohr se puede obtener directamente la tensión de corte máxima τ max = 5 y la


tensión normal máxima σ N max = σ I = 10 .
ˆ
e) Teniendo en cuenta que t i (n) = σ ij nˆ j , podemos obtener las componentes del vector tensión
1 ˆ 1 ˆ 1 ˆ
en el plano de normal nˆ = e1 + e2 + e3 :
3 3 3

 t 1 (nˆ )  3 5 0 1 8 
 (nˆ )  1     1  
t 2  = 5 3 0 1 = 8
 t (nˆ )  3 3 
0 0 0 1 6
 3 
b.2) La tensión normal:
1
1 1
σN = ti
(nˆ )
nˆ i = [8 8 0] 1 = 16
3 3 3
1

Universidad de Castilla- La Mancha Draft Por: Eduardo W. V. Chaves (2012)


Ciudad Real - España
3 TENSIONES 295

r r ˆ r ˆ r
= t (n) ⋅ t (n) − σ N
2 2
Para la componente tangencial podemos aplicar directamente σ S ,
8
r ˆ r (nˆ ) r (nˆ ) 1 1
[8 8 0] 8 = 128 . Luego:
2
= t ⋅ t = ti ti =
(nˆ ) (nˆ )
donde t (n)
3 3 3
0
2
r r ˆ r ˆ r 128  16  128 128
= t (n) ⋅ t (n) − σ N
2 2
σS = −  = ⇒ σS =
3  3 9 3

Ejemplo 3.27
El estado tensional en un punto del cuerpo viene dado por las componentes del tensor de
tensiones de Cauchy según su parte esférica y desviadora, respectivamente:
1 0 0 0 6 8 
σ ijesf = 0 1 0 ; σ ijdev = 6 0 0
0 0 1 8 0 0

Se pide:
a) Obtener las componentes del tensor de tensiones de Cauchy;
b) Determinar las tensiones principales ( σ I , σ II , σ III ) y las direcciones principales.
c) Obtener la máxima tensión de corte;
d) Dibujar el círculo de Mohr para: d.1) el tensor de tensiones de Cauchy ( σ ij ), d.2) Parte
esférica ( σ ijesf ) y; d.3) parte desviadora ( σ ijdev );
Solución:
1 0 0 0 6 8  1 6 8 
a) σ ij = σ ijesf + σ ijdev = 0 1 0 + 6 0 0 = 6 1 0
0 0 1 8 0 0 8 0 1 

En el Ejemplo 3.25 hemos obtenido los valores principales del tensor σ ijdev que es el
mismo del problema propuesto. Como el tensor y su parte desviadora tienen las mismas
direcciones principales, podemos obtener de forma automática las tensiones principales:
1 0 0 10 0 0  9 0 0 
σ′ij = σ′ijesf + σ′ijdev   
= 0 1 0  +  0 0 0  = 0 1 0 
0 0 1  0 0 − 10 0 0 11

Las direcciones principales son las mismas del tensor σ del Ejemplo 3.25.

Universidad de Castilla- La Mancha Draft Por: Eduardo W. V. Chaves (2012)


Ciudad Real - España
296 PROBLEMAS RESUELTOS DE MECÁNICA DEL MEDIO CONTINUO

d) Círculo de Mohr
σS ≡ τ σS ≡ τ
τ max = 10

σ dev σ dev
II = 0 σ dev = 10 σ dev σN
III = −10 I N σ I = σ II = σ III = 1

Parte desviadora Parte esférica

14444444444444442444444444444444
3

σS ≡ τ
τ max = 10

σ III = −9 σ II = 1 σ I = 11 σN

Observemos que la parte esférica lo que hace es desplazar el círculo de Mohr según el eje
σ N , no alterando así el valor de la tensión tangencial máxima.

Ejemplo 3.28
En un punto P del medio continuo el tensor de tensiones de Cauchy σ viene
representado por sus componentes cartesianas por:
1 1 0 
σ ij = 1 1 0 MPa ,
0 0 2

Se pide:
a) Determinar las tensiones principales y las direcciones principales en el punto P ;
b) Obtener la máxima tensión de corte;
c) Dibujar el círculo de Mohr para: c.1) el tensor de tensiones de Cauchy ( σ ij ), c.2) Parte
esférica ( σ ijesf ) y; c.3) parte desviadora ( σ ijdev );
d) i.) Encontrar el vector tensión en un plano que pasa por el punto dado cuya
r
dirección normal a este plano es n = 1,0eˆ 1 + 1,0eˆ 2 + 0eˆ 3 ;
ii.) Obtener también la tensión normal y tangencial en este plano.

Universidad de Castilla- La Mancha Draft Por: Eduardo W. V. Chaves (2012)


Ciudad Real - España
3 TENSIONES 297

f) Obtener los autovalores y autovectores de la parte desviadora del tensor de tensiones de


Cauchy ( σ dev ).
Solución:
a) Ver Ejemplo 3.14. Los autovalores son σ I = 2 , σ II = 2 , σ III = 0
b) y c)
 2 0 0 1 0 0 1 0 0 
4 2
σ′ijdev = σ′ij − σ ′ijesf = 0 2 0 − 0 1 0 = 0 1 0 
  
3 3
0 0 0 0 0 1 0 0 − 2

σS ≡ τ σS ≡ τ

τ max = 1
+
σN

σ III = −1,333 σ I , σ II = 0,667 σN


σ I = σ II = σ III = 1,333

Parte desviadora Parte esférica

14444444444444442444444444444444
3

σS ≡ τ

τ max = 1

σ III = 0 σ I , σ II = 2 σN

r
d) El vector tensión se obtiene a partir de t (n) = σ ⋅ nˆ , normalizando el vector obtenemos
ˆ
r
ˆ n 1 ˆ 1 ˆ
que: n = r = e1 + e 2 + 0eˆ 3 . Vector tensión:
n 2 2

t 1(nˆ )  1 1 0 1   2
 (nˆ )    1   1  
t 2  = 1 1 0 1  = 2  2 
t (nˆ )  0 0 2 2 0 0
 3     

Universidad de Castilla- La Mancha Draft Por: Eduardo W. V. Chaves (2012)


Ciudad Real - España
298 PROBLEMAS RESUELTOS DE MECÁNICA DEL MEDIO CONTINUO

Ejemplo 3.29
Las componentes de un estado de tensión en un punto P son:
 29 0 0
σ ij =  0 − 26 6  Pa

 0 6 9 

Descompónganse las componentes del tensor de tensiones en una parte esférica y otra
desviadora, y determínense los valores de las tensiones principales del tensor desviador.
Solución:
Considerando la descomposición aditiva del tensor de tensiones en una parte esférica y
desviadora:
σ ij = σ ijdev + σ ijesf

La parte desviadora viene dada por


σ11 − σ m σ12 σ13 
σ ijdev =  σ12 σ 22 − σ m σ 23 

 σ13 σ 23 σ 33 − σ m 

siendo la tensión media dada por:


1 ( 29 − 26 + 9)
σm = σ ii = =4
3 3
Resultando así:
 29 − 4 0 0   25 0 0
σ ijdev 
= 0 − 26 − 4 
6  =  0 − 30 6  Pa

 0 6 9 − 4   0 6 5 

Las componentes del tensor hidrostático son:


4 0 0
σ ijhid ≡ σ esf
ij = 0 4 0  Pa
0 0 4

Para comprobar las operaciones anteriores, la siguiente relación tiene que verificarse:
 25 0 0  4 0 0  29 0 0
σ ij = σ ijdev + σ ijesf =  0 − 30 6 + 0 4 0  =  0 − 26 6  Pa
     ✓
 0 6 5 0 0 4  0 6 9 

Obteniendo la ecuación característica del tensor de tensiones desviador:


σ ijdev − λδ ij = 0 
→ λ3 − λJ 2 − J 3 = 0

Con la solución de la ecuación cúbica anterior obtenemos las tensiones principales del
tensor desviador:
σ 1dev = 25 Pa
 dev
σ 2 = 6 Pa
σ dev = −31Pa
 3

Universidad de Castilla- La Mancha Draft Por: Eduardo W. V. Chaves (2012)


Ciudad Real - España
3 TENSIONES 299

Ejemplo 3.30
Descomponer el tensor de tensiones de Cauchy dado por sus componentes:
 12 4 0 
σ ij = σ 21 9 − 2 MPa
σ 31 σ 32 3 
en su parte esférica y desviadora.
Obtener los invariantes del tensor desviador
Obtener también la tensión normal octaédrica, y la tensión media en este punto.
Solución:
Debido a la simetría del tensor de tensiones de Cauchy:
12 4 0 
σ ij =  4 9 − 2 MPa

 0 − 2 3 

I σ 12 + 9 + 3 24
Tensión media σ m = σ oct = = = = 8.
3 3 3
La parte esférica y desviadora del tensor de tensiones son:
8 0 0 12 4 0  8 0 0 4 4 0 
σ ijesf = 0 8 0 ; σ ijdev = σ ij − σ ijesf 
=4   
9 − 2 − 0 8 0 = 4 1 − 2

0 0 8   0 − 2 3  0 0 8 0 − 2 − 5 

Los invariantes principales del tensor desviador son:


I σ dev ≡ J1 = 4 + 1 − 5 = 0 , como era de esperar, ya que la traza de cualquier tensor desviador
es cero.
1 −2 4 0 4 4
II σ dev = + + = −41 = − J 2
−2 −5 0 −5 4 1

o bien utilizando la definición: J 2 =


3
(
1 2 1
) (
I σ − 3 II σ = 24 2 − 3 × 151 = 41
3
)
III σ dev ≡ J 3 = det (σ dev ) = 44

Ejemplo 3.31
El estado tensional en un punto está dado por el tensor de tensión:
 σ aσ bσ
σ ij =  aσ σ cσ 
bσ cσ σ 

donde a , b , c son constantes y σ es un valor de tensión. Determinar las constantes a ,


b , c de tal manera que el vector tensión se anule en un plano octaédrico.
Solución:

Universidad de Castilla- La Mancha Draft Por: Eduardo W. V. Chaves (2012)


Ciudad Real - España
300 PROBLEMAS RESUELTOS DE MECÁNICA DEL MEDIO CONTINUO

1
Un plano octaédrico tiene el siguiente versor: nˆ i = [1 1 1] . El vector tensión en este
3
r (nˆ )
plano viene definido por t = σ ⋅ nˆ , en componentes:
t 1(nˆ )   σ aσ bσ 1 σ + aσ + bσ 0 a + b = 1
 (nˆ )    1  1  
 t 2  =  aσ σ c σ  1 =  aσ + σ + cσ  = 0 ⇒ a + c = −1
t (nˆ )  bσ cσ σ  3 1 3
 bσ + cσ + σ  0 b + c = −1
 3    

−1 −1 −1
resolviendo el sistema anterior obtenemos que, b = , c= , a=
2 2 2

Ejemplo 3.32
En un punto P del medio continuo el tensor de tensiones de Cauchy σ viene
representado por sus componentes cartesianas por:
 57 0 24
σ ij = σ 21 50 0  MPa ,
σ 31 σ 32 43
a) Determinar las tensiones principales y las direcciones principales en el punto P ;
b) Obtener la tensión tangencial máxima y la tensión normal máxima;
c) Dibujar el círculo de Mohr del estado tensional correspondiente;
r
d) Obtener el vector tensión t (n) en el plano octaédrico del espacio de Haigh-
Westergaard. Obtener también la tensión normal octaédrica y la tensión tangencial
octaédrica.
Solución:
Teniendo en cuenta que la simetría del tensor de tensiones de Cauchy:
57 0 24
σ ij =  0 50 0  MPa
24 0 43

Verificamos que la tensión σ 22 = 50 ya es una tensión principal y está asociada al


autovector nˆ ( 2) = [0 ± 1 0] . Para encontrar las otras tensiones principales resolvemos el
siguiente sistema:
57 − σ 24 σ1 = 25
=0 ⇒ σ 2 − 100σ + 1875 = 0 ⇒ 
24 43 − σ σ 3 = 75
Utilizando la definición de autovalor-autovector, podemos obtener los siguientes
autovectores:
Asociado al autovalor σ1 = 25 ⇒ nˆ (1) = [m 0,6 0 ± 0,8]

Asociado al autovalor σ 3 = 75 ⇒ nˆ (3) = [± 0,8 0 ± 0,6]


Circulo de Mohr en tensiones:
Reestructurando tal que σ I > σ II > σ III :
σ I = 75 , σ II = 50 , σ III = 25

Universidad de Castilla- La Mancha Draft Por: Eduardo W. V. Chaves (2012)


Ciudad Real - España
3 TENSIONES 301

b, c) El círculo de Mohr viene dibujado en la figura abajo:


σS ≡ τ
75 − 25
τ max = = 25
τ max = 25 2

σ III = 25 σ II = 50 σ I = 75 = σ N max σN

d) El espacio de Haigh-Westergaard está formado por las tensiones principales luego, el


r
vector tensor en este espacio viene dado por t (n) = σ ⋅ nˆ , cuya normal del plano octaédrico
 1 1 1 
tiene como componentes nˆ i =  :
 3 3 3

t 1(n)  75 0 0  1 75


r  (n)    1  1  
t (n) = σ ⋅ nˆ componente
 s → t 2  =  0 50 0  1 = 3 50
t (n)   0 0 25 3 1  25
 3    
Su módulo viene dado por:

( )
r 2 1 8750 r
t (n) = 75 2 + 50 2 + 25 2 = ⇒ t (n) = 54,00617
3 3
r
La tensión normal octaédrica viene dada por σ oct = t (n) ⋅ nˆ :
1
1
σ oct = [75 50 25] 1 = 50
3 3
1

Podríamos haber aplicado directamente la definición de tensión normal octaédrica:


Iσ 75 + 50 + 25
σ oct = = σm = = 50
3 3
La tensión tangencial octaédrica se puede obtener a través del teorema de Pitágoras:
r 2 8750
t (n)
2
τ oct = − σ oct = − 50 2 = 20,4124
3
También podríamos haber aplicado la definición:
1 1
τ oct = 2 I σ2 − 6 II σ = 2 × 150 2 − 6 × 6875 = 20,41241
3 3
donde I σ = 150 , II σ = 75 × 50 + 75 × 25 + 50 × 25 = 6875 .
e) Componentes de la parte esférica del tensor:

Universidad de Castilla- La Mancha Draft Por: Eduardo W. V. Chaves (2012)


Ciudad Real - España
302 PROBLEMAS RESUELTOS DE MECÁNICA DEL MEDIO CONTINUO

50 0 0 
Tr (σ )
σ ijesf = δ ij = σ m δ ij =  0 50 0 
3
 0 0 50

Su parte desviadora:
57 0 24 50 0 0   7 0 24 
σ ijdev = σ ij − σ ijesf =  0 50 0  −  0 50 0  =  0 0 0 
24 0 43  0 0 50  24 0 − 7 

f) Teniendo en cuenta que el tensor y su parte desviadora son coaxiales, es decir, presentan
las mismas direcciones principales, podemos utilizar el espacio principal para obtener los
valores principales del tensor desviador:
75 0 0  50 0 0   25 0 0 
σ′ijdev = σ′ij − σ ′ijesf     
=  0 50 0  −  0 50 0  =  0 0 0 
 0 0 25  0 0 50  0 0 − 25

3.1.6 Estado Tensional en Dos Dimensiones

Ejemplo 3.33
Considere el siguiente estado de tensión:

2
y

6
x

Figura 3.22:

Obtener el estado de tensión en este punto σ ij .


Solución:
En el estado de tensón plano σ ij (i, j = 1,2) se necesitan dos planos para definir
completamente el estado tensional en el punto:
σ x τ xy 
σ ij =  (3.78)
 τ xy σ y 

Universidad de Castilla- La Mancha Draft Por: Eduardo W. V. Chaves (2012)


Ciudad Real - España
3 TENSIONES 303

Según la Figura 3.22 verificamos que:


5

σx = 4

τ xy = 2
τ xy = 2

σy = 6

x
Figura 3.23:

Luego:
 4 2
σ ij =   (3.79)
2 6

Ejemplo 3.34
Considérese un material compuesto, constituido por matriz y fibras según dirección de 45 º
tal como se indica en la Figura 3.24. Este material compuesto puede romper si la tensión de
corte a lo largo de la fibra supera el valor de 3,8 × 10 6 Pa ( N / m 2 ) .
Para una tensión normal σ x = 2,8 × 10 6 Pa , determínese el valor máximo de σ y para que el
material no rompa.

σy

45º σx − 45º σx


y

σy
x

Figura 3.24: Material compuesto (matriz-fibra).

Universidad de Castilla- La Mancha Draft Por: Eduardo W. V. Chaves (2012)


Ciudad Real - España
304 PROBLEMAS RESUELTOS DE MECÁNICA DEL MEDIO CONTINUO

Solución: Este es un ejemplo típico de transformación de coordenadas. Es decir, tenemos


que considerar la tensión de corte máxima según la dirección θ = −45º . Para ello realizamos
la transformación de coordenadas siguiente:
σx − σy
τ ′xy ≡ τ ( θ ) = − sin 2θ + τ xy cos 2θ
2
⇒ σ y ≈ −4,8 × 10 6 Pa (compresión)
2,8 × 10 6 − σ y
τ ′xy ≡ τ ( θ = −45 º ) =− sin( −90 º ) = 3,8 × 10 6 Pa
2
Ver Ejemplo 1.96 del capítulo 1.

Ejemplo 3.35
Las tensiones que actúan en dos planos que pasan por el punto P están indicadas en la
Figura 3.25. Determínese el valor de la tensión de corte τ en el plano a − a y las tensiones
principales en este punto.
y

a b

τ
80 Pa
45 º
x
60 Pa 60 º
a

b
Figura 3.25: Estados tensionales en un punto, según los planos a y b .
Solución:
Para obtener el estado de tensión en un punto, en el caso de dos dimensiones,
determinamos las tensiones: σ x , σ y , τ xy , como se indica en la Figura 3.26.

y y

b a b
a σy σy
τ xy τ xy

80 Pa τ xy 80 Pa τ xy
σx
45º σx 45º σx
τ τ
x x
60 º τ
60 º xy

60 Pa 60 Pa
b a b a
a) b)

Figura 3.26: Estados tensionales en un punto, según los planos a y b .

Universidad de Castilla- La Mancha Draft Por: Eduardo W. V. Chaves (2012)


Ciudad Real - España
3 TENSIONES 305

Según la Figura 3.26, podemos determinar directamente σ x y τ xy descomponiendo el


vector tensión 60 Pa , ver Figura 3.26(b), i.e.:
σ x = 60 cos( 30 º ) = 51,962 Pa
τ xy = 60 cos( 60 º ) = 30 Pa

Para determinar la componente σ y , emplearemos las ecuaciones:


σx + σ y σx − σ y
σ′x ≡ σ ( θ) ≡ σ N = + cos 2θ + τ xy sin 2θ
2 2
σx − σ y
τ′xy ≡ σ S ≡ τ ( θ) = sin 2θ − τ xy cos 2θ
2
Reemplazando los valores numéricos en las expresiones anteriores:
51,962 + σ y 51,962 − σ y
σ ( θ = 45 º ) = + cos( 90 º ) + 30 sin( 90 º ) = 80 Pa
2 2
51,962 − σ y
τ ( θ = 45 º ) = sin( 90 º ) − 30 cos( 90 º )
2
La primera ecuación nos proporciona el valor de σ y :
σ y = 48,038 Pa

Una vez determinado σ y , podemos determinar τ (θ= 45º) :


τ ( θ= 45 º ) = 1,96 Pa

Las tensiones principales pueden determinarse a través de las componentes σ x , σ y , τ xy , tal


como se indica en las ecuaciones:
2
σx + σy  σx − σy 
σ (1, 2 ) = ±  
 + τ 2xy
2  2 

2
51,962 + 48,038  51,962 − 48,038  σ = 80,1Pa
σ (1, 2 ) = ±   + 30 2 ⇒  1
2  2  σ 2 = 19,9 Pa

Universidad de Castilla- La Mancha Draft Por: Eduardo W. V. Chaves (2012)


Ciudad Real - España
306 PROBLEMAS RESUELTOS DE MECÁNICA DEL MEDIO CONTINUO

Ejemplo 3.36
Dado el estado de tensiones σ x = 1Pa , τ xy = −4 Pa y σ y = 2 Pa . Obtener una gráfica de
ángulo-tensiones ( θ − σ x , σ y , τ xy ), siendo θ el ángulo de giro de la cuña dada en la Figura
3.27.
y

τ xy = 4 Pa σ y = 2 Pa
τ xy = −4 Pa
σx σ x = 1Pa
P
x

τ xy

σy

Figura 3.27: Estado tensional en un punto.

Solución:
Calculemos los distintos valores de σ ′x , σ′y , τ′xy utilizando las ecuaciones:
σx + σy σx − σy
σ ′x = + cos 2θ + τ xy sin 2θ
2 2
σx − σy
τ′xy = − sin 2θ + τ xy cos 2θ
2
σx + σy σy − σx
σ ′y = + cos 2θ − τ xy sin 2θ
2 2
Podemos calcular el ángulo correspondiente a la dirección principal a través de la ecuación:
2 τ xy 2 × ( − 4)
tan 2θ = = = 8 ⇒ (θ = 41,437 º )
σx − σ y 1− 2
y las tensiones principales:
2
σx + σy  σx − σy  σ1 = 5,5311P
σ1, 2 = ±  
 + τ 2xy ⇒ 
2  2  σ 2 = −2,5311Pa
Considerando las leyes de transformación, podemos obtener los distintos valores de
σ ′x , σ ′y , τ ′xy para distintos valores de θ . Haciendo θ variar de 0 hasta 360 º podemos
representar las tensiones σ ′x , σ ′y , τ′xy en función del ángulo, ver Figura 3.28. Podemos
observar que cuando θ = 41,437 º la tensión tangencial es cero ( τ xy = 0 ) y las tensiones
principales σ I = 5,5311Pa y σ II = −2,5311Pa .

Universidad de Castilla- La Mancha Draft Por: Eduardo W. V. Chaves (2012)


Ciudad Real - España
3 TENSIONES 307

x′
σ1
θ = 41,437 º
σ2

x′
θ = 131,437 º
8 σ2
Tensiones

6
σ1 = 5,5311 σ ′y

σy 2
τ ′xy
σx
0
0 50 100 150 200 250 300 350
θ
-2 σ ′x
45º x′

τ xy -4 σ 2 = −2,5311
θ = 86,437 º

-6

τ max = 4,0311

Figura 3.28: Tensiones en función del ángulo θ .

Ejemplo 3.37
a) Dado un campo de tensiones σ ij (i , j = 1,2) , y los siguientes valores:
t t t
2 2 2
m11 = ∫σ
t
11 x 3 dx 3 ; m12 = ∫σ
t
12 x 3 dx 3 ; m 22 = ∫σt
22 x 3 dx 3

− − −
2 2 2

y dado un nuevo sistema x1′ − x ′2 − x 3′ formado por una rotación alrededor de x3′ de un
ángulo θ , obtener la ley de transformación de mij (i, j = 1,2) para este nuevo sistema.
Solución:
Debido a la simetría de σ ij = σ ji , concluimos que m12 = m21 . La matriz de transformación
del sistema x1 − x 2 − x 3 al sistema x1′ − x ′2 − x 3′ viene dada por:
 cos θ sin θ 0
 cos θ sin θ 
a ij = − sin θ cos θ 0 2D
→ A= 
 0 0 1  − sin θ cos θ

Universidad de Castilla- La Mancha Draft Por: Eduardo W. V. Chaves (2012)


Ciudad Real - España
308 PROBLEMAS RESUELTOS DE MECÁNICA DEL MEDIO CONTINUO

Vamos utilizar la notación de Voigt, luego:


t t t t
2 2 2 2

∫ ∫ ∫ ∫
′ 
 m11 ′ 
 σ11 ′ 
 σ11  σ11   σ11 
{m ′} = m′22  =
  
σ′22  x3′ dx3′ =
 
σ′22  x3 dx3 = [M]σ 22  x3dx3 = [M]  
σ 22  x3 dx3
 m′  σ′  σ′  σ  σ 
 12   12   12   12   12 
−t −t −t −t
2 2 2 2

con eso, podemos concluir que:


′ 
 m11  m11 
{m ′} = m ′22  = [M]m 22  = [M]{m}
 
(3.80)
 m′  m 
 12   12 
donde la matriz [M] es la matriz de transformación para un tensor de segundo orden
cuando éste esté en la notación de Voigt, ver Ejemplo 1.96, y viene dada por:
 a11 2 a12
2
2a11 a12   cos 2 θ sin 2 θ 2 cos θ sin θ 
   
[M] =  a 21 2 a 22
2
2a 21 a 22  =  sin θ
2
cos θ
2
− 2 sin θ cos θ 
a a a 22 a12 a11 a 22 + a12 a 21  − sin θ cos θ cos θ sin θ cos 2 θ − sin 2 θ 
 21 11 
Además considerando que [M]−1 = [N ] , obtenemos {m} = [N ] {m ′}, donde
T T

 a11 2 a12
2
a11 a12   cos 2 θ sin 2 θ cos θ sin θ 
[N ] =  a 212 a 22
2
a 21 a 22
 
 =  sin θ
2
cos 2 θ − sin θ cos θ 

 2a a 2a 22 a12 a11 a 22 + a12 a 21   − 2 sin θ cos θ 2 cos θ sin θ cos 2 θ − sin 2 θ 
 21 11 
El mismo resultado (3.80) podría haber sido obtenido si considerábamos mij como un
tensor de segundo orden en el plano, y a través de la ley de transformación de un tensor de
segundo orden obtenemos que:
mij′ = a ik a jl m kl ; (i, j = 1,2) ó m ′ = Am A T
 m′ m12′   cos θ sin θ   m11 m12  cos θ − sin θ (3.81)
⇒  11 =

m12 m ′22   − sin θ cos θ m12 m 22   sin θ cos θ 

x3 = x3′

x2′
t
x3 =
2
x2

θ x1′
−t
x3 =
2 x1

Universidad de Castilla- La Mancha Draft Por: Eduardo W. V. Chaves (2012)


Ciudad Real - España
3 TENSIONES 309

3.1.7 Tensiones en Coordenadas Cilíndricas y Esféricas

Ejemplo 3.38
Demuéstrese que un cilindro cerrado de pared delgada de radio interno r y espesor t
sujeto a una presión interna p , ver Figura 3.29, tiene como estado tensional:
pr pr
σr = 0 ; σθ = ; σz =
t 2t
NOTA: Las expresiones anteriores sólo son válidas para un cilindro de pared delgada.

z
p

Figura 3.29: Cilindro cerrado bajo presión.


Solución:
Una vez adoptados los ejes de referencia de la Figura 3.29, planteamos el equilibrio de
fuerzas según las direcciones z , y y r .
Equilibrio de fuerzas según dirección z :

∑F z =0
p σz
2
p ( πr ) = σ z ( 2 πr )t
pr r pA = p (πr 2 )
⇒ σz =
2t
z

Equilibrio de fuerzas según dirección y :

L
∑ Fy = 0 σθ
2σ θ ( Lt ) = p ( 2 rL )
pr
⇒ σθ =
t
pA = p ( 2rL )
σθ
r y

Universidad de Castilla- La Mancha Draft Por: Eduardo W. V. Chaves (2012)


Ciudad Real - España
310 PROBLEMAS RESUELTOS DE MECÁNICA DEL MEDIO CONTINUO

Equilibrio de fuerzas según dirección r :


Podemos verificar que en la pared interna del cilindro la tensión radial ( σ r ) es igual a la
presión ( σ r = − p ) y en la pared externa está libre de presión σ r = 0 , luego:
− p ≤ σr ≤ 0

σr = − p σr = 0
r
p

r σ r << σ θ
para el caso >> 1 ⇒  luego, σ r ≈ 0 cuando comparado con σ z , σ θ .
t σ r << σ z
r
Experimentalmente se ha observado que la relación para despreciar σ r es ≥ 10 . En esta
t
situación:
pr pr
σz = = 5p ; σθ = = 10 p ; σr = − p
2t t
Podemos verificar también que la tensión σ θ es más grande, σ θ > σ z , es decir, que para un
material homogéneo, un cilindro rompería según dirección de σ z , como se indica en la
figura siguiente:

Pero si el material estuviera σθ


constituido por un material
heterogéneo, como por ejemplo, σz
matriz y fibras en la dirección de σ θ ,
la forma de rotura ya no estaría tan
definida.
Observemos también que las
tensiones obtenidas anteriormente
para el cilindro de pared delgada no
r
serán válidas si < 10 . El error
t
cometido ya será significativo.

Universidad de Castilla- La Mancha Draft Por: Eduardo W. V. Chaves (2012)


Ciudad Real - España
3 TENSIONES 311

Ejemplo 3.39
Demuéstrese que una esfera cerrada de pared delgada de radio interno r y espesor t ,
sometida a una presión interna p , ver Figura 3.30, presenta el estado tensional siguiente:
pr pr
σr = 0 ; σθ = ; σφ =
2t 2t
NOTA: Las expresiones anteriores sólo son válidas para una esfera de pared delgada.

x3 , z

ê r
x3
ê φ
r
θ ê θ

x1 x2
φ x2 , y

x1 , x

Figura 3.30: Esfera sometida a presión interna.


Solución:
Considerando los ejes adoptados en la Figura 3.30, planteamos el equilibrio de fuerzas
según dirección x , y , r .
Equilibrio de fuerzas según dirección x :

y
êθ ê r
∑F x =0
ê φ
− p ( πr 2 ) + σ φ ( 2 πr )t = 0
pr
2 ⇒ σφ =
pA = p (π r ) 2t

2r x
σφ

Universidad de Castilla- La Mancha Draft Por: Eduardo W. V. Chaves (2012)


Ciudad Real - España
312 PROBLEMAS RESUELTOS DE MECÁNICA DEL MEDIO CONTINUO

Equilibro de fuerzas según dirección y :

pA = p (π r 2 )
x
êθ êφ σθ
∑F y =0
ê r
− p ( πr 2 ) + σ θ ( 2 πr )t = 0
pr
⇒ σθ =
2t

2r

Equilibrio de fuerzas según dirección r :


Podemos verificar que en la pared interna de la esfera la tensión radial es igual a la presión
( σ r = − p ) y que la pared externa está libre de presión ( σ r = 0 ), luego:
− p ≤ σr ≤ 0

σr = − p σr = 0
r
p

r σ r << σ φ
Para el caso >> 1 ⇒  luego σ r ≈ 0 comparado con σ θ , σ φ .
t σ r << σ θ
r
Experimentalmente se ha observado que la relación para despreciar σ r es ≥ 10 , en esta
t
situación:
 pr
σ θ = 2t = 5 p

 pr
σ φ = = 5p
 2t
σ r = − p

Universidad de Castilla- La Mancha Draft Por: Eduardo W. V. Chaves (2012)


Ciudad Real - España
3 TENSIONES 313

3.2 Ejercicios Propuestos

Problema 3.1
El tensor de tensiones σ en la base cartesiana ê i viene representado por sus componentes:

 0,1 0,6 0,0


σ ij = 0,6 1,2 0,0 MPa (3.82)
0,0 0,0 0,3

Se pide:
r
a) Encontrar el vector tracción t según el plano 2 x1 − 2 x 2 + x 3 = 1 ;
r
b) Encontrar la magnitud del vector tracción t , vector tensión normal y tangencial
según el plano 2 x1 − 2 x 2 + x 3 = 1 ;
c) Obtener los invariantes principales I σ , II σ , III σ ;
d) Obtener la tensión de corte octaédrica.

Problema 3.2
Dibujar el círculo de Mohr para cada uno de los siguientes casos:
a) σ11 = 150 MPa (tensión uniaxial)
b) σ 22 = −110 MPa (compresión uniaxial)
c) σ11 = 50 MPa , σ 22 = 100 MPa (tensión biaxial)
d) σ11 = 50 MPa , σ 22 = −50 MPa (tensión biaxial)
e) σ11 = 8 MPa , σ 22 = σ 33 = −40 MPa (tensión triaxial)
f) σ11 = 50 MPa , σ 22 = −10 MPa , σ12 = σ 21 = 40 MPa , σ 33 = 30 MPa
Para cada caso encontrar también la tensión de corte máxima.

Problema 3.3
Las componentes del tensor de tensiones en un punto P , en el sistema de coordenadas
cartesianas, vienen dadas por:
 3 2 2
σ ij =  2 4 0 MPa (3.83)
 2 0 2

Encontrar:
a) El vector tracción en el punto P cuyo plano es normal al eje x1 ;

Universidad de Castilla- La Mancha Draft Por: Eduardo W. V. Chaves (2012)


Ciudad Real - España
314 PROBLEMAS RESUELTOS DE MECÁNICA DEL MEDIO CONTINUO

b) El vector tracción en P cuyo plano tiene normal igual a (1,−3,2) ;


c) Las tensiones principales de σ ;
d) Las direcciones principales de σ .

Problema 3.4
Consideremos que σ13 = σ 23 = σ 33 = 0 . Mostrar que si los ejes x1* y x2* son obtenidos a
partir de una rotación de α sobre el eje x3 , las componentes del tensor de tensiones en
este nuevo sistema vienen dadas por:
1 1
*
σ11 = (σ11 + σ 22 ) + (σ11 − σ 22 ) cos 2α + σ12 sin 2α (3.84)
2 2
1 1
σ *22 = (σ11 + σ 22 ) − (σ11 − σ 22 ) cos 2α − σ12 sin 2α
2 2
(3.85)
1
*
σ12 = − (σ11 − σ 22 ) sin 2α + σ12 cos 2α
2

Problema 3.5
¿Partiendo de qué principio se demuestra la simetría del tensor de tensiones?

Problema 3.6
Dadas las componentes del tensor de tensiones:
1 1 0
σ ij = 1 1 0
0 0 2

y sus valores principales:


2 0 0
σ ij = 0 2 0 (3.86)
0 0 0

Obtener los invariantes J1 , J 2 y J 3 .

Problema 3.7
Sabiendo que un punto se encuentra en un estado de tensión hidrostático y que la presión
media es ( − 4 Pa ), determinar las tensiones principales y las direcciones principales en este
punto.

Problema 3.8
El campo de tensión de un medio continuo viene representado por:

Universidad de Castilla- La Mancha Draft Por: Eduardo W. V. Chaves (2012)


Ciudad Real - España
3 TENSIONES 315

 1 0 2 x2 
σ ij =  0 1 4 x1  unidades de tensión (3.87)
 2 x 2 4 x1 1 

donde xi son las coordenadas cartesianas.


Se pide:
a) Despreciando las fuerzas másicas, ¿está el cuerpo en equilibrio?
b) Determinar el vector tensión que actúa en un punto ( x1 = 1, x 2 = 2, x3 = 3) según el
plano x1 + x 2 + x3 = 6 .
c) Determinar la proyección del vector tensión según la dirección normal y tangencial
al plano x1 + x 2 + x3 = 6

Problema 3.9
Si las componentes del tensor de tensiones de Cauchy en el sistema ortonormal ê1 , ê 2 , ê 3
son:
 σ11 σ12 σ13 
σ ij = σ12 σ 22 σ 23  (3.88)
σ13 σ 23 σ 33 

Si las tensiones principales son σ1 = 30,7259 , σ 2 = 13,8247 y σ 3 = 4,4494 y las direcciones


principales son:
para σ1 ⇒ nˆ (1) = [ 0,6720 0,7127 0,2010]
para σ 2 ⇒ nˆ ( 2 ) = [ − 0 ,4237 0,1474 0,8937] (3.89)
para σ 3 ⇒ nˆ (3) = [ 0,6073 − 0,6858 0,4010]

Obtener las componentes del tensor de tensiones de Cauchy σ ij en el sistema: ê1 , ê 2 , ê 3 .

Problema 3.10
El estado de tensión en un medio continuo respecto a los ejes cartesianos está dado por:
3 x1 x 2 5 x 22 0 
 
σ ij =  5 x 22 0 2 x3 
 0 2 x3 0 

Determinar el vector tensión que actúa en el punto P(2,1, 3 ) de un plano que es tangente
en P a la superficie cilíndrica x 22 + x32 = 4 .

Problema 3.11
Obtener las componentes del tensor de tensiones de Cauchy en el punto representado por
el elemento infinitesimal dado por la figura abajo:

Universidad de Castilla- La Mancha Draft Por: Eduardo W. V. Chaves (2012)


Ciudad Real - España
316 PROBLEMAS RESUELTOS DE MECÁNICA DEL MEDIO CONTINUO

Problema 3.12
El estado de tensión en un punto P del medio continuo se da esquemáticamente por:
x3

1
4
1
4 σ 22

1 1
x2

x1

Se pide:
Determinar el valor de la componente σ 22 del tensor de tensiones para que exista al menos
un plano que pase por P que esté libre de tensiones;
Determinar la dirección de dicho plano.

Problema 3.13

Universidad de Castilla- La Mancha Draft Por: Eduardo W. V. Chaves (2012)


Ciudad Real - España
4 Leyes Fundamentales de
la Mecánica del Medio
Continuo
4.1 Ejercicios Resueltos

Ejemplo 4.1
Demostrar el teorema del transporte de Reynolds partiendo de la siguiente expresión:
D D

Dt V
Φ dV =
Dt V ∫
Φ JdV 0 (4.1)
0

donde V es el volumen en la configuración actual, V0 es el volumen en la configuración de


referencia, J es el determinante del Jacobiano y Φ es un campo escalar que describe una
cantidad física de una partícula en el espacio por unidad de volumen en un instante de
tiempo t .
Solución:
D  DΦ DJ 

Dt V
Φ JdV0 =  J
V ∫Dt
+Φ dV 0
Dt 
0 0

 DΦ r

= J + JΦ ∇ xr ⋅ v dV 0 (4.2)
V0 
Dt 

 DΦ r

=  + Φ ∇ xr ⋅ v  dV
V 
Dt 

Ejemplo 4.2
r
Si PijL ( x , t ) representa alguna propiedad escalar, vectorial o tensorial cualquiera por unidad
de masa de un medio continuo, probar que la siguiente expresión es siempre válida:
r
D r DPijL ( x , t )

Dt V ∫
ρ PijL ( x , t ) dV = ρ
V
Dt
dV (4.3)
318 PROBLEMAS RESUELTOS DE MECÁNICA DEL MEDIO CONTINUO

Solución:
Partiendo del Teorema del transporte de Reynolds:
D r D r r ∂v p 
Dt V ∫ 
V 
Dt ∫
Φ ( x , t )dV =  Φ ( x , t ) + Φ ( x , t )
∂ x p
dV

Haciendo que Φ = ρ PijL y reemplazando en la ecuación anterior, resulta:
D D ∂v p 
Dt V∫ρ PijK dV ∫
=  (ρPijK ) + ρPijK
 Dt
V 
 dV
∂x p 
 D Dρ ∂v 

= ρ
V 
Dt
PijK + PijK
Dt
+ ρPijK k
∂x k
dV

 D  Dρ ∂v k  

=  ρ
V 
Dt
PijK + PijK 
 Dt
+ρ 
∂x k 
 dV

14243
=0
ecuación de continuida d

Con lo que concluimos que:


D  DPijK 
Dt V ∫
ρPijK dV = ρ

V 
Dt ∫  dV

c.q.d.

Ejemplo 4.3
Probar que la siguiente relación es válida:
∂ r r r r
ρa=(ρ v ) + ∇ xr ⋅ (ρ v ⊗ v ) (4.4)
∂t
r r r r r
donde ρ ( x , t ) es la densidad de masa, a ( x , t ) es la aceleración Euleriana, y v ( x , t ) es la
velocidad Euleriana.
Solución:
Partiendo del teorema del transporte de Reynolds:
D ∂Φ r

Φ dV = dV + Φ (v ⋅ nˆ ) dS
∫ ∫
Dt V V
∂t S
r
y considerando que Φ = ρ v obtenemos que:
r
D r ∂ (ρ v ) r r
∫ρ v dV = dV + ρ v ⊗ (v ⋅ nˆ ) dS
∫ ∫
Dt V V
∂t S
en notación indicial queda:
D ∂ (ρ v i )
Dt V∫ρ vi dV =
V
∂t ∫ ∫
dV + ρ v i (v k n k ) dS
S

D ∂ (ρ v i )
∫ρ 1
V
Dt
23
v dV = ∫ i
V
∂t S

dV + (ρ v i v k )n k dS
= ai
Aplicando el teorema de la divergencia para la integral de superficie obtenemos que:
∂ (ρ v i )  ∂ (ρ v i ) 
∫ ρ ai dV = ∫
V V
∂t
dV + ∫ (ρ v i v k ) ,k dV
V
⇒ ∫ ρ ai dV = ∫  V
+ (ρ v i v k ) ,k  dV
V  ∂t 
En notación tensorial:
r r
r  ∂ (ρ v ) r r  r ∂ (ρ v ) r r
∫V ρ a dV = V∫  ∂t + ∇ xr ⋅ (ρ v ⊗ v ) dV ⇒ ρa=
∂t
+ ∇ xr ⋅ (ρ v ⊗ v )
c.q.d.

Universidad de Castilla- La Mancha Draft Por: Eduardo W. V. Chaves (2012)


Ciudad Real – España
4 LEYES FUNDAMENTALES DE LA MECÁNICA DEL MEDIO CONTINUO 319

Ejemplo 4.4
Para un campo de velocidad dado por:
xi
vi = para t ≥ 0
1+ t
Se pide:
1) Encontrar la densidad de masa de una partícula en función del tiempo;
2) Probar que para el movimiento dado se cumple que ρ x1 x 2 x 3 = ρ 0 X 1 X 2 X 3 .
Solución:
1) Por la conservación de masa sabemos que:
Dρ ∂v Dρ dρ ∂v
+ρ k =0 ⇒ = = −ρ k
Dt ∂x k Dt dt ∂x k
Utilizando el campo de velocidad dado hallamos que:
∂v i 1 ∂x i δ ii 3
= = =
∂x i 1 + t ∂x i 1+ t 1+ t
luego
dρ 3ρ dρ 3dt
=− ⇒ =−
dt 1+ t ρ 1+ t
Integrando la relación anterior obtenemos que:
dρ 3dt
∫ ρ = ∫ − 1 + t ⇒ Lnρ = −3 Ln(1 + t ) + C1
La constante de integración C1 se obtiene con la condición inicial t = 0 donde se cumple
que la densidad de masa es igual a la densidad de masa de referencia ( ρ = ρ 0 ):
Ln ρ 0 = −3 Ln(1 + 0) + C1 ⇒ C1 = Ln ρ 0
 1   ρ0 
Ln ρ = −3 Ln(1 + t ) + Ln ρ 0 = Ln 3
 + lnρ 0 = Ln
  (1 + t ) 3


 (1 + t )   
Con lo que concluimos que:
ρ0
ρ=
(1 + t )3
2) Utilizando la definición de velocidad y aplicando el campo de velocidad obtenemos:
dx i x dx i dt
vi = = i ⇒ =
dt 1 + t xi 1 + t
Integrando la ecuación anterior resulta:
dx i dt
∫ xi
= ∫
1+ t
Lnx i = Ln(1 + t ) + C1 (4.5)
Aplicando la condición inicial, donde se cumple que: t = 0 ⇒ x i = X i , luego:
Ln X i = Ln(1 + 0) + C1 ⇒ C1 = Ln X i
Reemplazando el valor de C1 en la ecuación (4.5) obtenemos:
Ln x i = Ln(1 + t ) + Ln X i ⇒ Ln( x i ) = Ln[ X i (1 + t )]
con lo que concluimos que:
xi = X i (1 + t )
Expandiendo la relación anterior:

Universidad de Castilla- La Mancha Draft Por: Eduardo W. V. Chaves (2012)


Ciudad Real - España
320 PROBLEMAS RESUELTOS DE MECÁNICA DEL MEDIO CONTINUO

 x1
 x1 = X 1 (1 + t ) ⇒ (1 + t ) =
 X1
 x2
 x 2 = X 2 (1 + t ) ⇒ (1 + t ) =
 X2
 x
 x 3 = X 3 (1 + t ) ⇒ (1 + t ) = 3
 X3
ρ0
y considerando que ρ = (ver apartado 1 de este ejemplo), obtenemos:
(1 + t )3
ρ (1+3
12 t )(1+3
12 t )(1+3
12 t) = ρ 0
x1 x2 x3
X1 X2 X3

ρ x1 x 2 x 3 = ρ 0 X 1 X 2 X 3 c.q.d.

Ejemplo 4.5
r
Considérese un medio continuo y una propiedad φ ( x , t ) asignada por la densidad, i.e.
unidad de la propiedad por unidad de volumen. Obtener la tasa de esta propiedad de tal
forma que venga descrita por una integral de volumen de control conjuntamente con una
integral de superficie de control.
Solución:
Recordar de la tasa de una propiedad está relacionada siempre con las mismas partículas. A
través de la Derivada Material podemos obtener la tasa de una propiedad cuando ésta esté
en la descripción espacial. Luego, la variación total de la propiedad en volumen V que está
delimitado por la superficie S viene dada por:
D r D  D r r D 

Dt V ∫
φ ( x , t )dV =
V
Dt
(φ dV ) = dV
V 

Dt
φ( x , t ) + φ( x , t ) (dV )
Dt 
 D r r r 

= dV φ( x , t ) + φ( x , t )∇ xr ⋅ v dV  (4.6)
V 
Dt 
D r r r
=  φ( x , t ) + φ( x , t )∇ xr ⋅ v  dV

V 
Dt 
Podemos aplicar la definición de derivada material a la ecuación anterior, resultando que:
D r D r r r
φ( x , t )dV =  φ( x , t ) + φ ( x , t )∇ xr ⋅ v  dV
∫ ∫
Dt V V 
Dt 
r
∂ r ∂φ( x , t ) r r r r

=  φ( x, t ) + r ⋅ v ( x , t ) + φ( x , t )∇ xr ⋅ v  dV
V 
∂t ∂x 
r (4.7)
∂ r   ∂φ( x , t ) r r r

=  φ ( x , t )  dV +  ∫ r ⋅ v + φ( x , t )∇ xr ⋅ v  dV
V 
∂t  V 
∂x 
∂ r  r
=  φ ( x , t )  dV + [∇ xr ⋅ (φv ) ]dV
∫ ∫
V 
∂t  V

Podemos aplicar el teorema de la divergencia a la segunda integral del lado derecho de la


igualdad y obtener que:

Universidad de Castilla- La Mancha Draft Por: Eduardo W. V. Chaves (2012)


Ciudad Real – España
4 LEYES FUNDAMENTALES DE LA MECÁNICA DEL MEDIO CONTINUO 321

flujo de φ a través
de la superficie S
6 44744 8
r
D r ∂φ( x , t ) r (4.8)
∫φ ( x , t )dV = ∫ dV + ({ φv ) ⋅ nˆ dS

Dt V V 1
∂t4
42 3 S flujo de φ
local
r
∂φ( x , t )
como el término es local, la integral de volumen de la derecha de la igualdad es un
∂t
volumen de control y la integral de superficie es una superficie de control ya que las
r r
variables (φv ) están en las descripción Euleriana. El término (φv ) representa el flujo de la
propiedad φ . Cuando no hay fuente o sumidero de la propiedad se cumple que
D r
Dt V∫φ( x, t )dV = 0 . Verificar también que la ecuación (4.8) cuando la propiedad es la

densidad de masa se denomina de ecuación de continuidad de masa:

D r D r r r
ρ ( x , t )dV =  ρ ( x , t ) + ρ ( x , t )∇ xr ⋅ v  dV = 0
∫ ∫
Dt V V 
Dt 
(4.9)
∂ r r 
=  ρ ( x , t ) + ∇ xr ⋅ (ρv )  dV = 0

V 
∂t 

Si la ecuación anterior es válida para todo el volumen también lo será localmente, i.e.:
D r r r
ρ ( x , t ) + ρ ( x , t )∇ xr ⋅ v = 0 Ecuación de continuidad de masa (4.10)
Dt
or
∂ r r
ρ ( x , t ) + ∇ xr ⋅ (ρv ) = 0 Ecuación de continuidad de masa (4.11)
∂t

volumen de control r
(φv )
S
r r
V q n = [(φv ) ⋅ nˆ ] nˆ

r
r ∂φ( x , t )
x
∂t
superficie de control

Figura 4.1: Volumen y superficie de control.

Universidad de Castilla- La Mancha Draft Por: Eduardo W. V. Chaves (2012)


Ciudad Real - España
322 PROBLEMAS RESUELTOS DE MECÁNICA DEL MEDIO CONTINUO

Volumen material Volumen de control Superficie de control

t=0

v0

XP
X*

Superficie de control
Volumen material Volumen de control
t1

r
v( x * , t1 )

xP
x*
Superficie de control
Volumen material Volumen de control
t2

r
v( x * , t 2 )

xP x*

Figura 4.2: Volumen material vs. volumen y superficie de control.

Ejemplo 4.6
Las componentes del campo del tensor de tensiones de un medio continuo en equilibrio
vienen dadas por:
σ11 = x12 ; σ 22 = x 22 ; σ 33 = x12 + x 22
σ12 = σ 21 = 2 x1 x 2 ; σ 23 = σ 32 = σ 31 = σ13 = 0
Encontrar la fuerza másica (por unidad de volumen) que actúa en el continuo.
Solución: Aplicando las ecuaciones de equilibrio podemos obtener que:
r r
∇ xr ⋅ σ + ρb = 0
 ∂σ11 ∂σ12 ∂σ13
 + + + ρb 1 = 0
 ∂x1 ∂x 2 ∂x 3
2 x1 + 2 x1 + ρb 1 = 0
 ∂σ 21 ∂σ 22 ∂σ 23 
 + + + ρb 2 = 0 ⇒  2 x 2 + 2 x 2 + ρb 2 = 0
 ∂x1 ∂x 2 ∂x 3 ρb = 0
 ∂σ 31 ∂σ 32 ∂σ 33  3
 + + + ρb 3 = 0
 ∂x1 ∂x 2 ∂x 3
Con lo que concluimos que para satisfacer las ecuaciones de equilibrio hay que cumplir que:

Universidad de Castilla- La Mancha Draft Por: Eduardo W. V. Chaves (2012)


Ciudad Real – España
4 LEYES FUNDAMENTALES DE LA MECÁNICA DEL MEDIO CONTINUO 323

4 x1 = − ρb 1 ⇒ ρb 1 = −4 x1
4 x 2 = − ρ b 2 ⇒ ρ b 2 = −4 x 2
⇒ ρb 3 = 0
r
ρb = −4( x1 eˆ 1 + x 2 eˆ 2 )

Ejemplo 4.7
Sea el movimiento del continuo dado por las siguientes ecuaciones:
 x1 = X 1 + αtX 3

 x 2 = X 2 + αtX 3
 x = X − αt ( X + X )
 3 3 1 2
donde α es una constante. Encontrar la densidad de masa en la configuración actual (ρ )
en función de la densidad de masa en la configuración de referencia (ρ 0 ) .
Solución:
Queremos encontrar una función densidad de masa tal que: ρ = ρ (ρ 0 ) . Sabemos que la
ecuación de continuidad de masa en la forma Lagrangiana viene dada por ρ 0 = Jρ , donde
el determinante del Jacobiano puede ser obtenido directamente de las ecuaciones de
movimiento:
∂x1 ∂x1 ∂x1
∂X 1 ∂X 2 ∂X 3
1 0 αt
∂x i ∂x 2 ∂x 2 ∂x 2
J= F = = = 0 1 αt = 1 + 2(αt ) 2
∂X j ∂X 1 ∂X 2 ∂X 3
∂x 3 ∂x 3 ∂x 3 − αt − αt 1
∂X 1 ∂X 2 ∂X 3
Resultando así:
ρ0 ρ0
ρ= =
J 1 + 2(αt ) 2

Ejemplo 4.8
Dado el campo de velocidad:
v1 = x1 x3 ; v 2 = x 22 t ; v3 = x 2 x 3t
y el campo de tensiones:
 x 2 x1 − x 2 x3 0 
σ ij = α − x 2 x3 x 22 − x 2 
 0 − x2 x32 
donde α es una constante. Encontrar las fuerzas másicas (por unidad de volumen) de tal
forma que cumpla con el principio de la conservación del momento lineal.
Solución:
Del principio de la conservación del momento lineal obtenemos las ecuaciones del
movimiento:
r r r r r
∇ xr ⋅ σ + ρb = ρv& = ρa ⇒ ρb = ρa − ∇ xr ⋅ σ
El campo de aceleración:

Universidad de Castilla- La Mancha Draft Por: Eduardo W. V. Chaves (2012)


Ciudad Real - España
324 PROBLEMAS RESUELTOS DE MECÁNICA DEL MEDIO CONTINUO

r r r r
r ∂v ( x , t ) ∂v ( x , t ) r r ∂v i ∂v i
a= + r ⋅ v ( x, t ) ; ai = + vj
∂t ∂x ∂t ∂x j

donde
 0   x3 0 x1 
∂v i  2  ∂v i 
= x2  ; = 0 2x2 t 0 
∂t  ∂x j 
 x 2 x3   0 x3t x 2 t 
Luego:
∂v i ∂v i
ai = + vj
∂t ∂x j
 0   x3 0 x1   x1 x 3   0   x1 x 32 + x1 x 2 x 3 t 
 
=  x 22  +  0 2 x2t 0   x 22 t  =  x 22  +  2 x 23 t 
 x 2 x 3   0 x3t x 2 t   x 2 x 3 t   x 2 x 3   x 3 x 22 t 2 + x 22 x 3 t 2 
 x1 x 32 + x1 x 2 x 3 t 
 
= x2 + 2x2t
2 3

x x + x x 2t 2 + x 2 x t 2 
 2 3 3 2 2 3 
La divergencia del tensor de tensiones de Cauchy viene dada por:
 ∂σ11 ∂σ12 ∂σ 13
 + + = α ( x 2 − x3 )
 ∂x1 ∂x 2 ∂ x 3
 ∂σ 21 ∂σ 22 ∂σ 23
 + + = α(2 x2 )
 ∂x1 ∂x 2 ∂x 3
 ∂σ 31 ∂σ 32 ∂σ 33
 + + = α ( 2 x 3 − 1)
 ∂x1 ∂x 2 ∂x 3

Con lo cual, las fuerzas másicas quedan:


r r
ρb = ρa − ∇ xr ⋅ σ
ρb i = ρa i − σ ij , j
 x1 x 32 + x1 x 2 x 3 t   x 2 − x3 
   
ρb i = ρ  2
x2 + 2 x2 t 3
 − α  2 x2 
 x 2 x 3 + x 3 x 22 t 2 + x 22 x 3 t 2   2 x 3 − 1 
 

Ejemplo 4.9
Teniendo en cuenta el Principio de la conservación del momento angular, demostrar que:

[r r r r r
∫ ρ ( x ⊗ (a − b) − (a − b) ⊗ x dV =
r
] [∫ ( xr ⊗ tr *
r r
]
− t * ⊗ x dS
V Sσ

donde
r r r r r r
x - vector posición; ρ ( x , t ) -densidad de masa; a ( x , t ) -aceleración; b( x , t ) -fuerzas másicas
r r
(por unidad de masa); t * ( x , t ) - vector tracción prescrito (fuerza de superficie) en la
superficie S σ .

Universidad de Castilla- La Mancha Draft Por: Eduardo W. V. Chaves (2012)


Ciudad Real – España
4 LEYES FUNDAMENTALES DE LA MECÁNICA DEL MEDIO CONTINUO 325

Solución:
El principio de la conservación del momento angular establece que:
r r r r D r r r r

Sσ V

( x ∧ t * )dS + ( x ∧ ρb)dV =
Dt V
( x ∧ ρ v )dV = ( x ∧ ρ a )dV
V
∫ ∫
A continuación hacemos el producto vectorial de la expresión anterior con un vector
r r
arbitrario z , independiente de x , resultando:
r r r r r r r r r

V

z ∧ ( x ∧ ρ a )dV = z ∧ ∫

( x ∧ t * )dS + z ∧ ( x ∧ ρb)dV ∫
V
r r r r r r r r r

⇒ z ∧ ( x ∧ ρ a )dV =
V


z ∧ ( x ∧ t * )dS + z ∧ ( x ∧ ρb)dV∫
V

r r r
Se ha demostrado en el capítulo 1 que dados tres vectores a , b , c , se cumple que
r r r r r r r r
a ∧ (b ∧ c ) = (b ⊗ c − c ⊗ b) ⋅ a . Luego, la expresión anterior puede ser reescrita como:
r r r r r r r r r r r r r r r
∫ ( x ⊗ ρ a − ρ a ⊗ x ) ⋅ z dV = ( x ⊗ t * − t * ⊗ x ) ⋅ z dS + ( x ⊗ ρb − ρb ⊗ x ) ⋅ z dV
∫ ∫
V Sσ V
r r r r r r r r r r r r r r r
⇒ ρ ( x ⊗ a − a ⊗ x ) ⋅ z dV − ρ ( x ⊗ b − b ⊗ x ) ⋅ z dV = ( x ⊗ t * − t * ⊗ x ) ⋅ z dS
∫ ∫ ∫
V V Sσ
r r r
[ r r r r r r r r r
⇒ ρ x ⊗ (a − b) − (a − b) ⊗ x ⋅ z dV = ( x ⊗ t * − t * ⊗ x ) ⋅ z dS
∫ ] ∫
V Sσ

[  r  r r
]
 r r r r r r r r  r
⇒  ρ x ⊗ (a − b) − (a − b) ⊗ x dV  ⋅ z =  ( x ⊗ t * − t * ⊗ x ) dS  ⋅ z
∫ ∫
V  S σ 
Con eso, concluimos que:

∫ ρ [x ⊗ (a − b) − (a − b) ⊗ x ] dV = ∫ ( x ⊗ t
r r r r r r r r* r r
− t * ⊗ x ) dS
V Sσ

Ejemplo 4.10
1) Teniendo en cuenta la definición del tensor tensión media ( σ ) de un medio continuo:
V σ = σ dV ∫
V

Y partiendo del principio de que el continuo está en equilibrio estático, demostrar que se
cumple que:

σ=
1
2V
[r r
∫ ρ x ⊗ b + b ⊗ x dV +
r r
] 1
2V ∫
r r r r
( x ⊗ t * + t * ⊗ x ) dS
V Sσ

2) Teniendo en cuenta ahora que el volumen del continuo viene dado por V = V (1) − V ( 2) ,
ver Figura 4.3. Dicho continuo está sometido a una presión p (1) en la superficie S (1) , y una
presión p ( 2) en la superficie S ( 2) . Considerando que el continuo está libre de fuerzas
másicas, verificar que se cumple la relación:
−1
σ= (1) ( 2)
( p (1)V (1) − p ( 2)V ( 2 ) )1
(V −V )

Universidad de Castilla- La Mancha Draft Por: Eduardo W. V. Chaves (2012)


Ciudad Real - España
326 PROBLEMAS RESUELTOS DE MECÁNICA DEL MEDIO CONTINUO

n̂ (1)
S (1)

V (1) p (1)

S (2)

V ( 2)
n̂ ( 2 )

p (2)

Figura 4.3

Solución:
r r r
Teniendo en cuenta la ecuación de equilibrio ∇ xr ⋅ σ + ρb = ρa = 0 (principio de la
conservación del momento lineal). Para todo el continuo hay que cumplir que:


r
[ r
] r
x ⊗ ∇ xr ⋅ σ + ρb dV = 0
V
r r r r (4.12)
⇒ x ⊗ ∇ xr ⋅ σ dV + x ⊗ ρb dV = 0
∫ ∫
V V

En el capítulo 1, hemos demostrado que se cumplen las siguientes relaciones:

r r r* r
∫ (∇ ⋅ σ ) ⊗ x dV = ∫ (σ ⋅ nˆ ) ⊗ x dS − ∫ σ dV = ∫ t
V S V S
⊗ x dS − σ dV ∫
V
(4.13)

r r r r
∫ x ⊗ (∇ ⋅ σ ) dV = ∫ x ⊗ (σ ⋅ nˆ ) dS − ∫ σ ∫ ∫
T
dV = x ⊗ t * dS − σ T dV (4.14)
V S V S V
r
donde hemos tenido en cuenta que el vector tensión prescrito, t * = σ ⋅ nˆ . Reemplazando
(4.14) en la expresión (4.12), obtenemos que:

Universidad de Castilla- La Mancha Draft Por: Eduardo W. V. Chaves (2012)


Ciudad Real – España
4 LEYES FUNDAMENTALES DE LA MECÁNICA DEL MEDIO CONTINUO 327

r r r r
∫ x ⊗ ∇ xr ⋅ σ dV + x ⊗ ρb dV = 0 ∫
V V

r r r r r

S

⇒ x ⊗ t * dS − σ T dV + x ⊗ ρb dV = 0
V

V
(4.15)

r r r r
∫ ∫
⇒ σ T dV = x ⊗ t * dS + x ⊗ ρb dV
V S

V

También se cumple que:


r r r r

V

σ dV = t * ⊗ x dS + ρb ⊗ x dV
S

V
(4.16)

r r r r
Observemos que los tensores resultantes de las operaciones x ⊗ t * , x ⊗ ρb , no son
tensores simétricos. Esto quiere decir que en la expresión (4.12) no ha tenido en cuenta el
principio de la conservación del momento angular (simetría del tensor de tensiones de
Cauchy). Para garantizar que σ sea simétrico, hacemos que:

σ + σT 1 r r r r  1 r r r r 
∫ 2 ∫ ∫
dV =  t * ⊗ x dS + ρb ⊗ x dV  +  x ⊗ t * dS + x ⊗ ρb dV 
2 S  2  S 
∫ ∫
V  V V
(4.17)


⇒ σ sym dV =
1
2V
r r r r
∫ [
ρ x ⊗ b + b ⊗ x dV +
1 r r* r* r
2S
x ⊗ t + t ⊗ x dS] ∫[ ]
V

Teniendo en cuenta la definición del tensor tensión media, concluimos que:

σ + σT 1 r r r r  1 r r r r 
∫ 2 ∫ ∫
dV =  t * ⊗ x dS + ρb ⊗ x dV  +  x ⊗ t * dS + x ⊗ ρb dV 
2 S  2  S 
∫ ∫
V  V V


⇒ σ sym dV =
1
2V
r r r r
∫ [
ρ x ⊗ b + b ⊗ x dV +
1 r r* r* r
2S
x ⊗ t + t ⊗ x dS] ∫[ ]
V
(4.18)
⇒V σ =
1
2V ∫ [
r r r r
ρ x ⊗ b + b ⊗ x dV +
1 r r* r* r
2S
]
x ⊗ t + t ⊗ x dS ∫[ ]
⇒σ =
1
2V
[r r r
∫ ρ x ⊗ b + b ⊗ x dV +
r
] 1
2V
[∫ xr ⊗ tr *
r r
+ t * ⊗ x dS ]
V S

Si además el cuerpo está libre de fuerzas másicas, la expresión anterior se resume a:

σ=
1
2V
[∫ xr ⊗ tr *
r r
+ t * ⊗ x dS] (4.19)
S

Para el caso particular de la Figura 4.3 tenemos que V = V (1) − V ( 2) , S = S (1) + S ( 2) ,


r (1) r ( 2)
t * = − p (1) n̂ (1) , t * = − p ( 2 ) n̂ ( 2) . En este caso, la expresión (4.19) queda:

Universidad de Castilla- La Mancha Draft Por: Eduardo W. V. Chaves (2012)


Ciudad Real - España
328 PROBLEMAS RESUELTOS DE MECÁNICA DEL MEDIO CONTINUO

 r r 
∫[ ] ∫[ ]
1 *
r* r (1) r r* r* r (2) 
σ=  x ⊗ t + t ⊗ x dS + x ⊗ t + t ⊗ x dS 
2(V (1) − V ( 2 ) ) S (1) S ( 2) 
 
=
2(V (1)
1
( 2) 
− V ) S (1) ∫
r
[ r
] r
∫ [ r
− p (1) x ⊗ nˆ (1) + nˆ (1) ⊗ x dS (1) + − p ( 2 ) x ⊗ nˆ ( 2) + nˆ ( 2 ) ⊗ x dS ( 2 )  ]
S (2) 
 
∫[ ] ∫[ ]
−1 (1) r ˆ (1) ˆ (1) r (1) ( 2) r ˆ ( 2) ˆ (2) r ( 2) 
=  p x ⊗ n + n ⊗ x dS + p x ⊗ n + n ⊗ x dS 
2(V (1) − V ( 2 ) )  S ( 1) S ( 2) 

r r
Hemos demostrado en el capítulo 1 que se cumple ∫ ( x ⊗ nˆ + nˆ ⊗ x ) dS = 2V 1 , donde n̂ es
S

el versor normal exterior a la superficie. Para este ejemplo n̂ ( 2) es normal interior a la

∫ [x ⊗ nˆ ]
r r
superficie S ( 2) , debido a eso tenemos que ( 2)
+ nˆ ( 2 ) ⊗ x dS ( 2 ) = −2V ( 2 ) 1 ,
(2)
S
resultando que:
 
∫[ ] ∫[ ]
−1 (1) r
ˆ (1) + nˆ (1) ⊗ x dS (1) + p ( 2 ) x ⊗ nˆ ( 2) + nˆ ( 2 ) ⊗ x dS ( 2 ) 
r r r
σ=  p x ⊗ n
2(V (1) − V ( 2 ) )  S (1 ) S (2) 

=
−1
{
p (1) 2V (1) 1 − p ( 2) 2V ( 2 ) 1 }
2(V − V ( 2 ) )
(1)

= (1)
−1
{
p (1)V (1) − p ( 2 )V ( 2 ) 1 }
(V − V ( 2) )

Ejemplo 4.11
Dado el campo de velocidad:
v1 = ax1 − bx 2
v 2 = bx1 − ax 2
v3 = c x12 + x 22
donde a , b y c son constantes, se pide:
a) Comprobar si se cumple la ecuación de continuidad de masa;
b) ¿Es un movimiento isocórico?, es decir, ¿es un medio incompresible?
Solución:
Ecuación de continuidad de masa:
Dρ r
+ ρ (∇ xr ⋅ v ) = 0
Dt
donde:
r
∇ xr ⋅ v = vi ,i = v1,1 + v 2, 2 + v3,3
=a−a+0=0
r
El movimiento es isocórico, ya que ∇ xr ⋅ v = 0

Universidad de Castilla- La Mancha Draft Por: Eduardo W. V. Chaves (2012)


Ciudad Real – España
4 LEYES FUNDAMENTALES DE LA MECÁNICA DEL MEDIO CONTINUO 329

4.1.1 Problemas de Flujo

Ejemplo 4.12
Obtener la ecuación de energía para un movimiento de sólido rígido. Considérese que el
r
flujo de calor viene dado por q = −K (T ) ⋅ ∇ xr T , donde K (T ) es un tensor de segundo orden
denominado de tensor de conductividad térmica (propiedad del material), y considérese
∂u
también que c = , donde c es el calor específico (propiedad del material). Proporcionar
∂T
también la unidad del tensor K .
Solución: Para un movimiento de cuerpo rígido la potencia tensional es igual a cero, luego, la
ecuación de energía se resume a:
∂u ∂T r r
ρ u& = ρ =σ { : D − ∇ xr ⋅ q + ρr = −∇ xr ⋅ q + ρr
∂T ∂t =0
∂T r
⇒ ρc = −∇ xr ⋅ q + ρr
∂t
∂T
⇒ ρc = −∇ xr ⋅ [− K (T ) ⋅ ∇ xr T ] + ρr
∂t
Resultando así que:
∂T
∇ xr ⋅ [K (T ) ⋅ ∇ xr T ] + ρr = ρc
∂t
La ecuación anterior se conoce como ecuación de flujo de calor que se aplica a problema de
conducción térmica.
DT ∂T
NOTA: Cuando no hay transporte de masa se cumple T& ≡ = .
Dt ∂t
∂T  K
Teniendo en cuenta las siguientes unidades: [q] =
r J W 
= 2 , ∇ xr T = r  = , podemos
2
m s m  ∂x  m
verificar que para que haya compatibilidad de unidades hay que cumplir que:
[qr ] = [K ] ⋅ [∇ xr T ]
 J W   J W  K 
 m 2 s = m 2  =  s m K = m K  m 
    
 J W 
Luego, concluimos que [K ] =  = .
s m K m K 
NOTA: También es interesante destacar que debido a que la potencia tensional es nula,
podemos desacoplar el problema mecánico del problema térmico, es decir, podemos
analizar los dos problemas separadamente. ■

Ejemplo 4.13
Consideremos un problema de conducción térmica, ver Ejemplo 4.12, donde tenemos una
pared de espesor h en la cual la temperatura en la cara exterior ( x1 = 0 ) es igual a 38º C y
la temperatura en la cara interior ( x1 = h ) es igual a 21º C , ver Figura 4.4. Obtener el flujo
de calor para el caso definido por: problema estacionario, el campo de temperatura según

Universidad de Castilla- La Mancha Draft Por: Eduardo W. V. Chaves (2012)


Ciudad Real - España
330 PROBLEMAS RESUELTOS DE MECÁNICA DEL MEDIO CONTINUO

las direcciones x 2 y x3 es homogéneo, no hay fuente interna de calor, y el material es


homogéneo e isótropo.
x2

T ( A) = 38º C Datos:
h = 0,04m
T ( B ) = 21º C W
K = 0,19
mK
(Exterior) (Interior)

x1

r
q
h

Figura 4.4
NOTA: Cuando decimos que un problema es estacionario, nos referimos que el campo de
la incógnita es estacionario. En el caso de la ecuación de flujo de calor, la incógnita es la
temperatura.
Solución:
Como hemos visto en el Ejemplo 4.12 la ecuación de gobierno para este problema es la
∂T
ecuación ∇ xr ⋅ [K ⋅ ∇ xr T ] + ρr = ρc . Si consideramos el problema estacionario, tenemos
∂t
∂T
que = 0 . Si no hay fuente interna de calor esto implica que r = 0 . Con estas
∂t
simplificaciones la ecuación de flujo de calor se resume a ∇ xr ⋅ [K ⋅ ∇ xr T ] = 0 , además si el
r
material es homogéneo, el tensor con las propiedades térmicas K no varía con x
resultando que ∇ xr ⋅ [K ⋅ ∇ xr T ] = K : ∇ xr [∇ xr T ] = 0 o en notación indicial
[K ij T , j ],
i
= K ij ,i T , j + K ij T , ji = K ij T , ji = 0 . Expandiendo ésta última expresión, obtenemos
123
=0
que:
∂ 2T ∂ 2T ∂ 2T ∂ 2T ∂ 2T ∂ 2T
K 11 + K 12 + K 13 + K 21 + K 22 + K 23 +
∂x12 ∂x 2 ∂x1 ∂x3 ∂x1 ∂x1∂x 2 ∂x 22 ∂x3 ∂x 2
(4.20)
∂ 2T ∂ 2T ∂ 2T
+ K 31 + K 32 + K 33 2 = 0
∂x1∂x3 ∂x 2 ∂x3 ∂x3

Si el campo de temperatura según las direcciones x 2 y x3 es homogéneo, eso implica que


las componentes del gradiente de temperatura según estas direcciones son iguales a cero, i.e.
∂T ∂T
= = 0 . Para un material isótropo, las componentes del tensor de conductividad
∂x 2 ∂x3
térmica, ver Capítulo 5 del libro texto, vienen dadas por:

Universidad de Castilla- La Mancha Draft Por: Eduardo W. V. Chaves (2012)


Ciudad Real – España
4 LEYES FUNDAMENTALES DE LA MECÁNICA DEL MEDIO CONTINUO 331

K 0 0 
K ij =  0 K 0 
 0 0 K 

Con estas consideraciones la expresión (4.20) se reduce a:


∂ 2T 11 =K
∂ 2T
K 11 =0 K → ⇒K =0 (4.21)
∂x12 ∂x12

∂ 2T
Integrando una vez la expresión K = 0 obtenemos que:
∂x12

∂ 2T ∂T dT
K =0 integrando
 → K + q1 = 0 ⇒ q1 = −K
∂x12 ∂x1 dx1

Que es la ecuación de flujo de calor de Fourier. Observar que para este caso q1 es una
constate, es decir, es independiente de x1 . Integrando una vez más:
− q1 − q1
∫ dT = ∫ K
dx1 ⇒ T ( x1 ) =
K
x1 + C

Aplicando la condición de contorno, x1 = 0 ⇒ T = T ( A) , obteniendo así la constante de


− q1
integración C = T ( A) . Resultando que T ( x1 ) = x1 + T ( A) . Además para x1 = h tenemos
K
que
− q1 (T ( B ) − T ( A) )
T ( x1 = h) = T ( B ) = h + T ( A) ⇒ q1 = −K
K h
Fijemos que en este caso (unidimensional) el gradiente de temperatura es la pendiente de la
recta definida por la temperatura, y la temperatura varía linealmente en la pared, ver Figura
4.4.
Reemplazando los datos del problema, ver Figura 4.4, obtenemos el flujo:
(T ( B ) − T ( A) )  W  (21 − 38)( K ) W J
q1 = −K = −0,19  = 80,75 2 = 80,75 2
h  mK  0,04(m) m m s
Observar que la transformación de temperatura de grados Celsius para Kelvin viene dada
por K = º C + 273,15 , luego la diferencia de temperatura sea en grados Celsius o en Kelvin
será la misma. Observar también que el flujo de calor va en el sentido de mayor
temperatura hacia la región de menor temperatura.

Universidad de Castilla- La Mancha Draft Por: Eduardo W. V. Chaves (2012)


Ciudad Real - España
332 PROBLEMAS RESUELTOS DE MECÁNICA DEL MEDIO CONTINUO

NOTA: Supongamos que ahora tengamos dos paredes con propiedades distintas tal y
como se muestra en la Figura 4.5.

T ( A)

T (B )

T (C )
1 2

K (1) K ( 2) r
q
x1
h (1) h (2)

Figura 4.5
(T ( B ) − T ( A) )
Observemos que la expresión q1 = −K (1) sigue siendo válida. También es
h (1)
(T (C ) − T ( B ) )
válido para el material 2 : q1 = −K ( 2) . Para obtener el flujo tenemos que
h (2)
aplicar la compatibilidad de temperatura en la cara B , es decir:
(T ( B _ 1) − T ( A) ) q1 h (1)
q1 = −K (1) ⇒ T ( B _ 1) = T ( A) −
h (1) K (1)
(T (C ) − T ( B _ 2 ) ) q1 h ( 2)
q1 = −K ( 2) ⇒ T ( B _ 2) = T (C ) +
h ( 2) K ( 2)
T ( B _ 1) = T ( B _ 2 )
q1 h (1) q1 h ( 2 )
T ( A) − = T (C ) +
K (1) K ( 2)
Resultando que:
− (T (C ) − T ( A) )
q1 =
 h (1) h ( 2 ) 
 
 K (1) + K ( 2 ) 
 

Ejemplo 4.14
Considérese un problema de conducción térmica estacionario sin fuente interna de calor y
r r
donde el flujo de calor viene gobernado por la ley de Fourier ( q = −K ( x ) ⋅ ∇ xr T ), donde el
r
campo de tensor de conductividad térmica viene representado por K ( x ) , y es un tensor de
segundo orden arbitrario (no necesariamente simétrico).
a) Demostrar que el tensor de conductividad térmica es un tensor semi-definido positivo;
r
b) Verificar en que situación la parte antisimétrica de K ( x ) no afecta en el resultado del
problema de conducción de calor. Considerar la potencial tensional igual a cero.

Universidad de Castilla- La Mancha Draft Por: Eduardo W. V. Chaves (2012)


Ciudad Real – España
4 LEYES FUNDAMENTALES DE LA MECÁNICA DEL MEDIO CONTINUO 333

c) Que formato presenta K si el material es isótropo.


Solución:
a) Partimos de la desigualdad de conducción de calor que hay que cumplir siempre:
r
− q ⋅ ∇ xr T ≥ 0 − q i T,i ≥ 0
r
− (−K ( x ) ⋅ ∇ xr T ) ⋅ ∇ xr T ≥ 0 − (− K ij T, j )T,i ≥ 0
r
∇ xr T ⋅ K ( x ) ⋅ ∇ xr T ≥ 0 T,i K ij T, j ≥ 0

Recordar que un tensor arbitrario A es semi-definido positivo si se cumple que


r r r r r
x ⋅ A ⋅ x ≥ 0 para todo x ≠ 0 . Demostrando así que K ( x ) es un tensor semi-definido
r
positivo. Luego, como consecuencia los autovalores de K ( x ) serán todos valores reales
r
mayores o iguales a cero, i.e. K 1 ≥ 0 , K 2 ≥ 0 , K 3 ≥ 0 . También recordar que, ya que K ( x )
r
no es simétrico, el espacio principal de K ( x ) no constituye una base ortonormal.
Es interesante constatar que la parte antisimétrica no afecta en la desigualdad de
conducción de calor, ya que:
r
∇ xr T ⋅ K ( x ) ⋅ ∇ xr T ≥ 0
[ ]
∇ xr T ⋅ K sym + K anti ⋅ ∇ xr T ≥ 0
∇ xr T ⋅ K sym
⋅ ∇ xr T + ∇ xr T ⋅ K anti
⋅ ∇ xr T ≥ 0
∇ xr T ⋅ K sym ⋅ ∇ xr T + K anti : (∇ xr T ⊗ ∇ xr T ) ≥ 0

Note que K anti : (∇ xr T ⊗ ∇ xr T ) = 0 ya que el doble producto escalar entre un tensor


antisimétrico ( K anti ) y otro simétrico (∇ xr T ⊗ ∇ xr T ) resulta ser igual a cero, luego:
r
0 ≤ ∇ xr T ⋅ K ( x ) ⋅ ∇ xr T = ∇ xr T ⋅ K sym ⋅ ∇ xr T ≥ 0
r
Es decir, siempre se cumplirá la desigualdad de conducción de calor, sea K ( x ) simétrico o
no.
b) Para el problema planteado, la única ecuación de gobierno que queda es la ecuación de
energía:
Du r r
ρ ≡ ρu& = σ : D − ∇ xr ⋅ q + ρr = −∇ xr ⋅ q
Dt
donde u es la energía interna específica, σ : D es la potencial tensional, y ρr es la fuente
interna de calor por unidad de volumen. Luego:
ρu& = −q i ,i
= −(−K ij T, j ) ,i
= K ij ,i T, j + K ij T, ji
= (∇ xr ⋅ K T ) ⋅ (∇ xr T ) + K : ∇ xr (∇ xr T )
[ ]
= (∇ xr ⋅ K T ) ⋅ (∇ xr T ) + K sym + K anti : ∇ xr (∇ xr T )
= (∇ xr ⋅ K T ) ⋅ (∇ xr T ) + K sym : ∇ xr (∇ xr T ) + K anti : ∇ xr (∇ xr T )
= (∇ xr ⋅ K T ) ⋅ (∇ xr T ) + K sym : ∇ xr (∇ xr T )

Universidad de Castilla- La Mancha Draft Por: Eduardo W. V. Chaves (2012)


Ciudad Real - España
334 PROBLEMAS RESUELTOS DE MECÁNICA DEL MEDIO CONTINUO

donde hemos considerado la simetría de [∇ xr (∇ xr T )]ij = T,ij = T, ji . Si el material es


r
homogéneo eso implica que el campo de K no depende de ( x ) con lo cual el término
K ij ,i = 0 j . En esta situación quedamos con:

ρu& = K sym : ∇ xr (∇ xr T )
Luego, cuando el material es homogéneo la parte antisimétrica de K no afecta en el
resultado.
c) Un material isótropo tiene la característica que sus propiedades en un punto no cambian
si hacemos un cambio de base. Un tensor de segundo orden isótropo es un tensor esférico,
ver capítulo 1, luego el tensor K tiene que ser del tipo: K = K1 , donde K es una constante.
En forma de matriz:
1 0 0
K ij = K 0 1 0
0 0 1

Ejemplo 4.15
Supongamos un medio continuo donde en un punto material hay dos cantidades físicas por
unidad de volumen c s y c f tal que c = c f + c s , ver Figura 4.6, y se cumple también que
r r r
v = v f + v s . Considerando que es un proceso isotérmico, un medio incompresible, que la
propiedad c s no afecta en la velocidad del material f y que el campo c f es homogéneo, y
no hay fuente del material f . Demostrar que:

r ∂c s Ecuación Convección-
Q s − ∇ xr ⋅ (v f c s ) + ∇ xr ⋅ (D ⋅ ∇ xr c s ) = (4.22)
∂t Difusión
r
donde el flujo de la propiedad s viene dado por q ( D ) = −D ⋅ ∇ xr c s .

r
vf
Volumen de control c f

cs
r
v r
dV vs

Figura 4.6: Medio heterogéneo.

Solución:
Partiendo de la ecuación de continuidad de esta cantidad física:

Q=
∂Φ
∂t
+ ∇ xr
r
⋅ (Φv ) ⇒ Q=
∂ (c f + c s ) ∂
∂t
[ r r
+ r (c f + c s )(v f + v s )
∂x
] (4.23)

con Q = Q s + Q f . Luego:

Universidad de Castilla- La Mancha Draft Por: Eduardo W. V. Chaves (2012)


Ciudad Real – España
4 LEYES FUNDAMENTALES DE LA MECÁNICA DEL MEDIO CONTINUO 335

Qs + Q f =
∂ (c f + c s ) ∂
∂t
[ r
+ r (c f + c s )(v f + v s )
∂x
r
]
[ ]
f s
∂ (c + c ) ∂ f r f r r r
⇒ Qs + Q f = + r c v + c f v s + csv f + csv s
∂t ∂x
(4.24)
[ ]
f s
∂c ∂c r r r r
⇒ Qs + Q f = + + ∇ xr ⋅ c f v f + c f v s + c s v f + c s v s
∂t ∂t
 ∂c r  ∂c s
[ ]
f
r r r
⇒ Qs + Q f =  + ∇ xr ⋅ (c f v f ) + + ∇ xr ⋅ (c s v f ) + ∇ xr ⋅ c f v s + c s v s
 ∂t  ∂t

∂c f r
Si suponemos que para el material ( f ) no hay fuente, luego + ∇ xr ⋅ (c f v f ) = 0 , y
∂t
Q f = 0 , que es la ecuación de continuidad de la cantidad c f . Con eso quedamos con:

Qs =
∂c s
∂t
r
[ r r
+ ∇ xr ⋅ (c s v f ) + ∇ xr ⋅ c f v s + c s v s ] (4.25)

∂c s r r r
⇒ Qs = + ∇ xr ⋅ (c s v f ) + ∇ xr ⋅ (c s v s ) + ∇ xr ⋅ (c f v s ) (4.26)
∂t
∂c s r r r r
⇒ Qs = + ∇ xr ⋅ (c s v f ) + ∇ xr ⋅ (c s v s ) + ∇ xr c f ⋅ v s + c f ∇ xr ⋅ v s (4.27)
∂t
r
Si la cantidad física c f no cambia con x , campo homogéneo, luego el gradiente
r r
∇ xr c f = 0 . Si además, para el medio ( s ) lo consideramos incompresible ∇ xr ⋅ v s = 0 . Estas
simplificaciones nos indican que el material ( s ) no afecta en las propiedades y ni en el
campo de velocidad del material ( f ). Lógicamente si la cantidad del material ( s ) es
significativa esta aproximación ya no será válida. Con estas aproximaciones quedamos con:
∂c s r r ∂c s r r
Qs = + ∇ xr ⋅ (c s v f ) + ∇ xr ⋅ (c s v s ) = + ∇ xr ⋅ (c s v f ) + ∇ xr ⋅ q ( D ) (4.28)
∂t ∂t
r r
Observemos que el término (c s v s ) ≡ q ( D ) representa el flujo debido a la concentración del
r r
material ( s ), término difusivo. El término (c s v f ) ≡ q (C ) está relacionado con transporte de
r
masa, término convectivo. Teniendo en cuenta que q ( D ) = −D ⋅ ∇ xr c s la expresión (4.28)
queda:
∂c s r r
Qs = + ∇ xr ⋅ (c s v f ) + ∇ xr ⋅ q ( D )
∂t
∂c s r
⇒ Qs = + ∇ xr ⋅ (c s v f ) + ∇ xr ⋅ (−D ⋅ ∇ xr c s ) (4.29)
∂t
r ∂c s
⇒ Q s − ∇ xr ⋅ (c s v f ) + ∇ xr ⋅ (D ⋅ ∇ xr c s ) =
∂t
Demostrando así (4.22).

Universidad de Castilla- La Mancha Draft Por: Eduardo W. V. Chaves (2012)


Ciudad Real - España
336 PROBLEMAS RESUELTOS DE MECÁNICA DEL MEDIO CONTINUO

Ejemplo 4.16
Considérese un tanque lleno de agua
con concentración de sedimentos, ver
Figura 4.7. La concentración de
sedimentos (densidad de
concentración) viene dada por
( − kx t ) h
c( x3 , t ) = C t exp 3 , por unidad de

volumen, donde C y k son constantes


positivas. Se pide: a) Obtener la masa
total de sedimentos en el tanque; b)
Obtener el flujo de sedimentos
sabiendo que éste es solo función de
r r
x3 y del tiempo t , i.e. q = q( x3 , t ) . x3 x2
b

x1

Figura 4.7: Tanque con sedimentos.


Solución:
Para obtener la masa total tenemos que resolver la integral:
h b a h
( − kx3t ) ( − kx3t )

M = c s dV =
V
∫∫∫
0 0 0
C t exp ∫
dx1 dx 2 dx3 = ab C t exp
0
dx3

h
− C
= ab 
 k
exp
( − kx3t ) 


− C
= ab 
 k
C  − abC
exp ( − kht ) +  =
k k
exp ( − kht ) − 1 [ ]
0

Para obtener el flujo, podemos aplicar la ecuación de continuidad de la concentración:


∂c s r r ∂c s
Q= + ∇ xr ⋅ q ⇒ ∇ xr ⋅ q = q i ,i = − (4.30)
∂t ∂t
donde hemos tenido en cuenta que no hay fuente de sedimentos Q = 0 . Para este problema
el flujo no es dependiente de x 2 y x1 . Con estas condiciones q1,1 = q 2, 2 = 0 . Luego:

∂q1 ∂q 2 ∂q 3 ∂c s
q i ,i = q1,1 + q 2, 2 + q 3,3 = + + =−
∂x1 ∂x 2 ∂x3 ∂t
(4.31)
∂q ∂c s
⇒ 3 =−
∂x3 ∂t

donde
∂c s ∂
∂t
=
∂t
[
C t exp
( − kx3t )
= C exp]( − kx3t )
− C t k x 3 exp
( − kx3t )
y reemplazando en la
expresión (4.31) obtenemos que:

Universidad de Castilla- La Mancha Draft Por: Eduardo W. V. Chaves (2012)


Ciudad Real – España
4 LEYES FUNDAMENTALES DE LA MECÁNICA DEL MEDIO CONTINUO 337

dq 3 ∂c s ( − kx3t ) ( − kx3t )
=− = −C exp + C t k x3 exp
dx3 ∂t

∫ [− C exp ]dx
( − kx3t ) ( − kx3t )

⇒ dq 3 = + C t k x3 exp 3
(4.32)
C ( − kx3t ) C ( − kx3t ) C k x3 t ( − kx3t )
⇒ q 3 = exp − exp − exp + K3
kt kt kt
( − kx3t )
⇒ q 3 = −C x3 exp + K3
{
=0

r ( − kx3t )
El vector flujo viene dado por q = −C x3 exp eˆ 3 .

Universidad de Castilla- La Mancha Draft Por: Eduardo W. V. Chaves (2012)


Ciudad Real - España
338 PROBLEMAS RESUELTOS DE MECÁNICA DEL MEDIO CONTINUO

4.1.2 Movimiento de Sólido Rígido

Ejemplo 4.17
Obtener el momento lineal y el momento angular para un sólido sometido a un
movimiento de cuerpo rígido.
x3′
Sólido rígido
Bt
C.M .
x2′
r
r v
x3 x
x1′

O
x2
x1

Figura 4.8
Solución:
Para un movimiento de cuerpo rígido todas las partículas estarán sometidas a una misma
r
velocidad v . En el capítulo 2 obtuvimos que la velocidad para un medio continuo
sometido a un movimiento de cuerpo rígido viene dada por:
r r r r r
v = c& + ω ∧ ( x − c )
r
donde ω es el vector axil (velocidad angular) asociado al tensor antisimétrico W (tensor
spin).
Momento lineal:
r r
L = ∫ ρ v dV
V
r r
( r r
= ρ c& + ω ∧ ( x − c ) dV
∫ )
V
r r r r r
= ρ c& dV + ρ ω ∧ x dV − ρ ω ∧ c dV
∫ ∫ ∫
V V V
En notación indicial:
r& r
( r r
∫ ρ c + ω ∧ ( x − c) ) i ∫ ∫
dV = ρ c i dV + ρ  ijk ω j x k dV − ρ  ijk ω j c k dV ∫
V V V V

∫ ∫
= c i ρ dV +  ijk ω j ρ x k dV −  ijk ω j c k ρ dV
V V

V

= mc i +  ijk ω j m x k −  ijk ω j c k m
[
= m c i +  ijk ω j ( x k − c k ) ]
r r r  r  r r
donde notamos que:
V
∫ ρ ω ∧ x dV = ω ∧  ∫ ρ x dV  = ω ∧ (m x ) , siendo m la masa total y
V 
r
x k el vector posición del centro de masa (C.M.). Así obtenemos que:

Universidad de Castilla- La Mancha Draft Por: Eduardo W. V. Chaves (2012)


Ciudad Real – España
4 LEYES FUNDAMENTALES DE LA MECÁNICA DEL MEDIO CONTINUO 339

r r r r r
L = m c& + ω ∧ ( x − c )
r
[
(Momento lineal de sólido rígido)
] (4.33)
= mv
r r r r r
donde v = c& + ω ∧ ( x − c ) es la velocidad del centro de masa.

Momento angular:
r r r

H O = ( x ∧ ρv ) dV
V

∫ [x ∧ ρ (c + ω ∧ ( x − c))]dV
r r& r r r
=
V
r r r r r r r r (4.34)
= ρ x ∧ c& dV + ρ x ∧ (ω ∧ x ) dV − ρ x ∧ (ω ∧ c ) dV
∫ ∫ ∫
V V V

 r  r r r r  r  r r
=  ρ x dV  ∧ c& + ρ x ∧ (ω ∧ x ) dV −  ρ x dV  ∧ (ω ∧ c )
∫ ∫ ∫
V  V V 
r r r
Del capítulo 1 obtuvimos que: dados tres vectores a , b , c la siguiente relación se cumple
r r r r r r r r r r r r r
a ∧ (b ∧ c ) = (a ⋅ c )b − (a ⋅ b)c . Luego, a ∧ (b ∧ a) = [(a ⋅ a) 1 − (a ⊗ a) ]⋅ b , si a = c . Con
r r r r r r
r r r r r r r r r
es ρ x ∧ (ω ∧ x ) dV = ρ [( x ⋅ x )ω − ( x ⋅ ω ) x ] dV y en notación indicial:
∫ ∫
V V

∫ ρ [x
V
k ] ∫ [
x k ω i − x p ω p x i dV = ρ x k x k ω p δ pi − x p ω p x i dV
V
]
= ∫ ρ [x k ]
x k δ pi − x p x i ω p dV
V

∫ [ ]
= ρ x k x k δ pi − x p x i dV ω p
V
= I O ip ω p
En notación tensorial queda:
r r r  r r r r  r
∫ ρ x ∧ (ω ∧ x ) dV = ∫ ρ [( x ⋅ x ) 1 − ( x ⊗ x )] dV  ⋅ ω
V V 
r
= IO ⋅ ω
r r r r
donde I O = ∫ ρ [( x ⋅ x ) 1 − ( x ⊗ x )] dV es el tensor de inercia relativo al punto O . Como
V
podemos observar I O es un pseudo-tensor de segundo orden y simétrico. Es un pseudo-
tensor porque depende del sistema de referencia adoptado. Aunque sea un pseudo-tensor,
por comodidad nos vamos a referir simplemente por tensor de inercia. Las, cuyas
componentes son I O ij = ∫ ρ [x k x k δ ij − xi x j ] dV , y explícitamente:
V

I O 11 = ρ [( x1 x1 + x 2 x 2 + x 3 x 3 )δ 11 − x1 x1 ] dV = ρ x 22 + x 32 dV

V
∫ [
V
]
∫ [
I O 22 = ρ x12 + x 32 dV
V
]
∫ [
I O 33 = ρ x12 + x 22 dV
V
]

Universidad de Castilla- La Mancha Draft Por: Eduardo W. V. Chaves (2012)


Ciudad Real - España
340 PROBLEMAS RESUELTOS DE MECÁNICA DEL MEDIO CONTINUO

I O 12 = ρ [( x1 x1 + x 2 x 2 + x 3 x 3 )δ 12 − x1 x 2 ] dV = − ρ [x1 x 2 ] dV
∫ ∫
V V

I O 13 = − ρ [x1 x 3 ] dV

V

I O 23 = − ρ [x 2 x 3 ] dV

V
donde I O 11 , I O 22 , I O 33 , son conocidos como momentos de inercia, y I O 12 , I O 13 , I O 23 , son
conocidos como productos de inercia en la mecánica clásica.
Retomando la ecuación (4.34) podemos decir que:
r  r  r r r r  r  r r
H O =  ρ x dV  ∧ c& + ρ x ∧ (ω ∧ x ) dV −  ρ x dV  ∧ (ω ∧ c )
∫ ∫ ∫
V  V V 
r r& r r r r
= m x ∧ c + I O ⋅ ω − m x ∧ (ω ∧ c )
r r r r
(
= m x ∧ c& − ω ∧ c + I O ⋅ ω
r r r
r
)
Sumando y restando el término m x ∧ ω ∧ x en la relación anterior obtenemos:
r r r r r
(
H O = m x ∧ c& − ω ∧ c + I O ⋅ ω
r
)
r r r
[r r r r r
]
= m x ∧ c& + ω ∧ ( x − c ) − m x ∧ ω ∧ x + I O ⋅ ω
r
r r r r
[
r r r
= m x ∧ v − m ( x ⋅ x) 1 − ( x ⊗ x) ⋅ ω + IO ⋅ ω
r
]
r r r r
{[
r r
= m x ∧ v + m ( x ⊗ x) − ( x ⋅ x) 1 + IO ⋅ ω
r r r r r
r
] }
r
= m x ∧ v + I ⋅ω = m x ∧ v + HG
Definiendo así el Momento angular con respecto al punto O :
r r r r Momento angular con respecto al sistema
HO = m x ∧ v + HG (4.35)
que pasa por O
r r r r
NOTA: Verifiquemos que cuando c = 0 tenemos que H O = I O ⋅ ω .
Definimos también el tensor de inercia con respecto al centro de masa:
r r
[ r r
I = IO + m ( x ⊗ x) − ( x ⋅ x) 1 ]
Tensor de inercia con respecto al centro de
masa
(4.36)

Las componentes de Iij = I O ij + m[x i x j − ( x12 + x 22 + x 32 )δ ij ] vienen dadas explícitamente


por:
I11 = I O 11 − m( x 22 + x 32 ) ; I12 = I O 12 + m( x1 x 2 )
Teorema de los ejes
I 22 = I O 22 − m( x12 + x 32 ) ; I 23 = I O 23 + m( x 2 x 3 ) (4.37)
paralelos
I33 = I O 33 − m( x12 + x 22 ) ; I13 = I O 13 + m( x1 x3 )
Observemos que las expresiones anteriores constituyen el teorema de los ejes paralelos
(Teorema de Steiner) de la Mecánica Clásica.

Ejemplo 4.18
Obtener el principio de la conservación del momento lineal y del momento angular para un
sólido sometido a un movimiento de cuerpo rígido.
Solución: Para un movimiento de cuerpo rígido, ver Figura 4.8, todas las partículas estarán
r
sometidas a una misma velocidad v . Partiendo de la definición de principio de la
conservación del momento lineal:
r r D r r&

Sσ V

t * dS + ρ b dV =
Dt V
ρ v dV = L∫

Universidad de Castilla- La Mancha Draft Por: Eduardo W. V. Chaves (2012)


Ciudad Real – España
4 LEYES FUNDAMENTALES DE LA MECÁNICA DEL MEDIO CONTINUO 341

Recurrimos ahora la expresión del momento lineal para un movimiento de sólido rígido
r r
obtenido en el ejemplo anterior como L = m v , luego:
r r D r r& r& r



t * dS + ρ b dV =
Dt V ∫
ρ v dV = L = m v = m a
14442V 444 3
r
∑F
Resultando:
r r
∑F = ma
Consideremos ahora el principio de la conservación del momento angular:
r r* r r D r r D r r&
∫ (x ∧ t


)dS + ( x ∧ ρb)dV =
Dt V
( x ∧ ρ v )dV =
Dt ∫
HO ≡ HO
1444442V44444 3
r
∑ MO
Resultando:
r r& r r&
∑M O = H O , también es válido que ∑M G = HG
r
donde la expresión del momento angular H O para un movimiento de sólido rígido fue
r r
obtenida en el ejemplo anterior. Los conjuntos de ecuaciones anteriores, ∑ F=ma y
r r&
∑ M G = H G , nos informan que los siguientes sistemas son equivalentes:
r&
r r HG
F(n ) F( 2 )

G G
= r
ma

r
F(1) G - centro de masa

NOTA: Cuando estamos tratando de un movimiento de sólido rígido, las ecuaciones de


gobierno del problema se resumen a:
r r r r& Ecuaciones de gobierno para
∑F =ma y ∑M O = HO
movimiento de sólido rígido
(4.38)

Universidad de Castilla- La Mancha Draft Por: Eduardo W. V. Chaves (2012)


Ciudad Real - España
342 PROBLEMAS RESUELTOS DE MECÁNICA DEL MEDIO CONTINUO

Ejemplo 4.19
Consideremos una viga con las siguientes condiciones de carga y de apoyo:
P P

α α
= HA A B
A B

L L VA VB
2 2

Figura 4.9: Viga isostática.


Obtener las reacciones de apoyo VA , VB , H A .
Solución:
Aunque en la viga haya deformación (régimen de pequeñas deformaciones) y tensión, para
efecto de cálculo de las reacciones de apoyo de una viga isostática podemos considerar que
de un cuerpo rígido se tratara, y las ecuaciones necesarias, ver ecuaciones (4.38), son:
 ∑F =0
∑F
x
r r r   = H A + P cos α = 0 ⇒ H A = − P cos α
∑ ∑F
x
F=ma =0 ⇒  =0 ⇒ 
∑F
y
  = V A + V B − P sin α = 0 ⇒ V A = −V B + P sin α
∑F
y
 z =0

r


∑M x =0
r r&  L P sin α
∑ MA = HA =0 ⇒  ∑M y =0 ⇒ ∑M


z = V B L − P sin α
2
= 0 ⇒ VB =
2

 ∑M z =0
P sin α
pudiendo ahora determinar V A = −V B + P sin α = . Notar que teníamos 3 ecuaciones
2
y tres incógnitas. Si tenemos un sistema donde hay más incógnitas que ecuaciones, sistema
hiperestático, este procedimiento ya no es válido ya que las reacciones dependerán de la
deformada de la viga y ésta a su dependerán de la rigidez de la viga.

Ejemplo 4.20
Obtener la energía cinética para un sólido sometido a un movimiento de cuerpo rígido, ver
Figura 4.8.

Solución:
Para un movimiento de cuerpo rígido todas las partículas estarán sometidas a una misma
r r r r r
velocidad v = c& + ω ∧ ( x − c ) . Luego la energía cinética viene dada por:
1
K (t ) =
2V
r r
ρ (v ⋅ v )dV =

1
2V
r r r r r r
{[ r r
ρ c& + ω ∧ ( x − c) ⋅ c& + ω ∧ ( x − c ) dV
∫ ][ ]}
r r r r
Haciendo la siguiente suma de vectores x = x + x ′ , donde x es el vector posición del
r
centro de masa, y x ′ es el vector posición de las partículas del sólido con respecto al
sistema que pasa por el centro de masa.

Universidad de Castilla- La Mancha Draft Por: Eduardo W. V. Chaves (2012)


Ciudad Real – España
4 LEYES FUNDAMENTALES DE LA MECÁNICA DEL MEDIO CONTINUO 343

K (t ) =
1
2V
r r
{[ r r r r r r r r
][
ρ c& + ω ∧ (( x + x ′) − c) ⋅ c& + ω ∧ (( x + x ′) − c ) dV
∫ ]}
=
1
2V
r r
{[( r r r r r r
) r r r r
] [(
ρ c& + ω ∧ ( x − c ) + (ω ∧ x ′) ⋅ c& + ω ∧ ( x − c ) + (ω ∧ x ′) dV
∫ ) ]}
Resultando así que:
K (t ) =
1
2V
r r
∫ ( r r r r
)(
r r
ρ c& + ω ∧ ( x − c ) ⋅ c& + ω ∧ ( x − c) dV + )
+
1
2V ∫
r r
( r r r r
2ρ c& + ω ∧ ( x − c ) ⋅ (ω ∧ x ′) dV + )
1 r r r r
ρ (ω ∧ x ′) ⋅ (ω ∧ x ′) dV
+ ∫
2V
r r& r r r
Observemos que v = c + ω ∧ ( x − c) es la velocidad del centro de masa, resultando así que:
1 r r 1 r r r 1 r r r r
K (t ) = ρv ⋅ v dV +
∫ 2ρv ⋅ (ω ∧ x ′) dV + ρ (ω ∧ x ′) ⋅ (ω ∧ x ′) dV
∫ ∫
2V 2V 2V
r r r r
A continuación analizaremos separadamente el término ρ (ω ∧ x ′) ⋅ (ω ∧ x ′) dV . ∫
V
r r r r
∫ ρ ( [(ω ∧ x ′) ⋅ (ω ∧ x ′)])
V
i ∫
dV = ρ  ijk ω j x ′k  ipq ω p x q′ dV
V


= ρ (δ jp δ kq − δ jq δ kp )ω j x k′ ω p x ′q dV
V


= ρ (δ jp δ kq ω j x ′k ω p x ′q − δ jq δ kp ω j x k′ ω p x q′ ) dV
V


= ρ (ω j x ′k ω j x ′k − ω j x ′k ω k x ′j ) dV
V


= ρ ω k ( x ′s x ′s δ kp − x ′k x ′j ) ω j dV
V

 
= ω k  ρ ( x ′s x ′s δ kj − x ′k x ′j ) dV ω j

 
V 
= ω k I kj ω j
En notación tensorial:
r r r r r  r r r r  r
∫ ρ [(ω ∧ x ′) ⋅ (ω ∧ x ′)] dV = ω ⋅  ∫ ρ [( x ′ ⋅ x ′) 1 − ( x ′ ⊗ x ′)] dV  ⋅ ω
V V
r r
= ω⋅ I ⋅ω
donde I es el tensor de inercia según el sistema que pasa por el centro de masa.
r r r
Analizamos ahora el término ∫ 2ρv ⋅ (ω ∧ x ′) dV :
V

r r r r r r  r r r
∫ 2ρv ⋅ (ω ∧ x ′) dV = 2v ⋅ ω ∧ ρ ∫ x ′ dV  = 2v ⋅ ω ∧ m {xr′ = 0
V  V  =0
r
Observemos que el sistema x ′ pasa por el centro de masa luego, el vector posición del
r
centro de masa para el sistema x ′ es el vector nulo.
Resultando así que la expresión de la energía cinética para un movimiento de sólido rígido:

Universidad de Castilla- La Mancha Draft Por: Eduardo W. V. Chaves (2012)


Ciudad Real - España
344 PROBLEMAS RESUELTOS DE MECÁNICA DEL MEDIO CONTINUO

1 r r 1 r r r 1 r r r r
K (t ) = ρv ⋅ v dV +
∫ 2ρv ⋅ (ω ∧ x ′) dV + ∫ ρ (ω ∧ x ′) ⋅ (ω ∧ x ′) dV ∫
2V 2 2V
1V44424443
=0

1 1r r
= mv 2 + ω ⋅ I ⋅ ω
2 2
Energía cinética para un
1 1r r
K(t ) = mv 2 + ω ⋅ I ⋅ ω movimiento de sólido (4.39)
2 2 rígido
Representando las componentes del tensor de inercia como:
 
∫ [
 ρ x ′2 + x 3′ dV ] − ρ [x1′ x 2′ ] dV
∫ − ρ [x1′ x 3′ ] dV 

2 2

V   I11 − I12 − I13 


∫ ρ [x ′ + x ′ ] dV
V V
 
Iij =  − ρ [x1′ x 2′ ] dV
∫ 1
2
3
2
− ρ [x 2′ x 3′ ] dV  =  − I12
∫ I 22 − I 23 
 V   − I13 − I 23 I 33 
∫ [ ]
V V
 − ρ [x1′ x 3′ ] dV
∫ − ∫ ρ [x ′ x ′ ] dV 2 3 ρ x1′ + x 2′ dV  
2 2

 V V V 
La forma explícita de la energía cinética queda:
1 1
K(t ) = mv 2 + ω k Ikj ω j
2 2
 I11 − I12 − I13   ω1 
1 1  
= mv 2 + [ω1 ω2 ω 3 ] − I12 I 22 − I 23  ω 2 
2 2 − I
 13 − I 23 I 33  ω 3 

=
1
2
1
[
mv 2 + I11ω12 + I 22 ω 22 + I 33 ω32 − 2 I12 ω1 ω 2 − 2 I13 ω1 ω3 − 2 I 23 ω 2 ω3
2
]
1
K(t ) = mv 2 +
2
1
2
[
I11ω12 + I 22 ω 22 + I 33 ω32 − 2 I12 ω1ω 2 − 2 I13 ω1ω3 − 2 I 23 ω 2 ω3 ] (4.40)

Ejemplo 4.21
Considérese el pseudo-tensor de inercia, I O , con respecto al sistema ortonormal x1 x 2 x3 ,
ver Figura 4.10. a) Hacer la interpretación física del tensor de inercia. b) Dado otro sistema
ortonormal, representado por x1* x 2* x3* , obtener las componentes del tensor de inercia en
este nuevo sistema. c) Demostrar que el tensor de inercia es un tensor definido positivo.
DI O &
Para un sólido en movimiento, ¿en que situaciones la tasa del tensor de inercia, ≡ IO ,
Dt
es cero? x3
x2*
x3*
x1*

O x2

x1
Figura 4.10

Universidad de Castilla- La Mancha Draft Por: Eduardo W. V. Chaves (2012)


Ciudad Real – España
4 LEYES FUNDAMENTALES DE LA MECÁNICA DEL MEDIO CONTINUO 345

Solución:
El pseudo-tensor de inercia depende del sistema de coordenadas adoptado. Y por
definición viene dado por:

∫ [ ]
r r r r
I O = ρ [( x ⋅ x ) 1 − ( x ⊗ x )] dV
∫ ; I O ij = ρ x k x k δ ij − xi x j dV
V V

En componentes
 
∫ [ ]
 ρ x 2 + x 3 dV − ρ [x1 x 2 ] dV
∫ − ρ [x1 x 3 ] dV 

2 2

V 
∫ ρ [x + x ]dV
V V
I ij =  − ρ [x1 x 2 ] dV

2
1
2
3 − ρ [x 2 x 3 ] dV 

 V 
− ∫ ρ [x x ] dV ∫ [ ]
V V
 − ρ [x1 x 3 ] dV
∫ 2 3 ρ x12 + x 22 dV 
 V V V 

a) El tensor de inercia nos dar información de como la masa del sólido está distribuida
según el sistema adoptado.
El término ∫ ρ [x1 x 2 ] dV nos indica como la masa del sólido está distribuida según el plano
V
x1 − x 2 . Luego, Si el material es homogéneo, i.e. campo de densidad de masa constante, y
x1 − x 2 es un plano de simetría, es decir si la masa está distribuida por igual con respecto al
plano x1 − x 2 , el término ∫ ρ [x1 x 2 ] dV es igual a cero. Con eso, concluimos también que si
V
los planos x1 − x 2 , x1 − x 3 , x 2 − x3 , son planos de simetría, la matriz de inercia será una
matriz diagonal.
b) Vamos suponer que los sistemas dados están relacionados por la ley de transformación
xi* = Aij x j , donde Aij es la matriz ortogonal de transformación de base, luego, se cumple
que xi = A ji x *j . Pudiendo así expresar I O ij de la siguiente manera:

∫ [
I O ij = ρ x k x k δ ij − x i x j dV
V
]
∫ [
= ρ ( x k* x k* )Aip δ pq A jq − Aip x *p A jq x q* dV
V
]
∫ {[ ]}
= Aip ρ ( x k* x k* )δ pq − x *p x q* A jq dV
V

 
∫ [ ]
= Aip  ρ ( x k* x k* )δ pq − x *p x q* dV A jq
V 
= Aip I *O ij A jq
r r
Recordar que el término ( x ⋅ x = x k x k = x k* x k* ) es un invariante, i.e. no depende del sistema
adoptado, y también que la matriz de transformación, igual que un tensor ortogonal, sólo
es dependiente, si es el caso, del tiempo.

Abusando un poco de la notación, utilizamos también notación tensorial, pero hay que
tener en cuenta que estamos trabajando con las componentes del tensor, y no haciendo una
transformación ortogonal.

Universidad de Castilla- La Mancha Draft Por: Eduardo W. V. Chaves (2012)


Ciudad Real - España
346 PROBLEMAS RESUELTOS DE MECÁNICA DEL MEDIO CONTINUO

r r r r
I O = ρ [( x ⋅ x ) 1 − ( x ⊗ x )] dV

V

∫ [ ]
r r r r
= ρ ( x * ⋅ x * )A T ⋅ 1 ⋅ A − (A T ⋅ x * ⊗ A T ⋅ x * ) dV
V

∫ [ ]
r r r r
= ρ ( x * ⋅ x * )A T ⋅ 1 ⋅ A − (A T ⋅ x * ⊗ x * ⋅ A ) dV
V

⋅ {ρ [( x * ⋅ x * )1 − ( x * ⊗ x * )]}⋅ A dV
r r r r

= AT
V

 
∫ [ ]
r r r r
= A T ⋅  ρ ( x * ⋅ x * )1 − ( x * ⊗ x * ) dV  ⋅ A
V 
= A T ⋅ I *O ⋅ A

I O = A T ⋅ I *O ⋅ A Componentes del tensor de inercia tras un


(4.41)
I O ij = Aip I *O ij A jq cambio de base (rotación)

Luego, es válido que I *O = A ⋅ I O ⋅ A T , que son las nuevas componentes del tensor de
inercia en el sistema x1* x 2* x3* . Verifiquemos que es la misma ley de transformación de las
componentes de un tensor de segundo orden, donde A es la matriz de transformación del
sistema x1 x 2 x3 al sistema x1* x 2* x3* .
c) Un tensor definido positivo, por definición, sus autovalores son mayores que cero.
Partiremos de la expresión de la energía cinética obtenida en el Ejemplo 4.20:
1
K(t ) = mv 2 +
2
1
2
[
I11ω12 + I 22 ω 22 + I 33 ω32 − 2 I12 ω1ω 2 − 2 I13 ω1ω3 − 2 I 23 ω 2 ω3 ]
La energía cinética es un escalar y siempre positivo, sólo en dos situaciones la energía será
igual a cero, cuando no haya masa o cuando el cuerpo esté en reposo. Vamos adoptar un
sistema tal que el origen esté situado en el centro de masa y los ejes adoptados son ejes de
simetría (ejes principales de inercia) y que el cuerpo esté girando alrededor del origen,
centro de masa. En esta situación la expresión de la energía se resume a:
 I1 0 0   ω1 
1
K(t ) = [ω1 ω2 ω3 ]  0 I 2 0  ω 2 

2
 0 0 I 3  ω 3 
1 44424443
Autovalores del
tensor de inercia

=
1
2
[
I1ω12 + I 2 ω 22 + I 3 ω 32 > 0]
1
Si además tenemos un movimiento tal que ω 2 = ω 3 = 0 , nos quedamos con K(t ) = I1ω12 ,
2
luego, la única forma que la energía cinética sea siempre positiva es que I1 > 0 .
Análogamente, podemos concluir que I 2 > 0 , I 3 > 0 . Con eso concluimos que el tensor
de inercia es un tensor definido positivo.
d) Como el tensor de inercia depende del sistema adoptado, en las siguientes situaciones el
tensor de inercia para un sólido en movimiento no cambia con el tiempo:
1) Si el sistema adoptado está unido al sólido.

Universidad de Castilla- La Mancha Draft Por: Eduardo W. V. Chaves (2012)


Ciudad Real – España
4 LEYES FUNDAMENTALES DE LA MECÁNICA DEL MEDIO CONTINUO 347

2) Si el sólido está girando alrededor de un eje de simetría, por ejemplo si un cilindro está
girando alrededor del eje prismático, luego, durante el movimiento la distribución de masa,
con respecto a los ejes adoptados, no cambia con el tiempo:

r
ω sistema de referencia fijo en el
espacio

Ejemplo 4.22
Considérese un cilindro homogéneo de radio r y altura h = 3r con masa total igual a m .
Encontrar el tensor de inercia en el sistema Ox1′ x 2′ x3′ . El sistema Ox1′ x 2′ x3′ viene dado por
una rotación del sistema Ox1′′x 2′′ x3′′ de 45º a lo largo del eje x1′′ . Los sistemas Gx1 x 2 x3 y
Ox1′′x 2′′ x3′′ tienen las mismas orientaciones.

x3

x3′′
x3′ G x2
x1 h = 3r

r x2′
rG

45º
O x2′′

x1′′, x1′

Datos: Para el sistema de referencia Gx1 x 2 x3 se conoce el tensor de inercia y viene dado
por:
1 2 2 
12 m(3r + h ) 0 0 
 1  mr 2 2 0 0
I G ij = 0 2 2
m(3r + h ) 0 = 0 2 0
 12  2  
 1 2 2 

 0 0 1 
 0 0 m ( 3r + h ) 
12
Solución:

Universidad de Castilla- La Mancha Draft Por: Eduardo W. V. Chaves (2012)


Ciudad Real - España
348 PROBLEMAS RESUELTOS DE MECÁNICA DEL MEDIO CONTINUO

Primero obtenemos el tensor de inercia en el sistema Ox1′′x ′2′ x3′′ a través del teorema de
Steiner, ver ecuación (4.37) del Ejemplo 4.17. Después aplicamos una rotación al tensor
según ecuación (4.41) del Ejemplo 4.21.
A través de las ecuaciones (4.37) podemos decir que:
I11 + m( x 22 + x 32 ) = I ′O′ 11 ; I12 − m( x1 x 2 ) = I ′O′ 12
I 22 + m( x12 + x 32 ) = I ′O′ 22 ; I 23 − m( x 2 x3 ) = I ′O′ 23 (4.42)
I33 + m( x12 + x 22 ) = I ′O′ 33 ; I13 − m( x1 x 3 ) = I ′O′ 13

donde ( x1 , x 2 , x 3 ) son las coordenadas del centro de masa con respecto al sistema Ox1′′x ′2′ x3′′ .
r 3
Luego, definimos el vector rG = x1eˆ 1′′ + x 2 eˆ ′2′ + x 3 eˆ ′3′ = 0eˆ 1′′ + reˆ ′2′ + reˆ ′3′ . Con lo cual
2
obtenemos que:
 3  17
I ′O′ 11 = I11 + m( x 22 + x 32 ) = mr 2 + m  r 2 + ( r ) 2  = mr 2
 2  4
 3  13
I ′O′ 22 = I 22 + m( x12 + x 32 ) = mr 2 + m 0 2 + ( r ) 2  = mr 2
 2  4
1
[
I ′O′ 33 = I33 + m( x12 + x 22 ) = mr 2 + m 0 2 + r 2 = mr 2
2
] 3
2
I ′O′ 12 = I12 − m( x1 x 2 ) = 0
I ′O′ 23 = I 23 − m( x 2 x3 ) = 0
3 3
I ′O′ 13 = I13 − m( x1 x3 ) = m(r )( r ) = mr 2
2 2
Resultando así en las siguientes componentes del tensor de inercia en el sistema Ox1′′x ′2′ x3′′ :
34 0 0 
mr 2 
I O′′ ij = 0 13 − 6
4 
 0 − 6 6 

Teniendo en cuenta la matriz de transformación entre los sistemas Ox1′′x 2′′ x3′′ y Ox1′ x 2′ x3′ :
1 0 0 
A = 0 cos 45º sin 45º 

0 − sin 45º cos 45º 

y aplicando la relación (4.41) obtenemos que:


34 0 0 
mr 2 
I O′ ij = A I ′O′ A = Aip I ′O′ ij A jq
T
=  0 7 − 7 
8
 0 − 7 31 

Ejemplo 4.23
r r r r r r r
Teniendo en cuenta el momento angular H O = m x ∧ v + I ⋅ ω = m x ∧ v + H G , encontrar la
tasa del momento angular de tal forma que no tenga la necesidad de calcular en cada
instante de tiempo el tensor de inercia.

Universidad de Castilla- La Mancha Draft Por: Eduardo W. V. Chaves (2012)


Ciudad Real – España
4 LEYES FUNDAMENTALES DE LA MECÁNICA DEL MEDIO CONTINUO 349

r
HG r
ω
x3′
x2*
r x3*
HO x1*
x3
G x 2′
r
x

O x2 x1′
G - centro de masa

x1
Figura 4.11
Solución:
Recurriendo a la derivada material podemos decir que:
D r
Dt
r&
HO ≡ HO =
D
Dt
[
r r r
m x ∧ v + HG ]
=
D
Dt
[ r r
m x ∧v +
D r
Dt
]
HG [ ]
r r
Dx r r Dv r&
=m ∧v +m x ∧ + HG
Dt Dt
r r r r r&
= m v12
∧3v + m x ∧ a + HG
r
=0

Luego, obtenemos que:


D r r& r r r&
HO ≡ HO = m x ∧ a + HG (4.43)
Dt
r r
donde a es la aceleración del centro de masa. A continuación analizamos el término H& G .
Adoptamos un sistema móvil x1′ x ′2 x3′ pero con orientación fija y siempre paralelo al sistema
r
fijo en el espacio x1 x 2 x3 , ver Figura 4.11. Expresando las componentes de I y ω en el
sistema x1′ x ′2 x3′ , obtenemos que:
r r D r r& & r r&
H G′ = I ′ ⋅ ω ′ tasa
→
 H G′ ≡ H G′ = I ′ ⋅ ω ′ + I ′ ⋅ ω ′
Dt
r
Fijemos que como el sólido gira con respecto al sistema x ′ , con lo cual la distribución de
r
masa, con respecto al sistema x ′ , cambia, y a su vez el tensor de inercia también cambia.
Luego, a cada instante de tiempo tenemos que calcular el tensor de inercia. Este
procedimiento es muy costoso. Para solventar este problema adoptamos un nuevo sistema
r
x * , que también tiene origen en el centro de masa, ver Figura 4.11. A través de ley de
transformación de las componentes de los tensores, las siguientes relaciones son válidas:

Universidad de Castilla- La Mancha Draft Por: Eduardo W. V. Chaves (2012)


Ciudad Real - España
350 PROBLEMAS RESUELTOS DE MECÁNICA DEL MEDIO CONTINUO

r r r r
 H G* = A ⋅ H G′ ; H G′ = A T ⋅ H G*
 r r r r
Componente s ω * = A ⋅ ω ′ ; ω′ = A T ⋅ ω*
 *
I O = A ⋅ I O′ ⋅ A I O′ = A T ⋅ I O* ⋅ A
T
;
r r
donde A es la matriz de transformación del sistema x ′ al sistema x * .
r r
La tasa de H G′ = A T ⋅ H G* resulta:
D r
Dt
r&
H G′ ≡ H G′ =
D
Dt
[ r
] r r&
A T ⋅ H G* = A& T ⋅ H G* + A T ⋅ H G* (4.44)

Haciendo una analogía con la tasa de un tensor ortogonal, ver capítulo 2 del libro texto,
podemos decir que Ω = A& ⋅ A T ⇒ A& T = A T ⋅ Ω T , donde Ω T es un tensor antisimétrico
r r
y representa el tensor tasa de rotación del sistema x * con respecto al sistema x ′ . Pudiendo
así expresar (4.34) como:
r& r r&
H G′ = A T ⋅ Ω T ⋅ H G* + A T ⋅ H G*
r r& (componentes) (4.45)
= A T ⋅ Ω T ⋅ H G* + H G* 
 
r r r r
Recurriendo a la propiedad del tensor antisimétrico tal que Ω T ⋅ H G* = ϕ ∧ H G* , donde ϕ
r r
es el vector axil asociado al tensor antisimétrico Ω T , es decir, ϕ = ϕ (t ) es la velocidad
r
angular del sistema móvil x * . Resultando que (4.45) aún puede ser escrito como:
r& r r&
H G′ = A T ⋅ Ω T ⋅ H G* + H G* 
 
r r r (componentes) (4.46)
&
= A T ⋅ ϕ * ∧ H G* + H G* 
 

donde
r*
r& *
HG =
Dt
[
D * r * DI * r *
I ⋅ω =
Dt
] * Dω
⋅ω + I ⋅
Dt
DI *
Para que el término sea igual a cero, podemos tener dos posibilidades:
Dt
DI * r r r
1) = 0 si el sistema x * está unido al sólido. En este caso se cumple que ϕ = ω , es
Dt
decir, la velocidad del sistema móvil es igual a la velocidad angular del sólido.
DI *
2) = 0 si el sólido gira al rededor de un eje prismático, ver Ejemplo 4.21.
Dt

Universidad de Castilla- La Mancha Draft Por: Eduardo W. V. Chaves (2012)


Ciudad Real – España
4 LEYES FUNDAMENTALES DE LA MECÁNICA DEL MEDIO CONTINUO 351

4.2 Ejercicios Propuestos

Problema 4.1

Las ecuaciones obtenidas a través de las leyes fundamentales de la mecánica del medio
continuo se pueden resumir de la siguiente manera:
Dρ r
Principio de la conservación de la masa + ρ (∇ ⋅ v ) = 0 (47)
Dt
Principio de la conservación del r r
∇ ⋅ σ + ρb = ρv& (48)
momento lineal
Principio de la conservación del
σ = σT (49)
momento angular
Principio de la conservación de la r
ρ u& = σ : D − ∇ ⋅ q + ρr (50)
Energía
Principio de la Irreversibilidad r 1 1 1 r
ρη& ( x , t ) + σ : D − ρ u& − 2 q ⋅ ∇ xr T ≥ 0 (51)
(Desigualdad de Entropía) T T T
r
donde ρ es la densidad de masa, v es la velocidad, σ es el tensor de tensiones de Cauchy,
r
b son las fuerzas másicas, u es la energía interna específica o densidad de energía interna
por unidad de masa, η es la densidad de entropía por unidad de masa y por unidad de
r r
tiempo, T es la temperatura, q( x , t ) es el flujo de calor o vector del flujo no convectivo,
r
D es el tensor velocidad de deformación, r ( x , t ) es una función escalar que describe en
forma espacial el calor generado por las fuentes internas por unidad de masa y por unidad
de tiempo.
Se pide:
a) Simplificar las ecuaciones anteriores (47) a (51) para el caso ESTÁTICO, PROCESO
ISOTÉRMICO Y ADIABÁTICO.
b) Hacer un estudio del planteamiento del problema si el problema está bien planteado. En
caso contrario, ¿qué ecuaciones deben ser adicionadas al problema para que esté bien
planteado?
c) Escribir en notación indicial las ecuaciones (47) a (51).

Problema 4.2
Citar los principios fundamentales de la Mecánica del Medio Continuo. De cada principio,
¿qué ecuaciones se obtienen?

Universidad de Castilla- La Mancha Draft Por: Eduardo W. V. Chaves (2012)


Ciudad Real - España
352 PROBLEMAS RESUELTOS DE MECÁNICA DEL MEDIO CONTINUO

Problema 4.3

Universidad de Castilla- La Mancha Draft Por: Eduardo W. V. Chaves (2012)


Ciudad Real – España
5 Introducción a las
Ecuaciones Constitutivas
y PVCI
5.1 Ejercicios Resueltos

Ejemplo 5.1
Para un material termoelástico simple ¿cuáles son las ecuaciones constitutivas y variables
libres si tenemos en cuenta la energía libre de Helmholtz ψ ?
Solución:
Las ecuaciones constitutivas para un material simples están en función de las siguientes
variables libres:
ψ = ψ(F , T )
∂ψ ( F , T )
P(F , T ) = ρ 0
∂F
∂ψ ( F , T )
η(F , T ) = −
∂T
r r
q 0 = q 0 ( F , T , ∇ Xr T )

(ver libro texto). También se pueden presentar en función de las siguientes variables
ψ = ψ(F , T )
ψˆ = ψ ( E , T )
∂ψ ( F , T )
∂ψ ( E , T ) σ =ρ ⋅FT
S = ρ0 ∂F
∂E ∂ψ ( F , T )
; η(F , T ) = −
∂ψ ( E , T )
η( E , T ) = − ∂T
∂T r −1 r
r r q = J q 0 ( F , T , ∇ Xr T ) ⋅ F T
qˆ 0 = q 0 ( E , T , ∇ Xr T ) r
= J −1 F ⋅ q 0 ( F , T , ∇ Xr T )

Ejemplo 5.2
Para un determinado material elástico se conoce la expresión de la densidad de energía (por
unidad de volumen), y viene dada por:
354 PROBLEMAS RESUELTOS DE MECÁNICA DEL MEDIO CONTINUO

1
Ψ ( I E , II E ) = (λ + 2µ )I E2 − 2µ II E
2
donde λ , µ son constantes del material. I E = I E (E ) , II E = II E (E ) son los invariantes
principales, el primer y segundo invariante principal del tensor de deformación de Green-
Lagrange respectivamente. ¿Cuales son las ecuaciones constitutivas para este problema?,
justificar. Obtener también las expresiones explícitas de las ecuaciones constitutivas en
función de λ , µ , I E , II E .
Formulario
I E = I E ( E ) = Tr ( E )

II E = II E ( E ) =
1
2
[
( TrE ) 2 − Tr ( E 2 ) ]
∂I E
=1
∂E
∂ II E
= Tr ( E )1 − E T
∂E
Solución:
La expresión de la energía está SOLO en función del tensor de deformación de Green-
Lagrange (grandes deformaciones). Sabemos que las ecuaciones constitutivas son:
ψˆ = ψ ( E , T )
∂ψ ( E , T )
S = ρ0
∂E
∂ψ ( E , T )
η( E , T ) = −
∂T
r r
qˆ 0 = q 0 ( E , T , ∇ Xr T )

Teniendo en cuenta la expresión de la energía dada, concluimos que el problema es


independiente de la temperatura, ya que en la expresión de la energía dada no está en
función de la temperatura. Luego, sólo me quedo con la ecuación constitutiva de la tensión
y que podemos obtener como:
∂ψ ( E ) ∂Ψ ( I E , II E ) ∂Ψ ( I E , II E ) ∂I E ∂Ψ ( I E , II E ) ∂ II E
S = ρ0 = = +
∂E ∂E ∂I E ∂E ∂ II E ∂E
2 
(
=  (λ + 2µ )I E (1) + (− 2µ ) Tr ( E )1 − E T )
2 
Simplificando la expresión anterior, y teniendo en cuenta que E T = E , I E = Tr (E ) ,
obtenemos:
S = λI E 1 + 2µE
Ejemplo 5.3
1
Considerando la energía libre de Gibbs específica G(S , T ) = ψ ( E , T ) −
S : E como
ρ0
ecuación constitutiva de energía, obtener las demás ecuaciones constitutivas para un
material termoelástico simple.
NOTA: Partir del principio de que G(S, T ) no depende del gradiente de la temperatura.

Universidad de Castilla- La Mancha Draft Por: Eduardo W. V. Chaves (2012)


Ciudad Real - España
5 INTRODUCCIÓN A LAS ECUACIONES CONSTITUTIVAS Y PVCI 355

Solución:
Partimos de la forma alternativa de la desigualdad de Clausius-Duhem en la configuración
de referencia:

[ ]
& + T&η − 1 qr ⋅ ∇ r T ≥ 0
S : E& − ρ 0 ψ
T
0 X
(5.1)

Teniendo en cuenta la energía libre de Gibbs específica podemos decir que:


& (E,T ) −
& (S , T ) = ψ 1 & 1
G S:E − S : E&
ρ0 ρ0
& (S , T ) + 1 S& : E + 1 S : E&
& (E,T ) = G
⇒ψ
ρ0 ρ0
y reemplazando en la desigualdad (5.1) obtenemos que:
& 1 & 1  1r
S : E& − ρ 0 G (S , T ) + S:E + S : E& + T&η  − q 0 ⋅ ∇ Xr T ≥ 0
 ρ0 ρ0  T (5.2)
r
& (S , T ) − S& : E − ρ T&η − 1 q
⇒ −ρ 0 G 0 ⋅∇ XT ≥ 0
r
0
T

Notar que S& : E = E : S& se cumple. La desigualdad anterior nos sugiere que para una
variación de la energía libre de Gibbs tenemos que tener las siguientes relaciones:
Deformación para una variación de tensión; Entropía para una variación de temperatura;
Flujo de calor para un gradiente de temperatura.
& (S , T ) puede ser expresado como:
El término G
DG(S, T ) & ∂G(S, T ) & ∂G(S, T ) &
≡ G(S, T ) = :S + T
Dt ∂S ∂T
Y reemplazando en la ecuación (5.2) obtenemos que:
& (S, T ) − E : S& − ρ T&η − 1 q r
− ρ 0G 0 ⋅∇ XT ≥ 0
r
0
T
∂G(S, T ) & ∂G(S , T ) & 1r
⇒ −ρ 0 :S − ρ0 T − E : S& − ρ 0 T&η − q 0 ⋅ ∇ Xr T ≥ 0 (5.3)
∂S ∂T T
 ∂G(S, T )   ∂G(S, T )  1 r
⇒ − ρ 0 + E  : S& − ρ 0  + η T& − q 0 ⋅ ∇ Xr T ≥ 0
 ∂S   ∂T  T

La desigualdad anterior debe cumplir para todo proceso termodinámicamente admisible. Si


r r
tenemos un proceso isotérmico (T& = 0) y adiabático (q 0 = 0) , la desigualdad de entropía
queda:
 ∂G(S , T ) 
− ρ0 + E  : S& ≥ 0 (5.4)
 ∂S 
cuya desigualdad también debe cumplir para todo proceso. Luego si para una dada
variación S& se cumple la desigualdad anterior, podemos aplicar un proceso tal que S& = −S& ,
violando así el principio de la desigualdad de entropía. Luego, la desigualdad anterior nunca
será violada si y solo si
∂G(S, T ) ∂G(S, T )
ρ0 + E =0 ⇒ E = −ρ 0
∂S ∂S

Universidad de Castilla- La Mancha Draft Por: Eduardo W. V. Chaves (2012)


Ciudad Real - España
356 PROBLEMAS RESUELTOS DE MECÁNICA DEL MEDIO CONTINUO

Teniendo en cuenta la ecuación anterior (ecuación constitutiva de deformación) en la


desigualdad (5.3), obtenemos que:
 ∂G(S, T )   ∂G(S , T )  1r
− ρ0 + E  : S& − ρ 0  + η T& − q 0 ⋅ ∇ Xr T ≥ 0
 ∂S   ∂T  T
(5.5)
 ∂G(S, T ) & 1 r
⇒ −ρ 0  + η T − q 0 ⋅ ∇ X T ≥ 0
r
 ∂T  T
r
Si ahora tenemos un proceso donde ∇ Xr T = 0 (campo de temperatura homogéneo), la
desigualdad queda:
 ∂G(S, T ) 
− ρ0 + η T& ≥ 0
 ∂T 
La desigualdad anterior nunca será violada si y solo si
∂G(S, T ) ∂G(S, T )
+η=0 ⇒ η=−
∂T ∂T
Que es la ecuación constitutiva de entropía. Luego, las ecuaciones constitutivas son:
Ecuación constitutiva de energía G(S, T )
∂G(S, T ) ∂g(S , T )
Ecuación constitutiva de deformación E = −ρ 0 =
∂S ∂S
(5.6)
∂G(S , T )
Ecuación constitutiva de entropía η=−
∂T
r r
Ecuación constitutiva de flujo de calor q 0 = q 0 (∇ Xr T )

donde g = −ρ 0 G . Notar que nuestras variables libres son (S , T ) .

Ejemplo 5.4
a) Hacer el planteamiento de las ecuaciones de gobierno para un problema de sólidos con
las siguientes características: Proceso isotérmico y adiabático, régimen de pequeñas
deformaciones, y relación lineal entre tensión y deformación.
b) Una vez establecida la relación lineal entre tensión-deformación, obtener dicha relación
para que cumpla que σ (ε ) sea una función-de-tensores isótropa de valor tensor de
segundo orden.
Solución:
Para un proceso isotérmico y adiabático la temperatura y la entropía no juega ningún papel.
Para un régimen de pequeña deformaciones tenemos que:
r
Tensor de deformaciones: E ≈ e ≈ ε = ∇ sym u
Tensor de Tensiones: P ≈ S ≈ σ
F ≈1 ;ρ ≈ ρ0 ; ∇ Xr ≈ ∇ xr ≈ ∇ , con esta aproximación la densidad de masa deja
de ser incógnita.

Universidad de Castilla- La Mancha Draft Por: Eduardo W. V. Chaves (2012)


Ciudad Real - España
5 INTRODUCCIÓN A LAS ECUACIONES CONSTITUTIVAS Y PVCI 357

Teniendo en cuenta las ecuaciones fundamentales:


Ecuaciones Fundamentales de la Mecánica del Medio Continuo
(Configuración Actual)
Ecuación de continuidad de masa Dρ r
+ ρ (∇ xr ⋅ v ) = 0 (1 ecuación) (5.7)
(Principio de la conservación de la masa) Dt
Ecuaciones del Movimiento r r
(Principio de la conservación del momento ∇ xr ⋅ σ + ρb = ρv& (3 ecuaciones) (5.8)
lineal)
Simetría del tensor de Tensiones de Cauchy
(Principio de la conservación del momento σ = σT (5.9)
angular)
Ecuación de Energía r
ρ u& = σ : D − ∇ xr ⋅ q + ρr (1 ecuación) (5.10)
(Principio de la conservación de la Energía)
Desigualdad de Entropía r 1 1 1 r
ρη& ( x , t ) + σ : D − ρ u& − 2 q ⋅ ∇ xr T ≥ 0 (5.11)
(Principio de la Irreversibilidad) T T T

Solo quedamos con las siguientes ecuaciones:


1) Ecuaciones de Movimiento
r r
∇ ⋅ σ + ρb = ρv&
2) Ecuación de Energía
r r r
ρ 0 u& ( X , t ) = S : E& − ∇ Xr ⋅ q 0 + ρ 0 r ( X , t )
⇒ ρu& = σ : ε&
Du D
o en función de la energía libre de Helmholtz = [ψ + Tη ] = ψ& :
Dt Dt
ρψ& = Ψ
& = σ : ε&

donde Ψ es la densidad de energía. Verificamos a través de la desigualdad de entropía que


es un proceso sin disipación de energía, es decir, toda energía que se almacena debido al
incremento de ε se recuperar con la disminución de ε .
3) De las Ecuaciones Constitutivas (ver Ejemplo 5.1) solo quedamos con:
ψ = ψ (ε )
∂ψ (ε ) ∂Ψ (ε )
S≈σ =ρ = = σ (ε )
∂ε ∂ε
es decir, la energía ( ψ ) y la tensión son funciones solamente de la deformación. Si
∂ψ (ε ) &
calculamos la tasa de la energía libre de Helmholtz ψ& (ε) = : ε , y reemplazamos en la
∂ε
expresión de la energía ρψ& = Ψ
& = σ : ε& , concluimos que:

∂ψ (ε ) & ∂Ψ (ε ) & ∂Ψ (ε )
ρ :ε = : ε = σ : ε& ⇒ σ=
∂ε ∂ε ∂ε

Universidad de Castilla- La Mancha Draft Por: Eduardo W. V. Chaves (2012)


Ciudad Real - España
358 PROBLEMAS RESUELTOS DE MECÁNICA DEL MEDIO CONTINUO

Luego, la ecuación de energía es una ecuación redundante, es decir, si conozco la tensión


puedo conocer la energía y vise-versa. Resumimos así las ecuaciones de gobierno para el
problema propuesto:
Ecuaciones de Movimiento:
r r r &r& (3 ecuaciones)
∇ ⋅ σ + ρb = ρa = ρv& = ρu
Ecuación Constitutiva en Tensión:
∂Ψ (ε ) (5.12)
σ (ε ) = (6 ecuaciones)
∂ε
Ecuaciones Cinemáticas:
r
ε = ∇ sym u (6 ecuaciones)
r
Como incógnitas tenemos: σ (6), u (3), ε (6), un total de 15 incógnitas y 15 ecuaciones
luego, el problemas está bien planteado. Para que el conjunto de ecuaciones en derivada
parciales anteriores tenga solución única es necesario introducir las condiciones de
contorno e inicial, constituyendo así en un Problema de Valor de Contorno Inicial (PVCI). El
problema que acabamos de plantear es el Problema Elástico Lineal que es el tema del próximo
capítulo. Las condiciones de contorno e inicial para este problema son:
Condiciones de contorno en desplazamiento, en S u :
r r r r r r
u( x , t ) =u* ( x , t ) ui ( x, t ) = u i * ( x, t ) (5.13)
Condiciones de contorno en tensiones, en S σ :
r r r r
σ ( x , t ) ⋅ nˆ = t * ( x , nˆ , t ) σ jk n k = t j * ( x, t ) (5.14)
Condiciones iniciales ( t = 0 ):
r r r
u( x , t = 0) = u 0 r r
r r u i ( x , t = 0) = u 0 i ( x )
∂u 0 ( x , t ) r r r r r (5.15)
= u& 0 ( x, t ) = v 0 ( x ) u& 0 i ( x ) = v 0 i
∂t t =0

En el caso de un problema estático o casi-estático, las ecuaciones de Cauchy recaen en las


ecuaciones de equilibrio y las condiciones iniciales son redundantes.

B Sσ
Su
r r
dV t * ( x)
r r
ρb( x )

Figura 5.1: Sólido sobre acciones externas.


En el apartado Serie de Tensores, capítulo 1, hemos visto que podemos aproximar un
tensor a través de la serie:
1 1 ∂σ (ε 0 ) 1 ∂ 2 σ (ε 0 )
σ (ε ) ≈ σ (ε 0 ) + : ( ε − ε 0 ) + (ε − ε 0 ) T : : (ε − ε 0 ) + L
0! 1! ∂ε 2! ∂ε ⊗ ∂ε
∂σ (ε 0 ) 1 ∂ 2 σ (ε 0 )
≈ σ0 + : (ε − ε 0 ) + (ε − ε 0 ) T : : (ε − ε 0 ) + L
∂ε 2 ∂ε ⊗ ∂ε

Universidad de Castilla- La Mancha Draft Por: Eduardo W. V. Chaves (2012)


Ciudad Real - España
5 INTRODUCCIÓN A LAS ECUACIONES CONSTITUTIVAS Y PVCI 359

Considerando el punto de aplicación ε 0 = 0 , y σ (ε 0 ) = σ 0 = 0 , y además teniendo en


cuenta que la relación σ - ε es lineal, podemos despreciar los términos de orden superior,
obteniendo entonces que:
∂σ (ε 0 ) ∂ 2 Ψ e (ε 0 ) ∂σ ij ∂ 2 Ψ e (ε 0 )
σ (ε ) = :ε = : ε = Ce : ε σ ij = ε kl = ε kl = C eijkl ε kl
∂ε ∂ε ⊗ ∂ε ∂ε kl ∂ε ij ∂ε kl

∂ 2 Ψ e (ε 0 )
donde C e = es un tensor de cuarto orden simétrico y es conocido como tensor
∂ε ⊗ ∂ε
constitutivo elástico, que contiene las propiedades mecánicas del material.
Observemos que la energía tiene que ser de orden cuadrática para que la relación σ - ε sea
lineal, ver ecuación (5.12). Utilizamos la expansión en serie para representar la densidad de
energía, obtenemos que:
1 e 1 ∂Ψ e (ε 0 ) 1 ∂ 2 Ψ e (ε 0 )
Ψ e (ε ) = Ψ (ε 0 ) + : (ε − ε 0 ) + ( ε − ε 0 ) T : : (ε − ε 0 ) + L
0! 1! ∂ε 2! ∂ε ⊗ ∂ε
1 ∂ 2 Ψ e (ε 0 )
= Ψ e 0 + σ 0 : (ε − ε 0 ) + (ε − ε 0 ) T : : (ε − ε 0 ) + L
2 ∂ε ⊗ ∂ε
1 ∂ 2 Ψ e (ε 0 )
= ε: :ε
2 ∂ε ⊗ ∂ε
1
= ε : Ce : ε
2
donde también hemos considerado que ε 0 = 0 ⇒ Ψ e0 = 0, σ 0 = 0 .
NOTA 1: Aunque la ecuación de energía es redundante, a la hora de establecer un método
sea analítico o numérico para resolver el problema el punto de partida es a través de
principios energéticos, de ahí la importancia de estudiar la energía de un sistema.
NOTA 2: La simetría de C e se comprueba fácilmente. Presenta simetría menor debido a
la simetría de σ y ε :
σ ij = σ ji ⇒ C eijkl = C ejikl ; ε kl = ε lk ⇒ e
C ijkl e
= C ijlk

y la simetría mayor es debido a que:


e ∂ 2Ψ e ∂ 2Ψ e
C ijkl = = = C eklij
∂ε ij ∂ε kl ∂ε kl ∂ε ij
NOTA 3: Para una mejor ilustración del problema planteado, consideremos un caso
particular (caso unidimensional) donde las componentes del tensor de tensiones y de
deformaciones vienen dadas por:
σ 0 0   ε 0 0
σ ij =  0 0 0 ; ε ij = 0 0 0 ⇒ σ11 = C1111
e
ε11 ⇒ σ = Eε
 0 0 0 0 0 0

En este caso la relación lineal tensión-deformación viene dada por σ = Eε y la densidad de


1 1 ∂ 2 Ψ e ∂σ
energía Ψ e = σε = εEε , y = =E.
2 2 ∂ε∂ε ∂ε

Universidad de Castilla- La Mancha Draft Por: Eduardo W. V. Chaves (2012)


Ciudad Real - España
360 PROBLEMAS RESUELTOS DE MECÁNICA DEL MEDIO CONTINUO

Ψ e (ε) σ(ε)

Ψ σ
Energía almacenada
1
Ψ e = σε
E 2
1
σ0 = 0
ε ε ε ε
ε0 = 0

Figura 5.2: Relación tensión-deformación (caso unidimensional).


NOTA 4: Debemos enfatizar que en el caso de un proceso elástico la ecuación constitutiva
σ (ε ) es únicamente dependiente del valor actual de ε , i.e. es independiente de la historia
de deformación. ■
b) La función-de-tensor σ (ε ) será isótropa si se cumple que:
σ′ij (ε kl ) = σ ij (ε ′kl )

Teniendo en cuenta que la relación entre σ - ε viene dada, en notación indicial, por
e
σ ij (ε ) = C ijkl ε kl , concluimos que:

σ ′ij (ε kl ) = σ ij (ε ′kl ) 
 ⇒ C ′ijkl
e e
= C ijkl
C ′ijkl
e
ε ′kl = C ijkl
e
ε ′kl 

Es decir, el tensor de cuarto orden C e es un tensor isótropo. Un tensor de cuarto orden


e
isótropo simétrico tiene el formato C ijkl = λδ ij δ kl + µ(δ ik δ jl + δ il δ jk ) . Que en notación
tensorial viene dado por C e = λ1 ⊗ 1 + 2µI , donde I ≡ I sym es el tensor identidad
simétrico de cuarto orden, y los parámetros λ y µ son conocidos como las constantes de
Lamé. Como hemos visto en el Capítulo 1, un tensor de cuarto orden simétrico viene dado
en función de dos constantes ( λ , µ ). En el Capítulo 7 del libro de texto (Elasticidad
Lineal) veremos que es posible expresar C e en función de otros parámetros, e.g. ( E , ν ),
( κ , G ), donde E es el módulo de Young, ν es el coeficiente de Poisson, κ es el módulo volumétrico,
y G = µ es el módulo de elasticidad transversal. Los significados físicos de estos parámetros
serán dados en el mismo capítulo 7.
NOTA 5: En la Figura 5.3 se muestra la relación tensión-deformación para un material
isótropo. Es interesante observar que debido a que C e es independiente de la dirección los
tensores σ y ε comparten las mismas direcciones principales. Luego, para un material
isótropo tenemos que σ (ε ) = (λ1 ⊗ 1 + 2µI) : ε = λTr (ε )1 + 2µε :
∂Ψ e (ε )
σ (ε ) = lineal
→ σ (ε ) = C e : ε isótropo
 → σ (ε ) = λTr (ε )1 + 2µε
14424 ∂4
ε3
Elástico

Es interesante comparar con la ecuación constitutiva en tensión del Ejemplo 5.2 donde se
considera grandes deformaciones, pero mantiene una relación lineal entre la tensión y
deformación.

Universidad de Castilla- La Mancha Draft Por: Eduardo W. V. Chaves (2012)


Ciudad Real - España
5 INTRODUCCIÓN A LAS ECUACIONES CONSTITUTIVAS Y PVCI 361

σ ′22 ′
σ11

σ12 x1′

σ′ij = C ′ijkl
e
ε ′kl P

σ′ij = a ip a jq σ pq
ε ′22 ′
ε12 ′
ε11

ε 22 e
σ ij = C ijkl ε kl σ 22
ε12 σ12
ε11 σ11
P P

P
x1
ε ′22

σ′ij′ = C ′ijkl
′ e ε ′kl′
P
′′
ε11
σ ′22

Espacio
principal
P
′′
σ11
e
C ijkl = C ′ijkl
e
= C ′ijkl
′e - Material isótropo

x1′′

Figura 5.3: Relación tensión-deformación material isótropo.

NOTA 6: Denotamos la densidad de energía de deformación complementaria por Ψ e (σ ) el cual es


una función de σ , (ver Figura 5.4), y viene dada por:
1 ∂Ψ (σ 0 ) ∂ 2 Ψ e (σ 0 )
e
1 e 1
Ψ e (σ ) = Ψ (σ 0 ) + : (σ − σ 0 ) + (σ − σ 0 ) : : (σ − σ 0 ) + L
0! 1! ∂σ 2! ∂σ ⊗ ∂σ
1 ∂ 2 Ψ e (σ 0 )
= Ψ 0e + σ 0 : (σ − σ 0 ) + (σ − σ 0 ) : : (σ − σ 0 ) + L
2 ∂σ ⊗ ∂σ
1 ∂ 2 Ψ e (σ 0 ) 1 1 −1
= σ: : σ = σ :D e : σ = σ : C e : σ
2 ∂σ ⊗ ∂σ 2 2
Note que, si estamos tratando con un material elástico lineal se cumple que
Ψ e (σ ) = Ψ e (ε ) .

Universidad de Castilla- La Mancha Draft Por: Eduardo W. V. Chaves (2012)


Ciudad Real - España
362 PROBLEMAS RESUELTOS DE MECÁNICA DEL MEDIO CONTINUO

a) Material elástico lineal. Densidad de energía de deformación complementaria


σ(ε) 1
Ψ e (σ) = σE −1σ
2
σ
Ψ ( ε ) = Ψ ( σ)
e e
1
Densidad de energía - Ψ e (ε) = εEε
2
E
1
σ0 = 0
ε0 = 0 ε ε

b) Material elástico no-lineal.

Densidad de energía de deformación complementaria Ψ e (σ)

σ(ε)

Densidad de energía - Ψ e (ε)


Ψ e (ε) ≠ Ψ e (σ)

σ0 = 0 ε
ε0 = 0 ε

Figura 5.4: Densidad de energía de deformación complementaria (caso unidimensional).


NOTA 7: Notar que

Ψ e (σ) = σε − Ψ e (ε) tensorial


 → Ψ e (σ ) = σ : ε − Ψ e (ε ) = −ρ 0 G(σ ) = g(σ )

donde g(σ ) es la energía libre de Gibbs por unidad de volumen con signo contrario, (ver
ecuaciones en (5.6) del Ejemplo 5.2).

Ejemplo 5.5
En un ensayo de tracción simple los siguientes valores fueron obtenidos para la tensión-
deformación:

Universidad de Castilla- La Mancha Draft Por: Eduardo W. V. Chaves (2012)


Ciudad Real - España
5 INTRODUCCIÓN A LAS ECUACIONES CONSTITUTIVAS Y PVCI 363

Punto σ( Pa ) ε(×10 −3 )
1 6,67 0,667
2 13,3 1,33
3 20 2
4 24 3
5 22 3,6
Determinar el módulo de Young E y los puntos límites.
Solución:
Podemos verificar que los tres primeros puntos mantienen la misma proporcionalidad:
σ (1) σ ( 2 ) σ ( 3) 20
E= = ( 2 ) = ( 3) = = 10 000 Pa = 10 kPa
ε (1)
ε ε 2 × 10 −3
La gráfica tensión-deformación con los puntos dados se puede apreciar en la Figura 5.5. En
esta figura se señalan los punto: σ e - límite elástico; σ Y - punto de fluencia; σ u - punto de
tensión última. σ r - punto de ruptura.

σ(Pa ) 30
σu
25 σY σr
3; 24
σe 3,6; 22
20 2; 20

15
1,33; 13,3
10
0,667; 6,67
5

0 0; 0
0 0,5 1 1,5 2 2,5 3 3,5 4
0, 2% −3
ε(×10 )
Figura 5.5: Curva tensión-deformación.

Ejemplo 5.6
a.1) Escribir la densidad de energía de deformación a.2) y las ecuaciones constitutivas de
tensión de un material elástico linear e isótropo en notación de Voigt, a.2.1) en función de
Eν E
( λ , µ ), a.2.2) y en función de ( E , ν ) donde λ = y µ= . b) Si
(1 + ν)(1 − 2ν ) 2(1 + ν)
expresamos el tensor de deformación ε en notación de Voigt tal que {ε } = L(1) [ ] {u },
obtener la matriz L [ ].
(1)

c) Escribir las ecuaciones de movimiento en notación de Voigt.


Solución:
a.1) La densidad de energía de deformación viene dada por:
1 1 1 1
Ψ e (ε ) = ε : C e : ε = ε : σ = σ : ε = σ ij ε ij
2 2 2 2

Universidad de Castilla- La Mancha Draft Por: Eduardo W. V. Chaves (2012)


Ciudad Real - España
364 PROBLEMAS RESUELTOS DE MECÁNICA DEL MEDIO CONTINUO

donde hemos utilizado que σ = C e : ε . Notar que


σ ij ε ij = σ1 j ε1 j + σ 2 j ε 2 j + σ 3 j ε 3 j
123 123 123
σ11ε11 σ 21ε 21 σ31ε31
+ + +
σ12ε12 σ22ε 22 σ32ε32
+ + +
σ13ε13 σ23ε 23 σ33ε33

luego
1 1
Ψ e (ε ) = σ ij ε ij = (σ11ε11 + σ 22 ε 22 + σ 33 ε 33 + 2σ12 ε12 + 2σ 23 ε 23 + 2σ13 ε13 )
2 2
y
 ε 11 
ε 
 22 
1 1  ε 33  1
Ψ e (ε ) = σ ij ε ij = [σ11 σ 22 σ 33 σ12 σ 23 σ13 ]   = {σ } {ε }
T

2 2 2 ε
 12  2
 2ε 23 
 
 2ε13 
Luego, los tensores σ y ε en notación de Voigt están almacenados como sigue:
 σ11   ε 11 
σ  ε 
 22   22 
σ  ε 
{σ } =  33  ; {ε } =  33 
 σ12   2ε12 
σ 23   2ε 23 
   
 σ13   2ε13 
a.2.1) La ecuación constitutiva de tensión en notación de Voigt queda:
 σ11  λ + 2µ λ λ 0 0 0   ε11 
σ   λ λ + 2µ λ 0 0 0   ε 22 
 22  
 σ 33   λ λ λ + 2µ 0 0 0   ε 33 
σ = C e : ε Voigt
 →  =   ⇒ {σ } = [C ] {ε }
 σ12   0 0 0 µ 0 0   2ε12 
σ 23   0 0 0 0 µ 0  2ε 23 
    
 σ13   0 0 0 0 0 µ   2ε13 
(5.16)
Para mayores detalles de la ecuación anterior ver Ejemplo 1.95 en el capítulo 1, donde
también hemos obtenido que:
1 λ
ε= σ− Tr (σ )1
2µ 2µ ( 2µ + 3λ )
y

Universidad de Castilla- La Mancha Draft Por: Eduardo W. V. Chaves (2012)


Ciudad Real - España
5 INTRODUCCIÓN A LAS ECUACIONES CONSTITUTIVAS Y PVCI 365

 µ +λ λ λ 
 µ ( 2µ + 3λ ) − − 0 0 0
2µ ( 2µ + 3λ ) 2µ ( 2µ + 3λ )
 
 λ µ +λ λ
ε 11   − − 0 0 0  σ 11 

   2µ ( 2µ + 3λ ) µ ( 2µ + 3λ ) 2µ ( 2µ + 3λ )
 σ 22 
ε 22   λ λ µ +λ  
ε 33   − 2µ ( 2µ + 3λ ) − 0 0 0 
2µ ( 2µ + 3λ ) µ ( 2µ + 3λ )   33 
σ
 = 1  σ 
2ε 12   0 0 0 0 0   12  (5.17)
2ε 23   µ  σ 23 
   1  
2ε 13   0 0 0 0 0  σ 13 
µ
 
 1
0 0 0 0 0
 µ 
{ε } = [C ] −1 {σ }
a.2.2) Note que
Eν E E
λ + 2µ = +2 = (1 − ν)
(1 + ν)(1 − 2ν) 2(1 + ν) (1 + ν)(1 − 2ν)
E
λ= (ν )
(1 + ν)(1 − 2ν )
E E (1 − 2ν )
µ= =
2(1 + ν) (1 + ν)(1 − 2ν) 2
Luego, la ecuación (5.16) puede ser reescrita como:
(1 − ν) ν ν 0 0 0 
 σ11   0   ε11 
σ   ν (1 − ν) ν 0 0
 
 22   ν ν (1 − ν) 0 0 0   ε 22 
σ 33  E  (1 − 2ν) ε 
 =  0 0 0 0 0   33  (5.18)
σ (1 + ν )(1 − 2 ν )  2   2ε 12 
 12  (1 − 2ν )
σ 23   0 0 0 0 0  2ε 23 
   2  
 σ13   (1 − 2ν)   2ε 13 
 0 0 0 0 0 
 2 
Note que
Eν E E
λ+µ= + =
(1 + ν)(1 − 2ν) 2(1 + ν) 2(1 + ν)(1 − 2ν)
E  E Eν  E2
µ(2µ + 3λ ) =  2 + 3  =
2(1 + ν )  2(1 + ν) (1 + ν)(1 − 2ν )  2(1 + ν)(1 − 2ν)
λ+µ E 2(1 + ν)(1 − 2ν) 1
= =
µ(2µ + 3λ ) 2(1 + ν)(1 − 2ν) E2 E
λ Eν (1 + ν)(1 − 2ν ) ν
= =
2µ (2µ + 3λ ) (1 + ν)(1 − 2ν) E2 E
1 2(1 + ν) 1
= = 2(1 + ν )
µ E E
Luego, la ecuación (5.17) puede ser reescrita como:

Universidad de Castilla- La Mancha Draft Por: Eduardo W. V. Chaves (2012)


Ciudad Real - España
366 PROBLEMAS RESUELTOS DE MECÁNICA DEL MEDIO CONTINUO

ε 11   1 −ν −ν 0 0  σ 11 
0
  − ν  
ε 22   1 −ν 0 0 0  σ 22 
ε 33  1  − ν −ν 1 0 0 0  σ 33 
 =    (5.19)
2ε 12  E  0 0 0 2(1 + ν ) 0 0  σ 12 
2ε 23   0 0 0 0 2(1 + ν ) 0  σ 23 
    
2ε 13   0 0 0 0 0 2(1 + ν )  σ 13 

b) Según la definición ε ij = 12 (u i , j + u j ,i ) obtenemos que:

 ∂u1 1  ∂u1 ∂u 2  1  ∂u1 ∂u 3  


  +   + 
 ∂x1 2  ∂x 2 ∂x1  2  ∂x3 ∂x1  
 1  ∂u ∂u  ∂u 2 1  ∂u 2 ∂u 3  
ε ij =   1 + 2   + 
 2  ∂x 2 ∂x1  ∂x 2 2  ∂x3 ∂x 2  
 1  ∂u ∂u  1  ∂u 2 ∂u 3  ∂u 3 
  1 + 3 

 +  
 2  ∂x3 ∂x1  2  ∂x3 ∂x 2  ∂x 3 
 ∂u1   ∂ 
   0 0 
∂x1
   ∂x1 
 ∂u 2   0 ∂
0 
 ε11   ∂x 2   
ε   ∂x 2
  ∂ 
 22   ∂u 3   0 0  u1 
 ε 33   ∂x 3
{ε } =   =  ∂u ∂u  =  ∂ ∂
∂x3   
 u 2  ⇒ {ε } = [L(1) ]{u } (5.20)
 2ε12   1 + 2   0  u 
2ε 23   ∂x 2 ∂x1   ∂x 2 ∂x1  3

   ∂u 2 ∂u 3   ∂ ∂ 
 2ε 13   +   0
 ∂x3 ∂x 2   ∂x3 ∂x 2 

 ∂u1 ∂u 3   ∂ ∂ 
 ∂x + ∂x   ∂x 0
 3 1   3 ∂x1 

NOTA: Si adoptamos la notación ingenieril, i.e. x1 = x , x 2 = y , x3 = z , u1 = u , u 2 = v ,


u 3 = w , ε11 = ε x , ε 22 = ε y , ε 33 = ε z , 2ε 12 = γ xy , 2ε 23 = γ yz , 2ε 13 = γ xz , la ecuación anterior
queda:
 ∂u   ∂ 
 ∂x   ∂x 0 0
 ∂v   ∂ 
 εx    0 0
 ε   ∂y   ∂y 
 y   ∂w   ∂ u 
ε    0 0 
{ε } =  z  =  ∂u ∂z ∂v  =  ∂ ∂
∂z   v 
  ⇒ {ε } = [L(1) ]{u }
γ xy   +   0   w
γ yz   ∂ y ∂x   ∂y ∂x  
   ∂v ∂w   ∂ ∂
 γ xz   +  0 
 ∂z ∂y   ∂z ∂y 
 ∂u + ∂w   ∂ 0
∂
 ∂z ∂x   ∂z ∂x 

Universidad de Castilla- La Mancha Draft Por: Eduardo W. V. Chaves (2012)


Ciudad Real - España
5 INTRODUCCIÓN A LAS ECUACIONES CONSTITUTIVAS Y PVCI 367

r r r
c) Teniendo en cuenta las ecuaciones de movimiento ∇ ⋅ σ + ρb = ρv& = ρu && (ver ecuación
(5.12) en notación indicial σ ij , j + ρb i = ρu
&& i y su forma explícita:

σ ij , j + ρb i = σ i1,1 + σ i 2, 2 + σ i 3,3 + ρb i = ρu
&& i
 ∂σ11 ∂σ12 ∂σ13
 + + + ρb1 = ρ&u&1
 ∂x ∂x ∂x
σ11,1 + σ12, 2 + σ13,3 + ρb1 = ρu &&1 1 2 3
  ∂σ 21 ∂σ 22 ∂σ 23
⇒ σ 21,1 + σ 22, 2 + σ 23,3 + ρb 2 = ρu&& 2 ⇒  + + + ρb 2 = ρu&& 2
  ∂ x1 ∂ x 2 ∂ x 3
σ 31,1 + σ 32, 2 + σ 33,3 + ρb 3 = ρu 3
&&
 ∂σ 31 ∂σ 32 ∂σ 33
 + + + ρb 3 = ρu&& 3
 ∂x1 ∂x 2 ∂x3
Luego, si consideramos las componentes del tensor de tensiones en notación de Voigt, la
ecuación anterior queda:

 ∂ σ 
∂ ∂   11 
 0 0 0 
 ∂x1 ∂x 2 ∂x 3  σ 22  ρb  ρu &&1 
 0 ∂ ∂ ∂ σ 
 33  
1
  && 
 0 0    + ρb 2  = ρu 2
∂x 2 ∂x1 ∂x3 σ (5.21)
   12  ρb 3  ρu
&& 3 
 0 ∂ ∂ ∂  σ 
0 0 23
 ∂x 3 ∂x 2 ∂x1  σ 
 13 
[ ] {σ } + {ρb } = {ρu&&}
⇒ L(1)
T

Ejemplo 5.7
Considerando un material elástico lineal homogéneo e isótropo descrito en el Ejemplo 5.4,
obtener las ecuaciones de gobierno de tal forma que resulte en un sistema de tres
ecuaciones y tres incógnitas, a saber: u1 , u 2 , u 3 , (formulación en desplazamientos).
Solución:
Como visto en el Ejemplo 5.4 las ecuaciones de gobierno para un material elástico linear
homogéneo e isótropo en régimen de pequeñas deformaciones son:
Notación tensorial Notación indicial
Ecuaciones de Movimiento: Ecuaciones de Movimiento:
r r &r& (3 ecuaciones)
∇ ⋅ σ + ρb = ρv& = ρu σ ij , j + ρb i = ρu
&& i (3 ecuaciones)
Ecuación Constitutiva en Tensión: Ecuación Constitutiva en Tensión:
σ (ε ) = λTr (ε )1 + 2µε (6 ecuaciones) σ ij = λε kk δ ij + 2µε ij (6 ecuaciones) (5.22)

Ecuaciones Cinemáticas: Ecuaciones Cinemáticas:


r 1  ∂u i ∂u j 
ε = ∇ sym u (6 ecuaciones) ε ij = +  (6 ecuaciones)
2  ∂x j ∂x i 

Resultando en un sistema de 15 ecuaciones y 15 incógnitas.
La divergencia del tensor de tensiones de Cauchy ( ∇ ⋅ σ ) se puede obtener a través de las
ecuaciones constitutivas en tensión, i.e.:

Universidad de Castilla- La Mancha Draft Por: Eduardo W. V. Chaves (2012)


Ciudad Real - España
368 PROBLEMAS RESUELTOS DE MECÁNICA DEL MEDIO CONTINUO

σ ij = λε kk δ ij + 2µε ij
⇒ σ ij , j = (λε kk δ ij + 2µε ij ) , j
⇒ σ ij , j = λ , j ε kk δ ij + λε kk , j δ ij + λε kk δ ij , j + 2µ , j ε ij + 2µε ij , j
{ { { (5.23)
=0 j = 0i =0 j

⇒ σ ij , j = λε kk , j δ ij + 2µε ij , j
⇒ σ ij , j = λε kk ,i + 2µε ij , j
r
Si las propiedades mecánicas λ y µ son constantes en el material, i.e. no varían con x
∂λ ∂µ
(material homogéneo), luego, λ , j ≡ = 0 j y µ, j ≡ = 0 j . Podemos también expresar
∂x j ∂x j
los términos ε kk ,i y ε ij, j en función de los desplazamientos. Para ello utilizamos las
ecuaciones cinemáticas:
1  ∂u i ∂u j  1
ε ij = +
2  ∂x j ∂xi  2
( )
 ≡ u i , j + u j ,i divergenci 1
(
  a → ε ij , j = u i , jj + u j ,ij
2
)

Notar que
∂ 2ui
∂x j ∂x j
r r
[ ]
≡ u i , jj ≡ [∇ ⋅ (∇u)]i ≡ ∇ 2 u i (Laplaciano del vector u )
r

∂ 2u j ∂ 2u j r
u j ,ij ≡ = ≡ u j , ji ≡ [∇ (∇ ⋅ u)]i
∂xi ∂x j ∂x j ∂xi

1  ∂u k ∂u k  ∂u k
ε kk =  +  = gradiente
≡ u k ,k  → ε kk ,i = u k , ki = u j , ji
2  ∂x k ∂x k  ∂x k
Con eso la ecuación (5.23) puede ser reescrita como:

σ ij , j = λε kk ,i + 2µε ij , j ⇒ σ ij , j = λu j , ji + 2µ
1
2
(
u i , jj + u j , ji )
⇒ σ ij , j = (λ + µ)u j , ji + µu i , jj

Reemplazando las ecuaciones anteriores en σ ij , j + ρb i = ρu


&& i (ecuaciones de movimiento),
obtenemos que:
σ ij , j + ρb i = ρ&u& i ⇒ (λ + µ)u j , ji + µu i , jj + ρb i = ρu
&& i

Resultando así en 3 ecuaciones y 3 incógnitas ( u1 , u 2 , u 3 ):

(λ + µ )u j , ji + µu i , jj + ρb i = ρu
&& i
Ecuaciones de Navier (5.24)
r
(λ + µ )[∇ (∇ ⋅ u)] + µ[∇ ⋅ (∇u)] + ρb = ρu
r r &r&

NOTA 1: Las ecuaciones anteriores son conocidas como Ecuaciones de Navier o también
como Ecuaciones de Navier-Lamé. La forma explícita de las ecuaciones (5.24) se presentan
a continuación:
(λ + µ )u j , ji + µu i , jj + ρb i = (λ + µ )(u1,1i + u 2, 2i + u 3,3i ) + µ(u i ,11 + u i , 22 + u i ,33 ) + ρb i = ρu
&& i

Universidad de Castilla- La Mancha Draft Por: Eduardo W. V. Chaves (2012)


Ciudad Real - España
5 INTRODUCCIÓN A LAS ECUACIONES CONSTITUTIVAS Y PVCI 369

(λ + µ)(u1,11 + u 2, 21 + u 3,31 ) + µ(u1,11 + u1, 22 + u1,33 ) + ρb1 = ρu &&1



(λ + µ)(u1,12 + u 2, 22 + u 3,32 ) + µ(u 2,11 + u 2, 22 + u 2,33 ) + ρb 2 = ρu && 2

(λ + µ)(u1,13 + u 2, 23 + u 3,33 ) + µ(u 3,11 + u 3, 22 + u 3,33 ) + ρb 3 = ρu 3
&&

o aún:
 ∂  ∂u1 ∂u 2 ∂u 3   ∂ 2 u1 ∂ 2 u1 ∂ 2 u1 
λ µ   µ  
+ + + +  ∂x 2 + ∂x 2 + ∂x 2  + ρb1 = ρu1
( ) &&
 ∂x1  ∂x1 ∂x 2 ∂x3   1 2 3 

 ∂  ∂u1 ∂u 2 ∂u 3   ∂ u2 ∂ u2 ∂ u2 
2 2 2

(λ + µ)   + µ 
+ +  ∂x 2 + ∂x 2 + ∂x 2  + ρb 2 = ρu 2
&&
 ∂x 2  ∂x1 ∂ x 2 ∂ x 3   1 2 3 

(λ + µ) ∂  ∂u1 + ∂u 2 + ∂u 3  + µ ∂ u 3 + ∂ u 3 + ∂ u 3  + ρb = ρu
2 2 2
&& 3
 ∂x  ∂x ∂x ∂x   ∂x 2 ∂ x 2
∂ x 2  3
 3  1 2 3   1 2 3 
NOTA 2: Hemos demostrado en el Ejemplo 1.103 que se cumple la siguiente relación:
r r r r r
∇ ∧ (∇ ∧ a) = ∇ (∇ ⋅ a) − ∇ 2 a indicial
 →  ilq  qjk a k , jl = a j , ji − a i , jj
Luego, también se cumple que
r r r r r r
∇ ⋅ (∇u) ≡ ∇ 2 u = ∇ (∇ ⋅ u) − ∇ ∧ (∇ ∧ u) indicial
 → u i , jj = u j , ji −  ilq  qjk u k , jl
Con lo cual la ecuación (5.24) también se puede escribir como:
(λ + µ )u j , ji + µu i , jj + ρb i = ρu
&& i
⇒ (λ + µ)u j , ji + µ(u j , ji −  ilq  qjk u k , jl ) + ρb i = ρ&u& i
⇒ (λ + 2µ)u j , ji − µ ilq  qjk u k , jl + ρb i = ρu
&& i
Lo equivalente en notación tensorial:
r r r
&r&
(λ + µ)[∇ (∇ ⋅ u)] + µ[∇ ⋅ (∇u)] + ρb = ρu
[ ]
r r r r
⇒ (λ + µ)[∇ (∇ ⋅ u)] + µ ∇ (∇ ⋅ u) − ∇ ∧ (∇ ∧ u) + ρb = ρu
r r &r&

[ ]
r r r r r
⇒ (λ + 2µ)[∇ (∇ ⋅ u)] − µ ∇ ∧ (∇ ∧ u) + ρb = ρu &r&

Ejemplo 5.8
Consideremos el campo de deformación infinitesimal ε . a) Demostrar que:
r r
 qjk  til ε ij ,kl = 0 qt ∇ ∧ (∇ ∧ ε ) = 0 (5.25)

donde  ijk es el símbolo de permutación.


b) Demostrar que:
ε ij , kl + ε kl ,ij − ε il , jk − ε jk ,il = O ijkl (5.26)
c) Expresar las ecuaciones en (5.25) de forma explícita.
Solución:
1 ∂u 1  ∂u 
El tensor de deformación infinitesimal viene dado por ε ij =  + i  = (u j ,i + u i , j ) .
j

2  ∂xi ∂x j  2
r
Si derivamos con respecto a ( x ) obtenemos:
∂ε ij 1
= ε ij ,k = (u j ,ik + u i , jk )
∂x k 2

Universidad de Castilla- La Mancha Draft Por: Eduardo W. V. Chaves (2012)


Ciudad Real - España
370 PROBLEMAS RESUELTOS DE MECÁNICA DEL MEDIO CONTINUO

Notar que u i , jk = u i ,kj es simétrico en jk si multiplicamos por un tensor que es


antisimétrico en jk , i.e.  qjk = − qkj , éste se anula: u i , jk  qjk = 0 iq , luego:
1 1
 qjk ε ij ,k = (u j ,ik + u i , jk ) qjk = u j ,ik  qjk
2 2
r
Derivamos una vez más con respecto a ( x ) y obtenemos que:
∂ 1
( qjk ε ij ,k ) =  qjk ε ij , kl = u j ,ikl  qjk
∂x l 2
Notar que u j ,ikl = u j ,kil = u j ,kli es simétrico en il y  til = − tli es antisimétrico en il y si
multiplicamos ambos lados de la igualdad por  til obtenemos que:
1
 til  qjk ε ij ,kl = u j ,ikl  til  qjk = 0 jkt  qjk = 0 qt
2
b) Si en la ecuación anterior multiplicamos por ambos lados de la igualdad por  tab  qmn ,
obtenemos que:
 tab  qmn  til  qjk ε ij ,kl = 0 qt  tab  qmn = O abmn
Recordar que se cumple que  tab  til = δ ai δ bl − δ al δ bi y  qmn  qjk = δ mj δ nk − δ mk δ nj , con lo
cual:
 tab  qmn  til  qjk ε ij ,kl = O abmn
⇒ (δ ai δ bl − δ al δ bi )(δ mj δ nk − δ mk δ nj )ε ij , kl = O abmn
⇒ (δ ai δ bl δ mj δ nk − δ ai δ bl δ mk δ nj − δ al δ bi δ mj δ nk + δ al δ bi δ mk δ nj )ε ij ,kl = O abmn
⇒ ε am,nb − ε an,mb − ε bm,na + ε bn,ma = O abmn
que es lo mismo que:
ε am,bn + ε bn,am − ε an,mb − ε mb,an = O ambn
c) Notar que en (5.25) tenemos 6 ecuaciones independientes ya que 0 qt es simétrico.
Para el caso q = 1, t = 1 tenemos que 1 jk 1il ε ij ,kl y expandiendo el subíndice l obtenemos:
1 jk 1il ε ij ,kl = 1 jk 1i1ε ij , k1 + 1 jk 1i 2 ε ij ,k 2 + 1 jk 1i 3 ε ij ,k 3 = 1 jk 1i 2 ε ij ,k 2 + 1 jk 1i 3 ε ij ,k 3
Expandiendo el subíndice i
1 jk 1il ε ij ,kl = 1 jk 1i 2 ε ij ,k 2 + 1 jk 1i 3 ε ij ,k 3 = 1 jk 132 ε 3 j , k 2 + 1 jk 123 ε 2 j ,k 3 = −1 jk ε 3 j , k 2 + 1 jk ε 2 j ,k 3
Expandiendo los demás subíndices obtenemos:
1 jk 1il ε ij ,kl = −1 jk ε 3 j ,k 2 + 1 jk ε 2 j , k 3 = −123 ε 32,32 − 132 ε 33, 22 + 123 ε 22,33 + 132 ε 23, 23
= −ε 32,32 + ε 33, 22 + ε 22,33 − ε 23, 23 = ε 33, 22 + ε 22,33 − 2ε 23, 23 = 0
∂ 2 ε 33 ∂ 2 ε 22 ∂ 2 ε 23
= + −2 =0
∂x 22 ∂x32 ∂x 2 ∂x3

notar que ε 23, 23 = ε 32,32 . Dejamos para el lector las siguientes demostraciones:
Para el caso q = 2, t = 2

Universidad de Castilla- La Mancha Draft Por: Eduardo W. V. Chaves (2012)


Ciudad Real - España
5 INTRODUCCIÓN A LAS ECUACIONES CONSTITUTIVAS Y PVCI 371

 2 jk  2il ε ij ,kl = −ε 31,31 + ε 33,11 + ε11,33 − ε13,13 = ε 33,11 + ε11,33 − 2ε13,13 = 0


∂ 2 ε 33 ∂ 2 ε11 ∂ 2 ε 13
= + −2 =0
∂x12 ∂x32 ∂x1∂x3
Para el caso q = 3, t = 3
 3 jk  3il ε ij ,kl = ε11, 22 − ε12,12 − ε 21, 21 + ε 22,11 = ε11, 22 + ε 22,11 − 2ε12,12 = 0
∂ 2 ε11 ∂ 2 ε 22 ∂ 2 ε 12
= + −2 =0
∂x 22 ∂x12 ∂x1 ∂x 2
Para el caso q = 1, t = 2
1 jk  2il ε ij ,kl = −ε12,33 + ε13, 23 + ε 32,31 − ε 33, 21 = ε13, 23 + ε 23,13 − ε 33,12 − ε12,33 = 0
∂ 2 ε13 ∂ 2 ε 23 ∂ 2 ε 33 ∂ 2 ε12 ∂  ∂ε 23 ∂ε13 ∂ε 12  ∂ 2 ε 33
= + − − =  + −  − =0
∂x 2 ∂x3 ∂x1 ∂x3 ∂x1 ∂x 2 ∂x3 ∂x3 ∂x3  ∂x1 ∂x 2 ∂x3  ∂x1∂x 2
Para el caso q = 2, t = 3
 2 jk  3il ε ij ,kl = −ε11,32 + ε13,12 + ε 21,31 − ε 23,11 = ε13,12 + ε12,13 − ε 23,11 − ε11, 23 = 0
∂ 2 ε13 ∂ 2 ε 12 ∂ 2 ε 23 ∂ 2 ε11 ∂  ∂ε13 ∂ε12 ∂ε 23  ∂ 2 ε11
= + − − =  + −  − =0
∂x1 ∂x 2 ∂x1 ∂x3 ∂x1∂x1 ∂x 2 ∂x 3 ∂x1  ∂x 2 ∂x3 ∂x1  ∂x 2 ∂x 3
Para el caso q = 1, t = 3
1 jk  3il ε ij ,kl = ε12,32 − ε13, 22 − ε 22,31 + ε 23, 21 = ε12, 23 − ε13, 22 − ε 22,13 + ε 23,12 = 0
∂ 2 ε 12 ∂ 2 ε 13 ∂ 2 ε 22 ∂ 2 ε 23 ∂  ∂ε 12 ∂ε 13 ∂ε 23  ∂ 2 ε 22
= − − + =  − +  − =0
∂x 2 ∂x3 ∂x 2 ∂x 2 ∂x1∂x 3 ∂x1∂x 2 ∂x 2  ∂x3 ∂x 2 ∂x1  ∂x1∂x3
Reagrupando las 6 ecuaciones:

 ∂ 2 ε 33 ∂ 2 ε 22 ∂ 2 ε 23
S
 11 = + − 2 =0
 ∂x 22 ∂x32 ∂x 2 ∂x3
 2 2 2
S 22 = ∂ ε 33 + ∂ ε11 − 2 ∂ ε13 = 0
 ∂x12 ∂x32 ∂x1 ∂x3

 ∂ 2 ε11 ∂ 2 ε 22 ∂ 2 ε 12
S
 33 = + − 2 =0
 ∂x 22 ∂x12 ∂x1 ∂x 2 Ecuaciones de
 (5.27)
S = ∂  ∂ε 23 + ∂ε13 − ∂ε 12  − ∂ ε 33 = 0
2 Compatibilidad 3D
 ∂x 3  ∂x1 ∂x3  ∂x1 ∂x 2
12
∂x 2

 ∂  ∂ε 23 ∂ε 13 ∂ε 12  ∂ 2 ε 11
S 23 = − + + − =0
 ∂x1  ∂x1 ∂x 2 ∂x 3  ∂x 2 ∂x3

S = ∂  ∂ε 23 − ∂ε 13 + ∂ε 12  − ∂ ε 22 = 0
2

 ∂x 2  ∂x1 ∂x 3  ∂x1 ∂x3


13
 ∂x 2

Las ecuaciones anteriores en notación de Voigt quedan:

Universidad de Castilla- La Mancha Draft Por: Eduardo W. V. Chaves (2012)


Ciudad Real - España
372 PROBLEMAS RESUELTOS DE MECÁNICA DEL MEDIO CONTINUO

 ∂2 ∂2 − ∂2 
 0 0 0 
 ∂x32 ∂x 22 ∂x 2 ∂x3 
 ∂2 ∂2 − ∂2 
0 0 0
 S11   ∂x 2 ∂x12

∂x1∂x3   ε11  0
S   3
   
  22  ∂ 2
∂2 − ∂2   ε 22  0
 0 0 0   ε  0
 S 33   ∂x 22 ∂x12 ∂x1 ∂x 2   33  =  
 =
 S12   0 − ∂2 ∂2 ∂2   2ε12  0
2
∂ (5.28)
0 − 12 2 1 1
 S 23   ∂x1 ∂x 2 ∂x3
2
∂x1∂x3 2 
∂x 2 ∂x 3  2ε 23  0
      
 S13   − ∂ ∂ 2   2ε 13  0
2
∂2 ∂2
 0 0 1
2
− 12 2 1
2 
 ∂x 2 ∂x 3 ∂x1 ∂x3 ∂x1 ∂x1 ∂x 2 
 − ∂2 ∂2 ∂2 ∂2 
 0 0 1
2
1
2
− 12 2 
 ∂x1∂x3 ∂x 2 ∂x3 ∂x1∂x 2 ∂x 2 

{S } = [L( 2) ] {ε } = {0}
NOTA 1: Las ecuaciones (5.27) son conocidas como Ecuaciones de Compatibilidad. Las
ecuaciones de compatibilidad nos garantizan que el campo de desplazamiento es único y
continuo. En otras palabras, las 6 componentes del tensor de deformación no son
independientes y no pueden ser arbitrarias.

2
1 2 3 3
1
4 5 6 5
4 6
7 8 9
7 8 9

No cumple con las ecuaciones de


compatibilidad

Cumple con las ecuaciones de 1 2 3


compatibilidad 4 5 6
7 8 9

NOTA 2: Cuando utilizamos un método numérico para la obtención de la solución, e.g.


método de los elementos finitos, la forma de garantizar el cumplimiento de las ecuaciones
de compatibilidad es a través de la continuidad del campo de desplazamientos. Con lo que
respecta la técnica de los elementos finitos, al hacer el ensamblaje de los elementos finitos
(atamos los nodos) estamos de cierta forma garantizando que las ecuaciones de
compatibilidad se cumplan.
NOTA 3: Cuando el campo de desplazamiento no depende de una dirección, i.e.
r r
u = u( x1 , x 2 ) , las ecuaciones de compatibilidad reducen a:

∂ 2 ε11 ∂ 2 ε 22 ∂ 2 ε 12 Ecuación de
S 33 = + −2 =0 (5.29)
∂x 22 ∂x12 ∂x1∂x 2 Compatibilidad 2D

ya que ε i 3 = ε 3i = 0 .

Universidad de Castilla- La Mancha Draft Por: Eduardo W. V. Chaves (2012)


Ciudad Real - España
5 INTRODUCCIÓN A LAS ECUACIONES CONSTITUTIVAS Y PVCI 373

Ejemplo 5.9
a) Demostrar que las ecuaciones fundamentales para un material elástico lineal e isótropo
dadas por (5.22) pueden ser reemplazadas por seis ecuaciones y seis incógnitas ( σ ij ),
(formulación en tensión), cuyas ecuaciones son:
2(λ + µ )
σ ij ,kk +
(2µ + 3λ )
σ kk ,ij −
λ
( 2µ + 3λ )
σ ll ,kk δ ij = 2 (ρu [
&& i ) , j ]sym
[
− 2 ( ρb i ) , j ]
sym
(5.30)

b) o por
2(λ + µ )
σ ij ,kk +
(2µ + 3λ )
σ kk ,ij =
−λ
( 2µ + λ )
[
(ρb k ) ,k − (ρu ]
&& k ) , k δ ij + 2 (ρu
&& i ) , j [ ]sym
[
− 2 ( ρb i ) , j ]
sym

(5.31)
Eν E
c) Considerando que λ = , µ= , expresar las ecuaciones (5.30) y
(1 + ν)(1 − 2ν ) 2(1 + ν)
(5.31) en función ( E , ν) .
r
Consejo: Las ecuaciones cinemáticas ε = ∇ sym u pueden ser reemplazadas por
ε ij , kl + ε kl ,ij − ε il , jk − ε jk ,il = O ijkl (5.32)
Ver ejemplo Ejemplo 5.8.
Solución:
a) Obtenemos la inversa de la ecuación constitutiva en tensión ( σ = C e : ε ) para obtener:
−1 −1 −1
Ce : σ = Ce : C e : ε = I sym : ε = ε sym = ε ⇒ ε = Ce :σ
−1
Recordar que el tensor C e para un material isótropo viene dado por:
1 λ 1 λ
ε= σ− Tr (σ )1 indicial
 → ε ij = σ ij − σ ss δ ij .
2µ 2µ (2µ + 3λ ) 2µ 2µ(2µ + 3λ )
r ∂λ
Si consideramos que las propiedades mecánicas no varían con x , i.e. λ ,i = = 0i y
∂x i
∂µ
µ ,i = = 0 i obtenemos que:
∂xi

∂ 2 ε ij  1 λ  1 λ
≡ ε ij , kl =  σ ij − σ ss δ ij  = σ ij ,kl − σ ss , kl δ ij (5.33)
∂x k ∂xl  2µ 2µ(2µ + 3λ )  ,kl 2µ 2µ(2µ + 3λ )

Además, si multiplicamos la ecuación (5.32) por δ jk obtenemos que:

ε ij , kl δ jk + ε kl ,ij δ jk − ε il , jk δ jk − ε jk ,il δ jk = O ijkl δ jk


(5.34)
⇒ ε ik ,kl + ε kl ,ik − ε il , kk − ε kk ,il = 0 il

Observar que, según la ecuación (5.33) se cumplen que:


1 λ 1 λ
ε ik ,kl = σ ik ,kl − σ ss ,kl δ ik = σ ik ,kl − σ ss ,il
2µ 2µ (2µ + 3λ ) 2µ 2µ(2µ + 3λ )

Universidad de Castilla- La Mancha Draft Por: Eduardo W. V. Chaves (2012)


Ciudad Real - España
374 PROBLEMAS RESUELTOS DE MECÁNICA DEL MEDIO CONTINUO

1 λ 1 λ
ε kl ,ik = σ kl ,ik − σ ss ,ik δ kl = σ lk ,ki − σ ss ,il
2µ 2µ (2µ + 3λ ) 2µ 2µ(2µ + 3λ )
1 λ
ε il ,kk = σ il ,kk − σ ss ,kk δ il
2µ 2µ(2µ + 3λ )
1 λ 1 3λ
ε kk ,il = σ kk ,il − σ ss ,il δ kk = σ − σ
2µ 2µ(2µ + 3λ ) { 2µ kk ,il 2µ(2µ + 3λ ) ss ,il
=3

1 3λ  1 3λ  2µ
= σ ss ,il − σ ss ,il =  − σ ss ,il = σ ss ,il
2µ 2µ (2µ + 3λ )  2µ 2µ(2µ + 3λ )  2µ(2µ + 3λ )

∂ 2 ε ij 1 λ
≡ ε ij ,kl = σ ij ,kl − σ ss ,kl δ ij
∂x k ∂x l 2µ 2µ(2µ + 3λ )
Con lo cual la ecuación (5.34) queda:
ε ik ,kl + ε kl ,ik − ε il ,kk − ε kk ,il = 0 il
1  2λ λ 2µ 
 σ ik ,kl − σ ss ,il + σ lk ,ki − σ il , kk + σ ss ,kk δ il − σ ss ,il  = 0 il
2µ  (2µ + 3λ ) (2µ + 3λ ) (2µ + 3λ ) 
 2λ 2µ  λ
⇒ σ ik ,kl −  + σ ss ,il + σ lk , ki − σ il ,kk + σ ss , kk δ il = 0 il
 (2µ + 3λ ) (2µ + 3λ )  (2µ + 3λ )
2( µ + λ ) λ
⇒ σ ik ,kl − σ ss ,il + σ lk , ki − σ il ,kk + σ ss , kk δ il = 0 il
(2µ + 3λ ) (2µ + 3λ )

2( µ + λ ) λ
− σ ss ,il − σ il ,kk + σ ss ,kk δ il = −σ ik , kl − σ lk ,ki (5.35)
(2µ + 3λ ) (2µ + 3λ )

De las ecuaciones de movimiento σ ij , j + ρb i = ρu


&& i podemos obtener que:

σ ij , jk + (ρb i ) , k = (ρu
&& i ) ,k

Con lo cual se cumplen que:


σ ik ,kl + (ρb i ) ,l = (ρu
&& i ) ,l ⇒ − σ ik ,kl = (ρb i ) ,l − (ρu
&& i ) ,l

σ lk , ki + (ρb l ) ,i = (ρu
&& l ) ,i ⇒ − σ lk ,ki = (ρb l ) ,i − (ρu
&& l ) ,i .

Y notar que − σ ik ,kl − σ lk , ki = (ρb i ) ,l − (ρu&& i ) ,l + (ρb l ) ,i − (ρu&& l ) ,i = 2[(ρb i ) ,l ]sym − 2[(ρu&& i ) ,l ]sym
Reemplazando la ecuación anterior (5.35) obtenemos que:
2( µ + λ ) λ

(2µ + 3λ )
σ ss ,il − σ il ,kk +
(2µ + 3λ )
σ ss ,kk δ il = 2 (ρb i ) ,l[ ]
sym
[
− 2 (ρ&u& i ) ,l ] sym

Reestructurando la ecuación anterior y por hacer ( l = j ) obtenemos que:


2( µ + λ )
σ ij ,kk +
(2µ + 3λ )
σ kk ,ij −
λ
(2µ + 3λ )
σ ll ,kk δ ij = 2 (ρu [
&& i ) , j ]sym
[
− 2 (ρb i ) , j ] sym

Cuya ecuación es la misma que (5.30).


b) Partiendo de la ecuación anterior obtenemos que:
2( µ + λ )
σ ij ,kk +
(2µ + 3λ )
σ kk ,ij =
λ
(2µ + 3λ )
σ ll ,kk δ ij + 2 (ρu [
&& i ) , j ]
sym
[
− 2 ( ρb i ) , j ]
sym
(5.36)

Universidad de Castilla- La Mancha Draft Por: Eduardo W. V. Chaves (2012)


Ciudad Real - España
5 INTRODUCCIÓN A LAS ECUACIONES CONSTITUTIVAS Y PVCI 375

Nuestro objetivo ahora es obtener una expresión para σ ll, kk . Si multiplicamos la ecuación
(5.32) por δ jk δ li obtenemos que:
ε ij , kl δ jk δ li + ε kl ,ij δ jk δ li − ε il , jk δ jk δ li − ε jk ,il δ jk δ li = O ijkl δ jk δ li
⇒ ε ij , ji + ε ji ,ij − ε ii , jj − ε jj ,ii = 2ε ij ,ij − 2ε ii , jj = 0 (5.37)
⇒ ε ij ,ij − ε ii , jj = 0
Si recurrimos a la ecuación constitutiva inversa (ver ecuación (5.33)), podemos decir que:
1 λ 1 λ
ε ij ,ij = σ ij ,ij − σ ss ,ij δ ij = σ ij ,ij − σ ss ,ii
2µ 2µ(2µ + 3λ ) 2µ 2µ(2µ + 3λ )
(5.38)
1 λ  2µ 
ε ii , kk = σ ii ,kk − σ ss ,kk δ ii =  σ ii , kk
2µ 2µ(2µ + 3λ )  2µ (2µ + 3λ ) 
Con lo cual la ecuación (5.37) queda:
⇒ ε ij ,ij − ε ii , jj = 0
1 λ  2µ 
⇒ σ ij ,ij − σ ss ,ii −  σ ii ,kk = 0
2µ 2µ (2µ + 3λ )  2µ(2µ + 3λ ) 
 λ 2µ  (5.39)
⇒ σ ij ,ij −  + σ ii ,kk = 0
 (2µ + 3λ ) (2µ + 3λ ) 
 2µ + λ 
⇒ σ ij ,ij =  σ ii , kk
 (2µ + 3λ ) 
Si ahora recurrimos a las ecuaciones del movimiento σ ij , j + ρb i = ρu
&& i podemos obtener
que:
σ ij , ji + (ρb i ) ,i = (ρ&u& i ) ,i ⇒ σ ij , ji = (ρu
&& i ) ,i − (ρb i ) ,i

Con lo cual la ecuación en (5.39) queda:


 2µ + λ 
σ ij ,ij =  σ ii ,kk
 (2µ + 3λ ) 
 2µ + λ 
&& i ) ,i − (ρb i ) ,i = 
⇒ ( ρu  (2µ + 3λ ) σ ii ,kk (5.40)
 
(2µ + 3λ ) (2µ + 3λ )
⇒ σ ii ,kk = σ ll ,kk =
2µ + λ
(ρu [
&& k ) ,k − (ρb k ) ,k = −
2µ + λ
] (ρb k ) ,k − (ρu [
&& k ) , k ]
Reemplazando la ecuación (5.40) en la ecuación (5.36), obtenemos que:
2( µ + λ )
σ ij ,kk +
(2µ + 3λ )
σ kk ,ij =
λ
(2µ + 3λ )
σ ll ,kk δ ij + 2 (ρu
&& i ) , j [ ]sym
[
− 2 ( ρb i ) , j ]
sym

2( µ + λ ) (2µ + 3λ )
⇒ σ ij , kk +
(2µ + 3λ )
σ kk ,ij =
−λ
(2µ + 3λ ) 2µ + λ
(ρb k ) ,k − (ρu [
&& k ) ,k δ ij + 2 (ρ&u& i ) , j ] [ ] sym
[
− 2 (ρb i ) , j ]
sym

2( µ + λ )
⇒ σ ij , kk +
(2µ + 3λ )
σ kk ,ij =
−λ
( 2µ + λ )
[
(ρb k ) , k − (ρ&u& k ) , k δ ij + 2 (ρu ]
&& i ) , j [ ] sym
[
− 2 (ρb i ) , j ]
sym

(5.41)
Obteniendo así la ecuación en (5.31)
c) Tras algunas manipulaciones algebraicas podemos obtener que:

Universidad de Castilla- La Mancha Draft Por: Eduardo W. V. Chaves (2012)


Ciudad Real - España
376 PROBLEMAS RESUELTOS DE MECÁNICA DEL MEDIO CONTINUO

1 (1 − 2ν)
=
(2µ + 3λ ) E
λ (1 − 2ν ) Eν ν
= =
(2µ + 3λ ) E (1 + ν)(1 − 2ν) (1 + ν)
µ (1 − 2ν) E (1 − 2ν)
= =
(2µ + 3λ ) E 2(1 + ν) 2(1 + ν)
2(µ + λ ) ν (1 − 2ν) 1
=2 +2 =
(2µ + 3λ ) (1 + ν) 2(1 + ν ) (1 + ν )
E Eν E (1 − ν )
(2µ + λ ) = 2 + =
2(1 + ν ) (1 + ν)(1 − 2ν) (1 + ν )(1 − 2ν)
λ Eν (1 + ν )(1 − 2ν) ν
= =
(2µ + λ ) (1 + ν)(1 − 2ν) E (1 − ν ) (1 − ν )
con lo cual la ecuación (5.30):

σ ij ,kk +
1
(1 + ν )
σ kk ,ij −
ν
(1 + ν)
[
σ ll ,kk δ ij = 2 (ρu
&& i ) , j ]
sym
[
− 2 ( ρb i ) , j ]
sym
(5.42)

y (5.31) queda:

σ ij ,kk +
1
(1 + ν)
σ kk ,ij =
−ν
(1 − ν)
[
(ρb k ) ,k − (ρu ]
&& k ) ,k δ ij + 2 (ρu
&& i ) , j[ ]sym
[
− 2 (ρb i ) , j ]
sym
(5.43)

NOTA: Para un problema estático la ecuación anterior quedan:

σ ij ,kk +
1
(1 + ν)
σ kk ,ij =
−ν
(1 − ν )
[ ] [
(ρb k ) , k δ ij − 2 (ρb i ) , j ]sym
Ecuaciones de Michell (5.44)

que son conocidas como ecuaciones de Michell.


r
Si las fuerzas másicas no varían con x las ecuaciones de Michell se reducen a:
1
σ ij ,kk + σ kk ,ij = 0 Ecuaciones de Beltrami (5.45)
(1 + ν)

que son las conocidas ecuaciones de Beltrami.


&& k = 0 k ) , la ecuación (5.40) queda:
NOTA 2: Para un problema estático (u
(2µ + 3λ ) (1 + ν) (1 + ν) r
σ ll ,kk = − (ρb k ) , k = − (ρb k ) ,k ∇ ⋅ {∇ [Tr (σ )]} = − ∇ ⋅ (ρb) (5.46)
2µ + λ (1 − ν) (1 − ν)
La ecuación anterior también puede ser obtenida a partir de la ecuación (5.44) con ( i = j ),
i.e.:

Universidad de Castilla- La Mancha Draft Por: Eduardo W. V. Chaves (2012)


Ciudad Real - España
5 INTRODUCCIÓN A LAS ECUACIONES CONSTITUTIVAS Y PVCI 377

−ν
σ ii ,kk +
1
(1 + ν)
σ kk ,ii =
(1 − ν )
[ ]
(ρb k ) , k δ ii − 2 (ρb i ) ,i
{
[ ]
=3

 1   − 3ν 
⇒ 1 +
(1 + ν) 
σ ii ,kk =  [
− 2  (ρb k ) , k ]
  (1 − ν) 
(5.47)
 ( 2 + ν)   (2 + ν) 
⇒  σ ii ,kk = − [
 (ρb k ) ,k ]
 (1 + ν)   (1 − ν ) 
(1 + ν)
⇒ σ ii ,kk = −
(1 − ν)
[
(ρb k ) ,k ]
Notar que σ ii ,kk = σ kk ,ii y (ρb k ) ,k = (ρb i ) ,i .

Ejemplo 5.10
a) Demostrar que:
∂ 4Φ ∂ 4Φ ∂ 4Φ
+ 2 + =0 (5.48)
∂x14 ∂x12 x 22 ∂x 24
donde
∂ 2Φ ∂ 2Φ ∂ 2Φ
σ11 = ; σ 22 = ; σ12 = σ 21 = − (5.49)
∂x 22 ∂x12 ∂x1 ∂x 2
Considerar un material elástico lineal, un problema estático y sin fuerzas másicas.
Considerar también que el tensor de tensiones de Cauchy es dependiente únicamente de x1
y x 2 , i.e. σ = σ ( x1 , x 2 ) .
b) Demostrar si las ecuaciones de equilibrio se cumplen.
Solución:
a) En el Ejemplo 5.9 (ver ecuación (5.47)) hemos demostrado que:
(1 + ν)
σ ii ,kk = −
(1 − ν)
[
(ρb k ) , k = 0 ]
donde hemos considerado que (ρb k ) ,k = 0 . Para el problema propuesto tenemos que
i, k = 1,2 , con lo cual:
σ ii ,kk = 0
⇒ σ ii ,11 + σ ii , 22 = 0
⇒ σ11,11 + σ 22,11 + σ11, 22 + σ 22, 22 = 0
∂ 2 σ11 ∂ 2 σ 22 ∂ 2 σ11 ∂ 2 σ 22
⇒ + + + =0
∂x12 ∂x12 ∂x 22 ∂x 22
Utilizando la definición (5.49), concluimos que:

Universidad de Castilla- La Mancha Draft Por: Eduardo W. V. Chaves (2012)


Ciudad Real - España
378 PROBLEMAS RESUELTOS DE MECÁNICA DEL MEDIO CONTINUO

∂ 2 σ11 ∂ 2 σ 22 ∂ 2 σ11 ∂ 2 σ 22
+ + + =0
∂x12 ∂x12 ∂x 22 ∂x 22
∂ 2 ∂ 2Φ ∂ 2 ∂ 2Φ ∂ 2 ∂ 2Φ ∂ 2 ∂ 2Φ
⇒ + + + =0
∂x12 ∂x 22 ∂x12 ∂x12 ∂x 22 ∂x 22 ∂x 22 ∂x12
∂ 4Φ ∂ 4Φ ∂ 4Φ
⇒ + 2 + =0
∂x14 ∂x12 ∂x 22 ∂x 24
Q.E.D.
NOTA: En la literatura Φ es conocida como función de tensión de Airy.
b) Para el caso bidimensional las ecuaciones de equilibrio (sin fuerzas másicas) se reducen a:
 ∂σ11 ∂σ12
 ∂x + ∂x = 0
 1 2
σ ij , j = 0 i ⇒ σ i1,1 + σ i 2, 2 = 0 i ⇒ 
 ∂σ 21 + ∂σ 22 = 0
 ∂x1 ∂x 2
Utilizando la definición (5.49), obtenemos que:
 ∂σ11 ∂σ12  ∂ ∂ 2Φ ∂ ∂ 2Φ
 ∂x + ∂x = 0  − =0
 ∂x1 ∂x 2 ∂x 2 ∂x1 ∂x 2
2
 1 2
 ⇒ 
∂σ ∂σ
 21 + 22 = 0
2
− ∂ ∂ Φ + ∂ ∂ Φ = 0
2

 ∂x1 ∂x 2  ∂x ∂x ∂x ∂x 2 ∂x12
 1 1 2

Con lo cual demostramos que las expresiones de las tensiones dadas por (5.49) cumplen
con las ecuaciones de equilibrio.

Ejemplo 5.11
Considerando las ecuaciones de gobierno para un material elástico y lineal, obtener una
r
formulación equivalente solamente en función de u (desplazamientos) y σ (tensiones).
Utilizar la notación de Voigt.
Solución:
Teniendo en cuenta las ecuaciones de gobierno para un problema elástico y lineal:
Notación tensorial Notación de Voigt
Ecuaciones de Movimiento: Ecuaciones de Movimiento:
r r &r& (3 ecuaciones)
∇ ⋅ σ + ρb = ρv& = ρu [L ] {σ } + {ρb } = {ρu&&} (3 ecuaciones)
(1) T

Ecuación Constitutiva en Tensión: Ecuación Constitutiva en Tensión:


(5.50)
σ (ε ) = C e : ε (6 ecuaciones) {σ } = [C ]{ε } (6 ecuaciones)
Ecuaciones Cinemáticas: Ecuaciones Cinemáticas:
{ε } = [L(1) ] {u } (6 ecuaciones)
r
ε = ∇ sym u (6 ecuaciones)

donde las ecuaciones de notación de Voigt fueron obtenidas en el Ejemplo 5.6, con

Universidad de Castilla- La Mancha Draft Por: Eduardo W. V. Chaves (2012)


Ciudad Real - España
5 INTRODUCCIÓN A LAS ECUACIONES CONSTITUTIVAS Y PVCI 379

 ∂ ∂ ∂ 
 0 0 0 
 ∂x1 ∂x 2 ∂x 3 
[L ]
(1) T
= 0


∂x 2
0

∂x1

∂x3
0 

 
 0 ∂ ∂ ∂ 
0 0
 ∂x3 ∂x 2 ∂x1 

Para eliminar la deformación reemplazamos las ecuaciones cinemáticas en la ecuación


constitutiva, resultando:
{σ } = [C ]{ε }
⇒ {σ } = [C ] [L(1) ]{u }
⇒ [C ] {σ } = [1
−1
C4]24
−1
C ] [L(1) ]{u }
[3
=[1 ]
⇒ [C ] {σ } − L
−1
[ ]{u } = {0 }
(1)

Con lo cual quedamos con el siguiente sistema de ecuaciones:


[ ]
 L(1) T {σ } + {ρb } = {ρu&&}
 −1 (1)
[ ]
 [C ] {σ } − L {u } = {0 }

También podemos expresar como:


 [0 ]
 [L ]  {u } = − {ρb } + {ρu&&}
(1) T

 [ ]
− L
(1)
[C ]−1  {σ }  {0 } 

NOTA: La formulación anterior se conoce como Formulación Mixta. Y es interesante


observar que en la formulación en desplazamiento y en tensión obtenidas en los Ejemplo
5.7 y Ejemplo 5.9 respectivamente tenemos derivada segunda de la variable. En la
formulación mixta tenemos solamente derivada primera de los desplazamientos y tensiones
y notar también que no involucra derivadas de los parámetros mecánicos del material.

Universidad de Castilla- La Mancha Draft Por: Eduardo W. V. Chaves (2012)


Ciudad Real - España
380 PROBLEMAS RESUELTOS DE MECÁNICA DEL MEDIO CONTINUO

Ejemplo 5.12
Dados dos sistemas constituidos por el mismo material elástico lineal y con condiciones de
cargas distintas:
r
Sistema I Fuerzas de superficie - t *
r
Fuerzas másicas específica- b
Su B
r r
r dV t * ( x) Campo de tensión - σ
u* r r
ρb( x ) Campo de deformación - ε
r
n̂ Campo de desplazamiento - u

r
Sistema II Fuerzas de superficie - t *
r
Fuerzas másicas específica- b
Su B Sσ
r r r
dV t * ( x) Campo de tensión - σ
u* r
ρb Campo de deformación - ε
r
n̂ Campo de desplazamiento - u

Figura 5.6: Dos sistemas bajo cargas externas.

Demostrar el Teorema de Betti:

∫ σ : ε dV = ∫ σ : εdV
V V
Teorema de Betti (5.51)

Solución:
Teniendo en cuenta la ecuación constitutiva de tensión, σ = C e : ε , en notación indicial:
e
σ ij = C ijkl ε kl

Si a ambos lados de la ecuación multiplicamos por el campo ε obtenemos que:


σ ij ε ij = ε ij C eijkl ε kl Simetría
  Mayor de C
 → e
σ ij ε ij = ε ij C ijkl ε kl = ε kl C eklij ε ij
e
donde hemos aplicado la simetría mayor del tensor constitutivo elástico ( C ijkl = C eklij ).
Como los dos sistemas están constituidos por el mismo material se cumple que σ = C e : ε .
Con lo cual la relación anterior queda:
Notación tensorial
σ ij ε ij = ε ij C eijkl ε kl = ε kl C eklij ε ij = ε kl σ kl    → σ :ε = σ :ε

Si ahora integramos sobre todo el volumen, obtenemos el teorema de Betti:

∫ σ : ε dV = ∫ σ : ε dV
V V
(5.52)

Universidad de Castilla- La Mancha Draft Por: Eduardo W. V. Chaves (2012)


Ciudad Real - España
5 INTRODUCCIÓN A LAS ECUACIONES CONSTITUTIVAS Y PVCI 381

e
NOTA 1: La ecuación anterior solo se cumple si C ijkl = C eklij , es decir, si C e presenta
e
simetría mayor. En otras palabras, la condición C ijkl = C eklij impone la existencia de una
función de energía almacenada, tal que:
e ∂ 2Ψ e ∂ 2Ψ e
C ijkl = = = C eklij
∂ε ij ∂ε kl ∂ε kl ∂ε ij
NOTA 2: El teorema de Betti (Teorema de la Reciprocidad) es el punto de partida para la
obtención de la formulación de la técnica de los Elementos de Contorno.
NOTA 3: El teorema de Betti también puede ser expresada de otra forma que
demostramos a continuación.
1 ∂u  ∂u j  1
Recordar que ε ij =  i +  = (u i , j + u j ,i ) , con eso para el sistema II también se
2  ∂x j ∂xi  2
1
cumple que ε ij = ( ui , j + u j ,i ) . Con lo cual:
2

∫σ
V
ij ε ij dV ∫
= σ ij ε ij dV
V

1 1
2V ∫
σij (u i , j + u j ,i )dV =
2V ∫
σ ij ( ui , j + u j ,i )dV (5.53)

∫σ u
V
ij i , j dV ∫
= σ ij ui , j dV
V

donde σij u i , j = σ ij u j ,i y σ ij ui , j = σ ij u j ,i se cumplen debido la simetría de σ y de σ ,


respectivamente. Y además notar que:
( σ ij u i ), j = σij , j u i + σij u i , j ⇒ σ ij u i , j = ( σ ij u i ), j − σij , j u i
(σ ij ui ), j = σ ij , j ui + σ ij ui , j ⇒ σ ij ui , j = (σ ij ui ), j −σ ij , j ui

Con lo cual la ecuación (5.53) queda:

∫σ u
V
ij i , j dV
V

= σ ij ui , j dV

∫ ( σ u ),
V
ij i j ∫
− σ ij , j u i dV = (σ ij ui ), j −σ ij , j ui dV
V
(5.54)

∫ (σ u ),
V
ij i j ∫
V

dV − σ ij , j u i dV = (σ ij ui ), j dV − σ ij , j ui dV
V

V

Aplicando el teorema de la divergencia a las primeras integrales de cada lada de la ecuación,


obtenemos que:

∫ σ u nˆ dS − ∫ σ
S
ij i j
V
ij , j u i dV ∫
S

= σ ij ui nˆ j dS − σ ij , j ui dV
V
(5.55)
∫ ∫
⇒ t i u i dS − σ ij , j u i dV = t i ui dS − σ ij , j ui dV
S V

S V

r r
donde hemos aplicado la definición σ ⋅ nˆ = t y σ ⋅ nˆ = t . La ecuación anterior en notación
tensorial queda:
r r r r r r
∫ t ⋅ udS − (∇ ⋅ σ ) ⋅ udV = t ⋅ u dS − (∇ ⋅ σ ) ⋅ u dV
∫ ∫ ∫ (5.56)
S V S V

Universidad de Castilla- La Mancha Draft Por: Eduardo W. V. Chaves (2012)


Ciudad Real - España
382 PROBLEMAS RESUELTOS DE MECÁNICA DEL MEDIO CONTINUO

Si recurrimos a las ecuaciones de movimiento se cumple que:


r r r r r r
&r& && &r&
∇ ⋅ σ + ρb = ρu ⇒ − ∇ ⋅ σ = ρ (b − u ) y ∇ ⋅ σ + ρb = ρu ⇒ − ∇ ⋅ σ = ρ (b − u
&&)

Con lo cual la ecuación (5.56) queda:


r r r r r r r r r r
&&
∫ t ⋅ udS + ρ (b − u ) ⋅ udV = t ⋅ u dS + ρ (b − u

&&) ⋅ u dV
∫ ∫ Teorema de Betti (5.57)
S V S V

Notar que, si consideramos S = S u + S σ podemos decir que:


r r r r r r
∫ t ⋅ udS = ∫ t * ⋅ udS + ∫ t ⋅ u* dS
S Sσ Su
r r r r r r (5.58)
∫ t ⋅ u dS = ∫ t * ⋅ u dS + ∫ t ⋅ u * dS
S Sσ Su

Para el caso particular cuando el sistema está en equilibrio y en la ausencia de fuerzas


másicas, la ecuación (5.57) queda:
r r r r
∫ t ⋅ udS = t ⋅ u dS
∫ (5.59)
S S

Si además las fuerzas de superficies son fuerzas concentradas, podemos decir que:
r r r r
Fi loc u loc
i = Fi loc uiloc F loc ⋅ u loc = F loc ⋅ u loc (5.60)

Ejemplo 5.13
Considerando el problema planteado en la Figura 5.6, demostrar el Principio del Trabajo
Virtual:
r r r r r
∫ t * ⋅ u dS σ + ρ (b − u

&&) ⋅ u dV = σ : ε dV


1444442444443 V V
14243 Principio del Trabajo Virtual (5.61)
Trabajo externo virtual total Trabajo interno
virtual total

r r
con u = u* en S u .
Solución:
Partimos directamente de la relación:
1
∫σ
V
ij ε ij dV =
2V ∫ ∫
σ ij ( ui , j + u j ,i )dV = σ ij ui , j dV
V
(5.62)

Notar que (σ ij ui ), j = σ ij , j ui + σ ij ui , j ⇒ σ ij ui , j = (σ ij ui ), j −σ ij , j ui , luego:

∫σ
V
ij ε ij dV ∫ ∫
= σ ij ui , j dV = (σ ij ui ), j −σ ij , j ui dV
V V
(5.63)
∫ ∫
⇒ σ ij ε ij dV = (σ ij ui ), j dV − σ ij , j ui dV
V V

V

Aplicando el teorema de la divergencia a la primera integral del lado derecho, obtenemos


que:

Universidad de Castilla- La Mancha Draft Por: Eduardo W. V. Chaves (2012)


Ciudad Real - España
5 INTRODUCCIÓN A LAS ECUACIONES CONSTITUTIVAS Y PVCI 383

∫σ
V
ij ε ij dV ∫
V

V

= (σ ij ui ), j dV − σ ij , j ui dV = σ ij ui nˆ j dS σ − σ ij , j ui dV


V
(5.64)
∫ ∫
= t *i ui dS σ − σ ij , j ui dV
Sσ V
r
donde hemos aplicado la definición σ ⋅ nˆ = t * . La ecuación anterior en notación tensorial
viene dada por:
r r r
∫ σ : ε dV = t * ⋅ u dS σ − (∇ ⋅ σ ) ⋅ u dV
∫ ∫ (5.65)
V Sσ V
r r r r
Notar que ∇ ⋅ σ + ρb = ρu && ⇒ − ∇ ⋅ σ = ρ (b − u
&&) (ecuaciones de movimiento), con lo
cual, la ecuación (5.65) queda:
r r r r r
∫ t * ⋅ u dSσ + ρ (b − u
&&) ⋅ u dV = σ : ε dV
∫ ∫
Sσ V V
14243
144444244444
3
Total external virtual work Total internal
virtual work

que es conocido como el Principio del Trabajo Virtual. Observar que no hemos tenido que
recurrir la simetría mayor de C e para su demostración.
Para el caso particular cuando el sistema está en equilibrio y en la ausencia de fuerzas
másicas, la ecuación anterior queda:
r r
∫ t * ⋅ u dS σ = σ : ε dV
∫ (5.66)
Sσ V

Si además las fuerzas de superficies son fuerzas concentradas, podemos decir que:
Notación Tensorial Notación de Voigt
r
{F } {u }= ∫ {σ } {ε } dV
r (5.67)
F loc ⋅ u loc = σ : ε dV

loc T loc T

V V

donde {F loc }= {F1 , F2 ,..., Fn }T , {u loc }= {U 1 , U 2 ,..., U n }T , y la dirección de la componente


uiloc es igual a dirección de la componente Fi loc .
NOTA 1: En otras palabras, el Principio del trabajo Virtual afirma que: “Una estructura
está en equilibrio, bajo la acción de un sistema de fuerzas exteriores, si y solo si el trabajo
externo virtual total es igual al trabajo interno virtual total, cualquier que sea el campo de
r
desplazamiento virtual ( u )”.

F1 F2
F3
u
u

σ, ε σ, ε

REAL VIRTUAL

Universidad de Castilla- La Mancha Draft Por: Eduardo W. V. Chaves (2012)


Ciudad Real - España
384 PROBLEMAS RESUELTOS DE MECÁNICA DEL MEDIO CONTINUO

NOTA 2: El Principio del Trabajo Virtual se utiliza en las técnicas de discretización del
problema como por ejemplo la Técnica de los Elementos Finitos, en el cual la incógnita
fundamental es el campo de desplazamiento.
NOTA 3: Es de fácil demostración que la ecuación (5.61) también es válida para las tasas
& r
de los campos virtuales u , ε& , i.e.:
r r& r r r&
∫ t * ⋅ u dS σ + ρ (b − u

&&) ⋅ u dV = σ : ε& dV


1444442444443 V V
14243 Principio del trabajo virtual (5.68)
Trabajo externo virtual total Trabajo interno
virtual total

r
También es valida para una variación de los campos δ u , δε , i.e.:
r r r r r
∫ t * ⋅ δ u dS σ + ρ (b − u

&&) ⋅ δ u dV = σ : δε dV


1444444V2444444
3 V
14243 Principio del trabajo virtual (5.69)
Trabajo externo virtual total Trabajo interno
virtual total

NOTA 4: Podemos definir también el Principio del trabajo virtual complementario donde
las incógnitas fundamentales son tensiones (fuerzas):
r r &r& r
∫ t ⋅ u* dSur + ρ (b − u ) ⋅ udV =
∫ ∫ σ : ε dV Principio del trabajo virtual
Sur
1444442444443 V V
14243 (5.70)
Total internal
complementario
Total external complementary virtual work
complementary virtual work

r
con σ ⋅ nˆ = t * en S σ .

Ejemplo 5.14
Consideremos un sub-dominio caracterizado por un material homogéneo, elástico linear e
isótropo. Consideremos también que en unos puntos del contorno del sub-dominio hay
unas fuerzas concentradas aplicadas {F ( e ) }≡ {F loc } y que el campo de desplazamiento es
aproximado a través de {u( x )} = [N ( x )]{u ( e ) } donde {u (e ) }≡ {u loc } son los desplazamientos
r r

en los puntos donde se aplican las fuerzas concentradas. Demostrar que las ecuaciones
fundamentales para un problema elástico linear e isótropo pueden ser reemplazadas por:

{F }= [K ]{u }
(e) (e) (e)
donde [K ] = ∫ [B] [C ][B] dV
(e) T
(5.71)
V

donde [C ] es la matriz constitutiva elástica en notación de Voigt, y obtener la expresión de


[B( xr )] .
Nota: Usar el Principio del Trabajo Virtual.
Solución:
Podemos partir directamente de la ecuación (5.67), el cual es equivalente a:

Universidad de Castilla- La Mancha Draft Por: Eduardo W. V. Chaves (2012)


Ciudad Real - España
5 INTRODUCCIÓN A LAS ECUACIONES CONSTITUTIVAS Y PVCI 385

r r r
F loc ⋅ u loc = σ : ε dV = σ : (∇ sym u ) dV
∫ ∫
V V
r r r (5.72)
⇒ u loc ⋅ F loc = (∇ sym u ) : σ dV

V

La ecuación anterior en notación de Voigt queda:


r r
{ } {F }= ∫ {ε } {σ } dV
r
u loc ⋅ F loc = (∇ sym u ) : σ dV Voigt
T

T
 → u ( e ) (e)
(5.73)
V V

Notar que en la ecuación anterior ya está teniendo en cuenta la ecuación de equilibrio (ver
ecuaciones (5.65)-(5.67)). La ecuación constitutiva de tensión, en notación de Voigt, viene dada
r r
por {σ ( x )} = [C ]{ε ( x )} , donde el campo del tensor de deformación viene dado por
r r
ε ( x ) = ∇ sym u (Ecuaciones cinemáticas). Tenemos que hacer la representación de la parte
simétrica del gradiente del campo de desplazamiento ε ij = 12 (u i , j + u j ,i ) en notación de
Voigt. En el Ejemplo 5.6 hemos demostrado que:
 ∂u1   ∂ 
   0 0 
∂x1
   ∂x1 
 ∂u   0 ∂
 ε11  
2
0 
∂x 2   ∂x 2 
ε    
 22   ∂u 3   0 ∂ 
0  u1 
 ε 33  
r ∂x3
{ε ( x )} =   =  ∂u ∂u  =  ∂ ∂
∂x3   
 u 2  ⇒ {ε ( xr )} = [L(1) ]{u( xr )}
 2ε 12   1 + 2   0  u 
2ε 23   ∂x 2 ∂x1   ∂x 2 ∂x1  3

   ∂u ∂u  ∂ ∂ 
 2ε 13   2 + 3   0
 ∂x 3 ∂x 2 
 ∂x3 ∂x 2   
 ∂u1 ∂u 3   ∂ ∂ 
 ∂x + ∂x   ∂x 0
 3 1   3 ∂x1 

Luego
{ε ( xr )} = [L(1) ]{u( xr )} = [L(1) ][N ( xr )]{u (e) }= [B( xr )] {u (e) }
donde definimos que:
[B ( xr )] = [L(1) ][N ( xr )] (5.74)
Observar que {u (e ) } no depende de x , y el campo de tensión se puede obtener por:
r

{σ ( xr )} = [C ]{ε ( xr )} = [C ][B( xr )]{u (e ) }


Podemos utilizar la misma aproximación del campo de desplazamiento para aproximar el
campo de desplazamiento virtual, con lo cual también se cumple que:
{u ( xr )} = [N ( xr )]{u (e ) } ⇒ {ε ( xr )} = [B( xr )] {u (e) }
Luego, la ecuación (5.73) queda:

{u } {F }= ∫ {ε } {σ } dV = ∫ {[B( xr )]{u }} [C ][B( xr )]{u }dV


(e) T (e) T (e) T (e)

V V

o aún:

Universidad de Castilla- La Mancha Draft Por: Eduardo W. V. Chaves (2012)


Ciudad Real - España
386 PROBLEMAS RESUELTOS DE MECÁNICA DEL MEDIO CONTINUO

{u } {F }= ∫ {u } [B( xr )] [C ][B( xr )]{u }dV


(e) T (e) (e ) T T (e)
(5.75)
V

Observar que ni {u (e ) } ni {u (e ) } son dependientes de x , con lo cual:


r

 
{u } {F }= {u }  ∫ [B( xr )] [C ][B( xr )] dV {u }
(e) T (e) (e) T T (e)

 V
  (5.76)
{ } ∫ 
r T r
⇒ F ( e ) =  [B ( x )] [C ][B ( x )] dV  u ( e )

{ }
V 
⇒ {F } = [K ]{u }
(e) (e) (e)

[ ]
NOTA: K (e ) es conocida como matriz de rigidez del sub-dominio (elemento finito), y la
matriz [N ( x )] de la relación {u( x )} = [N ( x )]{u ( e ) } se conoce como la matriz que contiene
r r r

las funciones de forma. Las funciones de forma son las funciones definidas en un dominio que
r
nos permite obtener el valor de una función incógnita {u( x )} en cualquier punto del
dominio a través de los valores nodales conocidos {u (e ) } de la función incógnita.

Ejemplo 5.15
Si consideramos la energía potencial elástica total Π dada por:
r 1 r r r r
Π (u) = ∫σ : ε dV − t * ⋅ udS σ − (ρb) ⋅ udV
∫ ∫ Energía Potencial Total (5.77)
V
2 S V
σ

donde
1
U int = ∫ 2 σ : ε dV = ∫ Ψ
e
(ε ) dV (5.78)
V V

y
r r r r
U ext = ∫ t * ⋅ udS σ + (ρb) ⋅ udV ∫ (5.79)
Sσ V

Consideremos también que la primera variación de Π , denotada por δΠ , es igual a cero


para un valor estacionario de Π . Demostrar que, si δΠ = 0 es equivalente al valor
r
estacionario de Π , entonce Π (u) corresponde a un valor mínimo.
r r
NOTA: Considerar que durante el proceso de deformación las acciones externas ( t * , b )
no varían con la deformada. Considerar también un material elástico lineal.
Solución:
La primera variación ( δΠ ) podemos obtener como sigue:

Universidad de Castilla- La Mancha Draft Por: Eduardo W. V. Chaves (2012)


Ciudad Real - España
5 INTRODUCCIÓN A LAS ECUACIONES CONSTITUTIVAS Y PVCI 387

 1 r r r r 
δΠ = δ σ : ε dV − t * ⋅ udS σ − (ρb) ⋅ udV 
∫ ∫ ∫
 2 
V Sσ V 
1 r r r r
= δ σ : ε dV − δ t * ⋅ udS σ − δ (ρb) ⋅ udV
∫ ∫ ∫ (5.80)
V
2 S V σ

1 r r r r
= ∫ δ(σ : ε ) dV − t * ⋅ δudS σ − (ρb) ⋅ δudV
∫ ∫
V
2 S V
σ

Notar que:

δΨ e (ε ) =
1
2
1 1
[
δ(σ : ε ) = (δσ : ε + σ : δε ) = δ(C e : ε ) : ε + σ : δε
2 2
]
1
2
[ 1
2
] [ 1
= (C e : δε ) : ε + σ : δε = ε : C e : δε + σ : δε = [σ : δε + σ : δε ]
2
]
(5.81)
= σ : δε
∂Ψ e
= : δε
∂ε
∂Ψ e
donde hemos considerado que σ = (ver Ejemplo 5.4). Para régimen de pequeñas
∂ε
deformaciones, la ecuación anterior también puede ser escrita como:
∂Ψ e r r r
δΨ e (ε ) = : δε = σ : δε = σ : δ(∇ sym u) = σ : (∇ sym δu) = σ : (∇δu) (5.82)
∂ε
donde hemos utilizado la propiedad A sym : B = A sym : (B sym + B skew ) = A sym : B sym , Con lo
cual la ecuación (5.80) queda:
1 r r r r
δΠ = ∫δ(σ : ε ) dV − t * ⋅ δudS σ − (ρb) ⋅ δudV
∫ ∫
V
2 Sσ V
r* r r r
= σ : δε dV − t ⋅ δudS σ − (ρb) ⋅ δudV
∫ ∫ ∫
V Sσ V
r r r r
= δΨ e dV − t * ⋅ δudS σ − (ρb) ⋅ δudV
∫ ∫ ∫
V Sσ V
r r
La expresión Π (u + δu) puede ser obtenida como
r r r r r r r r
Π (u + δu) = Ψ e (ε + δε ) dV − t * ⋅ (u + δu)dS σ − (ρb) ⋅ (u + δu)dV
∫ ∫ ∫ (5.83)
V Sσ V

Utilizamos serie de Taylor para aproximar el término Ψ e (ε + δε ) , resultando:


∂Ψ e (ε ) 1 ∂ 2 Ψ e (ε )
Ψ e (ε + δε ) = Ψ e (ε ) + : δε + δε : : δε + ... (5.84)
∂ε 2 ∂ε ⊗ ∂ε
∂Ψ e (ε ) ∂ 2 Ψ e (ε )
Notar que : δε = δΨ e (ver ecuación (5.81)) y C e = (ver Ejemplo 5.4).
∂ε ∂ε ⊗ ∂ε
Con lo cual la ecuación (5.84) queda:

Universidad de Castilla- La Mancha Draft Por: Eduardo W. V. Chaves (2012)


Ciudad Real - España
388 PROBLEMAS RESUELTOS DE MECÁNICA DEL MEDIO CONTINUO

∂Ψ e (ε ) 1 ∂ 2 Ψ e (ε )
Ψ e (ε + δε ) = Ψ e (ε ) + : δε + δε : : δε + ...
∂ε 2 ∂ε ⊗ ∂ε
1
≈ Ψ e (ε ) + δΨ e + δε : C e : δε
2
Con lo cual la ecuación (5.83) puede ser reescrita como:
r r r r r r r r
Π (u + δu) = Ψ e (ε + δε ) dV − t * ⋅ (u + δu)dS σ − (ρb) ⋅ (u + δu)dV
∫ ∫ ∫
V Sσ V

1 r r r r r r
∫ ∫
= Ψ e (ε ) dV + δΨ e dV + δε : C e : δε dV − t * ⋅ (u + δu)dS σ − (ρb) ⋅ (u + δu)dV
∫ ∫ ∫
V V V
2 Sσ V
r r r r
= Ψ e (ε ) dV − t * ⋅ udS σ − (ρb) ⋅ udV +
∫ ∫ ∫
V Sσ V
r r r r 1
+ δΨ e dV − t * ⋅ δudS σ − (ρb) ⋅ δudV +
∫ ∫ ∫δε : C e : δε dV ∫
V S V V
2
σ

Notar que:
r r r r r
Π (u) = Ψ e (ε ) dV − t * ⋅ udS σ − (ρb) ⋅ udV
∫ ∫ ∫
V Sσ V

y
r r r r
δΠ = δΨ e dV − t * ⋅ δudS σ − (ρb) ⋅ δudV = 0
∫ ∫ ∫
V Sσ V

Con lo cual:
r r r 1
Π (u + δu) = Π (u) + δΠ +
V
2 ∫
δε : C e : δε dV

r r r 1
⇒ Π (u + δu) − Π (u) = δΠ +
V
2 ∫
δε : C e : δε dV

r r r 1
⇒ Π (u + δu) − Π (u) =
V
2 ∫
δε : C e : δε dV

donde hemos considerado que δΠ = 0 . Notar que el término δε : C e : δε > 0 siempre será
positivo para cualquier valor de δε ya que C e es un tensor definido positivo (ver Capítulo
1). Con lo cual garantizamos que:
r r r 1 r r r
∆Π = Π (u + δu) − Π (u) =
V
2 ∫
δε : C e : δε dV > 0 ⇒ Π (u + δu) > Π (u)
r
δΠ = 0 ⇒ Π (u) es un mínimo
NOTA 1: Para un sistema constituido un material elástico linear el punto de equilibrio
corresponde al valor mínimo de Π . Este es el conocido Principio de la Energía Potencial
Mínima.
NOTA 2: Para el caso particular donde las acciones externas vienen constituida por fuerzas
concentradas y en ausencia de fuerzas másicas, tenemos que:
r
Π (u) = U int + U ext =
1
2 ∫
σ : ε dV − F loc { } {u }T loc
Energía Potencial Total (5.85)
V

Universidad de Castilla- La Mancha Draft Por: Eduardo W. V. Chaves (2012)


Ciudad Real - España
5 INTRODUCCIÓN A LAS ECUACIONES CONSTITUTIVAS Y PVCI 389

Π (u )
F

Π
Π (u ( 2 ) )

Π (u )
δΠ = 0 ∂Π
=0
Π (u ( 3 ) ) ∂u
Deformada correspondiente al
equilibrio

u (2) u u ( 3)

NOTA 3: A través de la ecuación (5.85) podemos demostrar el Teorema de Castigliano- Parte


I:
r
∂Π (u) ∂U int ∂U ext ∂U int
{ } {u }
∂  F loc

T loc

= + = − =0
{ } ∂{u } ∂{u } ∂{u }
∂ u loc loc loc loc
∂{u } loc

⇒ {F }=
int
∂Uloc

∂{u } loc

donde {F }= {F , F ,..., F } , {u }= {U , U ,..., U } . Notar que en la expresión anterior


loc
1 2 n
T loc
1 2 n
T

el término U tiene que estar en función de {u } .


int loc

NOTA 4: Para una mejor ilustración del problema planteado, vamos considerar una barra
de longitud L y con área de la sección transversal A . Consideremos también que el campo
de tensión y de deformación son campos homogéneos y dados por:
σ 0 0   ε 0 0
σ ij =  0 0 0 ; ε ij = 0 0 0 ⇒ σ11 = C1111
e
ε11 ⇒ σ = Eε
 0 0 0 0 0 0

Consideremos también que el campo de desplazamiento viene representado por una


aproximación lineal ( u ( x) = a1 + a2 x ) y en las extremidades de la barra tenemos que:

F (1) ,u (1) F ( 2) ,u ( 2 )
1 2 x

Universidad de Castilla- La Mancha Draft Por: Eduardo W. V. Chaves (2012)


Ciudad Real - España
390 PROBLEMAS RESUELTOS DE MECÁNICA DEL MEDIO CONTINUO

El objetivo ahora es escribir el energía potencial total en función de u (1) , u ( 2) . Verificar que,
debido a las cargas concentradas, tenemos que:
U ext = F (1) u (1) + F ( 2) u ( 2 ) (5.86)
En este caso la relación lineal tensión-deformación viene dada por σ = Eε y la densidad de
1 1
energía Ψ e = σε = εEε . Luego, la energía interna total viene dada por:
2 2
1 1 1 1
∫ 2 σ : ε dV ∫ ∫ ∫
1D
U int = → U int = σε dV = Eεε dV = Eε 2 dV
V
2V 2V 2V

∂u1 ∂u ( x)
notar que ε11 = = = ε , con lo cual:
∂x1 ∂x
2
1 1  ∂u ( x) 
U int =
2V∫Eε 2 dV = E
2 V  ∂x  ∫  dV (5.87)

Nuestro objetivo ahora es expresar el campo de desplazamiento en función de sus valores


nodales. Para ello vamos utilizar la aproximación adoptada u ( x) = a1 + a2 x , donde:
u ( x = 0) = u (1) = a1 + a 2 x = a1  u (1)  1 0  a1 
 ⇒  ( 2)  =   
u ( x = L) = u ( 2 ) = a1 + a 2 x = a1 + a 2 L  u  1 L  a 2 

Ahora nuestro objetivo es definir los coeficientes a1 , a 2 . Para ello, obtenemos la inversa
de la relación anterior:
a1 = u (1)
u (1)  1 0  a1  a1  1  L 0 u (1)  
 ( 2)  =  inversa
→  =  ⇒
 
u  1 L  a 2 
  ( 2) 
a 2  L  − 1 1 u 

a 2 = u − u
 L
(
1 ( 2) (1)
)
con lo cual:

u ( x) = a1 + a 2 x = u (1) +
L
(
1 ( 2)  x
L
) x
u − u (1) x = 1 − u (1) + u ( 2)
L

(5.88)
x   x   u 
(1)
 r
⇒ u ( x) = 1 −      ( 2 )  = [N ( x )] u ( e )
L   L   u 
{ }

Con lo cual la ecuación (5.87) queda:
2 2

∫ ( )
1  ∂u ( x)  E  1 ( 2) (1)  E
∫  u − 2u (1) u ( 2 ) + u (1)  dV
( 2) 2 2
U int = ∫E
2 V  ∂x 
 dV =  u − u  dV = 2
2VL  2L V

(1) ( 2)
Notar que u yu no dependen de x , con lo cual:
E  ( 2) 2 E
− 2u (1) u ( 2 ) + u (1)  dV = 2  u ( 2) − 2u (1) u ( 2 ) + u (1) V
2 2 2
U int =
2L
u
2   2L  ∫ 
V
(5.89)
EAL  ( 2) 2 EA  ( 2 ) 2
− 2u (1) u ( 2 ) + u (1)  = − 2u (1) u ( 2 ) + u (1) 
2 2
= u
2 
u
2L  2L  
Luego, la energía potencial total (ver ecuación (5.85)) viene dada por las ecuaciones (5.86) y
(5.89), i.e.:
r
Π (u) = U int + U ext =
EA  ( 2 ) 2
2L
u

2


(
− 2u (1) u ( 2 ) + u (1)  − F (1) u (1) + F ( 2 ) u ( 2 ) = Π (u (1) , u ( 2 ) ) )

Universidad de Castilla- La Mancha Draft Por: Eduardo W. V. Chaves (2012)


Ciudad Real - España
5 INTRODUCCIÓN A LAS ECUACIONES CONSTITUTIVAS Y PVCI 391

Como buscamos un estado estacionario, hay que cumplir que:


 ∂Π (u (1) , u ( 2 ) )
 =
∂  EA  ( 2) 2
(1) 
u
2
(
− 2u (1) u ( 2 ) + u (1)  − F (1) u (1) + F ( 2) u ( 2 ) ) = 0
 ∂u (1) ∂u  2 L   

 =
EA
2L
( )
− 2u ( 2) + 2u (1) − F (1) = 0


 =
L
(
EA (1)
)
u − u ( 2 ) − F (1) = 0

( ) = 0
(1) (2)
 ∂Π (u , u ) =
∂  EA  ( 2) 2
u − 2u (1) u ( 2 ) + u (1)  − F (1) u (1) + F ( 2 ) u ( 2 )
2
 ( 2) 
∂u ( 2 ) ∂u  2 L   


 =
EA
2L
( )
2u ( 2 ) − 2u (1) − F ( 2 ) = 0



=
L
(
EA ( 2 )
)
u − u (1) − F ( 2 ) = 0

Reestructurando las ecuaciones anteriores en forma matricial obtenemos que:


EA  1 − 1 u   F 
(1) (1)

    =  
L  − 1 1  u ( 2 )   F ( 2 ) 
[ ]{ } { }
⇒ K (e ) u ( e) = F ( e)

Notar que K (e ) [ ] no tiene inversa ya que det[K ]


(e)
= 0 . Para que el problema tenga
solución única, tenemos que introducir las condiciones de contorno.
[ ]
Notar que la matriz K (e ) de la expresión anterior podría haber sido obtenida a través de
la ecuación (5.76) (ver Ejemplo 5.14), donde para este caso particular tenemos que
[C ] = E , y a través de la ecuación (5.88) podemos obtener que:

[B( xr )] = [L(1) ][N ( xr )] = ∂ 



x   x   − 1   1 
1 −    =    
∂x  L   L   L   L 
Luego:
 − 1  
  
[K ] ∫
(e ) r T r
= [B ( x )] [C ][B ( x )] dV = 
 L    − 1   1 

1  1 − 1
E     dV = E 2 
 1    L   L   dV
L − 1 1  ∫
V 
V
   V

 L  
E  1 − 1 E  1 − 1 EA  1 − 1
= 2 
L − 1 1 V
 ∫
dV = 2 
L − 1 1 
 V=  
L − 1 1 

Universidad de Castilla- La Mancha Draft Por: Eduardo W. V. Chaves (2012)


Ciudad Real - España
392 PROBLEMAS RESUELTOS DE MECÁNICA DEL MEDIO CONTINUO

NOTA 5:
Principio del Potencial de Energía Estacionario
En este ejemplo hemos establecido el Principio de la Energía Potencial Estacionaria, (ver
ecuación (5.77)):
r r r r r
Π (u) = Ψ e (ε ) dV − t * ⋅ udS σ − (ρb) ⋅ udV
∫ ∫ ∫ (5.90)
V Sσ V

1
donde hemos tenido en cuenta que Ψ e (ε ) = σ : ε . El funcional es estacionario si y solo si
2
r
δ ur Π (u) = 0 .

Principio Variacional de Hellinger-Reissner


En el Ejemplo 5.4 en la NOTA 7 hemos establecido que
Ψ e (σ) = σε − Ψ e (ε) tensorial
 → Ψ e (σ ) = σ : ε − Ψ e (ε ) = −ρ 0 G(σ ) = g(σ )
(5.91)
⇒ Ψ e (ε ) = σ : ε − g(σ )

donde g(σ ) es la densidad de energía libre de Gibbs con el signo invertido.

σ g(σ) - Energía almacenada complementaria

Ψ e (ε) - Energía almacenada

ε
Figura 5.7: Energías almacenadas.

Reemplazando el valor Ψ e (ε ) = σ : ε − g(σ ) en el funcional (5.90) obtenemos que:


r r r r r
Π (u) = Ψ e (ε ) dV − t * ⋅ udS σ − (ρb) ⋅ udV
∫ ∫ ∫
V Sσ V
r r r r r (5.92)
⇒ Π HR (u, σ ) = σ : ε − g(σ ) dV − t * ⋅ udS σ − (ρb) ⋅ udV
∫ ∫ ∫
V Sσ V
r r
Notar que σ : ε = σ : (∇ sym u) = σ : (∇u) . Con lo cual podemos obtener que:
r r r r r r
Π HR (u, σ ) = σ : (∇u) − g(σ ) dV − t * ⋅ udS σ − (ρb) ⋅ udV
∫ ∫ ∫ (5.93)
V Sσ V

Universidad de Castilla- La Mancha Draft Por: Eduardo W. V. Chaves (2012)


Ciudad Real - España
5 INTRODUCCIÓN A LAS ECUACIONES CONSTITUTIVAS Y PVCI 393

r
El Funcional (5.92) es estacionario para una variación arbitraria de u y desapareciendo en
S ur si y solo si σ satisface las ecuaciones de equilibrio. Y es estacionario para una variación
de σ si y solo si satisface la ecuación constitutiva (tensión-deformación). Este principio es
conocido como Principio Variacional de Hellinger-Reissner.
r r r r r r
δ ur Π HR (u, σ ) = σ : (∇δu) dV − t * ⋅ δudS σ − (ρb) ⋅ δudV = 0
∫ ∫ ∫
V Sσ V
r
∫ ∫ ∫
= σ ij (δu) i , j dV − t *i (δu) i dS σ − (ρb) i (δu) i dV = 0
V Sσ V
r (5.94)
∫ ∫ ∫ ∫
= σ ij (δu) i nˆ j dS − σ ij , j (δu) i dV − t *i (δu) i dS σ − (ρb) i (δu) i dV = 0
Sσ V Sσ V

∫[ ] ∫ [σ ]
r
= − σ ij , j + (ρb) i (δu) i dV + ˆ − t *i (δu) i dS = 0
ij n j
V Sσ

En el volumen obtenemos las ecuaciones de equilibrio: σ ij , j + (ρb) i = 0 i .

En la superficie S σ la condición de contorno: σ ij nˆ j − t *i = 0 i


r r

δ σ Π HR (u, σ ) = δσ : (∇ sym u) − δg(σ ) dV = 0
V
r ∂g(σ )
V

= (∇ sym u) : δσ −
∂σ
: δσ dV = 0 (5.95)

 r ∂g(σ ) 
V 

= (∇ sym u) −
∂σ 
: δσ dV = 0

r ∂g(σ )
En el volumen obtenemos las ecuaciones constitutivas: (∇ sym u) − =0.
∂σ

Principio Variacional de Hu-Washizu


El Principio de Hu-Washizu es una generalización del Principio de Hellinger-Reissner,
r
donde el funcional además de depender de (u, σ ) también depende del campo de
deformación ε , y viene dado por:

∫ [Ψ ]
r r r r r r r r
Π HR (u, σ , ε ) = e
(ε ) − σ : (ε − ∇ sym u) − (ρb) ⋅ u dV − (σ ⋅ nˆ ) ⋅ (u* − u)dS σ − t * ⋅ udS σ
∫ ∫
V S ur Sσ

(5.96)
y estacionario para las siguientes situaciones:
r
δ ur Π HR (u, σ , ε ) = 0 ⇒ Ecuaciones de equilibrio
 r r
δu = 0 ⇒ En S ur

 r Ecuaciones cinemáticas
δ σ Π HR (u, σ , ε ) = 0 ⇒ 
 Condición de contorno en S ur
 r
δ ε Π HR (u, σ , ε ) = 0 ⇒ Ecuaciones constitutivas tensión - deformación


Universidad de Castilla- La Mancha Draft Por: Eduardo W. V. Chaves (2012)


Ciudad Real - España
394 PROBLEMAS RESUELTOS DE MECÁNICA DEL MEDIO CONTINUO

Es decir:
r
ƒ δ ur Π HR (u, σ , ε ) = 0
r
δ ur Π HR (u, σ , ε ) = [∫ σ : (∇ δur ) − (ρbr ) ⋅ δur ] dV − ∫ tr ⋅ δur dS
sym *
σ
V Sσ

(u, σ , ε ) = ∫ [(∇ ⋅ σ ) ⋅ (δu) − (ρb) ⋅ δu] dV − ∫ t ⋅ δudS


r r r r r r *
⇒ δ ur Π HR σ
V Sσ

(u, σ , ε ) = ∫ [(∇ ⋅ σ ) − (ρb)]⋅ (δu) dV − ∫ t ⋅ δudS


r r r r r *
⇒ δ ur Π HR σ
V Sσ

r
ƒ δ σ Π HR (u, σ , ε ) = 0

∫[ ]
r r r r
δ σ Π HR (u, σ , ε ) = − δσ : (ε − ∇ sym u) dV − (δσ ⋅ nˆ ) ⋅ (u* − u)dS σ = 0

V Sur

∫ [ ] ∫ [nˆ ⊗ (u ]
r r r* r
⇒ δ σ Π HR (u, σ , ε ) = − (ε − ∇ sym u) : δσ dV − − u) : δσdS σ = 0
V Sur

r
ƒ δ ε Π HR (u, σ , ε ) = 0

∫[ ]
r
δ ε Π HR (u, σ , ε ) = δ ε Ψ e (ε ) − σ : (δ ε ε ) dV = 0
V

r  ∂Ψ e (ε )   ∂Ψ e (ε ) 
⇒ δ ε Π HR (u, σ , ε ) = 
V 

∂ε
: δ ε ε − σ : (δ ε ε ) dV = 
 V 
∂ε ∫
− σ  : δ ε ε dV = 0

Universidad de Castilla- La Mancha Draft Por: Eduardo W. V. Chaves (2012)


Ciudad Real - España
5 INTRODUCCIÓN A LAS ECUACIONES CONSTITUTIVAS Y PVCI 395

Ejemplo 5.16
Demostrar que:

∫t
r* r r& r r
[
( X , t ) ⋅ u dS 0σ + ρ 0 b( X , t ) − u

r
0 ] X
r
&r&( Xr , t ) ⋅ u& dV = P : ∇ r u& dV
0 ∫ (5.97)
Sσ V0 V0

r
donde u es un campo virtual de desplazamientos, y P es el primer tensor de tensiones de
Piola-Kirchhoff.

configuración
configuración de F actual
referencia S 0σ

V0
V
B0 dV0 r r r
t * ( X , t ) = t *0 B r r
r r dV t * ( x, t )
u( X , t )
r r
r r r u( x , t )
ρ 0 b( X , t ) = ρ 0 b 0 r r
ρb( x , t )

Solución:
r r r r
Aunque las variables t * ( X , t ) y b( X , t ) no sean variables intrínsecas de la configuración de
referencia como las variables ρ 0 , S 0 , V0 entre otras, por simplicidad vamos denotar
r r r r r r
t * ( X , t ) = t *0 y b( X , t ) = b 0 y para el campo de desplazamiento Lagrangeano.
Recordemos también que (ver Capítulo 2 del libro de texto):
r r r
D ∂ ∂x ( X , t ) ∂ ∂x ( X , t ) ∂u& i ( X , t) r
Fij ≡ F&ij = i
= i
= = u& i , J ( X , t )
Dt ∂t ∂X j ∂X j 142 ∂t 43 ∂X j
x&i

r r
ó F& = l ⋅ F = ∇ Xr u& ( X , t )
r r
y = F& ⋅ F −1 = ∇ Xr u& ( X , t ) ⋅ F −1
l
r r r r
F& −1 = − F −1 ⋅ l = − F −1 ⋅ ∇ Xr u& ( X , t ) ⋅ F −1 = − F −1 ⋅ ∇ xr u& ( x , t )

Teniendo en cuenta las relaciones anteriores, también se cumple para un campo virtual de
desplazamiento:
r& r r& r r& r
F& = ∇ Xr u ( X , t ) y F& −1 = − F −1 ⋅ ∇ Xr u ( X , t ) ⋅ F −1 = − F −1 ⋅ ∇ xr u ( x , t )

Con lo cual podemos decir que:


& r
∫ P : F dV = ∫ P 0 iJ F&iJ dV0 = PiJ u& i , J ( X , t ) dV0

V0 V0 V0

(PiJ u& i ) , J = PiJ , J u& i + PiJ u& i , J ⇒ PiJ u& i , J = (PiJ u& i ) , J − PiJ , J u& i
con lo cual:

Universidad de Castilla- La Mancha Draft Por: Eduardo W. V. Chaves (2012)


Ciudad Real - España
396 PROBLEMAS RESUELTOS DE MECÁNICA DEL MEDIO CONTINUO

r r r
∫ P : F& dV0 = PiJ u& i , J ( X , t ) dV0 = (PiJ u& i ( X , t )) , J − PiJ , J u& i ( X , t ) dV0
∫ ∫
V0 V0 V0

& r r
∫ P : F dV = ∫ (P
V0
0
V0
iJ ∫
u& i ( X , t )) , J dV0 − PiJ , J u& i ( X , t ) dV0
V0

& r r
∫ P : F dV = ∫ P
V0
0
S0
iJ
V0

u& i ( X , t )nˆ J dS 0 − PiJ , J u& i ( X , t ) dV0

donde hemos aplicado el teorema de la divergencia. En notación tensorial queda:

r r
∫ P : F dV = ∫ (P ⋅ nˆ ) ⋅ u& ( X , t ) dS − ∫ (∇ ⋅ P) ⋅ u& i ( X , t ) dV0
& r
0 i 0 X
V0 S0 V0

Recordar que la ecuación de movimiento en la configuración de referencia viene dada por:


r
&r&( Xr , t )
∇ Xr ⋅ P + ρ 0 b 0 = ρ 0 u ⇒
r
[
&r&( Xr , t )
− ∇ Xr ⋅ P = ρ 0 b 0 − u ]
r& r
y teniendo en cuenta que donde por definición se cumplen que F& = ∇ Xr u ( X , t ) y
r
t *0 = P ⋅ nˆ obtenemos:
r r
∫ P : F& dV0 = (P ⋅ nˆ ) ⋅ u& i ( X , t ) dS 0 − (∇ Xr ⋅ P ) ⋅ u& i ( X , t ) dV0
∫ ∫
V0 S0 V0

∫P :∇ r
X
r& r r &r& r
u ( X , t ) dV0 = t *0 ⋅ u ∫
r
&r&( Xr , t ) ⋅ u& ( Xr , t ) dV
( X , t ) dS 0 + ρ 0 b 0 − u i ∫ 0 [ ]
V0 S0 V0

Recordatorio: Recordemos del capítulo 4 del libro de texto que la potencial tensional
puede ser expresada de distintas formas, a saber:
1 1
V

w int (t ) = P : F& dV0 = S : E& dV0 = P : F& dV0 =
V V
2 V

S : C& dV0 =
V
J
P : F& dV ∫ ∫ ∫
0 0 0 0

ρ
= ∫ρ
V 0
P : F& dV = σ : D dV = { ∫
Jσ : D dV0 = τ : D dV0
V V0
∫ τ

V0

y D=l sym
[ r r sym
= ∇ xr u& ( x , t) ]
(ver Ejemplo 2.36)
NOTA 1: Recordar que ni P ni F& están en ninguna configuración, pero el escalar P : F&
si que están en la configuración de referencia.
NOTA 2: Teniendo en cuenta lo anterior. El trabajo interno virtual total también se puede
expresar como:
r& r sym r& r r& r
∫σ : D dV = σ : ∇ xr u ( x , t ) dV = σ : ∇ xr u ( x , t ) dV = P : F& dV0 = P : ∇ Xr u ( X , t ) dV0
∫ ∫ ∫ ∫
 
V V V V V 0 0

r& r sym r& r


Note que, debido a la simetría de σ se cumple que σ : ∇ xr u ( x , t ) = σ : ∇ xr u ( x , t ) .
 
NOTA 3: La ecuación (5.97) también es válida para una variación del campo virtual:


r r r r r
t * ( X , t ) ⋅ δ u dS 0σ + ρ 0 b( X , t ) − u
∫ [ r
0 X
r
&r&( Xr , t ) ⋅ δ u dV = P : ∇ r δ u dV
0 ] ∫ (5.98)
Sσ V0 V0

Universidad de Castilla- La Mancha Draft Por: Eduardo W. V. Chaves (2012)


Ciudad Real - España
7 Elasticidad Lineal
7.1 Ejercicios Resueltos

Ejemplo 7.1
El cilindro indefinido de la Figura 7.1 constituido por un material elástico lineal isótropo,
está sometido al siguiente estado de deformación (en coordenadas cilíndricas):
err = eθθ = a sin θ
a cos θ
e rθ = (7.1)
2
e zz = eθz = erz = 0

con eij son las componentes del Tensor de Almansi. Se pide:


r
Calcular el vector tracción t en el contorno, en coordenadas cilíndricas.
Hipótesis: 1) λ, µ son las constantes de Lamé; 2) Régimen de pequeñas deformaciones.

x3

Π
t

Π ê z
ê θ

t ê r

x2
r

x1

Figura 7.1.
392 PROBLEMAS RESUELTOS DE MECÁNICA DEL MEDIO CONTINUO

Solución:
Régimen de pequeñas deformaciones: e ≈ E ≈ ε
 a cos θ 
a sin θ 0
 ε rr ε rθ ε rz   2
 a cos θ 
ε( r , θ, z ) = ε rθ ε θθ ε θz  =  a sin θ 0 (7.2)
 2 
 ε rz ε θz ε zz   0 0 0
 

σ = λTr (ε )1 + 2µε (7.3)


Tr (ε ) = 2a sin θ (7.4)
luego,
 a cos θ 
 a sin θ 2
0
1 0 0  a cos θ 
σ = λ 2a sin θ 0 1 0 + 2µ  a sin θ 0 (7.5)
 2 
0 0 1  0 0 0
 

λ 2a sin θ + 2µa sin θ µa cos θ 0 



σ (r,θr,θ =  µa cos θ λ 2a sin θ + 2µa sin θ 0 
 (7.6)
 0 0 λ 2a sin θ
r
El vector tracción t :
r ˆ
t (n) = σ ⋅ nˆ (7.7)
nˆ = (1,0,0)

t 1(nˆ )   2λa sin θ + 2µa sin θ


 (nˆ )   
t 2  =  µa cos θ  (7.8)
t (nˆ )   0 
 3  

Ejemplo 7.2
El paralelepípedo de la Figura 7.2 se deforma de la manera indicada por las líneas de trazo.
Los desplazamientos vienen dados por las siguientes relaciones:
u = C1 xyz ; v = C 2 xyz ; ω = C 3 xyz (7.9)
Se pide:
a) Determinar el estado de deformación en el punto E , cuando las coordenadas del
punto E ′ en el cuerpo deformado son E ′(1,503; 1,001; 1,997) ;
b) Determinar la deformación normal en E en la dirección de la línea EA ;
c) Calcular la distorsión angular en E del ángulo recto formado por las líneas EA y
EF .
d) Determinar el incremento de volumen y la deformación volumétrica media.

Universidad de Castilla- La Mancha Draft Por: Eduardo W. V. Chaves (2012)


Ciudad Real - España
7 ELASTICIDAD LINEAL 393

G ≡ G′ F ≡ F′

E
D ≡ D′ 2m

E′

C ≡ C′
O ≡ O′
y

1,5m
A ≡ A′
B ≡ B′
1m
x

Figura 7.2
Solución:
a) El estado de deformación en función de los desplazamientos es:
1  ∂u i ∂u j 

ε ij = + (7.10)
2  ∂x j ∂x i 

Explícitamente en notación ingenieril:
 ∂u 1  ∂v ∂u  1  ∂ω ∂u  
 1 1    +   + 
 εx γ xy γ xz 
 ∂x 2  ∂x ∂y  2  ∂x ∂z  
2 2
1 1   
1 ∂v ∂u  ∂v 1  ∂ω ∂v  
ε ij =  γ xy εy γ yz  =   +   +  (7.11)
2 2   2  ∂x ∂y  ∂y 2  ∂y ∂z  
1 γ 1
γ yz ε z   1  ∂ω ∂u  1  ∂ω ∂v  ∂ω 
 2 xz   2  ∂x + ∂z   +  
2  ∂y ∂z 
2
   ∂z 
Para determinar el estado de deformación en cualquier punto necesitamos a priori
determinar el campo de desplazamientos.
Determinación de las constantes:
Reemplazando los valores dados para el punto E (1,5; 1,0; 2,0) , resulta:
u ( E ) = 1,503 − 1,5 = C1 (1,5)(1,0)(2,0) ⇒ C1 = 0,001
0,001
v ( E ) = 1,001 − 1,0 = C 2 (1,5)(1,0)(2,0) ⇒ C 2 = (7.12)
3
w(E ) = 1,997 − 2,0 = C 3 (1,5)(1,0)(2,0) ⇒ C 3 = −0,001

donde el desplazamiento del punto E : u ( E ) = (1,503 − 1,5; 1,001 − 1,0; 1,997 − 2,0) fue
utilizado.
Podemos escribir el Campo de Desplazamiento:

Universidad de Castilla- La Mancha Draft Por: Eduardo W. V. Chaves (2012)


Ciudad Real - España
394 PROBLEMAS RESUELTOS DE MECÁNICA DEL MEDIO CONTINUO

Notación Ingenieril Notación Científica


u = 0,001 xyz u1 = 0,001 X 1 X 2 X 3
0,001 0,001 (7.13)
v= xyz u2 = X1 X 2 X 3
3 3
w = −0,001 xyz u 3 = −0,001 X 1 X 2 X 3

∂u
εx = = 0,001 yz = 0,002 = ε11
∂x
∂v 0,001
εy = = xz = 0,001 = ε 22
∂y 3
∂w
εz = = −0,001xy = −0,0015 = ε 33
∂z
 ∂v ∂u  0,001 0,011
γ xy =  +  = yz + 0,001xz = = 2ε 12
 ∂x ∂y  3 3
 ∂w ∂u 
γ xz =  +  = −0,001yz + 0,001xy = −0,0005 = 2ε13
 ∂x ∂z 
 ∂w ∂v  0,001
γ yz =  +  = −0,001xz + xy = −0,0025 = 2ε 23
 ∂y ∂z  3
El campo de deformación queda:

 yz
1  yz 
 + xz 
1
(xy − yz ) 
 2 3  2 
1  yz  xz 1  xy 
ε ij = 0,001  + xz   − xz 
2  3  3 2 3 
 
1
 (xy − yz ) 1  xy  
 − xz  − xy
 2 2 3  
Estado de deformación en el punto E ( x = 1,5; y = 1,0; z = 2,0) :

 1 1    0,011 


 − 0,00025
 εx γ xy γ xz   0,002
2 2  6 
1 1    0,011  
ε ij =  γ xy εy 
γ yz  =  − 0,00125
E   0,001
 (7.14)
2 2   6 
1 γ 1  
 2 xz γ yz ε z  − 0,00025 − 0,00125 − 0,0015 
2   
b) Como visto la componente normal:
ˆ ⋅ε ⋅ M
ε Mˆ = M ˆ componente
 s → ε Mˆ = ε ij Mˆ i Mˆ j (7.15)
Expandiendo la expresión anterior y considerando la simetría del tensor de deformación:
ε Mˆ = ε 11 Mˆ 12 + ε 22 Mˆ 22 + ε 33 Mˆ 32 + 2ε 12 Mˆ 1 Mˆ 2 + 2ε 13 Mˆ 1 Mˆ 3 + 2ε 23 Mˆ 2 Mˆ 3 (7.16)
en notación Ingenieril:
ε Mˆ = ε x Mˆ 12 + ε y Mˆ 22 + ε z Mˆ 32 + γ xy Mˆ 1 Mˆ 2 + γ xz Mˆ 1 Mˆ 3 + γ yz Mˆ 2 Mˆ 3 (7.17)
La normal será dada por los cosenos directores de la dirección de la línea EA :

Universidad de Castilla- La Mancha Draft Por: Eduardo W. V. Chaves (2012)


Ciudad Real - España
7 ELASTICIDAD LINEAL 395

−1 −2
Mˆ 1 = 0 ; Mˆ 2 = ; Mˆ 3 = (7.18)
5 5
Reemplazando los correspondientes valores en la ecuación (7.17), resulta:
ε Mˆ = ε y Mˆ 22 + ε z Mˆ 32 + γ yz Mˆ 2 Mˆ 3
1 4 2
ε Mˆ = 0,001 + (−0,0015) + (−0,0025) (7.19)
5 5 5
−3
ε Mˆ = −2 × 10
c) Para el caso de pequeñas deformaciones, la distorsión en E del ángulo recto formado
por las líneas EA y EF , Θ = 90º ), será:

1 1  − 2Mˆ ⋅ ε ⋅ Nˆ 
ε Mˆ Nˆ = − ∆θ Mˆ Nˆ = −  ˆ ⋅ ε ⋅ Nˆ
=M componente
 s → ε Mˆ Nˆ = ε ij Mˆ i Nˆ j (7.20)
2 2 sin Θ 

Para mayores detalles de la expresión anterior ver Capítulo 2- Cinemática del Continuo
(pequeñas deformaciones). Expandiendo la expresión anterior y considerando la simetría
del tensor de deformación:
(
ε Mˆ Nˆ = ε 11 Mˆ 1 Nˆ 1 + ε 22 Mˆ 2 Nˆ 2 + ε 33 Mˆ 3 Nˆ 3 + ε 12 Mˆ 1 Nˆ 2 + Mˆ 2 Nˆ 1 + )
(7.21)
( ) (
+ ε 13 Mˆ 1 Nˆ 3 + Mˆ 3 Nˆ 1 + ε 23 Mˆ 2 Nˆ 3 + Mˆ 3 Nˆ 2 )
o en notación ingenieril:
γ Mˆ Nˆ
2
= ε x Mˆ 1 Nˆ 1 + ε y Mˆ 2 Nˆ 2 + ε z Mˆ 3 Nˆ 3 +
γ xy
2
(Mˆ 1 )
Nˆ 2 + Mˆ 2 Nˆ 1 +
(7.22)
γ
(
+ xz Mˆ 1 Nˆ 3 + Mˆ 3 Nˆ 1 +
2
γ yz
2
) (
Mˆ 2 Nˆ 3 + Mˆ 3 Nˆ 2 )
donde:
 −1 − 2
Mˆ i = 0  ; Nˆ i = [− 1 0 0] (7.23)
 5 5
resultando:
γ Mˆ Nˆ  0,011   −1  − 2
= ε 12 Mˆ 2 Nˆ 1 + ε 13 Mˆ 3 Nˆ 1 =  (−1)  + (−0,00025)(−1)  
2  6   5  5
γ Mˆ Nˆ (7.24)
= 5,96284793998 × 10 − 4
2
⇒ γ Mˆ Nˆ = 1,1925696 × 10 −3

Solución Alternativa
Podemos crea una base ortonormal asociada a los versores M̂ y N̂ a través del producto
vectorial Pˆ = Mˆ ∧ Nˆ . Luego, obtenemos así las componentes del versor P̂ :
eˆ 1 eˆ 2 eˆ 3
−1 −2 2 ˆ 1 ˆ  2 −1
Pˆ = M
ˆ ∧ Nˆ = 0 = e2 − e3 ⇒ Pˆi = 0  (7.25)
5 5 5 5  5 5
−1 0 0

Universidad de Castilla- La Mancha Draft Por: Eduardo W. V. Chaves (2012)


Ciudad Real - España
396 PROBLEMAS RESUELTOS DE MECÁNICA DEL MEDIO CONTINUO

Luego, la matriz de transformación del sistema X 1 X 2 X 3 para la base M̂ , N̂ , P̂ viene


dada por:
 −1 − 2
 Mˆ 1 Mˆ 2 Mˆ 3   0 5 5

  
A = a ij =  Nˆ 1 Nˆ 2 Nˆ 3  =  − 1 0 0  (7.26)
 Pˆ 2 −1
 1 Pˆ
2 Pˆ3   0
 5 5 

Aplicando la ley de transformación de las componentes de un tensor de segundo orden, es
decir, ε ij = a ik a jl ε kl o en forma matricial ε ′ = A ε A T :

  0,011   T
 −1 − 2   0,002   − 0,00025  −1 − 2
0   6  
0 
 5 5    0,011   5 5
ε ′ = − 1 0 0    0,001 − 0,00125  − 1 0 0  (7.27)
2 −1   6   2 −1
0 − 0,00025 − 0,00125 − 0,0015   0
 5 5     5 5 

 
Resultando:
γ Mˆ Nˆ
ε M̂ ε Mˆ Nˆ =
2

 
 − 2 × 10 −3 5,96284794 × 10 − 4 − 2,5 × 10 − 4
 
 −3 
ε ij = 5,96284794 × 10 − 4
′ 2 × 10 −3 − 1,75158658 × 10  (7.28)
 −4 
 − 2,5 × 10 − 1,75158658 × 10 −3 1,5 × 10 −3 
 

NOTA: Observar que no se trata de un caso de deformación homogénea, es decir, las


aristas que en la configuración inicial son rectas, en la configuración deformada no mas
serán rectas. Para obtener los versores deformados tenemos que aplicar la transformación
lineal mˆ = F ⋅ Mˆ y nˆ = F ⋅ Nˆ , donde F es el gradiente de deformación.
∆( dV )
d) Deformación volumétrica por definición es: εV = donde dV es un diferencial de
dV
volumen
Caso de pequeñas deformaciones:
∆ (dV )
εV = = εx + εy + εz ⇒ ∆ (dV ) = ε x + ε y + ε z dV (7.29)
dV
integrando podemos obtener el incremento de volumen:
2, 0 1 1, 5

∫ (ε )  xz 
∆V =
V
x + ε y + ε z dV = 0,001 ∫ ∫ ∫  yz +
z =0 y =0 x =0
3
− xy dxdydz

(7.30)

resultando:

∆V = 1,125 × 10 −3 m 3 (7.31)

Universidad de Castilla- La Mancha Draft Por: Eduardo W. V. Chaves (2012)


Ciudad Real - España
7 ELASTICIDAD LINEAL 397

Luego:
∆( dV ) 1,125 × 10 −3
εV = = = 0,375 × 10 −3 (7.32)
dV 1,5 × 1,0 × 2,0

Ejemplo 7.3
El estado de tensiones en un punto de una estructura que está constituida por un material
elástico, lineal e isótrropo, viene dado por:
 6 2 0
σ ij =  2 − 3 0 MPa
0 0 0

a) Determinar las componentes del tensor de deformación ingenieril. Considérese que el


módulo de elasticidad longitudinal ( E = 207GPa ) y el módulo de elasticidad transversal
( G = 80GPa )
b) Si un cubo de 5cm de lado está sometido a este estado tensional. ¿Cual será su cambio
de volumen?
Solución:
Las deformaciones pueden ser obtenidas partiendo de las siguientes relaciones:

εx =
1
E
[ ( )]
σ x − ν σ y + σ z = 3,333 × 10 −5

1
[ ]
ε y = σ y − ν(σ x + σ z ) = −2,318 × 10 −5
E
1
[ ( )]
ε z = σ z − ν σ x + σ y = −4,348 × 10 −6
E
(7.33)
1
γ xy = τ xy = 2,5 × 10 −5
G
1
γ xy = τ xy = 0
G
1
γ xy = τ xy = 0
G
donde el coeficiente de Poisson puede ser obtenido partiendo de la relación:
E E 207
G= ⇒ ν= −1= − 1 ≈ 0,29375
2(1 + ν) 2G 160
Luego:
33,24 12,5 0 

ε ij =  12,5 − 23,01 0  × 10 −6
 0 0 − 4,257 

Solución Alternativa
−1 (1 + ν) ν
En el libro texto (Chaves(2007)) hemos demostrado que C e = I − 1 ⊗ 1 , con
E E
eso obtenemos que:

Universidad de Castilla- La Mancha Draft Por: Eduardo W. V. Chaves (2012)


Ciudad Real - España
398 PROBLEMAS RESUELTOS DE MECÁNICA DEL MEDIO CONTINUO

−1  (1 + ν) ν  (1 + ν) ν (1 + ν ) ν
ε = Ce : σ =  I − 1 ⊗ 1 : σ = I :σ − 1 ⊗1:σ = σ − Tr (σ )1
 E E  E E E E
En componentes:
 σ11 σ12 σ13  1 0 0
(1 + ν)  ν
ε ij = σ12 σ 22σ 23  − Tr (σ ) 0 1 0

E  E
σ13 σ 23 σ 33  0 0 1
 6 2 0 1 0 0 33,24 12,5 0 
−6   −6   
= 6,251 × 10  2 − 3 0 − 4,2609 × 10 0 1 0 =  12,5 − 23,01 0  × 10 − 6
0 0 0 0 0 1  0 0 − 4,257

(1 + ν)  1  ν
where we have used = 6,25 × 10 −6  , Tr (σ ) = 4,25725 × 10 −6 .
E  MPa  E
En el régimen de pequeñas deformaciones la deformación volumétrica (lineal) es igual a la
traza del tensor de deformación:
DVL ≡ ε V = I ε = (33,24 − 23,01 − 4,257 ) × 10 −6 = 5,973 × 10 −6
Luego, la variación de volumen queda:
∆V = ε V V0 = 5,973 × 10 −6 (5 × 5 × 5) = 7,466 × 10 −4 cm 3

Ejemplo 7.4
Un paralelepípedo de dimensiones a = 3cm , b = 3cm , c = 4cm , constituido por un material
homogéneo elástico y lineal se aloja en una cavidad de la misma forma y dimensiones,
cuyas paredes son de un material lo suficientemente rígido para poderlo suponer
indeformable. Sobre la abertura de la cavidad de dimensiones a × b y a través de una placa
rígida de peso y rozamiento despreciables se aplica, perpendicularmente a ella, una fuerza
F = 200 N que comprime al bloque elástico. Si el coeficiente de Poisson es ν = 0,3 y el
módulo de elasticidad E = 2 × 10 4 N / cm 2 , calcular:
a) Las fuerzas laterales ejercidas por las paredes de la cavidad sobre el paralelepípedo;
b) La variación de altura experimentada por el mismo.

F
z

y
c

b
Figura 7.3

Universidad de Castilla- La Mancha Draft Por: Eduardo W. V. Chaves (2012)


Ciudad Real - España
7 ELASTICIDAD LINEAL 399

Solución:
En cualquier punto del cuerpo elástico habrán sólo tensiones normales, σ x , σ y y σ z . La
tensión σ z viene dada por:
200 200 200 N
σz = − =− =− (7.34)
ab 3× 3 9 cm 2
Observemos que debido a la simetría las tensiones σ x y σ y serán iguales, luego:

εx = ε y =
1
E
[ (
σx − ν σy + σz = 0 )] ⇒
1
E
[σ x − ν(σ x + σ z )] = 0
⇒ σ x − ν(σ x + σ z ) = 0 (7.35)
νσ z
⇒ σx =
(1 − ν )

obteniendo así:
νσ z 0,3  200  200 N
σx = = − =− (7.36)
(1 − ν ) (1 − 0,3)  9  21 cm 2

La fuerza que ejerce la pared sobre el cuerpo elástico viene dado por:
200
Fy = σ y a c = − × 3 × 4 = −114,28 N
21
(7.37)
200
Fx = σ x b c = − × 3 × 4 = −114,28 N
21
La deformación ε z viene dada por:

εz =
1
E
[ (
σz − ν σx + σy )]
1
= [σ z − 2νσ x ] (7.38)
E
1  200 200 
= 4 
− + 2 × 0,3 ×  = −8,25 × 10 − 4
2 × 10  9 21 

Luego, la variación de altura viene dada por:


∆ c = ε z c = −8,25 × 10 −4 × 4 = −0,0033cm (7.39)

Ejemplo 7.5
En la Figura 1a se representa un dispositivo de apoyo en fajan de una máquina. Dicho
aparato de apoyo está constituido por un bloque de neopreno de dimensiones ( 50 × 20cm ),
representado en la Figura 7.4b por el elemento ABCD.

Universidad de Castilla- La Mancha Draft Por: Eduardo W. V. Chaves (2012)


Ciudad Real - España
400 PROBLEMAS RESUELTOS DE MECÁNICA DEL MEDIO CONTINUO

a) b)
1,1
1,2
D
D’ C
C’ 1 20
A A’ B B’
1 1,1 x
50

Dimensiones en centímetros - cm

Figura 7.4

Bajo acción de cargas vertical y horizontal que transmite la máquina al apoyo, el bloque de
neopreno se deforma como se indica en la Figura 1b (A’B’C’D’) y aún se puede considerar
que el campo de desplazamiento ( u, v) está dado por unas ecuaciones lineales del tipo:
u = a1 x + b1 y + c1
v = a2 x + b2 y + c2
donde a1 , b1 , c1 , a 2 , b2 , c 2 son constantes a determinar.
Hipótesis:
1 – Material elástico lineal isótropo con el Módulo de elasticidad longitudinal igual a
1
1000 N / cm 2 y el módulo de elasticidad transversal igual a N / cm 2 .
0,0028
2 – Se supondrá que se trata de un estado plano de deformación.
Se pide:
a) Calcular las componentes del tensor de deformación y la deformación volumétrica
en cualquier punto;
b) Calcular las tensiones en cualquier punto;
c) Máxima tensión normal;
d) Determinar el Alargamiento unitario en la dirección de la diagonal AC .

Solución:
 u = a1 x + b1 y + c1
 (7.40)
v = a 2 x + b2 y + c 2
Según Figura 7.4 sacamos que:
u (0;0) = 1 = c1
u (50;0) = 1,1 = 50a1 + 1 ⇒ a1 = 0,002 (7.41)
u (0;20) = 1,1 = 20b1 + 1 ⇒ b1 = 0,005

luego

Universidad de Castilla- La Mancha Draft Por: Eduardo W. V. Chaves (2012)


Ciudad Real - España
7 ELASTICIDAD LINEAL 401

u = 0,002 x + 0,005 y + 1 (7.42)


Para desplazamiento vertical:
v(0;0) = 0 = c 2
u (50;0) = 0 = 50a 2 ⇒ a 2 = 0 (7.43)
u (0;20) = −1 = 20b2 ⇒ b2 = −0,05
v = −0,05 y (7.44)
Luego:
u = 0,002 x + 0,005 y + 1
 (7.45)
v = −0,05 y
a) Deformaciones
∂u ∂v ∂u ∂v
εx = = 0,002 ; εy = = −0,05 ; γ xy = + = 0,005 (7.46)
∂x ∂y ∂y ∂x

Deformación volumétrica lineal (pequeñas deformaciones):


DVL = ε V = ε x + ε y + ε z = I ε = −0,048 (7.47)
b) Tensiones
E E
G= ⇒ν= − 1 = 0,4
2(1 + ν) 2G

σx =
E
(1 + ν)(1 − 2v)
[
(1 − ν )ε x + νε y ]
= 3571,4286 × [(0,6) × 0,002 − 0,4 × 0,05] = −67,1428

σy =
E
(1 + ν)(1 − 2v)
[
(1 − ν )ε y + νε x ]
(7.48)
= 3571,4286 × [(0,6) × (−0,05) + 0,4 × 0,002] = −104,2857
τ xy = Gγ xy
1
= × 0,005 = 1,785714
0,0028
νETr (ε ) E
Una solución alternativa es utilizar: σ ij = δ ij + ε ij , donde:
(1 + ν)(1 − 2ν ) (1 + ν )

 εx 1
2
γ xy 1
2
γ xz   0,002 1
2
(0,005) 0
   
ε ij =  12 γ xy εy 1
2
γ yz  =  12 (0,005) − 0,05 0
 1 γ xz 1
γ yz ε z   0 0 0
2 2

νETr (ε ) N E N
= −68,571429 2 , = 714,285714 2
(1 + ν)(1 − 2ν) cm (1 + ν) cm

Universidad de Castilla- La Mancha Draft Por: Eduardo W. V. Chaves (2012)


Ciudad Real - España
402 PROBLEMAS RESUELTOS DE MECÁNICA DEL MEDIO CONTINUO

1 0 0  0,002 1
2
(0,005) 0
   
σ ij = −68,571429 0 1 0 + 714,285714  12 (0,005) − 0,05 0
0 0 1  0 0 0

 − 67,1428 1,785714 0 
 N
≈  1,785714 − 104,2857 0 
cm 2
 0 0 − 68,571

c) Tensiones principales
2
σx + σ y  σx − σy 
σ (1,2) = ±   + τ 2xy

(7.49)
2  2 
2
− 67,1428 − 104,2857  − 67,1428 + 104,2857  2
σ (1,2) = ±   + 5,35714 =
2  2  (7.50)
= −171,4285 ± 19,328675
 σ1 = −152,099824 N cm 2
 2 (7.51)
σ 2 = −190,757175 N cm
d) Alargamiento unitario
La diagonal ( AC ) mide inicialmente:

L0 = AC = 50 2 + 20 2 = 53,852cm (7.52)
Diagonal deformada

A′C ′ = 50,2 2 + 19 2 = 53,675cm (7.53)


∆L = A′C ′ − AC = −0,177cm (7.54)
El alargamiento unitario es:
∆L − 0,177
ε= = = −0,0033 (7.55)
L0 53,852

Ejemplo 7.6
Considérese una barra de sección cuadrada de lado a y longitud L . Las constantes
elásticas del material se suponen conocidas ( E y ν = 0,25 ). Se pide:
a) Para el caso de carga de la Figura 7.5(a), calcular la energía almacenada (densidad de
energía de deformación) en la barra durante la deformación y la energía total de
deformación;
b) Determinar la energía almacenada en el cambio de volumen y la correspondiente al
cambio de forma;
c) Mismo apartado a) para el caso de la Figura 7.5(b).

Universidad de Castilla- La Mancha Draft Por: Eduardo W. V. Chaves (2012)


Ciudad Real - España
7 ELASTICIDAD LINEAL 403

M
M
P P

L a
a
b) M
a) a
sección

Figura 7.5:
Solución:
Considerando un caso unidimensional:
σx
σ x = Eε x ⇒ ε x = (7.56)
E
Sabemos que la energía de deformación por unidad de volumen viene dada por:
1 1 1 σx 1 P2
Ψ e = σ : ε unidimensi
  onal → Ψ e = σ x ε x = σ x
 = (7.57)
2 2 2 E 2 EA 2
Luego, la energía total U viene dada por:
P2
Ψ ex (volumen) = L × A × ⇒
2 EA 2
(7.58)
P2L
⇒U =
2 EA
La energía de deformación (por unidad de volumen) también puede ser expresada por:
1 1
Ψe = I σ2 − II dev
6(3λ + 2µ) 2µ σ (7.59)
144244 3 1424 3
Ψ e
vol Ψ e
forma

Considerando:
σ x 0 0
P
σij =  0 0 0 → I σ = σ x = (7.60)
A
 0 0 0

Cálculo de II σ dev :
2
1 I2 σ
II σ dev = (3 II σ − I σ2 ) = − σ = − x (7.61)
3 3 3
Luego, la energía de deformación asociada al cambio de volumen:
1 (1 − 2ν) 2 (1 − 2ν) 2
Ψ e vol = I σ2 = Iσ = σx (7.62)
6(3λ + 2µ ) 6E 6E

(1 − 2ν ) P 2
Ψ e vol = (por unidad de volumen) (7.63)
6E A 2

Universidad de Castilla- La Mancha Draft Por: Eduardo W. V. Chaves (2012)


Ciudad Real - España
404 PROBLEMAS RESUELTOS DE MECÁNICA DEL MEDIO CONTINUO

Energía de deformación asociada al cambio de forma:


1 1 2(1 + ν)
Ψ e forma = − II σ dev = − II σ dev
2µ 2 E
(7.64)
(1 + ν)  σ x 
2
=− − 
E  3 

2
(1 + ν) σ x (1 + ν ) P 2
Ψ e forma = = (por unidad de volumen) (7.65)
E 3 3E A 2
Comprobación:
(1 − 2ν ) P 2 (1 + ν) P 2
Ψ e vol + Ψ e forma = + =
6E A 2 3E A 2
P2
= [(1 − 2ν) + 2(1 + ν)] =
6 EA 2
P2
= [1 − 2ν + 2 + 2ν] =
6 EA 2
P2
= =Ψe
2 EA 2
En el caso de sólo a flexión, y además teniendo las siguientes relaciones:
M y a4
σy = I=
I 12
12 M y
σy =
a4
σy
σ y = Eε y ⇒ ε y =
E
Energía de deformación:

1 1  12 M y σ y 
Ψ e = σ y ε y =  

2 2  a4 E 
1  12 M y 12 M y  (7.66)
=  
2  a4 Ea 4 
72 M 2 y 2
=
Ea 8
72 M 2 y 2
Ψe = (7.67)
Ea 8

Ejemplo 7.7
Bajo la restricción de la teoría de pequeñas deformaciones, para un campo de
desplazamientos dado por:
r
u = ( x1 − x3 ) 2 × 10 −3 eˆ 1 + ( x 2 + x3 ) 2 × 10 −3 eˆ 2 − x1 x 2 × 10 −3 eˆ 3

Universidad de Castilla- La Mancha Draft Por: Eduardo W. V. Chaves (2012)


Ciudad Real - España
7 ELASTICIDAD LINEAL 405

a) Determinar el tensor de deformación infinitesimal, el tensor de rotación


infinitesimal en el punto P(0,2 − 1) ;
Solución:
 ∂u1 ∂u1 ∂u1 
 
 ∂x1 ∂x 2 ∂x3  2( x − x )
1 3 0 − 2( x1 − x3 )
∂u i  ∂u 2 ∂u 2 ∂u 2  
= = 0 2( x 2 + x 3 ) 2( x 2 + x3 )  × 10 −3
∂x j  ∂x1 ∂x 2 ∂x3 
   
 ∂u 3 ∂u 3 ∂u 3   − x 2 − x1 0
 ∂x1 ∂x 2 ∂x3 
Aplicado al punto P(0,2 − 1)
 2( x1 − x 3 ) 0 − 2( x1 − x 3 )  2 0 − 2
∂u i 
= 0 2( x 2 + x3 ) 2( x 2 + x 3 )  =  0 2 2  × 10 −3
∂x j
P  − x 2 − x1 0   − 2 0 0 

∂u i
= ε ij + ω ij
∂x j
cuyas componentes:
Tensor de deformación infinitesimal Tensor spin infinitesimal
 2 0 − 2 0 0 0 
1  ∂u ∂u j   
 = 0 2 1  × 10 −3 ωij = 1  ∂u i −
∂u j  
 = 0 0 1  × 10 −3
(7.68)
ε ij =  i +
2  ∂x j ∂xi    2  ∂x j ∂xi   
 − 2 1 0   0 − 1 0 
   

Ejemplo 7.8
Bajo la restricción de la teoría de pequeñas deformaciones, para un campo de
desplazamientos dado por:
r
u = a ( x12 − 5 x 22 ) eˆ 1 + (2 a x1 x 2 )eˆ 2 − (0) eˆ 3
a) Determinar el tensor de deformación lineal, el tensor de rotación lineal;
b) Obtener las deformaciones principales;
c) Dado el módulo de elasticidad transversal G , ¿qué valor toma el módulo de Young E
para que haya equilibrio en cualquier punto?
NOTA: Las fuerzas másicas son despreciables.
Solución:
a) Considerando que u1 = a ( x12 − 5 x 22 ) , u 2 = 2 a x1 x 2 , u 3 = 0 , las componentes del
gradiente de los desplazamientos son:
 2 x1 a − 10ax 2 0
∂u i 
= 2ax 2 2ax1 0
∂x j 
 0 0 0
Descomponiendo de forma aditiva el gradiente de los desplazamientos en una parte
simétrica (tensor de deformación lineal - ε ij ) y en una parte antisimétrica (tensor spin
infinitesimal- ωij ):

Universidad de Castilla- La Mancha Draft Por: Eduardo W. V. Chaves (2012)


Ciudad Real - España
406 PROBLEMAS RESUELTOS DE MECÁNICA DEL MEDIO CONTINUO

∂u i
= ε ij + ωij
∂x j
donde
  2x a − 10ax 2 0  2 x1 a 2ax 2 0 
1  ∂u ∂u j  1  1 
ε ij =  i +  =  2ax 2 2ax1 0 +  − 10ax 2 2ax1 0 
2  ∂x j ∂xi  2 

  0 0 0  0 0 0 
 2 x1 a − 4ax 2 0
=  − 4ax 2 2ax1 0
 0 0 0
y
  2x a − 10ax 2 0  2 x1 a 2ax 2 0   0 − 6ax 2 0
1  ∂u ∂u j  1  1 
ωij =  i −  =  2ax 2 2ax1 0 −  − 10ax 2 2ax1 0  = 6ax 2 0 0
2  ∂x j ∂xi  2 

  0 0 0  0 0 0   0 0 0

b) Deformaciones principales.
2 x1 a − λ − 4ax 2
=0
− 4ax 2 2ax1 − λ
(
⇒ 2 x1 a − λ ) − (4ax )
2
2
2
=0
⇒ (2 x a − λ ) = (4ax )
2 2
1 2

λ1 = 2 x1 a − 4ax 2
⇒ 2 x1 a − λ = ±4ax 2 ⇒ 
λ 2 = 2 x1 a + 4ax 2
c) Partiendo de la ecuación de equilibrio:
r r
∇ ⋅σ + ρ Indicial
{ = 0  
b → σ ij , j = 0 i
r
=0

Expandiendo:
 ∂σ11 ∂σ12 ∂σ13
 + + =0
σ11,1 + σ12, 2 + σ13,3 = 0  ∂x1 ∂x 2 ∂x 3
  ∂σ 21 ∂σ 22 ∂σ 23
σ 21,1 + σ 22, 2 + σ 23,3 = 0 ⇒  + + =0
  ∂x1 ∂x 2 ∂x3
σ 31,1 + σ 32, 2 + σ 33,3 = 0  ∂σ 31 ∂σ 32 ∂σ 33
 + + =0
 ∂x1 ∂x 2 ∂x3

donde las componentes del tensor de tensiones son σ ij = λε kk δ ij + 2µε ij , siendo ε kk = 4 x1 a


luego σ ij = λ 4 x1 aδ ij + 2µε ij
σ11 = λ 4 x1 aδ 11 + 2µε11 = λ 4 x1 a + 2µ (2 x1 a ) = 4 x1 a (λ + µ )
σ12 = λ 4 x1 aδ 12 + 2µε 12 = 2µ(−4ax 2 ) = −8µax 2
σ13 = 0
Luego, la primera ecuación de equilibrio queda:

Universidad de Castilla- La Mancha Draft Por: Eduardo W. V. Chaves (2012)


Ciudad Real - España
7 ELASTICIDAD LINEAL 407

∂σ11 ∂σ12 ∂σ13


+ + =0
∂x1 ∂x 2 ∂x3
⇒ 4a(λ + µ) − 8µa = 0
⇒ λ + µ = 2µ
⇒λ = µ = G
G (3λ + 2G )
Además teniendo en cuenta la relación E = , dicha relación se puede obtener a
λ+G
νE E
través de las relaciones λ = , µ=G = . Luego, concluimos que:
(1 + ν)(1 − 2ν) 2(1 + ν)
G (3λ + 2G ) G (3G + 2G )
E= = = 2,5G
λ+G G+G

Ejemplo 7.9
En un punto de un suelo que podemos considerar como un sólido elástico lineal se conoce
la deformación volumétrica εV = −2 × 10 −3 , la deformación tangencial ε12 = − 3 × 10 −3 y la
deformación horizontal que es nula ε11 = 0 . El suelo está sometido a un estado de
deformación plana en el plano x1 − x 2 . Se pide:
a) Componentes cartesianas del tensor de deformación. Obtener las deformaciones
principales y la orientación de las mismas, definiendo el ángulo que forman con lo
sejes ( x1 , x 2 , x 3 ) .
1
b) Suponiendo que las constante elásticas son E = 50MPa , ν = , obtener las
4
componentes del tensor de tensiones y sus valores principales. Obtener asimismo
las direcciones en las que las tensiones normales y tangenciales son máximas o
mínimas y sus valores.
c) Obtener la densidad de energía elástica de deformación por unidad de volumen.
NOTA: Se recuerdan las expresiones de la elasticidad lineal:
σ = λTr (ε )1 + 2µε
Eν E
con λ = , µ=
(1 + ν)(1 − 2ν) 2(1 + ν)
Solución:
a) Las componentes del tensor de deformación infinitesimal son:
 0 − 3 × 10 −3 0
 
ε ij =  − 3 × 10 −3 ε 22 0
 0 0 0
 
Deformación volumétrica DVL ≈ ε V = I ε = ε11 + ε 22 + ε 33 = −2 × 10 −3 ⇒ ε 22 = −2 × 10 −3 . Con
lo cual:
 0 − 3 0
   0 − 3
ε ij =  − 3 − 2 0 × 10 −3 deformació
  n plana
→ ε ij =   × 10
−3

 0  − 3 − 2 
0 0
 

Universidad de Castilla- La Mancha Draft Por: Eduardo W. V. Chaves (2012)


Ciudad Real - España
408 PROBLEMAS RESUELTOS DE MECÁNICA DEL MEDIO CONTINUO

Deformaciones principales:
0−λ − 3 λ1 = 1
=0 ⇒ λ2 + 2 λ − 3 = 0 ⇒ 
− 3 −2−λ λ 2 = −3
Las deformaciones principales son:
ε1 = 1 × 10 −3 1 × 10 −3 0 
 ⇒ ε′ij =  −3 
ε 2 = −3 × 10 −3  0 − 3 × 10 

b) y

ε1 x′
y′ ε xy
ε yy

ε2 θ

Círculo de Mohr γ
εS = (×10 −3 )
2

(ε N = 0; ε S = 3 )

ε III = −3 εI =1
ε N × 10 −3

(ε N = 0; ε S = −2)

(ε N = −2; ε S = − 3 )

Observemos que el radio es R = (1 − (−3)) / 2 = 2 . Luego:


3
tg(2θ) = ⇒ 2θ = arctg( 3 ) ⇒ θ = 30º
1

Universidad de Castilla- La Mancha Draft Por: Eduardo W. V. Chaves (2012)


Ciudad Real - España
7 ELASTICIDAD LINEAL 409


b) Aplicando σ ij = λTr (ε )δ ij + 2µε ij , donde λ = = 20MPa ,
(1 + ν)(1 − 2ν)
E
µ= = 20 MPa , Tr (ε ) = −2 × 10 −3 . Luego:
2(1 + ν)

1 0 0  0 − 3 0
   
σ ij = λTr (ε ) 0 1 0 + 2µ − 3 − 2 0 × 10 3
0 0 1  0 0 0
 
 − 40 0 0   0 − 3 0 
  

=  0 − 40 
0  + 40 − 3 − 2 0  × 10 −3
MPa
  14 2 43

 0 0 
− 40  0 0 
0  =10 3 Pa
  
Resultando:
 − 40 − 40 3 0 
 
σ ij =  − 40 3 − 120 0  kPa
 0 0 − 40
 
Como el material es isótropo, las direcciones principales de las tensiones coinciden con las
direcciones principales de las deformaciones. Y además, recordemos que los autovalores de
σ y ε están relacionados, cuya expresión se demuestra a continuación.
Reemplazando el valor de σ = λTr (ε )1 + 2µε en la definición de autovalor, autovector:
σ ⋅ nˆ = γ σ nˆ

(λTr (ε )1 + 2µε ) ⋅ nˆ = γ σ nˆ
⇒ λTr (ε )1 ⋅ nˆ + 2µε ⋅ nˆ = γ σ nˆ
⇒ λTr (ε )nˆ + 2µε ⋅ nˆ = γ σ nˆ
⇒ 2µε ⋅ nˆ = γ σ nˆ − λTr (ε )nˆ
⇒ 2µε ⋅ nˆ = (γ σ − λTr (ε ) )nˆ
 γ − λTr (ε ) 
⇒ ε ⋅ nˆ =  σ nˆ
 2 µ 
⇒ ε ⋅ n = γ εn
ˆ ˆ

Luego:
γ σ − λTr (ε )
γε = ⇒ γ σ = 2µγ ε + λTr (ε )

Pudiendo así obtener los autovalores de σ :
γ (σ1) ≡ σ I = 2µγ (ε1) + λTr (ε ) = (40 × 10 6 ) × (1 × 10 −3 ) + (20 × 10 6 ) × (−2 × 10 −3 ) = 0
γ (σ2 ) ≡ σ II = 2µγ (ε2 ) + λTr (ε ) = (40 × 10 6 ) × (0) + (20 × 10 6 ) × (−2 × 10 −3 ) = −40 × 10 3 Pa
γ (σ3) ≡ σ III = 2µγ ε(3) + λTr (ε ) = (40 × 10 6 ) × (−3 × 10 −3 ) + (20 × 10 6 ) × (−2 × 10 −3 ) = −160 × 10 3 Pa

También podemos utilizar la expresión σ = λTr (ε )1 + 2µε en el espacio principal:

Universidad de Castilla- La Mancha Draft Por: Eduardo W. V. Chaves (2012)


Ciudad Real - España
410 PROBLEMAS RESUELTOS DE MECÁNICA DEL MEDIO CONTINUO

 − 40 0 0  1 0 0  0 0 0 
     
σ′ij =   0 − 40 0  + 40 0 − 3 0  × 10
142
−3
3 = 0 − 160
MPa
4 0  kPa
 0 0 − 40 0 0 0  =103 Pa 0 0 − 40

Círculo de Mohr en tensiones:

− 160 − 40 0 σ N (kPa)

1
c) La densidad de energía Ψ e = σ : ε . Podemos utilizar el espacio principal para obtener
2
la densidad de energía, i.e.:
σ1 0 0  0 0 0  ε 1 0 0  1 0 0
0  = 0 − 160 m
 σ2 0   0  × 10 3 Pa ; 0
 ε2 0  = 0 − 3 0 × 10 −3 m
 
 0 0 σ 3  0 0 − 40  0 0 ε 3  0 0 0
1 1
Ψ e = σ ij ε ij = (σ1ε1 + σ 2 ε 2 + σ 3 ε 3 )
2 2
1
[ ] m N m
= (0)(1) + (−160 × 10 3 )(−3 × 10 −3 ) + ( −40 × 10 3 )(0) = 240 Pa = 240 2
2 m m m
J
= 240 3
m

Ejemplo 7.10
Un sólido se halla sometido a deformación plana, siendo las componentes del tensor de
deformación lineal en un determinado punto:
− 2 3 0
ε ij =  3 − 10 0 × 10 −3

 0 0 0
Considérese que el sólido tiene un comportamiento elástico lineal e isótropo, definido por
módulo elástico de Young E = 10MPa y coeficiente de Poisson ν = 0,25 .
Se pide:
a) Obtener la deformación volumétrica y el tensor de deformación desviadora;
b) Obtener las deformaciones principales y las direcciones en que se producen;
c) Obtener las componentes del tensor de tensiones de Cauchy;
d) Obtener las máximas y mínimas tensiones normales;
e) Se sabe que el material rompe cuando en algún plano se alcanza una tensión tangencial
que supere 40 kPa . Verificar si se produce la rotura.

Universidad de Castilla- La Mancha Draft Por: Eduardo W. V. Chaves (2012)


Ciudad Real - España
7 ELASTICIDAD LINEAL 411

Solución:
a) Deformación volumétrica ( εV ):
ε V = I ε = Tr (ε ) = ( −2 − 10) × 10 −3 = −12 × 10 −3

Descomposición aditiva del tensor de deformación ε = ε esf + ε dev , donde la parte esférica:
− 4 0 0 
Tr (ε )
ε ijesf = δ ij =  0 − 4 0  × 10 −3

3
 0 0 − 4
y la parte desviadora queda:
 − 2 3 0  − 4 0 0  2 3 0
 
ε ijdev = ε ij − ε ijesf =   3 − 10 0 −  0 − 4 0   × 10 = 3 − 6 0 × 10 −3
   −3

 0 0 0  0 0 − 4  0 0 4



b) Las deformaciones principales obtenemos al resolver el determinante característico:
−2−λ 3
=0 ⇒ λ2 + 12λ + 11 = 0
3 − 10 − λ
Solución de la ecuación cuadrática:
− (12) ± (12) 2 − 4(1)(11) − 12 ± 10 λ (1) = −1,0
λ (1, 2 ) = = ⇒ 
2(1) 2 λ ( 2 ) = −11

Luego, las deformaciones principales son:


ε1 = −1,0 × 10 −3 ; ε 2 = −11,0 × 10 −3

Direcciones principales (ε ij − λ δ ij )n (jλ ) = 0 i (i, j = 1,2)

Dirección principal asociada al valor principal λ (1) = −1,0 :

 n1(1)  0 − n1 + 3n 2 = 0 ⇒ n1 = 3n 2


(1) (1) (1) (1)
− 2 − (−1) 3
   = ⇒ 
 3 − 10 − (−1) n (21)  0 3n1(1) − 9n (21) = 0

2 2 2 1
restricción n1(1) + n (21) = 1 , con eso obtenemos que (3n (21) ) 2 + n (21) = 1 ⇒ n (21) = , y que
10
3
n1(1) =
10
Dirección principal asociada al valor principal λ (1) = −11,0 :

 n1( 2)  0 9n1 + 3n 2 = 0


( 2) ( 2)
− 2 − (−11) 3
  = ⇒
 3 − 10 − (−11) n (22)  0 3n1( 2 ) + n (22 ) = 0 ⇒ n (22) = −3n1( 2 )

2 2 1 −3
Con la restricción n1( 2) + n (22) = 1 , obtenemos que n1( 2) = , y que n (22) =
10 10
Resumiendo así que:

Universidad de Castilla- La Mancha Draft Por: Eduardo W. V. Chaves (2012)


Ciudad Real - España
412 PROBLEMAS RESUELTOS DE MECÁNICA DEL MEDIO CONTINUO

 3 1 
ε1 = −1 × 10 −3 dirección
 principal
→ nˆ i(1) =  0
 10 10 
 1 −3 
ε 2 = −11 × 10 −3 dirección
 principal
→ nˆ i( 2 ) =  0
 10 10 
ε1 = 0 dirección principal
   → nˆ i(3) = [0 0 1]
c) Las componentes del tensor de tensiones de Cauchy vienen dadas por:
σ ij = λTr (ε )δ ij + 2µε ij

νE E
donde λ = = 4MPa , µ = G = = 4 MPa , Tr (ε ) = −12 × 10 −3 :
(1 + ν)(1 − 2ν ) 2(1 + ν)

 1 0 0 − 2 3 0 
 
σ ij =  4 × (−12) 0 1 0 + 2 × (4)  3 − 10 0  × 10 −3 MPa
  
 0 0 1  0 0 0 

 − 64 24 0 

=  24 − 128 0  kPa
 0 0 − 48

Como el material es isótropo las direcciones principales de las tensiones y deformaciones


coinciden. Las tensiones principales obtenemos trabajando en el espacio principal
σ′ij = λTr (ε )δ ij + 2µε ′ij :

 1 0 0  − 1 0 0 
 
σ′ij =  4 × (−12) 0 1 0 + 2 × (4)  0 − 11 0  × 10 −3 MPa
 
 0 0 1  0 0 0 

 − 56 0 0 

= 0 − 136 0  kPa
 0 0 − 48

d) Dibujamos el círculo de Mohr en tensiones con σ I = −48kPa , σ II = −56kPa ,


σ III = −136kPa :

σ S (kPa )

σ S max = 44

σ II = −56

σ III = −136 σ N (kPa)


σ I = −48

Podemos obtener la tensión tangencial máxima como:

Universidad de Castilla- La Mancha Draft Por: Eduardo W. V. Chaves (2012)


Ciudad Real - España
7 ELASTICIDAD LINEAL 413

σ I − σ III (−48) − (−136)


σ S max = = = 44kPa
2 2
Luego, el material alcanzará la rotura.

Ejemplo 7.11
El estado tensional en un punto del medio continuo viene dado a través de las
componentes del tensor de tensiones de Cauchy por:
 − 26 6 0 
σ ij =  6 9 0  kPa
 0 0 29
Considerando un material elástico, lineal, homogéneo e isótropo. Se pide:
a) Obtener los invariantes principales de σ ;
b) Obtener la parte esférica y desviadora de σ ;
c) Obtener los autovalores y autovectores de σ ;
d) Dibujar el círculo de Mohr en tensiones. Obtener también la tensión normal y la tensión
tangencial máxima.
e) Considerando un régimen de pequeñas deformaciones y teniendo en cuenta que el
material que constituye el medio continuo tiene como propiedades mecánicas
λ = 20000 kPa y µ = 20000kPa ( λ, µ son las constantes de Lamé). Obtener las
componentes del tensor de deformación infinitesimal;
f) Obtener los autovalores y autovectores de ε .
Solución:
I σ = 12 × 10 3
9 0 − 26 0 − 26 6
II σ = × 10 6 + × 10 6 + × 10 6 = −763 × 10 6
0 29 0 29 6 9

III σ = det (σ ) = −7830 × 10 9

Parte esférica y desviadora σ ij = σ ijdev + σ ijesf :


1 ( 29 − 26 + 9)
σm = σ ii = = 4 × 10 3 Pa
3 3
4 0 0 
σ ijhid ≡ σ esf
ij = 0 4 0  kPa
0 0 4 

 − 26 − 4 6 0   − 30 6 0 
σ ijdev 
= 6 9−4 0  =  6 5 0  kPa
 0 0 29 − 4   0 0 25

Resolviendo la ecuación característica, los autovalores son:


σ I = 29kPa , σ II = 10kPa , σ III = −27 kPa :
Los autovectores:

Universidad de Castilla- La Mancha Draft Por: Eduardo W. V. Chaves (2012)


Ciudad Real - España
414 PROBLEMAS RESUELTOS DE MECÁNICA DEL MEDIO CONTINUO

σ I = 29kPa dirección
  principal
 → nˆ i(1) = [0 0 1]
σ II = 10kPa dirección
  principal
 → nˆ i( 2 ) = [0,1644 0,98639 0]
σ III = −27 kPa dirección
  principal
 → nˆ i(3) = [0,98639 − 0,1644 0]

σ S (kPa )
29 − (−27)
σ S max = = 28
2

σ N (kPa)

σ III = −27
σ I = σ N max = 29

σ II = 10

−λ 1
σ ij = λTr (ε )δ ij + 2µε ij inversa
→ ε ij = Tr (σ )δ ij + σ ij
2µ(3λ + 2µ) 2µ
−λ
donde = −5 × 10 −9 ( Pa ) −1 , Tr (σ ) = 1,2 × 10 4 ( Pa)
2µ (3λ + 2µ)

1 0 0 − 26 6 0 
−9 4   −8 
ε ij = (−5 × 10 )(1,2 × 10 ) 0 1 0 + 2,5 × 10  6 9 0  × 10 3
0 0 1  0 0 29
1 0 0 − 26 6 0 
−5   −5 
= −6 × 10 0 1 0 + 2,5 × 10  6 9 0 
0 0 1   0 0 29
 − 7,1 1,5 0 

=  1,5 1,65 0  × 10 − 4
 0 0 6,65
Como el material es isótropo, el tensor de tensiones y el tensor de deformaciones tienen el
mismo espacio principal (mismas direcciones principales), luego:
−λ 1
ε ′ij = Tr (σ )δ ij + σ′ij
2µ(3λ + 2µ) 2µ
1 0 0 29 0 0  66,5 0 0 
−5   −5   
= −6 × 10 0 1 0 + 2,5 × 10  0 10 0  =  0 19 0  × 10 −5
0 0 1   0 0 − 27   0 0 73,5
Las direcciones principales del tensor de deformaciones son las mismas que las del tensor
de tensiones para un material isótropo.

Universidad de Castilla- La Mancha Draft Por: Eduardo W. V. Chaves (2012)


Ciudad Real - España
7 ELASTICIDAD LINEAL 415

Ejemplo 7.12
Demostrar que las ecuaciones constitutivas de tensión, para un material elástico lineal
homogéneo e isótropo, se puede representar por el conjunto de ecuaciones:
σ dev = 2µε dev

 Tr (σ ) = 3κTr (ε )
donde µ = G es la constante de Lamé, κ es el coeficiente de deformación volumétrica.
Solución:
σ = Ce : ε
= [λ1 ⊗ 1 + 2µI] : ε
= λTr (ε )1 + 2µε

σ = σ dev + σ esf = λTr (ε )1 + 2µ(ε dev + ε esf )


Tr (σ )
⇒ σ dev + 1 = λTr (ε )1 + 2µ (ε dev + ε esf )
3
Tr (ε ) Tr (σ )
⇒ σ dev = λTr (ε )1 + 2µε dev + 2µ − 1
3 3
 2µ  Tr (σ )
⇒ σ dev = λ +  Tr (ε )1 + 2µε dev − 1
 3  3
La traza del tensor de tensiones:
Tr (σ ) = σ : 1 = [λTr (ε )1 + 2µε ] : 1 = λTr (ε )3 + 2µTr (ε ) = (3λ + 2µ )Tr (ε )
Con lo cual:
 2µ  Tr (σ )
⇒ σ dev =  λ +  Tr (ε )1 + 2µε −
dev
1
 3  3

⇒ σ dev =  λ +
2µ  (3λ + 2µ )Tr(ε) 1
 Tr (ε )1 + 2µε −
dev

 3  3

⇒ σ dev =  λ +
2µ  (3λ + 2µ )Tr(ε) 1 + 2µε dev
 Tr (ε )1 −
14434444 424444444
3
3
=0

A las ecuaciones σ dev = 2µε dev tenemos que añadir la restricción:


 2µ  (3λ + 2µ )Tr(ε)
λ +  Tr (ε )1 − 1=0
 3  3
 2µ  Tr (σ )
⇒ λ +  Tr (ε )1 − 1=0
 3  3
 2µ 
⇒ Tr (σ )1 = 3 λ +  Tr (ε )1
 3 
⇒ Tr (σ )1 = 3κ Tr (ε )1
o
Tr (σ ) = 3κ Tr (ε )

Universidad de Castilla- La Mancha Draft Por: Eduardo W. V. Chaves (2012)


Ciudad Real - España
416 PROBLEMAS RESUELTOS DE MECÁNICA DEL MEDIO CONTINUO

σ33 σm dev
σ 33

σ 23 σ 23
σ13 σ 23 = + σ13 σ 23

σ13 σ13
σ12 σ 22 σm σ12 σ dev
22
σ12 σ12
σ11 σm dev
σ11

σ ij = λTr (ε )δ ij + 2µε ij
= Tr (σ )δ ij = 3κ Tr (ε )δ ij
+ σ ijdev = 2µε ijdev

ε 33 εm dev
ε 33

ε 23 ε 23
ε13 ε 23 = + ε13 ε 23

ε13 ε13
ε12 ε 22 εm ε12 ε dev
22
ε12 ε12
ε11 εm dev
ε11

Solución Alternativa:
Partiendo de la ecuación constitutiva en tensión para un material elástico, lineal,
homogéneo e isótropo: σ = σ (ε ) = λTr (ε )1 + 2µε . Además como estamos en régimen
linear se cumple que σ = σ (ε ) = σ (ε esf + ε dev ) = σ (ε esf ) + σ (ε dev ) , donde:
σ (ε esf ) = λTr (ε esf )1 + 2µε esf
 Tr (ε )  Tr (ε ) Tr (ε ) 2µ
σ esf = λTr  11 + 2µ 1 = λTr (ε )1 + 2µ 1 = (λ + ) Tr (ε )1 = κ Tr (ε )1
 3  3 3 3
Tr (σ )
1 = κ Tr (ε )1
3
Tr (σ )1 = 3κ Tr (ε )1

σ (ε dev ) = λ Tr (ε dev )1 + 2µε dev = 2µε dev


1424 3
=0

[
Observar que se cumple que Tr σ (ε esf ) = Tr σ esf ] [ ] = Tr[σ ] .

Universidad de Castilla- La Mancha Draft Por: Eduardo W. V. Chaves (2012)


Ciudad Real - España
7 ELASTICIDAD LINEAL 417

NOTA: Es interesante observar que: si en un material isótropo tenemos un estado


puramente esférico de comprensión:
p
− p 0 0 
p σ ij =  0 − p 0 
 ∴ Tr (σ ) = −3 p
 0 0 − p 
p

E
tenemos que Tr (σ ) = 3κ Tr (ε ) < 0 , y teniendo en cuenta que κ = , podemos
3(1 − 2ν)
concluir que: si ν > 0,5 eso implica que κ < 0 y como consecuencia Tr (ε ) > 0 , es decir, una
expansión, lo que no tiene sentido físico que en un material isótropo sometido a un estado
de compresión haya una expansión.

Ejemplo 7.13
Un paralelepípedo de cierto material elástico (coeficiente de Poisson ν = 0,3 y módulo de
elasticidad E = 2 × 10 6 N / m 2 que a cierta temperatura tiene de dimensiones a = 0,10m ,
b = 0,20m , c = 0,30m , ver Figura 7.6, se introduce en una cavidad de anchura b de paredes
rígidas, planas y perfectamente lisas, de tal forma que dos caras opuestas del paralelepípedo
estén en contacto con las paredes de la cavidad.
Una vez en esta posición se eleva la temperatura del prisma en ∆T = 30º C .
Se pide:
1) Calcular los valores de las tensiones principales en los puntos del paralelepípedo.
2) Hallar las componentes de las deformaciones.

Dato: coeficiente de expansión térmico del material es igual a 1,25 × 10 −5 º C −1 .


Solución:
Como el sólido puede deformarse libremente según las direcciones x , z , luego está libre
de tensiones normales σ x = σ z = 0 . El sólido está restringido al movimiento según la
dirección y luego ε y = 0 :

εy =
1
E
[ ] 1
[ ]
σ y − ν(σ x + σ z ) + α∆T = σ y + α∆T = 0
E
⇒ σ y = − Eα∆T

resultando que:
N
σ y = − Eα∆T = −2 × 10 6 × 1,25 × 10 −5 (30) = −750
m2
Componentes del tensor de tensiones:
0 0 0
σ ij = 0 − 750 0 Pa

0 0 0

Universidad de Castilla- La Mancha Draft Por: Eduardo W. V. Chaves (2012)


Ciudad Real - España
418 PROBLEMAS RESUELTOS DE MECÁNICA DEL MEDIO CONTINUO

νσ y
b) εx = εz = + α∆T = 4,875 × 10 − 4
E
El tensor de deformación queda:
 4,875 0 0 

ε ij =  0 0 0  × 10 − 4
 0 0 4,875

Datos:
a
a = 0,10m
b = 0,20m
c = 0,30m
E = 2 × 10 6 N / m 2
ν = 0,3
∆T = 30º C
α = 1,25 × 10 −5 º C −1
c
y

x
Figura 7.6.

Ejemplo 7.14
En el fondo de un recipiente, cuyo hueco interior es prisma recto, de base cuadrada, de
0,10 × 0,10m , se coloca un bloque de caucho sintético de 0,10 × 0,10 × 0,5m , tal como se
indica en la figura. El bloque ajusta perfectamente en el recipiente de paredes rígidas lisas.
Las características elásticas del caucho sintético son E = 2,94 × 10 6 N / m 2 y ν = 0,1 .
Sobre el caucho se vierten 0,004m 3 de mercurio, cuya densidad de masa es 13580kg / m 3 .
Se pide:
a) La altura H que alcanza el mercurio sobre el fondo del recipiente;
b) El estado tensional en un punto genérico del bloque de caucho
NOTA: Despréciese el peso del caucho. Considérese la aceleración de la gravedad
g = 10m / s 2 .

Universidad de Castilla- La Mancha Draft Por: Eduardo W. V. Chaves (2012)


Ciudad Real - España
7 ELASTICIDAD LINEAL 419

z Paredes rígidas
Paredes rígidas

Mercurio

Caucho
L0 = 0,5m

Figura 7.7.
Solución:
Primero calculamos la fuerza total del mercurio ejerce sobre el caucho:
 kg  m  kgm 
F = Vρg = 0,004(m 3 ) × 13580 3  × 10 2  = 543,20 2 ≡ N 
m  s   s 
La tensión normal según la dirección z viene dada por:
F 543,20 N
σz = − =− = −54,320 × 10 3 2
A (0,1 × 0,1) m
Según las direcciones x , y el caucho no se deforma: ε x = ε y = 0

εx =
1
E
[
σ x − ν (σ y + σ z ) = 0 ] ⇒ σ x = ν (σ y + σ z )

1
[
ε y = σ y − ν (σ x + σ z ) = 0
E
] ⇒ σ y = ν (σ x + σ z )

σ y = ν (σ x + σ z )
{[ ]
σ y = ν ν (σ y + σ z ) + σ z }
(ν 2 + ν ) ν
σy = σz = σ z = −6035,55Pa = σ x
2
(1 − ν ) (1 − ν)
La deformación normal según la dirección z :

Universidad de Castilla- La Mancha Draft Por: Eduardo W. V. Chaves (2012)


Ciudad Real - España
420 PROBLEMAS RESUELTOS DE MECÁNICA DEL MEDIO CONTINUO

εz =
1
E
[
σ z − ν (σ x + σ y ) ]
1
= {− 54320 − 0,1[2(−6035,55)]} = −0,0180656
2,94 × 10 6
b) Variación de la longitud del caucho:
∆L = L0 ε z = 0,5 × (− 0,018656 ) = −0,00903m
La altura H que alcanza el mercurio sobre el fondo del recipiente:
H = hmer + ( L0 − ∆L)
donde hmer viene dado por:
0,004
Vmer = b 2 × hmer = 0,004 ⇒ hmer = = 0,4m
0,1 × 0,1
Resultando así que:
H = hmer + ( L0 − ∆L)
= 0,4 + (0,5 − 0,00903)
= 0,891m

Ejemplo 7.15
En un ensayo de laboratorio de un material se han obtenido las siguientes relaciones:

 1   − ν 21   − ν 31 
ε x =  σ x +  σ y +  σ z
 E1   E2   E3 
 − ν 12   1   − ν 32 
ε y =  σ x +  σ y +  σ z (7.69)
 E1   E2   E3 
 − ν 13   − ν 23   1 
ε z =  σ x +  σ y +  σ z
 E1   E2   E3 

donde ν 12 = 0,2 , ν 13 = 0,3 , ν 23 = 0,25 , E1 = 1000 MPa , E 2 = 2000MPa , E3 = 1500MPa .


Sabiendo que el material analizado es un MATERIAL ORTÓTROPO. Obtener los valores
de ν 21 , ν 31 , ν 32 .
Solución:
La matriz constitutiva para un material ortótropo presenta el siguiente formato:

 C11 C12 C13 0 0 0 


C C22 C23 0 0 0 
 12
C C23 C33 0 0 0  Simetría Ortótropa
[C ] =  13  (7.70)
0 0 0 C44 0 0  9 constantes independientes
0 0 0 0 C55 0 
 
 0 0 0 0 0 C66 

Reestructurando (7.69) obtenemos que:

Universidad de Castilla- La Mancha Draft Por: Eduardo W. V. Chaves (2012)


Ciudad Real - España
7 ELASTICIDAD LINEAL 421

  1   − ν 21   − ν 31  
       0 0 0 
E  E2   E3 
 xx    1 
ε   σ xx 
 ε   − ν 12   1   − ν 32   
 yy   E      0 0 0  σ yy 
 ε zz   1   E2   E3   σ 
  =  − ν 13   − ν 23   1    zz  (7.71)
 2 ε xy        0 0 0  σ xy 
2ε yz   E1   E2   E3   σ 
   0 0 0 C44 0 0   yz 
 2ε xz    σ 
 0 0 0 0 C55 0   xz 
 0 0 0 0 0 C66 

Luego para un material ortótropo debe cumplir que:


 − ν 21   − ν 12   − ν 31   − ν 13   − ν 32   − ν 23 
  =   ;   =   ;   =  
 E 2   E1   E 3   E1   E3   E2 
obteniendo así que
ν 21 ν 12 E 2 ν 12 2000 × 0,2
= ⇒ ν 21 = = = 0,4
E2 E1 E1 1000
ν 31 ν 13 E 3 ν 13 1500 × 0,3
= ⇒ ν 31 = = = 0,45
E3 E1 E1 1000
ν 32 ν 23 E 3 ν 23 1500 × 0,25
= ⇒ ν 32 = = = 0,1875
E3 E2 E2 2000

Ejemplo 7.16
Dado un material elástico lineal, homogéneo e isótropo con las siguientes propiedades
elásticas:
E = 71 GPa
G = 26,6 GPa
Determinar las componentes del tensor de deformación y la densidad de energía de
deformación en un punto del cuerpo si las componentes del tensor de tensiones en este
punto son:
 20 − 4 5 
σ ij =  − 4 0 10  MPa
 5 10 15

Solución:
E
Obtenemos el coeficiente de Poisson partiendo de la relación: G=
2(1 + ν )
E
⇒ν= − 1 = 0,335
2G

Universidad de Castilla- La Mancha Draft Por: Eduardo W. V. Chaves (2012)


Ciudad Real - España
422 PROBLEMAS RESUELTOS DE MECÁNICA DEL MEDIO CONTINUO

1
ε 11 = [σ11 − ν (σ 22 + σ 33 )] = 1 9 [20 − 0,335 (0 + 15 )]10 6 = 211 × 10 −6
E 71 × 10
1 1
ε 22 = [σ 22 − ν (σ 11 + σ 33 )] = [0 − 0,335 (20 + 15 )]10 6 = −165 × 10 −6
E 71 × 10 9
1 1
ε 33 = [σ 33 − ν (σ 11 + σ 22 )] = 9
[15 − 0,335 (20 + 0 )]10 6 = 117 × 10 −6
E 71 × 10
1+ ν 1 + 0,335
ε12 = σ 12 = ( −4 × 10 6 ) = 75 × 10 −6
E 71 × 10 9
1+ ν 1 + 0,335
ε13 = σ 13 = 9
(5 × 10 6 ) = 94 × 10 − 6
E 71 × 10
1+ ν 1 + 0,335
ε 23 = σ 23 = (10 × 10 6 ) = 188 × 10 − 6
E 71 × 10 9
luego:
 211 − 75 94 
ε ij = − 75 − 165 188  × 10 − 6
 94 188 117 

La densidad de energía de deformación para un material elástico lineal e isótropo viene


dada por:
1 1 1
Ψ e = ε : C e : ε = ε : σ indicial
 → Ψ e = ε ij σ ij
2 2 2
Considerando la simetría de los tensores de tensión y de deformación la densidad de
energía de deformación resulta:
1
Ψe = [ε11 σ11 + ε 22 σ 22 + ε 33 σ 33 + 2ε12 σ12 + 2ε 23 σ 23 + 2ε13 σ 13 ]
2
1
= [( 211)( 20 ) + ( −165 )(0) + (117 )(15) + 2( −75)( −4) + 2(188)(10) + 2(94)(5) ]
2
= 5637 ,5 J / m 3
Podemos también obtener la energía de deformación utilizando la ecuación:
1 1 1 1
Ψe = I σ2 − − II σ dev = I σ2 + − J2
6(3λ + 2µ) 2µ 6(3λ + 2µ) 2µ

y considerando que: I σ = 3,5 × 10 7 ; II σ = −2,4933 × 1014 ; λ ≈ 5,3804 × 10 10 Pa ; µ = G ,


obtenemos:
Ψ e ≈ 5638 ,03 J / m 3
La diferencia entre los resultados obtenidos es debida a la aproximación numérica.

Ejemplo 7.17
Expresar la energía de deformación en función de los invariantes principales de ε .
Solución:

Universidad de Castilla- La Mancha Draft Por: Eduardo W. V. Chaves (2012)


Ciudad Real - España
7 ELASTICIDAD LINEAL 423

1
Ψe = ε :σ
2
1
= ε : [λTr (ε )1 + 2µ ε ]
2
λTr (ε )
= ε2:3 1 + µ ε :ε
2 1 Tr ( ε )
λ [Tr (ε ) ]2 λ [Tr (ε ) ]2
= + µ ε :ε = + µ Tr (ε ⋅ ε T )
2 2
λ [Tr (ε ) ]2 λ [Tr (ε ) ]2
= + µ Tr (ε ⋅ ε ) = + µ Tr (ε 2 )
2 2
Podemos sumar y restar el término µ[Tr (ε )]2 sin alterar la expresión:
λ [Tr (ε ) ]2
Ψe = + µ[Tr (ε )]2 + µ Tr (ε 2 ) − µ[Tr (ε ) ]2
2
1
{
= (λ + 2µ )[Tr (ε )]2 − µ [Tr (ε )]2 − Tr (ε 2 )
2
}
Considerando que los invariantes principales de ε son I ε = Tr (ε ) ,

II ε =
1
2
{ }
[Tr (ε )]2 − Tr (ε 2 ) , obtenemos que:
1
Ψe = (λ + 2µ )I ε2 − 2µ II ε = Ψ e ( I ε , II ε )
2

Ejemplo 7.18
Se conocen las respuestas de un sólido termoelástico lineal en equilibrio a un sistema de
r r (I ) r (I )
acciones I (b ( I ) , t * en S σ ; u* ; en S ur ; ∆T ( I ) ) y a otro sistema de acciones
r r ( II ) r ( II )
II (b ( II ) , t * en S σ ; u* ; en S ur ; ∆T ( II ) ) . Justificar (demostrar) cuál sería la respuesta
al sistema I + II .
Solución:
Como estamos en el régimen lineal se cumplen que:
r r r
b = b ( I ) + b ( II )
∆T = ∆T ( I ) + ∆T ( II )
r r ( I ) r ( II )
t* = t* + t*
r r ( I ) r ( II )
u* = u* + u*
Lo mismo para los campos:
r r r
u = u ( I ) + u ( II )
ε = ε ( I ) + ε ( II )
σ = σ ( I ) + σ ( II )
Partiendo de las ecuaciones de gobierno del problema termoelástico lineal en equilibrio:
ƒ Ecuaciones de Equilibrio:

Universidad de Castilla- La Mancha Draft Por: Eduardo W. V. Chaves (2012)


Ciudad Real - España
424 PROBLEMAS RESUELTOS DE MECÁNICA DEL MEDIO CONTINUO

r r r
[ r
] [ r
] r
∇ xr ⋅ σ + ρb = ∇ xr ⋅ (σ ( I ) + σ ( II ) ) + ρ (b ( I ) + b ( II ) ) = ∇ xr ⋅ σ ( I ) + ρb ( I ) + ∇ xr ⋅ σ ( II ) + ρb ( II ) = 0

ƒ Ecuaciones Cinemáticas:

ε = ε ( I ) + ε ( II ) =
2
[
1 r r (I ) r
] [1 r r
∇ x u + (∇ xr u ( I ) ) T + ∇ xr u ( II ) + (∇ xr u ( II ) ) T
2
]
=
1
2
{[ r
∇ xr u ( I ) + ∇ xr u ] [
r ( II ) r (I ) r ( II ) T
+ ∇ xr u + ∇ xr u ]}
{ [
1 r r ( I ) r ( II )
= ∇x u +u
2
] [r r T
]} {
2
r rT
+ ∇ xr (u ( I ) + u ( II ) ) = ∇ xr u + [∇ xr u] = ε
1
}
ƒ Ecuación Constitutiva:
σ = C e : ε + M∆T
donde M es el tensor de tensiones térmicas
σ = C e : ε + M∆T = C e : (ε ( I ) + ε ( II ) ) + M (∆T ( I ) + ∆T ( II ) )
= (C e : ε ( I ) + M∆T ( I ) ) + (C e : ε ( II ) + M∆T ( II ) )
= σ ( I ) + σ ( II )
Se comprobando así que se cumplen todas las condiciones. Luego, también se puede
aplicar el principio de la superposición al problema termoelástico lineal, como era de
esperar ya que estamos en el régimen lineal.

Ejemplo 7.19
Considérese una barra de 7,5m de longitud y 0,1m de diámetro que está constituida por un
1
material cuyas propiedades son: E = 2,0 × 10 11 Pa y α = 20 × 10 −6 .
ºC
Inicialmente la barra está a 15º C y la temperatura aumenta a 50 º C .
Se pide:
1) Determinar el alargamiento de la barra considerando que la barra pueda expandirse
libremente;
2) Suponga que la barra ya no puede alongarse libremente porque en sus extremos se han
colocado bloques de hormigón, ver Figura 7.8(b). Obtener la tensión en la barra.
Nota: Considerar el problema en una dimensión.

x
∆L

a) b)

Figura 7.8: Barra bajo efecto térmico.

Universidad de Castilla- La Mancha Draft Por: Eduardo W. V. Chaves (2012)


Ciudad Real - España
7 ELASTICIDAD LINEAL 425

Solución:
1) Para obtener el alargamiento debemos calcular previamente la deformación según la
dirección del eje de la barra:
ε ij = α (∆T )δ ij

Como se trata de un caso unidimensional sólo consideraremos la componente de la


deformación según el eje x , ε 11 = ε x , luego:
ε 11 = ε x = 20 × 10 −6 (50 − 15) = 7 × 10 −4
El alargamiento se obtiene según la integral:
L


∆L = ε x dx = ε x L = 7 × 10 − 4 × 7,5 = 5, 25 × 10 − 3 m
0

Observar que como la barra puede expandirse libremente, ésta está libre de tensión.
2) Si las extremidades no pueden moverse, surgirán tensiones uniformes que vienen dadas
por:
σ x = − Eε x = −2,0 × 1011 × 7 × 10 −4 = −1,4 × 10 8 Pa

Ejemplo 7.20
Dado un material elástico lineal, homogéneo e isótropo con las siguientes propiedades
mecánicas: E = 10 6 Pa (módulo de Young), ν = 0,25 (coeficiente de Poisson),
α = 20 × 10 −6 º C −1 (Coeficiente de dilatancia térmica).
Considérese que en un determinado punto del sólido se conoce el estado tensional y que
viene dado por:
12 0 4
σ ij =  0 0 0 Pa
 4 0 6

Se pide:
a) Determinar las tensiones y direcciones principales; Obtener la tensión tangencial
máxima.
b) Las deformaciones en dicho punto. Determinar también las deformaciones y direcciones
principales;
c) Determinar la densidad de energía.
d) Si ahora, a éste sólido sufre una variación de temperatura ∆T = 50º C , ¿Cual es el estado
de deformación final en este punto?
e) ¿Se puede decir que estamos ante un caso de Tensión Plana?
Solución:
Obtenemos los autovalores al resolver el determinante característico. Además observemos
que ya se conoce un autovalor σ 2 = 0 que está asociado a la dirección nˆ i( 2) = [0 ± 1 0] .
Luego, es suficiente resolver el determinante:
12 − σ 4
=0 ⇒ σ 2 − 18σ + 56 = 0
4 6−σ

Universidad de Castilla- La Mancha Draft Por: Eduardo W. V. Chaves (2012)


Ciudad Real - España
426 PROBLEMAS RESUELTOS DE MECÁNICA DEL MEDIO CONTINUO

Resolviendo la ecuación cuadrática obtenemos que:


18 ± 324 − 224 σ1 = 14
σ (1,3) = ⇒ 
2 σ 3 = 4
14 0 0
σ' ij =  0 0 0 Pa
 0 0 4

A continuación obtenemos las direcciones principales (autovectores que deben ser


versores, vectores unitarios), resultando:
 2 1 
σ1 = 14 autovector
 → nˆ i(1) =  0  = [0,8944 0 0,4472]
 5 5
σ2 = 0 autovector
 → nˆ i( 2) = [0 1 0]
 1 2 
σ3 = 4 autovector
 → nˆ i(3) =  0 −  = [0,4472 0 − 0,8944 ]
 5 5
Haciendo el cambio de nomenclatura tal que σ I > σ II > σ III , tenemos que σ I = 14 ,
σ II = 4 , σ III = 0 .
σ S (Pa )

σ S max = 7

σ N (Pa)

σ III = 0 σ II = 4
σ I = 14

Podemos obtener la tensión tangencial máxima como:


σ I − σ III (14) − (0)
σ S max = = = 7 Pa
2 2
Las componentes del tensor de tensiones de Cauchy vienen dadas por:
−λ 1
σ ij = λTr (ε )δ ij + 2µε ij inversa
→ ε ij = Tr (σ )δ ij + σ ij
2µ(3λ + 2µ) 2µ
−1
Recordar que σ = C e : ε , la inversa ε = C e : σ .
νE E
donde λ= = 4 × 10 5 Pa , µ=G = = 4 × 10 5 Pa , Tr (σ ) = 18 ,
(1 + ν )(1 − 2ν ) 2(1 + ν )
−λ
= −2,5 × 10 − 7 Pa
2µ (3λ + 2µ)

Universidad de Castilla- La Mancha Draft Por: Eduardo W. V. Chaves (2012)


Ciudad Real - España
7 ELASTICIDAD LINEAL 427

 1 0 0 12 0 4 
 
−6  
ε ij =  − 4,5 × 10 0 1 0 + 1,25 × 10  0 − 10 0  
−6 

 0 0 1  4 0 6  

10,5 0 5
=  0 − 4,5 0 × 10 − 6

 5 0 3
Para un material isótropo lineal las direcciones principales de las tensiones y deformaciones
coinciden. Las tensiones principales obtenemos trabajando en el espacio principal
−λ 1
ε ′ij = Tr (σ ) + σ ′ij :
2µ(3λ + 2µ) 2µ

 1 0 0 14 0 0  13 0 0
  
ε ′ij =  − 4,5 × 10 0 1 0 + 1,25 × 10  0 0 0  =  0 − 4,5 0  × 10 −6
−6   −6  
 0 0 1  0 0 4   0 0 0,5

1 1
La energía de deformación viene dada por Ψ e = σ : ε = σ ij ε ij . Podemos utilizar el
2 2
espacio principal donde se cumple que:
13 0 0 14 0 0 
ε ′ij =  0 − 4,5 0  × 10 − 6
 ; σ ′ij =  0 0 0 
 0 0 0,5  0 0 4

Luego, podemos utilizar el espacio principal para obtener la densidad de energía:


1 1
Ψ e = σ ij ε ij = [σ11′ ε11′ + σ ′33 ε ′33 ] = 92 × 10 −6 J3
2 2 m
Utilizando el principio de la superposición, podemos decir que:
ε ij = ε ij (σ ) + ε ij (∆T )
= ε ij (σ ) + α∆Tδ ij
Luego,
10,5 0 5 1 0 0 1010,5 0 5 
  −6 −6   
ε ′ij =  0 − 4,5 0 × 10 + 20 × 10 (50) 0 1 0 =  0 995,5 0  × 10 −6
 5 0 3 0 0 1  5 0 1003
Las direcciones principales del tensor de deformación infinitesimal son las mismas del
tensor de tensiones.
e) No podemos decir que se trata de un estado de tensión plana ya que no tenemos
información del estado tensional de todo el medio continuo. Sólo estaremos en el caso de
tensión plana cuando el CAMPO de tensión es independiente de una dirección.

Ejemplo 7.21
Considérese una barra donde en una de las extremidades se aplica una fuerza igual a
6000 N como se indica en la figura siguiente:

Universidad de Castilla- La Mancha Draft Por: Eduardo W. V. Chaves (2012)


Ciudad Real - España
428 PROBLEMAS RESUELTOS DE MECÁNICA DEL MEDIO CONTINUO

1m
100 m
1m =

y, v
6000
σy =
1×1

x, u F = 6000 N

z, w

Determinar ε x , ε y , ε z y el cambio de longitud en las dimensiones de la barra. Considere


que la barra está constituida por un material cuyas propiedades elásticas son:
Módulo de Young: E = 10 7 Pa ;
Coeficiente de Poisson: ν = 0,3
Considerar el material elástico, lineal, homogéneo e isótropo y régimen de pequeñas
deformaciones.
Solución:
Utilizando las expresiones de las deformaciones normales:

εx =
1
[ ( ν
σx − ν σy + σz = − σy = −)] (0,3)(6000 )
= −0,00018
E E 10 7
σ y 6000
1
[
ε y = σ y − ν (σ x + σ z ) = ]
= = 0,0006
E E 10 7
1
[ ( ν
)]
ε z = σ z − ν σ x + σ y = − σ y = −0,00018
E E
Los cambios de longitud en las dimensiones de la sección son:
u = w = −0,00018 × 1 = −1,8 × 10 −4 m
y de la longitud:
v = 0,0006 × 100 = 6,0 × 10 −2 m

Ejemplo 7.22
Una roseta a 45 º , como se indica en la Figura 7.9, destinada a medir la deformación en una
parte de una estructura, proporciona las siguientes lecturas:

Universidad de Castilla- La Mancha Draft Por: Eduardo W. V. Chaves (2012)


Ciudad Real - España
7 ELASTICIDAD LINEAL 429

ε x = 0,33 × 10 −3
ε ′x = 0,22 × 10 − 3
ε y = −0,05 × 10 − 3

¿Cuál es la tensión de corte máxima en el punto en cuestión? Sabiendo que el material


(elástico lineal e isótropo) que constituye la estructura tiene las siguientes propiedades
elásticas: E = 29000 Pa (Módulo de Young); ν = 0,3 (Coeficiente de Poisson). Se pide:
a) Determinar las deformaciones principales y las direcciones principales de las
deformaciones;
b) Determinar las tensiones principales y las direcciones principales de las tensiones.
c) ¿Que conclusión se puede sacar de las direcciones principales de las tensiones y de las
deformaciones?
Nota: Considerar el caso de deformación plana y el material elástico, lineal e isótropo.

x′

45º
45º
x
Figura 7.9: Roseta.

Solución:
Primero tenemos que obtener las componentes del tensor de deformación en el sistema
x, y, z . Para ello, utilizaremos la ley de transformación de coordenadas para obtener la
componente γ xy = 2ε12 . Recordando que en el caso bidimensional la componente normal
puede obtenerse como:
ε 11 + ε 22 ε 11 − ε 22
′ =
ε 11 + cos( 2θ) + ε 12 sin( 2θ) (7.72)
2 2

cuya expresión fue obtenido a través de la transformación de coordenadas:



 ε11 ′
ε 12 0  cos θ sin θ 0  ε11 ε 12 0 cos θ − sin θ 0
ε ′ ε ′22 0 =  − sin θ cos θ 0 ε12 ε 22 0  sin θ cos θ 0
 12
 0 0 0  0 0 1  0 0 0  0 0 1
Es interesante ver también el Ejemplo 1.96 (capítulo 1)
La expresión (7.72) en notación ingenieril:
εx + εy εx − εy γ xy
ε ′x = + cos( 2θ) + sin( 2θ)
2 2 2
Despejando γ xy obtenemos:

Universidad de Castilla- La Mancha Draft Por: Eduardo W. V. Chaves (2012)


Ciudad Real - España
430 PROBLEMAS RESUELTOS DE MECÁNICA DEL MEDIO CONTINUO

 ε ′x − ε x cos 2 ( 2θ) − ε y sin 2 ( 2θ) 


γ xy = 2  = 0,16 × 10 − 3
 sin( 2 θ ) 
 
Luego:
 0,33 0,08 0 
ε ij = 0,08 − 0,05 0  × 10 −3
 0 0 0 

Las tensiones:

σx =
E
(1 + ν )(1 − 2ν )
[ ]
(1 − 2ν )ε x + νε y = 12,0462 Pa

σy =
E
(1 + ν )(1 − 2ν )
[ ]
(1 − 2ν )ε y + νε x = 3,5692 Pa

E
τ xy = γ xy = 1,7846 Pa
2(1 + ν )

σz =

(1 + ν )(1 − 2ν )
[ ]
ε x + ε y = 4,684 Pa

Tensión de corte máxima:


2
 σx + σy 
τ max =   + τ 2xy = 4,5988 Pa

 2 
a) La ecuación característica para el tensor de deformación es:
λ 2 − 0, 28 λ − 2,29 × 10 −2 = 0 (× 10 −3 )
Los autovalores (las deformaciones principales) vienen dados por:
ε 1 = 0,346155 × 10 −3
ε 2 = −0,06615528 × 10 − 3
Los autovectores del tensor de deformación:
ε1 ⇒  0,9802 0,1979 0
ε 2 ⇒  − 0,1979 0,9802 0 
ε 3 ⇒  0 0 1 

b) Dadas las componentes del tensor de tensión:


12,0462 1,7846 0 
σ ij =  1,7846 3,5692 0  Pa
 0 0 4,684 

A través del determinante característico podemos obtener los autovalores, tensiones


principales:
σ 1 = 12,40654
σ 2 = 3,208843
σ 3 = 4,684
Los autovectores del tensor de tensiones son:

Universidad de Castilla- La Mancha Draft Por: Eduardo W. V. Chaves (2012)


Ciudad Real - España
7 ELASTICIDAD LINEAL 431

σ1 ⇒  0,9802 0,1979 0 
σ 2 ⇒ − 0,1979 0,9802 0 
σ 3 ⇒  0 0 1 

Comparando los autovectores del tensor de tensiones y de deformaciones concluimos que


son los mismos.
Las direcciones principales de tensión y deformación son coincidentes sólo para el caso de
material isótropo.
b) Solución Alternativa para la Componentes del tensor de tensiones:
Conocidas las componentes del tensor de deformaciones:
 0,33 0,08 0 
ε ij = 0,08 − 0,05 0  × 10 −3
 0 0 0 

Aplicamos la ecuación constitutiva: σ ij = λTr (ε )δ ij + 2µε ij , donde las constantes de Lamé


vienen dadas por:
νE
λ= = 16,7307692 × 10 3 Pa
(1 + ν )(1 − 2ν )
E
µ= = 11,15384615 × 10 3 Pa
2(1 + ν)

y Tr (ε ) = 0,27999972 × 10 −3 ≈ 0,28 × 10 −3 , con lo cual: σ ij = λTr (ε )δ ij + 2µε ij viene dado


por:
1 0 0  ε11 ε 12 ε13  1 0 0  0,33 0,08 0
    
σ ij = λTr (ε ) 0 1 0 + 2µ ε12 ε 22 ε 23  = λTr (ε ) 0 1 0 + 2µ 0,08 − 0,05 0 × 10 −3

0 0 1 ε 13 ε 23 ε 33  0 0 1  0 0 0
 12,0461 1,784615 0 

= 1,784615 3,5692 0  Pa
 0 0 4,6846
Como el material es isótropo σ y ε comparten las mismas direcciones principales, luego
podemos utilizar la misma expresión σ′ij = λTr (ε )δ ij + 2µε ′ij en el espacio principal, i.e.:

1 0 0 ε 1 0 0  1 0 0 0,346155 0 0
      
σ ′ij = λTr (ε ) 0 1 0 + 2µ  0 ε 2 0  = λTr (ε ) 0 1 0 + 2µ  0 − 0,0662 0 × 10 −3
0 0 1  0 0 ε 3  0 0 1  0 0 0
12,40752 0 0 
= 0 3,20783 0  Pa
 0 0 4,6846

Universidad de Castilla- La Mancha Draft Por: Eduardo W. V. Chaves (2012)


Ciudad Real - España
432 PROBLEMAS RESUELTOS DE MECÁNICA DEL MEDIO CONTINUO

Ejemplo 7.23
Una delta de roseta (aparato para obtener la deformación) tiene la forma de un triángulo
equilátero, y registra deformaciones longitudinales en las direcciones x1 , x1′ y x1′′ como se
muestra en la Figura 10.

x2
x1′′ x1′

30 º
60 º
30 º

60º 60 º
x1
Figura 10
Si las deformaciones medidas en estas direcciones son:
ε11 = −4 × 10 −4
′ = 1 × 10 − 4
ε11
′′ = 4 × 10 − 4
ε11
Determinar ε 22 = ε y , 2ε12 = γ xy , ε ′22 ≡ ε ′y . Mostrar que ε11 + ε 22 = ε11
′ + ε ′22 .

Hipótesis: Considerar caso de deformación plana.


Solución:
Utilizando la ley de transformación de las componentes de un tensor de segundo orden que
es independiente de una dirección, podemos decir que se cumple que:
ε 11 + ε 22 ε 11 − ε 22
′ =
ε 11 + cos( 2θ1 ) + ε 12 sin( 2θ1 ) (7.73)
2 2

ε 11 + ε 22 ε11 − ε 22
′′ =
ε 11 + cos( 2θ 2 ) + ε 12 sin( 2θ 2 ) (7.74)
2 2

donde θ1 = 60º y θ 2 = 120º . Luego, combinando las expresiones anteriores, eliminamos


ε 12 , resultando:

2 ε 
ε 22 =  ε 11 ′′ − 11  = 4,66667 × 10 − 4
′ + ε 11
3 2 

Una vez obtenido el valor de ε 22 = 4,66667 × 10 −4 , podemos reemplazar en la ecuación


(7.73) y obtenemos que:

Universidad de Castilla- La Mancha Draft Por: Eduardo W. V. Chaves (2012)


Ciudad Real - España
7 ELASTICIDAD LINEAL 433

1
γ xy = 2ε 12 = (4ε11′ − ε11 − 3ε 22 ) = −3,46410 × 10 − 4
3
ε 12 = −1,73205 × 10 − 4
Para obtener ε ′22 , primero determinando el ángulo de giro con respecto a x1 que es
θ 3 = 60º +90º = 150º , resultando:
ε 11 + ε 22 ε 11 − ε 22
ε ′22 = + cos( 2θ 3 ) + ε 12 sin( 2θ 3 ) = −0,33333 × 10 − 4
2 2
Comprobando así que:
′ + ε ′22 = 0,66667 × 10 −4
ε 11 + ε 22 = ε 11

Ejemplo 7.24
Considérese una sección de una presa que presenta el campo de desplazamiento dado por:

y, v

u ( x, y ) = −4 x 2 − y 2 + 2 xy + 2

v ( x, y ) = −4 y 2 − x 2 + 2 xy + 5 x, u

El material que constituye dicha estructura presenta las siguientes propiedades elásticas:
E = 100 MPa , G = 35,7 MPa , ν = 0, 4 y está sometido a un nivel de carga tal que se puede
considerar que está en el régimen de pequeñas deformaciones.
Se pide:
a) Obtener el campo de tensión;
b) Demostrar que si se cumplen las ecuaciones de equilibrio para el campo de
desplazamiento dado.
Solución:
a) Cálculo de las componentes del tensor de deformación:
∂u ∂v ∂u ∂v
εx = = −8 x + 2 y ; εy = = −8 y + 2 x γ xy = + =0
∂x ∂y ∂y ∂x
Luego, las componentes del tensor de deformación quedan:
− 8 x + 2 y 0 0
ε ij =  0 − 8 y + 2 x 0 
 0 0 0 

b) Para una presa, como ya hemos visto, podemos analizarla según la aproximación del
estado de deformación plana:

Universidad de Castilla- La Mancha Draft Por: Eduardo W. V. Chaves (2012)


Ciudad Real - España
434 PROBLEMAS RESUELTOS DE MECÁNICA DEL MEDIO CONTINUO

σ x   
1 − ν ν 0   εx 
  E  
 σ y  = (1 + ν )(1 − 2ν )  ν 1− ν 0  εy 
 τ xy  1− ν  
   0 0  γ xy
 2  
0,6 0,4 0  − 8 x + 2 y 
= 357 ,1428 0,4 0,6 0   − 8 y + 2 x  MPa
 0 0 0,3  0 

σ x  − 4 x − 2 y 
 
⇒  σ y  = 357 ,1428  − 2 x − 4 y  MPa
 τ xy   0 
 

σz =
(1 + ν )(1 − 2ν )
( )
ε x + ε y = 357 ,1428 × [( −8 x + 2 y ) + ( −8 y + 2 x )]

Las ecuaciones de equilibro quedan:


 ∂σ x ∂τ xy ∂τ xz
 + + + ρb x = 0 
 ∂ x ∂y ∂ z
− 4 + 0 + 0 + 0 ≠ 0 No cumple
 ∂τ xy ∂σ y ∂τ yz 

 ∂x
+
∂y
+
∂z
+ ρb y = 0 ⇒ 0 − 4 + 0 + 0 ≠ 0 No cumple


∂σ z
 ∂τ ∂τ yz ∂σ z 0 + 0 + +0=0
 xz + + + ρb z = 0  ∂z
 ∂x ∂y ∂z

Lo que indica que el campo de desplazamientos dado no cumple las ecuaciones de


equilibrio.

Ejemplo 7.25

Se considera un prisma cuadrangular regular cuyo material tiene de módulo de elasticidad


E = 27,44 × 10 5 N / cm 2 y coeficiente de Poisson ν = 0,1 . La longitud del lado de la sección
recta es a = 20cm . En ambas bases del prisma se colocan dos placas perfectamente lisas y
rígidas, de peso despreciable, unidas entre sí mediante cuatro cables de sección A1 = 1cm 2 y
módulo de elasticidad E1 = 19,6 × 10 6 N / cm 2 de longitudes iguales a la altura del prisma
l = 1m , simétricamente dispuesto, como indica en la Figura 7.11.
Sobre dos caras laterales opuestas del prisma se aplica una fuerza de compresión uniforme
p = 7350 N / cm 2 . Se pide calcular:
1. Tensión σ C en los cables;
2. Tensiones principales en el prisma;
3. Variación de volumen experimentada por el prisma.

Universidad de Castilla- La Mancha Draft Por: Eduardo W. V. Chaves (2012)


Ciudad Real - España
7 ELASTICIDAD LINEAL 435

a) Configuración de referencia
b) Configuración deformada
z

∆l p
p p

l = 1m

a
y
a

Figura 7.11
Solución:
Verifiquemos que el cable y el prisma tienen que deformarse, según dirección z , de igual
manera.
ε Pz = ε Cz
En el cable se cumple que:
σC
σ C = E1ε Cz ⇒ ε Cz =
E1
El campo de tensiones en el prisma vienen dados por:
 
0 0 0 
P 
σ ij = 0 − p 0 

0 0 − 4σ C A1 
 a 2 
Deformación del prisma según dirección z :

ε Pz =
1
[ (
σz − ν σx + σy =
1
)]  4σ c A1
− +ν

p
E E  a2 
Aplicando que ε Pz = ε Cz :
ε Pz = ε Cz
1  4σ c A1  σ
− +ν p = C
E  a2  E1
Tras algunas manipulaciones algebraicas obtenemos la tensión en el cable:

Universidad de Castilla- La Mancha Draft Por: Eduardo W. V. Chaves (2012)


Ciudad Real - España
436 PROBLEMAS RESUELTOS DE MECÁNICA DEL MEDIO CONTINUO

νE1 pa 2 0,1 × 19,6 × 10 6 × 7350 × 20 2 N


σc = = = 4900 2
( Ea 2 + 4 E1 A1 ) 5 2
(27,44 × 10 × 20 + 4 × 19,6 × 10 × 1)6
cm
La tensión normal según dirección z en el prisma queda:
4σ C A1 4 × 4900 × 1 N
σ Pz = − 2
=− 2
= −49 2
a 20 cm
0 0 0 
 N
σ ijP = 0 − 7350 0 
cm 2
0 0 − 49
Variación de volumen en el prisma:
∆V = ε V V 0
donde εV = I ε es la deformación volumétrica lineal (pequeñas deformaciones):
σx + σy + σz
εV = I ε = ε x + ε y + ε z = (1 − 2ν ) = −2,12857 × 10 −3
E
y V0 = 4 × 10 4 cm 3 es el volumen del prisma, resultando:
∆V = ε V V0 = (−2,12857 × 10 −3 )(4 × 10 4 ) = −85,1428cm 3

Ejemplo 7.26
Dos paralelepípedos iguales del mismo material y de dimensiones a × b × c , se colocan a
uno y otro lado de una placa lisa rígida adosados a ella por sus caras a × c , de tal forma que
sus ejes de simetría perpendiculares a dichas caras sean coincidentes. Ambos
paralelepípedos, junto con la placa, se introducen en una ranura de anchura igual a dos
veces la longitud de la arista b más el espesor de la placa. Las paredes de la ranura son
planas, rígidas y perfectamente lisas.
Se aplican respectivamente a los bloques en sus caras superiores y perpendiculares a ellas
fuerzas uniformemente repartidas p1 y p 2 por unidad de superficie.
Conociendo el módulo de elasticidad E y el coeficiente de Poisson ν , se pide calcular:
a) Las tensiones principales en ambos bloques
b) Las variaciones de longitud de las aristas de los bloques.

Universidad de Castilla- La Mancha Draft Por: Eduardo W. V. Chaves (2012)


Ciudad Real - España
7 ELASTICIDAD LINEAL 437

a
p1 p2

1 2

c
y

b b

x
Figura 7.12.

Solución:
Prisma 1
σ (x1) = 0 ; σ (y1) ; σ (z1) = − p1

Prisma 2
σ (x2 ) = 0 ; σ (y2) ; σ (z2 ) = − p 2

Por compatibilidad de tensión:


σ (y1) = σ (y2 ) = σ y

ε (y1) + ε (y2 ) = 0


1 (1)
E
[ ( 1
)] [
σ y − ν σ (x1) + σ (z1) + σ (y2 ) − ν σ (x2 ) + σ (z2 ) = 0
E
( )]
[ (1)
] [
⇒ σy − ν σz + σy − ν σz = 0 (2)
]
⇒ [σ + ν p ] + [σ
y 1 y + ν p2 = 0 ]
Resultando
ν( p1 + p 2 )
σy = −
2
Prisma 1:
ν( p1 + p 2 )
σ (x1) = 0 ; σ (y1) = − ; σ (z1) = − p1
2
Prisma 2:

Universidad de Castilla- La Mancha Draft Por: Eduardo W. V. Chaves (2012)


Ciudad Real - España
438 PROBLEMAS RESUELTOS DE MECÁNICA DEL MEDIO CONTINUO

ν( p1 + p 2 )
σ (x2 ) = 0 ; σ (y2) = − ; σ (z2 ) = − p 2
2
Las deformaciones en cada prisma viene dadas por:
Prisma 1:

ε (x1) =
1 (1)
E
[ (
σ x − ν σ (y1) + σ (z1) =
ν
2E
)]
[ν( p1 + p 2 ) + 2 p1 ]
ε (y1)
1
[ (
= σ (y1) − ν σ (x1) + σ (z1) =
E
ν
2E
)]
( p1 − p 2 )

ε (z1)
1
[ (
= σ (z1) − ν σ (x1) + σ (y1) =
E
)]
1 2
2E
[
ν ( p1 + p 2 ) − 2 p1 ]
Prisma 2:

ε (x2 ) =
E
[
1 ( 2)
(
σ x − ν σ (y2 ) + σ (z2 ) = )]
ν
2E
[ν( p1 + p 2 ) + 2 p 2 ]
ε (y2 )
1
[ (
= σ (y2) − ν σ (x2 ) + σ (z2 ) =
E
)]
ν
2E
( p 2 − p1 )

ε (z2 )
1
[ (
= σ (z2) − ν σ (x2 ) + σ (y2 ) =
E
)]
1 2
2E
[
ν ( p1 + p 2 ) − 2 p 2 ]

Variación de las aristas:


Prisma 1 Prisma 2

aν νa
∆a (1) = ε (x1) a = [ν( p1 + p 2 ) + 2 p1 ] ∆a ( 2 ) = ε (x2) a = [ν( p1 + p 2 ) + 2 p 2 ]
2E 2E
νb νb (7.75)
∆b (1) = ε (y1) b = ( p1 − p 2 ) ∆b ( 2 ) = ε (y2 ) b = ( p 2 − p1 )
2E 2E
∆c (1) = ε (z1) c =
2E
[
c 2
ν ( p1 + p 2 ) − 2 p1 ] ∆c ( 2 ) = ε (z2 ) c =
2E
[
c 2
ν ( p1 + p 2 ) − 2 p 2 ]

Ejemplo 7.27
Una presa de gravedad de perfil triangular está construida mediante hormigón de peso
5
específico γ ( γ es el peso específico del agua), siendo su forma y dimensiones
2
transversales las indicadas en la Figura 7.13. La solución de tensiones (campo de tensión)
para este problema de deformación plana es conocida y viene dada por:
σ11 = − γx 2
γ
σ 22 = ( x1 − 3 x 2 )
2
σ12 = − γx1
1
Considerar: Coeficiente de Poisson: ν = ; Módulo de elasticidad longitudinal E .
4
Se pide:

Universidad de Castilla- La Mancha Draft Por: Eduardo W. V. Chaves (2012)


Ciudad Real - España
7 ELASTICIDAD LINEAL 439

a) Representación gráfica de las fuerzas de superficies que debe ejercer el terreno sobre el
lado AB , para que la solución indicada sea la correcta;
b) Tensiones Principales en los puntos A y B . A partir del círculo del Mohr en tensiones,
obtener los valores extremos de las tensiones en los respectivos puntos.
c) Obtener el campo de deformación en la presa.

O x1

45º γ = ρg
g -aceleración de la gravedad
ρ - densidad de masa
h
kg m N
[γ ] = 3 2
= 3
γ 5 m s m
γ
2

A B

x2
Figura 7.13.
NOTA: Aunque en la literatura γ se denomina de peso específico, en realidad γ es el
r r r
módulo de la fuerza másica por unidad de volumen, i.e. γ = p = ρb = ρg , donde b es la

fuerza másica por unidad de masa b = [r ] N m


= 2 . Recordar que, en el Sistema Internacional
kg s
de Unidades el término “específico” se refiere siempre a “por unidad de masa”, que no es el caso
de γ , el término correcto sería densidad de peso, por decir algo, ya que en el SI el término
“densidad” siempre se refiere a “por unidad de volumen”.
Solución:
a) Campo de tensión y de deformación en la presa:
 − γx 2 − γx1 0   ε11 ε 12 0
 γ 
σ ij =  − γx1 ( x1 − 3 x 2 ) 0  ; ε ij = ε12 ε 22 0
 2 
σ 33   0 0 0
 0 0

Obtenemos la fuerza de superficie a través del vector tracción t (n) = σ ⋅ nˆ . Para el lado AB
ˆ

tenemos como normal el vector nˆ i = [0,1,0] :


 t 1 ( AB )   − γx 2 − γx1 0  0  − γx1 
 ( AB )   γ    γ 
t 2  =  − γx1 ( x1 − 3 x 2 ) 0  1 =  ( x1 − 3 x 2 )
 t ( AB )   2  2 
 3   0 0 σ 33  0  0 

Universidad de Castilla- La Mancha Draft Por: Eduardo W. V. Chaves (2012)


Ciudad Real - España
440 PROBLEMAS RESUELTOS DE MECÁNICA DEL MEDIO CONTINUO

Fuerza de superficie en la base de la presa:

h h
A B A B
− 3γh
− γh 2 − γh

( AB ) ( AB )
t1 (según dirección x1 ) t2 (según dirección x 2 )

O x1

45º

5
γ
2

B
A
ˆ
t (n)
x2

b) Fijemos que σ 33 ya es una tensión principal. Partiendo de σ = λTr (ε )1 + 2µε podemos


obtener σ 33 :
σ ij = λTr (ε )δ ij + 2µε ij ⇒ σ 33 = λTr (ε )δ 33 + 2µε 33 ⇒ σ 33 = λTr (ε )

A continuación determinamos Tr (ε ) . Para ellos hacemos el doble producto escalar de


σ = λTr (ε )1 + 2µε con el tensor identidad de segundo orden, resultando:
σ : 1 = λTr (ε )1 : 1 + 2µε : 1
⇒ Tr (σ ) = 3λTr (ε ) + 2µTr (ε ) = [3λ + 2µ ]Tr (ε )
Tr (σ ) σ + σ 22 + σ 33
⇒ Tr (ε ) = = 11
3λ + 2µ 3λ + 2µ
Luego la componente σ 33 queda definida como:

Universidad de Castilla- La Mancha Draft Por: Eduardo W. V. Chaves (2012)


Ciudad Real - España
7 ELASTICIDAD LINEAL 441

λ
σ 33 = λTr (ε ) = (σ11 + σ 22 + σ 33 )
3λ + 2µ
λ λ
⇒ σ 33 − σ 33 = (σ11 + σ 22 )
3λ + 2µ 3λ + 2µ
 λ  λ
⇒ σ 33 1 −  = (σ11 + σ 22 )
 3λ + 2µ  3λ + 2µ
λ
⇒ σ 33 = (σ11 + σ 22 )
2(λ + µ)
Reemplazando los valores de σ11 , σ 22 , obtenemos que:
λ
σ 33 = (σ11 + σ 22 ) = ν − γx 2 + γ ( x1 − 3x 2 ) = γ ν [x1 − 5 x 2 ] = γ [x1 − 5 x 2 ]
2(λ + µ)  2  2 8
λ
donde hemos considerado que ν = .
2(λ + µ)
El estado tensional en el punto A( x1 = 0; x 2 = h) viene dado por:
     
− γx 2 − γx1 0  − γh 0 0  − 1 0 0 
 γ   − 3hγ   −3 
σ ij( A) =  − γx1 ( x1 − 3 x 2 ) 0 = 0 0 =0 0  hγ
 2   2   2 
γ − 5hγ   − 5
 0

0 [x1 − 5 x 2 ]  0 0 0 0
8   8   8 
Fijemos que estas componentes ya son las tensiones principales en el punto A .
Círculo de Mohr en tensiones en el punto A :

σ S ( γh)

σ S max = 0,4375

− 1,5 −1 − 0,625 σ N (γh)

El estado tensional en el punto B( x1 = h; x 2 = h) viene dado por:


   
− γx 2 − γx1 0  − γh − γh 0  − 1 − 1 0 
 γ   γ   
σ ij( B ) =  − γx1 ( x1 − 3 x 2 ) 0  = − γh (h − 3h) 0  = − 1 − 1 0  γh
 2   2   − 1
γ γ 0 0 
 0

0 [x1 − 5 x 2 ]  0 0 [h − 5h] 
 2 
8   8 

Universidad de Castilla- La Mancha Draft Por: Eduardo W. V. Chaves (2012)


Ciudad Real - España
442 PROBLEMAS RESUELTOS DE MECÁNICA DEL MEDIO CONTINUO

Las tensiones principales en el punto B( x1 = h; x 2 = h) vienen dadas por:


−1− σ −1
=0 ⇒ (−1 − σ) 2 − 1 = 0 ⇒ (−1 − σ) 2 = 1 ⇒ ( −1 − σ) = ±1
−1 −1− σ
σ1 = −2
⇒
σ 2 = 0

σ S ( γh)

σ S max = 1

−2 − 0,5 0 σ N (γh)

c) Podemos obtener la expresión del campo de deformación partiendo de la expresión:


σ = λTr (ε )1 + 2µε :
σ = λTr (ε )1 + 2µε
⇒ 2µε = σ − λTr (ε )1
1 λ
⇒ε= σ− Tr (ε )1
2µ 2µ
Tr (σ )
Recordemos que anteriormente hemos obtenido que Tr (ε ) = , luego:
3λ + 2µ
1 λ
ε= σ− Tr (ε )1
2µ 2µ
1 λ
= σ− Tr (σ )1
2µ 2µ(3λ + 2µ)
También podemos expresar la relación anterior en función de los parámetros E y ν :
E 1 (1 + ν)
µ=G = ⇒ =
2(1 + ν ) 2µ E
µ(3λ + 2µ) 1 1
E= ⇒ =
λ+µ µ(3λ + 2µ) E (λ + µ)
λ λ 1 ν
= =
2µ (3λ + 2µ) 2 E (λ + µ) E
Luego:
1 λ
ε= σ− Tr (σ )1
2µ 2µ(3λ + 2µ)
(1 + ν) ν
ε= σ − Tr (σ )1
E E

Universidad de Castilla- La Mancha Draft Por: Eduardo W. V. Chaves (2012)


Ciudad Real - España
7 ELASTICIDAD LINEAL 443

La traza de σ viene dada por:


Tr (σ ) = σ11 + σ 22 + σ 33
γ  γ  5
= (− γx 2 ) +  ( x1 − 3 x 2 ) +  ( x1 − 5 x 2 ) = γ ( x1 − 5 x 2 )
2  8  8
1
Pudiendo así obtener las componentes del tensor de deformaciones con ν = :
4
5 5
ε ij = σ ij − γ ( x1 − 5 x 2 )δ ij
4E 32 E
 
− γx 2 − γx1 0  1 0 0
5  γ  5
ε ij =  − γx1 ( x1 − 3 x 2 ) 0 − γ ( x1 − 5 x 2 ) 0 1 0
4E  2  32 E
γ 0 0 1
 0

0 [x1 − 5 x 2 ]

8
 1 
− 8 ( x1 − 3 x 2 ) − x1 0
5γ  1 
=  − x1 − (−3 x1 + 7 x 2 ) 0
4E  8 
 0 0 0
 

Ejemplo 7.28
Un cubo metálico que tiene longitud de arista a = 0,20m se sumerge en el mar a una
profundidad z = 400m .
Conociendo el módulo de elasticidad longitudinal del metal E = 21× 1010 Pa , el coeficiente
de Poisson ν = 0,3 , calcular la variación de volumen que experimenta el cubo sumergido.
Considerar la aceleración de la gravedad igual a g = 10m / s 2 .
OBS.: Aunque la densidad de masa varía con la temperatura, salinidad, y presión
(profundidad) considerar la densidad de masa del agua del mar igual a ρ = 1027 kg / m 3 .

Universidad de Castilla- La Mancha Draft Por: Eduardo W. V. Chaves (2012)


Ciudad Real - España
444 PROBLEMAS RESUELTOS DE MECÁNICA DEL MEDIO CONTINUO

Solución:
Debido a la profundidad y a las dimensiones del cubo podemos tomar como una buena
aproximación que todo el cubo está sometido a una misma presión.

h = 400m p

p
p

Figura 7.14.

La presión podemos obtener a partir de la primera ley de Newton F = ma = Vρg (peso de


la columna de agua) y dividiendo por el área:
F Vρg Ahρg kg m kg m
p= = = = ρgh = 1027 3 10 2 400m = 4,108 × 10 6 2 2 = 4,108 × 10 6 Pa
A A A m s m s
Luego, las componentes del tensor de tensiones en el cubo vienen dadas por:
− p 0 0  − 4,108 0 0 
  
σ ij =  0 − p 0  =  0 − 4,108 0  MPa
 0 0 − p   0 0 − 4,108
Como sólo tenemos componentes normales de tensión y el material es isótropo, sólo habrá
componentes normales de deformación e iguales:

εz = εy = εx =
1
[ (
σx − ν σy + σz = )] 1
10
[− 4,108 − 0,3 (− 4,108 − 4,108 )] × 10 6
E 21 × 10
Resultando
ε z = ε y = ε x = −7,82 × 10 −6

En pequeñas deformaciones la deformación volumétrica lineal es igual a la traza del tensor


de deformaciones infinitesimal:
∆V
= DVL ≡ ε V = Tr (ε ) ⇒ ∆V = V0 Tr (ε ) = 0,2 3 × ( −2,346 × 10 −5 ) = −1,8768 × 10 − 7 m 3
V0

donde hemos considerado que Tr (ε ) = −2,346 × 10 −5 .

Universidad de Castilla- La Mancha Draft Por: Eduardo W. V. Chaves (2012)


Ciudad Real - España
7 ELASTICIDAD LINEAL 445

Ejemplo 7.29
Un cilindro macizo, de 0,05m de radio de la base y 0,25m de altura, está constituido por
un material elástico lineal, de módulo de elasticidad longitudinal E = 3 × 10 4 MPa y
coeficiente de Poisson ν = 0,2 . Dicho cilindro se sitúa entre los pistones de una prensa, que
se pueden considerar infinitamente rígidos, y todo ello se encierra en recipiente hermético,
como se indica en la Figura 7.15.
Se llena el recipiente con aceite, y mediante el mecanismo adecuado, se eleva la presión en
el fluido hasta 15MPa . Haciendo funcionar la prensa, se aplica una fuerza axil total de
F = 2,35619 × 10 5 N sobre las bases del cilindro. Esta fuerza axil es el resultado debido a la
acción de la prensa más el producido por la presión del aceite sobre los pistones, y se puede
considerar uniformemente repartido sobre las bases.
Se pide determinar, en un punto genérico del cuerpo:
a) Las componentes del tensor de tensiones;
b) Las componentes del tensor de deformaciones;
c) Las componentes del campo de desplazamientos ( u , v , w ).

0,25m
x x
A A′

Corte AA′
F

0,1m

Figura 7.15: Ensayo de compresión triaxial.

Solución:
a) Tensor de tensiones
F 2,35619 × 10 5
σz = − =− = −30 MPa
A π(0,05) 2
σ x = σ y = −15MPa

Las componentes del tensor de tensiones son:

Universidad de Castilla- La Mancha Draft Por: Eduardo W. V. Chaves (2012)


Ciudad Real - España
446 PROBLEMAS RESUELTOS DE MECÁNICA DEL MEDIO CONTINUO

 − 15 0 0 

σ ij =  0 − 15 0  MPa
 0 0 − 30
b) Para un material elástico, lineal, homogéneo e isótropo, tensiones normales sólo
producen deformaciones normales, luego:
 1
[ (
ε x = E σ x − ν σ y + σ z )]

 1
[
ε y = σ y − ν (σ x + σ z )
E
]

 1
[ (
ε z = E σ z − ν σ x + σ y )]

Sustituyendo los valores de las variables obtenemos las siguientes componentes para el
Tensor de Deformaciones
− 2 0 0
ε ij =  0 − 2 0  × 10 − 4

 0 0 − 8

c) Campo de desplazamientos
Como estamos en pequeñas deformaciones se cumplen las siguientes relaciones:
∂u ∂v ∂w
εx = ; εy = ; εz =
∂x ∂y ∂z
Integrando y obteniendo los valores de las constantes de integración obtenemos finalmente
el campo de desplazamientos:
u = −2 × 10 −4 x ; v = −2 × 10 −4 y ; w = −8 × 10 −4 z

Ejemplo 7.30
Un hexaedro regular, de 0,1m de lado, está constituido por un material cuyas propiedades
mecánica viene representadas por las constantes de Lamé: λ = 8333,33MPa ,
µ = 12500 MPa .
Mediante una máquina de ensayos adecuada se le impone la deformación representada en la
Figura 7.16, en la cual todas las caras continúan siendo planas, las caras AEFB y DHGC
pasan a ser rombos y las restantes continúan siendo cuadradas. En este estado se pide
calcular:
a) El campo de desplazamientos;
b) El campo de deformaciones;
c) El campo de tensiones;
d) El tensor de tensiones en el centro del hexaedro;
e) Las deformaciones principales en el centro del hexaedro;
f) Las tensiones principales en el centro del hexaedro;
g) Las acciones ejercida por la máquina de ensayo sobre las caras ABFE y BCGF .

Universidad de Castilla- La Mancha Draft Por: Eduardo W. V. Chaves (2012)


Ciudad Real - España
7 ELASTICIDAD LINEAL 447

H H′ G G′

tg(α ) ≈ α = 0,001
E
E′ F F′

α C = C′
D = D′ y

A = A′ B = B′

x
Figura 7.16: Hexaedro deformado.
Solución:
a) Según Figura 7.16 podemos verificar que sólo habrá componentes tangenciales de
deformación. Además verificamos también que no hubo desplazamientos según las
direcciones x y z , luego u = 0 , w = 0 . A través de una analogía de triángulos podemos
sacar el desplazamiento v :
v
tg(α ) ≈ α = 0,001 = ⇒ v( z ) = 0,001z
z
Campo de desplazamiento:
u = 0

v( z ) = 0,001z
w = 0

b) Teniendo en cuenta las componentes del tensor de deformaciones:
 ∂u 1  ∂v ∂u  1  ∂ω ∂u  
 1 1   +   + 
 εx γ xy γ xz   ∂x 2  ∂x ∂y  2  ∂x ∂z  
2 2 
1 1   1  ∂v ∂u  ∂v 1  ∂ω ∂v  
ε ij =  γ xy εy γ yz  =   +   + 
2 2   2  ∂x ∂y  ∂y 2  ∂y ∂z  
1 γ 1
γ yz ε z   1  ∂ω ∂u  1  ∂ω ∂v  ∂ω 
 2 xz   2  ∂x + ∂z   +  
2  ∂y ∂z 
2
   ∂z 
Concluimos que ε x = ε y = ε z = γ xy = γ xz = 0 y la componente γ yz viene dada por:
∂v ∂w
γ yz = + = 0,001
∂z ∂y

Universidad de Castilla- La Mancha Draft Por: Eduardo W. V. Chaves (2012)


Ciudad Real - España
448 PROBLEMAS RESUELTOS DE MECÁNICA DEL MEDIO CONTINUO

 1 1 
 εx 2
γ xy
2
γ xz 
1 1  0 0 0 
ε ij =  γ xy εy 
γ yz  = 0 0 0,0005
2 2 
1 γ 1 0 0,0005 0 
 2 xz γ yz ε z 
2 

E E′

v( z )
z

c) Campo de tensiones σ = λTr (ε )1 + 2µε


Considerando Tr (ε ) = 0 , λ = 8333,33MPa , µ = 12500MPa , resulta:
0 0 0 

σ ij = 2 × (12500) 0 0 0,0005 MPa
0 0,0005 0 
0 0 0 
= 0 0 12,5 MPa

0 12,5 0 

e, f) Deformaciones principales:
−ε 0,0005 ε 2 = +0,0005
=0 ⇒ ε 2 = 0,0005 2 ⇒ ε = ±0,0005 ⇒ 
0,0005 −ε ε 3 = −0,0005
Recordemos que en pequeñas deformaciones las direcciones principales de tensiones
coinciden con las direcciones principales de deformaciones, luego podemos aplicar la
expresión σ = λTr (ε )1 + 2µε en el espacio principal de deformación:
0 0 0 

σ ′ij = 2 × (12500) 0 0,0005 0  MPa

0 0 − 0,0005
0 0 0 

= 0 12,5 0  MPa
0 0 − 12,5

Universidad de Castilla- La Mancha Draft Por: Eduardo W. V. Chaves (2012)


Ciudad Real - España
7 ELASTICIDAD LINEAL 449

g) Para obtener la fuerza total en una cara, multiplicamos la fuerza de superficie por el área
de la respectiva cara. La fuerza de superficie se obtiene a través de la expresión del vector
tensión t (n) = σ ⋅ nˆ . Para la cara ABFE la normal viene dada por nˆ i = [1,0,0] , luego:
ˆ

 t 1 ( ABFE )  0 0 0  1 0


 ( ABFE )      
t 2  = 0 0 12,5 0 = 0
 t ( ABFE )  0 12,5 0  0 0
 3      

Para la cara BCGF la normal viene dada por nˆ i = [0,1,0] , luego


 t 1 ( BCGF )  0 0 0  0   0 
 ( BCGF )      
t 2  = 0 0 12,5 1  =  0  MPa
 t ( BCGF )  0 12,5 0  0 12,5
 3      
Si hacemos el mismo procedimiento para las demás caras verificamos que la representación
de las fuerzas de superficies viene indicada tal y como se muestra en la figura abajo:

z
H H′ G
G′

E E′ F F′

α C = C′
D = D′ y

A = A′ B = B′

Ejemplo 7.31

Sobre el prisma recto de la Figura 7.17 actúan las fuerzas F1 = 10 N y F2 = 2 N sobre las
10
caras indicadas. Las longitudes de las aristas del prisma son: AB = 4cm , AD = cm ,
3
AA′ = 2cm . Sabiendo que el material que constituye el prisma tiene como propiedades
N
mecánicas: Módulo de Young E = 2,5 × 10 6 2 y coeficiente de Poisson ν = 0,25 . Y
cm
1
propiedades térmicas α = 5 × 10 −8 .
ºC

Universidad de Castilla- La Mancha Draft Por: Eduardo W. V. Chaves (2012)


Ciudad Real - España
450 PROBLEMAS RESUELTOS DE MECÁNICA DEL MEDIO CONTINUO

Se pide:
a) Obtener las tensiones principales; b) Obtener las componentes del vector tensión en el
plano Π . ¿Es en el plano Π donde actúa la máxima tensión tangencial? Justificar la
respuesta. c) Obtener el valor de las fuerzas F1 y F2 que se deben aplicar para que no haya
desplazamiento, según las direcciones x1 y x2 , cuando el prisma esté sometido a una
variación de temperatura de ∆T = 20º C .

x2 A2

A1
F2
B

Π
D
A
F1
F1
x1

60º
A′
F2

x3
Figura 7.17.
a) Campo de tensiones
 F1 
A 0 0
 1  1,25 0 0
10 F2 N
A1 = 8,0 , A2 = 4 × ⇒ σ ij =  0 − 0 =  0 − 0,15 0
 
3  A2  cm 2
0   0 0 0 
0 0
 
 
Que son las propias tensiones principales.

Universidad de Castilla- La Mancha Draft Por: Eduardo W. V. Chaves (2012)


Ciudad Real - España
7 ELASTICIDAD LINEAL 451

b)
x2
r ˆ
t (n)

Π n̂
D
A

x1

60º
A′

x3

 3 1 
La normal (vector unitario) tiene componentes: nˆ i =  ; ; 0 . Luego, el vector
 2 2 
r ˆ
tensión t (n) viene dado por:
r ˆ
t (n) = σ ⋅ nˆ
ˆ
t i(n) = σ ij nˆ j
 3
 
1,25 0 0  2   1,0825 
1  
=  0 − 0,15 0  = − 0,075
ˆ
t i(n)
 2  
 0 0 0  0   0 
 
 
La componente normal:
r ˆ
σ N = t (n) ⋅ nˆ = t i(n) nˆ i
ˆ

 3
 
 2 
1 
σ N = [1,0825 − 0,075 0] = 0,9
 2 
 0 
 
 
La componente tangencial:
r ˆ 2 r ˆ 2
t (n) = σ 2N + σ 2S ⇒ σS = t (n) − σ 2N

donde

Universidad de Castilla- La Mancha Draft Por: Eduardo W. V. Chaves (2012)


Ciudad Real - España
452 PROBLEMAS RESUELTOS DE MECÁNICA DEL MEDIO CONTINUO

 1,0825 
r ˆ r (nˆ ) r (nˆ )
= t ⋅ t = t i t i = [1,0825 − 0,075 0]− 0,075 = 1,1775
2
(nˆ ) (nˆ )
t (n)
 0 

Luego:
r ˆ 2
σS = t (n) − σ 2N = 1,1775 − 0,9 2 = 0,60621778

Si dibujamos el círculo de Mohr de tensiones


τ

σ III = −0,15 0 σ I = 1,25 σ N ( N / cm 2 )

Verificamos que en cualquier punto del sólido la tensión tangencial máxima está en un
 2 2 
plano definido por la normal nˆ i =  ; ; 0 y tiene como valor máximo
 2 2 
σ I − σ III
τ max = = 0,7 > σ S
2
c) Consideremos el campo de deformaciones:
1+ ν ν
ε= σ − Tr (σ )1 + α∆T 1
E E
1+ ν ν
ε ij = σ ij − Tr (σ )δ ij + α∆Tδ ij
E E
Para el caso en particular Tr (σ ) = σ11 + σ 22 :
0 0 0  σ11 0 0 1 0 0
0 0 0  = 1 + ν  0 σ 22   ν 
0 + α∆T − Tr (σ ) 0 1 0
  E   E 
0 0 ε 33   0 0 0 0 0 1

Luego, montamos el siguiente sistema:


 1+ ν  ν  1+ ν  ν 
ε11 = 0 = E σ11 + α∆T − E Tr (σ ) = E σ11 + α∆T − E (σ11 + σ 22 )
    

ε = 0 = 1 + ν σ + α∆T − ν Tr (σ ) = 1 + ν σ + α∆T − ν (σ + σ )
 22 E
22  E  E
22  E
11 22 
   
Resolviendo el sistema anterior, obtenemos que:

Universidad de Castilla- La Mancha Draft Por: Eduardo W. V. Chaves (2012)


Ciudad Real - España
7 ELASTICIDAD LINEAL 453

Eα ∆T N
σ11 = σ 22 = − = −3,33333 2
(1 − ν) cm
Luego, las fuerzas vienen dadas por:
 F1 = σ11 A1 = −26,66666 N

 F2 = σ 22 A2 = −44,44444 N

Universidad de Castilla- La Mancha Draft Por: Eduardo W. V. Chaves (2012)


Ciudad Real - España
454 PROBLEMAS RESUELTOS DE MECÁNICA DEL MEDIO CONTINUO

7.2 Ejercicios Propuestos

Problema 7.1
¿En que casos podemos idealizar la estructura como problemas bidimensionales? Escribir
las simplificaciones de cada caso y dar ejemplos.

Problema 7.2
Sea Q e (N) el tensor acústico elástico definido como:
Q e (N) = N ⋅ C ⋅ N componente
 s → Q e jl = N i C ijkl N k

donde C es el tensor elástico para un material elástico lineal e isótropo dado por:
C = µ1 + (λ + µ)N ⊗ N cuyas componentes son: C ijkl = λδ ij δ kl + µ(δ ik δ jl + δ il δ jk ) .

Probar que:
Q e jl = µδ jl + (λ + µ)N j Nl

Problema 7.3
Hacer el planteamiento del problema elástico lineal, es decir, definir las ecuaciones que
gobiernan el problema elástico.

Problema 7.4
5) Definir el Estado de Tensión Plano y el Estado de Deformación Plano. Para cada
estado, ¿qué simplificaciones son consideradas? Dar ejemplos de estructuras que se pueden
aproximar por cada estado.

Problema 7.5
6) ¿Cuáles son las hipótesis para el planteamiento de la teoría de la elasticidad lineal?

Problema 7.6
Definir el comportamiento elástico lineal, elástico no-lineal y el inelástico. Para cada
régimen hacer una gráfica representativa tensión-deformación.

Problema 7.7
Definir material Anisótropo e isótropo. Definir los tipos de simetría que puede presentar el
material.

Universidad de Castilla- La Mancha Draft Por: Eduardo W. V. Chaves (2012)


Ciudad Real - España
7 ELASTICIDAD LINEAL 455

Problema 7.8
Consideremos una barra de sección circular donde aplicamos una fuerza en las
extremidades de misma dirección y sentido contrario, ver Figura 7.18. La barra está
constituida por un material homogéneo, elástico-lineal e isótropo.

x3
x2
A

F F

x1

Figura 7.18: Sistema de fuerzas I

a) Dado el sistema el sistema de fuerzas I representado por la Figura 7.18. Justificar y


explicar porque este sistema es equivalente al sistema de fuerzas II dado por la Figura 7.19.

x3
x2

F F
σ= σ=
A A
x1

Figura 7.19: Sistema de fuerzas II

b) Obtener las componentes del tensor de tensiones de Cauchy para el sistema de fuerzas
II.
c) Verificar si el sistema II verifica las ecuaciones de equilibrio;
d) Demostrara que en la superficie de la barra está libre de fuerzas de superficies.
e) Obtener las componentes del tensor de deformación infinitesimal y del tensor spin
infinitesimal;
f) Obtener la máxima tensión normal y la máxima tensión tangencial.

Universidad de Castilla- La Mancha Draft Por: Eduardo W. V. Chaves (2012)


Ciudad Real - España
456 PROBLEMAS RESUELTOS DE MECÁNICA DEL MEDIO CONTINUO

Problema 7.9
Dar detalles de la reducción de la matriz elástica ortótropa a la matriz isótropa

 C11 C12 C13 0 0 0 


C C22 C23 0 0 0 
 12
C C23 C33 0 0 0  Matriz Ortótropa
[C ] =  13 
0 0 0 C44 0 0 
0 0 0 0 C55 0 
 
 0 0 0 0 0 C66 

 C11 C12 C12 0 0 0 


C C11 C12 0 0 0 
 12 
C C12 C11 0 0 0  Matriz Isótropa
[C ] =  12 
0 0 0 1
2
(C11 − C12 ) 0 0 
0 0 0 0 1
(C11 − C12 ) 0 
 2

 0 0 0 0 0 1
2
( C11 − C12 ) 

Universidad de Castilla- La Mancha Draft Por: Eduardo W. V. Chaves (2012)


Ciudad Real - España
11 Fluidos
11.1 Ejercicios Resueltos

Ejemplo 11.1
Justificar si son ciertas o falsas las siguientes afirmaciones:
a) Si el campo de velocidad es estacionario, el campo de aceleraciones también lo es;
b) Si el campo de velocidad es uniforme, el campo de aceleraciones es siempre nulo;
c) Si el campo de velocidades es estacionario y el medio es incompresible el campo de
aceleraciones es siempre nulo.

Solución:
a) Velocidad estacionaria
r r
∂v ( x , t) r
=0
∂t
El campo de aceleraciones queda:
r
∂vi ( x , t )
a i = v&i = + v k v i ,k = v k vi , k
142 ∂t 43
=0i
r r
r r& ∂v ( x , t ) r r r r r r r r
a=v = + ∇ xr v ( x , t ) ⋅ v ( x , t ) = ∇ xr v ( x ) ⋅ v ( x )
∂t 144244 3
No depende del tiempo

Luego la suposición (a) es VERDADERA.


r r r
b) Campo de velocidad uniforme implica que v ( x , t ) = v (t ) , luego:
r r r r
r r& ∂v ( x , t ) r r r r ∂v ( x , t )
a=v = + ∇ x v ( x , t ) ⋅ v ( x, t ) =
r
∂t 1424 3 ∂t
=0

Luego, la suposición (b) es FALSA.


r r r
c) Campo de velocidad estacionario implica que v = v ( x ) , no es función del tiempo, y un
r r
medio incompresible viene caracterizado por ∇ xr ⋅ v ( x , t ) = 0 , con lo cual se concluye que:
r r
r r& ∂v ( x ) r r r r r r r r
a=v = + ∇ xr v ( x ) ⋅ v ( x ) = ∇ xr v ( x ) ⋅ v ( x )
∂t
Luego, la suposición (c) es FALSA.
458 PROBLEMAS RESUELTOS DE MECÁNICA DEL MEDIO CONTINUO

Ejemplo 11.2
Considerando

ρv&i = ρb i − p ,i + (λ* + µ * )v j , ji + µ * vi , jj Ecuación del movimiento


r r r r (11.1)
ρv& = ρb − ∇ xr p + (λ* + µ * )∇ xr (∇ xr ⋅ v ) + µ *∇ xr 2 v Navier-Stokes-Duhem

Demostrar la ecuación de vorticidad:


r
∂ω
∂t
r r
[
+ 2∇ xr ⋅ ( ω ⊗ v ) anti −
µ* r 2 r r
]
∇ ω = 0 Ecuación de vorticidad
ρ x
(11.2)

r r r r r
donde ω es el vector vorticidad y viene dado por ω ≡ rot (v ) ≡ ∇ xr ∧ v . ( )
Solución:
Teniendo en cuenta la derivada material de la velocidad Euleriana:
r r
Dvi ∂vi ∂v i ∂v r r& ∂v ( x , t ) r r r r
v&i ≡ = + v j = i + vi , j v j a=v = + ∇ xr v ( x , t ) ⋅ v ( x , t ) (11.3)
Dt ∂t ∂x j ∂t ∂t

Las componentes resultantes de la operación vi , j v j son las componentes de la siguiente


r r
operación (∇ xr v ) ⋅ v , ver capítulo 1, y también fue demostrado que:
r r r r r 1 r r
(∇ xr v ) ⋅ v = (∇ xr ∧ v ) ∧ v + ∇ xr (v ⋅ v )
2
r r r 1
= (∇ xr ∧ v ) ∧ v + ∇ xr (v 2 ) (11.4)
2
r r 1
= ω ∧ v + ∇ xr (v 2 )
2
Luego, la aceleración Euleriana también puede ser representada por:
r r
r r r& ∂v ( x , t ) r r 1 r 2
a ( x, t ) = v = + ω ∧ v + ∇ x (v ) (11.5)
∂t 2
cuya expresión ya ha sido demostrado en el Ejemplo 2.21 (capítulo 2).
Teniendo en cuenta (11.3) y (11.4), la expresión (11.1) queda:
ρv&i = ρb i − p ,i + (λ* + µ * )v j , ji + µ * vi , jj
r r r r
ρv& = ρb − ∇ xr p + (λ* + µ * )∇ xr (∇ xr ⋅ v ) + µ *∇ xr 2 v
r r
 ∂v r r 1 r 2  r 2r
⇒ ρ + ω ∧ v + ∇ x (v )  = ρb − ∇ xr p + (λ* + µ * )∇ xr (∇ xr ⋅ v ) + µ * ∇ xr v (11.6)
 ∂t 2 
r r
∂v r r 1 1 r (λ* + µ * ) r r r µ * r 2 r r
⇒ r 2
+ ω ∧ v + ∇ x (v ) − b + ∇ x p − ∇ x (∇ x ⋅ v ) − ∇ v =0
∂t 2 ρ ρ ρ x
A continuación tomamos el rotacional de la expresión anterior:
r
r  ∂v r r 1 r 1 (λ* + µ * ) r r r µ * r 2 r  r
∇ x ∧  + ω ∧ v + ∇ xr (v 2 ) − b + ∇ xr p −
r ∇ x (∇ x ⋅ v ) − ∇ v = 0 (11.7)
 ∂t 2 ρ ρ ρ x 

Universidad de Castilla- La Mancha Draft Por: Eduardo W. V. Chaves (2012)


Ciudad Real - España
11 FLUIDOS 459

Recordar del capítulo 1 Vol.1 que se cumple que:


[ ]
r r r r r r r
ƒ ∇ xr ∧ ∇ xr (v 2 ) = 0 , ∇ xr ∧ [∇ xr p ] = 0 , ∇ xr ∧ [∇ xr (∇ xr ⋅ v )] = 0 , donde hemos aplicado
la definición que el rotacional del gradiente de un escalar resulta el vector nulo;

ƒ
r
r
r
[ r r
] r r r r
[ r r
] r r r
∇ xr ∧ (∇ xr ∧ v ) ∧ v = (∇ xr ⋅ v )(∇ xr ∧ v ) + ∇ xr (∇ xr ∧ v ) ⋅ v − ∇ xr v ⋅ (∇ xr ∧ v )
;
⇒ ∇ xr ∧ [ω ∧ v ] = (∇ xr ⋅ v )ω + [∇ xr ω ]⋅ v − ∇ xr v ⋅ ω
r r r r r r r r

ƒ
r
[ 2r
]
r r r
[ r 2
r
] r
[ 2 r
∇ xr ∧ ∇ xr v = −∇ xr ∧ ∇ xr ∧ (∇ xr ∧ v ) = ∇ xr ∇ xr ∧ v = ∇ xr ω ; ]
r r
ƒ
r  ∂v  ∂ r r r ∂ω
∇x ∧  =
r ∇x ∧v = [ ; ]
 ∂t  ∂t ∂t
r
ƒ Considerando que el campo b es conservativo, y teniendo en cuenta que el
rotacional de cualquier campo vectorial conservativo es igual al vector nulo
r r r
∇ xr ∧ b = 0 .
Teniendo en cuenta todo lo anterior, la relación (11.7) resulta:
r r
∂ω r r r r r r µ* 2 r
+ (∇ xr ⋅ v )ω + (∇ xr ω) ⋅ v − ∇ xr v ⋅ ω − ∇ xr ω = 0 (11.8)
∂t ρ
Fijemos que se cumplen las siguientes relaciones:
(v i ω j ), i = vi , i ω j + vi ω j , i ⇒ v i , i ω j = (vi ω j ), i −vi ω j , i
(11.9)
(v i ω j ), j = v i , j ω j + vi ω j , j ⇒ vi , j ω j = (v i ω j ), j −v i ω j , j = (vi ω j ), j

que es lo mismo en notación tensorial que:


r r r r r r
(∇ xr ⋅ v )ω = ∇ xr ⋅ [ω ⊗ v ] − (∇ xr ω) ⋅ v
(11.10)
(∇ xr v ) ⋅ ω = ∇ xr ⋅ [v ⊗ ω ] − (∇ xr ⋅ ω)v = ∇ xr ⋅ [v ⊗ ω]
r r r r r r r r

donde hemos aplicado la definición que la divergencia del rotacional de un vector es nulo,
r r r
es decir, ∇ xr ⋅ ω = ∇ xr ⋅ (∇ xr ∧ v ) = 0 . Teniendo en cuenta (11.10), la expresión (11.8) queda:
r
∂ω r r r r r r µ* r 2 r r
+ (∇ xr ⋅ v )ω + (∇ xr ω) ⋅ v − ∇ xr v ⋅ ω − ∇ ω=0
∂t ρ x
r
∂ω µ* r 2 r r
+ ∇ xr ⋅ [ω ⊗ v ] − (∇ xr ω) ⋅ v + (∇ xr ω) ⋅ v − ∇ xr ⋅ [v ⊗ ω ] −
r r r r r r r r
⇒ ∇ ω=0
∂t ρ x
r
∂ω r r r r µ* r 2 r r
⇒ + ∇ xr ⋅ [ω ⊗ v ] − ∇ xr ⋅ [v ⊗ ω ] − ∇ ω=0 (11.11)
∂t ρ x
r
∂ω r r r r µ* r 2 r r
⇒ + ∇ xr ⋅ [ω ⊗ v − v ⊗ ω ] − ∇ ω=0
∂t ρ x
r

∂ω
∂t
[
r r
+ 2∇ xr ⋅ (ω ⊗ v ) anti − ]
µ* r 2 r r
ρ x
∇ ω=0

Demostrando así la ecuación de vorticidad (11.2).

Universidad de Castilla- La Mancha Draft Por: Eduardo W. V. Chaves (2012)


Ciudad Real - España
460 PROBLEMAS RESUELTOS DE MECÁNICA DEL MEDIO CONTINUO

Ejemplo 11.3
r
Hallar la fuerza de tracción E que actúa en la superficie cerrada S que contiene al
volumen V de un fluido Newtoniano donde la viscosidad volumétrica es nula.

r
n̂ t (nˆ )

Solución:
dE i = t i(nˆ ) dS
La fuerza de tracción total viene dada por la siguiente integral:

∫ ∫ ∫
ˆ
E i = t i( n ) dS = σ ij nˆ j dS = σ ij , j dV (11.12)
S S V

donde se ha empleado la relación σ ij nˆ j = t i( nˆ ) .


2
Si la viscosidad volumétrica es nula, entonces κ * = 0 ⇒ λ* = − µ * (condición de Stokes).
3
Teniendo en cuenta la ecuación constitutiva de un fluido Newtoniano y la condición de
Stokes obtenemos que:
σ ij = − pδ ij + λ* δ ij D kk + 2µ *D ij
2 *
= − pδ ij − µ δ ij D kk + 2µ *D ij
3
 D 
= − pδ ij + 2µ *  D ij − kk δ ij 
14423443
Dijdev

σ ij = − pδ ij + 2µ *D ijdev

Reemplazando la ecuación constitutiva anterior en la expresión (11.12) obtenemos que:


E i = ( − pδ ij + 2µ *D ijdev ) nˆ j dS
S

Aplicando el Teorema de Gauss:

Universidad de Castilla- La Mancha Draft Por: Eduardo W. V. Chaves (2012)


Ciudad Real - España
11 FLUIDOS 461

∫ (− pδ + 2µ D ) dV
* dev
Ei = ij ij ,j
V

= ∫ (− p δ + 2µ D ) dV
,j ij
* dev
ij , j
V


= ( − p ,i + 2µ *D ijdev, j ) dV
V

donde hemos tenido en cuenta que µ *,j = 0 j , es decir µ * es un campo escalar homogéneo
(material homogéneo). La expresión anterior en notación tensorial queda:

∫ [− ∇ ]
r
E= r
x p + 2µ * ∇ xr ⋅ (D dev ) dV (11.13)
V

Ejemplo 11.4
Considérese un fluido en reposo de densidad de masa ρ f . Demostrar el Principio de
Arquímedes: “Todo cuerpo sumergido en un fluido en reposo experimenta un empuje hacia arriba igual
al peso del volumen del fluido desalojado”.
Si el cuerpo tiene densidad de masa ρ s y el campo de fuerzas másicas viene dado por
b i = − gδ i 3 . Obtener la fuerza resultante que actúa en el cuerpo y la aceleración.
Solución:
r
[
En el Ejemplo 11.14 hemos demostrado que E = ∫ − ∇ xr p + 2µ *∇ xr ⋅ (D dev ) dV . Si el fluido ]
está en reposo tenemos que D dev = 0 y la presión termodinámica es igual a la presión
hidrostática, p = p 0 . Luego, quedamos con:

r
E= ∫ [− ∇
V
r
x p 0 ]dV (11.14)

r
E

V -volumen
p0 n̂

x3 r
Ws
x2

x1

Universidad de Castilla- La Mancha Draft Por: Eduardo W. V. Chaves (2012)


Ciudad Real - España
462 PROBLEMAS RESUELTOS DE MECÁNICA DEL MEDIO CONTINUO

El peso del volumen del fluido desalojado viene dado por:


r r

= ρ f bdV
f
W (11.15)
V

Aplicando la ecuación de equilibrio:


r r
∇ xr ⋅ σ + ρ f b = 0 σ ij , j + ρ f b i = 0 i
r
⇒ ∇ xr ⋅ σ = −ρ f b ⇒ σ ij , j = −ρ f b i
r (11.16)
⇒ ∇ xr ⋅ (− p 0 1) = −ρ f b ⇒ (− p 0 δ ij ), j = −ρ f b i
r
⇒ ∇ xr p 0 = ρ f b ⇒ p 0,i = ρ f b i

Teniendo en cuenta (11.14) y (11.15), concluimos que:


r r r
∫ ∫
= ρ f bdV = ∇ xr p 0 dV = − E
f
W (11.17)
V V

Demostrando así el principio de Arquímedes.


El peso del cuerpo, de densidad de masa ρ s , que está sumergido en el líquido viene dado
por:
r r
W s = ρ s bdV∫
V

La fuerza resultante que actúa en el cuerpo es:


r r r
R = E + Ws
r r

= − ρ f bdV + ρ s bdV
V

V

r

= (ρ s − ρ f )bdV
V

Cuyas componentes son:


 
 0 
 

V
∫ V

Ri = (ρ s − ρ f )b i dV = − g (ρ s − ρ f )δ i 3 dV = 

0 

 g (ρ f − ρ s )dV 
V 

Verificando así que: si el cuerpo tiene densidad de masa menor que la densidad de masa del
r r
líquido, por ejemplo si el cuerpo es un gas, el cuerpo asciende, i.e. ρ f > ρ s ⇒ R > 0 , y caso
r r
contrario en cuerpo desciende. Si tenemos en cuenta que R = m s a , donde m s es la masa
total del cuerpo sumergido, podemos obtener la aceleración del cuerpo como:
ρs g (ρ f − ρ s )
∫ g (ρ f − ρ s )dV ∫ g (ρ f − ρ s ) ∫ρ
s
dV dV
R3 ρs ρ s
g (ρ f − ρ s )
a3 = =V =V = V
=
ms ms ms ms ρs

Universidad de Castilla- La Mancha Draft Por: Eduardo W. V. Chaves (2012)


Ciudad Real - España
11 FLUIDOS 463

NOTA: Es interesante observar que si el medio ( f ) es tal que ρ f = 0 tenemos que


a 3 = − g , es decir, la aceleración es independiente de la masa. Comprobando así, como hizo
Galileo, que a través de experimento sencillo verificó que cuerpo en caída libre era
independiente de la masa. Por ejemplo, en la luna donde podemos considerar que la
densidad de masa del aire es igual a cero, dos cuerpos con distintas masas en caída libre, e.g.
una pluma y un martillo, tendrán la misma aceleración y alcanzarán la superficie de la luna
en el mismo tiempo.

Ejemplo 11.5
Probar que el tensor desviador de tensiones σ dev es igual τ dev , donde σ ij = − pδ ij + τ ij .
Solución
Si σ kk = −3 p + τ kk
σ kk
σ ijdev = σ ij − δ ij
3
(− 3 p + τ kk )
= − pδ ij + τ ij − δ ij
3
τ kk
= τ ij − δ ij
3
Luego
σ ijdev = τ ijdev

Ejemplo 11.6
Deducir la ecuación unidimensional de continuidad de masa para el flujo de un fluido
incompresible no viscoso a través de un tubo. Considere V el volumen comprendido entre
dos secciones transversales arbitrarias A y B .
B
n̂ B
r
vB

n̂ A
V
r
vA

Solución:
Para un medio incompresible, la densidad de masa es independiente del tiempo ρ& = 0 , y
r
además teniendo en consideración la ecuación de continuidad ρ& + ρv i ,i = ρ (∇ xr ⋅ v ) = 0 , se
puede decir que:
r
∇ xr ⋅ v = 0 v k ,k = 0 (11.18)

Universidad de Castilla- La Mancha Draft Por: Eduardo W. V. Chaves (2012)


Ciudad Real - España
464 PROBLEMAS RESUELTOS DE MECÁNICA DEL MEDIO CONTINUO

Tomando para un volumen V :


r
∫∇ r
x ⋅ v dV =0 ∫v k , k dV =0 (11.19)
V V

Aplicando el teorema de Gauss:


r
∫ v ⋅ n̂ dS = 0
S
∫v S
ˆ k dS
kn =0 (11.20)

Luego:
rA rB
∫v ⋅ nˆ A dS + ∫v ⋅ nˆ B dS = 0
SA SB

Velocidad uniforme y perpendicular a las secciones SA y SB


r r
v A = −v A nˆ A ; v B = v B nˆ B

Reemplazando la velocidad en la integral, resulta:


− v A nˆ A ⋅ nˆ A dS + v B
∫ ∫ nˆ
B
⋅ nˆ B dS = 0
SA SB

v AS A = vBS B

Ejemplo 11.7
El campo de velocidad de un gas en movimiento a través de una tubería, cuyo eje
prismático es x 2 , se define mediante sus componentes:

v1 = 0 ; v 2 = 0,02 x 2 + 0,05 ; v3 = 0

kg
Cuando el gas pasa por x 2 = 0 la densidad de masa ρ es igual a 1,5 . Hallar ρ para
m3
x 2 = 5m .
Solución:
r r r
El campo de velocidad es estacionario, v = v ( x ) . De la ecuación de continuidad de masa:
∂ρ r r
+ ∇ xr ⋅ (ρv ) = 0 ⇒ ∇ xr ⋅ (ρv ) = 0
∂t
r
Luego, podemos concluir que ρv es una constante:
(ρv 2 ) x2 = 0
= (ρv 2 ) x2 =5

v 2 ( x 2 = 0) = 0,02 × 0 + 0,05 = 0,05 y v 2 ( x 2 = 5) = 0,02 × 5 + 0,05 = 0,15 con eso:


(ρv 2 ) x2 = 0
= (ρv 2 ) x2 =5

kg
1,5 × 0,05 = ρ 0,15 ⇒ ρ ( x 2 = 5) = 0,5
m3
Solución alternativa:
r
∇ xr ⋅ (ρv ) = 0 indicial
 → (ρvi ) ,i = ρ ,i vi + ρvi ,i = 0

Universidad de Castilla- La Mancha Draft Por: Eduardo W. V. Chaves (2012)


Ciudad Real - España
11 FLUIDOS 465

∂ρ ∂v  ∂ρ ∂ρ ∂ρ   ∂v ∂v ∂v 
vi + ρ i =  v1 + v2 + v3  + ρ  1 + 2 + 3  = 0
∂x i ∂xi  ∂x1 ∂x 2 ∂x 3   ∂x1 ∂x 2 ∂x 3 
Resultando:
 ∂ρ 
 (0,02 x 2 + 0,05)  + ρ (0,02) = 0
 ∂x 2 
∂ρ 0,02
⇒ =− ∂x
ρ (0,02 x 2 + 0,05) 2
Integrando la expresión anterior, obtenemos que:
Lnρ = Ln(0,02 x 2 + 0,05) + LnC =
 C 
⇒ Lnρ = Ln 
 (0,02 x 2 + 0,05) 
C
⇒ρ=
(0,02 x 2 + 0,05)
Condiciones de contorno, x 2 = 0 ⇒ ρ = 1,5 , con lo cual obtenemos que C = 0,075 :
0,075 x2 = 5 0,075 kg
ρ=  → ρ= = 0,5 3
(0,02 x 2 + 0,05) (0,02 × 5 + 0,05) m

Ejemplo 11.8
Las componentes del tensor de tensión en un punto de un fluido Newtoniano, con una
viscosidad volumétrica nula, son:
 − 6 2 − 1
σ ij =  2 − 9 4  Pa
 − 1 4 − 3

Determinar las componentes del tensor de tensión viscoso.


Solución:
En el caso de viscosidad volumétrica nula (condición de Stokes) tenemos p = p = p 0 , y
además se puede obtener que:
σ ij = − pδ ij + τ ij
2 *
κ * = λ* + µ =0
3
σ ii = −3 p

p=−
σ ii
=−
(− 6 − 9 − 3) = 6
3 3
Luego:
τ ij = σ ij + pδ ij
 − 6 2 − 1  6 0 0   0 2 − 1
=  2 − 9 4  + 0 6 0  =  2 − 3 4  Pa
    
 − 1 4 − 3 0 0 6   − 1 4 3 

Universidad de Castilla- La Mancha Draft Por: Eduardo W. V. Chaves (2012)


Ciudad Real - España
466 PROBLEMAS RESUELTOS DE MECÁNICA DEL MEDIO CONTINUO

Ejemplo 11.9
σ kk
Determinar las condiciones bajo las cuales la presión normal media p = − = −σ m es
3
igual a la presión termodinámica p para un fluido Newtoniano.
Solución:
Fue deducido que:
1 σ kk
σ ijdev = 2µ *D ijdev ; σ kk = − p + k *D ii ; = − p = − p + κ *D kk
3 {3
−p

Así, para que la igualdad p = p se cumpla, hay que cumplir que:


D ii = 0 2 *
κ* = 0 ;  ; λ* = − µ
 Tr (D ) = 0 3

Ejemplo 11.10
σ kk
Determinar las condiciones bajo las cuales la presión normal media p = − es igual a la
3
presión termodinámica p , para un fluido Newtoniano.
Solución:
s ij = 2µ *D ′ij
σ kk = −3 p + 3µ *D ′kk
σ kk
κ* = 0 ⇒ p=− (11.21)
3
2 * 2
κ * = λ* + µ =0 ⇒ λ* = − µ *
3 3
D ′kk = 0

Ejemplo 11.11
Un fluido perfecto y barotrópico tiene una ecuación cinética de estado definida por
p
ρ = ρ0 + , donde k es constante. Obtener el campo de presión (distribución de presión)
k
en régimen cuasiestático (aceleración nula) bajo acción del campo gravitatorio
b i = [0 0 − g ] .
T

Solución:
Ecuación constitutiva de un fluido perfecto:
σ = − p1
Las ecuaciones del movimiento quedan:

Universidad de Castilla- La Mancha Draft Por: Eduardo W. V. Chaves (2012)


Ciudad Real - España
11 FLUIDOS 467

r r
∇ xr ⋅ σ + ρb = ρv& indicial
 → σ ij , j + ρb i = ρv&i = 0 i
{
= 0i
(− pδ ij ), j + ρb i = 0 i (11.22)
− p, j δ ij + ρb i = 0 i
r r
− ∇ xr p + ρb = 0 tensorial
← 
 − p , i + ρb i = 0 i

Considerando el vector de fuerzas másicas b i = [0 0 − g ] concluimos que:


 ∂p ∂p
(i = 1) ⇒ − + ρb 1 = 0 ⇒ = 0 ⇒ p = p ( x1 , x 2 , x3 )
 ∂x1 ∂x1
 ∂p ∂p
− p, i +ρb i = 0 i ⇒ (i = 2) ⇒ − + ρb 2 = 0 ⇒ = 0 ⇒ p = p ( x1 , x 2 , x3 ) (11.23)
 ∂x 2 ∂x 2
 ∂p dp ( x3 )
(i = 3) ⇒ − ∂x + ρb 3 = 0 ⇒ dx + ρg = 0
 3 3

Con lo cual hemos concluimos que el campo de presión es sólo función de la coordenada
x 3 , i.e. p = p( x3 ) .
El hecho de que sea un fluido barotrópico, eso implica que la densidad de masa es
únicamente función de la presión ρ = ρ ( p ) . Esta relación es precisamente la ecuación
cinética del enunciado:
p
ρ = ρ ( p) ⇒ ρ = ρ0 +
k
Luego:
dp( x3 )
+ ρg = 0
dx3
dp( x3 )  p
+ ρ 0 + g = 0 (11.24)
dx3  k
dp( x3 ) g
+ p = −ρ 0 g
dx3 k
La solución de esta ecuación diferencial es la suma de la solución homogénea y una
particular:
−g
dp( x 3 ) g ( x3 )
k
Solución homogénea: + p = 0 ⇒ p = C exp
dx3 k
Solución particular: p = −kρ 0
Luego:
−g
( x )
p = Cexp k 3 − kρ 0

Ejemplo 11.12
Un gas perfecto es un fluido ideal e incompresible en el cual en la ausencia de fuente de
calor, el movimiento es barotrópico, la presión es proporcional a ρ γ , donde γ es una
constante y γ > 1 . Demostrar que cuando r = 0 (fuente interna de calor), la energía interna
específica para un gas perfecto viene dada por:

Universidad de Castilla- La Mancha Draft Por: Eduardo W. V. Chaves (2012)


Ciudad Real - España
468 PROBLEMAS RESUELTOS DE MECÁNICA DEL MEDIO CONTINUO

1 p
u= + constante
( γ − 1) ρ
Solución:
Para el problema propuesto, la ecuación de energía se resume a:
r
ρ u& = σ : D − ∇ xr ⋅ q + ρr = σ : D
⇒ ρ u& − σ : D = 0
Para un gas perfecto la tensión es un tensor esférico y viene dado por:
σ ( p ) = − p1
donde p es la presión termodinámica. Luego, la ecuación de energía queda:
ρ u& − σ : D = 0
⇒ ρ u& + p1 : D = 0
⇒ ρ u& + pTr (D) = 0
r
⇒ ρ u& + p∇ xr ⋅ v = 0
Para un movimiento barotrópico la energía interna específica es una función de la densidad
de masa, u = u (ρ ) , luego:
r
ρ u& + p∇ xr ⋅ v = 0
∂u r
⇒ρ ρ& + p∇ xr ⋅ v = 0
∂ρ
Dρ r r
Teniendo en cuenta la ecuación de continuidad de masa + ρ∇ xr ⋅ v = 0 ⇒ ρ& = −ρ∇ xr ⋅ v ,
Dt
la ecuación de energía queda:
∂u r
ρ ρ& + p∇ xr ⋅ v = 0
∂ρ
∂u r r
⇒ −ρ ρ∇ xr ⋅ v + p∇ xr ⋅ v = 0
∂ρ
 ∂u  r
⇒  − ρ 2 + p ∇ xr ⋅ v = 0
 ∂ρ 12
 ≠0 3

con lo cual hay que cumplir que:


∂u ∂u p
− ρ2 + p=0 ⇒ = 2
∂ρ ∂ρ ρ

Como la presión es proporcional a ρ γ , podemos decir que p = p (ρ ) = kρ γ , donde k es


una constante de proporcionalidad, luego:
∂u ∂u kρ γ
− ρ2 + p=0 ⇒ = 2 = kρ ( γ − 2 )
∂ρ ∂ρ ρ
Integrando la expresión anterior, obtenemos que:
k ργ
u= + constante
( γ − 1) ρ
1 p
= + constante
( γ − 1) ρ

Universidad de Castilla- La Mancha Draft Por: Eduardo W. V. Chaves (2012)


Ciudad Real - España
11 FLUIDOS 469

Ejemplo 11.13
r
Un fluido se mueve alrededor de una esfera de radio R con velocidad v , siendo sus
componentes en coordenadas esféricas ( r , θ, φ ) dadas por:

 R 3 3R 
v r = c 3 − + 1 cos(θ)
 2r 2r 
 R 3 3R  (11.25)
v θ = c 3 + − 1 sin(θ)
 4 r 4 r 
vφ = 0

donde c es una constante positiva.


¿Es un movimiento isocórico? Probar la respuesta.
r
Nota: Dado un vector u , la divergencia de este vector en coordenadas esféricas es:
r r ∂u 1 ∂u θ 1 ∂u φ cot(θ) 2
div u ≡ ∇ xr ⋅ u = r + + + uθ + u r
∂r r ∂θ r sin(θ) ∂φ r r
Solución: Para demostrar que un movimiento es isocórico tenemos que probar que
r
∇ xr ⋅ v = 0
Obtenemos las siguientes derivadas:
∂v r ∂   R 3 3R    − 3R 3 3R 
= c 3 − + 1 cos(θ) = c + 2 + 1 cos(θ)
∂r ∂r   2r 2r    2r
4
2r 

∂v θ ∂   R 3 3R    R 3 3R 
= c 3 + − 1 sin(θ) = c 3 + − 1 cos(θ)
∂θ ∂θ   4r 4r    4r 4r 
Sacamos ahora la divergencia de la velocidad:
r ∂v 1 ∂v θ 1 ∂v φ cot(θ) 2
∇ xr ⋅ v = r + + + vθ + v r
∂r r ∂θ r sin(θ) {
∂φ r r
=0

 − 3R 3
3R  1  R 3 3R 
= c + + 1  cos(θ) + c
 + − 1 cos(θ) +
 2r
4
2r 2  r  4r 3 4r 
cos(θ) 1   R 3 3R   2   R 3 3R  
+ c 3 + − 1 sin(θ) + c 3 − + 1 cos(θ)
sin(θ) r   4r 4r   r   2r 2r  
Simplificando obtenemos que:
r
∇ xr ⋅ v = 0
Comprobando que es un movimiento isocórico.

Ejemplo 11.14
El fluido barotrópico del interior de la tubería de la figura tiene por ecuación cinética de
estado:
 ρ 
p = β Ln  ; (β y ρ 0 constantes)
 ρ0 

Universidad de Castilla- La Mancha Draft Por: Eduardo W. V. Chaves (2012)


Ciudad Real - España
470 PROBLEMAS RESUELTOS DE MECÁNICA DEL MEDIO CONTINUO

p - presión, ρ - densidad de masa.


Calcular en régimen estacionario la presión de salida p ( 2) en función de las demás variables
de la figura. Justificar la fórmula empleada.

p (1)
p( 2)
v (1)
v( 2)
S (1)
S (2)

p - presión, v - velocidad, S - área de la sección.

Solución:
Según el principio de conservación de masa:
D
Dt V ∫
ρdV = 0

Dada una propiedad Φ ( x , t ) se cumple que:


D r  DΦ ( x, t ) DdV 
Dt V ∫
Φ ( x , t )dV = 
V
∫ Dt
dV + Φ ( x , t ) 
Dt 
 DΦ ( x, t ) r r 
=  ∫ dV + Φ ( x , t )∇ xr ⋅ v ( x , t )dV 
V
Dt 
 DΦ ( x, t ) r r 
=  ∫ + Φ ( x , t )∇ xr ⋅ v ( x , t ) dV
V
Dt 
 ∂Φ ( x , t ) r r r r 
=  ∫ + ∇ xr Φ ( x , t ) ⋅ v ( x , t ) + Φ ( x , t )∇ xr ⋅ v ( x , t ) dV
V
∂t 
 ∂Φ ( x , t )
⋅ (Φ( x, t )v ( x, t )) dV
r r
= 
V 
∫ ∂ t
+ ∇ xr

∂Φ ( x , t ) r r
= ∫ ∫
dV + ∇ xr ⋅ [Φ( x, t )v ( x, t )]dV
V
∂t V

∂Φ ( x , t ) r r
= ∫
V
∂t
dV + ∫ [Φ( x, t )v ( x, t )]⋅ nˆ dS = 0
S

haciendo ρ = Φ ( x , t ) obtenemos que:


D r ∂ρ ( x , t ) r r
∫ρ ( x , t )dV = ∫ dV + [ρ ( x , t )v ( x , t )] ⋅ nˆ dS = 0

Dt V V
∂t S

∂ρ ( x , t )
Aplicando la condición de régimen estacionario = 0 , resultando así que:
∂t

Universidad de Castilla- La Mancha Draft Por: Eduardo W. V. Chaves (2012)


Ciudad Real - España
11 FLUIDOS 471

r r r r r r
∫ [ρ( x)v ( x)]⋅ nˆ dS = 0
S
⇒ ∫S [ρ( x )v ( x)]⋅ nˆ dS + S∫ [ρ( x )v ( x)]⋅ nˆ dS = 0
(1) (2)

⇒ ∫S − ρ (1) v (1) dS +
S
∫ ρ ( 2 ) v ( 2 ) dS =0
(1) (2)

⇒ −ρ (1) v (1) S (1) + ρ ( 2 ) v ( 2) S ( 2) = 0


Resultando que:

ρ (1) v (1) S (1) = ρ ( 2) v ( 2) S ( 2) (11.26)

Recordar que q = ρ v es el flujo de masa que tiene por unidad [q] =


r r r kg
.
m2s
Introduciendo la ecuación cinética de estado se obtiene la siguiente expresión:
 p  p
 ρ  p  ρ     ρ   
p = β Ln  ⇒ = Ln  ⇒ exp  β  =   ⇒ ρ ( x ) = ρ 0 exp  β 
 ρ0  β  ρ0   ρ0 
Luego:
ρ (1) v(1) S (1) = ρ ( 2) v( 2) S ( 2)
 p ( 1)   p( 2 ) 
   
 β   β 
ρ 0 exp  
v(1) S (1) = ρ 0 exp  
v( 2 ) S ( 2 )
 p ( 2 ) − p (1 ) 
  v(1) S (1)
 β 
⇒ exp  
=
v( 2 ) S ( 2 )
p( 2 ) − p(1)  v(1) S (1)  (11.27)
⇒ = Ln 
β  v( 2 ) S ( 2 ) 
 
 v(1) S (1) 
⇒ p( 2 ) − p(1) = β Ln 
 v( 2 ) S ( 2 ) 
 
 v(1) S (1) 
⇒ p( 2 ) = p(1) + β Ln 
 v( 2 ) S ( 2 ) 
 
NOTA: El caudal, muchas veces representado por Q , es el flujo total específico, i.e.:
r r r r
q ⋅ dS ρv ⋅ dS r r  m3 
Q= ∫ =∫ = ∫ v ⋅ dS Caudal   (11.28)
S
ρ S
ρ S  s 

 q r r
Verificamos las unidades [Q ] = 
⋅ dS kg m 3 2 m 3
∫ = 2
 S ρ  m s kg
m =
s
. En este ejemplo hemos

obtenido que ρ (1) v (1) S (1) = ρ ( 2) v ( 2) S ( 2) , que puede ser reescrito como:
ρ (1) v (1) S (1) = ρ ( 2) v ( 2) S ( 2) ⇒ ρ (1) Q(1) = ρ ( 2) Q( 2)
Para el caso particular de un medio incompresible, ρ (1) = ρ ( 2) , resulta que:
v (1) S (1) = v ( 2) S ( 2) ⇒ Q(1) = Q( 2 ) (ver Ejemplo 11.6)

Universidad de Castilla- La Mancha Draft Por: Eduardo W. V. Chaves (2012)


Ciudad Real - España
472 PROBLEMAS RESUELTOS DE MECÁNICA DEL MEDIO CONTINUO

Ejemplo 11.15
Partiendo de la ecuación de Navier-Stokes-Duhem, obtener la ecuación de Bernoulli:

p v2
gh + + = constante Ecuación de Bernoulli (11.29)
ρ 2

Hipótesis: Fluido no viscoso, incompresible. Campo de velocidad estacionario e irrotacional.


Solución:
r
Considerando el medio incompresible (∇ xr ⋅ v ) = 0 , y no viscoso (λ* = µ * = 0) , la ecuación
de Navier-Stokes-Duhem se resume a:
ρv&i = ρb i − p ,i + (λ* + µ * )v j , ji + µ * v i , jj
r r r r
ρv& = ρb − ∇ xr p + (λ* + µ * )∇ xr (∇ xr ⋅ v ) + µ *∇ xr 2 v (11.30)
r r
⇒ ρv& = ρb − ∇ xr p
Que es la ecuación de Euler del movimiento. La derivada material de la velocidad, ver
expresión (11.5), se resume a:
r
r& ∂v r r 1 r 2 1
v= + ω ∧ v + ∇ x (v ) = ∇ xr (v 2 )
∂t 2 2
r
∂v r
donde hemos considerado el campo de velocidad estacionario = 0 e irrotacional
∂t
r r r r r
∇ xr ∧ v = rot v = ω = 0 . Con eso la ecuación (11.30) puede ser reescrita como:
ρ r 1 r 2 r 1 r
∇ xr (v 2 ) = ρb − ∇ xr p ⇒ ∇ x (v ) − b + ∇ xr p = 0 (11.31)
2 2 ρ
Consideremos que la fuerza másica (campo conservativo) podemos representarla por
r
b = −∇ xr ϕ , donde ϕ es un potencial y además considerando que el campo de densidad de
 p 1
masa es homogéneo se cumple que ∇ xr   = ∇ xr p . Luego, (11.31) queda:
ρ ρ

 p v2  p v2
∇ xr  ϕ + +  = 0i
 ⇒ ϕ+ + = constante (11.32)
 ρ 2  ρ 2

Considerando que el potencial dado por ϕ = gh , donde g es la aceleración de la gravedad


y h la altura piezométrica, obtenemos la conocida ecuación de Bernoulli:
p v2
gh + + = constante
ρ 2

 v 2   p  N m 3 Nm J m 2
Verificando las unidades [gh] =  = = 2 = = = , que es la unidad de
 2   ρ  m kg kg kg s 2
energía específica, es decir, unidad de energía por unidad de masa.
Es interesante resaltar que la ecuación de Bernoulli es la aplicación de la conservación de la
energía y se supone que en el sistema no hay pérdida de energía:

Universidad de Castilla- La Mancha Draft Por: Eduardo W. V. Chaves (2012)


Ciudad Real - España
11 FLUIDOS 473

energía constante
energía en A energía en B

p
ρ A
p
ρ
v2 B

2 A
v2
gh A A 2 B

h B gh B

Ejemplo 11.16
Se considera un fluido perfecto e incompresible que circula por el canal de la figura en
régimen estacionario. Determinar el valor de H .
v (1) = 1m / s

h(1)
v( 2 ) = 2 m / s

H
h( 2)

Hipótesis: No se considera pérdida de energía.

Solución:
Ecuación de continuidad:
v (1) 1
v (1) h(1) = v ( 2 ) h( 2) ⇒ h( 2 ) = h(1) = h(1)
v( 2) 2
Ecuación de Bernoulli:
v (21) 
( H + h(1) ) + 0 + 
2g  v (22 ) − v (21) 1 3
 ⇒ H = h( 2 ) − h(1) + ⇒ H = − h(1) +
v (22)  2g 2 2g
h( 2 ) + 0 + 
2g 

Universidad de Castilla- La Mancha Draft Por: Eduardo W. V. Chaves (2012)


Ciudad Real - España
474 PROBLEMAS RESUELTOS DE MECÁNICA DEL MEDIO CONTINUO

Ejemplo 11.17
Un depósito circular de gran diámetro que está lleno de agua que vierte por un pequeño
orificio lateral situado a una altura H por debajo del nivel del agua del depósito. Si el
caudal vertido es Q , obtener el diámetro D del orificio.

Hipótesis: Considerar que H no


A varía con el tiempo (régimen
estacionario). Considerar que en la
H sección BB ′ la presión en el flujo
B es igual a la presión atmosférica:

( p − patm ) patm

C′ B′

C
B Presión Presión

Sección CC ′ Sección BB′

Solución:
El agua podemos considerar como un fluido perfecto e incompresible. Aplicando la
Ecuación de Bernoulli:
p v2
z+ + = ctte
ρg 2 g
donde se cumple que:
p atm 
Punto A ⇒ H+ +0 
ρg 
2 
⇒ v ( B ) = 2 gH
p v( B ) 
Punto B ⇒ 0 + atm +
ρg 2 g 
Teniendo en cuenta que el caudal viene dado por Q = v ( B ) S ( B ) , concluimos que:

πD 2 4Q
Q = v ( B ) S ( B ) = 2 gH ⇒ D=
4 π 2 gH

Universidad de Castilla- La Mancha Draft Por: Eduardo W. V. Chaves (2012)


Ciudad Real - España
11 FLUIDOS 475

Ejemplo 11.18
Considérese un tubería donde se ha introducido un tubo de pitot tal y como se indica en la
Figura 11.1. Obtener la velocidad en el punto 1 en función de h(1) y h( 2) . Considerar que
no hay pérdida en el sistema.

h( 2)

h(1)

v(1) v( 2) = 0
1  2 
h  p(1)  p( 2 )

Figura 11.1: Tubo de pitot


Solución:
Aplicando la ecuación de Bernoulli entre los puntos 1 y 2 , hay que cumplir que:
p (1) v (21) p( 2) v (22 )
gh + + = gh + +
ρ 2 ρ 2
p (1) v (21) p ( 2)
⇒ + =
ρ 2 ρ
2( p ( 2 ) − p (1) )
⇒ v (1) =
ρ
Considerando que las presiones en los puntos 1 y 2 vienen dadas respectivamente por:
p (1) = ρgh(1) ; p ( 2 ) = ρgh( 2 )

Con lo cual la velocidad v (1) viene definida por:

2( p ( 2) − p (1) ) 2(ρgh( 2 ) − ρgh(1) )


v (1) = = = 2 g (h( 2 ) − h(1) )
ρ ρ

Universidad de Castilla- La Mancha Draft Por: Eduardo W. V. Chaves (2012)


Ciudad Real - España
476 PROBLEMAS RESUELTOS DE MECÁNICA DEL MEDIO CONTINUO

Ejemplo 11.19
Considérese un fluido no viscoso e incompresible, y caracterizado por presentar el campo
de velocidad estacionario, irrotacional, e independiente de x3 . Expresar las ecuaciones de
gobierno del problema a través de un potencial de velocidad φ y de líneas de corrientes ψ .
Solución:
Potencial de velocidad: En este ejemplo podemos representar el campo de velocidad a través de
r
un potencial φ , i.e. v = ∇ xr φ . Con eso estamos considerando que el campo de velocidad es
r r r r r
conservativo, y como consecuencia el rotacional es igual a cero, i.e. ∇ xr ∧ v = rot v = ω = 0 .
Recordar que, un campo cuyo rotacional es cero no necesariamente implica que el campo
sea conservativo, pero todo campo conservativo el rotacional es igual a cero.
Observemos que la velocidad tiene la misma dirección que ∇ xr φ , y es normal a las
isosuperficies, i.e. normal a las superficies de φ = ctte .
Línea de corriente: Recordar que una línea de corriente ( ψ ) es la curva en el instante t donde
la tangente a esta curva (en cualquier punto) es paralela a la velocidad. Dos líneas de
corrientes no pueden cruzarse. Si el campo de velocidad es estacionario la línea de corriente
coincide con la trayectoria de la partícula.
Volumen de control
ψ (5) = ctte

r r ψ ( 4) = ctte
v ( x)
t Líneas de corriente

ψ ( 2) = ctte
∇ xr ψ
ψ (1) = ctte

r
x

Partiendo de la definición de diferencial total dψ y de gradiente ∇ xr ψ obtenemos la


r
relación dψ = ∇ xr ψ ⋅ dx .
r
v = ∇ xr φ
ψ + dψ
∇ xr ψ

ψ = ctte ⇒ dψ = 0
en la línea de corriente

Universidad de Castilla- La Mancha Draft Por: Eduardo W. V. Chaves (2012)


Ciudad Real - España
11 FLUIDOS 477

r
Observar que se cumple que ∇ xr ψ ⋅ ∇ xr φ = 0 . El diferencial total dx en la línea de corriente
en un punto tiene la misma dirección que la velocidad en este punto. Con eso se cumple
que:
r r r
dx ∧ v = 0
En componentes cartesianas:
eˆ 1 eˆ 2 eˆ 3
r r r
dx ∧ v = dx1 dx 2 dx3 = 0
v1 v2 v3
r
= (v3 dx 2 − v 2 dx3 )eˆ 1 + (v 3 dx1 − v1 dx3 )eˆ 2 + (v 2 dx1 − v1 dx 2 )eˆ 3 = 0
Componentes:
(v3 dx 2 − v 2 dx3 ) 0
(dx ∧ v ) i =  (v 3 dx1 − v1 dx3 )  = 0
r r

 (v 2 dx1 − v1 dx 2 )  0

Para este ejemplo el campo de velocidad es independiente de x3 , es decir, estamos en el


caso plano x1 − x 2 (2D). Con eso quedamos solo con:
 0  0 
r r 
( dx ∧ v ) i =  0  = 0 
  
(v 2 dx1 − v1 dx 2 ) 0

Con eso concluimos que:


v 2 dx1 − v1 dx 2 = 0 (11.33)
Teniendo en cuenta que en una línea de corriente se cumple que ψ = ctte ⇒ dψ = 0 y
r
además aplicando la definición dψ = ∇ xr ψ ⋅ dx , obtenemos que:
r
dψ = ∇ xr ψ ⋅ dx indicial
→ dψ = ψ ,i dxi = 0
⇒ dψ = ψ ,1 dx1 + ψ , 2 dx 2 + ψ ,3 dx3 = 0
∂ψ ∂ψ ∂ψ
⇒ dψ = dx1 + dx 2 + dx3 = 0
∂x1 ∂x 2 ∂x3
Para el caso 2D tenemos que:
∂ψ ∂ψ
dx1 + dx 2 = 0 (11.34)
∂x1 ∂x 2
Si comparamos las relaciones (11.33) y (11.34) concluimos que:
∂ψ ∂ψ
v1 = − ; v2 = (11.35)
∂x 2 ∂x1

r
1) Partiendo que el fluido es incompresible: (∇ xr ⋅ v ) = 0 obtenemos que:
∂v1 ∂v 2 ∂v3 2D ∂v1 ∂v 2
v i ,i =
+ + =0 → + =0
∂x1 ∂x 2 ∂x3 ∂x1 ∂x 2
r
Teniendo en cuenta que v = ∇ xr φ , concluimos que:

Universidad de Castilla- La Mancha Draft Por: Eduardo W. V. Chaves (2012)


Ciudad Real - España
478 PROBLEMAS RESUELTOS DE MECÁNICA DEL MEDIO CONTINUO

∂v1 ∂v 2 ∂ 2φ ∂ 2 φ
+ =0 ⇒ + =0 ⇒ ∇ 2xr φ = 0 (11.36)
∂x1 ∂x 2 ∂x12 ∂x 22
r r r r r
2) Partiendo del hecho que el fluido es irrotacional ∇ xr ∧ v = rot v = ω = 0 obtenemos que:

eˆ 1 eˆ 2 eˆ 3
r r r ∂ ∂ ∂ r
rot (v ) = ∇ xr ∧ v = =  ijk v k , j eˆ i = 0
∂x1 ∂x 2 ∂x3
v1 v2 v3 (11.37)
 ∂v ∂v   ∂v ∂v   ∂v ∂v  r
=  3 − 2 eˆ 1 +  1 − 3 eˆ 2 +  2 − 1 eˆ 3 = 0
 ∂x 2 ∂x3   ∂x3 ∂x1   ∂x1 ∂x 2 
Luego:
 ∂v 3 ∂v 2 
 − 
  ∂x 2 ∂x 3   0
  ∂v1 ∂v3    
  −   = 0
 
  ∂x3 ∂x1   0
 ∂v ∂v1   
 2
− 
 ∂x1 ∂x 2 
Como estamos en el caso 2D, la expresión anterior se resume a:
∂v 2 ∂v1
− =0
∂x1 ∂x 2
Teniendo en cuenta las relaciones (11.35) concluimos que:
∂v 2 ∂v1 ∂ 2ψ ∂ 2ψ
− =0 ⇒ + =0 ⇒ ∇ 2xr ψ = 0
∂x1 ∂x 2 ∂x1 ∂x 2
Con lo cual el problema queda planteado a través de las relaciones:

∇ 2xr φ = 0 ; ∇ 2xr ψ = 0 (11.38)

Universidad de Castilla- La Mancha Draft Por: Eduardo W. V. Chaves (2012)


Ciudad Real - España
11 FLUIDOS 479

11.2 Ejercicios Propuestos

Problema 11.1
Definir los siguientes conceptos:
a) Presión hidrostática.
b) Presión media.
c) Presión termodinámica.
Justificar en qué casos serán iguales la presión media y la presión termodinámica y cuándo
lo serán las tres.

Problema 11.2

2) Definir los siguientes conceptos:


a) Fluido de Stokes.
b) Fluido Newtoniano.
c) Fluido perfecto.
d) Fluido incompresible.

Problema 11.3:

Universidad de Castilla- La Mancha Draft Por: Eduardo W. V. Chaves (2012)


Ciudad Real - España
480 PROBLEMAS RESUELTOS DE MECÁNICA DEL MEDIO CONTINUO

Universidad de Castilla- La Mancha Draft Por: Eduardo W. V. Chaves (2012)


Ciudad Real - España
´
Bibliografia

ASARO, R.J. & LUBARDA, V.A. (2006). Mechanics of solids and materials. Cambridge University
Press, New York, USA.
BATRA, R. C. (2006). Elements of Continuum Mechanics. John Wiley & Sons Ltd., United
Kingdom.
CASANOVA, J.C. (1993). Ejercicios de elasticidad. Editorial UPV.
CHADWICK, P. (1976). Continuum mechanics concise theory and problems. George Allen & Unwin
Ltd.Great Britain.
CHAVES, E.W.V. (2009). Mecánica del medio continuo: Modelos Constitutivos. CIMNE, Barcelona,
España.
CHAVES, E.W.V. (2007). Mecánica del medio continuo: Conceptos básicos. CIMNE, Barcelona,
España, (1ª Edición (2007), 2ª Edición (2010), 3ª Edición(2012)).
GOICOLEA, J.M. Mecánica del medio continuo web: http://w3.mecanica.upm.es/mmc-ig/
HOLZAPFEL, G.A. (2000). Nonlinear solid mechanics. John Wiley & Sons Ltd. England.
MASE, G.E. (1977). Teoría y problemas de mecánica del medio continuo. McGraw-Hill, USA.
OLIVER, X. & AGELET DE SARACÍBAR, C. (2000). Mecánica de medios continuos para ingenieros.
Ediciones UPC, Barcelona, España.
OLIVER, X. & AGELET DE SARACÍBAR, C. (2000). Cuestiones y problemas de mecánica de medios
continuos. Ediciones UPC, Barcelona, España.
ORTIZ BERROCAL, L. (1985). Elasticidad. E.T.S. de Ingenieros Industriales. Litoprint. U.P.
Madrid.
PARKER, D.F. (2003). Fields, Flows and Waves: An introduction to continuum models. Springer-
Verlag London, UK.

You might also like